Vinay Kumar - Differential Calculus For JEE Main and Advanced PDF

You might also like

You are on page 1of 918

ps

el
eh
je
iit
@

FREE BOOKS FOR JEE & NEET =>(@iitjeeadv)


About the Author
Vinay Kumar (VKR) graduated from IIT Delhi
in Mechanical Engineering.
Presently, he is Director of VKR Classes,
Kota, Rajasthan.

ps
el
eh
je
iit
@

FREE BOOKS FOR JEE & NEET =>(@iitjeeadv)


Third Edition
ps
el
Vinay Kumar
eh

B.Tech., IIT Delhi


je
iit
@

McGraw Hill Education (India) Private Limited

FREE BOOKS FOR JEE & NEET =>(@iitjeeadv)


Published by McGraw Hill Education (India) Private Limited
444/1, Sri Ekambara Naicker Industrial Estate, Alapakkam, Porur, Chennai - 600 116

Differential Calculus for JEE Main & Advanced, 3/e

Copyright © 2020, by the McGraw Hill Education (India) Private Limited.

No part of this publication may be reproduced or distributed in any form or by any means, electronic, mechanical, photocopying,
recording, or otherwise or stored in a database or retrieval system without the prior written permission of the publishers. The pro-
ps
gram listings (if any) may be entered, stored and executed in a computer system, but they may not be reproduced for publication.
el
eh

This edition can be exported from India only by the publishers.


McGraw Hill Education (India) Private Limited
je
iit

1 2 3 4 5 6 7 8 9 7101775 23 22 21 20 19
@

Printed and bound in India

ISBN (13) : 978-93-5316-461-4
















ISBN (10) : 93-5316-461-3

Information contained in this work has been obtained by McGraw Hill Education (India), from sources believed to be
reliable. However, neither McGraw Hill Education (India) nor its authors guarantee the accuracy or completeness of any
information published herein, and neither McGraw Hill Education (India) nor its authors shall be responsible for any
errors, omissions,or damages arising out of use of this information. This work is published with the understanding that
McGraw Hill Education (India) and its authors are supplying information but are not attempting to render engineering or
other professional services. If such services are required, the assistance of an appropriate professional should be sought.

Typeset at Transforma Pvt Ltd., Chennai and printed at XXXXXXXXX

Cover printed at:

Cover Design: Meenu Ragav


Image source: DAL

Visit us at: www.mheducation.co.in


Write to us at: info.india@mheducation.com
CIN: U80302TN2010PTC111532
Toll Free Number: 1800 103 5875

FREE BOOKS FOR JEE & NEET =>(@iitjeeadv)


PREFACE
T his book is meant for students who aspire to join the Indian Institute of Technologies (IITs) and various other engi-
neering institutes through the JEE Main and Advanced examinations. The content has been devised to cover the syllabi
of JEE and other engineering entrance examinations on the topic Differential Calculus. The book will serve as a text book
as well as practice problem book for these competitive examinations.
As a tutor with more than eighteen years of teaching this topic in the coaching institutes of Kota, I have realised
the need for a comprehensive textbook in this subject.
I am grateful to McGraw-Hill Education for providing me an opportunity to translate my years of teaching experience
into a comprehensive textbook on this subject.
This book will help to develop a deep understanding of Differential Calculus through graphs and problem solving.
The detailed table of contents will enable teachers and students to easily access their topics of interest.
Each chapter is divided into several segments. Each segment contains theory with illustrative examples. It is followed
by Concept Problems and Practice Problems, which will help students assess the basic concepts. At the end of the theory
ps
portion, a collection of Target Problems have been given to develop mastery over the chapter.
el
The problems for JEE Advanced have been clearly indicated in each chapter.
eh

The collection of objective type questions will help in a thorough revision of the chapter. The Review Exercises
je

contain problems of a moderate level while the Target Exercises will assess the students’ ability to solve tougher problems.
iit

For teachers, this book could be quite helpful as it provides numerous problems graded by difficulty level which can be
given to students as assignments.
@

I am thankful to all teachers who have motivated me and have given their valuable recommendations. I thank my
family for their whole-hearted support in writing this book. I specially thank Mr. Devendra Kumar and Mr. S. Suman for
their co-operation in bringing this book.
Suggestions for improvement are always welcomed and shall be gratefully acknowledged.

Vinay Kumar

FREE BOOKS FOR JEE & NEET =>(@iitjeeadv)


ps
el
eh
je
iit
@

FREE BOOKS FOR JEE & NEET =>(@iitjeeadv)


CONTENT
About the Author ii

Preface v

 CHAPTER 1 LIMITS 1.1 – 1.178 


1.1 Introduction 1.1



1.2 Concept of Infinity 1.7



1.3 Theorems on Limits 1.11



1.4 One-sided limits 1.14



1.5 Determinate and Indeterminate Forms ps 1.20



1.6 Factorisation and Cancellation of Common Factors 1.23



el
1.7 Rationalization 1.27



eh

1.8 Limit Using Expansion Series of Functions 1.29





je

1.9 Standard Limits 1.33





iit

1.10 Algebra of Limits 1.45





Limits when x → ∞
@

1.11 1.48



1.12 Asymptotes 1.56



1.13 Limit of a Sequence 1.61



1.14 Limits of Forms (0 x ∞) and (∞ – ∞) 1.65



1.15 Limits of Forms 0 and ∞ 0 0
1.71




1.16 Limits of Form 1 1.73



1.17 Sandwich Theorem / Squeeze Play Theorem 1.77



1.18 L’ Hospital’s Rule 1.82



1.19 Geometrical Limits 1.96



1.20 Miscellaneous Limits 1.99



Target Problems for JEE Advanced 1.104


Things to Remember 1.114


Objective Exercises 1.116


Review Exercises for JEE Advanced 1.126


Target Exercises for JEE Advanced 1.127


Previous Year’s Questions (JEE Advanced) 1.129


Previous Year’s Questions (JEE Main Papers) 1.131


Answers 1.133


Hints & Solutions 1.140


FREE BOOKS FOR JEE & NEET =>(@iitjeeadv)
viii | Content


 CHAPTER 2 CONTINUITY OF FUNCTIONS 2.1 – 2.98 


2.1 Definition of Continuity 2.1



2.2 Continuity in an Interval 2.10



2.3 Classification of Discontinuity 2.20



2.4 Algebra of Continuous Functions 2.27



2.5 Properties of Functions Continuous on a Closed Interval 2.33



2.6 Intermediate Value Theorem (I.V.T.) 2.37



Target Problems for JEE Advanced 2.45


Things to Remember 2.51


Objective Exercises 2.52


Review Exercises for JEE Advanced 2.63


Target Exercises for JEE Advanced 2.65


Previous Year’s Questions (JEE Advanced) 2.67


Previous Year’s Questions (JEE Main Papers) 2.69


Answers 2.70


Hints & Solutions
ps 2.74


 CHAPTER 3 DIFFERENTIABILITY 3.1 – 3.116
el


eh

3.1 Introduction 3.1


je



3.2 Differentiability 3.6
iit



3.3 Reasons of Non-differentiability 3.11



@

3.4 Relation between Continuity and Differentiability 3.13





3.5 Derivability at Endpoints 3.19



3.6 Differentiability over an interval 3.20



3.7 Alternative limit form of the Derivative 3.26



3.8 Derivatives of Higher Order 3.33



3.9 Algebra of Differentiable Functions 3.36



3.10 Functional Equations 3.40



Target Problems for JEE Advanced 3.48


Things to Remember 3.54


Objective Exercises 3.55


Review Exercises for JEE Advanced 3.64


Target Exercises for JEE Advanced 3.66


Previous Year’s Questions (JEE Advanced) 3.68


Previous Year’s Questions (JEE Main Papers) 3.72


Answers 3.74


Hints & Solutions 3.79


 CHAPTER 4 METHODS OF DIFFERENTIATION 4.1 – 4.120


4.1 Introduction 4.1



4.2 Derivative using First Principles (ab initio) Method 4.1



FREE BOOKS FOR JEE & NEET =>(@iitjeeadv)
Content | ix



4.3 Derivative of Standard Functions 4.3



4.4 Rules of Differentiation 4.4



4.5 The Chain Rule 4.9



4.6 Logarithmic Differentiation 4.15



4.7 Derivative of Inverse Functions 4.19



4.8 Parametric Differentiation 4.24



4.9 Differentiation of Implicit Functions 4.25



4.10 Differentiation by Trigonometric Substitution 4.33



4.11 Derivatives of Higher Order 4.37



4.12 Successive Differentiation 4.46



4.13 Derivative of a Determinant 4.48



4.14 Properties of Derivative 4.51



4.15 L’Hospital’s Rule 4.56



Target Problems for JEE Advanced 4.57


Things to Remember 4.63


Objective Exercises 4.64


ps
Review Exercises for JEE Advanced 4.73


el
Target Exercises for JEE Advanced 4.75


eh

Previous Year’s Questions (JEE Advanced) 4.77




Previous Year’s Questions (JEE Main Papers)
je

4.79


Answers
iit

4.80


Hints & Solutions 4.86
@


 CHAPTER 5 TANGENT AND NORMAL 5.1 – 5.94 


5.1 Introduction 5.1



5.2 Rate Measurement 5.1



5.3 Approximation 5.7



5.4 Error 5.9



5.5 Tangent and Normal 5.11



5.6 Tangent to Parametric curves 5.20



5.7 Angle of Intersection 5.26



5.8 Common Tangents 5.30



5.9 Length of Tangent 5.35



Target Problems for JEE Advanced 5.38


Things to Remember 5.42


Objective Exercises 5.43


Review Exercises for JEE Advanced 5.50


Target Exercises for JEE Advanced 5.52


Previous Year’s Questions (JEE Advanced) 5.54


Answers 5.55


Hints & Solutions 5.59


FREE BOOKS FOR JEE & NEET =>(@iitjeeadv)
x | Content


 CHAPTER 6 MONOTONICITY 6.1 – 6.100


6.1 Definitions 6.1



6.2 Monotonicity over an Interval 6.4



6.3 Critical Point 6.15



6.4 Intervals of Monotonicity 6.17



6.5 Monotonicity in Parametric Functions 6.24



6.6 Algebra of Monotonous Functions 6.24



6.7 Proving Inequalities 6.27



6.8 Concavity and Point of Inflection 6.35



Target Problems for JEE Advanced 6.43


Things to Remember 6.49


Objective Exercises 6.50


Review Exercises for JEE Advanced 6.59


Target Exercises for JEE Advanced 6.60


Previous Year’s Questions (JEE Advanced) 6.62


Previous Year’s Questions (JEE Main Papers)
ps 6.63


Answers 6.64
el


Hints & Solutions 6.68
eh


 CHAPTER 7 MAXIMA AND MINIMA 7.1 – 7.200
je


iit

7.1 Introduction 7.1





@

7.2 Concept of Local Maxima and Local Minima 7.1





7.3 Fermat Theorem 7.4



7.4 The First Derivative Test 7.7



7.5 The First Derivative Procedure for Sketching the Graph of a Continuous Function 7.15



7.6 Second Derivative Test 7.18



7.7 Higher Order Derivative Test 7.23



7.8 Extrema of Parametric Functions 7.25



7.9 Operations on Functions having points of Extrema 7.26



7.10 Global Maximum and Minimum 7.28



7.11 Boundedness 7.40




7.12 Algebra of Global Extrema 7.42



7.13 Miscellaneous Methods 7.44



7.14 Optimisation Problems 7.49



7.15 Asymptotes 7.70



7.16 Points of Inflection 7.75



7.17 Curve Sketching 7.77



7.18 Isolation of Roots 7.85



7.19 Rolle’s Theorem 7.89



FREE BOOKS FOR JEE & NEET =>(@iitjeeadv)
Content | xi



7.20 Deductions of Rolle’s Theorem 7.95



7.21 Lagrange’s Mean Value Theorem 7.98



7.22 Corollaries of LMVT 7.104



7.23 Related Inequalities 7.106



7.24 Cauchy’s Mean Value Theorem 7.112



Target Problems for JEE Advanced 7.115


Things to Remember 7.125


Objective Exercises 7.128


Review Exercises for JEE Advanced 7.136


Target Exercises for JEE Advanced 7.137


Previous Year’s Questions (JEE Advanced) 7.139


Previous Year’s Questions (JEE Main Papers) 7.143


Answers 7.144


Hints & Solutions 7.156


ps
el
eh
je
iit
@

FREE BOOKS FOR JEE & NEET =>(@iitjeeadv)


ps
el
eh
je
iit
@

FREE BOOKS FOR JEE & NEET =>(@iitjeeadv)


1
CHAPTER

Limits

We denote neighbourhoods by N(a), N1(a), N2(a), etc.


1.1 Introduction

Since a neighbourhood N(a) is an open interval symmetric
Let us introduce the notion of limit of a function which plays about a, it consists of all real x satisfying a – δ < x < a + δ for
an important role in mathematical analysis and the study some δ > 0. The positive number δ is called the radius of the
ps
of calculus. The concept of limit of a function is one of the neighbourhood.
el
fundamental ideas that distinguishes calculus from algebra
We designate N(a) by N(a, δ) if we wish to specify its radius.
and trigonometry.
eh

The inequalities a – δ < x < a + δ are equivalent to –δ < x – a


We use limits to describe the way a function f varies. Some < δ, and to |x – a| < δ. Thus, N(a, δ) consists of all points x
je

functions vary continuously i.e. small changes in x produce whose distance from a is less than δ.
iit

only small changes in f(x). Other functions can have values


that jump or vary erratically. Meaning of x → a
@

Sometimes we face with problems whose solutions involve the The symbol x → a is called as 'x tends to a' or 'x approaches a'.
use of limits. According to a formula of geometry, the area of It implies that x takes values closer and closer to 'a' but not 'a'.
a circle of radius r is πr2. How is such a formula derived? The
usual way is to inscribe regular polygons in the circle, find
the areas of these polygons, and then determine the “limiting
value” of these areas as the number of sides of the polygons Sometimes we need to consider values of x approaching 'a'
increase without bound. Thus, even such seemingly simple from only one side of 'a'.
formula as that for the area of a circle depends on the concept If x approaches 'a' from the left of 'a' then we use the symbol:
of limit for its derivation. x → a– or x → a – 0.

We also use limits to define tangent to graphs of functions. Similarly, if x approaches 'a' from the right of 'a' then we use
This geometrical application leads to the important concept of the symbol : x → a+ or x → a + 0.
derivative of a function, which quantifies the way a function's
value changes.
The need for evaluating the limit of a function arises in science
and engineering when we come across situations where a Limit of a Function
function (denoting a physical quantity) is not defined at x = a, Let us consider a function y = f(x) of a continuous variable x.
however the value of the function as x takes values very very Suppose that the independent variable x approaches a number
close to ‘a’ symbolize a useful physical quantity, for example 'a'. This means that x is made to assume values which become
instantaneous velocity, acceleration etc. arbitrarily close to 'a' but are not equal to 'a'. To describe such a
situation we say that x tends to 'a' or x approaches 'a' and write
Neighbourhood of a Point x → a. If there is a number  such that as x approaches 'a',
Any open interval containing a point a as its midpoint is called either from the right or from the left, f(x) approaches , then
a neighbourhood of a.  is called the limit of f(x) as x approaches 'a'.

FREE BOOKS FOR JEE & NEET =>(@iitjeeadv)


1.2 Differential Calculus for JEE Main and Advanced

Informal Definition of Limit differing by little from zero. It is not a statement about the value
Let f(x) be defined on an open interval about 'a', except possibly of f(x) when x = 0. When we make the statements we assert
at 'a' itself. If f(x) gets arbitrarily close to  for all x sufficiently that, when x is nearly equal to zero, f(x) is nearly equal to . We
close to 'a', we say that f approaches the limit  as x approaches assert nothing about what happens when x actually equal to 0.
'a', and we write
lim f ( x )  . Caution
x a

(the abbreviation "lim" means "limit")


A wrong statement about limits :
It is evident that if a function has a limit for x → a, this limit is The number  is the limit of f(x) as x approaches a if f(x) gets
unique, since the values of the function corresponding to the closer to  as x approaches a.
values of x approaching 'a' must become arbitrarily close to Consider f(x) = x + [x] +1 at x = 0.
a constant and hence cannot be simultaneously close to more As x approaches 0, the function f(x) gets closer to
than one constant number. 1/2, from both sides but 1/2 is not the limit because f(x) does
An alternative notation for lim f ( x )   is f(x) →  as x → a not get arbitrarily close to 1/2. For example the function cannot
x a
attain 0.4995 or 0.5002 by using x sufficiently close to 0.
which is usually read as "f(x) approaches  as x approaches a".
Suppose we are asked to sketch the graph of the function f
Study Tip x3 − 1
given by f(x) = , x ≠ 1.
x −1
1. A number  is said to be a limiting value only if it is finite
For all values other than x = 1, we can use standard curve-

and real, otherwise we say that the limit does not exist or
sketching techniques. However, at x = 1, it is not clear what
dne (for brevity).
ps
to do. To get an idea of the behaviour of the graph of f near
2. Note that 'a' need not be in the domain of f.
el
x = 1, we can use two sets of x-values – one set that approaches

Even if 'a' happens to be in the domain of f, the
eh

1 from the left and one set that approaches 1 from the right, as

value f(a) plays no role in determining whether shown in table.
lim f ( x )  . It is only the behaviour of f(x) for x
je

x a
iit

near a that concerns us.



3. We also note that the three statements,
@

lim f(x) = , lim (f(x) – ) = 0, lim | f(x) –  | = 0, are all
x →a x →a x →a

equivalent.
x
Let us consider the function f(x) = .
x
This function is equal to 1 for all values of x except x = 0.
It is not equal to 1 when x = 0; it is in fact not defined at x = 0. When we plot these points, it appears that the graph of f is a
parabola that has a hole at point (1, 3), as shown in the figure.
For when we say that f(x) is defined for x = 0 we mean that we
Although x cannot equal 1, we can move sufficiently close to
can calculate its value for x = 0 by putting x = 0 in the formula
1, and as a result f(x) moves arbitrarily close to 3.
which defines f(x). In this case we cannot. When we put x = 0
in f(x) we get 0/0, which is meaningless. Using limit notation, we write lim f(x) = 3.
x→1
x
Thus f(x) = is a function which differs from y = 1 solely in It is read as "the limit of f(x) as x approaches 1 is 3."
x
that it is not defined for x = 0.
x
We have lim
x→0
= 1, since x/x is equal to 1 so long as x differs
x
from zero, however small the difference may be.
On the other hand there is of course nothing to prevent the
limit of f(x) as x tends to zero from being equal to f(0), the
value of f(x) for x = 0.
Thus if f(x) = x, then f(0) = 0 and lim
x →0
f ( x ) = 0.

Note: That the statement lim f ( x ) =  is a statement


x →0


about the values of f(x) when x has any value distinct from but

FREE BOOKS FOR JEE & NEET =>(@iitjeeadv)


Limits 1.3

x 2 − 3x + 2 x3 − 1 ( x 2 + x + 1)( x − 1) x2 + x + 1
Example 1: Evaluate lim = = .
x−2 x2 −1 ( x + 1)( x − 1) x +1


x→2

Solution: Consider the function f defined by So the behaviour of (x3–1)/(x2–1) for x near 1, but not equal to


x 2  3x  2 x2 + x + 1
f(x) = ,x≠2 1, is the same as the behaviour of .
x +1
x2
The domain of f is the set of all real numbers except 2, which x3 − 1 x2 + x + 1 .
lim = lim
has been excluded because substitution of x = 2 in the expression x →1 x 2 −1 x →1 x +1
0
for f(x) yields the undefined term .
0 Now, as x approaches 1, x2 + x + 1 approaches 3 and x + 1
On the other hand, x 2 – 3x + 2 = (x – 1) (x – 2) and
( x − 1)( x − 2) x2 + x + 1 3
f(x) = = x – 1, provided x ≠ 2
approaches 2. Thus, lim = , from which it
x−2
x →1 x +1 2
....(1) x3 − 1 3
The graph of the function y = x – 1 is a straight line L; so the follows that lim = .
x →1 x 2 − 1 2
graph of f(x) is the line L with a hole at the point (2, 1).
We can also express this as follows:

x3 − 1 3
As x → 1, 2 → .
x −1

2
Example 3: Find the limit of the function
ps


x + 1 , x < 0
f (x) =  as x→ 0.
el
2 − x , x ≥ 0

eh

Solution: Using the graph of y = f(x), we see that the




je

function approaches 1 as x approaches 0 from the left of 0. Also


the function approaches 2 as x approaches 0 from the right of 0.
iit
@

Although the function f is not defined at x = 2, we know its


behaviour from values of x near 2. The graph makes it clear
that if x is close to 2, then f(x) is close to 1. In fact, the values of
f(x) can be brought arbitrarily close to 1 by taking x sufficiently
close to 2.
x 2 − 3x + 2
We express this fact by writing lim = 1, which
x→2 x−2
Since the function does not approach the same level from both
x 2  3x  2
means that the limit of is 1 as x approaches 2. sides of x = 0, lim f ( x ) does not exist.
x2 x →0
x3 −1
Example 2: Let f(x) = 2
x −1
. How does f(x) behave
Formal Definition of Limit


when x is near 1 but is not 1 itself ? The conceptual problems in trying to give an exact meaning
Solution: There are two influences acting on the fraction to the expression lim f(x) = b revolve around phrases such as
x →a


(x3 – 1)/(x2 – 1) when x is near 1. On the one hand, the numerator “arbitrarily close,” “sufficiently near,” and “arbitrarily small.”
x3 –1 approaches 0; thus there is an influence pushing the After all, there is a no such thing in any absolute sense as a small
fraction towards 0. On the other hand the denominator x2 –1 positive real number. The number 0.000001 is small in most
also approaches 0; division by a small number tends to make a contexts, but in comparison with 0.000000000001 it is huge.
fraction large. How do these two opposing influences balance However, we can assert that one number is smaller than another.
out?
Moreover, the actual closeness of one number x to another number
We rewrite the quotient (x3 – 1)/(x2 – 1) as follows: When x ≠ 1, a is just the distance between them : it is |x – a|. One way to say that
we have a function f takes on values arbitrarily close to a number  is to

FREE BOOKS FOR JEE & NEET =>(@iitjeeadv)


1.4 Differential Calculus for JEE Main and Advanced

state that, for any small positive real number ε, there are numbers In Figure 2 the vertical band shown is not narrow enough
x such that | f(x) –  | < ε. We are stating that no matter what to meet the challenge of the horizontal band shown. But the
positive number ε is selected, 10–7, or 10–17, or 10–127, there are vertical band shown in Figure 3 is sufficiently narrow.
numbers x so that the distance between f(x) and  is smaller
than ε.
ε − δ definition
Let f be a function defined on an open interval containing a
except possibly at a and let  be a real number. The statement
lim f(x) =  means that for each ε > 0 there exists a δ > 0 such
x →a
that if 0 < |x – a| < δ, then |f(x) – | < ε.
The inequality 0 < |x – a | that appears in the definition is just
a fancy way of saying “x is not a.” The inequality |x – a | < δ
asserts that x is within a distance δ of a. The two inequalities Figure 1
may be combined as the single statement 0 < |x – a| < δ,
which describes the open interval (a–δ, a + δ) from which a
is deleted.
Here we wish to say that f(x) is arbitrarily close to  whenever x
is sufficiently close, but not equal to a. What does “sufficiently
close” mean? “ The answer is this : If an arbitrary ε > 0 is
chosen with which to measure the distance between f(x) and ,
then it must be the case that there is a number δ > 0 such that
ps
whenever x is in the domain of f and within a distance δ of
el
a, but not equal to a then the distance between f(x) and  is
less than ε. Figure 2
eh

This is illustrated on the graph of the function y = f(x) as shown


je

in the figure. Since from the inequality |x – a| < δ there follows


the inequality |f(x) – l| < ε, this means that for all points x that
iit

are not more distant from the point a than δ, the points M of
@

the graph of the function y = f(x) lie within a band of width 2ε


bounded by the lines y = –ε and y =  + ε.

Figure 3

Example 4: Prove that lim (3x + 1) = 7.


x→2


Solution: Let an arbitrary ε > 0 be given. For the


inequality |(3x + 1) – 7| < ε to be fulfilled it is necessary to
have the following inequalities fulfilled :
First ε > 0 is chosen arbitrarily. There must then exist a number ε ε ε
|3x – 6| < ε, |x – 2| < , – <x–2<
δ > 0 such that whenever x lies in the interval (a – δ, a + δ), 3

3 3
and x ≠ a then the point (x, f(x)) lies in the shaded rectangle. Thus, given any ε, for all values of x satisfying the

In other words, ε is the challenge. The response is δ. Usually, ε
inequality |x – 2| < = δ, the value of the function 3x + 1 will
the smaller ε is, the smaller δ will have to be. The geometric 3
significance of the definition is shown in the Figure 1. The differ from 7 by less than ε. And this means that 7 is the limit
narrow horizontal band of width 2ε is again the challenge. of the function as x → 2.
The response is a sufficiently narrow vertical band, of width
Example 5: Examine the limit of the function
2δ, such that the part of the graph within that vertical band


(except perhaps at x = a) also lies in the challenging horizontal x2 −1
y= as x tends to 1.
band of width 2ε. x −1

FREE BOOKS FOR JEE & NEET =>(@iitjeeadv)


Limits 1.5

Solution: The function is defined for every value of x Example 6: Consider the function





x2 −1
other than 1 and y = = x + 1, when x≠1.  x 2 + 1,
 x≥0
x −1 g(x) =  lim g(x) ≠ 1.
. Prove that x→0
Firstly consider the behaviour of the values of y for values of 
 − ( x 2
+ 1), x < 0
x greater than 1. Clearly, y is greater than 2 when x is greater
Solution To do this, we must establish the negation of
than 1.



the limit condition. There is an ε > 0 such that, for any δ > 0,
If, x, while remaining greater than 1 takes up values whose there is a number x in the domain of a g such that 0 < |x| < δ
difference from 1 constantly diminishes, then y, while and |g(x) – 1| ≥ ε. There are many possible choices for ε. To be
remaining greater than 2, takes up values whose difference 1
from 2 constantly diminishes also. specific, take ε = . We must now show that for every positive
2
In fact, difference between y and 2 can be made as small as we number δ, there is a nonzero number x in the open interval
like by taking x sufficiently near 1. (–δ, δ), such that the distance between g(x) and 1 is greater
1 δ
For instance, consider the number .001. Then than or equal to . See figure. Take x = – . This number is
2 2
|y – 2| = y – 2 = x + 1 – 2 < .001 ⇔ x < 1.001.
nonzero, lies in (–δ, δ), and furthermore
Thus, for every value of x which is greater than 1 and less than
1.001, the absolute value of the difference between y and 2 is
 δ δ2
g(x) = g  −  = – – 1 < – 1.
less than the number .001 which we had arbitrarily selected.  
2 4
Instead of the particular number .001, we now consider any 1
Hence |g(x) – 1| > 2 ≥ .
small positive number ε. Then ps 2
y – 2 = x – 1 < ε ⇔ x < 1 + ε.
el
Thus, there exists an interval (1, 1 + ε), such that the value of
eh

y, for any value of x in this interval, differs from 2 numerically,


by a number which is smaller than the positive number ε,
je

selected arbitrarily.
iit

Thus, the limit of y as x approaches 1 from the right is 2 and


@

we have lim y = 2.
x→1+
We now consider the behaviour of the values of y for values of
x less than 1. When x is less than 1, y is less than 2.
If, x, while remaining less than 1, takes up values whose
difference from 1 constantly diminishes, then y, while
remaining less than 2, takes up values whose difference from
2 constantly diminishes also. Notice in the definition that the hypothesis 0 < | x – a | simply
Let now, ε be any arbitrarily assigned positive number, however means x ≠ a. This condition releases the point of approach
small. We then have, a from the responsibility of having any image at all; and
even if there is an image f(a) the condition | f(a) –  | < ∈
|y – 2| = 2 – y = 2 – (x + 1) = 1 – x < ε is allowed to fail. Briefly, the idea of limit discounts what
so that for every value of x less than 1 but > 1 – ε, the happens precisely at a, but is vitally concerned with images of
absolute value of the difference between y and 2 is less than the "neighbours" of a.
the number ε. There is no practical need of applying ∈−δ definition
Thus, the limit of y, as x approaches 1, from the left is 2 and everywhere since it involves lengthy and complicated
lim y = 2.
we write x→ calculations. We shall later on derive some simple rules for
1 −
finding limits.
Combining the conclusions arrived at in the last two cases, we
see that corresponding to any arbitrarily assigned positive Example 7: Let the function f be defined as follows:


number ε, there exists an interval (1 – ε, 1 + ε) around 1, such  1
f   = 1 for every nonzero integer n,
 n

that for every value of x in this interval, other than 1 where
the function is not defined, y differs from 2 numerically by a f(x) = x for every other real number x. Prove that
number which is less than ε, i.e., we have | y – 2 | < ε for

lim f(x) does not exist.
any x, other than 1, such that |x – 1| < ε. x→0

FREE BOOKS FOR JEE & NEET =>(@iitjeeadv)


1.6 Differential Calculus for JEE Main and Advanced

Solution: The graph of f is the line y = x, with the There can, therefore, exist no number which differs from sin


exception that when x = 1/n, n a nonzero integer, y = 1. It is 1/x by a number less than an arbitrarily assigned positive
shown in the figure. number for values of x near 0. Hence lim (sin 1/x) does
x→0
Since f(0) = 0 and f(x) is close to 0 when x is close to 0, x ≠ not exist.
 2 1
1/n, we might suspect that lim f(x) = 0.  x sin , x ≠ 0
x→0 Example 9: f(x) =  x as x→ 0.




1 , x=0
Solution: The function oscillates between the graphs



of y = –x2 and y = x2. As x approaches closer and closer to
zero from either side, the two functions approach 0, and hence
f(x) → 0 as x→ 0.
Example 10: Find the limit of the function



x , x ∈Q
f (x) =  as x→ 2 and x→ 4.
 4 − x , x ∉Q
Solution: We draw the rough sketch of y = f(x).



However, this is not correct. For if we take the neighbourhood
N = (–1/2, 1/2) of 0, there is no deleted neighbourhood D = (-δ, 0)
∪ (0, δ) of 0 such that f(x) is in N for every x in D.
We can always find a positive integer n such that 1/n < δ and for
ps
this number f(1/n) is not in N since f(1/n) = 1. In a similar way,
el
we can show that lim f(x) is not equal to any other number.
eh

x→0 If x approaches 2 by taking rational values, then the function


Therefore, this limit does not exist. f(x) approaches 2 using y = x. And if x approaches 2 by taking
je

Example 8: Prove that lim (sin 1/x) does not exist. irrational values, then the function f(x) also approaches 2 using
iit


x→0
y = 4 – x. Hence, the limit of f(x) as x→ 2 is 2.
Solution: The graph of y = sin 1/x is drawn below.

@


lim f ( x ) = 2
x →2

However, if x approaches 4 by taking rational values, then the


function f(x) approaches 4 using y = x. And if x approaches 4
by taking irrational values, then the function f(x) approaches 0
using y = 4 – x. Since the function does not approach the same
level, the limit lim f ( x ) does not exist.
x →4
There are many such interesting functions that have unusual
The function oscillates between –1 and 1 more and more limit behaviour. An often cited one is the Dirichlet function
rapidly as x approaches closer and closer to zero from either side.
0, if x is rational
If we take any interval containing 0, however small it may be, f(x) = 
then for an infinite number of points of this interval the function 1, if x is irrational.
assumes the values 1 and –1. This function has no limit at any real number.

Practice Problems A

1. For the function graphed here, find the following limits:

FREE BOOKS FOR JEE & NEET =>(@iitjeeadv)
Limits 1.7

(i) lim f(x) (ii) lim f(x) x if x is rational




x→2 x→1 5. Let f(x) = 
− x


2. For the function graphed here, find the following limits: if x is not rational

(a) Does limx→1 f(x) exist ?



(b) Does limx→ f(x) exist ?



(c) Does limx→0 f(x) exist ?



(d) For which numbers a does limx→af(x) exist?



 x 2 if x is an irrational number
6. Let f(x) =  .
1 if x is an rational number


(i) lim f(x) (ii) lim f(x) (iii) lim f(x)



x→−2 x→0 x→−1
Prove that f(x) has a limit at the points x = 1 and


3. If lim f(x) = 5 must f be defined at x = 1 ? x = –1 and does not have a limit at other points.

x→1
If it is, must f(1) = 5 ? Can we conclude anything about 7. Find lim cot −1 x 2 .


the values of f at x = 1 ? Explain. x→0

4. If f(1) = 5, must lim f(x) exist ? If it does, then must lim sin 2 x
8. Find lim .

x→1 x→1

x→0 cos x
f(x) = 5? Can we conclude anything about lim f(x)?
x→1 9. Find lim cos π/x.

ps x→0
el
eh

the corresponding values yn of the function become arbitrarily


1.2 Concept of Infinity close to the number .

je

Suppose that n assumes successively the values 1, 2, 3,.... Then Let us consider a function of an integral argument. Usually
iit

as n gets larger and larger and there is no limit to the extent such an argument is denoted by the letter n and the values of
@

of its increase. However, large a number we may think of, a the function by some other letter supplied with a subscript
time will come when n has become larger than this number. indicating the value of the integral argument. For instance, if
It is convenient to have a short phrase to express this unending y = f(n) is a function of the integral argument n we write yn =
growth of n and we shall say that n tends to infinity, or n → ∞, f(n). Given such a function, we say that the values
When we say that n ‘tends to ∞’ we mean simply that n is y1 = f(1), y2 = f(2), .... , yn = f(n), .....,
supposed to assume a series of values which increase beyond assumed by the function form a sequence.
all limit.
If there is a sequence y1, y2, y3, ...., this assigns, to every natural
There is no real number ‘infinity’. This implies that the equation
n = ∞ is meaningless. A number n cannot be equal to ∞, because number n, a value yn = f(n). For instance, the terms of the
‘equal to ∞’ means nothing. So far in fact the symbol ∞ means
1 1 1
nothing at all except in the one phrase ‘tends to ∞’, the meaning geometric progression , , .... are the subsequent values
2 4 8
of which we have explained above. Later on we shall learn how
1
to attach a meaning to other phrases involving the symbol ∞, of the function f(n) = .
but we bear in mind 2n
We now discuss the question ‘what properties has f(n) for
(i) that ∞, by itself means nothing, although phrases
sufficiently large values of n ?’ i.e. ‘how does f(n) behaves as

containing it sometimes mean something,
n tends to ∞ ?’
(ii) that in every case in which a phrase containing the symbol
It may occur that, as n increases, the values yn = f(n) become

∞ means something it will do so simply because we have
arbitrarily close to a number . Then we say that the number  is
previously attached a meaning to this particular phrase by
the limit of the function f(n) of the integral argument n or that the
means of a special definition.
sequence y1, y2, .... , yn, .... has the limit , as n → ∞, and write
Limit of a Sequence lim f (n )   or lim y n  
n n
Definition  A number  is said to be the limit of the function
y = f(n) of the integral argument n or the limit of the sequence Consider the function 1/n for large values of n. Instead of
y1, y2, ...., yn, if for all sufficiently large integral values of n saying ‘1/n is small for large values of n’ we say ‘1/n tends

FREE BOOKS FOR JEE & NEET =>(@iitjeeadv)


1.8 Differential Calculus for JEE Main and Advanced

to 0 as n tends to ∞ . Similarly we say that ‘1– (1/n) tends to Note: The limit  may itself be one of the actual values
1 as n tends to ∞’.



of f(n). Thus if f(n) = 0 for all values of n, it is obvious that
We shall say that ‘the limit of 1/n as n tends to ∞ is 0’, a lim f (n ) = 0.
n →∞
1
statement which we express as lim  0 . On the other hand the limit itself need not (and in general will
n  n
not) be the value of the function for any value of n. This is
We shall also sometimes write ‘1/n → 0 as n → ∞’ which may sufficiently obvious in the case f(n) = 1/n. The limit is zero;
be read ‘1/n tends to 0 as n tends to ∞’. but the function is never equal to zero for any value of n.
In the same way we shall write A limit is not a value of the function. It is something quite
 1 distinct from these values.
lim 1 −  = 1 or , 1 – (1/n) → 1 as n → ∞. For the function f(n) = 1 the limit is equal to all the values of
n →∞  n


f(n).
Now let us consider f(n) = n2. Then ‘n2 is large when n is large’. For f(n) = 1+ (1/n), it is not equal to any value of f(n). For f(n)
And it is natural in this case to say that ‘n2 tends to ∞ as n tends = (sin 12 nπ)/n, (whose limits as n tends to ∞ is easily seen to be
to ∞’, and we write n2 → ∞. 0, since sin 12 nπ is never numerically greater than 1), the limit
Finally consider the function f(n) = – n2. In this case f(n) is is equal to the value which f(n) assumes for all even values of
large, but negative, when n is large and we say that ‘– n2 tends n, but the values assumed for odd values of n are all different
to – ∞ as n tends to ∞’ and write – n2 → − ∞ . from the limit and from one another.
Definition  The function f(n) is said to tend to the limit  Definition The function f(n) is said to tend to ∞ (positive


as n tends to ∞ , if, however small be the positive number ε, infinity) with n if, when any number M, however large, is
f(n) differs from  by less than ε for sufficiently large values assigned, we can determine N so that f(n) > M when n ≥ N;
ps
of n; that is to say if, however small be the positive number that is to say if, however large M may be, f(n) > M for sufficient
ε, we can determine a number N(ε) corresponding to ε, such large values of n.
el
that f(n) differs from  by less than ε for all values of n greater Another form of statement is, if we can make f(n) as large as
eh

than or equal to N(ε). we please by sufficiently increasing n.


The definition may be stated more shortly as follows : if, given A function may be always numerically very large when n is
je

any positive number, ε, however small, we can find N(ε) so very large without tending either to ∞ or to –∞. A sufficient
iit

that | f(n) –  | < ε when n ≥ N(ε), then we say that f(n) tends illustration of this is given by f(n) = (–1)nn.
to the limit  as n tends to ∞ , and write
@

Example 1: Find lim sin nθπ, if θ is rational.




n→∞
lim f (n ) =  Solution: Let f(n) = sin nθπ and θ = p/q, where p and q


n →∞
are positive integers. Let n = aq + b, here a is the quotient and
The definition of a limit may be illustrated geometrically b the remainder when n is divided by q.
as follows. The graph of f(n) consists of a number of points Then sin(npπ/q) = (–)ap sin(bpπ/q).
corresponding to the values n = 1, 2, 3.... Suppose, for example, p even. Then, as n increases from 0 to
Draw the line y = , and the parallel lines y =  – ε, y =  + ε q – 1, f(n) takes the values
at distance ε from it. Assume that for each positive ε , a number
N can be found, depending on ε , such that the part of the graph pπ 2pπ (q −1)pπ
0, sin , sin ......., sin

to the right of n = N lies within the band. Then we say that ‘as q q q
x approaches ∞, f(n) approaches ’ and write
When n increases from q to 2q–1 these values are repeated and
lim f (n ) =  so also as n goes from 2q to 3q – 1, 3q to 4q–1, and so on. Thus
n →∞
the values of f(n) form a cyclic repetition of a finite series of
different values. It is evident that when this is the case f(n) cannot
tend to a limit, nor to ∞ , nor to –∞, as n tends to infinity. The
limit does not exist.

Oscillating Functions
When f(n) does not tend to a limit, nor to ∞ , nor to –∞, as n tends
to ∞, we say that f(n) oscillates as n tends to ∞.
The simplest example of an oscillatory function is given by f(n)
= (–1)n , which is equal to 1 when n is even and to –1 when n
is odd. In this case the values recur cyclically.

FREE BOOKS FOR JEE & NEET =>(@iitjeeadv)


Limits 1.9

Now consider f(n) = (–1)n + n–1, the values of which are of an endless band of width 2ε. Assume that for each


–1 + 1, 1 + 12 , –1 + 13 , 1 + 14 , –1 + 15 , .... positive ε, a number D can be found, depending on
ε, such that the part of the graph to the right of x = D lies
When n is large every value is nearly equal to 1 or
within the band. Then we say that “as x approaches ∞, f(x)
–1, and obviously f(n) does not tend to a limit or to ∞ or to –∞
approaches ” and write as
and therefore it oscillates, but the values do not recur. It is to lim f(x) = 


be observed that in this case every value of f(n) is numerically x→∞

less than or equal to . We have a similar definition for lim f(x) = .


x→ −∞
If f(n) oscillates as n tends to ∞, then f(n) will be said to oscillate Note that the definition of the limit of a sequence can be
finitely or infinitely according as is or is not possible to assign regarded as a special case of the definition of the limit of a
a number K such that all the values of f(n) are numerically function, as its argument becomes infinitely large and assumes
less than K, only integral values.
i.e. | f(n) | < K for all values of n. Definition We assume that f(x) is defined for all x greater


For example, consider f(n) = n{1 + (–1)n}. than some number c. If for each number E there is a number
In this case the values of f(n) are D such that for all x > D it is true that f(x) > E, then we write
lim
0, 4, 0, 8, 0, 12, 0, 16,.... = ∞.
x→∞ f(x)

The odd terms are all zero and the even terms tend to ∞. Thus For each possible choice of a horizontal line, say, at height E,
f(n) oscillates infinitely. if we are far enough to the right of f, we stay above that line.
That is , there is a number D such that if x > D, then f(x) > E, as
But the function f(n) = sin nθπ, θ is rational, oscillates finitely.
illustrated in the figure.
Now consider the sequence
ps
π 2π 3π nπ
el
sin , sin , sin , ...., sin , ....
2 2 2 2
eh


It does not possess a limit since, for n = 1, 2, 3, 4, y n = sin
je

2
iit

consecutively takes on the values 1, 0, –1, 0 which then are


repeated indefinitely in the same order as n increases. Therefore
@

the number to which yn tends, as n → ∞, does not exist.


We have similar definitions for lim f(x) = − ∞,
Limit of a Function as x → ∞ x→∞

Definition  Let f(x) be defined for all x beyond some lim f(x) = ∞ and lim f(x) = − ∞.
x → −∞ x → −∞
number c. If for each positive number ε, there is a number D
such that for all x > D it is true that For example, if a > 1, the function y = ax becomes infinite for
| f(x) –  | < ε , then we write x→∞
lim f(x) = . x → ∞ i.e. lim ax = ∞.
x →∞

If 0 < a < 1 we have lim ax = ∞.
The positive number ε is the challenge, and D is a response. x → −∞
The smaller ε is, the larger D usually must be chosen. The Further, lim log a x = ∞ , if a > 1
x →∞

geometric meaning of the precise definition of x→∞
lim f(x) = 
is shown in the figure. lim log a x = − ∞ , if 0 < a < 1

x →∞

Note: We say that a limit, such as xlim


→∞
f(x), exists when


the limit is a real number, but not when the limit is ∞ or –∞.

Bounded Function
The function f(x) in a given interval of x is said to have
(i) a lower bound if there exists a finite number A, such that

f(x) ≥ A, for all values of x in the given interval.
Draw two lines parallel to the x axis, one of height (ii) an upper bound if there exists a finite number B such that

 + ε and one of height  – ε . They are the two edges f(x) ≤ B, for all values of x in the given interval.

FREE BOOKS FOR JEE & NEET =>(@iitjeeadv)


1.10  
Differential Calculus for JEE Main and Advanced

A function is said to be bounded in an interval if it has a lower Infinite Limits


bound as well as an upper bound. Suppose that when x → a the "limiting behaviour" of a function
Thus, a function is bounded if its values are contained between y = f(x) is such that its absolute value increases unlimitedly
two numbers A and B i.e. A ≤ f(x) ≤ B. then we say that the function f(x) approaches an infinitely large
Then, the graph of the function y = f(x) lies between the lines quantity as x → a.
y = A and y = B. Definition A function y = f(x) is said to approach infinity


Alternatively, a function f(x) is called bounded in a given as x → a if for all the values of x lying sufficiently close to a
interval of x if these exists a number M > 0 such that |f(x)| ≤ the corresponding values of the function f(x) become greater
M for all values of x in the given interval. If there is no such than any given arbitrarily large positive number.
number M, the function is called unbounded. If a function f(x) approaches infinity as x → a we write


The graph of a bounded function (considered within the given lim f ( x ) = ∞ .
interval) is completely contained between the straight lines y x→a

= –M and y = M. A function y = f(x) which approaches infinity as x → a does


For instance, the functions y = sin x and y = cos x are bounded not have a limit in the ordinary sense. To extend the ordinary
for all x while the function y = ax when considered on the terminology and to characterize the behaviour of the function
whole x–axis, is unbounded. whose absolute value |f(x)| increases indefinitely we say that
It should be stressed that when speaking of the boundedness the function y = f(x) tends to infinity or has an infinite limit.
of a function it is necessary to indicate the interval on which Suppose that a function y = f(x) tending to infinity as x → a
the function is considered. only assumed positive (negative) values in a neighbourhood
of the point c, then we say that the function f(x) approaches
ps
1 plus (minus) infinity as x → a. In these cases we write,
For example, the function y = is bounded in the interval
el
x respectively,
(1, ∞) and unbounded in the interval (0, 1).
eh

lim f ( x ) = ∞ and lim f ( x ) = − ∞


 π x→a x→a
je

The function y = tanx is bounded in the interval  0,  and


 4
 π Consider the following examples:
iit

unbounded in the interval  0,  .


 2
x +1
@

Definition The function f(x) is called bounded as x → a if (i) The function y = approaches infinity as x → 1, it
x −1


there exists a neighbourhood of the point a, in which the given
tends to – ∞ if x remains less than 1 and to ∞ if x remains
function is bounded. greater than 1.
Definition The function y = f(x) is called bounded as

x → ∞ if there exists a number N > 0 such that for all values of 1
(ii) The function y = has the limit ∞ as x → 0.
x satisfying the inequality x > N, the function f(x) is bounded. x2

From the definition of a bounded function f(x) it follows that −1
(iii) lim = − ∞.
if xlim f ( x ) = ∞ or lim f ( x ) = ∞ that is, if f(x) is an infinitely x →1 ( x − 1) 2

→a x →∞
large function, it is unbounded. The converse is not true: an
unbounded function may not be infinitely large. Note: In what follows, when speaking about a limit, we


For example, the function y = x sin x as x → ∞ is unbounded shall mean that the limit is finite unless it is stipulated that it
because, for any M > 0, values of x can be found such that |x is infinite.
sin x|>M. But the function y = x sin x is not infinitely large x2 − 4 x2 − 4
because it becomes zero when x = 0, π, 2π, .... For example, since lim = 4, we say that lim
x→2 x − 2 x→2 x − 2

Note: 1
exists. However, although lim 2 = ∞ , we do not say that

x→0 x
1. If a function f(x) increases as x → ∞ and remains bounded 1

then it possesses a limit. lim 2 exists.
x→0 x
2. The product of a bounded function by an infinitesimally Consider some more examples :

small quantity is an infinitesimally small quantity.
Let f(x) be an infinitesimally small quantity and g(x) a bounded lim cot −1 x = π , lim x 2 = ∞ , lim sin x dne.
x →∞ x →−∞ x →∞
function in a neighbourhood of the point a to which x tends i.e.
|g(x)| ≤ M. Then we have lim f ( x ) g ( x ) = 0 . Example 2: Show that lim e − x sin x = 0.
x→a


x →∞

FREE BOOKS FOR JEE & NEET =>(@iitjeeadv)


Limits 1.11

Solution: –e–x ≤ e–x sin x ≤ e–x The function oscillates between –∞ and ∞.


Hence the limit does not exist.
2x 2
Example 4: Evaluate lim .
x2 + 1



x →∞
Solution: We have



2x 2 2( x 2 + 1) − 2 2
= = 2−
x +1 2
x +1
2
x +1
2

We can see from the graph that lim e − x sin x = 0. 2


x →∞ and as x → ∞, →0.
x +1
2


x –1
Example 3: Evaluate lim 2 sin sin(x).


x→∞
π x π 2x 2
Solution: − .2 ≤ 2 sin–1sin(x) ≤ 2 x
x
Hence, xlim = 2.
2 →∞ x 2 + 1


2

Concept Problems A

1. Find the following limits for the function shown in the Find (i) lim f(x), (ii) lim f(x), (iii) lim f(x).

figure:

ps x2 x→0 x→3
Y
3. Find lim [αn], where α is any positive number and [.]

el
n→∞
denotes G.I.F.
eh

1
0 1 2 X
4. Find lim {n –(–1)n} + n{1–(–1)n}

je

n→∞

5. Find lim (sin nθπ)/{n(n–(–1)n)}, where θ is any real


iit

n→∞
(i) lim f(x) (ii) lim f(x) number.
@






x3 x→3

lim f(x) lim f(x) 6. Find lim sin nθπ, if θ is integral.


(iii) (iv) x→∞

n→∞
x




1
 1 7. Find lim .
 x  2 , x  2 n →∞ 1 1


cos 2 nπ + n sin 2 nπ
 2 2
2. Let f(x) =  x 2  5  2  x  3

 1
lim
 x  13 x  3 8. Find n →∞  21 1  .
n  cos nπ + n sin 2 nπ

  2 2 

(v) Sum rule


1.3 Theorems on Limits

lim (f + g) (x) = lim [f(x) + g(x)]

x →a x →a

Let a and b be real numbers and let n be a positive integer.
(i) lim b = b (ii) lim x = a (iii) lim xn = an = xlim
→a
f(x) + xlim
→a
g(x) =  + m,
x→a x→a x→a





Let f and g be two functions such that i.e., the limit of the sum of two functions is equal to the


lim f(x) = , lim g(x) = m. sum of their limits.
x →a x →a (vi) Difference rule


Then lim (f – g) (x) = lim [f(x) – g(x)]
x →a x →a

(iv) Constant Multiple rule

lim k(f(x)) = k. lim f(x) = k., = xlim
→a
f(x) – xlim
→a
g(x) =  – m,

x →a x →a

where k is independent of x. i.e., the limit of the difference of two functions is equal to


A constant factor may be taken outside the limit sign. the difference of their limits.

FREE BOOKS FOR JEE & NEET =>(@iitjeeadv)
1.12  
Differential Calculus for JEE Main and Advanced

The limit of an algebraic sum of two, three or, in general, (a) if q(a) ≠ 0, then lim f(x) = f(a).

any definite number of functions is equal to the algebraic x →a



sum of the limits of these functions. (b) if q(a) = 0 but p(a) ≠ 0, then lim f(x) does not exist.
x →a



(vii) Product rule

lim (fg) (x) = lim [f(x). g(x)] (c) if q(a) = 0 and p(a) = 0, then lim f(x) may or may not
x →a



x →a x →a

exist. This will be dealt later on in detail.
= xlim
→a
f(x). xlim
→a
g(x) = m,

i.e., the limit of the product of two functions is equal to Example 1: Evaluate




the product of their limits. lim (x + 2)
(i) x→2 (ii) lim x(x – 1)





x→2
The limit of a product of two, three or, in general, any
lim x + 4
2

definite number of functions is equal to the product of the (iii) x→2 (iv) lim cos (sin x)
x+2





limits of these functions. x→0

(viii) Quotient rule x 2 + 3x + 2


lim (f / g) (x) = lim [f(x) / g(x)] (v) lim
x →a x →a x2 −1



x→1

= xlim
→a
f(x) / xlim
→a
g(x) =  / m, (m ≠ 0) Solution:


(i) x + 2 being a polynomial in x, its limit as x → 2 is given


i.e., the limit of the quotient of two functions is equal to lim (x + 2) = 2 + 2 = 4.
by x→2

the quotient of their limits provided the limit of the divisor
is not zero. (ii) Again x(x – 1) being a polynomial in x, its limit as x → 2

(ix) Power rule is given by

lim x(x – 1) = 2 (2 – 1) = 2.
n
For any positive integer n, lim f ( x ) = n lim f ( x ),
ps

x→2
x →a x →a
(iii) We have lim x + 4 = (2) + 4 = 2.

2 2
el
provided xlim f ( x ) > 0 when n is even.
x→2 x + 2
2+2

→c

( ) ( )
eh

m/n
Also lim (f ( x ) )
m/n
= lim f ( x ) where m/n is a rational (iv) lim cos (sin x) = cos lim sin x = cos 0 = 1.
je

x →a x →a x →0


x→0
number and the limit on the right exists.
iit

(v) Note that for x = 1, the numerator of the given expression is a



The limit of a power is the power of the limit. nonzero constant 6 and the denominator is zero. Therefore,

@

(x) Composition rule 6


the given limit is of the form . Hence, we conclude that

If f is continuous* at x = , then 0
lim x + 3x + 2 does not exist.
2

lim g(f(x)) = g  lim f ( x ) = g () . x→1
x2 −1
x→ a  x → a 

Example 2: Evaluate the following limits:
 


Specially, xlim n (f(x) = n  lim f ( x ) = n 
→a
x→a  x 3 + 2x 2 − 1

(i) lim (2x2 – 3x + 4) (ii)
provided  > 0. x→5 lim
5 − 3x



x→−2

*
We shall learn about continuity in the next chapter. Solution:


A function is f(x) said to be continuous at x = a if xlim
→a
lim (2x2 – 3x + 4)
(i) x→5

f(x) = f(a).
lim (2x2) – lim (3x) + lim 4
= x→5
These results are of fundamental importance but their x→5

x→5

formal proofs will not be given in this book.
lim x2 – 3 lim x + lim 4
= 2 x→5 x→5 x→5

Note:
= 2(52) – 3(5) + 4 = 39


(i) For any polynomial p(x) = c0 + c1x + .... + cnxn and any
(ii) We start by using laws of limit, but their use is fully jus-

real number a,

tified only at the final stage when we see that the limits of
lim p(x) = c + c a + .... = c an = p(a). the numerator and denominator exist and the limit of the
x →a 0 1 n

denominator is not 0.
(ii) Consider the rational function

p( x ) x 3 + 2 x 2 − 1 xlim ( x 3 + 2 x 2 − 1)
f(x) = where p(x) and q(x) are polynomials. For any →−2
lim =

q( x ) x →−2 5 − 3x lim (5 − 3x )
real number a, x →−2

FREE BOOKS FOR JEE & NEET =>(@iitjeeadv)
Limits 1.13

vanish at x = 0, we obtain the limit by substituting x = 0 in


lim x 3 + lim 2 x 2 − lim 1
= x →−2 x →−2 x →−2 the function.
lim 5 − 3 lim x

x →−2 x →−2
03  3.02  0  3 3 1
lim x + 3x − x − 3 =
3 2
 
( −2) + 2( −2) − 1
3 1 2 x→0
x2 + x − 6 02  0  6 6 2
= =– .
5 − 3( −2)

11
Study Tip
Example 3: Find lim
3
2 x 2 − 10 .


x→3 Although such substitution produces the correct answer in
Solution: Because lim (2x – 10) = 2(3 ) – 10 = 8 and 2 2 this particular case, in many limits involving non-elementary


x→3 functions, it produces either an incorrect answer or no answer
lim 3 x = 2 we can write lim 3
2 x 2 − 10 = 2. at all.
x →8 x→3
Also do not assume from these problems that lim f(x) is
x→a
Direct Substitution invariably f(a).
We learned that the limit of f(x) as x approaches a does not n (1 + x ) − x 2 + 2
To evaluate the lim , we notice that the
depend on the value of f at x = a. It may happen, however, that x →0 cos(sin x ) + 1 + sin −1 x
the limit is precisely f(a). In such cases, we say that the limit
can be evaluated by direct substitution. That is n (1 + x ) − x 2 + 2
function is elementary and x = 0 lies in
cos(sin x ) + 1 + sin −1 x
lim f(x) = f(a). (substitute a for x)
ps
the domain. Hence, limit is equal to the function's value at x = 0,
x →a

el
Such well behaved functions are continuous at a – we will
examine this concept more closely in the next chapter. Here we n (1 + x ) − x 2 + 2 0−0+2
eh

i.e. lim −1
= =1.
discuss some limits that can be evaluated by direct substitution. x →0 cos(sin x ) + 1 + sin 1+1+ 0

x
je

In the next section, we will discuss techniques for evaluating


limits for which direct substitution fails. To evaluate lim [ x ] we cannot use direct substitution property
iit

x →2
Basic Elementary Functions as lim [ x ] = [2] = 2, since [x] is a non-elementary function. We
@

x →2
The following functions are called basic elementary functions : know that this limit does not exist.
(i) y = xp (ii) y = ax Sometimes, using direct substitution property on non-elementary




(iii) y = logax (iv) y = trigo x functions may give correct results as in lim [ x 2 ] = [0]2 = 0, but




–1
(v) y = trigo x. it is not advisable to use in general. x →0


Elementary Functions x can be treated as an elementary function since we can
write x = x 2 .
Elementary function is one which may be represented by
a single formula y = f(x), where f(x) is made up of basic
elementary functions and constants by means of finite number Thus, lim x x − 2 = 2 2 − 2 = 0.
x→2
of operations of addition, substraction, multiplication, division
and composition. x2 − 4
Note that piecewise defined functions are non-elementary. For Example 5: Find lim π .
x→1 cos x


example, [x], {x}, sgn x are non-elementary functions. 4
Solution: The given function is an elementary function
Limit of Elementary Functions


and the point x = 1 lies in the domain, and thus
Theorem For all elementary functions, limit at any point in

the domain is equal to the function's value at that point. x2 − 4 12 − 4
lim π = π =–3 2 .

x→1 cos x cos ·1
lim x + 3x − x − 3 .
3 2
4 4
Example 4: Find x→0
x +x−6
2


But the limit of this function as x → 2 can not be found by
Solution: Since the given function is an elementary means of the direct substitution since the denominator turns


function and the polynomial in the denominator does not into zero at the point x = 2.

FREE BOOKS FOR JEE & NEET =>(@iitjeeadv)


1.14  
Differential Calculus for JEE Main and Advanced

Concept Problems B


1. Show that lim
1
= 10. 5. Show that lim cos x  4 tan x  1 .


2  x  2x 4
x →0.01 x→0
x 2
2
2. lim 3x .
Evaluate x→−1 1
6. Why doesn't the limit lim x sin  0 follow from

2x − 1


x 0 x
3. lim cos x .
Evaluate x the product law of limits with f(x) = x and g(x) =
sin x − π

sin(1/x) ?
x
4. Evaluate lim .
cos 1 x

x0

exists a corresponding number δ > 0 such that for all x


1.4 One-Sided Limits satisfying a – δ < x < a ⇒ |f(x) – | < ε and we call it the

For functions that are defined piecewise, a two-sided limit at some left hand limit.
x, where the formula changes, is best obtained by first finding x
For instance, we see that lim  1 because |x|/x is equal
the one-sided limits at the number.
ps x 0 x 

Consider the behaviour of the function to –1 for all x to the left of zero.
el
x  1 if x  0 Right Hand Limit (R.H.L.)
eh

f (x)  
| x | 1 if x  0
Suppose that f is defined on the interval (a, c) immediately to

je

The graph of y = f(x) is shown below : the right of a. Then we say that the number  is the right hand
limit of f(x) as x approaches a from the right, and we write

iit

Y
f (x)  
@

1 f(a+) = xlim
a  ...(2)


0 X provided that we can make the number f(x) as close to  as we
–1 please merely by choosing the point x in (a, c) sufficiently
close to a.
As x → 0, f(x) does not approach a specific number. However, Formally, lim f ( x )   , if for every number ε > 0 there exists
x a
as x approaches 0 through positive numbers, f(x) → 1. Also, a corresponding number δ > 0 such that for all x satisfying a <
as x approaches 0 through negative numbers, f(x) → –1. This x < a + δ ⇒ |f(x) – | < ε and we call it the right hand limit.
behaviour illustrates the idea of one-sided limits, which will x
now be defined. For instance, we see that lim  1 because |x|/x is equal to
x 0 x 

Left Hand Limit ( L.H.L.) 1 for all x to the right of zero.


Suppose that f is defined on the interval (c, a) immediately to
One-Sided Limits and Two-Sided Limits
the left of a. Then we say that the number  is the left hand limit
of f(x) as x approaches a from the left, and we write Suppose that the function f is defined for x ≠ a in a
neighbourhood of the point a. Then the two-sided limit
f(a–) = lim f ( x )   ...(1) lim f ( x ) exists and is equal to the number  if and
x a


x →a
provided that we can make the number f(x) as close only if the one-sided limits lim f ( x ) and lim f ( x ) both exist
x a 
x a

to  as we please merely by choosing the point x in and are equal to .


(c, a) sufficiently close to a. This theorem is useful in showing that certain limits
We may describe the left hand limit in (1) by saying that f(x) →  do not exist, frequently by showing that the left hand
as x → a–, that is, as x approaches a from the left. The symbol and right hand limits are not equal to each other.
a– denotes the left hand side of a. And conversely, if there exists a limit  of a function at the point
Formally, lim f ( x )   , if for every number ε > 0 there a, then there exist limits of the function at the point a both on
x a the right and on the left and they are equal.

FREE BOOKS FOR JEE & NEET =>(@iitjeeadv)


Limits 1.15

Since the one-sided limits are not equal, the corresponding Study Tip
two-sided limit
x 1. When a function is defined only on one side of a point a, then


lim does not exist.
lim f(x) is identical with the one-sided limit, if it exists.

x →0 x
x →a

Let f(x) = 
 x
 3
x 1 For example, if f(x) = x , then f is defined only at and


2  x,
 x1 to the right of zero. Hence, xlim lim

→0 x = x  0 x = 0. Of 

The given function is defined on the entire number line. Let us course, lim x does not exist, since x is not defined
x  0
compute the one-sided limits of this function at the point x = 1.
when x < 0.
f(1– ) = lim ( x )  1
3
x 1 0
2. A function cannot have an ordinary limit at an endpoint of


its domain. It can have a one-sided limit.
f(1+) = x lim (2  x )  3 The function f(x) = 9 − x 2 has the interval –3 ≤ x ≤ 3 as
1 0

its domain. If a is any number on the open interval (–3, 3),


Thus, f(1–) ≠ f(1+). Hence, the given function has no limit at
then lim 9 − x 2 exists and is equal to 9 − a 2 .
the point x = 1. x →a
Now consider a = 3. Let x approach 3 from the left; then

Example 1: The graph of a function g is shown in the lim 9 − x 2 = 0. For x > 3, 9 − x 2 is not defined.
x  3


figure. Use it to state the values (if they exist) of the following
limits: Hence, lim 9 − x2 = lim 9 − x 2 = 0.
lim g(x) x →3 x  3
(a) x 
(b) x lim g(x) 
ps




2 2
Similarly, lim 9 − x 2 = lim 9 − x 2 = 0.
lim g(x) lim g(x)

(c) (d) x  3 x 3
el





x→2 x5
1 |x|
lim g(x) Consider the limit lim sin (e ) .
eh

(e) lim g(x) (f) x 0




x→5




x5
Even if f(0) = π/2 , the function is undefined on both sides
je

Y of x = 0. Hence there is no question of limit at x = 0.
iit

4 [ x 2 ]  2[ x ]  3
In the limit lim 2 , the function is
x 3 [ x ]  4[ x ]  3
@


3 y = g(x)
undefined in the right neighbourhood of x = 3. Hence

R.H.L. cannot be discussed. However, the limit is 1/3 since
1 L.H.L. can be evaluated as 1/3.
1
0 1 2 3 4 5 X
3. There is no shorthand for the two-sided limit of lim

x →0 x

1 1
even though we write lim    and lim   . We
Solution: From the graph we see that the values of g(x) x 0 x 
x  0 x 


approach 3 as x approaches 2 from the left, but they approach may only say that the limit does not exist.
1 as x approaches 2 from the right. Therefore, 2x  1
Note that we do not write lim   . However, it
lim g(x) = 3 and x 1 x  1

(a) x 2

(b) lim g(x) = 1 2x  1
x 2 would be correct to write lim  .

(c) Since the left and right limits are different, we conclude
x 1 x 1

that lim g(x) does not exist. x 3
x→2 Example 2: Find lim and
x 3  3x  2


x 1
The graph also show that
x 3

(d) lim g(x) = 2 and lim .
x5 x 2 ( x  1) 2 ( x  2)


(e) lim g(x) = 2 x 3
x5

Solution: lim
(f) This time the left and right limits are the same and so, we x 1 x 3  3x  2



lim g(x) = 2
have x→5 x 3
= lim 
x 1 ( x  1) 2 ( x  2)
Despite this fact, notice that g(5) ≠ 2.


FREE BOOKS FOR JEE & NEET =>(@iitjeeadv)
1.16  
Differential Calculus for JEE Main and Advanced

x 3 The right and left hand limits are equal. Thus, the limit exists
Now, lim   lim f(x) = 0.
and x→4
x 2 x  3x  23



x 3 The graph of f is shown in the figure.


lim 3 
x  2 x  3x  2


Y

x 3
∴ lim  does note exist
x  2 ( x  1) 2 ( x  2)

4. In some cases a function is defined at x = a however limit

does not exist at x = a and in some cases limit x → a exists but 0 4 X
f(a) is not defined. Note these situations in the graph of the
following function y = f(x) :
 sin[ x ]
 
 
Y  , for [ x ]  0
Example 4: If f(x) =  [ x ] ,



 0 , for [ x ]  0
2

then find lim f ( x ) where [.] denotes the greatest integer
1 function. x →0
0 1 2 3 4 4 X Solution:

At x = 0 , f (0 ) = 1 +
sin[ x ] sin (1)
lim f ( x )  lim   sin 1 .

At x = 1 , f (1–) = 0 ; f (1+) = 1 x 0 x 0 [x] (1)

At x = 2 , f (2+) = f (2–) = 1 ≠ f(2)
ps


At x = 3 , f (3+) = f (3–) = 2 = f(3) lim f(x) = 0 as it is given that f(x) = 0 for [x] = 0.
and x
el

0

At x = 4 , f (4+) = f (4–) = 1 ≠ f(4)
So, x→0
lim f(x) does not exist.

eh

At x = 5 , f (5–) → ∞ (limit dne).




5. In functions involving the greatest integer function,
je

fractional part function and signum function the following Substitution for One-sided Limits
iit

points must be noted : L.H.L. = xlim f ( x )  lim f (a  h )


a h 0

@

(i) f (x) = [ x ] and f (x) = {x} has no limit at all integers.


Substitute x = a – h, assuming h as a small positive quantity.


(ii) y = [f(x)] and y = {f(x)} may not have limit at points
Note that as x → a–, h → 0+. But we often write h → 0.


where f(x) is an integer.
|x| f ( x )  lim f (a  h )
(iii) f (x) = has no limit at x = 0. R.H.L. = xlim
a h 0



x
(iv) y = sgn(f(x)) may not have limit at points where Substitute x = a + h, assuming h as a small positive



f(x) = 0. quantity. As x → a+ , h → 0+.
Non-Existence of Limit x.sgn( x  1)
Example 5: Find lim
x 1
Three of the most common types of behaviour associated with


the non-existence of a limit.
1. f(x) approaches a different number from the right side of Solution: L.H.L. = lim x.sgn( x  1) Put x = 1 – h
x 1



a than it approaches from the left side. = hlim(1  h ) sgn(1  h  1)
0
2. f(x) increase or decreases without bound as x approaches


a. = hlim(1  h ) sgn( h )
0

3. f(x) oscillates too much as x approaches a.
1  h )(1)  1 .

= hlim(
0
 x4
 x4

if
Example 3: If f(x) =  determine
 8  2x
 x4 R.H.L.= lim x. sgn( x  1) Put x = 1 + h


if
whether lim f(x) exists. x 1

x→4
lim
Solution: Since f(x) = x − 4 for x > 4, we have x 4  = hlim(1  h ) sgn(1  h  1)
0



lim
f(x) = x 4 x − 4 = 4 − 4 = 0. 

1  h ) sgn h = lim(1  h ) .1 = 1
Since f(x) = 8 – 2x for x < 4, we have = hlim(
0 h 0


lim f(x) = lim (8 – 2x) = 8 – 2.4 = 0. Since, L.H.L. ≠ R.H.L., lim x sgn( x  1) does not exist.
x 4 x 4

x 1

FREE BOOKS FOR JEE & NEET =>(@iitjeeadv)


Limits 1.17

Example 6: Find xlim[ x] lim   2 cos(h)  = – 2


0 
→2 = h



Solution: L.H.L. = lim[2  h ] = lim 1  1 .  

R.H.L. = lim  2 sin   h   
5  
h 0


h0 h 0   4 4 
R.H.L. = hlim[ 2  h ]  lim 2  2 .
0 h 0 lim   2 cos(h)  = –2
= h 0 


Since, L.H.L. ≠ R.H.L., lim[ x ] does not exist.
Since, L.H.L. = R.H.L., the required limit is –2.

x →2
Example 7: Evaluate lim{x} .
x →2 Example 10: Evaluate the left hand and right hand limits




of the function
Y
 ( x 2  6 x  9)
 , x3
1

2
1

f(x) =  ( x  3) at x = 3.


+

x
x


Solution:
 0 , x3


–1 –h 0 1 2–h 2 X

Solution: The given function can be written as


{2–h} = 2 – h – 1 = 1 – h



| x  3 |
, x3
f(x) =  ( x  3)
We have {2 – h} = 2 – h – 1 = 1 – h
L.H.L. = lim{2  h}  lim(1  h )  1 .
h 2 h 0

 0 , x3

R.H.L. = hlim{2  h}  lim h  0 ∴ L.H.L. = x
lim f(x) = lim (3 – h)
0 h 0 3
ps h→0

Note that if h as a small positive quantity{–h} = 1 – h | 3 h 3| | h | h
el
 1  = lim = lim = lim = –1
Example 8: Find lim   x    {x 2 }  h 0 (3  h  3) h 0 (  h ) h 0  h

eh

x 2   2



and R.H.L. = x
lim f(x) = lim (3 + h)
je

3
Solution: L.H.L. =
h→0



| 3 h 3|
iit

|h| h
= lim = lim = lim = 1
= lim 2  {4  4h  h }
2 (3  h  3)
h 0 h →0 h

h →0 h
@

h 0

Hence the left hand limit and right hand limit of f(x) at

= hlim 2  {4h  h 2 } = lim 2 + {– h (4 – h)} x = 3 are –1 and 1 respectively.
0 h→0

2  1  (4h  h 2 ) , since 4h – h2 is a small positive cos[ x ], x  0
= hlim
0 Example 11: Let f(x) = 
| x | a , x  0





quantity.
=3 Find the value of a, given that x→0 lim f(x) exists, where

 1 [ . ] denotes the greatest integer function.
R.H.L. = lim   2  h     {(2  h ) 2 }
h 0  2
  lim f(x) exists
Solution: Since x→0


= hlim 2  {4  h  4h} 2
lim f(x) = lim f(x).
0 x0  x0 


⇒ lim f(0 – h) = lim f(0 + h)
= hlim 2  {4h  h 2 } h→0 h→0

0

⇒ lim |0 – h| + a = lim cos [0 + h]
lim 2 + 4h + h2 = 2 h→0

h→0
= h→0


a = cos 0 = 1
Since, L.H.L. ≠ R.H.L., the limit does not exist.

∴ a=1

Example 9: Find lim5 [sin x + cos x]


x
4 Study Tip
     If f(x) is an even function, then
Solution: L.H.L. = lim5  2 sin  x  4  
  


x
4 (i) lim f ( x )  lim f ( x )
x 0  x 0 


  5  
L.H.L. = hlim  2 sin   h   lim f ( x )  lim f ( x )
0   4 4  (ii) x a  x  a 


FREE BOOKS FOR JEE & NEET =>(@iitjeeadv)
1.18  
Differential Calculus for JEE Main and Advanced

If f(x) is an odd function then


sin −1 x
(i) xlim0 
f ( x )   lim f ( x ) .
x 0 
For x < 0, >1
x

lim f(x) exists, then lim f(x) = 0.
Further if x→0  sin 1 x 
x→0
∴ L.H.L.= lim   = 1. Hence P = 1.

x 0 
 x 


(ii) lim f ( x )   lim f ( x )
x a  x  a 

 1 1 
Example 13: Find lim  tan 1  cot 1 2 
 sin 1 x  x 0  x 



x
Example 12: Evaluate lim   where [ . ] denotes
x 0 
 x 


 1 1 
Solution: lim  tan 1  cot 1 2 
x 0  x 



the greatest integer function. x
1 1
 sin 1 x  = lim tan 1  lim cot 1 2
Solution: Let P = lim   x 0 x x 0


x 0 
x
 x 


1 1
= lim tan 1  0 = lim tan 1
sin −1 x


x 0 x x 0 x
For x > 0, sin–1 x > x ⇒ >1

x Now we evaluate the one sided limits :
 sin 1
x 1 1 
∴ R.H.L. = lim   =1 L.H.L. = lim tan 
x 0  x 0 x 2



 x
1 
R.H.L. = lim tan 1  .
ps
sin −1 x x 0
x 2
The function is even.
Finally, the limit does not exist.
el
x
eh
je

Concept Problems C
iit

@

Y
1. Find lim f(x) f(x)

x / 2 2
Y
f(x) 1
2

–1 0 1 2 3 4 5 X

1
4. The following figure shows the graph of a function f.

Decide which of the given limits exist and evaluate those
0 π/2 π X which do.
2. Find lim f(x) (a) lim f ( x ) (b) lim f ( x )
x 0 x →1


x →1


(c) lim f ( x ) (d) lim f ( x )
Y


x  2 x  2
f(x) Y
2
3
1
2
0 1 2 X
1
3. Find (i) lim f(x), (ii) lim f(x), (iii) lim f(x)

x→0 x→1 x→3 0 1 2 3 4 X

FREE BOOKS FOR JEE & NEET =>(@iitjeeadv)


Limits 1.19

5. The following figure shows the graph of a function f.


x2

Decide which of the given limits exist and evaluate those 10. Given that f(x) = , show that
( x − 1)( x − 2)


which do.
(a) lim f ( x ) (b) lim f ( x ) (i) lim f(x) = ∞, lim f(x) = –∞,



x →1 x →2



x1 x1
(c) lim f ( x ) (d) lim f ( x ) (ii) lim f(x) = –∞, lim f(x) = ∞,
x →3 x  4





x 2 x 2
Y
(iii) lim f(x) = 1 = lim f(x).



3 x x

11. Find the left and right hand limits of the function


2
ψ(x) =   x  1 for x  1, at the point x = 1.

 2 x  1 for x 1
1
0 1 2 3 4 X  2 x  3, x  2

12. Find the limit of f (x) =  at x = 2.
4  x , x  2
2
Use the graph of f and g to find the limits that exist.


6. 

Y
y = g(x)  2( x  1) if x  3
Y y = f(x) 2 
13. Find lim f ( x ) where f(x) = 4 if x  3
1
x →3  2
1 x  1 if x  3
0 1 2 X –1 0 1 2 X 14. Using graphs find the limit (if it exists):
ps

–2 x 2  2 ,
 x 1
(a) (x) =  , lim f ( x )
el
 1 ,
 x  1 x →1


lim f(x) + g(x), a = 0, 2
eh

(i) | x 5|
x →a (b) lim


x 5


x 5
je

(ii) lim f (x)


lim tan 2 x .


iit

x→2 1+ g ( x ) (c)
x  / 2


Evaluate the following limits :
@

7. 15. Suppose that f is an odd function of x.




1/( x  5)  x Does knowing that lim f(x) = 3 tell you anything about
(i) lim 10 (ii) lim sin –1
 log 3 3 

x 5    x 0





x1
lim f(x) ?
( x  1) −1 1 x 0
(iii) lim (iv) lim tan
( x  1)  2



x 1 x→0 x 16. Suppose that f is an odd function of x. Does knowing that

lim f(x) = 7 tell you anything about either lim f(x) or
8. Evaluate lim sin–1 (sec x). x  2 x 2

x→0

9. Evaluate lim cos–1(1 + tan x). lim f(x)?


x 2

x0

Practice Problems B

19. Evaluate the limits

17. Evaluate (i) lim f ( x ) (ii) lim f ( x ) sin[cos x ]

x →1 x 1 (i) lim sin–1 [sec x] (ii) lim




x→0 1+ [cos x ] x→0
x | x 3|
where f ( x )  . where [.] denotes the greatest integer function.
( x  x  6) | x |
2


 2 1
 x sin , x0
18. Evaluate the following limits : 20. If f(x) =  x , find lim f ( x )

(i) lim x sgn( x  1) x 2 x →0

 , x  0


x1
  
tan 2 x 21. Evaluate lim 1 – x + [x] – [1 – x],

(ii) lim  tan   x   x1
 



  8
x where [.] denotes the greatest integer function.

4

FREE BOOKS FOR JEE & NEET =>(@iitjeeadv)


1.20  
Differential Calculus for JEE Main and Advanced

2 n 1
x2 −1
 [x r ]  n  1 (i) lim
| x −1 |



r 1
x→1
22. Evaluate lim
1  [ x ] | x | 2 x 2{x} − 4

x →0 ¯
(ii) lim
[x] − 3



where [.] denotes the greatest integer function. x→2

sin{x  10}  | x3 |  x 3 
23. Find lim , (iii) lim     , a > 0
x8 {10  x}
x →a   a  

a





where {.} denotes the fractional part function. x  1 0  x 1

(iv) lim f(x) where f(x) = 
24. Find the left and right hand limits of the function 3x  2 1  x  3



x→1

1
φ(x) = 1
at the point x = 2. 27. In each of the following functions :


x  2 x 2 (a) Sketch the graph of the function f.



(b) For each integer n, evaluate the one-sided limits
25. Evaluate the one-sided limits (where [.] denotes the



lim f ( x ) and lim f ( x ) in terms of n.

greatest integer function): x  n x  n
1 [x 2 ] − 1 (c) Determine those values of a for which lim f ( x )
(i) lim (ii) lim x →a



x→1 ( x − 1) 2 x2 −1 exists.



  
x→1

(i) f (x)  
2 if x is not an integer,
1
(iii) lim (iv) lim x(–1)[1/x]


2  (1) if x is an integer,
x
2 − 21/ x



x→0 x→0
26. Evaluate the one-sided limits (where [.] denotes the
ps (ii) f(x) = [10x]

greatest integer function) :


1
el
(iii) f(x) = x – [x] – .


2
eh
je
iit

1.5 Determinate and Indeterminate (ii) lim(f ( x )  g ( x ))  



x a


@

Forms (iii) lim f ( x )g ( x )   (for b  0)


x a


Until now we have found limits using graphs or the direct
substitution property. We shall now classify the problems of (iv) lim
g( x )
  (for b  0)
limit in two ways: x a


f (x)
(i) Determinate form: In which the limit can be predicted f (x)
(v)  0.

lim
apriori without resorting to special methods. This is so x a


g( x )
named because two problems having the same form have Here we give a list of determinate forms. One should try to
equal limits. understand the meaning of these forms :
2x cos x
For example, lim and lim are in the same (i) 0 + c = c (ii) 0 + 0 = 0
x




x 1 x →0 ln | x |

tan (iii) c + ∞ = ∞ (iv) ∞ – c = ∞




2 (v) c – ∞ = – ∞ (vi) 0 + ∞ = ∞




form
1
and we see that both of them have the limit 0. (vii) 0 – ∞ = – ∞ (viii) ∞ + ∞ = ∞




∞ (ix) 0 × c = 0 (x) ∞ × c = ∞ , c > 0




Hence we call
1
as a determinate form and we are sure (xi) ∞ × ∞ = ∞ (xii) 0 × 0 = 0




∞ c 0
1 (xiii)    , c > 0 (xiv) =0, c ≠ 0
that any problem in




form will have limit 0. 0 c

f (x)  0
If lim f(x) = b and lim g(x) = 0 then lim is said (xv) 
 (xvi) 0





x →a g ( x )

x →a x →a 0
b (xvii) ∞∞ = ∞ (xviii) 0∞ = 0
to be of the form . If b > 0 and g(x) → 0+ then the




0 Consider some examples of determinate form :
limit is ∞. 1
(i) lim 2 = ∞
Note: Let lim f ( x )  b, lim g ( x )   , then we have x →0 x



x a

x a
1
(i) lim (f(x) + g(x))   (ii) lim 4 = ∞,
x →0 x




x a

FREE BOOKS FOR JEE & NEET =>(@iitjeeadv)


Limits 1.21

 1 1  x2 −1 Several indeterminate forms arise from the limit lim [f(x)]g(x)


(iii) lim  2  4  = lim = –∞, x →a
x→0  x x  x4


x→0
1. lim f(x) = 0 and lim g(x) = 0 type 00
x →a x →a



 1 
(iv) lim  2  2  = ∞, 2. lim f(x) = ∞ and lim g(x) = 0 type ∞0
x→0  x 




x →a x →a

 1  3. lim f(x) = 1 and lim g(x) = ± ∞ type 1∞


(v) lim 1  x   = ∞, x →a x →a



x 0  x2 


Each of these three cases can be treated either by taking the
 1   1  natural logarithm:
(vi) lim  2  2   1  x  2   = lim (1 – x) = 1,
x 0  x   x  let y = [f(x)]g(x),


x→0
x x
 x   1  1 then n y = g(x) n f(x)
(vii) lim    xlim
x   1  2 x 
    0,
  2  1 / x  or by writing the function as an exponential


2
(viii) lim (1  cos x )( x  2)  lim (1  1) 2  4 . [f(x)]g(x) = eg(x) n f(x)


x 0 x 0


In either method we are led to the indeterminate product g(x)
(ii) Indeterminate form: In which the limit cannot be n f(x), which is of type 0 · ∞.

predicted apriori. Here special methods are required to The reason why these forms carry the label "indeterminate" is not
find the limit. In this case two problems having the same that they cannot be resolved, but rather that there is no predicting
form may not have equal limits. what the limit of the given function will be, or indeed whether it will
For example, the limit of a quotient, exist at all, until the given function receives a thorough investigation.
ps

It is tempting to argue that an indeterminate form of type 0 .
f (x)
lim where lim f(x) = 0 and lim g(x) = 0, is ∞ has value 0 since “zero times anything is zero.” However,
el
x →a g ( x ) x →a x →a

this is fallacious since 0 . ∞ is not a product of numbers, but
rather a statement about limits.
eh

called an indeterminate form of type 0/0. Consider the



following limits in 0/0 form: The form (∞ + ∞), on the other hand, is determinate in the sense
je

x x2 x that unfailingly the sum function has limit ∞ if both summand


1 , lim  0 , lim 3   functions have limit ∞ at the prescribed point of approach.
iit

lim
x 0 x

x 0 x x 0 x
The following examples show how easy it is to construct
@

We notice that the result varies quite significantly even if they are in the quotient functions and sum functions with indeterminate
same indeterminate form. The word “indeterminate” here refers forms having an arbitrary real number C as a limit, or having
to the fact that the limiting behaviour of the quotient cannot be no limit whatever.
determined without further study. The expression “0/0” is just a
The form 0/0.
device to describe the circumstance of a limit of a quotient in
C sin x x sin(1 / x )
which both the numerator and denominator approach 0. lim  C while lim does not exist.

x 0 x →0

x
The important indeterminate forms are : x
0 , ∞ , 0 × ∞, ∞ − ∞, 0º, ∞º and 1∞ The form (∞/∞).


0 ∞ x (sin x  2)


Cx
lim C while xlim does not exist.


Note that ‘0’ doesn’t mean exact zero but it represents a value x

x  x
approaching towards zero. The form (∞ – ∞).
The behaviour of ‘1’ and ‘∞’ are similar. lim (x – (x – C)) = C while lim (x2 – x) does not exist.
x 

x 
Indeterminate Products
The above examples were artificially framed to point that
If lim f(x) = 0 and lim g(x) = ∞ (or – ∞), then it isn't clear
x →a x →a "indeterminate" means "unpredictable without special attention
what the value of lim f(x)g(x), if any, will be. There is a to the functions involved".
x →a There is a myth that circulates among students which states
struggle between f and g. If f wins, the answer will be 0; that all indeterminate forms of types 0º, ∞º, and 1∞ have value
if g wins, the answer will be ∞ (or – ∞). Or there may be a 1 because “anything to the zero power is 1” and “1 to any
compromise where the answer is a finite nonzero number. This power is 1.” The fallacy is that 0º, ∞º, and 1∞ are not powers
kind of limit is called an indeterminate form of type 0 · ∞. of numbers, but rather descriptions of limits. The following
We can deal with it by writing the product fg as a quotient : examples show that such indeterminate forms can have any
f g
fg = or fg = positive real value :

1/ g 1/ f
(a) lim [x(ln a)/(1 + ln x)] = a (form 0º)



This converts the given limit into an indeterminate form of x0
0
the type or ∞/∞ . (b) lim [x(ln a)/(1 + ln x)] = a (form ∞º)



0 x

FREE BOOKS FOR JEE & NEET =>(@iitjeeadv)


1.22  
Differential Calculus for JEE Main and Advanced

(c) lim [x(ln a)/(1 + ln x)] = 1 (form 1∞) (ii) ∞ ∞ ∞









x→1 (iii) 0 indeterminate





We can easily verify these results. lim f(x) lim g(x) lim [f(x)/g(x)]




x →a x →a x →a
Find whether the following limits are in determinate /
indeterminate form. Also evaluate the limit in case of (i) ≠0 0 ∞





determinate form. (ii)  ∞ 0





2 (iii) ∞  ∞
0





(i) lim (iv) 0 0 indeterminate
x 0 1  21/ x
2






(v) ∞ ∞ indeterminate





1 2 2
These results are valid even when x tends to
as x → 0, → 0, 21/ x → ∞ , → 0.
∞ a–. a+, ∞ or – ∞.

x


f (x)  5
sin 1 x  /2 Example 1: If lim = 3 then find lim f ( x ) .
 0 , since  →0 x 2 x  2



(ii) lim x→2
   


x 1 

tan x Solution: Since the denominator is approaching 0, we



2 can have limit only through 0 form.
(iii) lim (e  x )   , since ∞ + ∞ = ∞.
x
0
Hence lim f ( x ) must be 5.


x 
x→2
lim(1  sin x )1/ x
2

(iv) (1∞ indeterminate form)


f (x)  5


x 0
Note that if lim = 4 then also lim f(x) is 5.
1
x2 x 2 x→2
(v) [cos
lim(1  x ]) x2 is not in 1∞ indeterminate form
f (x) 0


x 0
Also if lim 2 = 2 then using form we have
exact 1
ps 0
since (exact 1)∞ = 1 (determinate form),
x→0 x
el
1 f (x)
(vi) [cos
lim(1 x ]) x = 1. lim f ( x )  0 and further lim = 0.
eh

x 0   x0 x→0 x


exact 1
je

 1 1  Example 2: Find the left hand and right hand limits of


  (∞–∞ indeterminate form)


(vii) lim  1
iit

x 0  x 2 sin 2 x 


the function f ( x )  as x → 3.
[x 2 ]  1  exact 0  x  21/( x  3)
@

(viii) lim 0  0  = 0
x 1 x  1   Solution: If x → 3–, then






Let us try to deduce conclusion about the limits: 1/(x – 3) → –∞ and 21/(x–3) →0.

lim [f(x) + g(x)] , lim [f(x).g(x)] and lim [f(x)/g(x)] based Consequently, lim f ( x )  1 / 3 .
x →a x →a x →a x  3
upon the individual behaviours of f(x) and g(x). Now if x → 3 , lim f ( x )  0 . +

Assume that  is a positive real number. x  3

lim f(x) lim g(x) lim [f(x) + g(x)] Example 3: Does the function





x →a x →a x →a
 1  1
(i)  ∞ ∞ y =  sin   sin  tend to a limit as x tends to 0?
 x  x




(ii)  –∞ –∞
Solution: The function is equal to 1 except when




(iii) ∞ ∞ ∞






(iv) –∞ –∞ –∞ sin(1/x) = 0; i.e. when x = 1/π, 1/2π,.... For these values the




(v) ∞ –∞ indeterminate formula for y assumes the meaningless expression 0/0, and




lim f(x) lim g(x) lim [f(x).g(x)] y is therefore not defined for an infinitely many values of x
x →a x →a x →a



near x = 0. Note that this is not the indeterminate form 0/0.
(i) ≠0 ∞ ∞ Hence the limit does not exist.




Concept Problems D

1. Find whether the following limits are in indeterminate (iii) lim x n x



form. Also indicate the form. x→0
1 1− x 1 1 
(i) lim (ii) lim (iv) lim   2 
x→1 1 − x 2 x→0  x x 






x→0 x

FREE BOOKS FOR JEE & NEET =>(@iitjeeadv)


Limits 1.23

(v) lim (sin x)x (vi) lim (n x)x lim e tan x  1 1
(iii) (iv) lim tan–1




x→0 x→0 





x x→0
1
1 tan x x
2 e
lim 1  sin x 
(vii) x→0
x
lim (1)1/x
(viii) x→0 5. Evaluate the following limits:





2. Suppose that limx → a f(x) = ∞ and limx → a g(x) = c, where sin 1 x 1

(i) lim lim
(ii) x→0
c is a real number. Prove each statement. x





x→1 ln | x |
(a) lim [f(x) + g(x)] = ∞
tan
x →a 2


e x ln 2  − 
1
(b) lim [f(x) g(x)] = ∞ if c > 0 (iii) lim (iv) lim x 2 1 − e x  2

x →a






x
ex x →0
 
(c) lim [f(x) g(x)] = – ∞ if c < 0
x →a x


6. Show that lim  x    .
Let lim f ( x )  0 with f(x) ≠ 0 for x ≠ a, lim g ( x )  b ≠ 0.
 


3.
x a
x  e
x a

g( x ) f (x) − 5
Prove that lim  7. If xlim
→4
= 1, find xlim
→4
f(x).
x−2


x a f (x)
f (x)
4. Prove that the following limits donot exist : 8. If lim 2 = 1, find


x 2 x
1
lim tan x f (x)
(i)  (ii) lim (2) x (i) lim f(x) and (ii) lim
x x 2 x 2 x







x→0
2 tan 2 x
ps
el
Concept Problems C
eh

je

9. Find xlim
x
1 
lim   ln x 
iit

(e)
2 |x − a|
→a

x 


x 0
@

10. Find the following limits by inspection.



(f) lim (x + x3)
x


x
(a) lim


x 0 ln x  x3
 3x 2
 1  1
x
x3 11. Evaluate lim  2 
(b) lim x   2 x  1 

x  e  x  


(c) lim (cos x)tan x 12. If f(x) = [x2 + 1][x + 1], where [.] denotes the greatest integer
x(  / 2) 



(d) lim (ln x) cot x
x0
function, find lim f(x).


x→1

METHODS OF EVALUATING LIMITS of the given expression otherwise we repeat the process till
we get rid of the indeterminate form (0/0).
x3
1.6 Factorisation and Cancellation of Example 1: Find lim
.
x2  9 x 3



Common Factors Solution: Here, the denominator tends to zero as


If f(x) and g(x) are polynomials such that f(a) = g(a) = 0, then x → 3 and the numerator also tends to zero. But since x2 – 9
= (x – 3) (x + 3), we have
f (x)
(x – a) is a factor of both f(x) and g(x). Now to solve lim ,
x →a g ( x ) x 3 x 3 1 1
 lim lim  lim 
we cancel the common factor (x – a) from both the numerator x 2  9 x  3 ( x  3)( x  3) x  3 x  3 6
x 3

and denominator, and again put x = a in the given expression. In the solution of this problem we cancel x – 3, and one may
If we get a meaningful number, then the number is the limit think that this is illegitimate since x → 3, and the division by

FREE BOOKS FOR JEE & NEET =>(@iitjeeadv)


1.24 Differential Calculus for JEE Main and Advanced

zero is not allowed. But this is not the case here : the functions x2  4
x 3 Example 4: Find lim  .
y 2 and y  1 coincide identically for all x ≠ 3, and



x→2 cos x
x 9 x 3 4
Solution: We put x – 2 = z, i.e. x = z + 2, and as
the definition of a limit of a function for x → a, does not involve



x → 2 we have z → 0.
the value of that function at the point x = a itself, and therefore z(z  4) z ( z + 4)
lim x  4
2

the limits of the above functions as x → 3 are equal to each lim lim
x→2  = z 0  = z→0 π
other. The essence of this transformation is that the limit of the cos x cos (z  2) − sin z
4 4 4
new function is found easier than that of the original function. π
x2 + x − 6 z ( z + 4)
z
Example 2: Evaluate lim π lim 4 16
x →−3 x+3 lim lim


= z→0 π = – 4 z→0 π . z→0 (z + 4) = – π .
Solution: We cannot apply direct substitution because − sin z sin z


the limit of the denominator is 0. 4 4

x2  x  6 (
lim x 2 + x − 3 = 0) Note:


x →−3 1. xn – an = (x – a)(xn–1 + xn–2a + xn–3a2 + xn–4a3 +...
x 3 lim ( x + 3) = 0



+ xan–2 + an–1)



x →−3

( )
where n is even or odd positive integer.
lim x 2 + x − 6 = 0

x  x   x →−3 2. xn + an = (x + a)(xn–1–xn–2a + xn–3a2 – xn–4a3 + ...

x lim ( x + 3) = 0 + (–1)n–1an–1)

x →−3

where n is odd positive integer. This formula is not applicable
Direct substitution fails here. when n is even.
ps
Because the limit of the numerator is also 0, the numerator 5
(1  x )3  1
Example 5: Find lim .
el
and denominator have a common factor of (x + 3). Thus, for


x0 x
all x ≠ – 3, we can cancel this factor to obtain Solution: Let us put 1 + x = y5. As x → 0, y → 1.
eh


x  x   ( x  3)( x  2) Then we have
= x – 2, x ≠ – 3
je

=
x x 3 (1  x )3  1 = lim y  1
3

5
iit

It follows that lim y 1 y  1


5

x0 x
@

x2 + x − 6
lim = xlim ( x − 2) = −5. y2 + y + 1 3
x →−3 x+3 →−3 lim =

Although correct, the second equality in the preceding
y →1 y + y3 + y 2 + y + 1
4
5

computation needs some justification, since cancelling the factor x3  x 2  x  1
x + 3 alters the function by expanding its domain. However, Example 6: Find lim
x 1 x 3  x 2  x  1


the two functions are identical, except at x = – 3. From our previous Solution: Here we have an indeterminacy of the form
discussions, we know that this difference has no effect on the


0/0. Let us factorize the numerator and the denominator of

limit as x approaches – 3. the function
x 6  24 x  16
Example 3: Find lim x3  x 2  x  1 x 2 ( x  1)  ( x  1)
x 3  2 x  12  lim


x→2 lim
x 1 x 3  x2  x  1 x 1 x 2 ( x  1)  ( x  1)
lim x  24 x  16
6
Solution: x→2
( x  1) 2 ( x  1)  lim x  1  0  0


x 3  2 x  12  lim .
x 1 x  1

x 1 ( x  1) ( x  1) 2 2
lim ( x  2)( x  2 x  4 x  8x  16 x  8)
5 4 3 2
= x→2 2
( x  2 x  6)( x  2) x 3  1000
Example 7: Find lim
168 x 3  20 x 2  100 x x 10


=
= 12
14 Solution: This is also an indeterminacy of the form 0/0.


Substitution for Limits x → a We have
Suppose that lim f(x) =  x 3  1000
x →a lim 3

Let x = a + t, then f(x) = f(a + t). x 10 x  20 x 2  100 x

If x → a then t → 0 and f(a + t) → , and we write
( x − 10)( x 2 + 10 x + 100)
lim f (a  t ) = . = lim
x →10 x (x − 10) 2

x a

FREE BOOKS FOR JEE & NEET =>(@iitjeeadv)
Limits 1.25

x 2  10 x  100 1 1
= lim = – lim =– 2
x ( x  10) x ( x  y  2)


x 10 y 2

x
Here, y is a variable, so that it might be thought that we are
The numerator of the fraction tends to 300 and the denominator
dealing with functions of two variables. However, the fact that
tends to zero. Consequently, the fraction in question is an
x as a variable plays no role in this problem; for the moment, x
infinitely large quantity and
can be considered a constant.
x 3  1000
lim does not exist. 2 x  23 x  6
x 10 x  2 x 2  100 x
3 Example 11: Evaluate lim

2 x / 2  21 x



x 2

lim
x 3  3x 2  x  3 2 x  23 x  6  0
Example 8: Find x→−3 Solution: lim x /2 1 x  form 0 
x x 6
2
2  




x 2


2
Solution: At the point x = – 3 both the numerator and 2 2x
 8  6.2 x
= lim


2x / 2  2


the denominator turn into zero. x 2
We have x2 + x – 6 = (x + 3) (x – 2) and
(2 x  2)(2 x  4)
x3 + 3x2 – x – 3 = (x + 3) (x2 – 1), and hence, on cancelling = lim
2x / 2  2

x 2
the factor (x + 3), we obtain
(2 x  2)(2 x / 2  2)(2 x / 2  2)
lim x  3x  x  3 = lim x − 1 = ( −3) − 1 = – 8
3 2 2 2
= lim
2x / 2  2

x 2
x→−3
x2  x  6 x→−3
x−2 −3 − 2 5
In the same manner we can find = 2 × 4 = 8.
ps

x 3  x 2 log x  log x  1
lim x  3x  x  3 dne, lim x  3x  x  3 = 0.
3 2 3 2
el
Example 12: Evaluate lim
x 1 x2 1


x→2 2
x x 6 x→1 2
x x 6
eh

Solution: The given limit = lim


 x  1   x  1 log x
3 2
je

Example 9: Evaluate


x 1 x2 1



lim  1  2 (2 x  3)  ( x − 1) ( x )
iit

x→2 
2
+ x +1 − ( x − 1)( x +1) logx
 x  2 x 3  3x 2  2 x  = lim
@

( x − 1)( x +1)

x→1
Solution: We have
 x  1  x + x +1   x +1 logx 


2

lim  1  2 (2 x  3)  = lim
x→2   x1  x  1 x +1
 x  2 x 3  3x 2  2 x 

12 +1+1  1+1 log1
lim  1  2 (2 x  3) 
3
= = .
= x→2
 x  2 x ( x  1)( x  2)  1+1 2

3x 2  ax  a  3
lim  x ( x  1)  2(2 x  3) 
= x→2 Example 13: If the limit lim exists,
x ( x  1)( x  2) x2  x  2


x 2
  find a and the limit.

  Solution: We see that the denominator → 0. For
lim  x  5x  6 
2


= x→2 0
 x ( x  1)( x  2)  the limit to exist, we must have the   form. Hence the
0

numerator must → 0, i.e.
lim  ( x  2)( x  3)  lim 3x2 + ax + a + 3 = 0
= x→2  

x2
 x ( x  1)( x  2) 

⇒ 12 – 2a + a + 3 = 0 ⇒ a = 15.

lim  x  3  = – 1 3x 2  ax  a  3 2 x 2  15x  18
= x→2 Now lim = lim
 x ( x  1)  x2  x  2 x 2 x2  x  2

2 x 2

 1 
3( x  2)( x  3)
1 1  = lim  1 .
Example 10: Find lim     x 2 ( x  2)( x  1)

 y  2  x  y  2 x  


y 2

f (x)
xx2y 1 Example 14: If lim  2 then evaluate the


lim
Solution: y · x2 x0
2 x ( x  y  2) y  2


following limits, giving explicit reasoning.

FREE BOOKS FOR JEE & NEET =>(@iitjeeadv)


1.26 Differential Calculus for JEE Main and Advanced

 f (x) 
(i) lim  f ( x )  , (ii) lim  ⇒ lim [A] = 0 and lim [A] = – 1
where [ . ] x0
x0  x 


x→ 0


x0
⇒ The given limit does not exist.


denotes the greatest integer function.
 f (x)  Example 15: Discuss the behaviour of
Solution: (i) Let l = lim f  x   = lim  2 · x 2 



x0  x  a 0 x m + a1x m +1 + .....a k x m + k


x0
φ(x) =
Now argument of G.I.F is tending towards zero and b0 x n + b1x n +1 + .... + b x n + 


f (x)
from positive side as lim  2 and x2 → 0+ where a0 ≠ 0, b0 ≠ 0 as x tends to 0 by positive or negative
x0 x 2


f (x) 2 values.
we have · x → 0+ ⇒ l = 0.
x2 lim φ(x) = 0.
Solution: If m > n, x→0



 f (x)   f (x)  lim φ(x) = a /b .
If m = n, x→0
(ii) We write lim   = lim  2 · x  0 0


x0  x  x0  x 
If m < n and n – m is even, φ(x) → ∞ or φ(x) → –∞ according
and assume A = f ( x ) · x as a0/b0 > 0 or a0/b0 < 0.
x2 If m < n and n – m is odd, then φ(x) → ∞ as x → 0+ and φ(x)
Now x → 0 ⇒ A → 0+
+ → –∞ as x → 0–, or φ(x) → –∞ as x → 0 and φ(x) → ∞ as

and x → 0– ⇒ A → 0– x → 0–, according as a0/b0 > 0 or a0/b0 < 0.

Concept Problems E
ps

el
eh

1. Evaluate the following limits:


(c) lim f ( x ) (d) lim f ( x )

x →−3 x →0




lim x  3x  9 x  2 ( x  1) 2
3 2
je

(i) (ii) lim


x→2
x3  x  6 ( x  1) 2  1 [x ]  1
2



  
x 0
iit

4. Evaluate limit where [.] denotes the greatest


x2 1

x1
(x  h )2  x 2 1 t
@

(iii) lim (iv) lim integer function.


h 0 1  t5



h t1 

2. Evaluate the following limits:


lim
x 2  9 x  20

5. Evaluate x→ where [.] denotes
| y 1 | y  1 5 x [ x ]

(i) l im


y 1
| y 1 | y 1 the greatest integer function.

x 3  27 6. Evaluate the following limits where [.] denotes the greatest

(ii) lim integer function :
| x  3 | ( x  3)


x 3
x  [x] x  [x]
(i) lim (ii) lim
Let f(x) = x + x − 2 find
2
x2 x2



3. x 2 
x 2 

x 2 + 2x − 3
(iii) lim
[ x 2 ]  [ x ]2 (iv) [x3 ]  x3
(a) lim f ( x ) (b) lim f (x) lim
[x]  x



x →−2 x →1 x 1 x2 1 x 10




Practice Problems D

7. Evaluate the following limits:
x 7 − 2x 5 + 1

(i) lim x − 3x + 2
4 (iii) lim
x →1 x 3 − 3x 2 + 2




x→1 x 5 − 4 x + 3

(iv) lim x − 6 x − 27
4 2
x 3 − 4 x 2 + 5x − 2
(ii) lim


x →1 ( x 2 − 1) 2 x →−3 x 3 + 3x 2 + x + 3


FREE BOOKS FOR JEE & NEET =>(@iitjeeadv)
Limits 1.27

8. Evaluate the following limits :


x 3 + cx 2 + 5x + 12

lim exists. Also find the corresponding
t t −t 2
x 2 − 7 x + 12


x →3
(i) lim − 4
+


t →1 t 1 t −1 limit.
10. Evaluate the one-sided limits in the following :
2 sin 2 x + sin x − 1


(ii) lim
x 2[x 2 ]


x →π / 6 2 sin 2 x − 3 sin x + 1 (i) lim
x→ 2
(1+ | x − 2 |)




1 − cot 3 x
(iii) lim
2 − cot x − cot 3 x x (e[ x ]+|x| − 2)


x →π / 4 lim
(ii) x→0
[ x ]+ | x |



9. Find a number c so that where [.] denotes the greatest integer function.


1.7 Rationalization  2  2
= lim   = = 1.

x→0  1 + x + 1 − x  2
If in any limit, the denominator or numerator involves the

radical sign then we can rationalize the irrational expression by
multiplying with their conjugates to remove the indeterminacy. (ii) We have lim
 (2 x − 3) x − 1

( ) 
x→1  
2x 2 + x − 3


4 − 15x + 1  
Example 1: Evaluate lim .
ps
2 − 3x + 1
( ) 
x→1


 (2 x − 3) x − 1
el
= lim 
lim 4 − 15x + 1 x→1 
(2 x + 3)( x − 1) 
eh
Solution:
2 − 3x + 1
x→1
 



( )
je

(4 − 15x + 1)(2 + 3x + 1)(4 + 15x + 1)  (2 x − 3) x − 1 


= lim  
iit

= lim
(2 − 3x + 1)(4 + 15x + 1)(2 + 3x + 1)
( )( )
x→1

x→1 
(2 x + 3) x − 1 x +1 

 
@

15 − 5x )
× 2 + 3x + 1 =
( 5
= lim
x→1
(3 − 3x )  

4 + 15x + 1 6
 2x − 3 
= lim
x→1 
(
(2 x + 3) x + 1  )

Example 2: Evaluate the following limits:
 


lim 1+ x − 1− x
(i) x→0 −1 −1
= = .


x

(5)(2) 10

(ii) lim
(2 x − 3) ( x −1 ) 1− x 
Example 3: Evaluate lim sin −1  
2x + x − 3  1− x 
x→1




2
x →1
0
Solution: (i) The given limit takes the form when

−1 1 − x



x → 0. Rationalizing the numerator, we get 0
Solution: lim sin  
1+ x − 1− x  1− x 


x →1
lim
x→0
x  1− x 


  = sin–1  lim 
x →1 1 − x 
lim  1 + x − 1 − x × 1 + x + 1 − x 

= x→0
 1 + x + 1 − x 

x
 1− x 
  = sin–1  lim 
 x →1 (1 − x )(1 + x ) 

lim  (1 + x ) − (1 − x )
= x→0 
 x 1+ x + 1− x ( ) 

 1  1 π
  = sin–1  lim = sin–1 = .
 x →1 1 + x 

2 6
  x−2
lim  2x  Example 4: Evaluate xlim
( )
= x→0
x 1+ x + 1− x 
→2 +

x2 − 4 + x − 2



 

FREE BOOKS FOR JEE & NEET =>(@iitjeeadv)


1.28 Differential Calculus for JEE Main and Advanced

x−2 1 1
Solution: xlim = = – .
4+4+4


→2 +

x2 − 4 − x − 2 12


3
x2 − 23 x +1
x−2 . ( x − 4 + x − 2)
2 Example 6: Calculate lim
x →1 ( x − 1) 2



= lim
x → 2+

x − 4 − x − 2 ( x − 4 + x − 2)
2 2
3
Solution: We substitute x = t



( x − 2)( x − 4 + x − 2 )
2 Then, for the variable t, the expression under the limit sign
lim can be written in the form
=
x → 2+ ( x 2 − 4) − ( x − 2)
t 2 − 2t + 1

= lim ( x − 2)( x − 4 + x − 2 ) ( t 3 − 1) 2
2

x2 − x − 2


x→2 +

The number to which the new variable t tends, as


x → 1, can be found as the limit of the function
= lim ( x − 2)( x − 4 + x − 2 )
2

t(x) = 3 x as x → 1,

x→2 +
( x − 2)( x + 1)


i.e. lim t ( x ) = lim 3 x = 1
x 2 − 4 + x − 2 = 0. x →1 x →1


= lim
Thus we have

x → 2+ x +1
t 2 − 2t + 1 ( t − 1) 2
Note: lim = lim
t →1 ( t 3 − 1) 2 t →1 ( t − 1) 2 ( t 2 + t + 1) 2

n −1 n −2 1 n −3 2 n −1
1 1
ps

(x n − a n )( x n + x n an + x n an + ..... + a n ) = ( x − a) = lim
1
=
1
el
t →1 ( t 2
+ t + 1) 2
9
1 1 n −1 n −2 1 n −3 2 .
eh
(x n + a )( x n
n − x n an + x n an − ...) = ( x + a ) if n is odd x − 2a + x − 2 a
Example 7: Evaluate x lim
je

→ 2a +

x 2 − 4a 2


3 (7 − x ) − 2
iit

Example 5: Evaluate lim x − 2a + x − 2a 


x →−1 ( x + 1) 0


lim  form 
@

Solution: x → 2a +
x − 4a
2 2 0

Solution:
3 (7 − x ) – 2
x − 2a x − 2a


(7 − x ) − 8 = lim + lim
x → 2a + x → 2a +
x − 4a
2 2
x 2 − 4a 2

=
(7 − x ) + (7 − x )1/3 .2 + 4
2/3

( x − 2a )( x + 2a )
= lim
( x + 1) x → 2a +

x 2 − 4a 2 ( x + 2a )

= ...(1)
( x + 1) 2/3 + ( x + 1)1/3 .2 + 4


( x − 2a )
+ lim
3 (7 − x ) − 2 ( x − 2a )( x + 2a )

x → 2a +

lim

x →−1 ( x + 1) x − 2a 1
= x lim +


( x + 1) → 2a +
( x − 2a )( x + 2a ).2 2a 4a

= – lim
+ 1)(7 − x ) + (7 − x ) .2 + 4 x − 2a

x →−1 ( x 2/3 1/ 3
1
= lim +
x → 2a +
x + 2a 2 2a

[from (1)] 2 a

1 1 1
= lim
=0+ = .

x →−1 (7 − x ) 2 / 3 2 a 2 a
+ (7 − x )1/ 3 .2 + 4

Concept Problems F

1. Calculate the value of the function at several points near x = 0 and hence estimate the limit


x of f(x) as x → 0.
f(x) =
x +1 −1

FREE BOOKS FOR JEE & NEET =>(@iitjeeadv)
Limits 1.29

2. Evaluate the following limits:


4 + x + x2 − 2

(i) lim x 3. Evaluate lim
x +1


5− x − 5+ x x →−1


x→0

3− x 4. Evaluate lim x +1
(ii) lim


x→ −1
6 x + 3 + 3x


4 − 2x − 2
x→9 2

3
x2 − 3 x 5. Find numbers a and b so that


(iii) lim
ax + b − 1


x→1 ( x − 1) lim = 1.
x →0 x
(iv) lim 5−x − 2




x→1
2−x −1

Practice Problems E

 x −3  x 2 − 2x + 6 − x 2 + 2x − 6
6. Evaluate lim log a  8. Evaluate lim .
x + 6 − 3

x →3  x →3 x 2 − 4x + 3

7. Evaluate the following limits :
x + 7 − 3 2x − 3

9. Evaluate lim
1 + x + x − 7 + 2x − x


2 2 x→ 2 3
x + 6 − 2 3 3x − 5
(i) lim
x 2 − 2x
x→2
ps


10. Evaluate the following limits :

1+ 2 + x − 3 x 2 + 8 − 10 − x 2
el
(ii) lim
(i) lim


x→ 2
x−2
eh

x →1


x2 + 3 − 5 − x2
(iii) lim x +1
8 + 3x − 2
je

3
x→ −1 4


x + 17 − 2 (ii) lim
iit

x → 0 4 16 + 5x − 2


1+ x − 1− x
(iv) lim
@

x →0 3 1 + x − 1− x 5
1 + x )3 − 1


(iii) lim .
x→0


(1 + x ) 3 (1 + x ) 2 − 1

1.8 Limit Using Expansion Series of x 3 2x 5


(vi) tan x = x + + + ........



Functions 3 15
x3 x5 x7
In this method basically we use the series expansion of sin (vii) tan–1x = x − + − + .......


x, cos x, tan x, log(1+x), ax, ex, etc. to evaluate the limit. 3 5 7
Following are some of the frequently used series expansions: 12 3 12.32 5 12.32.52 7
(viii) sin-1x = x + x + x + x + .......


3! 5! 7!
x 1n a x 2 1n 2a x 3 1n 3a
(i) ax = 1+ + + + .........a > 0 x 2 5x 4 61x 6


1! 2! 3! (ix) sec x = 1 + + + + ......


2! 4! 6!
x x 2 x3
(ii) ex = 1 + + + + ............ nx x2
(x) (1 + x)n = 1 + + n (n − 1) +.... for –1< x< 1, n ∈ Q.


1! 2! 3!


1 2
x 2 x3 x 4  1 11 2 
(iii) ln (1+x) = x − + − + .........for − 1 < x ≤ 1 (xi) (1 + x)1/x = e 1 − x+ x + .....


2 3 4  


2 24
x3 x5 x7 Note: Using the above expansions, we can find other
sin x = x − + − + .......

(iv)

expansion series. For example to find the expansion series of


3! 5! 7!
1 − cos 2 x
x2 x4 x6 sin2x, we write, sin2x = and use the expansion series
(v) cos x = 1 − + − + ...... 2


2! 4! 6! of cos x with x replaced by 2x.

FREE BOOKS FOR JEE & NEET =>(@iitjeeadv)


1.30 Differential Calculus for JEE Main and Advanced

Maclaurin's Theorem  x3   x3 
 x + 3 ........ −  x − 3 ! .......
x2 x3    
f(x) = f(0) + xf'(0) + f "(0) + f '" (0) + ... = xlim
→0
x3


2! 3!
xn n 1 1 3
........ + f (0) + ........  +  x + ........
n! lim  3 3! =
1
+
1 1
= .
x→0 x3


3 6 2
Example 1: Expand sin x in powers of x.
1 − x


e x + ln 
 e 
Solution: Here f(x) = sin x, Hence f(0) = 0,


f'(x) = cos x, f'(0) = 1, Example 5: Evaluate lim
tan x − x




f"(x) = – sin x, f"(0)= 0, x→0


f'"(x) = – cos x, f"'(0) = – 1 1 − x
e x + ln 



 e 
.............

 nπ  Solution: lim
fn(x) = sin  x +  fn(0) = sin nπ x→0 tan x − x



 2


2
e x + ln(1 − x ) − ln e
nπ n = lim [form 0/0]
x sin tan x − x


x→0
x3 x5 2 + ...
Thus sin x = x – + − .... +
x 2 x3  x 2 x3 

3! 5! n!
1+ x + + + ... +  − x − − − ... −1
Example 2: Expand ln(cos x) in powers of x. 2! 3!  2 3 


= lim
Solution: Here f(x) = ln(cos x),
ps x→0  3 5
x 2x
 x + 3 + 15  − x


f'(x) = – tan x = – t, say,
 
el

f"(x) = – sec2 x = – (1 + t2),
eh

1

f'"(x) = – 2 tan x sec2 x = – 2t (1 + t2), − x 3 (1+ terms containing x and its higher powers)

f(4)(x) = – 2 (1 + 3t2) (1 + t2) = – 2 (1 + 4t2 + 3t4), 6
= xlim
je

→0 1 3

f(5)(x) = – 2 (8t + 12t3) (1 + t2) x (1+ terms containing x and its higher powers)

iit

= – 2(8t + 20t3 + 12t5), 3



f(6)(x) = – 2 (8 + 60t2 + 60t4) (1 + t2) 1
@

=– .

= – 2(8 + 68t2 + 120t4 + 60t6) 2

Hence, 5 sin x − 7 sin 2x + 3 sin 3x
Example 6: Evaluate lim
f(0) = 0, and x 2 sin x


x →0

f'(0) = f(3)(0) = f(5)(0) = ...= 0, also
5 sin x − 7 sin 2x + 3 sin 3x

f"(0) = – 1, f(4)(0) = –2, f(6)(0) = – 16. Solution: lim

x →0 x 2 sin x


x2 x4 x6
Hence ln(cos x) = – −2 − 16 .............  x3   ( 2 x )3   (3x )3 
2! 4! 6! 5 x − + .... − 7  2 x − + .... + 3  3x − + ....
 3!   3!   3!! 
= lim
ex − 1 − x x →0 
x2  x −
x 3
+ ....

Example 3: Evaluate lim
x2  3! 


x→0

e −1− x x 5x 3 56 x 3 81x 3
Solution: lim − + −
x→0 3! 3! 3!


x2 = lim
 
x →0  x 3 
x2 x 3 1 − + ...
 1 + x + 2 ! ....... − 1 − x  3! 
 
= lim
−5 + 56 − 81

x→0 x2
= = −5 .
1 x x 1 2 3!
= xlim + +
→ 0 2! 3!
............ = . x2
4! 2 n (1 + x ) − sin x +


Example 7: Evaluate lim 2
Example 4: Evaluate tan x − sin x


x→0 x tan x sin x


x3 x2
tan x − sin x n (1 + x ) − sin x +
Solution: lim Solution: lim 2


x→0 x3 x→0


x tan x sin x

FREE BOOKS FOR JEE & NEET =>(@iitjeeadv)


Limits 1.31

 x 2 x3   x3 x5  x2 x 7 − 2x 5 + 1
 x − 2 + 3 ..... −  x + 3! + 5! + ..... + 2 Example 11: Find lim
x→1 x 3 − 3x 2 + 2
.
   



= xlim
→0
3 tan x siin x 0
x . . Solution: This is of the form if we put x = 1.



x x 0
1 1 1 Therefore we put x = 1 + h and expand.
= + = .
3 6 2 x 7 − 2x 5 + 1 (1 + h )7 − 2(1 + h )5 + 1
lim = lim
cos x 3 − 1 + n (1 + x 6 ) x→1 x 3 − 3x 2 + 2 h→0 (1 + h )3 − 3(1 + h ) 2 + 2


Example 8: Evaluate lim
x 2 (e x − 1 − x 2 )
2
x →0


(1 + 7 h + 21h 2 + ...) − 2(1 + 5h + 19h 2 + ...) + 1
= lim
cos x 3 − 1 + n (1 + x 6 ) h→0 (1 + 3h + 3h 2 + ...) − 3(1 + 2h + h 2 ) + 2
Solution: lim
x 2 (e x − 1 − x 2 ) −3h + h 2 + ...
2
x →0


= lim
x6 x12 −3h + ....


h→0
1− + ........ − 1 + x 6 −
2 2 −3 + h + ...
= lim = lim = 1.
  h→0 −3 + ....


x →0 x2 x4
x 2 1 + + ....... − 1 − x 2 
 1 2  a cos x + bx sin x − 5
Example 12: If lim exists, find a,
x →0 x4


1 b and the limit.
+ .....  0
2 Solution: As x → 0, x4 → 0 the limit must be in  
= lim =1.  0


x→0  1  form. Hence lim acos x + bx sin x – 5 = 0.

ps
 2 + ....... x→0
⇒ a – 5 = 0 ⇒ a = 5.
el
esin x − sin x − 1  x2 x4   x3 
5 1 − + ....... + bx  x − ...... − 5
eh

Example 9: Find lim


x2  2 2   3 


x →0
Limit = lim
je

x →0 x4
e sin x
− sin x − 1
 5 2  5 b 4
iit

Solution: lim
x →0 x2  b −  x +  −  x + ....


2 24 6 
@

 sin x sin 2 x sin 3 x  = lim


1 + 1 + + + ..... − sin x − 1 x →0 x4
 2 3  4
For limit to exist, x must cancell from the numerator. Hence
lim
= x→
x2 we assume the coefficients of all powers of x

0

 sin x   1 sin x 
2
1 1 5 5
= lim   + + ... = = . less than 4 to be zero ⇒ b − = 0 ⇒ b = .
x→0  x   2   2 2

3 2 2
 5 b 4
(7 + x )1/3 − 2  −  x + ........
Example 10: Evaluate lim 24 6  5 b
x −1 Now the limit = lim −


x →1 4
=
x →0 x 24 6
Solution: Put x → 1 + h
5 5 −5


(8 + h )1/3 − 2 ∴ Limit = − = .
lim 24 12 24
h→0 h
Ae x − B cos x + Ce − x

 h
1/ 3 Example 13: If lim = 2 , find A,
x →0


2 . 1 +  −2 B and C. x sin x
 8
= lim Solution: The given limit is equal to

h→0 h


 x2   x2   x2 
 1 1   h  2 A 1 + x + ....... − B  1 − + ....... + C  1 − x + .......
 − 1     2   2   2 

 1 h 3 3   8  lim
x →0  3 
2 1 + . + + ....... − 1 xx −
x
.......
 3 8 1 . 2   3 

 

= lim  A + B + C 2
( A − B + C) + ( A − C) x +   x


h→0 h 2
= lim
1 1 x →0  x2 
= lim 2 × = . x 2 1 − ......
h→0

24 12  3 

FREE BOOKS FOR JEE & NEET =>(@iitjeeadv)


1.32 Differential Calculus for JEE Main and Advanced

For limit to exist, x2 must cancel from the numerator. Hence


 1 11 
we assume A – B + C = 0 ...(1) = e 1 − x + x 2 + .....
 


2 24


and A – C = 0 ...(2)



A+ B+C 1
∴ Limit = = 2. ...(3) (1 + x )1/ x − e + ex
2


2 Now lim
On solving these three equations, we get A = 1, B = 2, C = 1. x →0 x2
1 11 2 1
Example 14: Show that x+ e(1 −
x + ....) − e + ex


2 24 2 11e
1
(1 + x )1/ x − e + ex 11e = lim = .
x →0 x2


2 24
lim = .
x →0 x2 f (x)

24
Example 15: Let f(x) be a function such that lim = 1.
x →0 x



Solution: Let y = (1 + x)1/x Find the values of a and b such that


1 x (1 + a cos x ) − b sin x
∴ ln y = ln(1 + x ) lim =1.

x x →0 {f ( x )}3
1 1 2 1 3 
=  x − x + x − ... x (1 + a cos x ) − b sin x
Solution: Since, lim =1

x 2 3 x →0 {f ( x )}3


1 1
= 1− x + x 2 − ..... ,   x2 x4    x3 x5 
2 3 x 1 + a 1 + +  − b x − + − ....

1 1   2! 4!    3! 5! 
1− x + x 2 −...... ⇒ lim =1
Now y = e 2 2
ps x →0 {f ( x )}3
1 1  a b  a b
el
1− x + x 2 −......
x (1 + a − b) + x 3  +  + x 5  −  + ...
= e. e 2 2  2! 3!  4! 5!
eh

⇒ lim =1
  1 1 2  x →0 {f ( x )}3
= e 1 +  − x + x − ...... + .....
je

 

 2 3 (1 + a − b)  a b   a b
+  +  + x 2  −  + ...
iit


2  2 ! 3!  4! 5!
 x
2
1 1 1 2  ⇒ lim =1
+  − x + x − ...... + .... x →0  f (x) 
3
@



2 2 3   
 x 
 1  1 1 2 
= e 1 − x +  +  x + ..... Since the limit exists we must have 1 + a –b = 0 and –
a b
+
   

2 3 8
=1 ⇒ –3a + b = 6 2 ! 3!


Solving these, we get a = –5/2 and b = –3/2.

Concept Problems G

1. Evaluate the following limits :
e x sin x − x (1 + x )

1 (iii) lim
3
1+ x −1− x x3


x→0
(i) lim 3
x →0 x2


3
1 + 3x − 1 + 2 x
e sin x − x − x x 2 (iv) lim
x2


(ii) lim 2 x→0
x →0 x + x ln(1 − x )


3 tan x − 3x − x 3
x2 3. Evaluate lim
cos x − 1 + x →0 x5


(iii) lim 2
x →0 x3


sin −1 x − tan −1 x
2 x − ln(1 + 2 x ) 4. Evaluate lim
x3


(iv) lim x→ 0
x →0 x2


2. Evaluate the following limits : (2 − x )(e x − x − 2)
5. Evaluate lim

x →0 x3

2 x - sin -1 x
(i) lim


2 x + tan -1 x
x →0 6. For what values of constants C and D is it true that

3 sin x − sin 3x lim( x −3 sin 7 x + Cx −2 + D) = −2
(ii) lim x →0
x − sin x


x→0

FREE BOOKS FOR JEE & NEET =>(@iitjeeadv)


Limits 1.33

Practice Problems F


7. Evaluate the following limits :
sin x − x + x 3 / 6 − x 5 / 120

(1 + x ) − (1 − x )
2 (ii) lim
(sin x )7



x →0
(i) lim


x→0 (1 + x 3 ) − (1 + x )
e x cos x − 1 − x
(iii) lim
sin( x 2 )



3
1+ x2 − 4 1 − x2 x→0
(ii) lim
x + x2


x→0 x2
x cos3 x − ln(1 + x ) − sin −1
31+ 3x − 1 2
(iii) lim (iv) lim
x3



x →0
x→0 (1 + x ) 50
− 1 − 50 x


1 1+ x
ln 2 (1 + x ) − x 2 2 sin x + ln − 3x
(iv) lim 10. Evaluate lim 2 1− x
x →0 x3


x →0 x5


8. Evaluate the following limits : x4 3

m
1 + ax − 1 + bx
n x 3
e4 − sin 2 x 2
(i) lim (m, n ∈ N) 11. Evaluate lim

x7 x →0


x →0 x
12. Find the values of a and b so that
m
1 + ax n 1 + bx − 1

(ii) lim (m, n ∈ N) (1 + ax sin x ) − b cos x
ps
lim
x →0


x
x→0 x4
el
3
1 + 2x − 4 1 + 9x

(iii) lim may tend to a definite limit and also find limit.
eh

x →0


x 13. For what values of the constants a, b is ,
1− 1−

2  sin 3x a 
je

lim  + 2 + b = 0 ?
x→0  x 3 

x
iit

4 4 1 − x 2 − 4e x
2

(iv) lim
tan −1 x − x 1  1 1 + ax 


x →0
@

3

+ bx 
14. If lim exists and has the value
9. Evaluate the following limits :  1+ x

x→0 x 1

x3 x5 1 2 3
sin x (1 − cos x ) − + equal to , then find the value of − + .
(i) lim 2 8 a  b
x →0 x7


1.9 Standard Limits e x −1

In particular lim =1
Following are some basic limits which are used frequently

x→0 x
in solving the limits. ln(1+ x )
sin x tan x (iii) lim =1

(i) lim = 1 = lim x→0 x

x→0 x x→0 x
log a (1+ x )
−1 −1 lim = loga e, ( a > 0, a ≠ 1)
lim tan x = lim sin x x

x→0
= x→0 x→0 x

x
(1 + x ) n − 1
[ where x is measured in radians ] (iv) lim =n

x →0

1 − cos x (1 − cos x )(1 + cos x ) x
Also, lim = lim
2
x 2 (1 + cos x ) xn − an

x = n a n −1 .
x→0 x→0
lim
2 x →a x−a
(sin 2 x )  sin x  1

= lim 2 = lim  
x→0 x (1 + cos x ) x→0  x  (1 + cos x )

Study Tip
1
= . (i) These limits help in finding limits in 0/0 form and most of
2


them are based on x approaching 0. In case, if x approaches
a x −1 a, we use the substitution x = a + t so that t approaches 0
(ii) lim = ln a (a > 0).

x→0 x when x approaches a.

FREE BOOKS FOR JEE & NEET =>(@iitjeeadv)


1.34 Differential Calculus for JEE Main and Advanced

(ii) These formulae are also applicable when a function, say πx



tan
180 . πx
f(x) has been replaced for x. But it must be ensured that
as x approaches a, f(x) must approach 0.
πx 180 .
sin f ( x )
180 π
For instance, lim
x →a f (x)
= 1, if f(x) approaches 0 as = lim =

x→0 x 180
x approaches a.


e3 x − 1
Example 2: Evaluate lim
Caution



x→0 x/2

sin f ( x ) e3 x − 1 e3 x − 1
Solution: lim = lim 2 × 3 = – 6.
But lim where f(x) and g(x) approach 0 as x



x →a g( x ) x→0 x/2 x→0 3x
approaches a, should not be immediately taken as 1. sin 2 x
Example 3: Compute lim



sin 2x sin 2 x sin 3x
x→0
For example, lim = lim .2 =2 sin 2x
x 2x lim

x→0 x→0 Solution: We have x→0



sin f ( x ) sin 3x
Further, lim should not be taken as 1 when sin
x →a f (x)  sin 2 x 2 x 3x 
= lim  . .
f(x) approaches 0 as x approaches a, but f(x) itself does not

x→0  2 x 3x sin 3x 
approach 0.  sin 2 x  2  3x 
sin(π − x ) =  lim  . .  lim ,x≠0
 2 x→0 2 x  3 3x→0 sin 2 x 

≠ 1, since π – x does not approach
For example, lim
ps
x→0 π−x −1
2  sin 3x  2
el
0 as x approaches 0. In fact, the limit is equal to 0. =1. . lim = ×1= .
3 3x→0 3x  3
eh

Why Radian Measure Is Used


x3 − 8
je

Throughout calculus, angles are measured in radians, as is Example 4: Evaluate lim


x→2 x 2 − 4


customary in calculus. If we measured angles in degrees instead,
iit

the formulas for the limits of the trigonometric functions would Solution: The given expression is of the form


@

be more complicated. Each formula would have an extra factor, x 3 − (2)3 x 3 − (2)3 x 2 − (2) 2
π/180, as we will now show. x − (2)
2 2 =
x−2
/
x−2

Earlier, it was shown that when angles are measured in radians,
x3 − 8 x 3 − ( 2) 3 x 2 − (2) 2
sin θ ⇒ lim = lim / lim
= 1. x2 − 4 x−2 x−2

lim x→2 x→2 x→2
θ→0 θ
When angles are measured in degrees, this limit is not 1. xn − an

Let sin θ denote the sine of an angle of θ degrees. The following = 3(22)/(2(21)) (using lim = nan–1 )
x →a x−a

table suggests that the limit is much smaller (angles measured = 12 / 4 = 3.

in degrees; data to four significant figures): 1 + x1/ 3
Example 5: Find xlim .
θ
→−1 1 + x1/ 5


10 5 1 0.1
(x )
5
sin θ 0.1736 0.08716 0.01745 0.001745 1/ 5
− ( −1)
Solution: We have lim = 5( −1) 4 = 5
1/ 5
− ( −1)


x →−1 x
sinθ
0.01736 0.01743 0.01745 0.01745
θ ( x1/3 )3 − ( −1)
and lim = 3 (–1)2 = 3
sin θ x →1 x1/ 3 − ( −1)


The data suggest that lim is about 0.01745. We can
θ→0 θ

x1/ 3 − ( −1) 5
find that the limit is precisely π/180. Hence the limit = lim = .
x →−1 x1/ 5 − ( −1)

3
tan x °
Example 1: lim Example 6: Evaluate
x→0


x


π a x + h + a x − h − 2a x
Solution: x ° = x radian lim , a>0
h→ 0 h2




180
πx  a h + a −h − 2 
tan x °
tan Solution: Limit = lim ax  
= lim 180  h2 


Now lim h→0
x→0 x→0

x x

FREE BOOKS FOR JEE & NEET =>(@iitjeeadv)


Limits 1.35

tan x − sin x
 a 2 h − 2a h + 1   2 
2 Example 12: Evaluate lim
x a −1 x3



x x→0
= lim a   = a  h 
h →0 ahh2  

  lim tan x − sin x
x 2
= a ln a. Solution: x→0
x3




a x − aa
Example 7: Evaluate lim , a > 0. tan x (1 − cos x )
x→ a x − a = lim


x3


x→0
Solution: Put x = a + h
2 x


a a+h − a a a a (a h − 1) lim tan x . 2 sin 2
Limit = lim = lim = aa lna = x→0


h →0 h h→0 h x3
2
 sin x   x
Example 8: Evaluate lim  . sin
x →0  x   tan x 1 2 = 1 .


= lim .  
x


x→0 x 2 2
sin x  sin x   
Solution: Since, 0 < <1,   = 0 in the 2 
 x 


neighbourhood of x = 0. x
sin(π cos 2 x )
 sin x  Example 13: Evaluate lim
x2



Hence lim  = 0. x →0
x →0  x  

 tan −1 x 
Solution: Here πcos2x → π as x → 0.


With a similar reason, lim   = 0. We change πcos2x to πsin2x, so that it tends to 0.

x →0
 x  sin(π cos 2 x ) sin{π(1 − sin 2 x )}
lim = lim
 tan x 
ps x →0 x2 x →0 x2

Example 9: Evaluate lim  .
x →0  x  


el
Solution: Since, 1 <
tan x
< 2 in the neighbourhood of sin(π − π sin 2 x ) sin(π sin 2 x )
= lim = lim
eh


x x →0 x2 x →0 x2

 tan x 
x = 0, 
tan x 
 sin(π sin x ) π sin x 
2 2
je

 = 1. Hence, xlim  x  = 1. = lim  × × 


 x  → 0   x →0  π sin 2 x x 2 
iit

 1

 sin −1x 
With a similar reason, lim   =1.
@

x →0 sin(π sin 2 x ) sin 2 x


× π ×

 x  = lim lim
x →0 π sin 2 x x →0 x 2

 −2x  = 1 × π × 1 = π.
Example 10: Evaluate lim 
x →0  tan x 
, where [.] denotes




greatest integer function. x sin{x}
Example 14: Evaluate lim , where {x}
x −1


Solution: We know when x → 0 x→1


x −x denotes the fractional part of x.
⇒ <1 ⇒ > −1
Solution: As x → 1– , {x} can be replaced by 1– h,

tan x tan x


−2 x where h is a small positive quantity.
⇒ –2 < < −1
As x → 1+ , {x} can be replaced by h, where h is a small

tan x
positive quantity.
 −2x 
So, lim   = –2. x sin{x} 1− h
x →0  tan x 

∴ lim = lim sin(1 − h ) = – ∞
ln x − 1 x →1− x −1 h → 0 −h +

Example 11: Evaluate lim .


x →e x − e


x sin{x} sin h
lim = lim (1 + h )
ln x / e x →1+ x −1 h →0 +
h
Solution: Limit = lim

x →e  x 


= 1×1=1
e  − 1

e  Since, L.H.L. ≠ R. H.L. the limit does not exist.
ln t cos x ln( x − a )
= lim (putting x/e = t) Example 15: Evaluate lim
t →1 ( t − 1) e x →a ln(e x − ea )



ln (1 + y ) 1 cos x ln( x − a )
Put t = 1 + y = lim = Solution: lim Put x = a + h

y→ 0 ey e x →a ln(e x − ea )


FREE BOOKS FOR JEE & NEET =>(@iitjeeadv)
1.36 Differential Calculus for JEE Main and Advanced

ln h 2
= cos a · lim   sin t  
h →0 a+h
ln(e − ea )  sin  

2
 2.   2   . sin t   1
= lim .
t 0  sin t  4.t 2 2


ln h
= cos a · lim  2 
 a  e h − 1   

h →0
ln  e ·   · h
  h   Example 19: Find the limiting value of



ln h
tan 2 x − 2 sin x
= cos a · lim as x tends to zero.
x3


h →0  e h − 1

a + ln   + ln h sin 2 x − 2 sin x cos 2 x
 h  Solution: lim
x 3 cos 2 x



x→0
1
= cos a · lim = cos a. 2 sin x[cos x − cos 2 x ]
ln(e h h )

h →0 a
+ +1 = lim
x 3 .1


ln h ln h x→0

3x
1 − cos 2 x 2 sin sin x
Example 16: Evaluate lim = 2 lim
2 2
x2



x x→0

x→0

1 − cos 2 x 3 sin
3x
sin x
Solution: f(x) = = 2 lim 2 . 2 = 3.
x


x→0 x

ps 3x
2 2
2
2 sin 2 x 2 | sin x |
= Example 20: Let a = min [x2 + 2x + 3, x ∈ R] and
el
= ,
x x



n
eh

 a r bn r .
sin x cos x
 sin x , if 0 < x < π / 2, b = lim , then find the value of
but |sin x| =  − sin x , if − π / 2 < x < 0 x 0 e x  e x r 0
je



iit

 sin x  Solution: a = minimum value of (x + 1)2 + 2




Hence f(0 ) = lim f ( x ) = lim  − 2

=− 2 ⇒
@

x →0 x →0  x 
− − a=2

sin 2 x 1
 sin x  b = lim = .
x 0 2(e 2 x  1).2 x

and f(0+) = lim f ( x ) = lim  2 = 2. 2
x →0 x →0 
+
x  +

2x
n cos x n n r n
Example 17: Evaluate lim 1 1
 a r bn r =  22 r
.
x →0 4
 2r  2 


1+ x −12 Now =
n
r 0 2 r 0
n (1 + cos x − 1)(cos x − 1)
Solution: Limit = xlim
→0
{(1 + x )
2 1/ 4
}x
−1 1 n
 4r =
1


(cos x − 1) 2 = n
[1 + 4 + 42 + ...... + 4n]
n

x 2 2 r 0 2
1  1 1  4n1  1  4n 1  1
= 1     2 . = n  = .
 2 2  3  3.2n


4
e x  ln( x  e)
n cos(sin t ) Example 21: Evaluate lim .
ex  1


Example 18: Evaluate lim 2
x 0


t →0 t
  x 
n cos(sin t ) (e x  ln  e  1   
Solution: lim Solution: lim   e 
t →0 t2


x 0  ex  1 


x  
 (sin t )   2  sin t    x 
n 1  2 sin 2   2 sin  
 2 
 lim 
2  
.  (e x  1) ln 1  ( x e)  
t 0  sin t  t2 = lim   
2 sin 2   x 0  x x 

 2 

FREE BOOKS FOR JEE & NEET =>(@iitjeeadv)
Limits 1.37

= 1 – lim
ln 1  ( x e) 
   cos 1 x ·    cos 1 x 
 x
x =

x0 lim


  cos 1


e
x1
1 x2

=1–
1
. 1    cos x 
1

e = lim
2 


x1
1 x2
 n (cos 3x ) 2 sin x 
Example 22: Evaluate lim  . x x  . 1 cos 1 ( x )
x 0  x2 e e 


= lim 
2  1 x2


x1
Solution:

n (cos 3x ).2 sin x n (cos 3x ).2 sin x.e x put cos–1(–x) = θ ⇒ x = – cos θ



lim lim
x 0 x 2 (e x  e  x ) x 0 x 2 (e 2 x  1) 1  1 
= lim = lim

2  0 
| sin  | 2  0 sin  


 n (1  cos 3x  1)
= lim   (cos 3x  1)
 (cos 3x  1)

x→0 1
= .
2 π

2 sin x 1 
  
x  e2 x  1  2  tan x  sin{tan 1 (tan x )}
  2 x  Example 25: Solve lim
 2x    tan x  cos 2 (tan x )


x
2

  2 3x 
 2 sin 2 

ps Solution: Here, RHL
n  
 


(1 cos 3 x 1)
= lim 
tan x  sin{tan 1 (tan x )}
el
(cos 3x  1)

x→0  2x 2
= lim
tan x  cos 2 (tan x )
eh




2 sin x 1  x
   2
 e 1  
je

x 2 x

 x   tan x  sin( x  )
iit

  = lim
tan x  cos 2 (tan x )



x

@

9 9 2
= 1 × (–1) × ×2×1=– . π

4 2 {∵ tan–1 (tan x) = x – π, when x > }

2
1− x
Example 23: Evaluate lim . sin x
x →1− (cos −1x) 2 1


tan x 1+ 0
= lim = = 1.
Solution: Put cos x = y and x → 1 ⇒ y → 0
-1 – + 
2 1+ 0

x cos (tan x )
1


2
1− x 1 − cos y tan x
lim −1 = lim
x →1−
y→0 y2 2 Now, LHL

(cos x)

Now rationalizing the numerator, we get tan x  sin{tan 1 (tan x )}
(1 − cos y) = lim
tan x  cos 2 (tan x )



= lim 2 x
y → 0 y (1 + cos y ) 2

1  cos y 1 1 1 1 tan x  sin( x )
= ylim . lim = . = . = lim
2 y 0 1   tan x  cos 2 (tan x )

0


y cos y 2 2 4

x
2

Example 24: Evaluate lim


sin 1    cos 1 x .  1 
as tan (tan x )  x , when x  
x 1
 2




1 x 2
     
Solution: sin x
  


1
sin 1   cos 1 x   cos 1 x tan x 1 0
= lim = =1
1 0
 x
lim  cos 2 (tan x )



x
x1
  cos 1 1 x2 2 1
tan x

FREE BOOKS FOR JEE & NEET =>(@iitjeeadv)
1.38 Differential Calculus for JEE Main and Advanced

tan x  sin{tan 1 (tan x )} 1


.
1
=
1
∴ lim = 1. =
2
.


 tan x  cos (tan x )2 2 2

x
2
Hence, limit does not exist.
xh
(x  h) x x
Example 26: Evaluate lim Use of Substitution
h→0 h


Sometimes in solving limit problem we convert lim f(x) by
(x > 0). x →a

x  h  n (x  h) x n x substituting x = a + h or x = a – h as lim f(a + h) or lim
e e h→0 h→0
Solution: lim f(a – h) according to the need of the problem.
h→0



h
e e x − e3
 1
x  h  n (x  h)  x n x
x n x     Example 28:  Compute lim
= lim e
 h 
x→3 x −3

h→0
Solution: Put y = x – 3. So, as x → 3, y → 0.



 es  1
= x
x
 lim e x − e3 e3 y  e3
s  0 s Thus lim = lim

x→3 x −3 y→0 y
x  h  n (x  h)  x n x
lim  e3 . e y − e3
h→0 h 
= lim
y

y→0
where s = x  h  n (x  h)  x  n x
e y −1
  h  = e3 lim = e3 . 1 = e3 .
x  h  nx  n 1     x nx
ps


y→0 y
x   x 
= x . hlim
el
→0 h ln tan x

Example 29: Find lim
eh

x   / 4 1  cot x



x
 ( xh  x)
= x  lim  n x
h  0 Solution: Put x = t + π/4
je

h



x/h  ln tan( t   / 4)
iit

(x  h)  h lim
 lim  n 1    t 0 1  cot( t   / 4)
h0 x  x 
@



 1  tan t 

x  n x 1  ln  
= x   .  1  tan t 
 2 x x  = lim

t 0  cot t  1 

1  
1  cos x  cot t  1 
Example 27: Evaluate lim
x 0 ln(1  tan t ) ln(1  tan t )


x
= lim + lim
t 0 2 tan t t 0 2 tan t

1  cos x
Solution: lim 1  tan t 1  tan t


x 0 x
1 2
1  cos x 1 = [1.1 + 1.1] = = 1.
= lim .

2 2
x 0 x 1  cos x

 π
cos  x + 
2 | sin( x / 2) | 1  6
= lim . Example 30: Evaluate lim
x 0


2( x / 2) 1 x → π / 3 (1 − 2 cos x ) 2 / 3


cos x
Now, we have 
1 sin( x / 2) 1 Solution: Putting x – z


LHL = lim . . 3
x 0 
2 x/2 1 cos x  
cos   z 
1 1 1  2 
= .  . = lim
z 0 (1  2 cos z  3 sin z ) 2 / 3


2 2 2
1 sin( x / 2) 1  sin z
and RHL = lim . . = lim
x 0


2 x/2 1  cos x z 0 (1  cos z  3 sin z) 2 / 3

FREE BOOKS FOR JEE & NEET =>(@iitjeeadv)
Limits 1.39

z z xn x


− ex  n a
2 sin   cos   Solution: l = lim e
  2 x−a



2 x→a
= lim 2/3 2/3

z 0
  z   z  z  e x  n a e x (  n x   n a )  1 x ( n x −  n a )
 2 sin  2   sin  2   3 cos  2    
        = lim
x→a x ( n x   n a )
.
x−a


  z 1/ 3  z  h
21/ 3 sin    cos   (a + h) ln 1 + 
   
2 2 −21/ 3 .0.1 a
= a . lim  a  = aa.
= zlim =0. h→0


=
( )
0

2 / 3 h
 z 
2/3
z

 2 sin 2  3 cos  2  3
e xn a
−e an x
     m = lim
x→a x−a


(1  tan )
Example 31: Evaluate lim
(1  2 sin ) ea  n x e x  n a  a  n x  1
  / 4
 . xn a − an x


= lim 
xn a  an x x−a


x→a
(1  tan )  0
Solution: Let P = lim  form 0 
(1  2 sin )   = aa . lim (a  h )  n a  a  n (a  h )


x→0


h→0
π h
Put θ = +h 
  
= aa (ln a − 1)
a/h
4

h
= aa lim
h→0
 n a   n 1  a 
 

 
1  tan   h  ps Now l = m ⇒ a = e2
∴  4 

P = lim
h 0  


1  2 sin   h  Example 33: Let
el  

 4 
sin −1 (1 − {x}).cos −1 (1 − {x})
eh

 1  tan h 
f(x) = , then find lim f(x) and
1  2{x}.(1 − {x}) x0 

je

 1  tan h   0
lim f(x), where {x} denotes the fractional part of x.
= hlim  form 0 
iit

0  1 1    x0 

1 2  cos h  sin h 
 2 2 
@

Solution: We have f(x)




−2 tan h sin −1 (1 − {x}).cos −1 (1 − {x})
lim =
h →0 (1 − cosh) (1 + cos h ) 2{x}.(1 − {x})

(1 − tan h ) − sin h
=
(1 + cos h ) ∴ lim f(x) = lim f(0 + h)


x0  h→0
tan h
= –2 lim
h 0  sin 2 h  sin (1  {0  h}).cos 1 (1  {0  h})
1

(1  tan h )   sin h  = lim
h 0 2{0  h}.(1  {0  h})

 1  cos h 
sin h sin 1 (1  h ).cos 1 (1  h )
= –2 lim = lim
 sin 2 h  2h .(1  h )

h 0

h 0
(1  cos h  sin h )   sin h 
 1  cos h  sin 1 (1  h ) cos 1 (1  h )
= hlim . lim
0 (1  h ) h 0

1 2h
= –2 lim
h 0  sin h  (In second limit put cos–1 (1 – h) = θ

(cos h  sin h )   1

⇒ 1 – h = cos θ)
 1  cos h 

1
sin (1  h ) cos 1 (cos )
1 = lim . lim
(1  h ) 0 2(1  cos )

= –2 . = 2. h 0
(1 − 0)(0 − 1)


sin 1 (1  h ) 
xx − ax = lim. lim (∵ θ > 0)
(1  h ) 0 2 sin( / 2)

Example 32: Let lim = l, a > 0 and h 0
x→a x − a


= sin–1 1 . 1 = π/2
a x − xa

lim = m, a > 0 . If l = m then find the value of 'a'. and lim f(x) = lim f(0 – h)
x−a

x→a x→−0 h→0

FREE BOOKS FOR JEE & NEET =>(@iitjeeadv)


1.40 Differential Calculus for JEE Main and Advanced

We show that the limit P does not exist :


sin 1 (1  {0  h}).cos 1 (1  {0  h})
= lim | sin h| | sin(0  t |
h 0 2{0  h}.(1  {0  h}) L.H.L. = lim  lim

h 0 t 0 (0  t )


 
h
1 1
sin (1  h  1).cos (1  h  1) sin t
= lim  1
= lim
h 0 2( h  1).(1  h  1) t

t 0


−1 sin h | sin(0  t ) |
sin 1 h lim cos h and R.H.L. = lim  lim
= lim . h →0
2(1 − h ) h 0 h t 0 (0  t )  

h 0 h
sin t
π/2 π = lim 1
=1. = . t 0


t

2 2 2
∵ L.H.L. ≠ R.H.L. Hence P does not exist.
1  sin 2 x ∴ The limit L also does not exist.
Example 34: Evaluate lim
(  4x )


x  / 4
Special Limits
1  sin 2 x Let us evaluate some limits with the help of the following
Solution: Let L = lim
(  4x ) special limits :


x  / 4
x  sin x 1 ex  1  x 1
1 1  sin 2 x (i) lim 3
 (ii) lim 2






= lim x0 x 6 x 0 x 2
4 x  / 4   

 4  x e x  e x  2x 1
  (iii) lim 
ps
x 0 3


x 3

h x  tan x
el
Put x = 1
(iv) lim 

4 3

3
eh

x0 x
  ex  1  x3
3

1  sin 2   h 
je

Example 35: Evaluate lim


4  x 0 sin 6 2x


1
∴ L = lim
iit

4 h 0    ex  1  x3
3

  h
4  4
@

Solution: Let l = lim


 x 0 sin 6 2x


(1  (cos 2h ) ) ex  1  x3
3

1
=  lim = lim
x 0 ( 2 x )6

4 h 0

h
Now put x3 = t
1 (1  (cos 2h ) ) (1  (cos 2h ) )
=  lim . 1 et  1  t

4 h (1  (cos 2h ) ) l= lim again put t = 2y

h 0
64 t 0 t 2


(form
0
) 1 e2 y  1  2 y
0 = lim

64 y 0 4y2

1 1  cos 2h
=  lim 1 (e y  1) 2  2e y  2  2 y
4 h 0 h (1  (cos 2h ) ) = lim

64 y 0 4y2

1 2 | sinh |   y 2 
=  . lim 1 1 e 1  1 ey  y  1
= lim    
4 h 0 h (1  (cos 2h ) ) 64 y 0  4  y 

2 y2 

 
1 1 | sin h| 1 1 1 
=  . 2 lim lim ∴  .64 
64  4 2
4   h l=

h 0 h 0


(1+ (cos2h) )
1 1 1 l 1 1
=  . 2. P P ⇒ = l= .

4 4 2 4.64 128

2
| sin h| e x  e x  2x
where P = lim . Example 36: Evaluate lim
x 0 x  sin x



h →0 h

FREE BOOKS FOR JEE & NEET =>(@iitjeeadv)


Limits 1.41

Solution: lim
e x  e x  2x
= lim
 e  1  t
t

= lim
et  t  1

1
x 0 x  sin x


2
t 0 t 0 t2


 e 1  2
t 2
  .t
e x  e x  2x l1
 t 
= lim = l (say), where
x 0 x  sin x

2
. x3 (using special limit(ii))


x3
e x − e − x − 2x x-sinx Caution
l1 = lim 3
and l2 = lim 3
x →0 x →0

x x 1 ln (1  y) 1  1 1
Doing lim   ·     0 using
x − sin x 3t − sin 3t y0  y y y  y y
Now l2 = lim = lim Put x =3t
x→0 x3 t→0 27 t 3 ln (1  y)
lim  1 , is not correct.


3t − (3 sin t − 4 sin t ) 3 y0 y
= lim
27 t 3 (1  x )1/ x  e

t→0
Example 38: Evaluate lim
3( t  sin t ) x 0



4 sin 3 t 1 4 x
= lim  lim = l2 +
3 t 0 3
(1  x )1/ x  e

t→0 27 t 27 t 9 27
Solution: lim
x 0


8l2 4 1 x
⇒ = ⇒ l2 = .
 ln(1 x ) 1 

9 27 6
e x  1 e
e x  e x  2x  
Now l1 = lim Put x = 3y
ps   , assume M = ln(1 x )
x 0 x3 = lim 1


x

x 0 x
el
e3 y − e −3 y − 6 y eM  1 e M eM
eh

= lim 3 = lim . = lim



y→0 27 y x→0 x
x 0 M

x
je

e   
3
y
 e y  3 e y  e y  6 y ln(1  x )  x e
=−
iit

= lim = e. lim
3
x2 2

27 y x 0

y→0
@

3 {using previous example}


 e2 y  1  1  e y  e y  2 y 

8
= lim    lim   1 1
27  2y  y 0 9
 y3  Example 39: Evaluate lim − 2

y→0
−1 2


x→0 (sin x) x
8 1
= + l1 1 x2 1
−.

27 9 Solution: Solving lim 2
. −1 2
x2


x→0 x (sin x)

8l1
=
8 ⇒ l1 = 1 1 1
9 27 3 −

= lim 2 = 0, is wrong.
x2

x→0 x
1 6
∴ The required limit = . = 2 .
The correct way is to put x = sin θ

3 1
1 1
 1 ⇒ lim − 2
Example 37: Evaluate lim x  x 2 n 1   x→0 (sin −1
x) 2
x
 x


x

 1 1 sin 2   2
1  
x
 = lim = lim
Solution: lim x 1  n 1    Put x = 1/y 0 2 sin 2  0 2 sin 2 

x    x  


(sin   ) (sin   )
 1 n (1  y)  = lim
= lim    0 4

y0  y y2 


sin     1 1
Put ln(1+y) = t ⇒ 1+y = et = 2. lim = 2     .
0  3
 

6 3

⇒ y = et –1 ⇒ as y→0, t→0 (using special limit(i))


 1 
t sin x − x 2 − {x}.{− x}
= lim    Example 40: Evaluate lim
t 0  e t 1 (e  1) 
t 2
x cos x − x 2 − {x}.{− x}



 x→0

FREE BOOKS FOR JEE & NEET =>(@iitjeeadv)


1.42 Differential Calculus for JEE Main and Advanced

where {.} denotes the fractional part function.


2 t  n t  1  t2
sin h − h − {h}{−h}2 = 8 t3
2 t . (1  t 2 )3



+
Solution: f(0 ) = lim
h cos h − h 2 − {h}{−h}


h→0
2 t n t  1  t 2
= . 4 t2
sin h − h 2 − h (1 − h ) (1  t )3 (1  t )3


= lim
h cos h − h 2 − h (1 − h )

h→0
1 2 t n t  1  t 2
= . 4 t2
2 (1  t )3 (1  t )3


sin h − h
= lim as t → 1 , y → 0
h cos h − h Put t = 1 + y;

h→0


1 2 (1  y)  n (1  y)  1  (1  y) 2
h − sin h h3 1 1 Limit = lim
= lim · = ·2 = . 2 y→0  y3
h3 h (1 − cos h ) 6

h→0 3
Put 1 + y = ez ; as y → 0, z → 0
− sin h − h 2 − (1 − h )h 1 2 z ez  1  e2 z
Similarly f (0 ) = lim – =− lim
−h cos h − h 2 − (1 − h )h 2 z → 0  ez  1  . z3



h→0
 z 
(sin h  h )
= lim 1 e2 z − 2 z ez − 1
h→0  h (1  cos h )

= lim
2 z→0 z3

sin h
1 2 1 e z (e z − e − z − 2 z) 1
h lim
= lim = = 1.
ps = = .l
h0 1  cos h 2 2 z→0 z3


2
el
Since f (0–) ≠ f (0+), the limit does not exist. e3 u − e − 3 u − 6 u

 1  x  x
l = lim when z = 3 u
eh

27 u 3

u→0
x  n 2

Example 41: Evaluate lim (eu  e  u )3  3 (e u  e  u )  6 u


je

x→0 x3


= lim
27 u 3
iit

u→0

Solution: Let 1 + x2 − x = t


3
@

⇒ x → 0 , t → 1  e2 u  1  1
= lim   + l
1 − t2


u→0  2 u  9
Also 1 + x2 = t2 + x2 + 2 t x ⇒ x = .
2t


8 8 1
1  t2 ⇒ l = ⇒ l =
 n t 9 27 3
2t
The given expression = 8 t3 1
(1  t 2 )3 ∴

The required limit = .
6

Concept Problems H

1. Evaluate the following limits :
xn −1
(m, n ∈ N)

sin x o
tan x 3 (ii) lim
xm −1


x→ 1
(i) lim (ii) lim




x→0 x x→0 x
log 2 (1+ x )
2. Evaluate the following limits : (iii) lim



sin 2 x x→0 x
(i) lim (ii) lim sin 8x cot 3x
ln (1 + 8x )




x→0 5 x x→0
(iv) lim
1  cos 5x ln (1 + 7 x )


sin 3x x→0
(iii) lim (iv) lim



x sin 2 x 2 4. Evaluate the following limits :
x 0 3x

3. Evaluate the following limits : ln (1  x ) ln sec x
(i) lim (ii) lim

e4 x  1 x  0 3x  1 x2



x→0
(i) lim


x  0 tan x 5. Evaluate the following limits :

FREE BOOKS FOR JEE & NEET =>(@iitjeeadv)
Limits 1.43

(1  x )1/ m  (1  x )1/ n sin 3x  sin x ln(1  3x sin x )


(i) lim (iii) lim (iv) lim
ln( x  1) x 0 tan x 2



x 0


x→0 x
a x − bx 10. Evaluate the following limits :


(ii) lim 8x  7 x
x 1− x2


x→0 (i) lim x x
x 0 6  5




6. Evaluate the following limits :

(i) lim
sin 2 x sin 2 3x
(ii) lim


x→0 tan 3x ln (1  2 x )
2



x 0

cos x  sec x
(ii) lim (5  1) (4  1)
x2


x0 (iii) lim
 0 (3  1) (6  1)



cos 7 x  cos 9 x
(iii) lim
x0 cos x  cos 5x 1  2x  1


(iv) lim
n (1  sin 4 x ) tan 3x



x→0
(iv) lim
1 sin 5 x


e x→0 11. Evaluate the following limits :


7. Find whether the following limits exists: (1  cos ) 2

(i) lim
1 1  cos x
tan 3   sin 3 


 0
(i) lim
x→0 x 1  cos x


x (1  1  x 2 )
1
cot 1 (ii) lim
ps x 0
1  x 2 (sin 1 x )3


(ii) lim x


el
x0 x 1  cos x
| x −1 |  for x  0
eh

(iii) lim x2
−1
x→1 tan ( x − 1) 12. Find lim f(x) if f(x) = 



 x
je

x→0
for x  0
(iv) lim x4 sin (2 x )  2 x  x 2
iit


x0
8. Evaluate the following limits : 13. Evaluate the following limits :
@


sin x  cos x

e tan x  e x (i) lim
(i) lim x  / 4   4x


x  0 tan x  x


n cos x sin(e x 1  1)
(ii) lim (ii) lim
x 1


ln x


x→0 4 1  x2 1
1  sin 3x  1 3
7  x3  3  x 2
(iii) lim (iii) lim
ln (1  tan 2 x ) x 1


x 0 x 1


cos x
1 x  x2 1 (iv) lim


(iv) lim x  / 2 3
(1  sin x ) 2


x→0 sin 4 x
9. Evaluate the following limits : 14. Evaluate the following limits :

x ln x  1

1  x sin x 1 1  cos 5x (i) lim (1  x ) tan (ii) lim
(ii) xlim  x e




(i) lim x1 2 x e
x 0 x2 0 1  cos 3x


x3  x 2  2
(iii) lim (1  x ) log x 2 (iv) lim
x 1 sin( x  1)



x1

Practice Problems G

sin(1  x )  sin(1  x ) 1  tan x  1  sin x 1
15. Show that lim = 2cos 1. 16. Prove that lim 3
=

x 0

x x 0 x 4

FREE BOOKS FOR JEE & NEET =>(@iitjeeadv)


1.44 Differential Calculus for JEE Main and Advanced

cos( x e x )  cos( xe  x ) 26. Evaluate the following limits :


17. Evaluate lim
x 0 3 sin 2   sin 2 


x (i) lim
   2  2



tan([2 ]x 2 )  tan([ 2 ]) x 2
18. Evaluate lim where [.]
sin 2 x
x 0  ya y 

(ii) lim  sin
2a 
. tan
y a 



denotes the greatest integer function. 2

sin( n ) 27. Evaluate the one-sided limits and find whether the limit
19. Evaluate lim (m and n positive integers) exists:
(sin ) m

 0


x sin( x − [ x ])
sin(a  2h )  2 sin(a  h )  sin a (i) lim
x −1




x→1
20. Evaluate lim
h2

h 0
1 − cos 2( x − 1)
21. Evaluate the following limits : (ii) lim
x −1




x→1
n (1+ a sin x )
(i) lim {x}sin( x  2)


x→0 sin x 28. Evaluate lim , (where {.} denotes the
( x  2) 2


x  2
1 − cos x fractional part function.)
(ii) lim
x0 sin 4 3 x


29. Evaluate the following limits :

cos 1 (1  x )
2
sin 3x
(iii) lim (i) lim
ln cos( 2 x − x ) 2







x→0 x 0 x
ps
x
2 sin x  sin 2 x 1  sin
el
(iv) lim 2
x3 (ii) lim


x0
x x
eh

x  x
22. Evaluate the following limits : cos  cos  sin 
2 4 4

je

3 sin x  x 2  x 3
(i) xx  aa
iit

lim
tan x  2 sin x  5x 2 4 (iii) lim (a  0)


x→0
x a x  a


@

1  cos x  2 sin 1 x  sin 3 x 30. Evaluate the following limits :


(ii) lim

3 tan 1 x  x 2  x 5


x 0
ln(1  x  3x 2  2 x 3 )
1 1 (i) lim
sin 2 x  (sin x )  (tan
2
x) 2
 3x  4 x 2  x 3 )


x 1 ln(1
(iii) lim


x→0 3x
x x 1
23. Evaluate the following limits : (ii) lim
x 1 x ln x



sin 3
x ln (1  3x ) 31. Evaluate the following limits :
(i)
 
lim



x 0 1 2
(tan x) e 53 x
1
  cos 1 x
(i) lim
x 1


x  1
e x  33x
2

(ii) lim
x 0  x2 


ln x
sin    sin x (ii) lim
x 1 x4 1


 2 
32. Evaluate the following limits :

logsec x / 2 cos x
(iii) lim 4 2  (cos x  sin x )5
(i) lim


x→0 logsec x cos x / 2
1  sin 2 x


x/4
1 − cos 7 (π − x ) sin( x   / 6)
24. Evaluate lim , n = 1, 2. (ii) lim
x →π 5 ( x − π)n

3  2 cos x


x/6
25. Evaluate the following limits : 33. Evaluate the following limits :


1  tan x 3
x2 − xx
log10 (1  2 x  3x 2  4 x 3 )
(i) lim (ii) lim (i) lim
x 3 / 4 1  2 cos 2 x → 0 1 − cos x x 0 log10 (1  x  2 x 2  7 x 3 )






x

FREE BOOKS FOR JEE & NEET =>(@iitjeeadv)


Limits 1.45

tan  (ii) Existent × Non-existent → no comment



(ii) lim For example, consider lim (x – 1).[x]
 0 3


(1  cos ) 2


x→1

L.H.L. = 0.(–1) = 0, RHL = 0.0 = 0.


cos x  cos x
3


(iii) lim The limit exists even if one of the individual limits
sin 2 x


x 0


does not exist. However, this may not always hold.
1  cos x cos 2 x For example, the limit
(iv) lim
x 0 x2


lim (cosx).21/x does not exist.


x→0

1.10 Algebra of Limits In fact, all the limit questions in 0 × ∞ form cannot



be determined in advance.
Earlier, we had seen the theorem on limits. But we face some (iii) Non-existent × Non-existent → no comment
difficulty in applying these theorems in several situations. We



should take some precautions while applying the theorems. For example, consider lim [ x ].[ x ]
x 0


1. lim(f ( x )  g ( x )) = lim f ( x )  lim g ( x ) L.H.L. = (–1).0 = 0, RHL = 0.(–1) = 0.


x a x a x a

The limit exists even if both the individual limits do not


(i) Existent + Existent → Existent exist. However, this may not always hold.


 sin x  tan x  For example, the limit lim [ x ].[ x ] does not exist.
lim   x 1

x 0 
x 
L.H.L. = 0.(–1) = 0, RHL = 1.(–2) = –2.

   ta n x 

 lim f (x) 
sin x
= lim    lim 
x 
ps
x 0  x  x 0 

lim  f ( x ) / g ( x )   x a

 lim g(x) 
3.
el
x a

= 1 + 1 = 2.

x a
(ii) Existent + Non-existent → Non-existent
eh


 sin 3 x  tan x  Existent
 Existent
je

lim   Existent (non  zero)


x 0  x3
 
iit

In all other cases, we need to examine the limit carefully.


3
@

 sin x   ta n x 
= xlim    xlim   4. If lim f(x) = , then
0  x  0  x 3 

x →a

= 1 + ∞ = ∞.
 
1 1
lim  f ( x ) 
1/ n
 lim f ( x )

n
= λn .
 sin x 1

x a x a
Also, lim   sin  is non-existent.
x 0  x x

If n is an odd integer, then there is no concern. If n is even and 
(iii) Non-existent + Non-existent → No comment is positive, again there is no concern. If n is even and  is zero,


For example, consider lim ([x] + [–x]) we need to examine the domain of the function.

x→0
lim (sin x)1/3 = 01/3 = 0

L.H.L. = –1 + 0 = –1, RHL = 0 + (–1) = –1. x→0

The limit exists even if both the individual limits lim (cos x)1/2 = 11/2 = 1


do not exist. However, this may not always hold. x→0

For example, consider lim (2[x] + [–x]) lim (sinx)1/2 = 01/2 = 0. Note that the function is defined only

x→0 x→0
L.H.L. = 2(–1) + 0 = –2, RHL = 2. 0 + (–1) = –1. in the right neighbourhood of x = 0. Thus, the limit evaluated

We see that the limit does not exist. is actually the right hand limit lim (sin x )1/ 2 = 0.
  

x 0
2. lim(f ( x ) . g ( x )) = lim f ( x ) . lim g ( x )

x →a x a x a
However, lim( sin 2 x )1/ 2 is not equal to 01/2 i.e. 0, since the
(i) Existent × Existent → Existent x 0


 sin x. tan 1 x  function (− sin 2 x )1/ 2 is not defined either in the left or the
lim  
x 0  x2 right neighbourhood of x = 0.
 

 

m
 ta n 1 x 
m
 sin x  lim  f ( x )  n  lim f ( x ) n
, where m and n are integers.
= lim    lim  
x 0  x  x 0 
x a x a
 x 

= 1 × 1 = 1. Here also we need to examine the domain of the function.

FREE BOOKS FOR JEE & NEET =>(@iitjeeadv)
1.46  
Differential Calculus for JEE Main and Advanced

Note: wrong. We should not calculate the limit of some term which



is not as a separate factor to the problem.
A. If lim[f ( x )  g ( x )] exists then we can have the following
x c 1 − cos(1 − cos x ) 1

cases: (ii) We have lim = by proceeding with
x4


x→0 8
(i) If lim f ( x ) exists, then lim g ( x ) must exist.
x →c x →c  x


1  cos  2 sin 2 
Since g = (f + g) – f, we have by the theorem on  2.

limits


4
lim g ( x )  lim(f ( x )  g ( x ))  lim f ( x ) and this exists.
x
x c x c x 0  1  cos x 2 

1  cos  .x 
(ii) Both lim f ( x ) and lim g ( x ) do not exist.  x2 
x →c x →c However, solving as lim


4


x→0 x
Consider lim[ x ] and lim{x} .
1 − cos( x 2 / 2)

x →1 x →1
1  cos x 1
where [.] and {.} represent greatest integer and = lim [since lim  ]
x4 x2


fractional part function, respectively. Here both the
x→0 x0 2
limits do not exist but lim([ x ]  {x})  lim x  1 x2 x2
exists.
x 1 x 1 2 sin .sin
= 4 4 = 1 is wrong, although the answer
B. If lim f ( x )g ( x ) exists, then we can have the following
2
x x 2 8
x →c

16 . .
cases: 4 4
(i) Both lim f ( x ) and lim g ( x ) exist. Obviously, then may be correct.
x →c x →c


ps
e x  cos x  1  1
2

e x  cos x
2

lim f ( x )g ( x ) exists.
x →c (iii) lim = lim
el
x2 x 0 x2

x0
(ii) lim f ( x ) exists and lim g ( x ) does not exist.
eh

x →c x →c


(e x  1) 1  cos x
2

1 3
1 = lim 2
 lim 2
 1 
x 0 x 0

je

Consider f(x) = x; g(x) = . x x 2 2



sin x
e x − cos x
2
iit

lim f ( x ) = 0 exists but lim g ( x ) does not exist. However, solving as lim
x →0 x →0

x2

@

x→0
Here lim f ( x ) . g ( x ) exists and equals 1 but this may
x →0

not be always true. = lim = 1 (as cos x → 1 as x → 0) is wrong.

x→0
(iii) Both lim f ( x ) and lim g ( x ) do not exist.
x →c x →c


esin 2 x  esin x
1 if x  0 (iv) lim

x0 x
Let f(x) =  and
2 if x  0

esin 2 x sin x  1 sin 2 x  sin x
2 if x  0 lim
x 0 sin 2 x  sin x
· = 1·[2 – 1] = 1

g(x) =  . Here lim f ( x ) and lim g ( x ) x
1 if x  0 x →0 x →0

However, solving as

do not exist while lim f ( x ).g ( x ) exists and sin 2 x sin x
x →0
esin 2 x − esin x 2x
.2x
−e x
.x
equals 2. But this may not be always true. lim = lim e
x

x→0 x→0
x
Caution
e 2x
−e x
e (e −1)
x x
= lim = lim =1
1  cos x cos 2 x

3 x→0 x x→0 x
(i) To prove lim 2

is wrong although the end result may be correct.

x 0 x 2

we rationalize the numerator first and proceed. 5. Composition Law

1 − cos x cos 2x It is tempting to write
However, solving as lim 2

x→0 x lim x2  9  lim ( x 2  9) ...(1)
1 − cos x x  4 x  4


= lim using lim cos 2x = 1 is
x2  (4) 2  9  25  5 .

x→0 x→0

FREE BOOKS FOR JEE & NEET =>(@iitjeeadv)
Limits 1.47

But can we simply "move the limit inside the radical" in (1) ?  1 x 
To analyze this question, let us write Now consider lim n  
x 1  1 x
g(x) = x and f(x) = x2 + 9.  

1 x 1 1
Then the function that appears in (1) is the composite function lim  and y = λnx is continuous at x = .
1 x 2


x 1 2
g (f ( x ))  f ( x )  x 2  9 .  1 x  1
Hence our question is whether or not ∴ lim n 
 1 x   n .
x 1 2


 
lim g (f ( x ))  g ( lim f ( x )) .
x a x a But one should note that even if limit of f(x) does not exist,

The next limit law answers this question in the affirmative, the limit of g(f(x)) may exist.
provided that the "outside" function" g meets a certain For example, let f(x) = sgn(x), whose limit does not exist at
condition; if so, then the limit of the composite function g(f(x)) x = 0, and g(x) = x2.
as x → a may be found by substituting into the function g the We see that lim g (f ( x )) = lim (sgn( x )) 2 = 1.
x →0
limit of f(x) as x → a. x →0
A. If lim f ( x )  b and g(x) is continuous at x = b, then Also, the limit of g(f(x)) may exist even if g(x) is discontinuous.
x a

 
For example, let f(x) = x2 + 1, whose limit at x = 0 is 1, and
lim g (f ( x ))  g lim f ( x ) = g(b). g(x) = [x]. We know that g(x) is discontinuous at x = 1.
x a x a
Proof: Since g is continuous at b, for every neighbourhood However, lim g (f ( x )) = lim[ x 2  1] = 1.
x →0 x 0

N of g(b) there exists a neighbourhood C of b such that g(y)
is in N for every y in C.
ps  x2 1 
Example 1: Evaluate lim   , [.] is G.I.F.
x 1  4( x  1) 


In turn, since l im f(x)=b there exists a deleted neighbourhood  
el
x →a
D of a such that f(x) is in C for every x in D. Combining x2 1 1

eh

these remarks, we see that for every neighbourhood N of Solution: lim


x 1 4( x  1)


2
g(b) there exists a deleted neighbourhood D of a such that
je

f(x) is in C, and therefore g(f(x)) is in N, for every x in D. 1


y = [x] is continuous at x = .
iit

This proves the result.



2
1
1
@

A function such as y = 2 , is a composition of the Hence the limit is   = 0.


x +3 2
polynomial function f(x) = x2 + 3 and the reciprocal function
g(x) =
1 Study Tip
x
1 1 We know that the function |x| is continuous everywhere; thus,
i.e., g(f(x)) = = 2 . it follows that if lim g ( x ) exists then
f (x) x +3 x →a
Thus, y = g(f(x)). Therefore, since lim f(x) = f(a) and g is
x →a lim g(x)  lim g(x)
x a x →a

1 That is, a limit symbol can be moved through an absolute
continuous at f(a), lim y = .
x →a a2 + 3 value sign, provided the limit of the expression inside the
 x2 1 absolute value signs exists. For example,
 , x 1
Let f(x) =  x  1 , g(x) = sin–1 x .
3 ,x 1 4 lim 5 − x 2 = lim(5 − x 2 ) = | − 4 | = 4 .


x →3 x →3

To find lim g (f(x)) , we ask two questions. An useful application of composition law is the following:
x →1
Let L  lim  f ( x ) 
g(x)

Firstly, whether lim f ( x ) exists . The answer is yes : lim f ( x ) = 2. x a


x →1 x →1
⇒ ln L  lim g ( x ).lnf ( x )
Secondly, whether g(x) is continuous at x = 2. x a

x
We know that sin–1 is continuous. ⇒ L = elim g ( x ).nf ( x )
x →a

4
Thus, using the above theorem, B. If lim f ( x )  b , with f(x) ≠ b for every x in some
π

x a
lim g (f ( x )) = g(2) = . neighbourhood of 'a' and if lim g ( x )  c , then
x →1 6

x b

FREE BOOKS FOR JEE & NEET =>(@iitjeeadv)


1.48  
Differential Calculus for JEE Main and Advanced

lim g  f ( x )   c . (This theorem is sometimes useful if = lim g  1  2h 


x a h 0


g is discontinuous at b.) = lim 2(–1 + 2h) + 3 = 1.


h→0
 x2 1
sin x  , x 1 Thus, lim g  f ( x )  = 1.
Let f(x) = , g(x) =  x  1 x 2

x 3
 ,x 1 6. Domination Law
To find lim g (f ( x )) , we ask two questions. Firstly, whether
x →0 If two function f(x) and g(x) satisfy the inequality


lim f ( x ) exists. The answer is yes: lim f ( x ) = 2. Secondly, f(x) < g(x) ...(1)



x →0 x →0
x2 −1 for all the values of x belonging to a neighbourhood of a point
whether lim g( x ) exists. We have lim g ( x ) = lim = = 2. a except possibly at a, then
x →1 x →1 x →1 x −1
Thus, using the above theorem, lim g (f ( x )) = lim g ( x ) = 2. lim f ( x )  lim g ( x ) ...(2)
x →0 x →1 x a x a



Note that the above limit has been evaluated even if g(x) is provided that the limits of both functions, as x → a, exist.
discontinuous. Further, lim g (f ( x ))  g lim f ( x )
x 0
 x 0
 Note that a strict inequality connecting functions may lead to
a non-strict inequality for their limits. For example, if 0 < |x|
 x2 1 < 1 then x2 < |x|. But nevertheless,
 , x 1
Now let f(x) = 1, g(x) =  x  1 . lim x 2  lim | x |  0 .
3  x 0 x 0

 , x 1
Find lim g (f ( x )) .
ps sinx tanx
x →0 Now consider f(x) = and g(x) = .
x x
The theorem is not applicable since the function f(x) attains
el
the value 1 in the neighbourhood of '0'. We have f(x) < g(x) in the neighbourhood of x = 0,
eh

We see that g(f(x)) = 3. Hence lim g (f ( x )) = 3. sin x tanx


x →0
x ≠ 0. Since lim = lim = 1,
x →0
x x →0 x
je

1  x ,  1  x  2
Example 2: If f(x) =  we have lim f ( x )  lim g ( x ) .
iit

 2 x  5, 2  x  3 x 0 x 0


 Thus, we see the necessity of the equality sign in the
@

 x 2  1 ,  2  x  1
g(x) =  domination law.
2x  3 ,  1  x  1


The theorem remains of course true if the strict inequality
find (i) lim g  f ( x )  and (ii) lim g  f ( x )  .
x 1 x 2 f(x) < g(x) entering into its formulation is replaced by the
Solution: (i) lim f ( x ) = lim 1  x = 0. non-strict inequality f(x) ≤ g(x).
x →1 x 1 In the case x → ∞ we must require that inequality (1) should



Also, g(x) is continuous at x = 0. hold for x > N where N is a fixed number.
 

Hence lim g  f ( x )   g lim f ( x ) = g(0) = 3. An important particular case of the theorem arises when one of
x 1 x 1

the two functions g(x) and f(x) is a constant. For definiteness,
(ii) lim f ( x ) does not exist. The theorem is not let g(x) = constant. Then the theorem reads :
x →2


applicable. We evaluate the one sided limits : If f(x) < M (or f(x) ≤ M) for 0 < |x – c| < δ where δ is a
L.H.L. = lim g  f ( x )  fixed number then lim f ( x )  M provided that the limit of
x  2

x c
= lim g  f (2  h )  the function f(x) exists.

h 0

= lim g 1  (2  h ) 
h 0
1.11 Limits when x → ∞

= lim g (1  h )

h 0

Sometimes we will not be concerned with the behaviour of g(x)
= lim 2(1  h )  3  1 . near a specific value of x, but rather with how the values of f(x)
h 0

behave as x increases without bound or decreases without bound.
R.H.L. = lim g  f ( x )  This is sometimes called the end behaviour of the function

x  2
= lim g  f (2  h )  because it describes how the function behaves for values of x

h 0 that are far from the origin. To investigate the end behaviour of
= lim g  2(2  h )  5  = lim g  4  2h  5  a function we find limits of the function as x → –∞ or as x → ∞.

h 0 h 0

FREE BOOKS FOR JEE & NEET =>(@iitjeeadv)


Limits 1.49

To discuss the end behaviour of a function we assume that Solution: (a) Divide numerator and denominator of
the function is defined on an infinite interval.



x 1
by x2 (the highest power of x in the denom-
Let the independent variable x of a function y = f(x) increase ( x  3) ( x  2)
indefinitely. This means that x is made to take on the values 1 / x  1 / x2
becoming greater than any given positive number. In such a inator), obtaining
(1  3 / x ) (1  2 / x )
case we say that x becomes infinitely large or that x approaches
infinity and write x → ∞. If x decreases indefinitely, that is, Hence, as x → ∞,
becomes less than any given negative number, we say that x
0−0 0
becomes negatively infinite or that it approaches minus infinity y→ = =0
and write x → –∞. (1 + 0) (1 − 0) 1


Definition A number λ is said to be the limit of the function As x → − ∞, the factors x – 1, x + 3, and x – 2 are negative,
and, therefore, y → 0–. As x → ∞, those factors are positive,

y = f(x) as x → ∞ if for all sufficiently large values of x the
corresponding values of the function f(x) become arbitrarily and, therefore, y → 0+.
close to the number λ. ( x  2) (1  x )
(b)
x 3


Limits as x Tends to – ∞
We can easily understand the meaning of the statement: f(x) − x 2 − x + 2 − x − 1 + 2 /x
= = , after dividing
approaches λ as x → – ∞ .
x −3 1 − 3/x
In fact, if x = – t and f(x) = f(– t) = g(t) then t tends to ∞ numerator and denominator by x (the highest power of x in
as x tends to – ∞, and the question of the behaviour of f(x) the denominator).
ps
as x tends to – ∞ is the same as that of the behaviour of g(t) As x → ∞, 2/x and 3/x approach 0, and –x – 1 approaches
as t tends to ∞. ∞. Thus, the denominator approaches 1 and the numerator
el
2x  1 approaches ∞.
eh

Example 1: Evaluate lim .


x 1 As x → –∞, x + 2 and x – 3 are negative and 1 – x is positive ;


x
je

Solution: Note that both the numerator and the denom- so, y → ∞. As x + 2 and x – 3 are positive and 1 – x is negative;
so, y → –∞. It should be emphasized that the symbols ∞ and


iit

inator approach infinity as x approaches infinity.


–∞ are not real numbers. The phrase “f(x) approaches ∞” is
@

lim (2 x − 1) → ∞ equivalent to saying that “f(x) approaches the unapproachable”


x →∞
2x − 1 or “f(x) increases without bound”.
lim
x →∞ x +1 “The symbols ∞ and –∞ are used here to encapsulate a
lim ( x + 1) → ∞ particular way in which limits fail to exist. It is important, to
x →∞
keep in mind that “∞” is not a number. The limit does not exist.

To resolve this problem, we divide both the numerator and the To say, for example, that f(x) → ∞ as x → a+ is to indicate
denominator by x. After dividing, the limit may be evaluated that lim f(x) does not exist and that it fails to exist because
as follows: x a 
the values of f(x) increase without bound as x approaches a
2x  1
2x  1
from the right. Furthermore, since ∞ and –∞ are not numbers,
lim  lim x
it is inappropriate to manipulate these symbols using rules
x  x  1 x  x  1
of algebra. For example, it is not correct to equate (∞ – ∞)
x
with 0.

1
lim 2  lim
= lim
2 x  (1 / x )
=
x  x 
x How Limits at Infinity Can Fail to Exist?
x  x  (1 / x ) 1


lim 1  lim Limits at infinity can fail to exist for various reasons. One
x  x  x possibility is that the values of f(x) may increase or decrease
20 without bound as x → ∞ or as x → –∞. For example,
= 2.
1 0 the values of f(x) = x3 increase without bound as x → ∞

and decrease without bound as x → –∞. We denote this by
Example 2: Determine what happens to y as x → –∞ writing


and x → ∞.
lim x3 = ∞, lim x3 = – ∞,
x 1 ( x  2) (1  x )


x x
(a) y = (b)
( x  3) ( x  2) x 3 3
lim (–x3) = ∞


lim (–x ) = –∞,


x x

FREE BOOKS FOR JEE & NEET =>(@iitjeeadv)


1.50  
Differential Calculus for JEE Main and Advanced

If the values of f(x) increases without bound as x → ∞ or as because both x and x – 1 become arbitrarily large and so their
x → – ∞, then we write product does too.
lim f(x) = ∞ or lim f(x) = ∞ Example 3: Discuss the behaviour of 2x3 – 11x2 + 12x

x



x
when x is large.
and if the values of f(x) decreases without bound as x → ∞
or as x → – ∞, then we write Solution: First consider x positive and large. The three



terms, 2x3, –11x2 , and 12x, all become large in absolute
lim f(x) = –∞ or lim f(x) = – ∞
value. To see how 2x3 – 11x2 + 12x behaves for large positive

x x

Limits at infinity can also fail to exist because the graph of the x, factor out x3.
function oscillates indefinitely in such a way that the values  11 12 
of the function do not approach a fixed number and do not 2x3 – 11x2 + 12x = x 3  2   2  ...(1)
 x x 



increase or decrease without bound; the trigonometric functions
sin x and cos x have this property. In such cases we say that Now, since 11/x and 12/x2 → 0 as x→ ∞
the limit fails to exist because of oscillation.  11 12 
lim  2   2   2
Note: If f(x) → l as x → ∞, and l is not zero, then f(x)cosxπ x   x x 
and f(x)sinxπ oscillate finitely as x → ∞. If f(x) → ∞ or


f(x) → –∞, then they oscillate infinitely. The graph of either Moreover, as x → ∞ , x3 → ∞,
function is a wavy curve oscillating between the curves y =  11 12 
f(x) and y = –f(x). Thus lim x 3  2   2   

x   x x 
End Behaviour of a Polynomial Hence lim (2x3 – 11x2 + 12x) = ∞
ps x
The end behaviour of a polynomial matches the end behaviour
Now consider x negative and of large absolute value. The
el
of its highest degree term.
More precisely, if cn ≠ 0 then argument is similar. Using (1), and the fact that
eh

 11 12 
lim (c0 + c1x + ... + cnxn) = lim cnxn lim x3 = – ∞ and lim  2   2   2
je


x x x x   x x 
iit

lim (c0 + c1 x + .... + cnxn) = lim cnxn it follows that lim (2x3–11x2 + 12x) = – ∞

x x
x
@

We can get these results by factoring out the highest power This completes the discussion. It is interesting to graph f(x) =
of x from the polynomial and examining the limit of the
2x3–11x2 + 12x and see what is happening for |x| large. The
factored expression.
figure is shown below.
 c0 c  Since lim (2x3–11x 2 + 12x) = ∞ the graph rises arbitrarily
Thus, c0 + c1x + .... + cnxn = xn   n11  ...  c n 
x  x
n
x
high as x → ∞.
As x → –∞ or x → ∞, it follows that all of the terms with
positive powers of x in the denominator approach 0, so the Since lim (2x3–11x2 + 12x) = − ∞ , the graph goes arbitrarily
x
above limits are certainly plausible. far down as x → – ∞ ,
For example lim (7x5 – 4x3 + 2x – 9)
x

= lim 7x5 = – ∞

x

and lim (–4x8 + 17x3 – 5x + 1)



x

= lim – 4x8 = –∞

x

Let us find lim (x2 – x).


x
Note that we cannot write
lim (x2 – x) = lim x2 – lim x = ∞ – ∞.

x x x
The above example generalizes to any polynomial function
The limit laws can't be applied to infinite limits because f(x) = anxn + an–1xn–1 + ..... + a0.
(∞ – ∞) is indeterminate. However, we can write Let f(x) be a polynomial of degree atleast 1 and with the
lim (x2 – x) = lim x (x – 1) = ∞ leading coefficient an positive. Then

x x

FREE BOOKS FOR JEE & NEET =>(@iitjeeadv)


Limits 1.51

lim f ( x )   We obtain
x 
5x 3  2 x 2  1 5x 2  2 x  1 / x

If the degree of f is even, then lim f ( x )   lim = lim =∞
x   x  3x  5 x  35/ x
But if the degree of f is odd, then lim f ( x )   where the final step is justified by the fact that
x 
1 5
5x2 – 2x → ∞, → 0, and 3 + →3
End Behaviour of a Rational Function


x x
x 3 + 6 x 2 + 10 x + 2 as x → –∞.
Let us determine how f(x) = Let f(x) be a polynomial and let axn be its term of highest
2x 3 + x 2 + 5 degree. Let g(x) be another polynomial and let bxm be its term
behaves for arbitrarily large positive number x. of highest degree.
As x gets large, the numerator x3 + 6x2 + 10x + 2 grows large, f (x) ax n
influencing the quotient to become large. On the other hand, Then lim  lim
x  bx m


x  g ( x )
the denominator also grows large, influencing the quotient to
become small. An algebraic step will help reveal what happens f (x) ax n
and  lim
lim
to the quotient. We have x  g ( x ) x  bx m


 6 10 2  The proofs of these facts are similar to the argument used in
x 3 1+ + 2 + 3 
x 3 + 6x 2 +10x + 2  x x x  the previous example. In short, when working with the limit
f(x) = = of a quotient of two polynomials as x → ∞ or as x → −∞ ,
3 2
2x + x + 5  1 5
x3  2 + + 3  disregard all terms except the one of highest degree in each of
 x x  ps
the polynomials. The next example illustrates this technique.
6 10 2 Alternatively, divide each term of the numerator and
1  
el
 x x 2 x 3 for x ≠ 0 denominator by x k where k is the highest power of
eh

1 5

2  3 x in numerator and denominator. Then use the result
x x c
 0 where c is a constant and k > 0.
je

lim
Now we can see what happens to f(x) when x is large. x  x k
iit

As x increases, 6/x → 0, 10/x2 → 0, 2/x3 → 0, 1/x → 0 and (i) First of all simplify the given expression.

5/x3 → 0.
@

(ii) Then divide each term of the numerator and denominator



1+ 0 + 0 + 0 1 by xm, where x is the independent variable and m is the
Thus, f(x) → = .
2+0+0 2 highest power of x.
c
So, as x gets arbitrarily large through positive values, (iii) Then expression of the form k tends to 0, where c is a

x
x 3 + 6 x 2 + 10 x + 2 1 constant and k is a positive number since
the quotient approaches . c
2x + x + 5 3 2 2 when x → ∞ or x → – ∞, → 0, k > 0.
xk

The technique used in this example applies to any function
that can be written as the quotient of two polynomials i.e. a Example 5: Examine the following limits:


rational function. 3x 4  5x 2
A useful technique for determining the end behaviour of (a) lim
x   x 4  10 x  5


rational function f(x) = p(x)/q(x) is to factor and cancel
x 3  16 x
the highest power of x that occurs in the denominator q(x) (b) lim 4
x  5x  x 3  5x


from both p(x) and q(x). The denominator of the resulting
fraction then has a (nonzero) limit equal to the leading x4  x
(c) lim
coefficient of q(x), so the limit of the resulting fraction can x  6 x 3  x 2


be quickly determined. The following examples illustrate Solution: By the preceding observations,


this technique. 3x 4  5x 2 3x 4
(a) lim  lim  lim (3) = –3
5x 3 - 2x 2 +1 x   x 4  10 x  5 x  x 4 x 


Example 4: lim
x  x  16 x
3
x3


3x + 5 1
(b) lim = lim  lim 0
5x  x  5x x  5x
4 3 4 x  5x


x 
Solution: Divide the numerator and denominator


by the highest power of x that occurs in the denominator, x4  x x4 x
(c) lim = lim  lim  
x  6 x  x 3


namely x. 3 2 x  6x x  6

FREE BOOKS FOR JEE & NEET =>(@iitjeeadv)


1.52  
Differential Calculus for JEE Main and Advanced

In short, let f(x) = a0xm + a1xm–1 + .... + am –1x + am, lim f(x) = 3 and lim f(x) = 3.


and g(x) = b0xn + b1xn–1 + .... + bn –1x + bn be two polynomial x x

functions. Then We see that the line y = 3 is a horizontal asymptote of the


f (x) a 0 3x 2
(i) lim  if m = n graph of f(x) = .
x2 +1



x  g ( x ) b0
f (x)
0  ∞
(ii) lim if m < n Limits of Form  ∞ 



x  g ( x )

f (x) f (x)
(iii) lim   if m > n. Consider the limit lim where f(x) → ∞ and g(x) → ∞



x  g(x ) x →a g ( x )
(It is ∞ if and only if a0 and b0 have the same signs. as x → a. Here a may be finite or infinite.

f (x) 
(iv) lim   if m > n. This is called   form.




x   g ( x )
a One way to handle this indeterminate form is to write the
(The correct sign is the sign of (–1)m–n 0 .
b0 1
a 0 x m  a1x m 1  ....
∴ l im limit lim
f (x)
as lim
g( x )
which enters
x  b0 x n  b1x n 1  ... x →a g ( x ) x →a 1

 a0 f (x)
 b , when m  n
0
ps
 0 into   form which can be solved by methods
 0, when m  n 0
el

=  discussed earlier.
eh

a0
 , when m  n and 0

b0 cot x 
 For example, lim 
je

x 0 cot 2 x  
 a0
iit

 , when m  n and 0 tan x 0


 b0 = xlim 0
@

0 tan 2 x  

Thus, the limit of the ratio of two polynomials is one of the
which can be evaluated using standard limits to get 1/2.
following : the ratio of leading coefficients of the polynomials,
if their degrees coincide; infinity, if the degree of the numerator Another way is to find the largest term occurring in the
is greater than that of the denominator; zero, if the degree of numerator and denominator and divide them by this term and
the numerator is less than that of the denominator. evaluate the limit using the basic result:
1
Definition of Horizontal Asymptote If a term p(x) → ∞ as x → a then its reciprocal →∞
p( x )
The line y = L is a horizontal asymptote of the graph of f if as x → a.
lim f(x) = L or lim f(x) = L. x  2 cot x   
x x For example, lim  
x 0 cos x  cot x   
3x 2 x
Consider the graph of f(x) = . 2
x2 +1 x tan x  2
= lim cot x = lim = 2.
x 0 cos x x 0 sin x  1

1
cot x
x 2  4x  5
Example 6: Evaluate lim
x 3  3x 2  2 x


x

Solution: Dividing the numerator and denominator by




x3 which is the highest power of x.
The limit of f(x) as x approaches – ∞ or ∞ is 3. 1 4 5
 2 3
As shown in the figure, we see that the value of f(x) approaches = lim
x x x
3 as x increases without bound (x → ∞). Similarly, f(x) 3 2

x
1  2
approaches 3 as x → – ∞. These limits at infinity are denoted by x x

FREE BOOKS FOR JEE & NEET =>(@iitjeeadv)


Limits 1.53

000 Solution: In both parts it would be helpful to manip-


0



=
1 0  0 ulate the function so that the powers of x are transformed to

powers of 1/x. This can be achieved in both cases by dividing
x 3  4x 2  5 the numerator and denominator by |x| and using the fact that
Example 7: Evaluate lim
x 2  3x  2


x
x 2 = |x|.
Solution: Dividing the numerator and denominator by (a) As x → ∞, the values of x under consideration are positive,



x3 which is the highest power of x. so we can replace |x| by x where helpful. We obtain
4 5
1  3 x2  2 x2  2 / | x |
x x lim = lim
= lim x  3x  6 x  (3x  6)/ | x |



x 1 3 2

 2 3
x x x
x2  2 / x2
1 0  0 = lim
=  x  (3x  6) / x



000

Hence, the limit does not exist.
lim 1  2 / x 2

1 2 / x2
x  2 x  3x  4
3 2
= lim = x 
Example 8: Find lim x  3  6 / x lim (3  6 / x )


x 4 x 3  3x 2  2 x  1


x 

Solution: This is an indeterminacy of the form ∞/∞. lim (1  2 / x ) 2


1 0 1


x 
Let us divide the numerator and the denominator of the func- = = 
lim (3  6 / x ) 30 3

tion by the leading power of x, i.e. by x3.
ps x 
x 3  2 x 2  3x  4 (b) As x → –∞, the values of x under consideration are
el
lim

x   4 x 3  3x 2  2 x  1 negative, so we can replace |x| by –x where helpful. We
eh

obtain

1  2 / x  3 / x2  4 / x2 1
je

 lim  . x2  2 x2  2 / | x |
4  3 / x  2 / x  1/ x 2 3 = lim

x 4 lim x  (3x  6)/ | x |
iit


x  3x  6

3x 4  2
@

Example 9: lim . x2  2 / x2
x = lim


x  3x  4
8
(3x  6) / ( x )

x 

Solution: Let us divide the numerator and the denom- 2


1


inator by x4. = x2 1
lim 

3x 4  2 x  3  6 / x 3  
lim 
x

x
x 8  3x  4  3
x 
 x  

3  2 / x4 3 Example 12: Evaluate lim 
3
x 
 lim   3. x


x 
x  x3 x 
1  3/ x  4/ x7 8 1
 

2 x 1  ....inf inity 
 x 2  2x  1  2 x 1  
Example 10: Find the value of lim  2 
x  2 x  3x  2 
x


  Solution: Let y =


3
x
x 2  2x  1 1 x+ 3
Solution: Since, lim  and
x+
x
2 x 2  3x  2


x  2
x+3 x
2 x 1
2x  1  x 2  2 x  1  2 x 1 1 ....inf inity
lim  1 we have lim  2  = .
x  2 x  1 x   2 x  3x  2  x
  2 Then y 

y
x
Example 11: Find x 2/3


x2  2 x2  2 x 5/ 3
(a) lim (b) lim ⇒ y=
3x  6 3x  6 x 5/ 3 + y

x 




x

FREE BOOKS FOR JEE & NEET =>(@iitjeeadv)


1.54  
Differential Calculus for JEE Main and Advanced

⇒ y2 + (x5/3)y – x5/3 = 0 1
3  2t 2 .

x 5/ 3
 x 10 / 3
 4x 5/ 3
t2
∴ y = lim
1  2t



t 0
2
t
 x 5 / 3  x10 / 3  4 x 5 / 3
= y (∵ y > 0) 3  2t 2 t
2 = lim


t 0 (1  2 t ) | t |


4x 5/ 3
= 3

2( x10 / 3 + 4 x 5 / 3 + x 5 / 3 ) = =– 3.
−1


e1 x  1
2

2
= Example 15: Evaluate lim
2 arc tan x 2  



 4  x 

1  5/ 3   1
 x  e1 x  1
2

0
Solution: lim  
2 arc tan x    0 
2



2 2 x 
∴ lim y    1.
x  1 0 1

2
e1 x  1 e1 x − 1
2 2

= lim = lim
Study Tip   x −2 cot −1 x 2

2   tan 1 x 2 
x

2 
1
As x → ∞ we get → 0. Hence in many problems 1 e1 x − 1
2
ps
x = – lim
x 2 1

tan −1 2
el
1 1
we write the expression in terms of and apply → 0.
x
eh

x x
To present this in a simple form we use the substitution x = 1/t. 1
As x → ∞, t → 0+
je

Hence, Put x =

t
iit

lim f(x) = lim f(1/t) and 1 et − 1 t 1



x t0 = – lim . −
=– .
@

t →0 2 1


t +
tan t 2
lim f(x) = lim f(1/t), if these limits exist.  1  2x 

t0
 2 x
x 7

Example 16: Find xlim  


For the latter limit we may also write   (1  8x 3 )1/ 3
 


lim f(x) = lim f(– 1/t) using the 1 
 x  2

x t0
substitution x = – 1/ t. Solution: lim    1 = −2 = –1.

7
1/ 3
2x (8)1/ 3


x 

1 1/ x 0  1 
Example 13: Evaluate lim x sin  3  8 
x 


x  x
1 x  cos x
Solution: lim x sin Example 17: Find lim
x  x  sin x




x  x
1 Solution: Here sin x occurs in the given expression and
As x → ∞, t → 0+


Put x = when x → ∞, sin x oscillates between –1 and 1.


t
1 1
sin t Let x = ∴t= and when x → ∞, t → 0
= lim = 1.

t x


t0 t
x  cos x
3x 2  2 Now lim
x  x  sin x

Example 14: Find lim
x   x2


1 1
cos 1  t cos
3x 2  2 1 t t
Solution: lim 
x   x2 t 1  lim t


= lim
1 t 0 1

t 0
−1 sin 1  t sin
As x → – ∞, t → 0+ 1 t t
Put x =
t 

t 1 t

FREE BOOKS FOR JEE & NEET =>(@iitjeeadv)


Limits 1.55

1 Study Tip
1  t cos
t  1  0 1
= lim
1 1 0 For sufficiently large values of x we have

t 0
1  t sin logax << xp << ax << x! where


t a>1 p > 0 a > 1 x ∈ N.


1 1 What we mean by this is that the function xp is much larger than
∵ when t → 0, t sin → 0 and tcos → 0


t t logax for large values of x when p > 0 and a>1.
sin x cos x As x→ ∞, both logax (a > 1)and xp approach ∞, but xp (p > 0)
Note:  lim lim 0 . is much larger than logax.
 x 


x x x
log a x
Note: The graphs of y = ex and y = ln x suggest that as Hence, lim = 0 , for a > 1.


x → ∞, ex → ∞ much more rapidly than ln x → ∞. Thus, x x
when x is large positive, ex is so much larger than ln x that (ln x )
the quotient ex/(ln x) is large positive. Specially, lim 0
x  x
ex
Hence lim = ∞. np an
x  ln x Also, lim = 0 for p > 0 and a > 1, and lim 0.
n  a n n  n !
The exponential function is notable for its very rapid rate of
n
increase with increasing x. In fact, ex increases more rapidly lim 2 = 0 .
For example, n
as x → ∞ than any fixed power of x. n!
x2 4 x  2nx
Thus, lim =0.
ps Example 18: Evaluate lim
x
e x 2  22 x 1


x x 
el
In fact, if n is a positive integer then nx
1 2
eh

ex
 , lim x e n x
0. 4 x  1  2.0   1
lim x  Solution Limit = lim .
02
je

x  x 2
xn


x  2

 2
iit

4x
@

Concept Problems I

1. Evaluate the following limits: 3. Evaluate the following limits:


x 4  5x 3  7 tan x tan 1 x
(i) lim (i) lim (ii) lim
x  2 x 5  3x 4  1






x tan 3x x  x
(2 x  3) (3x  5) (4 x  6) 2
(ii) lim
x  3x 3  x  1 x  sin x


(iii) lim tan–1(x2 – x4) (iv) lim
x  cos 2 x



x x
(2 x 3  4 x  5) ( x 2  x  1)
(iii) lim 1
x  ( x  2) ( x 4  2 x 3  7 x 2  x 1) Show that lim x tan 1 = lim x cot 1 x = 1


4.

x x x
2 x 4  3x 2  5 x  6
(iv) lim 5. Evaluate the following limits:
x 3  3x 2  7 x  1


x 

cos x 9x 6 − x
2. Evaluate the following limits: (i) lim lim
(ii) x →
x3 + 1

x  −∞



(i) lim
2x x2 1
x 5 3
x+


  x 1   x x x x2 1
(ii) lim cos log   (iii) lim (iv) lim
  x  x 1 3x  6


x  x 


x

[ x ] 1 6. Show that lim xn e–x = 0 for all positive integers. Hence


(iii) lim x 



x  x2 ( 6 x 1)/( 3 x  2 )
 3x  2 x  1  x1000
(iv) lim  2  find lim .
 x x2  ex


x  x

FREE BOOKS FOR JEE & NEET =>(@iitjeeadv)


1.56  
Differential Calculus for JEE Main and Advanced

Practice Problems H


2x 2  3 2x 2 + 3 x ex
7. Evaluate lim and lim (i) lim (ii) lim
x  ln 2 x





x  4 x  2 x →−∞ 4 x + 2 x5 x 

8. Assuming that m and n are positive integers, find ln x ln x
(iii) lim (iv) lim

x  3





2  3x n x x 0 1 / x 

lim
x 1 xm . 13. Evaluate the following limits :


x  sin 2 x

9. Evaluate the following limits: (i) lim



x  x
2 x3x5x
(i) lim x (2  sin x )
3x  2  3 2 x  3


x (ii) lim
x2 1



x 
 1 1 
(ii) lim x4  cos x  1  2  x (2  sin 2 x )
 2x 


x (iii) lim
x  x 1



( x  3) 40 (5x  1)10
(iii) lim ln100 x
(3x 2  2) 25


x 
(iv) lim

x5


x 
5
x 7  3  4 2x 3  1
(iv) lim ( x  1)10  ( x  2)10  ....  ( x  100)10
x 


6
x8  x 7  1  x 14. Find lim
x  x10  1010

ps
10. Find lim logx–1 x . logx (x + 1).logx+1 (x + 2).
15. Find the limit of the sequence {xn} which is defined by

x
el
log x −1 x5. the recursive relation:
logx+2 (x + 3) ........
eh

xn+1 = xn(2 – xn) ∀ n ≥ 1, where x1 is an arbitrary number




2 x  3 3 x  4 4 x  .......  n n x satisfying the inequalities 0 < x1 < 1.
je

11. Find lim


x  (2 x  3)  3 (2 x  3)  ....  n (2 x  3)

1
(3x 4  2 x 2 ) sin  | x |3  5
iit

12. Evaluate the following limits: 16. Evaluate lim x .


| x |  | x |  | x | 1
@

x  3 2


1.12 Asymptotes We can also write lim
1
  but there is no corresponding

x 0 | x|
Infinite Limits
1
Consider the function f(x) = 1/x. As x approaches 0 from the statement for lim .
x →0 x
right, 1/x, which is positive, becomes arbitrarily large. The
notation for this is Let f be the function given by f(x) = 3/(x–2). From figure we
can see that f(x) decreases without bound as x approaches 2
1 from the left, and f(x) increases without bound as x approaches
lim .
x 0 x 2 from the right. This behaviour is denoted as

3 3
As x approaches 0 from the left, 1/x, which is negative, has lim = – ∞ and lim = ∞
x 2 x2 x 2 x  2 

arbitrarily large absolute values. The notation for this is
1
lim   .
x 0 x 

The behaviour of 1/x, is quite different from that of
1/x2. Since x2 is positive whether x is positive or negative and
since 1/x2 is large when x is near 0, we have
1 1
lim 2
  and lim 
x 0 x x 0 x2

1
In this case we may write lim   , meaning that
x2 x 0

“as x → 0, both from the right and from the left, 1/x2 becomes
arbitrarily large through positive values.”

FREE BOOKS FOR JEE & NEET =>(@iitjeeadv)


Limits 1.57

If it were possible to extend the graphs toward infinity, we would If the function is a quotient of two functions then the vertical
see that the graph becomes arbitrarily close to the vertical line asymptote occurs at a number where the denominator is 0 and
x = 2. This line is a vertical asymptote of the graph of f. the numerator is not 0.
Example 1: Determine the points x = a for which each Theorem Let f and g be continuous on an open interval



denominator is zero. Then see what happens to y as x → a– containing a. If f(a) ≠ 0, g(a) = 0. and there exists an open
and as x → a+. interval containing a such that g(x) ≠ 0 for all x ≠ a in the
x 1 f (x)
(a) y = interval, then the graph of the function given by h(x) =
( x  2) ( x  2) g( x )

has a vertical asymptote at x = a.
( x  2) ( x  1)
(b) y =
( x  3) 2 Example 2: Determine all vertical asymptotes of the



graphs of the functions:
( x  2) (1  x )
(c) y =
(b) f(x) = x  1 (c) f(x) = cot x
2
x3 1

(a) f(x) =
2( x + 1) x2 1
Solution:

(a) The denominator is zero for x = –2 and x = 1. Solution:



As x → –2–, y → –∞ ; as x → –2+, y → ∞. (a) When x = – 1, the denominator is 0 and the numerator is

As x → 1–, y → ∞ ; as x ∞ 1+, y → –∞.
not 0. Hence, we can conclude that x = – 1 is a vertical

(b) The denominator is zero for x = 3 asymptote.

As x → 3–, y → ∞ ; as x → 3+, y → ∞.
ps
(b) By factoring the denominator as

(c) The denominator is zero for x = 3.

(x2 – 1) = (x – 1) (x + 1)

As x → 3–, y → ∞; as x → 3+, y → –∞.
el

we can see that the denominator is zero at x = – 1

eh

Vertical Asymptotes

and x = 1. Moreover, because the numerator is not 0 at
If f(x) approaches infinity (or negative infinity) as x these two points, we conclude that the graph of f has two
je

approaches a from the right or the left, then the line x = a is vertical asymptotes.
iit

a vertical asymptote of the graph of f. (c) By writing the function in the form

@

The line x = a is called a vertical asymptote of the curve cos x


f(x) = cot x =
y = f(x) if atleast one of the following statements is true : sin x

lim f(x) = ∞ lim f(x) = ∞ lim f(x) = ∞ we conclude that vertical asymptotes occur at all


x →a x →a − x a  values of x such that cos x ≠ 0 and sin x = 0. Hence,
lim f(x) = – ∞ lim f(x) = – ∞ lim f(x) = – ∞ the graph of this function has infinitely many vertical
x a 

x →a x →a − asymptotes. These occur when x = nπ, where n is an
integer.
Note: If a function f has a vertical asymptote as


x = a, then f is discontinuous at a. The theorem requires that the value of the numerator at x = a
be nonzero. If both the numerator and the denominator are 0
at x = a, then we obtain the indeterminate form 0/0, and we
cannot determine the whether a vertical asymptote exists at
x = a or not, without further investigation.

Example 3: Determine all vertical asymptotes of the




x 2  2x  8
graph f(x) = .
x2  4
Solution: We simplify the expression, as follows :


x 2  2x  8 ( x  4)( x  2) x+4
f(x) = = = , x ≠ 0.
x2  4 ( x  2)( x  2) x+2

At all values of x other than x = 2, the graph of f coincides


with the graph of g(x) = (x + 4)/(x + 2). Thus, we conclude
that there is a vertical asymptote at x = – 2, as shown in the
figure. From the graph, we can see that

FREE BOOKS FOR JEE & NEET =>(@iitjeeadv)


1.58  
Differential Calculus for JEE Main and Advanced

x 2 + 2x − 8 x 2  2x  8
lim = − ∞ and lim 
x → 2− x2 − 4 x 2

x2  4

Note: From the definition it follows that the graph of a



function of x can have atmost two horizontal asymptotes – one
Note that x = 2 is not a vertical asymptote.
to the right and one to the left.
We can see in the figure that the function f(x) = 1/(x2 + 1)
Horizontal Asymptotes
approaches the same horizontal asymptote to the right and to
Bearing in mind the geometrical interpretation of the limit the left. This is always true of rational functions. Functions that
of a function, as x → ∞, we can say that if are not rational, however, may approach different horizontal
lim f(x) = , the curve y = f(x) has the straight line asymptotes to the right and to the left.
x  ps
y =  as its horizontal asymptote for x → ∞.
el
The line y =  is called a horizontal asymptote of the curve
eh

y = f(x) if either
je

lim f(x) = L or lim f(x) = L



x x
iit
@

In a number of cases the function f(x) behaves in a different


manner as x → ∞ and as x → –∞ .

9x 2  1
For instance, for the function f(x) = defined for
x 1
all x ≠ 1, we have

9x 2  1 9x 2  1
lim  3 , lim 3
x  x 1 x  x 1

Therefore, when investigating for horizontal asymptotes of
functions, we usually consider both lim f ( x ) and lim f ( x ) .
x  x 

Example 4: Determine the limits




3x  2 3x − 2
(a) lim (b) lim
x  x →−∞


2x  1
2
2x 2 + 1
For example, the line y = 2 is a horizontal asymptote to
Solution
2x  1

the right in the curve f(x) = since lim f(x) = 2. (a) For x > 0, we can write x = x 2 . Thus, dividing both
x 1

x
the numerator and the denominator by x produces
By taking the limit as x → – ∞, we see that y = 2 is also a
horizontal asymptote to the left. The graph of the function is 3x  2
lim
shown in the figure. x 
2x 2  1

FREE BOOKS FOR JEE & NEET =>(@iitjeeadv)
Limits 1.59

2
3
x 30 3
= lim =  .
20

x 1 2
2 2
x
2
(b) For x < 0, you can write x = – x . Thus,

dividing both the numerator and the denominator by x

produces Fig. 1
3x − 2 2 2
3− 3−
3x − 2 x x x
= = =
2x 2 + 1 2x 2 + 1 2x 2 + 1 − 2 + 1

− x2 x2 x2

and we can take the limit as follows.

2
3
3x − 2 x Fig. 2
lim
x →−∞
= lim
2x + 1
2 x→ −∞

1
 2
x2
3−0 3
= = .
− 2+0 − 2

ps
el
eh
je
iit

Fig. 3
@

Use of Asymptotes in Graphing


Some examples of graphing rational functions will show the
usefulness of asymptotes.
1
Example 5: Draw the graph of f(x) = .
x ( x − 1)


Fig. 4
Solution: Note that when x = 0 or when x =1 the denom-


inator x(x–1) is 0. Thus 0 and 1 are not in the domain of the
function. More important, when x is near 0 or near 1 the quo-
tient 1/[x(x–1)] has large absolute value. Thus the lines x = 0 and
x = 1 are vertical asymptotes for the graph. To decide how the
graph approaches these asymptotes, it is necessary to exam-
ine the sign of f(x) for x near 0 and for x near 1. Consider x
near 0. If x is a very small positive number, then
1 1
f (x) 
x x 1


is the product of 1/x, which is a large positive number, and 1/
Fig. 5
(x–1), which is near 1/(–1) = –1. So, for x small and positive, f(x)
is negative and of large absolute value; that is, limx→0+ f(x) = –∞. If x is near 0, but negative, then 1/x is negative and of large
This fact is recorded in Fig. 1 absolute value. Again, 1/(x–1) is near –1. Thus,

FREE BOOKS FOR JEE & NEET =>(@iitjeeadv)


1.60  
Differential Calculus for JEE Main and Advanced

1 1
lim f ( x )  lim 
x 0 x 0 x x 1

This fact is recorded in Fig.2.
Next, how does f(x) behave near x = 1? 4x 2
y=
Consider first x near 1 but larger than 1. x2 +1
1 1
Then f ( x ) = is a large positive number, since 1/x is
x x −1 The notion of an asymptote can be extended to include curves
near 1 and x – 1 is a small positive number. Thus, as well as lines. Specifically, we say that f(x) is asymptotic
to g(x) as x → ∞ if
lim f ( x )  
x 1 lim [f(x) – g(x)] = 0


x

Similarly, lim f ( x )   and that f(x) is asymptotic to g(x) as x → –∞ if

x 1
lim [f(x) – g(x)] = 0


These two facts are recorded in Fig. 3. Piecing together these x
three figures suggests that the graph of f for x in or near the Informally stated, if f(x) is asymptotic to g(x) as x → ∞, then
interval (0,1) looks something like Fig. 4. How high the curve the graph of y = f(x) gets closer and closer to the graph of
goes for x in (0,1) is the type of question answered in a later y = g(x) as x → ∞, and if f(x) is asymptotic to g(x) as x →
chapter with the aid of derivative. –∞, then the graph of y = f(x) gets closer and closer to the
How does f(x) behave when |x| is large? graph of y = g(x) as x → –∞.
For example, if
ps
1 1
Since lim  0 and lim 0 f(x) = x2 +
2
and g(x) = x2
x  x ( x  1) x  x ( x  1)
x −1
el

then g(x) is asymptotic to g(x) as x → ∞ and as x → –∞ since
eh

the x axis is a horizontal asymptote (both for x positive and


for x negative). In both cases the graph approaches the x axis 1
lim [f(x) – g(x)] = lim =0
je

from above, not from below, since the function is positive when x x − 1

x
|x| is large. The graph of f must look something like Fig. 5.
iit

1
lim [f(x) – g(x)] = lim =0
Example 6: Discuss and sketch the graph of the x  x  1

@

x


4x 2 This asymptotic behaviour is illustrated in the following figure,
equation y = 2 .
x +1 which also shows the vertical asymptote of f(x) at x = 1.
Solution: The graph is symmetric w.r.t. the y axis, since


x occurs with even powers only.
x2
Since, 0 ≤ < 1, 0 ≤ y < 4 for every number x.
x2 +1
Thus the graph lies between the lines y = 0 and y = 4.
4x 2 4
Since, lim = lim = 4,
x  x 2
 1 x  1  1 / x2
the line y = 4 is an asymptote of the graph of the function. The
graph may be drawn as shown in the figure.

Concept Problems J

1. Determine the vertical and horizontal asymptotes of the
x2 −1 x2 1

graph of the given functions : (i) f(x) = (ii) f(x) =
x2 − 4 x2  4




2x 2 2x 2 3. Use asymptotes to sketch the graphs:

(i) f(x) = (ii) f(x) =
( x + 2) 2
x2 + 9 1 1 x ( x  1)




(i) y = (ii) y = (iii) y =
x −x
2
x −x
4 2
x2 1





2. Sketch the graph of each of the following functions,

indicating the vertical and horizontal asymptotes. 4. Sketch the graph of (i) y = xe–x (ii) y = x ln x.

FREE BOOKS FOR JEE & NEET =>(@iitjeeadv)
Limits 1.61
3(n  1)
1.13 Limit of a sequence Example 3: Evaluate lim ,n∈N
n  (n  1)3 3n




Here we consider limit of a sequence f(n) as n tends to ∞. (3n  1)(3n  2)(3n  3)
Solution: Limit = lim
We say that the sequence approaches the limit λ if for all (n  1)3



n 
sufficiently large integral values of n the corresponding values
of f(n) become arbitrarily close to the number λ.  1  2  3
3  n 3  n 3  n 
If f(n) → ∞ we may say that f(n) diverges to ∞ . If f(n) = lim     = 27
does not tend to a limit or to ∞ , then it oscillates finitely n  3


 1
or infinitely: in this case we say that the sequence oscillates 1  n 
 
finitely or infinitely.
The only difference between the ‘tending of n to ∞’ and Example 4: Find



‘tending of x to ∞’, is that x assumes all values as it tends to 2n n 1 n n (1) n
∞ , i.e. that the point x coincides in turn with every point of lim cos  
n  2n 2  1 2n  1 1  2n n 2  1
the number line to the right of its initial position, whereas n


tended to ∞ by a series of jumps. We can express this distinction  2 1  11/ n 
by saying that x tends continuously to ∞. Solution: lim   n cos  2  1 / n 
n   2  1 / n 2   



There is a very close correspondence between functions of x and
functions of n. Every function of n may be regarded as a selection 1 (1) n 1
from the values of a function of x.    
(1 / n  2) (1  1 / n ) n 
2
We can easily see that [x] → ∞ , x–[x] oscillates finitely, and
     
   
[x] + √{x–[x]} → ∞ as x → ∞. ps 1  2   11/ n 
One simple remark may be inserted here. The function f(x) = x = lim  2 
cos  
n  n  2  1 / n   2 1 / n 


– [x] oscillates between 0 and 1, as is obvious from its graph. It
el
is equal to zero whenever x is an integer, so that the function f(n) 1 (1) n 
  
eh

derived from it, is always zero and so it tends to the limit zero. (1 / n  2) (1  1 / n 2 ) 
The same is true if f(x) = sin xπ.
je

2 1 1 1 
f(n) = sin nπ = 0. = 0    cos    = 0.
iit

 2 2 1 

2
It is evident that f(x) → l or f(x) → –∞ involves the corresponding
@

property for f(n), but that the converse in often untrue. Example 5: Evaluate


Note: If f(n) is a sequence such that lim f (n )  a , then 1.n + 2 .(n − 1) + 3(n − 2)..... + n .1
n  lim


n →∞ n3
we have lim f (n  1)  a .

n 
n

n 1  n
2  r(n  r  1)
Example 1: Find lim r 1
Solution: Limit = lim


n  4 n3


n3  1  n n 

n n
n  11 / n2  1 / n  (n  1) r   r 2
 
r 1 r 1
 
Solution: lim = lim
n 



n 3/ 4 4
1  1 / n3  4 1 / n n3

n 

(n  1)n (n  1) n (n  1)(2n  1)
1/ 4
 11/ n 2
 1/ n  . = lim
2

6

 11/ n 1/ n 
= lim n n  n3

n 
4


4 3

1 1 1
=   .

n! 2 3 6
Example 2: Find lim
n  (n  1)!  n !


Example 6: If the value of


n!  3  3  3  3
Solution: lim lim n 3  1   1   1   .... 1  
n  (n  1)!  n !


n   7  8  9  n

n! 1 is k then find the sum of digits of k.
(n  1)! n 1 n 3
= lim  lim 0 . Solution: lim n3
4 5 6 7
    ......
n  n! n  1

1 1


n 7 8 9 10 n
(n  1)! n 1

FREE BOOKS FOR JEE & NEET =>(@iitjeeadv)


1.62 Differential Calculus for JEE Main and Advanced
2n 1

n

n 1 n 1 0 1
4  5  6n 3 Solution: Limit = lim  1.



n  n
= nlim = 120 = k. 2 1
 (n  2)(n  1)n 1


n 1
Thus the sum of digits of k = 3.
Example 10: Find

 1 



Example 7: Find the value of  ln 1  2  . lim  1  1   1  1   1  1   1 
n      ....  1  2 
n 2  n 


 22   32   42   n 



 1  Solution: We have limit
Solution: We have  ln 1  n 2 



n 1 n n 1


n
n2 1 n
n 2
= lim  n2
= lim  n · n


  n2 1  
  n 1   n 1   n  n  2 n  n 2 n 2
=  ln  2  =
   ln   n   n    1 2 3 n 1   3 4 5 n 1 

n 2  n  n 2  
= lim  · · ...   2 · 3 · 4 ..... n 
n   2 3 4 n  



  n 1   n 1  
=   ln  n    ln 
 n 
 1 n +1 1
n 2 


= lim · = .


n  n 2 2

  n 1   n 
=   ln    ln   Example 11: Calculate lim sin( n 2  1) .
 n 1    
n 


n 2   n 

Solution: We have


 1 2  2 3
=  ln  ln    ln  ln  + .....
ps  1   1 1
1/ 2

 2 3  3 4 n 2  1  n 1  2   n 1  . 2  ........ 

   
el
n 2 n
1  n 
eh

= ln – lim ln   1
n   n 1  = n + 2n + .........

2

je

1   
= ln – ln 1 = – ln 2 .
Hence sin(  n  1)  sin  n   ....... 
2
iit

2
 2n 
@

xn
Example 8: Evaluate lim .   
n  n !  ....... 


= (–1)n sin 
 2n 

Solution: Since x is fixed number, there will be a pos-


itive integer N such that |x| < N.    
|x| The sequence {(–1)n} is bounded and sin   .......   is
Let
N
= q ( 0 < q < 1). Now, we have   2n 

infinitesimally small, and therefore the product of these two
xn x x x x x x x sequences is an infinitely small sequence. Thus,
 . . ....... . . ...
n! 1 2 3 N 1 N N 1 n
lim sin( n 2  1)  0
n 

.
|x| |x| |x| |x| n · 3n 1
< . . ........ . q. q.....q 
N −1 Example 12: If lim

1 2 3 n 1
n  n ( x  2)  n · 3
n
3
n


3
|x|
[since ≤ q for n ≥ N] where n ∈ N, then find the number of integral values of 'x'.

N 1 N Solution: We have
|x|
q n  N 1


=
( N  1) ! n · 3n 1

lim  .
n  n ( x  2) n  n · 3n 1  3n 3
| x |N−1
is a fixed quantity independent of n, while

where Dividing both numerator and denominator by n. 3n,
( N − 1) !
1 1
qn–N+1 approaches 0 as n approaches ∞. lim n

n 
xn  x2 1 3
Hence, we have lim = 0.  3  3 n
n  n !  

Clearly for the limit to exist, we must have
2n 1  n n
Example 9: Evaluate lim x−2
n  1  2n ⇒


n  –1< <1 –1<x<5



3

FREE BOOKS FOR JEE & NEET =>(@iitjeeadv)


Limits 1.63

∴ The possible integral values of 'x' are 0, 1, 2, 3, 4 5. If f(n) and g(n) both oscillate finitely as n tends to infinity,




Hence there are 5 integral values of 'x'. then f(n) + g(n) must tend to a limit or oscillate finitely
as n tends to infinity.
Behaviour of Two Functions as n Tends to Infinity
For example


Results (i) f(n) = (–1)n, g(n) = (–1)n+1,




1. If f(n) → ∞ as n tends to infinity and g(n) ≥ f(n) for all (ii) f(n) = g(n) = (–1)n.



values of n, then g(n) → ∞ as n tends to infinity. 6. If f(n) oscillates finitely, and g(n) infinitely as n tends to


2. If f(n) → 0, as n tends to infinity and |g(n) | ≤ |f(n) | for infinity, then f(n) + g(n) oscillates infinitely as n tends to

all values of n, then g(n) → 0 as n tends to infinity. infinity.
3. If lim |f(n)| = 0, then lim f(n) = 0. 7. If both f(n) and g(n) oscillate infinitely as n tends to infinity,



n n
then f(n) + g(n) may tend to a limit, or to ∞ , or to –∞ , or
4. If f(n) tends to a limit or oscillates finitely as n tends to oscillate either finitely or infinitely as n tends to infinity.

infinity , and |g(n)| ≤ |f(n)| when n ≥ N, then g(n) tends Theorem If lim f(n) = a and lim g(n) = b, then
to a limit or oscillates finitely as n tends to infinity.


n n

5. If f(n) tends to ∞ or to –∞ or oscillates infinitely as n tends lim f(n)g(n) = ab.



n
to infinity , and |g(n)| ≥ |f(n)| when n ≥ N, then g(n) tends
to ∞ or to –∞ or oscillates infinitely as n tends to infinity. Theorem If lim f(n) = a and lim g(n) = b and b is not

n n
Behaviour of Sum of Two Functions as n Tends f (n ) a
zero, then lim = .
to Infinity ps n g ( n ) b
Theorem If f(n) and g(n) tends to limits a, b, then f(n) + Results

g(n) tends to the limit a + b.
el
1. If f(n) → ∞ and g(n) oscillates finitely as n tends to
The argument is roughly like this : ‘when n is large, f(n) is

infinity, then f(n)g(n) must tend to ∞ and –∞ or oscillate
eh

nearly equal to a and g(n) to b, and therefore their sum is


infinitely as n tends to infinity.
nearly equal to a + b’.
je

Examples of these three possibilities may be obtained by



Results
iit

taking f(n) to be n and g(n) to be one of the three functions


1. If f(n) tends to a limit, but g(n) tends to ∞ or to – ∞ or 2 + (–1)n, –2–(–1)n, (–1)n
@

oscillates finitely or infinitely as n tends to infinity, then 2. If f(n) and g(n) oscillate finitely as n tends to infinity,

f(n) + g(n) behaves like g(n) as n tends to infinity. then f(n)g(n) must tend to a limit (which may be zero) or
2. If f(n) → ∞ and g(n) → ∞ or oscillates finitely as n tends oscillate finitely as n tends to infinity.

to infinity, then f(n) + g(n) → ∞ as n tends to infinity. In this For example,

statement we may obviously change ∞ into –∞ throughout. (i) f(n) = g(n) = (–1)n,



3. If f(n) → ∞ and g(n) → –∞ as n tends to infinity, then f(n) (ii) f(n) = 1 + (–1)n, g(n) = 1 – (–1)n,



+ g(n) may tend either to a limit or to ∞ or to –∞ or may 1 1
oscillate either finitely or infinitely as n tends to infinity. (iii) f(n) = cos nπ , g(n) = sin nπ


3 3
These five possibilities are illustrated in order by 3. A particular case of the theorem which is important is


(i) f(n) = n, g(n) = –n, that in which g(n) is constant. The theorem then asserts


(ii) f(n) = n2, g(n) = –n,


(iii) f(n) = n, g(n) = –n2, simply that lim kf(n) = ka if lim f(n) = a.
n n


(iv) f(n) = n + (–1)n , g(n) = –n, 4. If f(n) → ∞ as n tends to infinity then kf(n) → ∞ or kf(n)


(v) f(n) = n2 + (–1)nn , g(n) = –n2.

→ –∞ , according as k is positive or negative, unless k = 0,


4. If f(n) → ∞ and g(n) oscillates infinitely as n tends when of course kf(n) = 0

to infinity, then f(n) + g(n) may tend to ∞ or oscillate for all values of n and lim kf(n) = 0. And if f(n)
infinitely as n tends to infinity, but cannot tend to a limit,

n

or to –∞, or oscillate finitely. oscillates finitely or infinitely as n tends to infinity, then



For g(n) ={f(n) + g(n)} – f(n); and, if f(n) + g(n) behaved so does kf(n), unless k = 0.

in any of the three last ways, it would follow, from the Note:
previous results, that g(n) → –∞ , which is not the case.


1. If f(n) is positive and f(n + 1) ≥ K.f(n), where K > 1, for
As examples of the two cases which are possible, consider

all values of n, then f(n) → ∞ as n tends to infinity.
(i) f(n) = n2, g(n) = (–1)n n, (ii) f(n) = n, g(n)= (–1)nn2. Here For f(n) ≥ K.f(n–1) ≥ K2f(n–2) ≥ ... ≥ Kn–1f(1) from which

again the signs of ∞ and –∞ may be permuted throughout.

the conclusion follows at once, since Kn → ∞

FREE BOOKS FOR JEE & NEET =>(@iitjeeadv)


1.64  
Differential Calculus for JEE Main and Advanced

Note that the same result is true if the conditions are f(n) oscillates infinitely if x ≤ –1


satisfied only when n ≥ N. and f(n) → 0 if –1 < x < 0.


2. If f(n) is positive and f(n + 1) < Kf(n), where 0 < Example 14: Prove that if x is positive then



K < 1, then lim f (n ) = 0. This result is also true if the n
x → 1 as n → ∞
n 


conditions are satisfied only when n ≥ N. Solution: Suppose, for example, x < 1.



3. If |f(n + 1)| ≤ K |f(n)| when n ≥ N and 0 < K < 1, then Then x , x , 3 x ,..... is a decreasing sequence, and n x > 1

lim f (n ) = 0. for all values of n. Thus n x → l , where l ≥ 1. But if l > 1
n 
f (n 1) we can find values of n, as large s we please, for which n x
4. If f(n) is positive and lim = l > 1, then f(n) → → l or x > ln, and since ln → ∞ as n → ∞, this is impossible.
n 

f (n )
∞ as n tends to infinity. Example 15: Prove that n
n → 1as n → ∞.



f (n +1) Solution:  1)  n n 1 ( n n
For we can determine N so that >K>1



if (n+1)n < nn+1 or (1 + n–1)n < n, which is certainly satisfied

f (n )
when n ≥ N ; we may, for example take K half way if n ≥ 3 . Thus n n decreases as n increases from 3 onwards

between 1 and l and apply 1. and since it is always greater than unity, it tends to a limit
f (n 1) which is greater than or equal to unity. But if n n → l , where
5. If lim = l, –1 < l < 1,
l > 1, then n > ln , which is certainly untrue for sufficiently

n  f ( n )

then lim f (n ) = 0. large values of n, since ln /n → ∞ with n.


n 

ps
Example 16: The sequence of real numbers x1, x2,


Example 13: Determine lim nrxn, where r is any x3, ..... satisfies lim (x2n + x2n+1) = 315 and lim (x2n +


el
n n  n 
positive integer.
Evaluate lim (x2n/x2n+1).
eh

x2n–1) = 2003.
n 

Solution: Let f(x) = nrxn.


Solution: We have lim (x2n+1 – x2n–1)
je

If x = 0 then f(n) = 0 for all values of n, and lim nrxn = 0. In




n n 
all other cases
iit

= lim [(x2n + x2n+1) – (x2n + x2n–1)]


f (n  1)  n 1  n 

 lim  x  x
@

lim
n  f (n ) n   n  = 315 – 2003 = – 1688 ; it follows that x2n+1 → ∞ as

n → ∞.

First suppose x is positive. x x  x 2 n 1
Then f(n) → ∞ if x > 1 and f(n) → 0 if x < 1. Then lim 2 n  lim 2 n  1  1
n  x 2 n 1 n 

x 2 n 1
If x = 1, then f(n) = nr → ∞.
since x2n + x2n+1 → 315 while x2n+1 → –∞.

Secondly, suppose x is negative. 1. Evaluate the following limits :
Then |f(n)| = nr |x|n tends to ∞ if |x| ≥ 1 and to 0 if |x| < 1.

Concept Problems K

2n  5.6n 12 + 22 + 32 + .... + n 2
(i) lim (ii) lim
3n  6n n3




n  n

(n  1) 4  (n  1) 4 5n 1  3n  22 n
(ii) lim (iii) lim
3n 3  5 5n  2n  3n  3


n 


n
1
n 3  3n 2  n  3
(iii) lim 2n
n2  n  6 (iv) lim


n n  1


2n 1
( n 2  1  n )2
(iv) lim 
3. Using the set x n  (2n  1) , show that the sequence

n  3
n6  1

2
2. Evaluate the following limits: {sin xn} diverges. Hence prove that the function sin x

 1 does not have a limit as x → ∞.
(i) lim n 2 1  cos  4. Prove that the following sequences tend to zero as n
 n


n

tends to infinity:

FREE BOOKS FOR JEE & NEET =>(@iitjeeadv)


Limits 1.65

n 3n 5. Prove that n (n !)   as n → ∞.
(a) x n = , (c) x n  n 51


n
, (b) xn =

2 n!

Practice Problems I


6. Evaluate the following limits:  1 1 1 

14  24  34  .....  n 4 (iii) lim    ......  
n   1.2.3 2.3.4 n (n  1)(n  2) 


(i) lim
n  n5



9. For x > 0 and ≠ 1 and n ∈ N, evaluate,
(n  1)3  (n  1)3


(ii) lim  1 1
n  ( n  1) 2  ( n  1) 2 lim    ......


n  log x 2 . log x 4 log x 4 . log x 8
4
n5  2  3 n 2  1


(iii) lim 1 
 
. log x 2 
n  5


n  2  n 1
4 3
log x 2 n 1 n

(iv) lim (n  2)! (n  1)!



1 9 
n  ( n  2)! ( n  1)! f ( n )   , n ∈ N and f(n) > 0 for


10. If f(n + 1) =


2 f (n ) 
n 3  2n 2  1  3 n 4  1 all n ∈N, then find lim f (n )
7. Find lim n 
n  4

n 6  6n 5  2  5 n 7  3n 3  1 ps
8. Evaluate the following limits: 11. Evaluate the following limits:

 1 n 

1 1 1 2
1    .....  n (i) lim   ...... 
el
n 1  n 2
1 n 2 
1 n2 


lim 2 4 2
eh

(i)
n  1 1 1  1  2  3  4  .........  2n 



1    .....  n
3 9 (ii) lim  

je

3 n  n2 1 


 1 1 1 
iit

(ii) lim    .......    x x x 


n   1.33.5 (2n  1)(2n  1)  lim  cos cos .......cos n 


(iii)
 2 
@

2 4


n

1.14 Limits of Forms (0 x ∞) and (∞ – ∞) 1


Example 1: Evaluate lim x tan

x  x


(0 x ∞) Form Solution: This is an ∞. 0 indeterminate form, but we


Consider the limit lim f(x).g(x) where f(x) → 0 and convert it into a 0/0 form by using the substitution u = 1/x
x →a
and noting that u → 0 as x → ∞
g(x) → ∞ as x → a. Here a may be finite or infinite.
This is called (0 x ∞) form. 1 1 tan u
lim x tan  lim tan u  lim = 1.
One way to handle this indeterminate form is to write the limit x  x u 0 u u 0 u

f (x) 0
lim f(x).g(x) as lim which enters into   form or Example 2: Evaluate lim x (nx  n ( x  2))
0 x 


x →a x →a 1
g( x ) x 1
g( x ) Solution: Limit = lim x.n Put x =
which is in    form. x  x2



lim t
x →a 1 
  n
1
f (x)
= lim 1  2 t = lim n (1  2 t ) .2 = – 2.
These can be solved by methods discussed earlier. t 0 t 0

 
t 2t
For example, lim sin2x. cotx is in (0 x ∞) form.
x→0  b 
sin 2 x 0 Example 3: Evaluate lim a x sin  x , a > 1
x  a 


lim s i n 2 x . c o t x = lim  0  which can be
x→0 x 0 tan x  
evaluated using standard limits to get 2.   b 
 sin  x  
1
Solution: Limit = lim   a   .b  b .
1
 1) x   


(e x b
and lim x (e x  1) = lim = 1.  x 
x  x  1/ x  a 

FREE BOOKS FOR JEE & NEET =>(@iitjeeadv)


1.66  
Differential Calculus for JEE Main and Advanced

 x 1    1
3
Example 4: Evaluate lim x  tan 1  .
x   x  2 4   n t 


Limit = l im  2 
 x 1  t  t
Solution: Limit = lim x  tan 1  tan 1 1
x   x2    nt 
3


= l im
 x 1  t  t2



 x  2 1  3
1
= lim x.tan    nt 
 1  x  1  =  lim  2 / 3  = 0.

x 
t   t 


  n 1
Example 9: Evaluate lim n2 ( n a  a ) , a > 0.
 1  1  
n



 tan  
 2x  3     1  1 n 1
= lim x  , lim n2 ( n a 
  2 x  3 
x   1 Solution: Put n = a)

y n



 2x  3 
 
ay  a
y
y 1

= lim
1 1 y→0 y2


= lim  .

x  3 2
2 1  a
y
y 1 1
= lim a  
x y
y→0


Note: lim xnx  0 y2

 
x 0


ay 1  a
y2
ps y 1

nt 1 = lim
Proof: We have lim 0. Put t =
el
y→0

2
t  t y

x
eh

y2 y2

n
1 a y1
1 a y1
1
je

x = lim = lim
⇒ 0 .  y  1
lim y→0 2 y→0 y2

x 0 1 y
y 1

iit

x = ln a.

@

Example 10: Evaluate:


 lim xnx  0


⇒ x 0  

  1  1  1 
lim n 2   1  cos   1  cos   1  cos  .... 
⇒ lim xnx  0 . n 
  n  n  n 
x 0  
  
Example 5: Evaluate lim sin xn (tan x ) 1
x 0
Solution: Put =x




n
Solution: Limit = l i m cos x (tan x . n (tan x ))
x0  


  1  1  1 
= 1 × 0 = 0. lim n 2  1  cos  1  cos  1  cos  .... 

x 2 n ( x  x 2 ) n 
  n  n  n 
Example 6: Evaluate l i m .  



x0 tan x
x x.( x  x 2 ) (1  cos x ) (1  cos x ) (1  cos x ).....
Solution: Limit = l i m . n ( x  x 2 )
x0 tan x ( x  x 2 ) = lim




x 0 x2
= lm
x
.
1
tan x 1  x
x 0

( x  x 2 ) n ( x  x 2 )  (1  cos x ) 2
1

1 1
 _.....
1  cos x

2 2 23
= 1 × 1 × 0 = 0. = lim = lim
x 0 2 x 0 x2

x

Example 7: Evaluate l i m x 3 (nx ) 2


x0
Solution: Limit = l i m x.( xnx )2 = 0 × 0 = 0. (1 − cosx ) . (1 + cos x )
= lim
(1 + cos x )


x0
x →0 x2


Example 8: Evaluate l i m x 2 (nx )3


x0 2
 sin x 
= (1) ⋅
1 2 1 1
1 = xlim   ⋅ xlim = .
Solution: Put x = . As x→ 0, t → ∞. →0  x  → 0 (1 + cosx ) 1+1 2




t

FREE BOOKS FOR JEE & NEET =>(@iitjeeadv)


Limits 1.67

One should transform the difference f1(x)–f2(x) into the product


x 2 (1  a )  x (a  b)  (1  b)
 f (x)  = lim 2
f1(x) 1  2  and first evaluate the indeterminate form ( x  1)


x 
 f1 ( x ) 
Since, limit of the above expression is a finite nonzero number,
f2 (x)   degree of numerator = degree of denominator
 the form   ;
f1 ( x )   ⇒ 1–a=0 ⇒ a=1




∴ Putting a = 1 in the limit we get,


f2 (x)
if lim = 1, then we reduce the expression to the  x (1  b)  (1  b)
x → a f1 ( x ) lim 2
x 1


x 

1
f2 (x) ⇒ –(1 + b) = 2 ⇒ b = –3



f1 ( x )  0 Hence, a = 1 and b = –3 .
form  the form 0  . Example 14: Find a and b if
1  
 (x 



f1 ( x ) lim 2
 x  1)  ax  b  0
x 


Example 11: Evaluate lim 
1

1 
 Solution: We have lim ( x 2  x  1)  ax  b  0
x 
 


x x 1 x 
x 0 


1
Put x = −
0
Solution: Limit = lim 1  x  1

t
0
x 0 x 1  x  


  1 1  a 

 1 x 1   1  x  1 lim   2   1   b   0

  t t  t
t 0

= lim
ps
x 0 1  x  1  x  1

x
 (1  t  t 2 )  a  bt 
el
⇒  
1 x 1 1 lim  0
eh

t 0 t
= lim
x 0 x 1  x .2
= . 
 


2
As t → 0, numerator → 1 + a and denominator → 0
je

Example 12: Find Since limit is finite, numerator must tend to 0.




iit

lim ( x 2  8x  3  x 2  4x  3 ) . ∴ 1 + a = 0 ⇒ a = –1

@

x
Substituting a = –1 in (1), we get


Solution: Here we have an indeterminacy of the form  (1  t  t 2 )  1  bt 
 


∞–∞. Multiply and divide the given expression by lim  0

t 0 t

 

x 2 + 8x + 3 + x 2 + 4x + 3 .
 (1  t  t )  1 
2

lim x 2  8x  3  x 2  4x  3 ⇒ lim  b  0
x t 0  t 

 

 lim ( x  8x  3  x  4x  3 ) ( x  8x  3  x  4x  3 )
2 2 2 2

x
( (1  t  t 2 )  1)( (1  t  t 2 )  1)
x 2  8x  3  x 2  4 x  3 ⇒ lim b
t 0

t ( (1  t  t 2  1)

x 2  8x  3  x 2  4 x  3
 lim  
x   1 t  t2 1 
x 2  8x  3  x 2  4 x  3 ⇒ lim  b
t 0

 t ( (1  t  t )  1) 
 
2

4x
 lim 1
x 
x  8x  3  x 2  4 x  3
2 ⇒ = b.

2 1

4x Hence a = –1 and b = .
 lim =
4
= 2. 2
x 
1  8 / x  3x  1  4 / x  3 / x
2 2
Example 15: Find lim ( x  4 x  x  4 x )
2 2

2
x 


 x2 1 
Example 13: If lim   ax  b   2 , find the Solution: lim ( x 2  4 x  x 2  4 x )

x  x  1  x 




values of a and b.  
 x 1
2   x 2  4x  x 2  4x  x 2  4x  x 2  4x 
 
 ax  b  = lim
Solution: We have lim   x 
x  4x  x  4x
x  x  1
2 2
 


FREE BOOKS FOR JEE & NEET =>(@iitjeeadv)
1.68  
Differential Calculus for JEE Main and Advanced

On considering the degree of the leading term, it follows that


( x 2  4x )  ( x 2  4x ) 1 5
= lim –p – = –2. So p = .
x 
x 2  4x  x 2  4x

3 3
 1 
Example 18: Evaluate lim  cos ec3 x  3 
8x x 0  x 



= xlim

x 2  4x  x 2  4x  1 

Solution: We have lim  cos ec3 x  3  , [form∞-∞]
8 x 0  x 



lim
x 3  sin 3 x
= lim 
= x 
x  4x x 2  4x 2 1 1 
    lim


x 2
x2 x  0  sin 3 x 3  x  0 x 2 sin 3 x
[When a negative number is to be written as square root of  x 3  sin 3 x   x 3 
a number, negative sign is put before the sign of square root, = lim   . 
x0 
 
  sin x  


for example – 4 = – 16 ].  x6
8 8
1im  4. x 3  sin 3 x
x 4 4 2  lim
1  1 x0 x6

x x


( x 2  x sin x  sin 2 x ) ( x  sin x )
= lim .

Example 16: Find xlim ( 3 x3  x 2  3 x3  x 2 ) . x 0 x2 x4



   x  sin x 
sin x   sin x 
2

Solution: xlim = lim  1    .


x 0   x   x    x4 
3 2 1/3 3 2 1/3
  ((x + x ) – (x – x ) ) 

 


(x3  x 2 )  (x3  x 2 )
ps
 x 3
x 5 
= xlim  x  x   ....... 
 ( x 3  x 2 ) 2 / 3  ( x 3  x 2 )1/ 3( x 3  x 2 )1/ 3 ( x 3  x 2 ) 2 / 3
el

3! 5!
= lim 3  

2x 2
eh

x 0 x4
= xlim
  2 1 1 2
 1 x 
 1  1  3   1  1  3  1  1  3   1  1  3 
je

3   ....... 
 x   x   x   x   =∞.

=
lim 
 
3! 5!
iit

  x 0 x
 1 1 
@

2 Example 19: Evaluate lim   .


= . x1  ln x x 1 


3

Example 17: Find p so that  1 1 
Solution: lim   


 ln x x 1 


x1
lim x p ( 3 x  1  3 x  1  2 3 x )

x   1 1
is some nonzero real number. = lim   
  

h0 ln (1 h ) h
1
Solution: Make the substitution t = . Then the limit h  ln(1  h ) h


equals to x = lim lim
h 0 h2 h 0 ln (1  h )

 1 1 1
lim t  p  3  1  3  1  2 3  Put ln (1 + h) = t
t 0  t t 


 t ⇒ 1 + h = et


 

 lim t  p 
1
3 31  t  31  t  2 . et  1  t et  1  t
t 0  = lim 2
= lim ...(1)
t2

t 0 t 0
 et  1  2

We need the degree of the first nonzero term in the Maclaurin's   ·t
 t 
expansion of 31  t  3 1  t  2 . We have e t  1  t
Also,  = lim ...(2)
1 1 t2


t 0
31  t  1  t  t 2  ......... ,

3 9 Adding (1) and (2)
3 1 t  1 
1 1 et  e t  2 e 2 t  2e t  1
t  t 2  ......... 2 = lim = lim
2

3 9 t 0 t t 0 t2

2 2 2
It follows that  t  31  t  2   t  .........  et  1 
⇒ = 1
31
. = lim   = 2
9 

t 0
 t  2

FREE BOOKS FOR JEE & NEET =>(@iitjeeadv)


Limits 1.69

Example 20: Evaluate xlim



(x − n cosh x) = lim (1) n cos n   n 2  n  



n 


x
e e
 
x
where cosh x = . = lim (1) n cos   n  n 2  n 
2 n 


 n 
  ex  e x  = (1) nlim
n
cos  
Solution: xlim  x   n     
n n n
2 



 
2   



 
= xlim
  [ n e − n (e + e ) + n 2 ]
x x −x
 


ex = (1) n lim cos   =0
n   


lim n + n 2 1
= x
ex  e x  1 1 

 n 
1   Example 23: Evaluate
= xlim  
  n  1  e  2 x  + n 2



   

1
 n 4
 
= 0 + n 2 = n 2. lim  4 (r  1)(r  2)(r  3)   n 


n   
  r 1  
Example 21: Find  


Solution: Limit
lim  n 2  n  1   n 2  n  1    

    1 

n 
  (n  1)(n  2)(n  3)(n  4)  4 
where [.] denotes the greatest integer function. = lim  4.   n
n   4 

ps  
Solution: n < n2 + n +1 < n + 1
el
1
Put n =


 n2  n 1  = n t
eh

Hence
  1

je

= lim {(1  t )(1  2 t )(1  3t )(1  4 t )} 4 1


∴  = lim  n2  n 1  n 

iit

t 0 t

n
1
 
@

n 1
 = 1 = lim (1  10 t  .......)  1
4
= lim 
n  


 n  n 1  n 
2 2 t 0 t
10
1  t  .......  1
Example 22: Evaluate lim cos  n  n , when n is
2
  =
lim 4

n  t 0


an integer. t

 
10 5
Solution:  = lim cos  n  n
2 = = .

n  4 2


Concept Problems L

1. Evaluate the following limits: lim x.ln(x + x2)
(i)


x0



7
(i) lim x1/10 . sin (ii) lim sin x .ln (tan x )


x →0 x x →0


x (iii) xlim ln x ln( x  1)
(ii) lim (a  x ) sec 1


x a
  
2a
3. Evaluate the following limits:
 ax x 

(iii) lim  sin
2a 
tan
x a  lim  1  12 


2 (i) x 2  x2 
x3  8 


x x 
(iv) lim 3 sin(3 )
x 


(ii) xlim ( 3 x 1  3 x )
2. Evaluate the following limits: 



FREE BOOKS FOR JEE & NEET =>(@iitjeeadv)
1.70  
Differential Calculus for JEE Main and Advanced

which clearly behaves like 3x + 2 for large x. Thus the


(iii) lim ( 4 x 2  9  2 x )


x  limit in question is 2, not 0.” Settle the argument.


(iv) l im ( x  1  x ) x 7. Evaluate the following limits:


x 


lim n (
(i) n
Evaluate the following limits: n 2 + 1 – n)



4.

(i) lim (cosec x  cot x )  1  2  3  ......  n n 
(ii) lim   
x0
n2



n   2



(ii) lim (cosec x  cot x )
x   1  3  5  7  ...  (2n  1) 2n  1 


(iii) lim   
 n 1



 n   2 
(iii) lim (sin x  cos x ) tan   x
x  / 4 4 

(iv) lim
3
n3  n  3 n3  1



n 
lim 1 1
(iv) x→0 – 8. Evaluate the following limits:

sin x tan x


5. Evaluate the following limits: (i) lim ( x − ln( x 2 + 1))
x →∞




(i) lim ( 3x 2  1  2 x )
(ii) lim x [( x  1)  ( x  1) ]
1/ 3 2/3 2/3


x 
x 


(ii) lim ( x  2 x  5  x )
3 3 2  e x  e x 1
x  (iii) lim   x tan 


 x
x 
 e e
x  x

(iii) lim ( x  3x  17  x )
5 5 4
Evaluate the following limits:
x  9.
ps



lim t ( t a 1) a 0
el
(i)
(iv) lim ( x 2  ax  b  x 2  cx  d ) t 


x 


eh

6. Two citizens are arguing about (ii) lim x n sin a , n  0





x  x
je

 3x 2  2 x 
lim   3x   a 
iit

  2 x tan  x 
 x5

x 
 lim 2 
The first claims, “For large x, 3x is small in comparison (iii) x
@

1


to 3x2, and 5 is small in comparison to x. So (3x2 + 3x)/ x sin  
x
(x + 5) behaves like 3x2/x = 3x. Hence the limit in question 1
is 0.” His companion replies, “ Nonsense. After all, (iv) lim x −2 (1.001) − x
x0


3x 2  2 x 3x  2
 ,
n m
10. Show that lim x (nx ) = 0 , m, n ∈ N
x5 1  (5 / x ) x 0


Practice Problems J

11. Evaluate the following limits : 12. Evaluate the following limits :


 x3 x2   1 2 
(i) lim  2   (i) lim  2  4 
x 1   1 
 3x  4 3x  2  



x


 x 1 x

(ii) lim  x 2  2x  1  x 2  7 x  3  (ii) lim  3


x 3  3x 2  x 2  2 x 


x 
x 

 3x (2 x  1)(3x  x  2) 
2 2
(iii) lim    
   

x   2 x  4x 2  


 1 (iii) lim  
x
x 0  4 x  1) 


 2 x (e
 1 1 
 2
(iv) xlim 
 2  x ( x  2) 2
  

 lim
(iv) x  3 ( x  a ) ( x  b ) ( x  c)  x 


x 3 x 2

FREE BOOKS FOR JEE & NEET =>(@iitjeeadv)
Limits 1.71

lim ( 2
x  x  1 + ax – b) = 0, then find the constants then lim  f ( x ) 
g (x)
13. If x  AB .

xa
a & b.
Also lim  f ( x ) 
g (x) lim g ( x ) n ( f ( x ))
14. Evaluate e x a

xa

lim (n6 + 6n5 + 12n4 + 1)1/3 – (n4 + 4n3 + 6n + 1)1/2 Consider the calculation of the limit, as x → a, of the

n
exponential power function [f(x)]g(x), where the functions
 x2 1 
15. If xlim   ax  b   0 find a and b. f(x) and g(x) are defined in a neighbourhood of the point a,
  x  1 
 

and f(x) > 0.
16. For a certain value of B, the limit lim ( x  5x  3)  x
4 3 B
x  The following cases are possible here :

is finite and nonzero. Find B and then compute the limit. 1. If lim f(x) = b > 0, lim g ( x )  c , then lim f ( x )  bc .
g(x)
x →a x a x a


17. Evaluate the following limits :
If xlim f ( x )  0 , lim g( x )  c  0 (or  ), then

2. a x a


  
lim  2 x tan x 
(i)
x
 cos x  lim f ( x )g ( x ) = 0.



x →a

 
2
(ii) lim ln x  ln x 2  1  x  3. lim f ( x )  0 , lim g ( x )  c  0 (or  )
x a x a
x   


g(x)
      then xlim
→a
f (x) = ∞.
(iii) lim n.cos   .sin  
 4n   4n 


lim f ( x )  0, lim g ( x )  0 , t h e n lim f ( x )g ( x ) i s a n
n
4. x a x a x →a
 
ps

2
(iv) lim n sin  ln cos  indeterminate form of type 00
n 


 n
el
5. If lim f ( x )  , lim g ( x )  c  0 (or ) , then
x a x a
18. Evaluate the following limits:

eh

g(x)
lim x3 lim f ( x )
x →a
= ∞.
je

(i) { x 2  1  x 4  x 2}
x


f ( x )  , lim g ( x )  c  0 (or  ) , then
iit

lim
(ii) x e x ( e x +1 – e x −1 )
6. If xlim
a x a



@

lim x3/2 ( x +1 + x −1 – 2 x )
(iii) x lim f ( x )g ( x ) = 0.
x →a


19. Evaluate the following limits : 7. If xlim f ( x )  , lim g ( x )  0 , then lim f ( x )g ( x ) is an
a x a x →a


(i) lim (sin x  1  sin x ) indeterminate form of type ∞0.
x 



8. If xlim f ( x )  1, lim g ( x )   , then lim f ( x )g ( x ) is an
 x 1 x  a x a x →a

(ii) lim x  tan 1  tan 1
x   x2 x  2  indeterminate form of type 1∞.

 1 1 1 
(iii) xlim x 5 sin     3  Note: If h > 0, then lim (1 – h)n = 0 and
  x  x 6x 

 n


lim (1 + h)n → ∞.
1.15 Limits of Forms 0 and ∞
n

0 0
1 x
 sin 2x 

Example 1: Find lim 
x 0  x 


Calculating the Limits of Exponential Power
Functions Solution: Here,


We first note that  sin 2 x 
lim   2 and xlim
x 
(1 + x) = 1.
x 0  →0

 0, 0  a  1
 1, a 1 1 x
 Hence, lim 
sin 2x 
 21  2 .
lim a x  
x 
 , a 1
x  x 0 

dne a0 x2
 x 1 
If lim f ( x ) = A > 0 and lim g ( x ) = B, a finite quantity Example 2: Find lim  
x   2 x  1 


x→a x→a

FREE BOOKS FOR JEE & NEET =>(@iitjeeadv)


1.72  
Differential Calculus for JEE Main and Advanced

Solution: We have n x
lim n f ( x )  lim


1 =0
1 x  x  x


x 1 x 1
lim  lim  Therefore,
x  2 x  1 x  1 2

2 lim x1/ x  lim .f ( x )  lim en f ( x )  e0  1 .
x x  x  x 


n 2
and lim x 2
  . Example 6: Evaluate lim x 1 nx .
x 



x 
x2 n 2
Therefore, lim  x  1  0 Solution: lim x 1 nx  0
form 
x   2 x  1  x 




n 2
= lim .nx
x  1 nx
Limits of Form 00 e


n 2
Example 3: Evaluate lim ( x  x ) .
2
2 x lim
x 0  = x  1
1



e nx

Solution: xlim (x  x ) 2 x2
= en2 = 2.
0 




= lim x n ( x  x )
2 2
Example 7: Evaluate x lim (2x + 3x)1/x.

 
x  0



e

x
lim
Solution: x (2x + 3x)1/x  0 form
(using lim x n x = 0)


( x  x 2 ) n ( x  x 2 ) 
1 x 
lim  

= x 0
e We choose the largest term in the base and divide the base


x0
= e0 = 1. with it. Here 3x is the largest term.

1
ps 1/ x
Example 4: Evaluate lim (1  x 2 ) n (1 x )   x 
Limit = lim  3x   2   1 


x 1
el
x    3  
1   
eh

1/ x
Solution: lim (1  x 2 ) n (1 x )  x 
= lim 3 1   2  



x 1 
je

x   

lim
1
n (1 x 2 )  3 
=
iit

x 1 n (1 x ) x
as x → ∞,   → 0 since 0 <
e 2

2
 n (1 x ) n (1 x ) 
< 1.
@

3

= lim    3
e
x 1
 n (1 x ) n (1 x ) 

= 3(1 + 0)0 = 3.
 n  

1/ x
= e1 + 0 = e.
Example 8: Evaluate lim
x
 2nx , n ∈ N.

x 


Caution Solution: For n = 1, 2, 3, we have 2 > n . n


1/ x
 x 
lim(sin 2 x )
x x
2

=1 Limit = lim 2   n   1 n = 2n.


 2  

x  n
 


x 0
Note that this is not in 00 indeterminate form. The exponent For n ≥ 4, we have 2n < n .
is exactly 0. 1/ x
 n 
x 
lim ( x 2  x )sin
2
x
=0 2
Hence limit = xlim n    1

x 0 = .
   n  
This is not in 00 indeterminate form. The base is exactly 0.  
The limit lim (sin 2 x  cos 2 x  1) x  x cannot be evaluated
2
Example 9: Find


x 0
 
sin 2 x
since the function is undefined as both base and exponent are lim 1cos ec x  2cos ec x  ...  n cos ec x
2 2 2

x→0
exactly 0. Solution: Let y = cosec2x. As x → 0, y → ∞


Limits of Form ∞ 0
Limit = lim (1y + 2y + ... ny)1/y ( ∞ 0 form)
y
1
1/ y
 1  y  2  y y 
   n 1 
Example 5: Find the lim x x . 1/ y
 lim n y
      ...    1


x  
Solution: The limit leads to the indeterminate form ∞ 0 . y   n   n   n  


1/ y
1
We let f(x) = x and find xlim n f ( x )  1  y  2  y  n 1 
y 
x  lim n       ...    1

y   n 

n  n  

1
n x 
Since n =
f (x) =
nxx 0
= n. 1 = n.
x


FREE BOOKS FOR JEE & NEET =>(@iitjeeadv)
Limits 1.73

Concept Problems M


1 x 2x
1. Evaluate lim  1  x  1 x  1  x 1
x→1   4. Evaluate xlim  

 2x    x 2 


2n 
2. Evaluate lim sin n  
5. Evaluate lim (4n + 5n)1/n



n
 3n  1 


n 
1
x2
3. Evaluate lim 
x2  6. Evaluate xlim (cot x ) ln x
 0 


x   2 x  1 

Practice Problems K

x 1
 x 1  10. Evaluate lim (1  cos ec3 x ) tan x
Evaluate lim  2  x 0


7.
x 1  x  1 

 
11. Evaluate
x

( 2 x 1) lim x ( x )
x 0


lim  x  2 x  1   x 1
2
ps
8. Evaluate x
 2  12. Evaluate  (e 
1/ n


lim  n
 2 x  3x  2  )  ( e ) n
el

n
1/ln n 1
  
eh

9. Evaluate lim  cosh  1 13. Evaluate lim (tan x)tan 2x



n   n  x /2


je

x
where coshx = e  e .
x
iit

14. Evaluate lim (1x + 2x + 3x + ...... 99x)1/x





2 x
@

1.16 Limits of Form 1∞ Let xlim f ( x )  1 and lim g ( x )   then


a xa

1 x
 1 g ( x ) f ( x ) 1
lim  f ( x ) 
lim
l im (1  x ) x  l im  1    e ,
g (x)
e x a

x 0 x   x xa

Let  = lim 1  f ( x )  1
1 g(x)
(1  x ) x  e . Proof:
To prove xlim xa

0
f ( x ) 1
The limit is in the indeterminate form 1∞.  g (x) 
1 = lim 1   f ( x )  1 f ( x ) 1 
We let f(x) = (1 + x ) x and find x xa  

Lim n f ( x ) .
0  
1 1 Put f (x) – 1 = y, as x → a, y → 0
Since n f ( x ) = n (1 + x ) x = n (1 + x ) lim g ( x ) [ f ( x ) 1]
 1 x a

 =  lim 1  y  y 
x ∴
 y 0 

n (1  x )  0 
lim n f ( x )  lim 0 = 1
    g ( x ) f ( x ) 1

x 0 x 0 x lim
= e x a


1
This is an important formula for solving 1∞ form.
lim (1  x ) x  lim f ( x )  lim en f ( x )  e .

x 0 x 0 x 0
Example 1: Calculate lim (1 + sin 4x)cosx.
Generally, it is useful to remember that


x0
x ax Solution: First notice that as x → 0+, we have
 a  1


lim 1 +  = lim (1 + x ) = lim 1 +  1 + sin 4x → 1 and cot x → ∞, so the given limit is
a/x
=e .
a
x →∞  x x →0 x →∞  x indeterminate.

FREE BOOKS FOR JEE & NEET =>(@iitjeeadv)


1.74  
Differential Calculus for JEE Main and Advanced

Let y = (1 + sin 4x)cosx tan


x

Then n y = n[1 + sin 4x)cotx] = cot x n (1 + sin 4x) Example 4: Evaluate xlim  a 2a .
→a  2  



 x
lim n y = lim n (1 + sin 4x )
x0 
x0  x

tan x tan
 a 2a
Solution: xlim
→a  2  
= lim n (1 + sin 4 x ) · sin 4 x · 4 x  x



Put x = a + h


x0 sin 4 x 4 x tan x


So far we have computed the limit of n y, but what we want   h 
tan   
is the limit of y. To find this we use the fact that y = en y lim 
= h→0 h   2 2a 
1  


lim (1 + sin 4x)cosx = lim en y = e4  (a  h ) 
x0 x0

 h 
 cot  
x lim  h   2a 
 x 2  5x  4  = h→0
1  a  h 


Example 2: Evaluate lim  2   
x   x  3x  7 


   cot
h 
. 1
h 
 1
= lim
2 a  a  h
Solution: By means of division of the numerator of h 0



e


the fraction by the denominator, we isolate the integral part:  h  2a
 2a  
= lim   . = e–2/π .
x  5x  4
2
8x  3 h 0
 tan h  a  h

1  . e  2a 
x 2  3x  7 x 2  3x  7

2/ x
 a x  bx  cx 
Thus, when x → ∞ the given function is a power whose base Example 5: Evaluate lim 
x 0  


tends to unity and the exponent, to infinity (an indeterminacy where a, b, c > 0.  3 
x x
ps 2/ x
 x 2  5x  4   8x  3  a b c 
x x x

of the form 1 ). lim  2  = lim 1  2 
Solution: We have lim 

(form 1∞ )
el
x   x  3x  7 
 x  3x  7  x 0 


x 

   3 
eh

x (8x 3)
 a x  bx  cx  2
 x2  3 x  7
 x2  3 x  7
=  1.
 8x  3 
8x 3 lim
x 0  x
je

3

 lim 1  2  e

x   x  3x  7  
iit

2  a x  b x  c x 3 

lim  
 = 3 x 0  x 

e
@

8  3/ x

 x2  3 x  7
 1  3/ x  7 / x 2 2  a x 1 b x 1 c x 1 
 
 8x  3  lim 
8x 3
=
 lim 1  2  3 x 0  x x x 

 e
x   
 x  3x  7   2  a x 1 b x 1 c x 1 
  = lim  lim  lim 
3  x 0 x x 0
x x 0
x 

e

8x  3
Since  0 as x → ∞ and we have = 3(2/3) ln (abc) = eln( abc ) = (abc)2/3.
2/3

x 2  3x  7


x2  3 x  7
Example 6: Find the value of
 8x  3 
8x 3


lim  1  2  e . 1
 x  3x  7      ln x

x 
 lim  tan   ln x   .

8  3/ x x 1  4 
Taking into account that xlim 8,
 1  3 / x  7 / x2 1
x     ln x
 x 2  5x  4  Solution: lim  tan   ln x   (1∞ form)
we get lim  2  e
8
4 


x 1 
x  x  3x  7 
  1

Example 3: lim (tan x) tan 2x


= lim 1  2 tan(ln x )  ln x
 



x
4
x 1  1  tan(ln x ) 
lim (tan x 1) tan 2 x
Solution: Limit = x (using formula) 2 tan(ln x ) 1
e .


4
= x 1 {1 tan(ln x )}
lim ln x

2 tan x e
= lim(tan x 1)
x

1 tan 2 x tan(ln x ) 1

e 4
= 2 lim ln x
.
1 tan(ln x )
e x 1

tan  / 4 1
= 2 = e–1 =
1(1 tan  / 4 )
= e2 . (1) = e2

e e

FREE BOOKS FOR JEE & NEET =>(@iitjeeadv)
Limits 1.75

 a 1 n b 
n
l n ({sin x}·{cos x} + 1)
Example 7: Find the value of lim  f(x) = where {x} denotes fractional

n   {sin x}{cos x}


(a , b > 0) .  a 
part function, prove that

 a 1 n b 
n
     
(ii) f (0–) = f 
 2 
(i) f 
 2 
Solution: lim   (1∞ form) = f (0+)





n  a    



 
n lim
 n b 1  and show that f(x) does not exist.
= lim 1  
x→0
n   a 

 ln(sin h  cos h  1)
Solution: f (0+) = lim
sin h  cos h



lim  n b 1  h0
= e n 
  . n ln(sin h ·cos h  1)

 a  lim
= h0 sin h
1  b1/ n  1 


lim · h cos h
= e n 
  h
a  1 / n 

= lim ln(sin h ·cos h  1)
1h
h 0


1
= log e b = b1/a.
ea =e log e b1/ a
1


= lim (sin h ·cos h  1  1) = 1.
n (1  x  x 2 4
) hh0
Example 8: Evaluate xlim   
→0
(e x
 1)x ln(cos h ·sin h  1)


f 
 2 
= lim = f (0+).
ps

h0 cos h ·sin h
 n (1  x 2  x 4 )  
f (0 ) = lim ln  (1  sin h ) cos h  1 = ln 2
el
lim
Solution: x→0  ex  1 




(1  sin h ) cos h
eh

x2   h0
 x 
⇒ lim f ( x ) does not exist.
je

 
1
= xlim

x→0
→0
l n 1  x 2  x 4 x2
iit

  
As before f 
 2 
is same as f (0–)
(1  x )
@

1
= l n xlim  x4  1
2

→0 e x2  

n
 3 n 
= xlim x +x 2 4
= xlim
Example 11: Evaluate L = lim  3  r 2 


(1 + x2) = 1 n  n
→0 →0  r 1 

x2 n
3
Example 9: Evaluate Solution: We first note that lim
n  3  r2


n


r 1
   3 n (n  1)(2n  1)
ln  tan   ax   = lim
. = 1.
   n  n 3

4 . 6
( b  0)

lim
x0 sin bx Hence the limit L is in 1∞ form
lim  ( n  1)( 2n  1) 
   ∴ L= e   1 n
n 

ln  tan   ax    2


2n
  4  
Solution: lim
= elim (2n  3n  1  2n )
2 2
sin bx


x0 n 
bx

bx 2n
1 3n 1 3
= lim = .
= lim ln  tan    ax   e2
n 
bx e 2n

 4 

x 0   
Example 12: Evaluate



1       
= lim  tan   ax   tan  sec 2  
   4 lim      2  bx 

x0 bx 4 2
x→0 sin  
ax tan ax     2a   2  ax   .

lim
= x . 1  tan   ax  .tan   sec 2 
bx ax  4  4 b  

lim sin 2    
0 2  bx

Solution: L = x→0
Example 10: For a function   2  a x  





FREE BOOKS FOR JEE & NEET =>(@iitjeeadv)
1.76  
Differential Calculus for JEE Main and Advanced

 cos 2  So, the term of least degree in f(x) is of degree 2.


=  lim sec 2
2  bx 2  ax = e −2 ,
e x 0
i.e., f(x) = a2x2 + a3x3 + ......





cos
where  = x→0
lim 2a x  x 2  f (x) 
1/ x

cos  Now, lim 1   = e2


x 0 


2bx


 x2 
sin     
lim 2 2  a x   x2 f (x )  1 x2 f (x )
= x→0 ⇒ x 0 
lim  = e2 ⇒ lim = e2
sin     e  x x
x 0

2
x3

e




2 2  b x 
  ∴ x 2  f (x) = 2
2 2  ax lim − a x 2 − b x lim




lim
= x→0 = x→0 x 0 x3
  2 − ax − bx

2 2  bx
− a2
x 2  a 2 x 2  a 3 x 3  .....
= a. ∴L = e b2
⇒ lim =2
x 0 x3





b
Example 13: Find a polynomial of the least degree, ⇒ a2 = –1, a3 = 2 and a4, a5 ....... are any arbitrary constants.



Since, we want a polynomial of the least degree, we choose
1/ x
 x 2  f (x) 
lim 1  

such that x 0  x2 = e2. f(x) = – x2 + 2x3.

1/ x
 x 2  f (x) 
Solution: lim 1   exists only when it is
x 0 x2


 
ps
x 2  f (x)
in 1∞ form i.e. lim = 0.
el

x 0 x2
eh
je

Concept Problems N
iit

@

1. Show that
(iii) xlim (ax + x)1/x

→0
if x  0


1/ 2
1 x  1
lim  1  x  1 x   2 / 3 if x 1 lim  1  tan x  sin x
 2x  (iv) x→0  

    1  sin x 


1 if x 
4. Evaluate the following limits :
Evaluate the following limits :

2. x4
 x6

(i) lim
 x 
x
 x 1 


x
(i) lim    
x   1  x 


3
x 1 (ii) lim x 4  ln x

 3x  4  3


(ii) lim x 1
 

x   3x  2 
x
(iii) lim 1 
1 
 ( n  0)
x
 1 1
(iii) lim  sin  cos 


x   x
x   xn 


x
1/( e x
1 x )
  (iv) lim(cos x )
tan 2 x
(iv) lim 1  3 cot 2 x x 0


x 2


3. Evaluate the following limits :

(i) lim (1 + sin πx)cot πx


x→1

(ii) lim (2  cos )cos ec 


2

 0


FREE BOOKS FOR JEE & NEET =>(@iitjeeadv)
Limits 1.77

Practice Problems L


5. Evaluate the following limits: 1/ x
lim  x  1  cos x 

n (i)  
lim  cos m x  , m, n ∈ N x→0



(i) x2  x 


x0 1/ln( x 1)
(ii) lim (1  ln(1  x ))
2

x 0



2
x
(ii) lim  x sin 1  1   1


x  x    (iii) xlim  xn  an  x a
→a  n 1 
 


x  ( x  a )na 
(iii) lim cos n x
lim  e   1  x  


n  n (iv) x   
 1  e   e 1  x  



nx
 1x
 21 x  31 x  ..........  n1 x 
(iv) lim  1 
x   9. Evaluate the following limits :


 n 


 e/ n  e / n 
6. Evaluate (i) lim  1
n
n 2 



1/ x n
 1 a x 1    
lim  .  where a > 0, a ≠ 1. lim  n   sin 1 
x   x a  1  (ii) n  

   n  1  n 


7. Evaluate the following limits:
ps
lim {cos(n(n–1) – n(n + 1))} ( n+1)

2
(iii) n
el


1/ x
 tan x   a 
(i) lim  lim tan n  π − 4 + 1 − 1  
x 
eh


x  0 (iv) n    4 n 


je

2
cot x
(ii) lim (cos x ) 2x
x →0 lim 1  l  m 
10. Find λ & µ if x  e2 .


iit

 2 
 x 

x
cos ec x
(iii) lim  1  tan x 
@

11. For A and B positive constants, define



x  0  1  sin x  f(x) = (ex + Ax)B/x.



1/ x 2 (a) Compute L1 = xlim f ( x ) and L = lim f ( x )
 1  →0 2 x 


(iv) lim  tan x 
x 0   (b) What is the largest value of A for which the equation


 x 


L1 = B L2 has a solution ? What are L1 and L2 in this
8. Evaluate the following limits: case ?

1.17 Sandwich Theorem/Squeeze Play

Theorem
Suppose that g(x) ≤ f(x) ≤ h(x) for all x in some open interval
containing a, except possibly at x = a itself. If

lim g(x) = xlim h(c) =  then lim f(x) = .
x →a →a x →a

Note: x2 x2
For example, if 1 – ≤ f(x) ≤ 1 + for all x ≠ 0,

(i) This is true also when one or both of the above inequalities 4 2

are strict. we can find xlim f(x), even if we donot have the
→0
(ii) This theorem is also applicable to one sided limits and

limits at infinity. formula of f(x). We can see in the figure that the graph of f(x)
(iii) The theorem holds even if  is finite or infinite. is squeezed between the graphs of the other two functions.

FREE BOOKS FOR JEE & NEET =>(@iitjeeadv)
1.78  
Differential Calculus for JEE Main and Advanced

1
Let f(x) = x cos3 . As x → 0, f(x) → 0.
x
1
Also |f(x)| = x cos3 approaches zero because
x
1
– |x| ≤ x cos3 ≤ |x| and


x
– |x| and |x| approach 0 as x → 0.
Since xlim


2 lim 2
→0 (1 – (x /4)) = 1 and x →0 (1 + (x /2)) = 1, Example 1: Show that lim x 2 sin
1
 0.
x 0



x
Sandwich Theorem implies that xlim →0
f(x) = 1. Solution: First notice that we cannot use



sin x
We shall now prove the standard limit xlim =1 1 x 2 · lim sin 1
→0
x lim x 2 sin = xlim
→0
x →0 x →0


x x
geometrically, with the help of Sandwich Theorem.
We take a circle of unit radius and suppose that the central because limx → 0 sin (1/x) does not exist.
1
angle x expressed in radians is contained within the limits 0 However, since –1 ≤ sin ≤ 1.
x
and π/2. It will be enough to consider only x > 0, since
sin x is an even function. We have, as illustrated by the figure,
1
x – x2 ≤ x2 sin ≤ x2

Proof: We shall compare the areas of three regions in the x
figure : two triangles and a sector. ps
el
eh
je

We know that
iit

On inspecting the figure we find that, area of ∆OAP <


@

lim x 2  0 and lim( − x 2 ) = 0



area of sector OAP < area of ∆OAT x 0 x →0

sin x x tan x Taking g(x) = – x2, f(x) = x2 sin (1/x), and h(x) = x2 in the
< < ⇒ sin x < x < tan x
Sandwich theorem, we obtain


2 2 2
1
1<
x
<
1 lim x 2 sin  0.
⇒ (0 < x < π/2) x 0

x
sin x cos x lim sin x .



Example 2: Determine the limit x
x


⇒ sin x
cos x < <1 Solution: Since – 1 ≤ sin x ≤ 1, it follows that

x


Since, xlim cos x  1 , and lim 1 = 1 
– 1 ≤ sin x ≤ 1 , x > 0,
0

x0 x x x
Using Sandwich Theorem we have x lim sin x = 1  where x
lim (– 1/x) = 0 and lim (1/x) = 0. Therefore, by
0 x
x sin x
sin y Sandwich Theorem, we can obtain lim
Put x = – y, then lim = 1. = 0.
x x


y 0 y
Hence, lim sin x = 1.
x→0 x
Zero Limit Theorem
A function f approaches the limit 0 as x tends to a if and only
if |f | approaches the limit 0 as x tends to a.
lim |f(x)| = 0 ⇔ lim f(x) = 0.
x →a x →a

Proof: Since –|f(x)| ≤ f(x) ≤ |f(x)|, and –|f(x)| and |f(x)| both
have limit 0 as x approaches a, xlim →a
f(x) = 0 by Sandwich
Theorem.

FREE BOOKS FOR JEE & NEET =>(@iitjeeadv)


Limits 1.79

Definition of e
Example 3: Evaluate l im [ x ] .


x  x  1
n

Solution: We have x – 1 < [x] ≤ x, The number e is defined as the limit lim 1   .
n   n


⇒ 1  1  [ x ]  1 for x > 0. The number e is irrational and therefore cannot be expressed
x x precisely by a finite decimal. Its approximate value is e ≈
 1 2.718
Now lim  1    1. Therefore by Sandwich theorem x
x   x  1
It can be shown that the function y  1   has
l im [x]
=1 .  x
x  x a limit not only when its argument runs through natural numbers
(that is x = n where n = 1, 2, 3, ....) but also when it varies
Example 4: Let a function f(x) be such that | f(x) | ≤ M continuously and approaches – ∞ or ∞. In all the cases the


for any x ≠ 0. Prove that lim x f(x) = 0. limit is the same number e.
x→0
Solution: We have | x f(x) | ≤ | x | . M Proof: Let n ≤ x < n + 1


⇒ – M | x | ≤ x f(x) ≤ M | x | for any x ≠ 0. ⇒ 1+
1
<1+
1
≤1+
1



n +1


lim M | x | = 0 and lim – M | x | = 0,
Since x→0 x n
n 1

x→0 n x
 1   1  1
by Sandwich theorem, x→0
lim x f(x) = 0. ⇒ 1    ≤ 
n  1   x   n
1 1

  

x  7 sin x n 1 −1
Example 5: Evaluate lim using ⇒  1   1 
< 1  x  x
1
x  2 x  13 1  n  1   1 + 
ps


n + 1

Sandwich Theorem.  
el
n
Solution: We have –1 ≤ sin x ≤ 1 for all x.  1  1
≤ 1   1  


eh

⇒ –7 ≤ 7 sinx ≤ 7  n  n


⇒ x – 7 ≤ x + 7 sinx ≤ x + 7 Now as x → ∞, n also tends to ∞ and in such a case both the
je

Dividing throughout by –2x + 13 (a negative quantity for right hand and left hand side of the above inequality tend to e.
iit

large values of x), we get Thus by Sandwich theorem.


@

x 7 x7 x7  1
x
  . lim  1   = e.
2 x  13 2 x  13 2 x  13

x   x

We evaluate the limits of the extreme functions as x tends to
infinity. Since both the limits are equal to –1/2, by Sandwich Example 8: Evaluate



theorem the limit of the desired function is also –1/2.  n n n n 
lim   2   ....  2 
n  n 2  1 n  2 n2  3 n n
x 2 (2  sin 2 x )
Example 6: Evaluate lim
x  10


x 
Solution: Let f (n)


2x 2 x 2 (2  sin 2 x ) 3x 2 n n n n
Solution:   = 2 + 2 + 2 + .... + 2
x  10 x  10 x  10 n +1 n + 2 n + 3 n +n



2x 2 3x 2 Note that f(n) has n terms which are decreasing Suppose h (n)
lim   and lim 
x  x  10 x  x  10

=  n  n  n  ....  n 

x 2 (2  sin 2 x )  n2 1 n2 1 n2 1 n2 1 
Hence, by Sandwich Theorem lim .
x  x  10 n2
h (n) = (obviously f (n) < h (n) ) and g (n)
1  2 x
2
, x Q n2 +1
lim
Example 7: Find x→0 f(x) if f(x) = 
= lim 
 1  x
4
, xQ n n n n 


   ....  
Solution: Consider | x | ≤ 1.  n2  n n2  n n2  n n2  n 

n



x4 ≤ x2 ≤ 2x2.
n2

Now 1 ≤ f(x) ≤ 1 + 2x2 ∀ x. = (obviously g (n) < f (n) )
n2 + n


lim 1 + 2x2 = 1. By Sandwich theorem, lim f(x) = 1
Hence g (n) < f (n) < h (n)

x→1 x→0

FREE BOOKS FOR JEE & NEET =>(@iitjeeadv)


1.80  
Differential Calculus for JEE Main and Advanced

lim g(n) = 1 = lim h (n) Thus, we have


Since n n n
1 1.3.5....(2n  1)  n2 2
Hence using Sandwich Theorem n
lim f (n) = 1.    n 1
2 


2.4.8...2n 2.4.8....2n  
n
Example 9: Let xn = 
1 lim x .
. Calculate n Now, we have
n2  k n


k 1 n
Solution: We have 1  n2  2
lim  0 and lim   n 1 



n
1 n
1 n
1 n  2.4.8.....2 n n  2 
∑ <∑ <∑ Hence, by Sandwich theorem, we have
k =1 n2 + n k =1 n2 + k k =1 n2 1.3.5....(2n  1)
lim 0
n n  2.4.8....2n
Thus < xn < 1


n2 + n [ x ] + [2 x ] + [3x ] + .... + [nx ]
Example 11: Find nlim 



n 1 n2
Since nlim  lim  1, where [ . ] denotes the greatest integer function.
 n 
n2  n 1
1
n Solution: We know that, x – 1 < |x| ≤ x



we have lim x n  1 in accordance with Sandwich theorem. ⇒ 2x – 1 < [2x] ≤ 2x

n  ⇒ 3x – 1 < [3x] ≤ 3x


....................
Caution

....................

⇒ nx – 1 < [nx] ≤ nx
ps

On the other hand, the general term in the expression for xn ∴ (x + 2x + 3x + .... + nx) – n
el
1 < [x] + [2x] + ..... +[nx] ≤ (x + 2x + .... + nx)
is equal to (k = 1, 2,...., n).

eh

n +k
2
n
xn (n +1) x.n (n +1)
1 ⇒ –n<  [r x ] ≤
je

 0 , if follows that

Since lim 2 r 1 2
n 
n k
2
iit

⇒ lim x  1  1  – 1 ≤ lim [ x ] + [2 x ] + .... + [nx ]


  n  
@

2  n  n n n2

1 1 1
lim    ......  
n   

 n 1 n 2 n n lim x  1  1 
2 2 2
 ≤ n  
2  n

1 1
= lim  lim +.....
n 
n 2  1 n  n 2  2 x lim [ x ] + [2 x ] + .... + [nx ] ≤ x

⇒ ≤ n
n2

1 2 2
+ nlim = 0 + 0 + ... + 0 = 0

n2  n lim [ x ] + [2 x ] + .... + [nx ] = x

Thus, n
The latter method has error in the reasoning. When we add n2 2
infinite number of small quantities, the sum may not always Example 12: Evaluate


tend to zero.  [12 x x ]  [22 x x ]  .....  [n 2 x x ] 
Example 10: Using Sandwich Theorem, show that lim  lim 
x 0  n 

n3
 



1.3.5....(2n  1)
lim  0. where [ ] denotes the greatest integer function.

n  2.4.8......2 n

Solution: We can see that Solution: We have 12xx – 1 < [12xx] ≤ 12xx




1.3.5....(2n − 1) 1 22xx – 1 < [22xx] ≤ 22xx

n
> ............
2.4.8.....2n

2.4.8......2

............

n
 n (1  2n  1)  n x – 1 < [n2xx] ≤ n2xx
2 x

1.3.5....(2n  1)  2 n 
 Adding the above inequations,


2.4.8....2n 21 2 .....n x x n 2  n [n 2 x x ] x x n 2
Using AM. – GM. inequality in the numerator.  

n3 n3 n3
1.3.5....(2n  1) n n
⇒  n (n  1)(2n  1) 1  2 x x ] n (n  1)(2n  1)
2.4.8....2 n n ( n 1) xx    xx

3 2 3
2 2 6n n n 6n 3

FREE BOOKS FOR JEE & NEET =>(@iitjeeadv)


Limits 1.81

Now, as n tends to infinity, we have  [n 2 x x ]  1


  lim x
x
xx [n 2 x x ] x x lim  lim
 lim  x 0  n 


 3 x 0 

 n  3
3 n  n 3 3
ln x

Hence, by Sandwich Theorem, we have =
1
lim e 1/ x (since 1/x is much larger than lnx)


[n 2 x x ] x x 3 x 0 

lim 
n  n3 3 = 1 e0 = 1 .

Now, the required limit


3 3

Concept Problems O

1. If 5 − 2x 2 < f(x) < 5 − x2 for x ∈ [–1, 1], find g(x) ≤
1
sin (50π 3 x ) ≤ h(x) for all values of

x2 1
lim (x)
x→0 f x except zero. What can you say about lim sin
x  x2
2. It can be shown that the inequalities
(50π 3
x) ?

x2 x sin x
1   1 hold for all values of x close
2  2 cos x 6. If b – |x – a| ≤ f(x) ≤ b + |x – a| in the neighbourhood of

6

x sin x x = a then find lim f(x).
to zero. What does this tell you about xlim  0 2  2 cos x
? ps x →a

3. If x4 ≤ f(x) ≤ x2 for x in [–1, 1] and x2 ≤ f(x) ≤ x4 for  x if x is rational


 2
Let f ( x )  
el
7.

x < –1 and x > 1, at what point c do you automatically 

 0 if x is irrational
eh

know xlim→ c f(x) ? What can you say about the value of the
limit at these points? Use Sandwich theorem to prove that xlim f (x)  0 .
je

By considering the inequality 0



4.

iit

x2 x2 x4 1 1
1  cos x  1   x 2 cos x 2 sin
@

Show that lim



2! 2! 4! 8. x = lim x

x→0 sin x
lim 1  cos x  0 .
prove that x→0
x→0 sin x

x
lim f(x) if 4 x  1  f ( x )  4 x  3x for all x > 5.
2
5. Find formulae for function g and h such that 9. Find x

g(x) → 0 and h → 0 as x → ∞ and such that x2

x

Practice Problems M

10. Evaluate the following limits: 2 1 − cos x 1
1
– x <

11. Show that < for all x close to
 2 2

2 24 x 2
lim x2  cos 2 x  x 2 cos 
(i) x→0  x  lim 1 − cos x .
x = 0 and hence find x→0



x2
12. Prove that
n cos(e n )

(ii) lim
2x  3 2 x 2  5x  3


n  2n  1 lim  lim
x 4 x 2 18x 10

x  3 x  1/ 2
x
2e x + e − x − 3x
lim
(iii) x where [.] is G.I.F. x
[e x + x 2 ]


sin 2
 lim 2

1 x 0 x2
xsin
(iv) x
lim


x0 sinx 
13. Evaluate the following limits:

FREE BOOKS FOR JEE & NEET =>(@iitjeeadv)
1.82  
Differential Calculus for JEE Main and Advanced

derivatives, provided that the given conditions are


(i) lim x + sin x
x + cos x x f '( x )


satisfied. In other words, suppose that lim = A, then
x sin x ! x →a g '( x )
(ii) lim f (x)
x 1  x 2


lim
x →a g ( x )
= A where A may be a number, or ∞ or – ∞.
(iii) lim x 3  x 2 sin 
It is especially important to verify the conditions regarding

x→0 x
1 the limits of f and g before using L'Hospital's Rule.
(iv) lim x e cos

x L'Hospital's rule is also valid for one sided limits and for limits


x0
at infinity or negative infinity : that is, "x → a" can be replaced
lim [1 x ] + [2 x ] + [3 x ] + .... + [n x ] where
2 2 2 2
14. Evaluate n by any of the symbols x → a+, x → a, x → ∞ or x → – ∞.

n3
[.] denotes the greatest integer function and x ∈ R Special Case Proof
For the special case in which f(a) = g(a) = 0, f' and g' are
15. Let the inequality g(x) < f(x) < 1 – cos x + g(x) hold for
continuous, and g'(a) ≠ 0, it is easy to see why L'Hospital's

g( x ) rule is true. It is more difficult to prove the general version
all x in the neighborhood of x = 0. If lim  1, find
x 0 x of L'Hospital's rule.
f (x)
lim .
x →0 sin x
x b b If f and g are differentiable and f(a) = g(a) = 0 and g'(a)
16. If a and b are positive, then prove that lim    ,
x 0 a  x  a 

b x f ( x ) f '(a )

lim    0 . How do the functions behave when ≠ 0, then lim
ps x a
.
x 0 x  a   g ( x ) g '(a )
x → 0– ? Proof : Using the definition of the derivative we have
el
x3 , x0
 f (a + h ) − f (a )
eh

17. Find lim f(x) where f(x) =  1 lim


 x sin x , x  0 f (a  h )

x→0 f '(a ) hh →0
 = lim
je

=
lim ( x 2  x )  1  where [.] denotes the greatest g '(a ) g (a + h ) − g (a ) h  0 g (a  h )
18. Evaluate x→0 x lim
iit

  h →0

h
integer function.
@

since f(a) = g(a) = 0



( n 1) 2
1 f (x)
19. Use Sandwich theorem to evaluate the limit lim
n 
 . = lim .

x →a g ( x )

k n 2 k
Notice that we have assumed g'(a) ≠ 0.
1.18 L’ Hospital’s Rule

Comprehensive Proof of L’Hospital’s Rule
Suppose f and g are differentiable on an open interval
containing a, except possibly at a itself. Assume that g′(x) ≠ 0 We first present L’Hospital’s Rule as a theorem about one-
for all x in the open interval containing a, except possibly sided limits.
at a. Suppose that L’Hospital’s Rule I. Let f and g be functions which are
lim f(x) = 0 and lim g(x) = 0
x →a x →a
differentiable on a non-empty open interval (a, b) with g′(x)

≠ 0 for every x in (a, b). If , in addition,
or that lim f(x) = ± ∞ and xlim
→a
g(x) = ± ∞ lim f(x) = lim g(x) = 0,
x →a (i) x a  x a 


0
(In other words, we have an indeterminate form of type
0 f ( x ) f (x)
∞ (ii) lim = L, then lim = L.
x a g( x ) x a  g(x )




or )

f (x) f '( x ) Proof:
Then lim = xlim , provided the limit on the right
x →a g ( x ) →a g '( x ) We may assume that f(a) = g(a) = 0. (If this is not the case to
side exists or is ∞ or – ∞ begin with, we simply define, or redefine, the values of f and
g to be zero at a.) Thus we ensure that f and g are continuous
Note: In many texts, the name L'Hospital is spelt as
on [a, x]. Let x be an arbitrary number in (a, b). Then f and g


.
are continuous on [a, x] (recall that differentiability at a point
L'Hospital's rule says that the limit of a quotient of implies continuity) and are differentiable on (a, x). Moreover,
function is equal to the limit of the quotient of their the derivative g′ does not take on the value zero in (a, x).

FREE BOOKS FOR JEE & NEET =>(@iitjeeadv)


Limits 1.83

Moreover, the derivative g′ does not take on the value zero in The hypotheses have been taken as weak as possible. If, as
(a, x). Hence, by the Generalized Mean Value Theorem and frequently happens, the functions f and g are also continuous
the fact that f(a) = g(a) = 0, we obtain at a, then (i) can be replaced by the simpler condition f(a)
f ( y) f ( x )  0 f ( x ) = g(a) = 0.
  ,
g( y) g ( x )  0 g ( x ) x1/ 2  a1/ 2

Example 2: Evaluate lim , where a > 0.
x a x1/ 3  a1/ 3



for some number y in (a, x). As x approaches a from the right,
so also does y, and hence Solution: If f(x) = x1/2 – a1/2 and if g(x) = x1/3 – a1/3, then
f ( y)



f (x) 1 1
lim = lim = L. This completes the proof. the derivatives are given by f′(x) = and g′(x) = ,
x a g ( x )

x a g( y) 

2x1/ 2 3x 2 / 3
Generalized Mean Value Theorem and it is clear that f and g are differentiable (and hence
continuous) and g′ is not zero on an open interval containing
Assume that a < b, and let f and g be functions which are a. Moreover f(a) = g(a) = 0. Hence, by L’Hospital’s Rule,
continuous on the closed interval [a, b] and differentiable on
the open interval (a, b). If g′(x) ≠ 0 for every x in (a, b) then 1
2/3 1/ 6
there exists a real number c in (a, b) such that x1/ 2
a 1/ 2
2 x1/ 2  3a  3a
lim = lim .
f (b)  f (a ) f (c) x a x1/ 3  a1/ 3 x a 1 3a1/ 2 2
 . 2/3
g (b)  g (a ) g(c) 3x

Example 1: Compute lim x  2 . Caution


x 2 x2 
ps
Solution: Let f(x) = x − 2 and g(x) = x − 2 . While applying L'Hospital's rule we are not supposed to


el
Obviously, lim f(x) = x
lim g(x) = 0, and since 
differentiate f(x)/φ(x) as a fraction. The numerator and
2 x 2
denominator must be differentiated separately.
eh

1 1
f′(x) = and g′(x) =
, x a
je

2 x−2 Example 3: Compute lim where a ≠ 0.



2 x
x2  a2


x a
f and g are differentiable, and g′ does not take on the value zero
iit

on any open interval with left endpoint equal to 2. We obtain Solution: The fact that
@



1
f ( x ) x−a xa 1
lim = lim 2 x = = if x ≠ a,
x  2 g( x ) x −a 2( x  a )( x  a )
2 x+a



x 2 1 

2 x2 immediately implies that


x2 0 x−a 1 1
= lim  = 0. lim = . = xlim
x  2 x →a →a
x −a x+a

2 2

x 2 2a
x 2
And it follows by L’Hospital’s Rule that lim = 0. Of course, the same answer is obtained by L’Hospital’s Rule.
x2 x  2
If we let f(x) = x – a and g(x) = x2 – a2, then f(a) = g(a) = 0
It is a simple matter to verify that L’Hospital’s Rule I remains and f′(x) = 1 and g′(x) = 2x. Hence
true if (a, b) is replaced throughout by (b, a), and lim is x a  lim x − a = lim 1 = 1 .
lim x →a x →a
replaced throughout by x a . This fact, significant in itself, x2 − a2

 2x 2a
also implies the following two-sided form of L’Hospital’s Rule. It is important to realize that L’Hospital’s Rule II can be
L’Hospital’s Rule II. Consider an open interval containing the applied only if lim f(x) = xlim
→a
g(x) = 0. For example, if f(x) =
x →a
number a, and let f and g be functions differentiable and with x2 + 3x – 10 and g(x) = 3x, then
g′(x) ≠ 0 at every point of the interval except possibly at a. If
f (x) x 2  3x  10 0
(i) lim f(x) = lim g(x) = 0, lim = lim  = 0,
x →a x →a x →2 g ( x )

x 2 3x 6


but
f ( x )
(ii) lim = L, then f ( x ) 3x  3 7
x a g( x )  .


lim = lim
g( x )
x 2 

x 2 3 3
lim
f (x)
= L. x  tan 2 x
x →a g ( x ) Example 4: Evaluate xlim

 0 x  tan 2 x


FREE BOOKS FOR JEE & NEET =>(@iitjeeadv)
1.84  
Differential Calculus for JEE Main and Advanced

Solution: Since we have the indeterminate form 0/0, we Rule: If the limit of f(x)/ g(x) as x → a takes the form
0/0, differentiate the numerator and denominator separately


apply L'Hospital's rule to give:
w.r.t. x and obtain a new function f'(x)/g'(x). Now as x → a,
x  tan 2 x 1  2 sec 2 2 x 1  2 if it again takes the form 0/0, differentiate the numerator and
lim  lim   3.
x  0 x  tan 2 x x  0 1  2 sec 2 2 x 1 2 denominator again w.r.t. x and repeat the above process, till
indeterminate form persists.
n (1  2h )  2h
Example 5: Evaluate lim The following examples illustrate the repeated use of
h 0 h2



L'Hospital's rule.
n (1  2h )  2h  0
Example 8: Evaluate lim 3x  x  2 .
1/ 3
Solution: lim  form 0 
h 0 h 2  




2 x 1 3x  6x  3
2
2 Solution: Let f(x) = 3x1/3 – x – 2 and g(x) = 3x2 – 6x + 3.



= lim 1  2 h [by L'Hospital's rule] Then f(1) = g(1) = 0, and the derivatives are given by f′(x) =
h 0 x–2/3 – 1 and g′(x) = 6x – 6. However, the value of

2h
2h f ( x ) 2 / 3
1
= lim 1  (1  2h ) = lim  2 . lim
x 1 g( x )
= lim x
x 1 6 x  6




h 0 h h 0 h

1  cos x is not obvious because f′(1) = g′(1) = 0. Taking derivatives


Example 6: Evaluate lim 2
again, we get f′′(x) = – x–5/3 and g′′(x) = 6, and it follows that


x 0 x4
3
1  cos x 0 2
Solution: xlim 0 f ( x )  x 5 / 2
0 x4  
pslim 1



=
x 1 g( x ) lim 3  .

sin x 1
el
x 1 6 9
= lim 0 [by L'Hospital's rule]
x →0 3
  Thus two applications of L’Hospital’s Rule yield



4x
eh

=∞
f ( x ) f ( x )

je

f (x) 1
Note:
I f t h e h y p o t h e s e s o f L’ H o s p i t a l ’s R u l e lim = lim = lim =– .
x →1 g ( x ) x 1 g( x ) x 1 g( x )


iit

9

II are satisfied for the functions f′ and g′, that is for
the derivatives of f and g, respectively, then we can
@

Example 9: Evaluate lim e  e  t .


2t t

f ( x ) f ( x )


t 0 t2
conclude that lim = lim . Solution: Recognizing the indeterminate form 0/0 we
x a g( x ) x a g( x )


apply L'Hospital's rule to yield
This fact suggests the possibility of applying L’Hospital’s Rule
more than once, and in some problems it is necessary to take e2 t  et  t 2e 2 t  e t  1
second or higher derivatives to find the limit. lim  lim

t 0 t2 t 0 2t
sin x  x
Example 7: Evaluate lim 2e 2 t  e t 3
x 0 x3  lim 



t 0 2 2
sin x  x 0
Solution: lim 0 in which we were obliged to use (and justified in using) the
x3 x 0  



quotient of the second derivatives, since the quotient of the
cos x  1  0  first derivatives turned out to be in the indeterminate form 0/0.
= lim  0  [by L'Hospital's rule]
 

x  0 3x 2 In point of logic one should understand that we properly
 sin x 1 applied L'Hospital's rule to the quotient of derivatives first and
= lim  [by L'Hospital's rule] thereupon applied it again to the given quotient function. The
x 0 6 x

6
If f'(a) = f"(a) = ..... = fn–1(a) = 0 fact that we write equalities form left to right should not be
allowed to confuse the order of logical application.

and g'(a) = g"(a) = ..... = gn–1(a) = 0

but fn(a) and gn(a) are not both zero, then by repeated application
Caution
of L'Hospital's rule, we have
Before applying L'Hospital's rule at any stage be sure that
f (x) f n (x) the form is 0/0. Do not go on applying this rule even if the
lim  lim n
x  a g( x ) x  a g (x) form is not 0/0.

FREE BOOKS FOR JEE & NEET =>(@iitjeeadv)
Limits 1.85

ex  e x  x 2  2 then
f (x)
lim= L.
Example 10: Evaluate lim x 



x 0 sin x  x
2 2 g( x )


Solution: We obtain An analogous theorem holds if (a, ∞) is replaced by (–∞, a)


e x  e x  2x e x  e x  2x and if lim is replaced throughout by lim .
lim  lim x x
x  0 sin x cos x  2 x x  0 sin 2 x  2 x

By repeated uses of L'Hospital's Rule, we get Proof:
The result is a corollary of L’Hospital’s Rule I and the Chain
e x  e x  2 e x  e x 1
lim  lim Rule. Let t = , and set
x  0 2 cos 2 x  2 x  0 4 sin 2 x

x
ex  e x 11 2 1 1 1
lim    F(t) = f   = g(x) and G(t) = g   = g(x). Since
x  0 8 cos 2 x 8(1) 8   t


4 t

t approaches 0 from the right if and only if x increases without
Caution bound,
lim lim  1  lim
Criticize the following use of L'Hospital Rule : t0 F(t) = t0 f  t  = x f(x) = 0,
 


 
x3  x 2  x  2
lim lim G(t) = lim g  1  = lim f(x) = 0.
x 2 x 3  3x 2  3x  2 t0 t 
t0
  x

ps

3x 2  2 x  1 1  1 
 lim By the Chain Rule, F′(t) = f′     2  and
t  t 
el
x  2 3x 2  6x  3
1
G′(t) = g′      .
eh

6x  2 6 1
 lim  lim  1  t   t2 

x  2 6x  6 x 2 6
je

Hence
The second equation is an incorrect use of L'Hospital's Rule,

iit

 1  1  1
f    2  f  
F( x ) t
@

since lim (3x – 2x – 1) = 7 and lim (3x2–6x+3) = 3. So,


2
 t  t 
x →2 x →2 lim = lim = lim
t 0 G( x )

t 0 
 1  1  t 0
1
7
the correct limit should be . g     2  g  
3  t  t  t
xe x  log(1  x ) The last limit exists and is equal to L since
Example 11: Evaluate lim
x2 x 0


1
Solution: Since the form is 0/0, we shall apply f  
f x
lim   = lim


L'Hospital's rule. t
= L.
t 0 1  g x


x
xe x  log(1  x ) xe x  e x  {1 / (1  x )} g  
lim  lim , t
x 0 x2 x 0 2x F( t )
[form 0/0] By L’Hospital’s Rule I it follows that lim = L,
t 0 G ( t )


xe  e  e  {1 / (1  x ) 2 }
x x x and the proof is complete.
 lim
x 0 2 An important observation is that all the forms of L’Hospital’s
Rule developed so far are valid whether L is finite or not.

0 111 3
  This fact requires no new proof and has really already
2 2 been established. The reason is that the basic conclusion of

A variation of L’Hospital’s Rule, not difficult to prove, is the L’Hospital’s Rule I is the
following: f (x) f ( x )
equation lim = lim ,and this holds good whether
L’Hospital’s Rule III. Let f and g be differentiable on an x a g ( x )

x a g( x )


open interval (a, ∞) with g′(x) ≠ 0 for x > a. If f ( x )


lim f(x) = lim g(x) = 0, or not lim is finite or infinite.
(i) x x x a g( x )



f ( x ) There is another significant variation of L’Hospital’s Rule,
(ii) lim = L, whose proof requires the Generalized Mean Value Theorem.
x  g( x )


FREE BOOKS FOR JEE & NEET =>(@iitjeeadv)
1.86  
Differential Calculus for JEE Main and Advanced

It states that the condition (i) can be replaced by xlim


→a
|g(x)| g (c) f (c)
+|L| + .


= ∞. The specific statement which we prove is the following: g(x ) g( x )

L’Hospital’s Rule IV. Let f and g be functions which are Hence, the inequalities established in the first paragraph of
differentiable on a non-empty open interval (a, b) with g′(x) the proof imply that
≠ 0 for every x in (a, b). If f (x)
− L ≤ ε(1 + ε) + | L |ε + ε.
g( x )


(i) lim |f(x)| = lim |g(x)| = ∞,
x a  x a 
Since the right side of this inequality can be made arbitrarily


f ( x ) small by taking ε sufficiently small, it follows
(ii) lim = L, f (x)
x a  g( x )


that lim = L, and the proof is complete.
x a g ( x )


f (x)
then lim = L. It is not difficult to derive variations of the preceding theorem
x a g ( x )



analogous to the modified versions described earlier. Thus,
Proof: with the obvious changes in the hypotheses, this last form of
Let ε be an arbitrary positive number. By hypothesis (ii), there
L’Hospital’s Rule also holds for two-sided limits and with a
exists a real number c in (a, b) such that
or L (or both) replaced by ± ∞.
f ( x )
 L < ε, for every x in (a, c). 3
x 1
g( x ) Example 12: Compute lim .



x  x  4


By hypothesis (i) there exists a real number d in (a, b), which Solution: Let f and g be the functions defined by
we shall for convenience assume to be in (a, c), such that,


f(x) = 3 x +1 and g(x) = x + 4, respectively. Since f′(x) =
for every x in (a, d), the following three inequalities hold :
ps
1
and g′(x) = 1, we see that f and g are differentiable
el
f (c) g (c)
g(x) ≠ 0, < ε, <ε 3( x + 1) 2 / 3



g( x ) g(x )
eh

on the interval (1, ∞) and that g′(x) ≠ 0. Moreover, x lim |g(x)|
(see Figure). It is a consequence of the last inequality that = lim |x + 4| = ∞, and
je

x 1
f ( x )
iit

lim
x  g( x )
= 3( x  1) 2 / 3 = 0.

lim
@

g (c) x  1
1− < 1 + ε, for every x in (a, d).
ln x

g(x )
Example 13: Evaluate lim
Now let x be an arbitrary real number in (a, d). By the x  x


Generalized Mean Value Theorem, there exists a real number Solution: Since lnx approaches ∞ as x approaches ∞,


y in (x, c) such that L'Hospital's Rule implies that
f ( x )  f (c) f ( y) lnx 1/x 1
 . lim  lim = lim = 0 .

x  x x  1 x  x
g (c)  g (c) g( y)

n ( x  1)
Hence Example 14: Evaluate lim
f ′( y) x  x2


f(x) = (g(x) – g(c)) + f(c).
g′( y) n ( x  1) 

Dividing by g(x), which cannot be zero, we get Solution: lim 
x  2
 



x
f ( x ) f ( y)  g (c)  f (c) 1
 1   .
[by L'Hospital's rule]
g ( x ) g( y)  lim x  1
=

g(x )  g(x )


x  2 x
An equivalent equation is
f (x)  f ( y)  g (c)  = lim 1 =0
1
–L=   L  1   x  
( x  1)2 x


 g( y)  g(x )   

g( x )
x
g (c) f (c)
+ .–L Example 15: Evaluate lim x
x  e



g( x ) g( x )
Solution: Since ex approaches ∞ as x approaches ∞,
From the general properties of the absolute value, it follows that


L'Hospital Rule implies that
f (x) f ( y) g (c) x 1
L  L 1− lim  lim 0
g( x ) g( y) g(x ) x  e x x   ex


FREE BOOKS FOR JEE & NEET =>(@iitjeeadv)
Limits 1.87

ln x x and f'(x) and g'(x) satisfy all the requirements that have been
Note: lim  0 and lim 0. stated for f(x) and g(x), we can then pass to the ratio of second

x  x x  e x

derivatives, etc. However, it should be borne in mind that the
f (x)
x100 limit of the ratio may exist, whereas the ratios of the
Example 16: Evaluate lim g( x )
x  ex derivatives do not tend to any limit.


x100    If f(x) = x2sin 1 , g(x) = x, then we have
Solution: lim 
x  e   x x



we apply L'Hospital's rule repeatedly 100 times lim f ( x ) = 0.

x→0 g ( x )


100. 99.........2. 1
= lim =0 f '( x ) 1 1
ex

x  Here = 2x sin – cos
g '( x ) x x
Study Tip which oscillates when x → 0. Thus f/g may tend to a limit when
f′/g′ does not. Our condition is sufficient but not necessary.
While evaluating xlim
→ a {f(x)/g(x)}, when it is of the form
∞/∞, it is sometimes necessary to change it into the form 0/0, x  sin x
otherwise the process of differentiating the numerator and the Example 19: Evaluate lim
x  x  cos x


denominator will never end.
Solution: This limit has the indeterminate form ∞/∞. If


we try to apply L’Hospital’s rule, we get
ln x
Example 17: Evaluate lim 
ps x  sin x 1+ cosx


x  0 cot x 
lim = lim
x  cos x
x 

1+ sinx
el
x
ln x
Solution: We have, lim , [form ∞/∞] The limit on the right does not exist, because both sinx
eh


x  0 cot x 

1/ x and cos x oscillate between –1 and 1 as x → ∞. Recall that


= lim , [form ∞/∞]
L’Hospital’s rule applies only if this limit exists. This does not
je

x  0  cos ec 2 x




mean that the limit of the original expression does not exist or
iit

= lim  sin x ,
2
[form 0/0] that we cannot find it. It simply means that we cannot apply


@

x0 x 
L’Hospital’s rule. To find this limit, factor out an x from the
2 sin x cos x 2  0  1 numerator and denominator and proceed as follows :
= lim  0  sin x 
x0 x 1 


1 1
x  sin x x 
lim  lim 
Example 18: Evaluate xlim m –x x  x  cos x x   cos x 
 x e . x 1 
x 



xm
Solution: We have lim x m e  x  lim

, sin x
1


x x ex x  1 0  1
= lim .
[form ∞/∞] x  cos x 1  0

1

m 1 x
= lim mx [form ∞/∞]


x x 1
e x 2 sin 0
Example 20: Evaluate lim x  
m(m  1) x m  2 0


 lim [form ∞/∞] x →0 sin x


x ex Solution: Applying L'Hospital's rule, we get


......... ......... .........
1 1 1 
m(m  1) ( m  2) ...3.2.1 lim m ! = 0, 2 x sin  x 2 cos   2 
 lim  x x x 

x ex x   ex lim

x 0 cos x
[∵ ex → ∞ when x → ∞] 1 1
2 x sin  cos

Note: = lim x x . This limit does not exist.



f '( x ) x 0 cos x
If the quotient again yields an indeterminate form,
g '( x ) But this does not mean that the limit of the given function
at the point x = a, of one of the two above-mentioned types does not exist. We try another method.

FREE BOOKS FOR JEE & NEET =>(@iitjeeadv)


1.88  
Differential Calculus for JEE Main and Advanced

1 1 2 / (1  x 2 )  2 
x 2 sin
x 2 sin  lim = lim  . x 
x = lim x 0 x  0 sin x / cosx x  0  1  x2 
lim = =0.  tanx 
x →0 sin x x →0 sin x

1


x  2  x 
  lim
 x  0 1  x2   lim   2 1 2
This implies that L'Hospital's rule is inapplicable in this    x  0 tan x 
question.


x 5  2x 3  4x 2  9x  4
2  2 x  sin 2 x Example 24: Evaluate lim
x 1 x 4  2x 3  2x  1



Example 21: Evaluate xlim  ( 2 x  sin 2 x )esin x Solution: Since the form is 0/0, we shall apply




2  2 x  sin 2 x L'Hospital's rule.

Solution: xlim  5x 4  6 x 2  8x  9
 (2 x  sin 2 x )esin x   Limit  lim



,
2  2cos 2 x x 1 4x 3  6x 2  2
= xlim
 ( 2  2 cos 2 x )esin x  ( 2 x  sin 2 x )esin x .cos x [This is in 0/0 form. So, we again apply L´Hospital´s rule]
4 cos 2 x 20 x 3  12 x  8
 lim ,
= xlim x 1 12 x 2  12 x

 esin x (4 cos x  2 x  sin 2 x ) cos x
As x→ ∞, cosx becomes zero at several points. Hence, [This is in 0/0 form. So, we again apply L´Hospital´s rule]
the function is undefined at several points. Therefore the 60 x 2  12 60  12 48
limit cannot be evaluated. This implies that L'Hospital's rule  lim    4.
x  1 24 x  12 24  12 12

is inapplicable. We try to find the limit by another method.
ps
2  2 x  sin 2 x    sin x x 2  2 cos x  2
el
2
lim = xlim 1  2 x  sin 2 x  e Example 25: Evaluate lim .
x  (2 x  sin 2 x )esin x    x0 x sin 3 x


eh

The first bracket approaches 1 as x→ ∞, but e −sin x oscillates x 2  2 cos x  2


je

between 1/e and e. Hence the limit does not exist. Solution: lim
x0 x sin 3 x


iit

Using L’Hospital’s Rule with Other Limit Properties  x 2  2 cos x  2 x3 


@

 lim  . 
( cos x)sin4x x 0 x4 sin 3 x 
Example 22: Evaluate lim 
x 3cosx


x0

3
Solution: This limit has the form 0/0, but direct application  x  x 2  2 cos x  2
 lim   lim


of L’Hospital’s rule leads to a real mess. Instead, we compute the x  0  sin x  x  0 x4
given limit by using the product rule for limits along with two

simple applications of L’Hospital’s rule. Specifically, using the x 2  2 cos x  2 , ∵ lim x  1
 lim  x  0 sin x 
product rule for limits (assuming the limits exist), we have x0 x4  

(1  cos x ) sin 4 x 2 x  2 sin x
lim  lim .

x 0 x 3 cos x x0 4x 3
(by L´Hospital´s rule for the form 0/0)

  
 1  cos x   sin 4 x   1 

=  lim   xlim   lim
 2
 x  cos x  2  2 cos x

  
x  lim
x 0 0 x 0

 sin x   4 cos 4 x   1  x0 12 x 2


[form 0/0]
=  lim   lim   xlim 

 x 0 2 x   x 0   0 cos x 

1 2 sin x 1 sin x 1 1 .
 lim  lim  .1 

1 x  0 24 x 12 x  0 x 12 12
=   (4)(1)=2.
2

cos 2 x
log(1  x 2 ) Example 26: Evaluate lim
x  1/ 2 e 2 x  2ex


Example 23: Evaluate lim


x  0 log cos x
cos 2 x
 0
Solution: Since the form is 0/0, we shall apply Solution: lim form 
e 2 x  2ex 
x  1/ 2
0





log(1  x 2 )
L´Hospital´s rule. lim 2 cos x .(  sin x )
x  0 log cos x
= x lim
 1/ 2 2e 2 x  2e

FREE BOOKS FOR JEE & NEET =>(@iitjeeadv)
Limits 1.89

  sin 2x  0 x cos x ln cos x


= xlim ,  form   2 lim
1/ 2 2e 2 x  2e   x 0 sin x . (1  x 2 ) ln(1  x 2 )


0


= lim 2 cos 2x  2 (1)   .
2
2 2 x cos x ln cos x
 2 lim . lim . lim
x  0 sin x x 0 1 x 2 x 0 ln(1  x 2 )

x 1/ 2 2c
4e 4e 2e


Example 27: Evaluate the limit,  2  1  1  lim
ln cos x
, [form 0/0]


 x2 )



x  0 ln(1
 1  2 x )(cos x  1)
2
2x 2
(e
lim .
 sin 3x  ln(1  3x )  x 4 1
. ( sin x )

x 0

 2 lim cos x
Solution: x 0 1
. (2 x )


 1  cos x  (e 2 x  1  2 x 2 ) x 2 1  x2
2

lim 
L= x→0  lim
 x 2  x 0 x 2 · x 2 sin 3x  ln(1  3x ) 


 sin x 1  x 2 

1
 2 . lim  .  = 1.
e  1  2t 2 x  0  x
2t 2
1


x cos x
=– lim · lim 
sin 3x  ln(1  3x )

 2 
2 t 0 t x 0

where x2 = t Example 30: Find the values of a and b in order that





4(e y  1  y) x2 x (1  a cos x )  b sin x
1
= – ylim · lim lim may be equal to 1.
x 0 sin 3x  ln(1  3x ) x3

y2 x 0
2 0

where 2t = y
ps
x (1  a cos x )  b sin x

x2 Solution: We have lim
el
1
x3


=– · 2 · lim x 0
x0 sin 3x  ln(1  3x )

eh

2
[form 0/0, so we shall apply L'Hospital's rule]
x2

Now l = lim
je

.
sin 3x  ln(1  3x ) 1  a cos x  ax sin x  b cos x ...(1)

x0
 lim
iit

x 0 3x 2
Using L'Hospital's rule, l = 2 .
@

Now the denominator of (1) → 0 as x → 0. Therefore if the



2 9
Hence, L = – . numerator of (1) does not tend to 0 as x → 0, then the given
9 limit cannot be equal to 1. Hence for the given limit to be
Example 28: Evaluate lim tan x.log e sin x . equal to 1 the numerator of (1) must also → 0 as x → 0.
x  / 2


∴ 1 + a – b = 0 or a – b = –1 ...(2)


log e sin x  0
Solution: limit = lim Now if 1 + a – b = 0, then (1) takes the form 0/0. Hence by
 form 0 
x  / 2   applying L'Hospital's rule to (1), the given limit is equal to


cot x
1 a sin x  a sin x  ax cos x  b sin x
.cos x
[using L'Hospital's rule ] lim
= lim sin x x 0 6x

x   / 2  cos ec x
2

ax cos x  (b  2a ) sin x , [form 0/0]
= lim   sin x.cos x  =0  lim

x  / 2 x 0

6x
ln ln(1  x 2 ) a cos x  ax sin x  (b  2a ) cos x
Example 29: Evaluate lim  lim
x 0 6


x  0 ln ln cos x

ln ln(1  x 2 ) [by L'Hospital's rule]

Solution: We have, lim [form ∞/∞] a  b  2a 6  3a


x  0 ln ln cos x   1 [as given]

1 1 6 6
. . (2 x ) ∴ b – 3a = 6. ...(3)
ln(1  x ) 1  x 2
2


 lim Adding (2) and (3), we have –2a = 5 or a = –5/2.
x 0 1 1
. . ( sin x ) ∴ b = a + 1 = (–5/2) + 1 = –3/2.
ln cos x cos x

Hence a = – 5/2, b = –3/2.

FREE BOOKS FOR JEE & NEET =>(@iitjeeadv)
1.90  
Differential Calculus for JEE Main and Advanced

m n
Other Indeterminate Forms Example 33: Evaluate lim x (ln x ) , where m, n are
x 0 



Form 0 × ∞ positive integers.
m n
This form can be easily reduced to the form 0/0 or to the Solution: We have lim x (ln x ) , [form 0 × ∞]
form ∞/∞. x  0



lim f(x) = 0 and lim g ( x )   (ln x ) n , [form ∞/∞]
Let
x →a x a  lim


x m

x  0
Then we can write lim f ( x ). g ( x ) n (ln x ) n 1 (1 / x )  n (ln x ) n 1 
x →a
 lim  lim   . ,
f (x) , x  0 mx  m 1 x 0  m


x  m 
 lim [form 0/0]


x  a 1 / g( x ) [form ∞/∞, if n > 1]


n 2
or  lim
g( x ) , [form ∞/∞]  n  (n  1) (ln x ) .(1 / x )
 lim    .


x  a 1 / f (x) x 0  m  
mx  m 1
Thus lim f ( x ). g ( x ) is reduced to the form 0/0 or ∞/∞ which n (n  1) (ln x ) n  2
 lim  1
x →a
2
.
can now be evaluated by L'Hospital's Rule or otherwise. x 0 
m2 x m
n (n  1) (n  2)...upto n factors (ln x ) x  n ,
Example 31: Find lim cotx . log secx  lim  1 .
n
x  / 2
.
x m


x 0 
mn
Solution:
lim cot x log sec x [0 . ∞ form] [by repeated application of the above process]
x  / 2



ps n! m
= lim
log sec x
[form 0/0] . x  (1) n n ! . lim x m  0 .
 lim (1) n
mn
el
x  0 mn x  0 
x  / 2 tan x


Form (∞ – ∞)
eh

1
. sec x tan x
= lim sec x This form can be reduced to the form 0/0 or to the form ∞/∞.
 lim sin x cos x  0
je

x  / 2 sec 2 x x  / 2 When lim f(x) = ∞ and xlim→a
g(x) = ∞, then
x →a
iit

lim [f(x) – g(x)], [form ∞ – ∞]


@

Example 32: Evaluate lim (π – 2 tan –1 x) ln x x →a






x

(  2 tan 1 x )  0  1 1 
 lim   
lim  form 0  x  a 1 / f (x ) 1 / g( x ) 
Solution: x  1  



nx

{1 / g ( x )}  {1 / f ( x )}
2  lim [form 0/0]



x  a {1 / f ( x )} . {1 / g ( x )}
1 x2
Applying L’ Hospital’s Rule = lim 1 1 Now this can be evaluated by applying L'Hospital's rule or
x 
 otherwise.
(ln x ) 2 x  1 1
Example 34: Find lim   .
2
 x0  sin x x


= lim 2 x (ln x ) 


x  1  x 2   Solution: If x  0 , then sin x  0


1
= 2(ln x ) 2  2 x.2 ln x. and 1  1    
x sin x x

lim 
x  2x Similarly, if x  0 , sin x  0
 
= lim (ln x )  2 ln x
2
 form   and
1 1
− = → – ∞ – (–∞) = – ∞ + ∞
 


x  x sin x x
1 2 Neither form reveals what happens in the limit. To find out,
2 ln x. 
= x x we first combine the fractions.
lim

x  x 1 1 x  sin x
 
1

= lim [2ln x + 3] = 0. sin x x x sin x

x  x 2 and then apply L'Hospital's rule to the result.

FREE BOOKS FOR JEE & NEET =>(@iitjeeadv)


Limits 1.91

 1 1 x  sin x 0  1 
lim    lim    lim  2    1
x 0 sin x x  x 0 x sin x 0 x  0  sec x 



1  cos x 0 ∴ L = e–1 = 1/e.
 


= lim
x0 sin x  x cos x 0  xx ) .
x
(x )


Example 37: Calculate lim ( x
x 0 



sin x 0
= lim  0.
x0 2 cos x  x sin x Solution: We first calculate xlim

2 xx:
0 



 1 1  it is just exp( lim x ln x).
Example 35: Evaluate lim  2  x  0
x   x

tan 2 x 


ln x
Now lim x ln x  lim by L'Hospital's Rule
 1 1 


 x 0 
x 0 
1/ x
Solution: xlim 
  x 2

tan 2 x 
(∞ – ∞)



1/ x
   lim  lim  x  0
x 0 
1 / x 2 x  0


 2   0
= lim  tan x  x 
2
 form 0  Hence lim xx = 1.
x 0    


2
2 tan x x0
x .x 2 
 x 2

 xx )  1 .
x
(x )
Therefore lim ( x
(tan x  x )(tan x  x ) x 0 

= lim

x 0 x4  
Example 38: Find lim e  e  2 cos   4
ps
(tan x  x )(tan x  x )


= lim 0 4
el
x 0 x . x3

e  e    2 cos   4  0
eh

tan x  x Solution: lim  form 0 


= 2 lim 0  4
 


x 0 x3
je

sec 2 x  1 e  e    2 sin 
iit

= 2 lim [by L'Hospital's rule] = lim (by L’ Hospital’s Rule)


43

0

x 0 2
3x
@

e  e    2 cos 
1  cos 2 x = lim

= 2 lim 0 122
x 0 cos 2 x 3x 2

 
2 sin 2 x 2 = lim e  e  2 sin 
24

= lim = . 0
2
3 x →0 1. x

3  
= lim e  e  2 cos  =
4 1
= .

Note: 0 24 24 6


The indeterminate forms 1∞, ∞0 are evaluated by first taking
Example 39: Determine lim (cot x)1/log x
logarithms and then finding the limit of the logarithm of the


x→0

power [f1 ( x )]f ( x ) (which requires evaluating a form like 0.∞ ).


2 Solution: Let y = (cot x)1/log x.


1
Example 36: Evaluate xlim (cosec x)1/ln x log y = log (cot x)

→0 log x


lim log y = lim log cot x 
Solution: Let L = xlim (cosec x)1/ln x [form 1∞] ⇒ 

 
→0 x0 x→0

log x



∴  n L  lim
1
( n cos ec x ) , [form ∞/∞]  cos ec 2 x


x 0  n x = (by L’ Hospital’s Rule)
lim cos x


(1 / cosec x) (-cosec x cotx) x 0 
1/ x
 lim

x 0 1/ x x 1
= lim . =–1

x 0 sin x cos x


 x  ,
 lim  
[form 0/0]
⇒ log lim y = – 1 ⇒ lim y = e–1 = 1/e.


x  0  tan x  x0 x0

FREE BOOKS FOR JEE & NEET =>(@iitjeeadv)
1.92  
Differential Calculus for JEE Main and Advanced

Example 40: Find x→0lim 1  sin x  cos x  log(1  x ) However, in (2) we get L =
1
from where we can get the


x tan 2 x L
Solution: The inconvenience of continuously diffe- limit. We have L2 = 1 ⇒ L = ± 1.


rentiating the denominator, which involves tan2 x as a factor, Since the function is positively valued, the limit is non

-
may be partially avoided as follows. We write negative ⇒ L = 1.
1  sin x  cos x  log(1  x ) Here are some more examples:
x tan 2 x 9x  1 x


(i) lim (ii) lim
x  x 1





2 x 0
1  sin x  cos x  log(1  x )  x 

sin x
= .  
 tan x  cotx

x3 (iii) lim
1  sin x  cos x  log(1  x )



x 0 cosecx
so that lim
x 0 x tan 2 x Leibnitz’s Formula for Differentiation of Integrals


1  sin x  cos x  log(1  x ) lim  x 
2
To differentiate an integral function, one way is to find the
= lim
x→0  tan x  integral and then differentiate as follows:
 

x 0 x3
x2 x2
1  sin x  cos x  log(1  x ) t3 x 6 ( 2 x )3 x 6  8 x 3

2
= lim .1 Let f(x) = t dt =   
x3

x 0 3 3 3 3
2x 2x
1  sin x  cos x  log(1  x )
= lim 6 x − 24 x 2 5

x 0 3 Hence, f '(x) = = 2x5 – 8x2.
x
3
To evaluate the limit on the R.H.S., we notice that the numer-
ps
But a better approach is given by Leibnitz rule:
ator and denominator both become 0 for x = 0.
el
d  
v( x )
1  sin x  cos x  log(1  x )   f ( t )dt  = f(v(x)) dv – f(u(x)) dv .

eh

lim
x 0 x3 dx  u ( x ) 



  dx dx
cos x  sin x  [1 / (1  x )]
je

The proof will be dealt in the chapter of definite integral. We


= lim
iit

x 0 3x 2 can now differentiate the same function as follows:



d  2 
(by L’ Hospital’s Rule) x 2
@

 sin x  cos x  [1 / (1  x ) 2 ]   t dt  = (x2)2 . 2x – (2x)2 . 2 = 2x5 – 8x2.


du  2 x 

= lim
x 0 6x
 x 2

 t dt
2
 cos x  sin x[2 / (1  x )3 ] 3 1
= xlim   . Example 41: Evaluate lim
2x
0 6 6 2

x 0 x3


x2
Cycling
 t dt
2
L'Hospital's rule does not help in evaluating the following limits. 0
When we apply it we just keep on cycling. We need to find lim 2 x 0
Solution: x 0 x 3  



the limits using some other method.
sec x    We apply L'Hospital's rule with the help of Leibnitz rule
Consider L  lim  ...(1)
 tan x  


x
2 2 x 5  8x 2 2 3 8 8
Limit = lim  x   .
sec x tan x

x 0 2
3x 3 3 3
= lim
sec 2 x

x
2 x3
tan x
= lim sec x

 ...(2)
 ntdt
x2
x  



2 Example 42: Evaluate lim
x  x4


2
sec x sec x
= lim = lim =L ...(3) x3
x  sec x tan x x  tan x
 ntdt


2 2
x2

Applying L'Hospital's rule again and again, we get the question 
Solution: xlim  
back in (3).  x4



FREE BOOKS FOR JEE & NEET =>(@iitjeeadv)
Limits 1.93

(nx 3 )3x 2  (nx 2 ).2 x  3 x  1 ( x  1) ( x 3/ 2  1)  1


= lim = lim    . 2

x  4x 3 x 1  x  1 x 1 x  1  x  x  1
[by L'Hospital's rule] 
= lim
9 xnx  4nx 
 3
x 1   ( x  1)   ( x 3/ 2  1))  

=  lim    lim    lim 
x  1   x 1 x  1   x 1 x  1  

x  4x 2 x 1

 
(9 x  4)nx 1
= lim lim
4x 2

x 
. x 1 x  x  1
2


 4  nx (by L' Hospital’s rule)
= lim  9  


x   x  4x

= lim  ( x )1/ 31  ( x )1/ 21  ( x )3/ 21  .
1 1 3 1
x 1  3  12  1  1


= 9 × 0 = 0. 2 2

Example 43: Find the constants 'a' (a > 0) and 'b' 1 1 3  1
=  (1) 2 / 3  (1) 1/ 2  (1)1/ 2  .


such that ,   3



3 2 2
X
t 2 dt
 at
1 1 3 1
=    .
 239  1 7
=  . 3  9 .
lim 3 2 2 3  


x→0
0
1. 6
bx  sin x

0 3
Example 46: If lim 2 n n x dx = k then find the
Solution: form hence using L`Hospital rule  5


x 1


0 ps n  n
x2 1
lim a x value of   , where [ . ] is the greatest integer function.
el
l = x→0 for existence of limit k
b  cos x

eh

lim b − cos x = 0 ⇒ b=1 2n n3x


x→0
n



je

Solution: lim dx (∞ × 0)
n  x5  1



lim x2 lim 1
iit

hence x→0
. x→0 =1
1 − cos x a+x 1
Put n =
@


2 t
=1 ⇒ a=4

a x 2
x
x   cos t 2 dt  t dx
x 1
1 5
Example 44: Evaluate 0


0 t

lim . lim 0
x 0 x  6x
3 t 0 
t 3
 
x
Applying L’ Hospital’s rule
x   cos t 2 dt 0
Solution: [form ] 2 1


0 0
lim
x 0 x  6x
3
t  2  1
  2  t  
32  t  1
Applying L'Hospital's rule we get
1 1  t 

5 5
lim t t
x
d
1 
dx 0
cos t 2 dt
1  cos( x 2 )
t 0  3t 2

= lim = lim 4 1
x 0 3x 2  6 x 0 3( x 2  2) 

7
= k.
lim 32  t 1  t5 =
5
=
1  cos 0

0 t 0 
24
=   0. 3
3(0  2) 6

 1   24 
3
x  x  x x 3
∴  k    7  = 3.
   

Example 45: Evaluate lim 2
 x e x dx 


x 1 x3  1 0
2


Example 47: Evaluate lim  
( 3 x  1)  ( x  1)  ( x 3/ 2  1)  0  x   x 2 x 


  0 e dx 
2
lim  0 form 
Solution: x 1 ( x  1)( x 2  x  1)    


FREE BOOKS FOR JEE & NEET =>(@iitjeeadv)
1.94  
Differential Calculus for JEE Main and Advanced

e  x  1
2
2
Solution: If y =  g ( x )dx  , then 3 2
x

  
= . lim



0 5 4 x 0 x2
Again using L' Hospital's rule, we get
 2   g ( x )dx  . g(x)
dy x



 0 

dx 3 lim −e − x .(−2 x )
2

3 3
= = .1= .
10 x→0


x (2x ) 10 10
(  e x dx ) 2
2

0  
Now lim   form   2 x (1  t 3 ) 1 dt 
   x
x  x 2x2


e dx 
0 Example 49: Find lim  4 x .
x  3 1
  (1  t ) dt 



 3x 
2   e x dx  (e x . 1) 2  e x dx
x x 2 2 2

 0  0  
 lim    form  Solution: Both the numerator and denominator tend to
 
2 2
x  e2 x ex



zero as x → ∞, and hence L' Hospital's rule applies.
2
2x 1
2e x . 1 1 x 1  t 3 dt
 lim x2
 lim  0.
x  e .2 x x  x
4x 1

3x 1  t 3 dt
 1 x t2 1 1 
Example 48: Evaluate lim 
x 0  x5
0 e dt  4
 
3x 2  2 1


x 
ps
= 1  8x 1  x3
3
x
3 e  t dt  3x  x 3 

4 3
el
2

0
Solution: We have lim 0  form 0  
  1  64 x 3
1  27 x 3


eh

x 0 x5
2 1
je

Using L' Hospital's rule, we get 



= 1/ x  8 1/ x 1
3 3
iit

3e x  3  3x 2
2

e  x  1  x 2  form 0 

2
4 3
lim = 
@

lim  0  1 / x  64 1 / x  27
3 3
x 0 5x 4 x 0 x4 
Again using L' Hospital's rule, we get 2
1


8 108
3 lim e  x .(2 x )  0  2 x 
2

= = 4 3 .
7

5 x→0 

4x 3 64 27

Concept Problems P

1. Evaluate the following limits:
ln (1 + e −2 x ) a1/ x  b1/ x

a ( x ln a  1)  1
x (iii) lim (iv) lim
ln (1 + e − x ) x  ln{x / ( x  1)}


(i) lim x →−∞


x 0 x2
3. Evaluate the following limits:
ln(tan 2 2 x )

(ii) lim
2x  x sin 3x


x →0 ln(tan 2 x ) (i) lim


x  5x 2  2 x  1
ln(sin x )
(iii) lim
x 0 ln(sin  x ) 2 x  sin x


(ii) lim
3x  sin x


2. Evaluate the following limits: x 

ln (1 + e − x ) (ii) lim ln (1  e )
x
(i) xx  x
lim (iii) lim



x →∞ e− x x  x x  1 1  x  ln x


FREE BOOKS FOR JEE & NEET =>(@iitjeeadv)
Limits 1.95

ln x x
4. Show that lim  0, m > 0. (iii) lim 1 1  4 t 2 dt
.

x  m
x



x  sin x
x 1 x 1
5. Show that lim exists even if
x  2 x  sin x

7. Evaluate the following limits:


1  cos x x
lim
x  2  cos x
does not exist.
0 4  t 4 dt

(i)
lim



6. Evaluate the following limits: x  x3

x2

x2 t2
(i) lim 0
sin x dx
(ii)
lim
x
e dt




x 0 x3 x 1 x 1
x 1
(ii) lim 0 (tan x ) 2 dx
(iii) lim 0
x
1  t 2 dt


x 



x2  1 x  x2

Practice Problems N

ps
el
8. Evaluate the following limits: 14. Evaluate the following limits:


eh

2
3t
(i) lim (i) lim
1
sin x.sin 1
x  x2 
je

3(1  t )1/ 3  t  3


t 


x 0 6
x
iit

(ii) lim x  2 x  1 .
1/ 3

(ii) lim 1 tan x.tan 1 x  x 2  


@


1/ 2x  x  3x  2

6 x 0 x
9. Evaluate the following limits:

sin 2 x  2 sin 2 x  2 sin x 15. Suppose that F is a function differentiable on the open
(i) lim

x 0 cos x  cos x 2 1



interval (0, ∞) and such that F′(x) = , for every x > 0,
x
 1 ln(1  x )  Show that
(ii) lim   
x  0  x (1  x ) x2 


F( x ) F( x )
log a n (a) lim =0 (b) lim =0
 0 for a > 1, α > 1 x 




10. Prove that lim 
x x  x2
n 

n
(ln x ) n n F( x )
11. Show that lim = 0 and lim x  0 for all (c) lim = 0, for every positive integer n.
x  xn



x →∞ n x  e x
positive integers n.
a sin 2 t a
 e dt  
2

(1) n esin t dt
12. Prove that xlim (x – n)cosecxπ = , n being any 16. Evaluate lim y y x .

→n x→0


integer. x
17. Evaluate

 (1) n  

1
13. Prove that lim cos ecx  
x n x  n    

 ( x n )  π 4 1/ h2
+h e π /3

(1) 
n lim ∫0
3 cos3 x dx − ∫
0
cos3 xdx
.
h→0

= , n being any integer. h 4 e1/ h
2

6

FREE BOOKS FOR JEE & NEET =>(@iitjeeadv)


1.96  
Differential Calculus for JEE Main and Advanced

2
1.19 Geometrical Limits  tan( x / 2)   sin x 
 x / 2   x  x3

   .


Example 1: A circular arc of radius 1 subtends an angle = lim
x 0 ( x  sin x ) 4


of x radians, 0 < x < π/2 as shown in the figure. The point R is
the intersection of the two tangent line at P and Q. Let T(x) be 2
the area of triangle PQR and let S(x) be the area of the shaded = 1 lim  tan( x / 2)  lim  sin x  .
4 x→0  x / 2  x→0  x 


region then find
T( x )
(i) T(x) (ii) S (x) (iii) lim x3 3
x →0 S( x ) . lim = .



( x − sin x )


x→0 2
Example 2: A tangent line is drawn to a circle of radius



unity at the point A, and a segment AB is laid off whose length
is equal to that of the arc AC . A straight line BC is drawn to
intersect the extension of the diameter AO at the point P .
Prove that:
 (1  cos )
(i) PA = (ii) lim PA = 3

  sin  0


ps
el
eh

tan   =
x PR
Solution: (i) In ∆ OPR,




2 1
je

∴ PR = tan x = RQ
iit

2
( Length of tangent from a point outside the circle are
@

equal)
and ∠ PRQ = (π – x)

∴ T(x) = Area of ∆ PQR
Solution: See the labelled figure below:




1
= . (PR) (RQ) sin (π – x)

2
x
= 1 . tan2   sin x ...(1)
2


2
x sin x
= tan   –
2

2
(ii) S(x) = area of sector OPQ – area of ∆ OPQ

1 1
= (1)2 . x – . (1)2. sin x

2 2
(i) From the figure,

( x − sin x )   sin 
=
tan φ = ...(1)

2
 x  sin x 1  cos 


tan   
lim T( x ) = lim 2 2 θ
(iii) x→0 Now in ∆ BPA , tan θ =
( x  sin x )

x→0 PA


S( x )
2
tan 2 ( x / 2) sin x   (1  cos )
= lim ⇒ PA = =
( x  sin x ) tan    sin 


x 0


FREE BOOKS FOR JEE & NEET =>(@iitjeeadv)
Limits 1.97

 (1  cos ) 1 1
lim lim
(ii) 0   sin  3 2 0 
(2)3 
. 1

3 =    2  .... 


2 3 
1  cos  lim 3
= lim
0 0
3
  sin 

2 1 12 3
=  
1  cos 


1 2 8 4
Now lim
0
=
2

2
Example 4: The figure shows a fixed circle C1 with



  sin  3 t − sin 3 t equation (x – 1)2 + y2 = 1 and a shrinking circle C2 with radius
Let  = lim
0 = tlim
→0 r and centre the origin. P is the point (0, r), Q is the upper
3 27 t 3

point of intersection of the two circles, and R is the point of
where θ = 3t intersection of the line PQ and the x axis. What happens

-
3 t − (3 sin t − 4 sin 3 t ) to R as C2 shrinks, that is, as r → 0+?
= tlim
→0
27 t 3

t − sin t 4
= tlim
→0 3 +

9t 27
 4 8 4 1
= ⇒ ⇒ =
+ =
ps

9 27 9 27 6
el
3
Since lim Solution: We find the coordinates of the point Q by
eh

= 6,
0   sin 



solving (x – 1)2 + y2 = 1, and x2 + y2 = r2. On
je

lim PA = 1 . 6 = 3. 2
0 substraction, we get x = r .
iit

2 2
@

Example 3: In the figure shown, let f(θ) = area of ∆ Points P, Q and R are collinear. Hence mPR = mPQ


ABC and g(θ) = area of sector OBC – area of ∆OAC. Find
r2
the limit of ratio of f(θ) and g(θ) as C moves to B along the r 1 r
r0 4
circumference. ⇒ 
0x r2

0
2
r2
2
⇒ x=
r2

1− 1−
4
r2
Solution: We need to find Now, rlim x  lim 2


0 r 0
  1/ 2
1  r2 
(r  r cos )r sin  1  1  
f ()  4 
lim  lim 2
0 g ()

0 1 2 1

r2
r   r sin  .r cos 

2 2 = lim 2 = 4.
r 0 1 r2



 1  cos   sin  1  1  .  ....


   0 2 4
= lim   
2
 
  sin  cos  0

0 

Example 5: Let (tan α) x + (sin α) y = α and




3 (α cosec α) x + (cos α) y = 1 be two variable straight lines, α

FREE BOOKS FOR JEE & NEET =>(@iitjeeadv)


1.98  
Differential Calculus for JEE Main and Advanced

being the parameter. Let P be the point of intersection of the 2


lines. Find the limiting position of P when α → 0. 
= lim  lim 2 = 2
Solution: Here two straight lines, (tan α) x + (sin α)  0   0 


tan tan


y = α and (α cosec α) x + (cos α) y = 1 have their point of 2 2
intersection as,
 cos   sin    x tan  Again as, lim y = lim   x tan 
x= and y = 0
sin 


sin    sin   0

∴ when α → 0, we obtain the point P.
  x 

 cos   sin   0 form  = lim  
cos  
lim x =
Hence, 0 lim 0   0  sin 


 0 sin     
 sin   cos   cos   x
= lim = lim  lim =1–2
 0 cos   1  0 sin   0 cos 


(applying L' Hospital’s rule)

   [as lim x = 2]
0
  2 sin cos 

 sin 
= lim = lim  2 2
∴ lim y = –1.
 0 2 sin 2  / 2


 0 
0
2 sin 2
2 Hence, in limiting position, P is (2, –1).
ps
Concept Problems Q
el

eh

1. Show that the perimeter of a regular polygon of 5. A straight line AB moves so that the sum of its
je

θ


4R intercepts OA, OB on two fixed straight lines OX, OY
iit

n sides is given by P(θ) = sin where




2π 2 is constant. If P be the ultimate intersection of two
θ=
@

is the central angle subtended by one side consecutive positions of AB, and Q the point where
n AB is met by the bisector of the angle XOY, then
of the polygon. Hence show that a circle of radius R has
prove that AP = QB.

circumference 2πR.
2. A straight line AB moves so as to include with two fixed 6. A segment AB = a (see fig.) is divided into n equal parts,


straight lines OX, OY a triangle AOB of constant area. each part serving as the base of an isosceles triangle with
Prove that the limiting position of the intersection of two base angles α = 45°. Show that the limit of the perimeter
consecutive positions of AB is the middle point of AB. of the broken line thus formed differs from the length
3. Find the limit of the sum of the lengths of the ordinates of AB despite the fact that in the limit the broken line

of the curve y = e–x cos πx, drawn at the points x = 0, “geometrically merges with the segment AB”.
1, 2, ...., n, as n → ∞.
4. Find the limit of the sum of the areas of the squares

constructed on the ordinates of the curve y = 21–x as on
bases, where x = 1, 2, 3, ...., n, provided that n → ∞.

Practice Problems O

7. The point C1 divides a segment AB =  in half ; the point 8. The side a of a right triangle is divided into n equal parts,


C2 divides a segment AC1 in half ; the point C3 divides a on each of which is constructed an inscribed rectangle
segment C2C1 in half ; the point C4 divides C2C3 in half, (see fig.). Determine the limit of the area of the step-like
and so on. Determine the limiting position of the point figure thus formed if n→∞.
Cn when n → ∞.

FREE BOOKS FOR JEE & NEET =>(@iitjeeadv)


Limits 1.99

of the substance available at the commencement of each


interval and to the length of the interval. Assuming
that the quantity of substance at the initial time is Q0,
determine the quantity of substance Qt(n) after the elapse
of time t if the increase takes place each nth part of the
time interval   t .
9. A weight hanging by a spring is made to vibrate by n

applying a sinusoidal force, and the displacement at time Find Q t  lim (Q) t .
(n )
n 


t is given by
C 11. A straight line AB of constant length moves with its
(sin t  sin t )


f(t) = extremities on two fixed straight lines OX, OY which
 

2 2
are at right angles to one another. Prove that if P be the
where C, α, β are constants such that α ≠ β. What happens ultimate intersection of two consecutive positions of AB,

to the displacement as β → α ? You may assume that α and N the foot of the perpendicular from O on AB, then
is fixed. AP = NB.
10. A certain chemical process proceeds in such a fashion 12. If PCP' be any fixed diameter of an ellipse, and QV any


that the increase in quantity of a substance during each perpendicular to this diameter ; and if the tangent at Q
interval of time τ out of the infinite sequence of intervals meet CP produced in T, prove that the limiting value of
(iτ, (i + 1)τ) (i = 0, 1, 2, ....) is proportional to the quantity the ratio TP: PV, when PV is infinitely small, is unity.
ps
el
1.20 Miscellaneous limits  nx   0 
lim  lim
1  nx 
= lim   =0
eh

n   x 0 n  1  0 

Repeated Limits
je

The operations of proceeding to the limit zero with each of


Caution
iit

two variables x and y may or may not be commutative when


@

applied to a function f(x,y). Thus


Here we note that interchange of limits is giving different
lim { lim ( x  y)} = lim x  0 , results. Hence it is advisable not to interchange the order of
x 0 y 0 x 0

limits indiscriminately.
lim { lim ( x  y)} = lim y  0
y 0 x 0 y 0 Representation of Functions Using Limits

but on the other hand
lim xn.
Example 2: Simplify the function y = n
xy x


lim { lim ( )}  lim  lim 1  1
x 0 y 0 x  y x 0 x x 0

Solution: When x2 < 1 i.e –1 < x < 1,


xy y lim x 2 n  lim ( x 2 ) n  0
lim { lim ( )}  lim  lim(1)  1 n  n 
xy

y 0 x 0 y 0 y y 0

When x2 > 1 i.e. x < –1 or x > 1,
Hence the limits may or may not be commutative.
 nx  lim x 2 n  lim ( x 2 ) n   .
n  n 

Example 1: Evaluate lim  lim  and
x 0  n  1  nx 




When x2 = 1 i.e. x = ±1,


 nx 
lim  lim 
n   x 0 1  nx  
lim x 2 n  lim ( x 2 ) n 1 .

 

n  n 
 nx   x 
Solution: xlim  lim  = lim  lim
0  n  1  nx     0, if x2  1
x 0 n  1


  x 




 n  ∴ lim x 2 n   1, if x2  1
x n 

= lim   = 1. 
 , if x2  1
x 0  x  



FREE BOOKS FOR JEE & NEET =>(@iitjeeadv)


1.100  
Differential Calculus for JEE Main and Advanced

 0, if 1  x  1
Example 4: If F( x )  lim x f ( x )  g ( x ) , find F(x)
2n

  1, if x  1



n  x 2n  1

, if x  1 or, x  1
 in terms of f(x) and g(x).
Here y is defined only when –1 < x ≤ 1. It is equal to 0 when x 2 n .f ( x )  g ( x )
–1 < x < 1 and to 1 when x = 1. Note that the points x = 1 Solution: ∴F(x) = lim
x 2n  1




n 
and x = –1 are the points of change in definition.
 0.f ( x )  g ( x )
 1  x 2n   , 1  x  1
Example 3: Evaluate lim lim   0 1

x 1 n   1  x 2 n
 1.f ( x )  g ( x ) ,


  x  1
 11
 1  x 2n  = 
Solution: Let f(x) = lim  


n  1  x 2 n   f (x )  g(x )


   x 2n , x  1 or x  1
We first simplify this function. We have  1
 1  2n
 0, if 1  x  1  x
lim x 2 n   1, if x  1  g( x ), if  1  x  1
n 

, if x  1 or, x  1  f ( x )  g( x )
 
∴ F(x) =  , if x  1
Hence, f(x) 2


1  0 f ( x ), if x  1or x  1
; 1  x  1
1  0
ps

 0 1
Example 5: Evaluate lim lim tan x 2 ( x  1) n.sin x
el
2 n
=  ; x  1, x  1
 0 1

x  ( x  1)


x 0 n 
eh

1  1
1  1 ; x  1
je

 tan x 2  ( x  1) n .sin x
Solution: Let f(x) = lim
iit

n  x 2  ( x  1) n


 1 
 1 
@

2n
using lim  x  tan x 2  ( x  1) n .sin x
 = 0 when x < – 1, x > 1.  lim , left nghd of x  0
n   1  x 2  ( x  1) n
  1   n 
 x 2n  
 tan x 2
f(x) =   sin x
1 , 1  x  1  lim ( x  1)
n
  n  , right nghd of x  0
= 1 , 1  x  1 
x2
 1

0 , x  1  ( x  1) n
 
The graph of f(x) can be drawn as follows:  tan x 2
 , left nghd of x  0
f(x) =  x 2

sin x , right nghd of x  0


Since xlim f ( x )   , lim f ( x )  0 , the required limit does


0 
x 0 

not exist.
Example 6: Let f: R → R be defined by


 
Now we find the limit of f(x) as x → 1. f ( x )  lim  lim (cos m !x )  n  .
m   n  

lim f ( x )  1 Prove that f(x) is 1 for rational values of x and, 0 for irrational
x 1

values of x.
lim f ( x )  1 .

x 1 Solution: Let x be a rational number say p/q, where p,


 1 x 2n  q are integers prime to each other. By taking m sufficiently
Thus, the limit lim lim   does not exist. large, m! πx can be made an even integral multiple of π, so
x 1 n  1  x 2 n
  that cos m! πx = 1.

FREE BOOKS FOR JEE & NEET =>(@iitjeeadv)


Limits 1.101

Example 8: Evaluate
Hence f(x) = nlim 2n
 (1) = 1 where x is rational.



 n n n 
If x is irrational, then cos m! πx will always lie between –1 lim   2  ....... 2 
n   n 2  12 n 2 2
2n 
and 1 for any integral value of m so that


(cos m! πx)2n = (rm)2n where |rm| < 1 1  n2 n2 n2 
 2 2  2  

for a fixed value of m. Solution: nlim
n  n 1
.......
 n  22 2n 2 



Thus, f(x) =
lim lim (r )2n = 0, when x is irrational.
m n  m 1 1 1 1 
lim  2
 2
 ..... 2 
Hence, f(x) is 1 for rational values of x and, 0 for irrational n  n 1
= 2 n 
 1    1    1    


values of x.
 n n n 
Example 7: Show that the function f: R → R defined by 1
1 n


1 1 1

= n  n 
 sin 2 (n ! x )  lim  dx  tan 1 x 0
f ( x )  lim  lim  is equal to 0 when x is r 1 r
2
0 1 x 2
1  


n   t 0 sin 2 ( n ! x )  t 2 
  n
rational and to 1 when x is irrational.
Solution: Let x = p/q be a rational number. Then by = tan–11 – tan–10 = π .


4


taking n sufficiently large n! πx can be made an integral
multiple of π so that sin n! πx = 0.  
Example 9: Evaluate lim
n n


0 n 
Hence f ( x )  tlim
0 0  t 2
 0 when x is rational. ps n
1  1.2.3........n 
If x is irrational, then 0 < sin2 n! πx < 1. Solution: nL  lim n  
n  n  n.n.n.......n 


el
1
Then f ( x )  nlim lim  1 n
eh

1 2 3
 t 0 1  t 2 / sin 2 ( n ! ) = lim
n   n n  n n  n n  ......n n 

n
je

1 1
= 1 when x is irrational.
iit

 nxdx  1 .
=
1+ 0 =


@

0
Thus, f(x)=0 when x is rational and 1 when x is irrational.
Hence, L = e–1.
Summation of infinite series using definite 1 2n r
integral as the limit of sum Example 10: Evaluate lim
n  n
 2 2


r 1 n  r
From the definition of definite integral, we have
Solution: On dividing numerator and denominator by n,
1 ψ(x)  r 
b

∑ f  n  = ∫ f (x )dx


lim 1 2n r
n →∞ n
 2 2

r = φ( x ) a we get lim
n  n
where, r 1 n  r

(i) Σ is replaced by ∫ sign, 1 2n r/n





r = lim
n  n
r 1 1  ( r / n )
2

(ii) is replaced by x,


n
1 Now we find
(iii) is replaced by dx,


n 1 2n
(iv) To obtain the limits of integration, we use a = lim  0 and b = lim 2

n  n n  n


φ( x ) ψ (x)
a = lim and b = lim 1 2n r/n x

2


n →∞ n n →∞ n lim dx
n  n 0
The limiting value is obtained by finding the definite integral. r 1 1  ( r / n ) 1 x2
2

Special case of definite integral as limit of sum
2

1 n
r
1 = 1+ x2
 f     f ( x )dx

lim 0
n  n r 1  n  0 = 5 −1 .


FREE BOOKS FOR JEE & NEET =>(@iitjeeadv)
1.102  
Differential Calculus for JEE Main and Advanced

Example 11: If A = [ai j ]n × n, where ai j = i100 + j100, then lim f ( x ) lim g ( x )


lim  = 1 2


n x a x a
  1m 2   2 m1 .
find lim
 a ii .
x a lim u ( x )
x a
lim v(u )
x a
m1 m2
i 1
n  101
n
n  [x 2 ] sec x 
Example 13: Evaluate lim  
 a ii  a11  a 22  ......  a nn x 0  (sin 2 x ) x x cot x 



Solution: We have 2




i 1
n  [x 2 ] sec x 
Now, lim  a ii = lim 
2 1100  2100  ......  n100  Solution: xlim 
0  (sin 2 x ) x
2

x cot x 



i 1 n  101 
n  101 n
n
 lim[ x 2 ] lim sec x 
1 =  x 0 x 0 
100
r
100 2  
1

= 2 x dx 


  n 
100 2 x 2

= 2 lim .  lim (sin x ) lim x cot x 


n  101  x 0 x 0 

n r 1 0
 0 1
Example 12: Evaluate the limit =  
 1 1



lim n ( ) (11 . 22 .... n n ) . 1 sin x n (1  x )
− 1+ 1
2
1
n
1
n2

x →∞ Example 14: Evaluate xlim
0 x 2 tan x x sec x


Solution: Taking the logarithm of the expression inside
ps


the limit, we find that it is Solution: 1 sin x n (1  x )
el
lim


x 0 x 2 tan x x sec x
eh

1 1 1 n
1 n
k k
 1   ln n  2
2 n
 k ln k  n  n ln  n  . sin x n (1  x )
je

n k 1 k 1
x x
iit

= lim
x 0 tan x

We can recognize this as the integral x ln x dx, Evaluating x sec x
@

this integral using integration by parts, we find that x x


1 1 1x 1 1 1
0 x ln dx  2 x 0 2 dx   4 .
1
2
ln x 0  =  


1 1
Therefore, the original limit is e–1/4. sin 2 x  x tan x 2
Example 15: Evaluate lim
x 0 cos ec3 x


Limit in Matrices and Determinants cot x

 f (x) g(x )  sin 2 x  x tan x 2


Let A =   then we have Solution: xlim
0
 u(x) v( x )  cos ec3 x


cot x
Here the limit of each of the functions does not exist
 lim f ( x ) lim g ( x )  simultaneously. Therefore, the above limit can be evaluated
lim A   
x a x a
 lim u ( x ) by finding limit of the expansion of the determinant.
x a lim v( x ) 

 x a x a 
∴ limit = lim (sin 2 x cot x + x tan x2 cosec3x)
Note that the limit exists if and only if the limits of each of

x→0
the functions exist simultaneously.
f (x) g(x )  sin 2 x x tan x 2 
Let ∆ = then we have
= lim   

x 0  tan x sin 3 x 
u ( x ) v( x )
 2 sin 2 x x tan x 2 
lim   lim (f(x)v(x) – g(x)u(x)).  
x a x a 2

= xlim  2x  x x  = 2 + 1 = 3.
0  tan x sin 3 x 

If the limits of each of the functions exist simultaneously then
the above limit can be also be evaluated as  x 
 x3 

FREE BOOKS FOR JEE & NEET =>(@iitjeeadv)


Limits 1.103

Concept Problems R


1. Define the function without limit
(iii) y = lim x 2 + a 2

y  lim (cos 2 n x ) .



a →0
n 

2. Define the following functions without limit: (iv) y  nlim

(arc cot nx )



(i) y  lim n 1  x n (x ≥ 0)
n  lim x 2 n ( x 2 + 1) + 2 x , find lim f(x).



3. Let f(x) = n x→1


x 1 + x 2n
(ii) y  lim (x ≥ 0)
x  1  xn


Practice Problems P

ps
tan  x 2  ( x  1) n sin x n
el
4
4. Define f (x)= lim
n  x  ( x  1)
2 n , and find 10. Evaluate lim
n 
 n  4k .
eh


lim f ( x ) . k 1
x→0
je

3
5. Sketch the graph of the following functions for n
2  2k 
11. Evaluate lim  n  n  1
iit

x≥0

n 
k 1
lim  1 
(a) φ(x) = n
@

x  1 x 
n


n
3 3i
n
lim log e (1+ x )
(b) Ψ(x)= n→∞
12. Find the limit lim
x →∞
∑n 1+
n

i =1


log e n
n  i2 π 
[ x x ] , x  0 13. Evaluate lim
i
∑ n 2 sin  n 2  .
 x →∞

6. Let f(x) =  t lim f(x)
, find x→0 i =1
 sin x 

 lim   ,x  0 1/ n
 t   x  lim  (2n )! 
14. Evaluate n n
 n !. n 

where [.] is G.I.F.
 
ax 2  bxy  cy 2  15. Evaluate


7. Evaluate lim  lim 2 
 y 0 dx  exy  fy 
2

x 0   lim e
 1/ k
(e1/ k ) 2 (e1/ k )3 (e1/ k ) k 
2 3  ...... 
k  2
k2 k2

 k k 


 ax 2  bxy  cy 2 

– lim  lim 2 
, ad ≠ 0, cf ≠ 0.
y 0 
2
  

 x 0 dx exy fy  2n
10n
16. Evaluate lim
n 
 ( n  2k ) 2 .

1 k 1
lim 1
then find the values of x.
8. If
 
x  n
sin 1 x 1

3n / 2
  k 
Simplify the function φ(x) = n
lim (2/π) arc tan(nx). 17. Evaluate lim
n 
 n sin  n 
.

9. k 1

FREE BOOKS FOR JEE & NEET =>(@iitjeeadv)
1.104  
Differential Calculus for JEE Main and Advanced

Target Problems for JEE Advanced

Problem 1: If α and β are the roots of the quadratic (cos ) x  (sin ) x




equation ax2 + bx + c = 0, then evaluate Problem 2: Evaluate lim
x2



x 2

1  cos(cx  bx  a ) 
 2
 Solution: Limit
lim  



x
1

 2(1  x ) 2


= lim (cos )  (sin )  cos   sin 
 x x 2 2


Solution: ax2 + bx + c = 0 has roots α and β, then x 2 x2


a b x 2 x 2
= lim cos ((cos )  1)  sin ((sin )  1)
2 2
 c  0

2
x2


x x x 2

1 1 Put x – 2 = t
i.e., cx2 + bx + a = 0 has roots and


α β

 cos α ) t − 1 ((sin α ) t − 1)
lim
= t →0 cos 2
α  +
 t →0
lim sin 2
α
 b a  1  1
 t  t
⇒ c  x2  x    c  x    x  
 c    

c
= cos2α ncosα + sin2α n(sinα).

ps
1  cos(cx 2  bx  a ) 
  xx  aa

el
lim   Problem 3: Evaluate lim
a x  aa


x a

x 1/  
 2(1  x ) 2


eh

enx  ena
x a
je

 2  cx 2  bx  a   lim
 sin   
Solution: Limit = x a (a x a  1)
(x  a )


aa
iit

  2  (x  a)
= lim  
@

(1  x )
e 

x 1/  2
  xnx
 eana
  = lim
  a na ( x  a )

x a a

 cx  bx  a 
sin 
2
 = lim 
eana e xnx  an a  1  ( xnx  ana )

= 2 a na .( xnx  ana )( x  a )
a
  x a

lim
x 1/  (1  x ) a a  xnx  ana 
= lim
c 1  x a a a .na .( x  a )

1 
sin   x    x   
2     put x – a = t

= lim

x 1/   1 ( t  a )n ( t  a )  ana
  x   = lim
  t 0 (na ) t

c 1  1  c 1 a (n ( t  a )  na )  t.n ( t  a )
sin   x    x    = lim
x   (na ) t

  t 0
2     
 . lim  
2
= lim
x 1/  c 1  1 x 1/  

x   x    t
2     n  1  
 a   lim n ( t  a )
= lim
t 0 t t 0 na

na  
c 1 1  
a
  
= 2  c  1 1 .
1.     1

 2     = + 1 = logae + 1.
na

FREE BOOKS FOR JEE & NEET =>(@iitjeeadv)
Limits 1.105

Problem 4: The integral value of n for which the Problem 6: Evaluate a, b, c and d, if




x3 lim .
cos 2 x  cos x  e x cos x  e x  x ( x 4 + ax 3 + 3x 2 + bx + 2
lim 2
x 4 − 2 x 3 − cx 2 + 3x − d ) = 4

x 0 xn
is a finite nonzero quantity.
Solution: Here, lim ( x 4 + ax 3 + 3x 2 + bx + 2



x 
Solution: The limit is equal to


− x 4 + 2x 3 − cx 2 + 3x − d ) = 4 (∞ − ∞ form)
   


1 2 4 6 2 4
lim n 1  x  x  x  ...  1 1  x  x  ..  Rationalizing

x →0 x  2 ! 4 ! 6 !   2 ! 4 ! 
(a  2) x 3  (3  c) x 2  (b  3) x  (2  d )
 x2 x3  x3  lim
 1  x    .....     x 
x 4  ax 3  3x 2  bx  2  x 4  2 x 3  cx 2  3x  d
 2! 3!  2 
   Since, limit is finite, so the degree of the numerator must be


 x2 x4 x6   x3 2x 5  x3 2. So, a – 2 = 0
       x  x    ..... 
2
i.e., a = 2.
 lim 
2 ! 4 ! 6 !   3! 5 !  2
x0 xn (3  c) x 2  (b  3) x  (2  d)
lim
 x3 x4 x5 x5  x3 x 
x 4  ax 3  3x 2  bx  2  x 4  2x 3  cx 2  3x  d 
     .....  
2 2 12 24 2
 lim   Dividing numerator and denominator by x2.

ps
x0 xn We get
el
= nonzero if n = 4.
(3  c)  (b  3) / x  (2  d ) / x 2

lim
eh

Problem 5: Evaluate x  a 3 b 2 2 c 3 d
1  2  3  4  1  2  3  4


lim 4 ( x  p)( x  q )( x  r )( x  s)  x
je

x x x x x x x x
x 

iit

3+ c
4 ( x  p)( x  q )( x  r )( x  s)  x ⇒ .
Solution: xlim
@


 2
3+ c


It is given that =4 ⇒ c=5
( x  p)( x  q )( x  r )( x  s)  x 4 2
= lim
( x  p)( x  q )( x  r )( x  s)  x )

x  ( 4
Hence, a = 2, c = 5 and b, d are real numbers.

.
1
( ( x + p)( x + q )( x + r )( x + s) + x ) 2
x  ln  x2 1  x 

Problem 7: Evaluate lim


x0 x3
a 4  b4
[using a – b =
(a  b)(a 2  b 2 )
] Solution: Put ln  x2 1  x = t  ...(1)




Ax 3  Bx 2  Cx  D
x2 +1 – x = e
t –t
lim or x2 +1 + x = e
 

( x  p)( x  q )( x  r )( x  s)  x
x 4

On subtraction, 2x = e–t – et

. 1
 e t  e t 

( ( x + p)( x + q )( x + r )( x + s) + x 2 )
or x =  


[where A = Σp, B = Σpq, C= Σpqr and D= Πp]  2 

B C D As x → 0, t → 0 from (1).
A  2  3

lim x x x e t  e t
x   p  q  r  s  t e t  e t  2t

4 1 lim
 x  1  x  1  x  1  x   1    lim 2 t 0
 e2 t  1 
3
     t 0 3 =
 e e 
t  t 3
2.t  
A 1     2 t 
= (1  1)(1  1)  4 (p  q  r  s) .  2 

FREE BOOKS FOR JEE & NEET =>(@iitjeeadv)
1.106  
Differential Calculus for JEE Main and Advanced

e t  e t  2t  e2 t  1  Problem 9: Find the value of



  lim  as lim  1
t 0 2t 3  2
 t 0 2 t   2
 x

2

(put t = 3y) lim  3 sin 1 x  tan 1 x  .


x 0  x 


e3 y  e 3 y  6 y
= lim
y 0 2.27 y3

3
Solution: sin–1x = x + x +
3 5 5 7 .....
x + x
e   
3



y
 e y  3 e y  e y  6 y 6 40 112
= lim

y 0 3 3 5 7
54 y
and tan–1x = x 
x x x
  .........


 y
   3 5 7
3
y
 e e 3(e y  e  y  2 y) 
  lim   
2
54 y3 54 y3  2
 x

y 0
(1∞ form)
2




 lim  sin 1 x  tan 1 x 


x 0  x 3 
  2y 3 
8 e 1  1 e y  e y  2 y  L 
( say )

= lim     2 2
y  0  54 2 y3  =
x 2  x 3
 x  tan 1 x  1
1

2y  9 lim sin
 

 e
x 0


or 
4 
 ⇒
8

4

1
. Now L = 2 lim

2 sin 1 x  tan 1 x  x 3 

27 9 9 27 6

5
ps x0 x
 x3 3 5 x3 x5 
n
r3  8 2  x   x  ..  x   .....   x 3
 r3  8
el
= 6 40 3 5
Problem 8: Evaluate lim 2 lim  



n 
eh

r 3 x0 x5
n
r3  8 1
je

Solution: lim 
=–

n  r3  8 2
iit


r 3
1
 r  2   r  2r  4 
@

n 2 Limit = = e–1/2.
= lim    2 

  e
r 3  r  2   r  2r  4 
n 

Problem 10: A sequence of numbers xn is determined


 r  2  n  r  2r  4 
n 2


= lim      2  x n 1  x n  2
n 
r 3  r  2  r 3  r  2r  4  by the equality x n  and the values x0 and x1.
2
 1 2 3 4 5 (n  5) (n  4) (n  3) (n  2)  Compute xn in terms of x0, x1 and n. Also prove that
= lim  ....  ×
n  5 6 7 8 9 (n  1) (n ) (n  1) (n  2) 
lim xn =
x 0 + 2 x1
19 28 39 
.
 (n 2  2n  4) ( n 2  3) ( n 2  2n  4)  n
   .... 2  2  2  3
 7 12 19
 ( n  6n  12) ( n  4n  7) (n  2n  4)  
x n 1  x n  2

 1.2.3.4 
(n 2  3)(n 2  2n  4)  Solution: x n 
= lim    2


n   ( n  1) n ( n  1)( n  2) 7 . 12 
 
x1 + x 0 x + x1 x + x2
2  (n 2  3)(n 2  2n  4) 
  x2 = ; x3 = 2 , x4 = 3 etc.
= lim  

7 n      2 2 2
 ( n 1) n ( n 1)( n 2 ) 

  3  2 4  
x 0 − x1
x2 – x1 =
 1  2  1  n  2  

2   n  n   2
= lim   x1 − x 2
7 n 
  1  1  1 1  1   1  2   ( x 0 − x1 )
x3 – x2 = =–
  n   n   n  

  2 22
2 (1  0)(1  0  0) 2 x 2 − x3 ( x 0 − x1 )
=  . x4 – x3 = =+
7 (1  0)1(1  0)(1  0) 7 23

2

FREE BOOKS FOR JEE & NEET =>(@iitjeeadv)


Limits 1.107

Problem 12: If α1, α2, ...., αn are the roots of equation


= – ( x 0 − x1 )



x5 – x4 = xn + nax – b = 0, show that (α1 – α2)(α1 – α3) ....(α1 – αn) =

24 n(α1n–1 + a).
  



− x n −1
= (–1)n · ( x 0 − x1 ) Solution: Since α1, α2, ..., αn are the roots of equation
x
xn – xn – 1 = n − 2




2 2n xn + nax – b = 0 we have
add ——————————————————— xn + nax – b = (x – a1)(x – a2)...(x–an)


xn – x1

x 0 − x1 x n  nax  b
 1 1 1 1  ⇒ = (x – α2)(x – α3)....(x – αn)
1  2  2  3  ......  (1) n  2 
n
x  1


=
 2 2 2 

2
lim x 0  x1  1 
n (xn – x1) = lim   x n  nax  b lim [(x–α )(x–α )...(x–α )]
 1  (1 2)  ⇒ lim

n 2 = x
x 1 x  1 2 3 n

lim x – x = x 0 − x1 × 2
nx n 1  na
n n 1

2 3
lim = (α1 – α2)(α1 – α3)...(α1 – αn)
lim x = x 0 − x1 + x = x 0 + 2 x1

x  1 1
n n 1 (using L’ Hospital’s rule on L.H.S.)

3 3

⇒ (α1 – α2)(α1 – α3)....(α1 – αn) = na1n–1 + na.
Problem 11: For n ∈ N, let xn be defined as

ps


n xn
 1
1  n  = e, then find nlim nx n 1  (n  1) x n  1
el
xn .

  Problem 13: Find lim

eh

(e x  e) sin x


n xn x 1
Solution: We have  1  
1 if n = 100.
= e ...(1)
je



 n
nx n ( x  1)  ( x n  1)
iit

Taking log on both sides of equation (1), we get Solution: l = lim


x 1 (e x  e) sin x


@

(n + xn) ln  1   = 1
1

  x = 1 + h so that as x → 1, h → 0


n Put


⇒ n + xn =  1
1
∴ l = – lim

h · n(1  h ) n  (1  h ) n  1 

ln 1+  e(e  1) sin h
h


h 0
 n

 
1
⇒ l = – lim  n · h 1  n C1 h  n C2 h 2  n C3 h 3  ...
xn =  1 – n ...(1) x 1 

l n 1 +


 
n
n +1 
 1  n C1 h  n C 2 h 2  n C3 h 3  ....  1 
 

Let =u ⇒ nu = n + 1



n 1
.
 e  1   sin h 

1 h
⇒ n= e (h 2 )   
u −1 

 h   h 
lim x n = lim  1  1 
∴ n  n 2  n C2  2n 2  n (n  1) 
u1   n u u 1 

=– =–  
e  2e 

0
(u  1)  ln u
= lim  0  form
(u  1) ln u
  n2  n n (n 1)


u1
=– =–
2(e) 2(e)

1 1
1
u u2 1
= lim u  1 = lim = .  5050 
u1 1 1

u1 2
 ln u  If n = 100 then l = –  .
u u2 u  e 

FREE BOOKS FOR JEE & NEET =>(@iitjeeadv)


1.108  
Differential Calculus for JEE Main and Advanced

Problem 14: Let f(n , θ) 1 a 


= lim   n 


 2      2   n   n ! n! 



= 1  tan  1  tan 2 2  1  tan 3  ....
 2  2   2   1 1 a 
= lim    n 1  [using (1)]
θ  


n   n ! ( n 1)! ( n 1)! 
to n factors . Show that nlim f (n , θ) = .
 tan θ  1 1 1 1 1 a 
= lim     ......    1
θ cos θ n   n ! (n  1)! (n  2)! ( 2)! 1 ! 1 ! 


Solution: 1 − tan2
cos 2 θ
= ; {a1 = 1 as given}


2 2
cos θ
 1 1 1 1 1 1
    ......    =
θ 2 = nlim
  n ! ( n  1)! ( n  2)! ( 2)! 1 ! 1
e
1 − tan 2 =


2
2 θ
and so on.

2 cos 2
2  1 1 1 
Hence f(n , θ) as, e  1     ........ 


θ  1! 2 ! 3 ! 
cos θ θ
cos cos n −1
θ 2 ...... 2 lim x , when x 2 = a + x
= Problem 16: Solve n
cos 2 . 2 θ θ n n n–1
and



2 cos cos 2 n
2 2
2 x0 = a.
cos θ 1 Solution: Here, x0 = a , x1 = a + a which shows
.


θ
= cos cos θ θ θ x1 > x0
....cos n cos n
ps

2
2 2 2
2 2 ∴ assuming xn > xn – 1 ⇒ x n−1 – a < xn – 1


el
θ θ θ θ   
But cos
. cos 2 . cos 3 ...... cos n  4a  1  1  4a  1  1 
⇒  x n 1    x n 1   <0
eh

2 2 2 2

 2 
 2 

sin θ
je

= n 4a + 1 + 1
2 sin θn


iit

2 xn – 1 < .
θ



2
Hence f(n , θ) = cot θ . 2n tan n
@

2 ∴ lim x = lim x = l
 tan n n n– 1 n n

2
= cot θ .

  ⇒ l2 – l – a = 0





2n
1  1  4a
⇒ l= , since l ≥ 0
Now lim f(n, θ) = θ cot θ.

n 2
Problem 15: Evaluate ∴ l = 1 + 1 + 4a




2
 a 1  a2 1   an 1 
lim  1   ........  , 1  4a  1
n  lim x n 
 a1   a 2   an 

or .

n  2
where a1 = 1 and an = n (1 + an – 1) ∀ n ≥ 2.

 an 1  a+b
 a1  1   a 2  1  Problem 17: If a1 = ab1 , b1 = ,
Solution: nlim    ......  



2
 a1   a 2   an 


a1 + b1
a a2 = a1b 2 , b2 = ...
We know, an – 1 + 1 = n ...(1) 2


n
and so on for (a > b > 0) then prove that
 a  a  a   a  1
lim  2   3   4  .....  n 1 
n   2   3   4   n  1  a1.a 2 .....a n a 2  b2
lim a n  lim b n 

n  n   a 2  b2 .
1 an tan 1  

a
= lim n 1 = lim [using (1)]  b 
n  ( n  1)!  


n  n !

FREE BOOKS FOR JEE & NEET =>(@iitjeeadv)


Limits 1.109

θ 
Solution: Put b = a cos θ , we get b1= a cos2 and 2 sin n 1


θ 2  π
= 2θ = 2 · π = .
2
a1 = a cos l = 2(n + 1) 2 sin = 

2 n1 3
2 6
θ θ θ 2n 1
b2 = a cos cos2 and a2 = a cos cos θ

2 4 2 4 1n
Similarly we get  3n
Cn 
  a
Problem 19: Given nlim
  Cn 
= where a and b
θ θ θ 
2n



an = a cos cos ...... cos and b

2 4 2n are relatively prime, find the value of (a + b).
θ θ θ
bn = a cos cos .... cos2 n 3n
Cn (3n )! n !n !

2 4 2 Solution: Consider = ·
2n n !(2n )! (2n )!



a sin θ Cn
⇒ an = n and
2 sin θ / 2n 3n (3n  1)......(2n  1)(2n )!


=
(2n )!(2n )(2n  1)....(2n  1)n !


bn = a sin θ cos θ / 2
n

2n sin θ / 2n ( 2n + 1)( 2n + 2)( 2n + 3).....( 2n + n )
=
a sin  a sin  . ( n + 1)( n + 2)( n + 3)..........( n + n )

Now, lim a n  lim 
n  n  2 sin  / 2
n n
 1n
 
Let P = lim 1  n  1  n  ...... 1  n  
ps
n  n 1 n2
 nn    
a (a 2  b 2 ) a 2  b2
lim a n  
el
1
⇒ n  a cos (b / a )  a 2  b2 . Taking logarithm, ln P
eh

tan 1  

 b 
  1  n   n   n 
lim ln 1   ln 1    ......  ln  1  n  n  
je

= n n   n  1   n  2   
and lim b n  lim a sin  cos  / 2  a sin  .
n
iit

n  n n  n
2 sin  / 2  1  n  1  1 
@

where Tr = ln 1+  = ln  1+
a 2  b2 n  n+r  n  1+( r n ) 
lim a n  lim b n 
⇒ n  n   a 2  b2 . 1 n  1 
tan 1   ∴ ln P = lim  ln 1  

n  n r 1  1  (r n ) 

 b 
 
1
π
1
 1   x+2 
Problem 18: Let x0 = 2 cos and xn = 2  x n 1 , n = 1,
= ∫ ln 1+ 1+ x  dx = ∫ ln  x+1  dx



6 0 0
( n 1)
2, 3, ........ find lim 2 · 2  xn . 1 1
n  = ∫ ln(x + 2)dx − ∫ ln(x +1)dx

π 0 0
Solution: Let = θ.


6 = (x + 2) ln (x + 2) − (x + 2)
1
0

 π θ
x1 = 2 + x0 = 2  2 cos   = 2 cos = 2 cos
6 12 2 – (x +1) ln (x +1) − (x +1) 1

0
  π θ
x2 = 2 + x1 = 2  2 cos   = 2cos = 2 cos
22 = [3 ln 3 – 3]– [2 ln 2 – 2]– [(2 ln 2– 2) – (0 – 1)]
 
12 24

= [ln 27 – 3] – [ln 4 – 2] – [ln 4 – 1]
   π θ

x3 = 2 + x2 = 2  2 cos   = 2 cos = 2 cos = [ln 27 – 1] – [2 ln 4 – 1]

 
24 48 23 = ln 27 – ln 16

27 27
∴ ⇒ P=



ln P = ln
θ  



xn = 2 cos ; 2 − xn = 2  2 cos = 2 sin 16 16
2n 2n 2n1 ∴ a = 27 ; b = 16 ⇒ a + b = 43.




FREE BOOKS FOR JEE & NEET =>(@iitjeeadv)
1.110  
Differential Calculus for JEE Main and Advanced

Alternative:
2n 3  3n 2  n n 1  1 
3n   1

16 32  2n  1 
Cn (3n )! n !n ! =


2n
= · 24

Cn n !(2n )! (2n )!
n 3 n 2 5n 1  n 
 (3n )(3n  1)(3n  2).............  3n  (3n  1)   =   
12 8 48 16  2n  1 
=



.  n (n  1)(n  2).............  n  (n  1)  
n 3 n 2 5n f ( n )

1
= + + + (given)


. A B C D

[2n (2n − 1)(2n − 2)........2n − (2n − 1)]2
So A = 12, B = 8, C = 48 ; D = 16


3n n n
=  
(3n ) (n ) 27 Hence, A + B + C + D = 84

 16 

4n
( 2n ) Problem 21: Evaluate



1n
 27  n  27 1  n 1n  2n  n 2n  3n  n 3n  4n  ......  n (m  1) n  m n 
∴ lim   = .  

n   16 
lim lim  

  16 m  n 

m2


Solution:
n
r4


Problem 20: Let  (2r  1)(2r  1) 1  n 1n  2n  n 2n  3n  n 3n  4n  ......  n (m  1) n  m n





r 1 lim lim  
m  n  m2
3 2  
= n + n + 5n + f (n ) , (A, B, C, D,∈ N)
ps
 

A B C D 1
n
2
n
 m 1 
n
el
 1  2 n    1  3n    1  ......  m n   1 
where f (n) is the ratio of two linear polynomials such that  2 3  m  
= lim  lim
eh


1 m  n 
 m2 
lim f (n )  . Find the value of (A+B+C+D).  
je

n 2  
r4
iit

Solution: Tr = 1  2  3  4  ....  m
(2r  1)(2r  1) =


lim
@

m2

m 

1  16r  1  1  4
  1 n n
=   2
16  (2r  1)(2r  1)      0 as n  ;    0 as n  ;

  2  3


1  (4r 2  1)(4r 2  1)  1  n 
Tr =    m 1 
16  4r 2  1 .....;    0 as n   


 m  
1  2 1 
1 1
=  (4r  1)   m(m 1)
= lim = 1 = 1.
2  m 
16  (2r  1)(2r  1)  lim

m  2 m 


2m 2
1  2 1  (2r  1)  (2r  1)    (1  [ x ])1/{x} 
1/{x}
=  (4r  1)    Problem 22: Evaluate lim  
2  (2r  1)(2r  1)  

16  x 0 


 e 
1  2 1 1 1  if it exist, where {x} denotes the fractional part of x.
=  4r  1    
16  2  (2r  1) (2r  1)   lim f(x)

Solution: L.H.L. = x 0



1/{0  h}
1 1 1  1 1   1  {0  h})1/{0  h} 
∴ Sn =  r 2  16 1  32   2r  1  2r  1  = lim f(0 – h) = lim  

h 0

4 h→0  e 
n (n + 1)(2n + 1) n 1/(1 h )
= +  (1  1  h )1/(1 h )  2
= lim   =  .

4· 6 16 h 0 e

 e 
1  1 1 1  1 1  lim f(0 + h)
  1        .....     R.H.L. = lim f(x) = h→0
32  3 3 5  2 n  1 2 n  1   x0 

FREE BOOKS FOR JEE & NEET =>(@iitjeeadv)


Limits 1.111

1/{h} 1/ h We choose x and y such that x + ny = u and x – ny = v


 (1  {h})1/{h}   (1  h )1/ h 
= lim   = lim   where u, v ∈ R and n ∈ N.
h 0  h 0 

 e   e  2
(3) becomes, |f(u) – f(v)| ≤ for arbitrary n ∈ N
=
1
lim
h →0
((1  h ) 1/ h
 e) 3n
ee 2


h i.e., as n → ∞ ⇒ |f(u) – f(v)| ≤ lim


n  3n
1  e(1/ h )ln(1 h )  e 
=
ee
lim
h →0   ⇒ |f(u) – f(v)| ≤ 0 ⇒ f(u) = f(v)


 h 


Hence, f is constant.
 ln(1 h )  h  Problem 24: The function un takes on the following
e h 1



= e lim
h →0   1 1 1

 h  values : u1 = , u2 = + .....,
  4 4 10
ln( 1+ h ) 1 1 1
(e h −1)  ln(1+ h ) − h  un = + 2 + ........ n ....
lim .  3 +1 3 +1 3 +1


= h →0
 ln(1+ h ) − h   h2 ...(1)
e  


h 1
Prove that lim u n  .
ln(1  h )  h n  2
Now let P = lim ...(2) 1 1 1 1 1 1
Solution: We have < ;    ; .....

h 0 h2  

replacing h by –h then 4 3 4 10 3 9
1 1 

n
ln(1  h )  h 1    
P = lim ...(3)
ps 3   3  
1 1 1
h2 Thus, un < + 2 + ...... n < 


h 0
el
3 3 3 1
adding (2) and (3), 1
eh

3
1   1  
n n
ln(1  h 2 ) ln(1  h 2 )  1 3 
1  3  . 1  
je

2P = lim = – lim = –1 un < <


  2   3  
  
h 0 (h 2 ) 2


h 0 h2 
iit

1   1 n 
1     = 1 .
@

∴ P=– from (1), lim u < lim


n 2  
 3 


n n
  
2
R.H.L. = e1.(–1/2) = e–1/2.
e x sin( 2007 ) x  1
L.H.L. ≠ R.H.L. Hence P does not exist. Problem 25: Let L = lim and
x 0 x ln(1  x )


Problem 23: Suppose that function f : R → R satisfies 
M = lim x  2 x   3
x 3  x 2  1  3 x 3  x 2  1  


n
the inequality,  3 {f (x  ky)  f (x  ky)} ≤ 1
k x  
k 1 then find the value of LM.
for every positive integer n and for all x, y ∈ R.
e x sin( 2007 ) x  1 x sin( 2007 x )
Prove that f is a constant function. Solution: L = lim ·
x ln(1  x )


x 0 x sin( 2007 x )
Solution: Replacing n by (n – 1) in,


sin(2007 x ) (2007 x )
n = lim · = 2007.
 3k {f (x  ky)  f (x  ky)} ≤1 ...(1) x0 (2007 x ) ln(1  x )


k 1
lim  x [( x 3  x 2  1)1 3  ( x 3  x 2  1)1 3  2 x ]
M = x
we get:
n 1   13 13 
1 1   1 1 
 3k {f (x  ky)  f (x  ky)} ≤1 ...(2) = lim  x  x 1  
x x 3 
 x 1   3   2x 


k 1
x    x x  
Subtracting (1) and (2) we get, 1
Put x =

|3n {f (x + ny) – f(x – ny)}| ≤ 2 t

2 [(1  t  t 3 )1/ 3  (1  t  t 3 )1/ 3  2]
⇒ |{f (x + ny) – f(x – ny)}| ≤ ...(3) = lim


n
3 t 0 t2

FREE BOOKS FOR JEE & NEET =>(@iitjeeadv)


1.112  
Differential Calculus for JEE Main and Advanced

1  1 x x
= lim  2 
1  ( t  t 3 )  ( t  t 3 )2 Problem 27: Let f (x) = tan secx + tan .
t 0 t  3




2 22
1 1  x x x x x
1 ( t  t 3 )  ( t  t 3 )2  2  sec + tan sec + ..... + tan sec
3 9  2 23 22 2n 2n −1
2 2    x
t  ...... and g (x) = f (x) + tan where x   ,  and
=
lim 9 2  2 2
n


2
t 0 t n N. Evaluate the following limits:
2 1 1
=
9  g( x )  x (b) lim  g ( x )  x
2
2 (a)
∴ lim    






LM = × 2007 = 2 × 223 = 446. x0  x  x0  x 
9

1
  g( x )  x 3

Problem 26: Let an = 2 cos  1 then show that lim (c) lim  



2n


n x0  x 
2 cos   1  x
(a1a2a3....an–1an) = , θ ∈ R. sin  x   x
3 sin
 2 2
Solution: Consider an · a n–1 · a n–2 ............a2 · a 1 Solution: T1 = x = x




cos cos x cos cos x
     
ps 2 2
=  2 cos n  1  2 cos n1  1
   

2 2 x
el
= tanx – tan

     2
eh

..........  2 cos  1  2 cos  1 x


   x

2 2
2
je

Similarly, T2 = tan – tan 2



       2 2
iit

=  2 cos  1  2 cos n  1  2 cos n 1  1 x


 2n    x

2 2 T3 = tan – tan
@

22

1 23
       
.......  2 cos 2  1  2 cos  1   



 2  2   2 cos n  1 x x
 2  Tn = tan n −1 –


     2 2n
Now  2 cos n  1  2 cos n  1
_________________________

 2  2 
x
f (x) = tanx – tan
 
 

2n
= 4 cos n  1 = 2  1  cos n 1   1
2
x
 2  ⇒ f (x) + tan = g (x) = tan x

2

2n

= 2 cos +1 1

Now (a) L = lim  tan x 


n1

2 x

x0  x 
     
Similarly,  2 cos n 1  1  2 cos n1  1
 2   2  1  tan x 
x
tan x  x
Limit
= x  x 
= x 0
x2
e

e
 2     
=  4 cos n 1  – 1 = 2  cos n  2  1 – 1 tan x  x
   

2 2 Consider l = lim
x2


x 0
2 cos 1
2n 2 tan x  x
= = lim lim x  0 ⇒ L = e0 = 1.

x3

x 0 x 0
n
(2 cos   1)
Hence  ai      tan x 
1/ x 2
 tan x  1
i 1  1 2
 2 cos n  1 x 0 
(b) = lim
lim    x x
  e


2 x0  x 
n
2 cos   1
∴ lim
n 
 ai  . = limit
tan x − x
= e1/3.

3 x→ 0
x3

i 1 e

FREE BOOKS FOR JEE & NEET =>(@iitjeeadv)


Limits 1.113

1 4  3a n 1 4  3a n
 tan x  x 3

= el ∴ an +2 – an +1 = 
(c) lim  3  2a n 1 3  2a n
x 




x0 
(4  3a n 1 )(3  2a n )  (4  3a n )(3  2a n 1 )
1  tan x  =
1 
limit
x 3  x (3  2a n 1 )(3  2a n )


where l = x 0




tan x  x tan x  x 1 a n 1  a n
= lim = (3  2a )(3  2a )  0
lim
= x . = dne.
4 x3 x {∵ an + 1 > an}

x 0 x 0
n 1



n
⇒ limit does not exist.
∴ an+2 – an+1 > 0


n
x an+2 > an + 1 whenever an + 1 > an
 3n 1 sin3 3n


Problem 28: Let f (x) = lim and ∴ The sequence of values an is increasing and since
n 




n 1 a1 = 1, an > 0 for all n,


g (x) = x – 4 f (x). Evaluate lim 1  g ( x ) 
cot x
. Now let, l = nlim a n  lim a n 1
x 0  n 

Solution: Using sin 3θ = 3 sinθ – 4 sin3θ 4  3a n


lim a n 1  lim
3  2a n



n  n 


x 1  x 
T1 = sin3 =  3 sin  sin x  4  3l
4   ⇒


3 3 lim
l = n

3  2l
x 3  x x ⇒ 3l + 2l2 = 4 + 3l
T2 = 3sin3 2 =  3 sin 2  sin 

3 4  3

3 l2 = 2
ps

and so on........ ∴ l= .

el

1 x  Problem 30: Find the values of a, b and c so that
3 sin  sin x 



eh

T1 = 
4 ae x  b cos x  ce  x
3  2


lim

x 0 x sin x
je

1 2 x x
3 sin 2  3 sin  x
 Solution: We have lim ae  b cos x  ce  2 ...(1)
x
T2 =
iit

4 3

3


x 0 x sin x
@

1 n x n 1 x  As x → 0, numerator → a – b + c
Tn =  3 sin n  3 sin n 1  and denominator → 0
4 3 

3
Since limit is finite a – b + c = 0
1 n x  or b = a + c ...(2)
∴ f (x) = lim  3 sin n  sin x 


4 From (1) and (2)
n  

3
ae x  (a  c) cos x  ce  x
 x  lim 2
x sin n

1   x 0 x sin x
=  lim
3  sin x  = 1 (x – sin x)
4  n  x 

4 ...(3)
  ⇒
 3n 
a (e x  1) c(e  x  1)
g (x) = x – 4 f (x) = sin x 
x (  x ) (a  c) sin 2 x
⇒ lim  lim 2
Now lim (1  sin x )
cot x lim (cot x )(sin x )
= =e x 0 sin x x 0 (1  cos x ) x sin x
x 0 e x→0

4 + 3a n a (e x  1) c(e  x  1)
Problem 29: If an = 1 and an + 1 =, n ≥ 1, 
x (a  c) sin 2 x ...(4)
3 + 2a n ⇒ lim x  lim 2



x 0 sin x x 0 (1  cos x ) x
then show that an+2 > an + 1 and if an has a limit l as n → ∞,
Since limit is finite, as x → 0,

then evaluate nlim an .
 a (e x − 1) c(e − x − 1)
− = a – c must be 0
Solution: a1 = 1 x (− x )



43 7 ∴ a–c=0 ⇒a=c ...(5)
and a2 =  1



3 2 5
a (e x  e  x  2) 2a sin x
∴ a2 > a1 lim  lim 2
x 0 (1  cos x ) x

x 0 x sin x
Assuming an +1 > an


FREE BOOKS FOR JEE & NEET =>(@iitjeeadv)
1.114 Differential Calculus for JEE Main and Advanced

From (4), c = 1 and from (2), b = 1 + 1 = 2.


a (e x  1) 2 2a sin x


⇒ lim  lim 2
x 0 (1  cos x ) x

x 0 e x x sin x Problem 31: Let a1, a2, ....., an be sequence of real



 ex  1 
2 numbers with an + 1 = an + 1 + a 2n and a0 = 0. Prove that


a    a  4
x  lim  n  
⇒ lim   lim
2a sin x
2 n   2 n 1  
x 0 (1  cos x ) x .

x 0 x  sin x 
e . 
 x  Solution: Here, an + 1 = an + 1 + a 2n ,



a (1) 2 2a.1 let an = cot (αn) ⇒ an + 1 = cot(αn) + cosec(αn) ⇒ an + 1
⇒  2

1.1 2
∴ a=1 cos( n )  1 2 cos 2 ( n / 2)  
  cot  n  = 1

From (5), c = 1 sin( n ) 2 sin( n / 2) cos( n / 2)  2 
And from (2), b = 1 + 1 = 2

Alternative : Putting n = 1, a1 = cot (α1) and a1 = a0 +
Here we use L’ Hospital’s rule
ae x  b cos x  ce  x = 1 + a 02 = 1
We have lim 2 ...(1) π
x 0

x sin x
As x → 0, numerator → a – b + c and ⇒ cot(α1) = 1 or α1 = 4

denominator → 0 ∴ a–b+c=0   
Again, a2 = cot  1  = cot  


⇒ b=a+c ...(2) 8
 2 


From (1) and (2)
ps
ae x  (a  c) cos x  ce  x  0     
a3 = cot  2  = cot  
el
lim  2  form   4.22 
 2 

x 0 x sin x  0
eh

    

Using L’ Hospital’s rule
a4 = cot  3  = cot   .........
ae x  (a  c) sin x  ce  x
je

 2   4.23 

lim 2 ...(3)
( x cos x  sin x )   
iit


x 0
As x → 0, numerator → a – b + c an cot  .
 4.2n 1 

@

and denominator → 0
Since limit is finite a – c = 0 Put =x

⇒ a=c

...(4)
  


From (3) and (4) cot  n 1 
 an 
ae x  2a sin x  ae  x  0 Hence, lim   = lim  4.2 
2 n   2 n 1 

lim
x cos x  sin x  form 0  n  2n 1
 


x 0
Again using L’ Hospital’s rule
ae x  2a cos x  ae  x =
2

lim
x 0  x sin x  cos x  cos x

a  2a  a  a  4
⇒ 2 ∴ lim  nn1   .
0 11
 

n   2

∴ a=1

Things to Remember
1. Theorems on Limits (iv) xlim [f(x) . g(x)] = m

Let f and g be two functions such that →a


lim f(x) = , lim g(x) = m. Then
x →a x →a (v) lim  [f(x) / g(x)] =  / m, (m ≠ 0)

x a

(i) lim k(f(x)) = k., (vi) For any positive integer n,
x →a


(ii) xlim
→a
[f(x) + g(x)] =  + m lim n f ( x ) = n lim f ( x ), provided lim f ( x ) > 0 when
x →a x →c
x →a


(iii) lim [f(x) – g(x)] =  – m n is even.
x →a


FREE BOOKS FOR JEE & NEET =>(@iitjeeadv)
Limits 1.115

 
m/n 5.
Also lim  f ( x ) m / n  lim f ( x )


where m/n is a rational lim sin x = 1 = lim tan x
(i)

x a x a x→0


x→0
number and the limit on the right exists. x x
(vii) If lim f ( x )  b and g(x) is continuous at x = b, then −1
lim tan x = sin −1 x
x a

 

= x→0
x x


lim g (f ( x ))  g lim f ( x ) = g(b).
x a x a
a x −1
(ii) = ln a (a > 0).
(viii) If , with f(x)¹ b for every x in some x



neighbourhood of 'a' and if lim g ( x )  c , then lim ln(1+ x ) = 1
x b (iii) x→0
lim g  f ( x )   c . (This theorem is sometimes useful if g is


x
x a

log a (1+ x )
discontinuous at b. lim = loga e, ( a > 0, a ≠ 1)


x→0 x
(ix) Domination Law : If two function f(x) and g(x) satisfy
(1  x ) n  1

the inequality f(x) < g(x) for all the values of x belonging (iv) lim =n


to a neighbourhood of a point a except possibly a then x 0 x
lim f ( x )  lim g ( x ) provided that the limits of
lim x  a
n n
x a x a
 n a n 1 .

both functions, as x → a, exist.
x →a
x a

2. For all elementary functions, limit at any point in the
lim xnx  0

domain is equal to the function's value at that point. (v) x 0

3. The important indeterminate forms are :
ps 1 x
 1

∞ (vi) l im (1  x ) x  l im 1 
0
  e
∞ , 0 × ∞, ∞ − ∞, 0º, ∞º and 1 x 
, ∞
el
0 x 0 x  

eh

4. Following are some of the frequently used series


a 0 x m  a1x m 1  ....

expansions :
6. l im
je

x 1n a x 2 1n 2 a x 3 1n 3a b0 x n  b1x n 1  ...

x 
(i) a x  1     .........a  0
iit

1! 2! 3!
 a0
 b , when m  n
@

x x 2 x3
(ii) e  1     ............
x
 0

1! 2 ! 3!
 0, when m  n
x 2 x3 x 4 
(iii) ln (1+x) = x     .........for  1  x  1 =  a0
 , when m  n 0

2 3 4 and

b0
x3 x5 x7 
(iv) sin x  x     .......  a0
 , when m  n 0
3! 5 ! 7 ! and
x2 x4 x6  b0
(v) cos x  1     ......

2! 4! 6! sin x cos x
x 3 2x 5 7. (i) lim  lim 0


(vi) tan x = x + + + ........ x  x x  x

3 15 (ii) In fact, if n is a positive integer then


x3 x5 x7 ex
(vii) tan–1x = x     ....... lim n   , lim x n e x  0 .


3 5 7 x 


x  x

12 3 12.32 5 12.32.52 7 (iii) For sufficiently large values of x we have


(viii)sin-1x = x + 3! x + 5! x + 7 ! x + .......


logax << xp << ax << x! where


a>1 p > 0 a > 1 x ∈ N.

x 2 5x 4 61x 6
(ix) sec x = 1 + + + + ...... (iv) lim log a x = 0 , for a > 1.


2! 4! 6! x


x
nx x2 a
n = 0 for a > 0,
(x) (1 + x)n = 1 +  n (n  1) +.... for –1< x< 1 (v) lim


1 2 n  a n

 1 11  n
(xi) (1 + x)1/x = e 1  x  x 2  .....  (vi) lim a  0
 


2 24 n  n !

FREE BOOKS FOR JEE & NEET =>(@iitjeeadv)


1.116 Differential Calculus for JEE Main and Advanced

8. The line x = a is called a vertical asymptote of the curve containing a, except possibly at a.

y = f(x) if atleast one of the following statements is true: Suppose that
lim f(x) = ∞ lim f(x) = ∞ lim f(x) = ∞


x →a x →a x a  lim f(x) = 0 and lim g(x) = 0


x →a x →a


lim f(x) = – ∞ lim f(x) = – ∞ lim f(x) = – ∞ or that f(x) = ± ∞ and lim g(x) = ± ∞.


x →a x →a x a 


x →a
9. The line y = L is called a horizontal asymptote of the (In other words, we have an indeterminate form of type

curve y = f(x) if either


0 ∞
lim f(x) = L or lim f(x) = L or ).
x 


x 0
 0, 0  a  1 f (x)
Then lim = lim
f '( x )
, provided the limit on
 1, g( x ) x →a
x →a g '( x )


a 1
10. lim a x  
 the right side exists or is ∞ or – ∞.
 , a 1

x 
15. Leibnitz’s formula for differentiation of integrals
dne a0 d 
v( x ) 
  f ( t )dt  = f(v(x)) dv – f(u(x)) dv .
11. If lim f ( x ) = A > 0 and lim f ( x ) = B, a finite quantity dx  u ( x ) 


du dx
 

x→a x→a

then lim  f ( x ) 
g (x)
A . B
 0, if −1 < x < 1
xa 2n 
16. lim x  1, if x = ±1
Also, lim  f ( x ) 
g (x) lim g ( x ) n ( f ( x ))
n 
e x a
∞, if x < −1 or, x > 1
xa 

ps
12. Let lim f (x) = 1 and lim g(x) = ∞ then 1 (x)  r 
b

 f     f (x)dx


x →a x →a 17. lim
el
r  ( x )  n  a
n  n

lim (f (x) ) g (x) f ( x ) − 1
g (x) lim
=e
eh

( x ) (x )
x →a

x →a
where, a = lim and b = lim .
13. Sandwich Theorem : Suppose that g(x) ≤ f(x) ≤ h(x) for n  n n  n
je


all x in some open interval containing c, except possibly
iit

at x = c itself.  f (x) g(x ) 


18. Let A =   then we have
If lim g(x) = lim h(c) = l, then lim f(x) = l.  u(x) v( x ) 
@


x →c x →c x →c

14. L'Hospital's rule: Suppose f and g are differentiable  lim f ( x ) lim g ( x ) 

lim A   
x a x a
on an open interval containing a, except possibly at a
 lim u ( x ) lim v( x ) 

x a
itself. Assume that g′(x) ≠ 0 for all x in the open interval  x a x a 

Objective Exercises
Single Correct Answer Type

 sin x  (B) 1



1. The value of lim   , (where [.] denotes greatest (C) a cannot be determined


x 0  x 

  (D) none of these


integer function) is
esin h − (1 + sin h )
(A) 0 (B) does not exists lim
(tan (sin h ))
4. 2 is



h →0 −1
(C) –1 (D) 1





 x2 
2. The value of lim   , (where [.] denotes the (A) 1 (B) 2



x 0  sin x tan x 

  (C)
1
(D) none of these




greatest integer function) 2
(A) 0 (B) 1
  · (1 + a
x b
lim 1  a 2





(C) does not exist (D) none of these 5. 2 x is (a, b R)
x  )





3. If lim ( x 4  x 2  1  ax 2  a ) exists then a is equal to (A) √b (B) b
x 



(A) –1 (C) b2 (D) none of these






FREE BOOKS FOR JEE & NEET =>(@iitjeeadv)
Limits 1.117

 2x 2  1  2  tan x  sin x   x 3
6. lim x 2  tan 1 2  tan 1 2  is lim
14. x→0 lim is equal to
  

x 


x 2 x0 x5
3 3 (A) 1/4 (B) 1/2
(B) −






(A) (C) 1/3 (D) None of these
5


5





5 5
(C) (D) − lim 1  cos x  2 sin x  sin 3 x  x 2  3x 4




3 3 15. x→0 equals
tan 3 x  6 sin 2 x  x  5x 3


7. If f(x) = 0 be a quadratic equation such that

 32 f (x) (A) 1 (B) 2
f(–π)= f(π) = 0 and f     , then lim





x  sin(sin x ) (C) 3 (D) 4
 
2 4





is equal to lim log cos x
16. x→0 is equal to
(A) 0 (B) π p
cos x − cos x
q






(C) 2π (D) None of these
q−p




pq
(A) (B)
20
q+p pq





8. lim  cos 2 n ( x  10) is equal to
n 


x 1 pq pq
(A) 0 (B) 1 (C) (D)
q−p p−q







(C) 19 (D) 20




 3x  17. The limit
1  sin   

 1  1  1   1 
1  5   1  2   1  4  ....  1  2 
9.  1  x 2  is equal to lim

lim n    5  5  

n

1  cos x 5 
x 1
ps
is equal to
(A) 0 (B) 1

el
(A) 0 (B) 5/4



(C) 2 (D) None of these




(C) 4/5 (D) 1/5
eh




10. If f : (1, 2) → R satisfies the inequality




1/ x
 xn ( 2x 1)
 (2x  1) x sin x 

cos(2 x  4)  33 x 2 | 4x  8 | lim  e
je

 f (x)  , ∀ x ∈ (1, 2). 18. The value of x→0   is


e xnx

x2  

iit

2
equal to
Then lim f ( x ) is
@

x  2 (A) e (B) 1 n2







16(A) e


(B)
cannot be determined from the given information (C) en2 (D) None of these






1/ n
(C)
–16  n! 


(D)
does not exist 19. lim   (m ∈ N) is equal to
n  n



 2  (mn ) 
 
11. lim (1  x ) x  (where {x} denotes the fractional part of (A) 1/em (B) m/e




x 0


 
 (C) em (D) e/m




x) is equal to    ay     by   
 exp  xn 1     exp  xn 1  
(A) e2 – 7 (B) e2 – 8  lim   x    x   


lim
20. y→0  x
2
(C) e – 6 (D) None of these  y 





12. The value of (A) a + b (B) a – b





 3 1 1  (C) b – a (D) – (a + b)
x  4x   x  2x 2 
lim   3   is




   
x 2   x  8  x2 x  2  
3
1  tan 1 3x  3 1  sin 1 3x

     lim
21. x→0 is equal to
1  sin 1 2 x  1  tan 1 2 x

(A) 1/2 (B) 2




(C) 1 (D) None of these (A) 1 (B) –1








 
(C) 2 (D) None
ax




13. xlim
a  a 2 − x 2 cot  2  is equal to (n (1  x )  n 2)(3.4 x 1  3x )
ax 22. lim

  equals
x→1
[(7  x )1/ 3  (1  3x )1/ 2 ].sin( x  1)

a 2a
(A) (B) 9 4 9 e
π π




(A) n (B) n





4 e 4 4
a 4a
(C) − (D)
π π (C) 4 n e (D) None of these








9 4

FREE BOOKS FOR JEE & NEET =>(@iitjeeadv)


1.118 Differential Calculus for JEE Main and Advanced

 1   x 3x 
2m / x
x 5 tan  2   3 | x |2 7 lim  sin  cos 
32. The value of the limit x→0 is
23. lim  x  is equal to  m m



x
| x |3 7 | x | 8 (A) 1 (B) 2





1 (C) e6m (D) ln 6m





(A) π  
lim  x  x 2  3x cos 1  is equal to
(B)
π





33. x
 |x|


1  
(C) – (D) None of these
π




(A) 3/2 (B) –3/2
  






cos 2 1  cos 2 1  cos 2 .........cos 2 ( x )  (C) –1 (D) none of these





lim
24. x→0 34. lim x + x + x sin(1 / x ) is equal to
2 2
x
  x  4  2 



sin     
(A) 0 (B) 2






  x   (C) –2 (D) none of these





(A) 2 (B) 2 x 2 n sin n x
lim
35. If x→0 is a non zero finite number, then





x 2 n − sin 2 n x


(C) – 2 (D) None of these

 a x  ax  1  




n must be equal to
25. If lim 2 2
a2x2 1 (A) 1 (B) 2

x





(C) 3 (D) none of these




  36. If b < 0, b ≠ –1 and a is a positive constant then
lim  x  x  x  x
= K. x   then the value of K is ax

lim equals
(A) 1 (B) a x
| x |  b2 x 2  x
ps





(C) 2a (D) None of these 1




1
el
lim (x–4
26. The value of x (n x)10) is (A) (B)
| b | −1 −b − 1






eh

(A) 1 (B) 0 1 1




(C) (D)
1 1 b −1




je

1− | b |
(C) (D) –
2


2


1/ n
m 
iit

x −1 37. lim   r n  is equal to, (n ∈ N)


tan( x − 1).log e x n   

27. lim  r 1 
@

is equal to
| x − 1 |3
x→1

(A) m (B) m/2





(A) 1 (B) – 1 (C) em (D) em/2








(C) 3 (D) None of these 38. If k is an integer such that





lim 1  1  1  1  + x n 1  a1/ x , a > 0, a ≠ 1    kπ  
n n
kπ  
28. x→0
x  x 2 x 
lim   cos  −  cos   = 0, then
n   4  6 


(A) a (B) 1 (A) k is divisible neither by 4 nor by 6







(C) 1 + a (D) None of these (B) k must be divisible by 12, but not necessarily by 24






1
 n 4n (1) n  (C) k must be divisible by 24


29. lim  2  n  is equal to (D) either k is divisible by 24 or k is divisible neither by
n  n  2 2 1



  4 nor by 6
(A) 2 (B) 1 sin x
lim





(C) 0 (D) None of these 39. x   1  where [.] denotes greatest
cos 1  (3 sin x  sin 3x ) 





2
x ; x  0  4 
30. If f(x) =  and g(x) = f(x) + |x|. Then integer function, is equal to
 x ; x  0


2
lim (log (A) (B) 1
x)g(x) is π




x0 |sin x|

(A) 0 (B) 1 4
(C) (D) does not exist





π




(C) 1/2 (D) not exist
cot 1 ( x  a log a x )




31. The value of the limit lim n2  n
a n 1

a (a > 0) is 40. The value of lim
x  sec 1 (a x log x a )
(a > 1) is equal to


n
(A) n a (B) ea (A) 1 (B) 0










(C) e–a (D) none of these (C) π/2 (D) does not exist








FREE BOOKS FOR JEE & NEET =>(@iitjeeadv)
Limits 1.119

n 49. Let P(x) = a1x + a2x2 + a3x3 + ...... + a100x100, where a1 =


n pnq 


41. lim   , p, q > 0 is equal to 1 and ai ∈ R i = 2, 3, 4, ...., 100.
n   

 2  100 1  P ( x )  1
then lim has the value equal to


(A) 1 (B) pq x0 x




pq 1
(C) pq (D) (A) 100 (B)





100




2
(C) 1 (D) 5050
cot 1 ( x  1  x )





lim 50. Assume that lim1
f(θ) exists and
42. x    2 x  1  x  is equal to


sec 1  

 f () 2  2  1
  x  1   2    2
 2 
  3  3


(A) 1 (B) 0
holds for certain interval containing the point





π


(C) (D) non-existent θ = –1 then lim f(θ)




2 1
(tan({x}  1)) sin{x} (A) is equal to f(–1) (B) is equal to 1
43. The value of lim , where { x }





{x} ({x}  1) (C) is non-existent (D) is equal to –1

x0





denotes the fractional part function, is 6 x 2 (cot x )(cos ec2 x )
(A) is 1 (B) is tan 1 lim
51. x0      has the value equal to





sec  cos x   tan    1

(C) is sin 1 (D) is non-existent
  4 sec x  




lim {x}
44. x→0 (n ({x} + |[x]|)) is equal to

(A) 6 (B) –6





(A) 0 (B) 1
ps
(C) 0 (D) –3








1 52. Suppose that a and b are real positive numbers then the
(C) ln 2 (D) ln
el




2 1/ t
 b t 1  a t 1 
eh

where [ ] is the greatest integer function and { } is the value of lim   has the value equals to
t 0 

fractional part function.  ba 
je

  100 x   99 sin x  
45. The value of  lim     is (where [.] (A) a ln b − b ln a (B) b ln b − a ln a
iit

 sin x   x   b−a b−a








 x 0
@

1
denotes greatest integer function)
(D)  b  b a
b
(A) 199 (B) 198 (C) b ln b - a ln a 
 aa









(C) 197 (D) None of these  




 2x   1 x 2 
 1  n
46. The value of lim   1  2  is 53. If f(x) = cot–1  2 
and g(x) = cos–1   , then
 1 x
2
 1 x  

n 0  2 
n
n 

(A) 1 (B) 2 f ( x )  f (a )  1
lim , where  0  a   is




(C) 4 (D) none x  a g ( x )  g (a )

 2




47. The value of (A) 1 (B) –1





lim
16  26  36....... n 6 (C) 2 (D) 1
 




2
(12  22  32.......n 2 ) 13  23  33  .......n
n3

n
n (1  x  x 2 )  n (1  x  x 2 )
54. lim is equal to
(A) 14 (B) 21 x 0 sec x  cos x





7 8 (A) 1 (B) – 1




(C) 132 (D) 12 (C) 0 (D) ∞








17 7 2x
48. Let a, b, c are non zero constant number then 55. If f(x)= nlim
 
tan–1 nx, then value of lim [f(x) – 1] is,

x→0

a b c where [.] represents greatest integer function
cos  cos cos
lim r r r (A) 0 (B) –1




r b c equals (C) 1 (D) does not exist
sin sin




1 cos( x 1)
r r  3 
56. The value of lim  x  2 x  x  1 
2 ( x 1) 2
a 2  b2  c2 c2  a 2  b2 is
(A) (B) x→1  

2bc
2
 x  2x  3 




2bc
(A) e (B) e1/2
(C) b  c  a
2 2 2





(D) independent of a,b,c (C) 1 (D) none of these




2bc




FREE BOOKS FOR JEE & NEET =>(@iitjeeadv)
1.120 Differential Calculus for JEE Main and Advanced

x2
1 lim e
tan x
− ef {f [ f ( x )]}
∫ sin x
2
57. If f(x) = , x→0 is equal to 66. If lim dx is a non zero definite number, then
1− x tan x − f {f [f ( x )]}


x→0 0
(A) 0 (B) 1 xn




(C) – 1 (D) none of these value of n is





7 29 133 5n  2 n 
 (A) 1 (B) 3
58. lim   2  3  .....   equals





10 10 10n  (C) 5 (D) 4

n  10 





(A) 3/4 (B) 2 67. If a, b and c are real numbers then the value of


1 t 




(C) 5/4 (D) 1/2
lim ln   (1  a sin bx )c x dx  equals




t 
1  t 
x2 t 0


2
 0 
59. lim  tan 
  dt is equal to
 1 t
2

x→0
x  ab
(A) abc (B)





sin 2x c
(A) 0 (B) 1 bc ca





(C) 1/2 (D) does not exist (C) (D)





a b





r 3  (r 2  1) 2 68. If C0, C1, C2, .........Cn are binomial coefficients then lim
60.  (r 4  r 2  1)(r 2  r) is equal to

n

r 1  2 n 
2 2 2
(A) 3/2 (B) 1  Cn    c n 1    Cn  2  ......( 1) n   C0  is
  
3  
3  
3 




(C) 2 (D) infinite
(A) 0 (B) 1




729 x  243x  81x  9 x  3x  1
ps




61. If lim 3
= k(ln 3) , then k (C) –1 (D) 2




x3  sin [ x ]  2 x 

el
x
is equal to  if [ x ]  0
69. If f(x)=  [x] , where [.] denotes the
eh



(A) 4 (B) 5 if [ x ]  0
 0




(C) 6 (D) none
je

greatest integer function, then lim f(x) is






sin x − (sin x )sin x x→0
iit

62. lim is equal to


x / 2 1 − sin x ln sin x (A) 0 (B) 1





@

(A) 1 (B) zero (C) − 1 (D) none of these










(C) 2 (D) 2/3 1
70. Let f(x) = lim . Then the set of




n  2n

3 1 
xn x
5
63. lim x 2  is equal to   tan 2 x 
 
x 
n 0 n !

values of x for which f(x) = 0, is
(A) 1 (B) e
(A) |2x| > 3 (B) | (2 x ) |< 3




(C) 2e–1 (D) 0








64. If {tn} be a sequence such that tn = , S denote the
2n (C) |2x| ≥ 3 (D) |2x| ≤ 3




3n + 1 n

2 n 4 1
sum of the first n terms and
n !  5n 5 1
n Sn 1  Sn 71. The value of lim  n  is equal to
  lim , then n   n 


n  2 n
k (A) 1 (B) 0




2/5
k 1 1
(C)   (D) e2/5
  lim   2 2  3 2  .......  (n  1) n 1 equals e





n 
(A) 18 (B) 9 1
72. The value of lim ([12x + 12] + [22x + 22] + [22x + 22]




n3

(C) 3 (D) 6 n 




n + ....... + [n2x + n2]), is (where [.] denotes the greatest
1.n  (n  1)(1  2)(n  2)(1  2  3)  ..1. r integer function)
r 1
65. lim x 1
n4 (B) x +

n  (A)




is equal to 3 3

x 1
(A) 1/12 (B) 1/24 (C) + (D) none of these








(C) 1/6 (D) 1/48 3 3




FREE BOOKS FOR JEE & NEET =>(@iitjeeadv)
Limits 1.121

73. The value of (A) 0 (B) 1







(C) 2 (D) 3





 1 1 1 1 
lim     ........  75. A0 is an equilateral triangle of unit area, A0 is divided into
nb 
, is
na na  1 na  2


n  

four equal parts, each an equilateral triangle, by joining
the mid points of the sides of A0. The central triangle is
a b
(A) log   (B) log   removed. Treating the remaining three triangles in the
b a





same way of division as was done to A0, and this process
(C) log (ab) (D) none of these is repeated n times. The sum of the area of the triangles




74. Let f(x) be defined for all x ∈ R such that removed in Sn then lim Sn is
n

  1   (A) 1/2 (B) 1
lim f ( x )  ln 1  f ( x )   ln(f ( x ))   0 ,





x 0   e   (C) –1 (D) 2





then f(0) is

Multiple Correct Answer Type for JEE Advanced

76. Which of the following limits vanish? 80. The true statement(s) is / are

1

lim x 4 (A) If lim f(x) = 0, then there must exist a number d


x x →c

1 such that f(d) < 0.001.
(A) sin lim
ps


x x  /2 (B) lim f(x) = L, is equivalent to lim (f(x) – L) = 0.
x →c x →c


(B) (1 − sin x) . tan x
el



2 x2  3 (C) lim (f(x) + g(x)) may exist even if the limits lim
eh

x →a x →a


(C) lim . sgn (x)
x2  x  5


x
(f(x) and lim (g(x) do not exist.
je

x →a
[ x ]2 − 9
(D) lim
iit

(D) If lim f(x) exists and lim (f(x) + g(x)) does not exist,
x2 − 9


x3 x →a x →a


@

77. The true statement(s) is / are then lim g(x) does not exist.
(A) If f(x) < g(x) for all x ≠ a, then lim f(x) < lim g(x).

x →a
x →a x →a


81. Which of the following functions have a graph which lies
(B) If lim f(x) = 0 and |g(x)| ≤ M for a fixed number M

between the graphs of y = |x| and


x →c
and all x ≠ c, then lim f(x)· g(x) = 0. y = – |x| and have a limiting value as x → 0.

x →c (A) y = x cos x (B) y = |x| sin x




(C) If lim f(x) = L, then lim |f(x)| = |L| and conversely 1 1
x →c


x →c (C) y = x cos (D) y = x sin




if lim |f(x)| = |L| then lim f(x) = L. x x
x →c x →c
(D) If f(x) = g(x) for all real number other then x = 0 82. The false statement(s) is / are



and lim f ( x ) = L, then lim g ( x ) = L. (A) If P(x) is a polynomial, then the function f(x) = P(x)
x →0 x →0 x −1


78. Which of the following functions has two horizontal has a vertical asymptote at x = 1.

asymptotes (B) A polynomial function has no vertical asymptote and


x 2x
a rational function has atleast one vertical asymptote.
(A) y = (B) y = (C) If f(x) has a vertical asymptote at x = 0, then f is
x +1




x2 +1


undefined at x = 0.
sin x
(C) y  (D) y = cot–1(2x + 1) (D) A function can have move than two horizontal


x2 1




asymptotes.
79. Which of the following functions has a vertical asymptote 83. The function(s) which have a limit as n → ∞


at x = – 1. 2 2
| x2 −1 | x 2 − 6x − 7  n 1   n 1 
(A)   (B) (1) n  
(A) y = (B) y =  n 1   n 1 




x +1 x +1




x2 +1 sin( x + 2) n2 +1 n2 1
(C) y = (D) y = (C) (D) (1) n




x +1 x +1 n n




FREE BOOKS FOR JEE & NEET =>(@iitjeeadv)
1.122 Differential Calculus for JEE Main and Advanced

84. The function(s) which have a limit as x → ∞ 1


2 x 2 n sin x

sin xπ x
(A) (B) a cos2xπ + b sin2xπ 89. Let f(x) = lim then which of the following
n  1 x 2n






x
(C) x sin xπ (D) tan xπ alternative(s) is/are correct ?




(A) lim x f(x) = 2
85. Which of the following limits exist? (where [.] indicates



x

greatest integer function all throughout) (B) lim f(x) does not exist



1n
sin[ x ]  en  x→1
(A) lim (B) lim   (C) lim f(x) does not exist
 




n 




x→1 [ x ]
 x→0
(D) lim f(x) is equal to zero.
(C) lim sin(sin 1
x )  (D) lim sin (sin x ) 
1



x
 




x  2 x
x1
 ax  1 
    90. Consider the function f (x) =   where
 bx  2 


86. Given l1 = lim cos–1 sec  x    ;

  4  a2 + b2 ≠ 0 then lim f ( x )

x


4 x
    (A) exists for all values of a and b
 cos ec  x    ;




l2 = lim sin–1 (B) is zero for a < b
   4 



x
4 (C) is non existent for a > b



1 1
       
l3 = lim tan–1 cot  x    ; 
e a e b
  4  (D) is or if a = b




x
4 mx  n for
2
x0

   
ps
l4 = lim cot–1  tan  x    91. Let f (x) =  nx  m for 0  x  1 where m, n ∈ R
   4 

 nx 3  m for

el
x
4 x 1
eh

where [x] denotes greatest integer function then which then which of the following must be correct

of the following limits exist (A) lim f ( x ) exist for all values of m and n.
je


(A) l1 (B) l2 x→0




(B) lim f ( x ) exists only if m = n.
iit

(C) l3 (D) l4


x→0




87. Give a real valued function f such that (C) lim f ( x ) exists for all values of m and n.
@




x→0
 tan 2 x
 2 for x0 (D) lim f ( x ) exists for no values of m and n.


 ( x  [ x ])
2 x→1
 sin m t
f(x) =  x0 2x

1 for
92. lim (m, n, ∈ N) equals


x →0 x tn

+

 {x}cot{x} for x0


 (A) 0 if m ≥ n (B) ln 2 if n – m=1





(C) +∞ if n – m = 1 (D) None of these

 




where, [.] is the integral part and {.} is the fractional part
x 2  x  1  ax  b  0 , then for k ≥ 2, k ∈

of x, then 93. If lim
x

(A) lim f(x) = 1


x→0 N which of the following is/are correct ?
(B) lim f(x) = cot 1 (A) 2a + b = 0





x0
(B) a + 2b = 0

 


(C) lim sec 2 n  k !b   1
2
(C) cot–1 lim f ( x ) =1 n 





x 0
(D) none of these (D) lim sec 2 n  k !a   1
n 




88. lim f(x) does not exist when

x →c x  cos 2 x
where, [.] is the greatest integral part and {.} is the 94. Let l1 = lim and
x  sin x

x

fractional part of x, then
1
(A) f(x) = [ [x] ] – [2x – 1], c = 3 h dx
l2 = lim  h 2  x 2 . Then


(B) f(x) = [x] – x, c = 1


h0
1


(C) f(x) = {x}2 – {–x}2, c = 0 22



tan(sgn x ) (A) both l1 and l2 are less than


(D) f(x) = ,c=0 7


sgn x (B) one of the two limits is rational and other irrational.


FREE BOOKS FOR JEE & NEET =>(@iitjeeadv)
Limits 1.123

(C) l2 > l1 Reason (R) : Let the triangle have side length 1 and



(D) l2 is greater than 3 times of l1. radius of circles be r. Then 2(n – 1) r + 2r 3


95. In which one of the following cases, limit tends to e n (n −1)
= 1. There are circles. the area ratio

1


2
(A) lim x 1
x
 n (n  1)
which approaches 3π


x 1
=
2 3 (n  ( 3  1)) 2


x
 1 6
(B) lim  1   as n→ ∞.
x   x



x 3 101. Assertion (A) : Let f : (0, ∞) → R be a twice continuously
 x4



(C) lim   differentiable function such that
x   x  2 


|f′′(x) + 2x f′(x) + (x2 + 1) f(x) | ≤ 1 for all x.


1 Then lim f(x) = 0.
(D) lim 1  f ( x )  f ( x ) when lim f ( x ) → 0


x


x  x
Reason (R) : Applying L’Hospital’s rule twice


x2
Assertion (A) and Reason (R)
f ( x )e 2
Code: on the function we get lim f(x) = 0.


x2 x
(A) Both A and R are true and R is the correct explanation e2


of A.
102. Assertion (A) : A circle C1 is inscribed in an equilateral


(B) Both A and R are true but R is not the correct triangle ABC with side length 2. Then circle C 2 is


explanation of A. inscribed tangent to BC, CA and circle C1. An infinite
ps
(C) A is true, R is false. sequence of such circles is constructed, each tangent to


(D) A is false, R is true. BC, CA and the previous circle. The sum of areas of all the
el



96. Assertion (A) : If [x] denotes the greatest integer function infinitely many circles is .
eh



x 8
then lim does not exist. 1 1
je

x [x] Reason (R) : Radius of C1 is , that of C2 is and



3 3 3
iit

Reason (R) : lim [ x ] does not exist.


1

x
radius of the remaining circle each shrink by a factor .
@

97. Assertion (A) : The value of 3





n 3  e1/ x  1 
 2n (n 2  3n  2)
1 103. Assertion (A): lim[ x ]   where [.] represents greatest
is .
 e 1 
x 0 1/ x


n 1
2
integer function does not exist.
Reason (R) :  e1/ x  1 


n 3 
1  2 1  Reason (R) : lim   does not exist.
x 0  e1/ x  1 
 2n (n 2  3n  2)   2n  n  1  n  2 

 
n 1 n 1

sin cot 2 x 1

2 104. Assertion (A) : lim
98. Assertion (A) : lim ( x  x  x  x )  3 3 2 3 3 2
(  2x ) 2


x  2 2


x  3
 1  sin  tan 
Reason (R) : lim  x   3 x 3  x 2  = 0 and Reason (R) : lim  1 and lim  1 , where θ
x    0  0 


3
is measured in radians.
 1 
lim  x   3 x 3  x 2  = 0 105. Consider the function f (x) = cos–1[cot x] where [ ]
x   

3

indicates greatest integer function.
99. Assertion (A) : lim x x − x x ( x

) = –1 Assertion (A) : lim f ( x ) exists







x → 0+
x
x 2
Reason (R) : lim x (x – 1) = –1
Reason (R) : Both lim f ( x ) and lim f ( x ) are finite.

x0





100. Assertion (A): An equilateral triangle is filled with n, x x
 
2 2
rows of congruent circles. The limit of the ratio of area
of circle to the area of triangle as Comprehension - 1
3π Let f(x) = x[x[x]], x ∈ [–1, 3] where [.] represents the greatest
n → ∞ is . integer function.

6

FREE BOOKS FOR JEE & NEET =>(@iitjeeadv)


1.124 Differential Calculus for JEE Main and Advanced

113. If [ . ] represents greatest integer function, then which


106. If the value of x so that f(x) = 13 is a , a, b ∈ N and


of the following, is identical to f of (x) for x ∈ (π, 2π)

b
x
g.c.d. (a, b) = 1 then the value of a + b is : (A) sgn (x) (B)






(A) 14 (B) 18 x





(C) 22 (D) None of these sin x




(C) sinπ[x] (D)





107. The number of points of where limit of f(x) does not sin x

exist is :
114. The limit of function g(x) = sgn f(x) as x tends to 0 is
(A) 3 (B) 4






(C) 5 (D) None of these (A) 0 (B) 1










(C) – 1 (D) does not exist





108. The value of lim f(2sin x) is :


x
2 Comprehension - 4
(A) 2 (B) 4 x
t 2 dt
 (a  t r )1/ p




(C) 8 (D) None




Suppose lim 0
  where p ∈ N, p ≥ 2, a > 0, r >
Comprehension - 2 x 0 bx  sin x
Consider a new definition for approximating the values of a 0 and b ≠ 0.
function f(x) in the neighbourhood of a point x = a Approx
x →a 115. If  exists and is non zero then

f(x) = lim f(x) whenever [ | lim f(x) (A) b > 1 (B) 0 < b < 1





x a  x a  (C) b < 0 (D) b = 1




– lim f(x) | ] ≤ 1, otherwise Approx f(x) is said to be
116. If p = 3 and  = 1 then the value of ‘a’ is equal to
ps
x a  x →a

(A) 8 (B) 3
el
non-existent. Here [.] denotes greatest integer function.




(C) 6 (D) 3/2
eh



109. For which of the following functions, Approx f(x) exists :
117. If p = 2 and a = 9, b = 1 and  exists then the value of

x2 −1 2{x} − 4

 is equal to
je

(A) Approx | x − 1 | (B) Approx


[x] − 3




iit

x→1 x→2 (A) 3/2 (B) 2/3






1 (C) 1/3 (D) 7/9
@

(C) Approx (D) None of these






1




x 0
2  2x Comprehension - 5
[ x ]| x |
x (e  2) Let a1 > a2 > a3 ....... an > 1 ; p1 > p2 > p3..... > pn > 0; such
110. The value of Approx is
[ x ] | x | that p1 + p2 + p3 + ......+ pn = 1.

x 0
(A) –1 (B) 0 Also F(x) = (p1a1x + p2a2x + ....... + pnanx)1/x




(C) 1 (D) None 118. lim F(x) equals





x0
 [x x ]  a, x0
 (A) p1 n a1 + p2 n a2 + ..... + pn n an
111. Let f(x) =  t
. The complete set


 sin x 
 lim  , x0

 (B) a1p + a p2 + .....a pn
  x 
1 2


n
t


of the values of ‘a’ for which Approx exists is
(C) a1 · a 2 + ..... a n
p1 p p2

n
x→0



(A) (0, 2] (B) (–2, 2) n
 a r pr




(C) [–1, 1] (D) None of these (D)






r 1
Comprehension - 3
119. lim F(x) equals
Consider the function f(x) =  sin x  x
1

0. Where [ . ] x
x 
(A) n a1 (B) ea 1



represents greatest integer function.

(C) a1 (D) an




112. Which of the following function will be periodic lim F(x) equals
120.

(A) f(x) (B) f(|x|)

x





  (A) n an (B) ea 1

(C) f(–x) (D) f  | x |  






 




2 (C) a1 (D) an




FREE BOOKS FOR JEE & NEET =>(@iitjeeadv)
Limits 1.125

Match the Columns for JEE Advanced


121. Column - I Column - II


1
lim cos2 (π ( n 3 + n 2 + 2n )) where n is an integer, equals.
3
(A) (P)




n 2
1
lim n sin(2π 1 + n ) (n ∈ N) equals.
2
(B) (Q)




n 4
 (n  1) 
lim (–1)n sin (π n 2 + 0.5n + 1 )  sin is (where n ∈ N). (R) π
2n 
(C)





n
x
 xa 
(D) If lim   = e where 'a' is some real constant then the (S) non existent
x   x  a 




value of 'a' is equal to.

122. Column - I Column - II



(A) If lim ( ( x − x − 1) – ax – b) = 0, where a > 0, then there
2
(P) y = –3
x 




exists atleast one a and b for which point (a, 2b) lies on the line

(1 + a 3 ) + 8e1/ x
(B) If lim = 2, then there exists atleast one (Q) 3x–2y–5= 0
+ (1 − b3 ) e1/ x




x→0 1 ps
a and b for which point (a, b3) lies on the line

el
(C) If lim ( ( x  x  1) – ax2 – b) = 0, then there exists
4 2
(R) 15x–2y–13=0
eh

x 




atleast one a and b for which point (a, –2b) lies on the line
je

x7  a7
iit

(D) If lim = 7, where a < 0, then there exists atleast one (S) y = 2
x  a x  a




@

a for which point (a, 2) lies on the line.



123. Column-I Column-II



(A) If f(x) = |x – a| + |x – 10| + |x – a – 10|, where a ∈ (0, 10), (P) 0




then the minimum value of f is (Q) 1



x (1  cos 2 x ) 2  a (sin x  tan x ) 2
(B) lim is equal to (R) 4
tan 5 x  a sin 8 x




x0

n a sin 2 (n !)
(C) lim , 0 < a < 1, n ∈ N, is equal to (S) 10
n 1




n

(D) The number of solutions of the equation (T) depends on a






loga(2a – x) = loga3 x, a > 0, a ≠ 1 is

124. Column - I Column - II


(A) lim ( x  x  x  x ) equals (P) –2




x
sin 2 x  2 tan x
(B) The value of the limit, lim is (Q) –1
ln(1  x 3 )




x0

(C) lim (ln sin3 x – ln(x4 + ex3)) equals (R) 0






x0
(D) Let tan (2π | sin θ | ) = cot (2π | cos θ | ), where θ ∈ R (S) 1




 2 
and f(x) = ( | sin θ | + cos θ | )x. The value of lim   equals

x  f ( x ) 

(Here [ ] represents greatest integer function)



FREE BOOKS FOR JEE & NEET =>(@iitjeeadv)
1.126 Differential Calculus for JEE Main and Advanced

125. Column - I Column - II




1  cos 2 x
(A) lim equals (P) 1
ex  ex  x
2
x 0




1/ x
 (3 / x )  1 
(B) If the value of lim   can be expressed in the (Q) 2
x 0  (3 / x )  1 






form of ep/q, where p and q are relative prime then (p + q) is equal to



tan 3 x  tan x 3
(C) lim equals (R) 4
x5




x0

x  2 sin x
(D) lim (S) 5
x  2 sin x  1  sin 2 x  x  1




x0 2

Review Exercises for JEE Advanced


1. Evaluate the following limits : 8. Evaluate lim (logsinx sin 2x)log sin 2x.



tan(a  2h )  2 tan(a  h )  tan a x0
(i) lim
h 0 h2 9. Evaluate



3
1  tan 1 3x  3 1  sin 1 3x 1  (tan x  sin x )  (tan x  sin x )  ..
(ii) lim
x 0 1 1
ps lim


1  sin 2 x  1  tan 2x x→0 1  x 3  x 3  ..........
el

2. Evaluate the following limits :
1 − x 02

1  cos x cos 2 x cos 3x
eh

(i) lim 10. Find lim , where



n x1x 2 x 4 .........x n


x 0 sin 2 2 x
je

(sin x  tan x ) 2  x (1  cos 2 x ) 2 1+ xr


iit

(ii) lim xr+1 = .


8 tan x  5 sin x

8 5
2


x→0
@

n n 1 n 2
3. Evaluate the following limits :
1.  r  2. r  3.  r  ....  n.1

1  1  x 2 cos x 11. Evaluate lim 1 1 1
(i) lim n  n4

4


x→0 tan x
n
12 + 22 + 32 + ... + r 2
(ii) lim
1  x sin x  cos 2 x 12. If Tr =
13 + 23 + 33 + ... + r 3
and Sn = ( −1) r Tr then find ∑

r =1


x 0 x
tan 2 lim Sn.
2 x

 
4. Evaluate the following limits :

1  x2 x2 x2 x2  13. Evaulate lim ln x 2
1 x2 1  x .
(i) lim 1  cos  cos  cos 

cos x
x8  4  x 1


x→0 2 4 2

 x2  x3 + 1
sin sin tan   14. Prove that the graph of f(x) = is asymptotic to the

(ii) lim  2  x


x→0
n cos 3x graph of g(x) = x2 using lim (f(x) – g(x)).
x
5. Evaluate
  x 

sin(3x  a )  3 sin(2 x  a )  3 sin( x  a )  sin a  x 1 
15. Evaluate lim x  tan 1   tan 1  
x2 x2
lim
x      

3
x→0 x

(1 − x )(1 − x ).....(1 − x )
2 2n
6. Evaluate lim (sin x-tan x) 2 + (1-cos 2x) 4 + x 5
[(1 − x )(1 − x 2 )......(1 − x n )]2 16. Evaluate lim

x→1 x →0 7 tan 7 x + sin 6 x + 2 sin 5 x

7. Evaluate 17. Find


n
  1  1   1 

1
lim (n  1)  n    n  2  ..........  n  n 1   (A) lim f ( x ) (B) lim f ( x )
  2     
2

nn 2 2




n x →1/ 2 x →3/ 2

FREE BOOKS FOR JEE & NEET =>(@iitjeeadv)
Limits 1.127

| x 1 | 2 1
 x | x  1 | 2  27. Simplify the function y = lim
n  1  n sin 2 x
where f ( x )  2 x



1 28. Simplify the function
x2


x  ( x )  n( x ) sin 2 x
18. Prove that y = lim
n  1  n sin 2 x



1 x  x 2
1 2  2 cos x
lim  lim  log 0.005 . x tan x  2 x  1
x 0 x x  / 4 sin( x   / 4) 29. Show that the function f(x) = does not
x 1



1 11 2 tend to a unique limit as x → ∞, through all real values,
(1 + x )1/ x − e + ex − ex but if x ranges through the sequence of values x = nπ +
2 24 π
19. Evaluate lim (n = 0, 1, 2, ....), then f(x) → 3.
x →0 x3

4
 2 30. The function f(x) and φ(x) are such that f(0) = φ(0) = 0.
20. If λn + 1 = 2   n , λ0 = x , find lim r


, for By considering the case where
x 4 x  4


f (x)
r = 1 and 3. f(x) = x2 sin 1/x, φ(x) = tan x, show that lim may
x→0 ϕ( x )


x n f ( x ) + g( x ) f '( x )
21. Simplify the function φn (x) = as n → ∞. exist when lim does not exist.
x→0 ϕ '( x )
x n +1

31. Show that the function
 a sin x   b tan x 

22. Evaluate lim   f ( x )  n( x ) sin 2 x
x   x 
where a, b are positive
x 0  y = lim

n  1  n sin 2 x

integers and [ ] denotes the greatest integer function.
ps
is equal to f(x) when x is an integer, but is equal to φ(x)


x in every other case.
el
23. If f1(x) = + 10 , ∀ x ∈ R and defined by fn(x) =
32. Evaluate

2
eh

1n
f1{fn–1(x)}, ∀ ≥ 2. S then evaluate lim f n ( x ) .  (n 3  13 )(n 3  23 )......(n 3  n 3 ) 
n  lim  
je

1/ x
n  
 n 3n 
 1 a x 1 
iit

where a > 0, a ≠ 1.

24. Evaluate lim  .  x3
x  x a  1 

   3x 2
 1  x
@

1
33. If lim   = k, then find the value of [6 nk],
 3x  1 
2

25. Simplify the function x 

 

y = 1 – lim lim {cos(m !x )}2 n where [.] is greatest integer function.

m  n 

1 2 x 2  5  3 2 x 3  3x  5
34. If lim = b, then find the value
x (1 − x ) 3
3

26. Let f(x) = . x 2 3
x  6  5 4x 2  6x  4

f (x) of 230b.
Find m = lim and b = lim (f(x) – mx). Hence cos 4 x  a cos 2 x  b


x x x 35. If lim = a finite quantity, then find
x 0 x4

show that lim (f(x) – mx – b) = 0.
x the value of ab + 1

Target Exercises for JEE Advanced


 x x 1  x  1 
x
  1 
1 x
1. Find lim   4. Evaluate: lim x ln  e 1   
x x 1  
  x

x 

  x

 ( x 3 − 2ax 2 − a 2 x + 2a 3 ) ( x − 2a ) −1 + 2a ( x + a ) 2  (3  ax )5 / 2  b nx  c sin( x  1)
2. lim  −  5. If lim = 2, then find the
− a  x 1 ( x  1) 2

x→a 

3
− 2
x

x a x

1
value of the expression a2+ b2 + c2.
 (a x  a  1)( x p  a p )  x a 1  1 1  ax 
 6. If the lim 3    exists and has the value
3. Evaluate p 1  x0 x  1  x 1  bx 

 ( x  a ) (ln a )a
2 p


1 2 3
where a > 0, p ∈ Q equal to l, then find the value of   .
a l b



FREE BOOKS FOR JEE & NEET =>(@iitjeeadv)
1.128 Differential Calculus for JEE Main and Advanced

1 1 19. Find values of α and β such that


7. Evaluate lim x2 (sin x. sin3 – (x tan3 )sgn (x))
(ax 2  2bx  c  x   =0 and prove that

x x x lim
x 



1 1
n cos 2 n  sin 2 n
8. Find lim
n   2 1
2 2
2 1 
lim x  (ax 2  2bx  c  x     ac  b 2

x  2a a
n  cos n  n sin n 
 2 2 
20. Show that the functions y = 1 – x – [x] – [1–x] and y
If an + 1 = (an + x)–1, a0 = 0, and bn + 1 = (bn + y)–1, b0 = 0,


9.
= 1 – x – lim (cos 2 n 1 2x ) are identical.

a n  bn n 
find lim for n = 2, 3.
xy x y
2 2
21. If f(x + y) = f(x) + f(y) for all x, y ∈ R and f(1) = 1,


π
π 4 1/ x2 2f (tan x )  2f (sin x )
+x e
4 then find lim
∫ cos t dt − ∫ cos tdt
n n
0
4 x 0 x 2 f (sin x )
10. Evaluate lim 0
,n∈ N n n
1 C
 n k (k k 3)  e  2

x→0
x 4e x
2
22. Show that lim
n 



k 0
n x  x  23. Calculate the limit of the sequence {xn} if it is defined
 n  
 1 x 

(1  x ) 2
by the recursive relation :
11. Evaluate lim
n (1  x 2 )  2 x 1 a 


x0
xn + 1 =  x n   ∀n ≥ 1, a > 0,

12. Let g(x) = f(x – 1) + f(x + 1) where
ps 2 xn 

where x1 is an arbitrary negative number.
1 | x |, | x | 1

el
f(x) =  find lim g(sin x). 24. Show that if –1 < x < 1, then
| x | , | x |  1 


eh

x m(m  1)....(m  n  1) n  m  n
2 un = x   x

n! n
je

 3 1
 r r  tends to zero as n → ∞ .
r 
n
13. Prove that lim  cot 1 
iit



n  25. If cn, sn denote the sums of the first n terms of the series

r 1  2

2

@

  1
+ cosθ + cos2θ +...., and
(1  cos x )(e x  cos x ) 2
14. Evaluate lim . sin θ + sin 2θ + sin 3θ .......
xn
x 0



and θ is not a multiple of 2π, then prove that
15. If α, β are two distinct real roots of the equation ax3 +

lim (c1 + c2 + ....+cn)/n = 0,

x – 1 – a = 0, (a ≠ –1, 0), which are not equal to unity

n
then prove that 1 θ
lim (s1 + s2 + .... + sn)/n = cot


n 2 2
ln((1  a ) x 3  x 2  1  a )  
lim 1 x
is a  1  .
 26. Draw a graph of the function y defined by the equation
 1)( x  1) 

x1/  (e

1
x 2 n sin x  x 2
 −1 
4 1 − 3x + x 2  x4 −1  y = lim 2
16. lim  −1
−  +3 −1  n  x 1
2n
x →1  x −x
2
1 − x3  x −x
3

 
27. Show that the graph of the function

17. If 2f(sin x) + 2 f(cos x) = tan x , then evaluate lim x n φ ( x ) + x − n Ψ( x )

x→−1 y = lim
n →∞ xn + x −n

 1  2 −1
1− x f(x) if f   = . is composed of parts of the graphs of φ(x) and Ψ(x),

 2 2 together with (as a rule) two isolated points.
18. Find 28. Prove that the function y which is equal to 0 when x is



−1
−1  rational and to 1 when x is irrational, may be represented
  a + x  2 4 ax 
lim    − 3/ 4 1/ 4 1/ 2 1/ 2 1/ 4
x →a   4 a − 4 x 
in the form
 x − a x + a x − a 3/ 4  y = lim sgn{sin2(m!πx)}
 


m 2
−( 2 )log a 8
} where sgn x = lim arctan(nx ) .
m  

4

FREE BOOKS FOR JEE & NEET =>(@iitjeeadv)
Limits 1.129

29. Evaluate 32. If f(x) = lim {sinx + 2sin2x + 3sin3x +....nsinnx} then


cos 2 (1  cos 2 (1  cos 2 (...cos 2 ( x )))...)


n
lim
x 0  ( x  4)  2 
1



 evaluate lim {(1  sin x ) 2 f ( x )}(sin x 1)
x x  / 2
 
30. Let the sequence xn be defined as 33. The function un attains the values



x0 = a 1 1 1 1
u1 = ;u = + + ,

x1  a  a 2 2


2 2.4 2.4.6

1 1 1
x2  a  a  a un = + + .... ,.....




2 2.4 2.4....(2n )

x3  a  a  a  a Prove that lim un < 2.


n

..................................., find lim x n . 1k  2k  ....n k 1
n  

34. Prove that, if k > 0, lim .
n k 1 k 1


n 
1 1 1
[ x ]  [2 x ]  [3x ]  .....  [nx ]
2 3 n F(n )
31. Evaluate lim Hence evaluate lim , where F(n) is the
n  12  22  32  ......n 2 (n 1) 2 k  2


n 

where [.] denotes the greatest integer function coefficient of xn–2 in (x – 1k) (x – 2k) .... (x – nk).

ps
el
Previous Year's Questions (JEE Advanced)
eh
je

A. Fill in the blanks


iit

x2

1. If f(x) = sin x, x ≠ nπ, n = 0, ± 1, ± 2, ± 3, 0 cos t 2 dt


[IIT - 1997]
@

6. lim = ..................





............. x 0 x sin x
= 2, otherwise
ln(1  2h )  2 ln(1  h )

and g(x) = x2 + 1, x ≠ 0, 2 7. lim = ......... [IIT - 1997]
h2


h 0

= 4, x = 0
B. True / False

= 5, x = 2
[IIT - 1986]

then lim g[f(x)] is............. 8. If lim [f(x)g(x)] exists then both lim f(x) and


x→0 x →a


x →a
 4 1 2  lim g(x) exist [IIT - 1981]
 x sin  x   x 


x →a
2. lim     = ....... [IIT - 1987]


x  (1 | x |3 ) 
  C. Multiple Choice Questions with
  ONE correct answer
3. ABC is an isosceles triangle inscribed in a circle of radius
x  sin x

r. IF AB = AC and h is the altitude from A to BC then 9. If f(x)= , then lim f(x) is
x  cos 2 x

the triangle ABC has perimeter x

P = 2( 2hr − h
2
+ 2 hr ) and area (A) 0 (B) ∞




(C) 1 (D) None





A = ...... also lim
A
= ....... [IIT - 1989] [IIT - 1979]
h →0 P3  



1 2 n
x4
10. lim    ....   is equal to
 x6

n  1  n 2 1 n2 1 n2 
4. lim   = .................. [IIT - 1990]
x   x  1 


1
(A) 0 (B) –




1/ x 2
2
 
lim  1  5x 2 
2
1
5. = .................. [IIT - 1996] (C) (D) none of these
 


 1  3x  [IIT - 1984]




x→0 2

FREE BOOKS FOR JEE & NEET =>(@iitjeeadv)


1.130 Differential Calculus for JEE Main and Advanced

sin[ x ] ((a − n )nx − tan x ) sin nx


11. If f(x) = , [x] ≠ 0 20. If lim = 0, where n is non


[x] x2


x→0
= 0, [x] = 0 zero real number, then a is equal to

where [x] denotes the greatest integer less than or equal n +1

to x, then lim f(x) equals (A) 0 (B)





x→0 n
(A) 1 (B) 0 1





(C) n (D) n +
(C) –1 (D) none of these





[IIT - 2003] n
[IIT - 1985]




n 21. The value of lim ((sin x)1/x + (1 + x)sin x), where x > 0 is
12. lim x = 0 for


x→0

x ex (A) 0 (B) –1
(A) No value of n [IIT - 2006]





(C) 1 (D) 2







(B) n is any whole number sec 2 x



(C) n = 0 only f ( t )dt



(D) n = 2 only [IIT - 1992] 2



22. lim equals
13. lim   equals 2




x x

x→0  tan 1 2 x  4 x  2

16
(A) 0 (B) 1/2 (A) 8 f(2) (B) 2 f(2)




(D) ∞ [IIT - 1992] π




(C) π




1/ x
14. lim    (C) 2 f(1/2) (D) 4f(2) [IIT - 2007]
tan   x   equals
ps
x→0 





π

  4 
el
x
1 t ln(1  t )
(A) 1 (B) – 1 23. The value of lim  dt is




eh

x 0 3
t4  4

(C) e2 (D) e x 0
[IIT - 1993]




je

1
1 2n r (A) 0 (B)
15. lim 




12
iit

equals

n  n
r 1 n  r 1
2 2
(C) (D) 1 [IIT - 2010]
@

24





64
(A) 1 + 5 (B) –1 + 5





1
(C) –1 + 2 (D) 1 + 2 24. If lim 1  x ln (1  b 2 )  x = 2bsin2θ, b > 0 and
[IIT - 1997] x   





x tan 2 x − 2 x tan x θ ∈ (–π, π), then the value of θ is
16. lim is equal to

(1 − cos 2 x ) 2

x→0
π π
(A) 2 (B) – 2 (A) ± (B) ±
3








1 1 4
(C) (D) – [IIT - 1999]
π π





2 2
(C) ± (D) ± [IIT - 2011]





x 6 2
 x 3 
17. For x ∈ R, lim   equals
x  x  2   x2  x 1 

25. If lim   ax  b   4 then,
 
 x 1

(A) e (B) e x 





(C) e–5 (D) e5 [IIT - 2000]





(A) a = 1, b = 4 (B) a = 1, b = – 4
sin(π cos x ) 2




18. lim equals (C) a = 2, b = –3 (D) a = 2, b = 3
[IIT - 2012]




x2

x→0

(A) – π (B) π
D. Integer Answer Type Questions



(C) π/2 (D) 1 [IIT - 2001]






19. The integer n for which 26. Let m and n be two positive integers greater than 1.


(cos x − 1)(cos x − e x )  ecos( a ) − e   e
n

lim is a finite nonzero number is If lim  = −   then the value of


m
is
n 

x→0 x a →0  a m
  2 n
(A) 1 (B) 2




(C) 3 (D) 4 [IIT - 2002] [2015]






FREE BOOKS FOR JEE & NEET =>(@iitjeeadv)
Limits 1.131

27. Let a, b ∈ R be such that 1 − x (1 + |1 − x |)  1 



31. Let f(x) = cos  , for x π 1
x 2 sin bx |1 − x |  1 − x 


lim = 1. Then 6(a + b) equals [2016]
x → 0 ax − sin x


then


E. M ultiple C hoice Q uestions with ONE (A) lim f ( x ) does not exist
x →1−



or MORE THAN ONE correct answer
(B) lim f ( x ) = 0
x →1−



1
(1 − cos 2 x) (C) lim f ( x ) = 0
x →1+



2
28. The value of lim
x → 0− x

(D) lim f ( x ) does not exist [2017]
x →1+




(A) 1 (B) –1




(C) 0 (D) none of these




[IIT - 1991] F. Subjective Problems

1 − cos 2(x − 1) a  2 x  3x
29. lim 32. Evaluate lim , a  0
x −1 x a 3a  x  2 3x

x →1

[IIT - 1978]
(A) exist and it equals 2
(a + h )2 sin (a + h ) − a 2 sin a



(B) exists and it equals – 2 33. Evaluate : lim
ps

h →0


h
(C) does not exist
[IIT - 1980]
el



(D) none of these [IIT - 1998]
eh



35. Use the formula

x2
a− a −x − 2 2
ax − 1 2x − 1
je

4 lim = loge a to find lim [1982]


30. Let L = lim , a > 0. If L is finite, then x → 0 (1 + x )1/ 2 − 1
iit

x →0


x
x →0 x4

@

 n  1 cos1    n.
(A) a = 2 (B) a = 1 2 1
35. Find the value of lim





x   n

1 1
(C) L = (D) L = [IIT - 2004]




64 32


[IIT - 2009]

Previous Year's Questions (JEE Main Papers)

1. lim
(1 - cos 2 x ) (3 + cos x )
is equal to tan( x - 2) { x 2 + (k - 2 ) x - 2 x}
x Æ0
3. If lim = 5, then k


x tan 4 x x Æ2 x2 - 4 x + 4


(a) 1/2 (b) 1 is equal to




(a) 0 (b) 1
(c) 2 (d) –1/4




(c) 2 (d) 3 [2014, online]





[2013]





sin (p cos x ) 2 2
e x - cos x
2. lim 2
is equal to 4. If lim is equal to
x Æ0


x sin 2 x


x Æ0
(a) p/2 (b) 1 3




(a) 3 (b)
(d) p




(c) –p [2014] 2





FREE BOOKS FOR JEE & NEET =>(@iitjeeadv)
1.132 Differential Calculus for JEE Main and Advanced

5 cot x - cos x
(c) (d) 2 [2015] 11. lim equals
(p - 2 x)3





x Æp / 2



4
(1 - cos 2 x )(3 + cos x ) (a) 1/4 (b) 1/24
5. If lim is equal to







x Æ0 x tan 4 x (c) 1/16 (d) 1/8 [2017]






(a) 4 (b) 3 12. For each t ŒR, let [t] be the greatest integer less than






1 or equal to t. Then
(c) 2 (d) [2015]
Ê È1˘ È2˘ È15 ˘ˆ





2
lim x Á Í ˙ + Í ˙ º + Í ˙˜
xÆ0 + ËÎ x˚ Î x˚ Î x ˚¯


1
6. Let p = lim 1 + tan 2
xÆ0+
( x ) 2x , then log p is equal to (a) is equal to 0




(b) is equal to 15



(a) 2 (b) 1 (c) is equal to 120






1 1 (d) does not exist (in R) [2018]




(c) (d) [2016]





2 4 x tan 2 x - 2 x tan x
13. lim equals

xÆ0 (1 - cos 2 x)2

2x
Ê a 4ˆ
7. If lim Á1 + - 2 ˜ = e3. Then a is equal to
x Æ• Ë x x ¯ (a) –1/2 (b) 1/4






3
(a) 2 (b)
ps (c) 1/2 (d) 1 [2018]









2
el
1 2 (27 + x)1/ 3 - 3
(c) (d) [2016, online] 14. lim equals
eh

xÆ0 9 - (27 + x)2 / 3









2 3
je

1 1
(1 - cos 2 x )2 (a) - (b)
iit

8. L = lim is




x Æ 0 2 x tan x - x tan 2 x
3 6


@

1 1 1
(a) 2 (b) - (c) - (d) [2018]





6 3




2

(c) –2 (d)
1
[2016, online] 1 + 1 + y4 - 2
15. lim





2 yÆ0 y4


3x - 3 1
9. lim is equal to (a) exists and equals
2x - 4 - 2


x Æ3


4 2
(b) does not exist


(a) 3 (b) 1/ 2 1




(c) exists and equals


3 1 2 2
(c) (d) [2017, online]
1





2 2 2
( )
(d) exists and equal [2019]
2 +1



2 2
1a + 2a +  + n a
10. If lim = =
[n + 1] a -1
[(na + 1) + (na + 2) +  + (na + n)] 16. For each x ŒR, let [x] be the greatest integer less than

nƕ


1 or equal to x. then
for some positive real number a, then a is equal to

60
x([ x ] + | x |) sin [ x ]
(a) 7 (b) 8 lim is equal to
x Æ0 -

|x|




15 17 (a) –sin 1 (b) 0
(c) (d) [2017, online]









2 2 (c) 1 (d) sin 1 [2019]





FREE BOOKS FOR JEE & NEET =>(@iitjeeadv)
Limits 1.133

17. For each t ŒR, Let [t] be the greatest integer less than x cot (4 x )


or equal to t, then 19. lim is equal to
x Æ0 sin 2 x cot 2 (2 x )



Êp ˆ (a) 2 (b) 0
(1 - | x | + sin |1 - x |) sin Á [1 - x ]˜





Ë2 ¯ (c) 4 (d) 1 [2019]






lim
x Æ1+ |1 - x | [1 - x ] cot 3 x - tan x

20. lim is
cos ( x + p / 4)



x Æp / 4
(a) equals –1 (b) equals 1




(c) does not exists (d) equals 0 [2019] (a) 4 (b) 8 2










(d) 8 (d) 4 2 [2019]






18. Let [x] denote the greatest integer less than or equal to


x. Then p - 2 sin -1 x
21. lim is equal to
x Æ1- 1- x



tan (p sin x ) + (| x| - sin ( x[ x ]))
2 2
lim 1 p
x Æ0 x2

(a) (b)





2p 2
(a) equal p (b) equals 0




2
(c) equals p + 1 (d) does not exist (c) (d) p [2019]
p









ps
ANSWERS
el
eh
je
iit

Practice Problems—A Concept Problems—C


@

1. (i) 1 (ii) does not exist 1. 1







2. (i) 0 (ii) does not exist 2. L.H.L. = 1, R.H.L. = 2, lim f(x) does not exist.

x →1




(iii) –1 3. (i) lim f(x) = does not exist




3. No, we donot conclude anything. x→0

4. No, no, we donot conclude anything. (ii) lim f(x) = 0,


x→1

5. (a) No (b) No
(iii) lim f(x) = 1




(c) Yes (d) 0


x→3




7. π/2 8. 0 4. (a) 1 (b) 1








9. does not exist (c) 1 (d) 2





5. (a) 2 (b) 2
Concept Problems—A




(c) 1 (d) 2




1. (i) – ∞ (ii) –∞ 6. (i) does not exist, 0 (ii) 0








(iii) 1 (iv) 1 
(ii) 




2. (i) does not exist (ii) –5 7. (i) 0




2




(iii) 4 (iii) 1/2 (iv) does not exist





3. ∞ 4. does not exist. 8. limit does not exist since function is undefined in the





5. 0 6. 0 neighbourhood.




7. does not exist 8. 0 9. 0 11. 0 , 1






12. does not exist 13. 8
Concept Problems—B



14. (a) 3 (b) does not exist



2. –1 3. 1/π (c) does not exist




4. 0 15. –3


6. since limit of g(x) does not exist. 16. lim f(x) is not known but lim f(x) = –7.
x  2

x 2

FREE BOOKS FOR JEE & NEET =>(@iitjeeadv)
1.134 Differential Calculus for JEE Main and Advanced

Practice Problems—B Concept Problems—F


17. (i) –1/3 (ii) 1 1. 2






18. (i) does not exist (ii) 0 2
2. (i) − 5 (ii)




π





3
19. (i) (ii) 0 1 1




2 (iii) (iv)


20. 0 21. –2 3




2




22. 0 23. does not exist.
3. –1/4 4. 1




24. 1/2, 0





25. (i) ∞, ∞ (ii) ∞,0 5. a = 2, b = 1






(iii) 1/2, 0 (iv) 0, 0
Practice Problems—E




26. (i) –2, 2 (ii) 1, 4




(iii) a2, a2– 1 (iv) 2, 5 6. loga6







27. (i) 2, 2 (ii) 10n –1, 10n 1
7. (i) 7 /4 (ii)








1 1 8 3
(iii) ,– (iii) 32 (iv) 1.2




2 2




Concept Problems—D 8. –
1
9.
34





0 3 23
1. (i) determinate form (ii) form 10. (i) 2/3 (ii) 1/6




0



(iii) (0 × ∞) form (iv) (∞ – ∞) form (iii) 9/25






(v) (0)º form (vi) (∞)º form
Concept Problems—G




(vii) (1)∞ form (viii) determinate form
ps




5. (i) 0 (ii) 0 1. (i) –1/9 (ii) – 2/3
el







(iii) 0 (iv) 0 (iii) 0 (iv) 2








eh

7. 7 2. (i) 1/3 (ii) 24







8. (i) 4 (ii) –2 (iii) 1/3 (iv) – 1/2
je








2
Practice Problems–C 3. 4. 1/2
iit




5
9. 0 if a < 0, dne if a ≥ 0 5. limit does not exist.
@


10. (a) 0 (b) ∞
Practice Problems—F




(c) 0 (d) – ∞




(e) ∞ (f) – ∞ 7. (i) –1 (ii)0






11. ∞ 12. does not exist (iii) ∞ (iv)–1







8. (i) a/m–b/n (ii)a/m + b/n
Concept Problems—E




(iii) –19/3 (iv)∞




15 9. (i) 1/80 (ii)–1/5040
1. (i) (ii) – ∞ (limit dne)




(iii) 1/2 (iv)– 11/6




11




(iii) 2x (iv) ∞ (limit dne) 10. 13/60 11. 1/2







2. (i) 0 (ii) ∞ (limit dne) 1 1 9
12. − , 1,




3. (a) 0 (b) 3/4 13. –3,
2



2 24




(c) dne (d) 2/3




4. ∞ (limit dne). 1 3 1
14. 72, a = ,b= ,l= −

4

5. f (5+) = 1 , f (5–) = 0 ; limit dne 4 32

6. (i) 1 (ii) – ∞ (limit dne) Concept Problems—H





(iii) 0 (iv) 1
π




Practice Problems—D 1. (i)
180
(ii) 0




7. (i) 1 (ii) –1/4 2. (i) 2/5 (ii) 8/3








(iii) 1 (iv) –7.2 (iii) – 3/2 (iv) 25/6








1 n
8. (i) (ii) –3 3. (i) 4 (ii)




4




m
(iii) 3/4 9. c = – 6 limit = 4.




8
10. (i) 2, 4 (ii) 0, –1 (iii) log2e (iv)








7

FREE BOOKS FOR JEE & NEET =>(@iitjeeadv)


Limits 1.135

1 1 1
4. (i) (ii) 1 31. (i) (ii)





ln 3




2 2

1 1 a
5. (i) − (ii) ln 32. (i) 5 2 (ii) 1









m n b 33. (i) –2 (ii) ∞





2 (iii) –1/12 (iv) 3/2





6. (i) (ii) –1




3
Concept Problems—I
(iii) 4 (iv) 4/5 1. (i) 0 (ii) 8










3 (iii) 2 (iv)





7. (i) No (ii) No 2. (i) 25 (ii) 1









(iii) No (iv) 0 (iii) 1 (iv) 9









8. (i) 1 (ii) –2 3. (i) 3 (ii) 0









(iii) 3/4 (iv) 1/8 
(iii) 

(iv) 1



9. (i) 1/2 (ii) 25/9 2









(iii) 2 (iv) 3 5. (i) 0 (ii) –3








10. (i) ln(8/7)/ ln(6/5) (ii) 9/4 (iii) 1 (iv) – 1/3








(iii) (ln 5 .ln 4)/(ln 3 .ln 6) (iv) 1/3 6. 0







11. (i) ∞ (ii) 1/2
Practice Problems—H




12. 1/2

13. (i) − 2 / 4 (ii) 1
ps
2 , − 2




7.

el
1 4
(iii) − (iv) ∞ 4




eh

4
 0, if m  n
2 1
14. (i) (ii)  3, if m  n
je

π 




e 8. lim it  
 , if m  n and n  m odd
iit

(iii) – ln 2 (iv) 5




, if m  n and n  m even
@

Practice Problems—G
17. – 2 18. tan10–10 2 1
9. (i) (ii)








19. 0, if n > m, 1 if n = m, ∞ if n < m. 3 24

20. – sin a (iii) 510/325 (iv) ∞





1
21. (i) a (ii) 324 10. 5 11. 2








12. (i) ∞ (ii) ∞
(iii) – 6 (iv) 1





(iii) 0 (iv) 0







22. (i) 3 (ii) 2/3 13. (i) 1 (ii) 0




(iii) 2/3




(iii) does not exist (iv) 0


23. (i) 3/5 (ii) 3 ln 3




14. 100 15. 1




(iii) 16





16. –2
24. 0 for n = 1; 4.9 for n = 2


25. (i) –1/3 (ii) ln 4 Concept Problems—J




sin 2β 1. (i) x = –2, y = 2 (ii) y = 2
26. (i) (ii)  a





2β 



Y

27. (i) –∞, 1 , No (ii) − 2 , 2 , No






28. 1 1
2. (i)

1


29. (i) 2 (ii)
2




–2 –1 1 2 X
(iii) aa ln ea


30. (i) 1/3 (ii) 1




FREE BOOKS FOR JEE & NEET =>(@iitjeeadv)
1.136 Differential Calculus for JEE Main and Advanced

Y Practice Problems—I
1
6. (i) (ii) 3





5
1
(ii) (iii) 0 (iv) 1






–2 2 X 4
7. 1 8. (i)




3
1 1
(ii) (iii)





Y 2 4
9. log 22 x 10. 3




y= 21
x –x 1
1 11. (i) − (ii) –1





3. (i). 2
2


1
2 X
sin x
Asymptote x=0 Asymptote x=1 (iii)



x
Concept Problems—L
–4

1. (i) 0 (ii) 2a / π




Y
(iii) a/π (iv) 0




y= 41 2 2. (i) 0 (ii) 0




x –x
ps (iii) 0



1
3. (i) – 1/2 (ii) 0




el
1 2 X (iii) 0 (iv) 1/2
(ii)




2
–1
eh


Asymptote x = –1 Asymptote x = 1 4. (i) 0 (ii) 0




(iii) − 2 (iv) 0
je




–4
5. (i) ∞ (ii) 2/3
iit




(iii) – 3/5 (iv) (a – c)/2




@

6. Both are wrong. The limit is –13.



Y 1 1
7. (i) (ii) −




Asymptote y = 1 2 2
(iii) y= x(x–1) 3


2
x +1 (iii) – (iv) 0




2
–2 –1 0 1 2 3 X 8. (i) ∞ (ii) 4/3




Y (iii) 2



9. (i) ln a


(ii) ∞ for n > 1 ; a for n = 1 ; 0 for n < 1


4. (i) e–1 (iii) a (iv) 0






1 2 X Practice Problems—J
Y 11. (i) 2/9 (ii) − 5




2
(iii) − 1 (iv) ∞




(ii) 2


12. (i) 1/2 (ii) 2
1




X
π2 a+b+c
(iii) (iv)
Concept Problems—K




8 3
1. (i) 5 (ii) 8/3 1
13. 1, 14. 2




(iii) ∞ (iv) 4



2




2. (i) 1/2 (ii) 1/3 15. a = 1, b = –1. 16. 5







(iii) 5 (iv) 1/2 4




FREE BOOKS FOR JEE & NEET =>(@iitjeeadv)
Limits 1.137

1 Concept Problems—O


17. (i) –2 (ii) ln  
2




1. 5 2. limit is 1
π





2
(iii) (iv) –π /4 3. limit is 1 at x = –1 and 1; limit is 0 at x = 0.




4


1 1
1 5. g(x) = − , h(x) = , limit = 0
x2 x2


18. (i) (ii) 1




4 2
6. b




1
(iii) −
Practice Problems—M


4
19. (i) 0 (ii) 1/2 8. 0 9. 4








(iii) 1/120 10. (i) 0 (ii) 0






(iii) 2 (iv) dne
Concept Problems—M





1
2 11.
1. 2. 0 2





3 13. (i) 1 (ii) 0





3. 0 4. 0 (iii) 0 (iv) 0







1 x
5. 5 6. 14. 15. 1
e








3
Practice Problems—K 16. they have same limits as before.

1 1
ps
17. 0 18. 1
7. 8.







4 4 19. 1
el
9. e2 10. 1

Concept Problems—P



12. eπ
eh

11. 0




13. 1 14. 99 1
1. (i) (lna)2 (ii) 1
je








2
Concept Problems—N
iit

(iii) 1
(i) e–1


2. (ii) e
@

2. (i) 1 (ii) 1








2
(iii) − (iv) e3 a
3
e (iii) 2 (iv) ln




b




1 e 3. (i) 0 (ii) 2/3
3. (i) (ii)








e (iii) –2


(iii) ae (iv) 1 5. 1/2





4. (i) e5 (ii) 1 6. (i) 2/3 (ii) π2/4




∞ if n < 1; e if n =




(iii) 1; 1 if n > 1.


(iv) 1/e (iii) 5




Practice Problems—L 7. (i) 1/3 (ii) 1/e




(iii) 1/2


m2 n
5. (i) − (ii) e1/6
Practice Problems—N




e 2

(iii) e −x2 / 2
(iv) n! 8. (i) – 9 (ii) 2/3








6. 0 if 0 < a < 1; lna if a > 1. 1

7. (i) 1 (ii) e–1/2 9. (i) 4 (ii) –




2




(iii) 1 (iv) e–1/3
14. (i) 1/18 (ii) 2/3




–1/2
8. (i) e (ii) e1/2








e 16. esin 2
y 17. 0
(iii) e1 (iv)



e1− e




n −1
9. (i) 1 (ii)
e 2a
Concept Problems—Q




–2 2α
(iii) e (iv) e e 1




3. 4. 1
10. λ = 1, µ ∈ R e −1



3

FREE BOOKS FOR JEE & NEET =>(@iitjeeadv)
1.138 Differential Calculus for JEE Main and Advanced

Practice Problems—O 19. A 20. B 21. B







22. B 23. C 24. B







7. lim ACn  8. ab 25. A 26. B 27. D










n  3 2 28. D 29. C 30. B







Ct 31. A 32. C 33. B
9. cos t







34. D 35. B 36. B

2







37. A 38. D 39. A







n
10. Q(t n )  Q0 1  kt  , where k is the proportionality factor 40. A 41. B 42. A








n   43. D 44. C 45. B







(law of compound interest) ; Qt = Q0ekt. 46. B 47. D 48. C







49. B 50. D 51. D
Concept Problems—R







52. D 53. B 54. A







1. If x ≠ kπ (k∈Ι), then cos2x < 1 and y = 0 ; but if x = kπ, 55. B 56. D 57. B







58. C 59. A 60. A

then cos2 x = 1 and y = 1.







61. C 62. B 63. B

(i) y = 1 when 0 ≤ x ≤ 1; y = x when 1 < x < ∞.







2. 64. D 65. B 66. D








(ii) x > 1 67. A 68. A 69. B








(iii) y = |x|
π 70. A 71. C 72. C








(iv) y = π when x < 0; y = when x = 0; y = 0 when 73. B 74. A 75. B
2








x > 0. 76. ABD 77. BD 78. ABD






3. 2 79. CD 80. ABCD 81. ABCD







82.
ps ABCD 83. A 84. A
Practice Problems—P






85. ABCD 86. AB 87. BC






88. BC 89. ABD 90. BCD
el






sin x x0 91. BC 92. ABC 93. BCD
eh

lim  0 ; lim = π






4. f(x) = ; x −
94. ABCD 95. ABD 96. D







x→ 0

0
97. A 98. A 99. B
je

tan x 2






x0 100. C 101. A 102. D
x2
iit






103. B 104. D 105. D






; limit does not exist 106. B 107. A 108. A
@








109. C 110. A 111. B






5. (a) (b) 112. D 113. C 114. A










115. D 116. A 117. B






118. C 119. C 120. D






121. (A)–(Q) ; (B)–(R) ; (C)–(P) ; (D)–(P)
a c


6. 0 7. d f  122. (A), (Q) ; (B)–(P) ; (C)–(R); (D), (S)





  123. (A)–(S) ; (B)–(R) ; (C)–(P) ; (D)–(Q)
8. x ∈ [0, sin 1)


124. (A)–(S) ; (B)–(P) ; (C)–(Q) ; (D)–(R)




9. φ(x) = 1, (x > 0); φ(x) = 0, (x = 0; φ(x) = –1, (x < 0). 125. (A)–(R) ; (B)–(S) ; (C)–(P) ; (D)–(Q)


10. ln 5 11. 40
REVIEW EXERCISES for JEE ADVANCED



14
12. 13. 1/ π




3
2 sin a
14. 4 15. 1 1. (i) (ii) –1




cos3 a



e
16. 8 17. 1 2. (i) 7/4 (ii) − 4




5




3. (i) 1/3 (ii) 6
OBJECTIVE EXERCISES




1 1
4. (i) 8
(ii) −




2 9
1. A 2. A 3. B






4. C 5. B 6. B 2n !
5. –cos a 6.






7. C 8. B 9. A



(n !) 2






10. C 11. A 12. A






13. D 14. A 15. B 7. e2 8. 2










16. C 17. B 18. B 9. (1/2) 10. cos–1x0









FREE BOOKS FOR JEE & NEET =>(@iitjeeadv)
Limits 1.139

1 2 24. If m is a positive integer, u n = 0 for n > m.



11. 12. –
u mn



24 3
Otherwise n 1  x   x , unless x = 0
1 un n 1
13. – 1 15.




2 1, n  I ψ( x ), n ∈ I
16. 1/2 25.  26. 
0, n  I  φ( x ), n ∉ I



2
17. (a) − 3 2 (b) 5
28. If x is rational then sin 2 (m!nx), and therefore




2 3


sgn{sin2(m!πx)}, is equal to zero from a certain value
−7e
19. 20. 4–2, 4–4 of m onwards; if x is irrational then sin 2(m!πx) is



16 always positive, and so sgn {sin2(m!πx)} is always
1 equal to 1
21. φ(x) = f(x), (|x| > 1); φ(x) = g(x), (|x| < 1); φ(x) = {f(x)
30. 1 + 1 + 4a

2
+ g(x)}, (x = 1); and φ(x) is undefined when x = –1. 29. 4
π




2
22. a+b–1

31. 0 32. e





23. 20

24. 1 if 0 < a < 1; a if a > 1 34. (k + 1)–2



PREVIOUS YEAR'S QUESTIONS
25. The function is equal to 0 when x is rational and to 1

when x is irrational.
27. The function is equal to 0 except when x is an integer, ps (JEE ADVANCED)

and then it is equal to 1.

2 1
el
26. m = 1, b = – 1. 1 2. –1 3. 4. e5

3







128r
eh

28. The function is equal to φ(x) unless x is an integer, and 5. e2 6. 1 7. –1 8. F




then it is equal to Ψ(x).






je

9. C 10. B 11. D 12. B









30. The former limit is 0, the second does not exist. 13. B 14. C 15. B 16. C
iit








 17. C 18. B 19. C 20. D
3







32. 4 · e 33. 4
@

3 21. C 22. A 23. B 24. D












25. B 26. 7 27. 2 28. B







34. 400 35. 256
2



29. C 30. AC 31. B, D 32.
TARGET EXERCISES for JEE ADVANCED







3 3
33. a2 cos a + 2a sin a 34. 2 ln 2



1. 1 2
35. 1–

–1/a for a ≠ 0, for a = 0 there is no limit. π

2.

p-1+ alna 3
3. 4.
e 2a QUESTIONS FROM PREVIOUS



2
YEAR'S (AIEEE/JEE MAIN PAPERS)
5. 41 6. –24



7. –1 8. does not exist



10. If n is even, limit does not exist. If n is odd limit is 0)

11. ∞ 12. 3 1. (c) 2. (d) 3. (d) 4. (b)




n3








 0 if 5. (c) 6. (c) 7. (a) 8. (a)









14. 1 / 2 if n3 16. 3 9. (b) 10. (a) 11. (c) 12. (c)



  if








 n3 13. (c) 14. (c) 15. (a) 16. (a)








1 17. (d) 18. (d) 19. (d) 20. (c)








17. 18. a2
21. (c)



2


21. 1 ln 2 23. − a



2

FREE BOOKS FOR JEE & NEET =>(@iitjeeadv)


HINTS & SOLUTIONS

Objective Exercises
Single Correct Answer Type

1. Lim 
 sin x 
 Lim
e sin h

 1 1  sin 2 h 
x 0  x 

1
 sin x
h 0 2 tan sin h
RHL as x  0 ,  1 1




x
 sin x  2


So, Lim  0 Hence, C is correct.
x 0  x 





 b

x
sin x Lim 1  a 2  b
as x  0 ,  1
 
LHL 5. x  x
1 a2





x
 sin x 
0  
x
So, Lim   As 1  a 2   for x  
x 0  x 



 


Hence, B is correct.


Hence, A is correct.
  2x 2  1  

 x2  6. Lim x 2  tan 1  2   tan 1 2   0   form
 
 x 2 

2. Lim   x   

x 0  sin x.tan x 
 
x x  2x 2  1 
as x  0 ,   1 tan 1  2 1
  tan 2
RHL
ps




sin x tan x  x 2  0
 Lim  form
el
x x x  1
LHL as x  0 ,   1 x2
0
eh



sin x tan x

Apply L – Hospital Rule.
 
je

x2
 x  2  4x    2x  1 2x  

So, Lim  0 2 2
iit
  

x 0  sin x tan x  1 1
  Lim 
x  2 x 3
x  2 2
 2x 2  1  2
2
@

Hence, A is correct. 1   2 

 x 2 
3. Lim x 4  x 2  1  ax 2  a

x 3  6x 

x  1 1 3
Lim    3  
     
2 x  2 2 2
2x 2  1  x 2  2 5 5
x 4  x 2  1  ax 2  a
Lim


x 
x 4  x 2  1  ax 2  a Hence, B is correct.

1  a  x   2a  1 x  1  a 
2 4 2 2 2 7. 
f  x   a x 2  2 

Lim
   3a 32
x  2
x 4  x 2  1  ax 2  a f  
2

4 4
For limit to exist, coeff of x 4  0  a 2  1
Hence, a  1
( )
− 2a 2 + 1 x 2 + 1 − a 2 ( ) ⇒ − (2a + 1)
2

Lim  x    x  
x →∞
x 4 − x 2 + 1 + ax 2 + a 1+ a Lim
x   sin sin x

Put x =    h
⇒ It exist only for a = 1
( −2π + h )(h ) × sinh = 2π


Hence, B is correct. Lim
h →0 − sin sinh sinh

esin h  1  sin h  Hence, C is correct.
Lim
 tan 
4.

h 0 2
1 20

Lim  cos 2 n  x  10 
sin h
8.
n 

Apply L – Hospital Rule x 1

esin h cos h  cos h  Lim cos 2 n 9  cos 2 n 8  ....  cos 2 0  cos 2 1  ...  cos 2 10 
Lim n   
h 0 1
2 tan 1 sin h   cos h All except cos20 will tend to zero & cos20 = 1

1  sin 2 h Hence, B is correct.


FREE BOOKS FOR JEE & NEET =>(@iitjeeadv)
Limits 1.141

 3x  π
1  sin    tan θ / 2
 1  x 2   0 form a sin θ θ/ 2 × 2
9.
Lim Lim × 2 ×

1  cos x θ→ 0 +
π θ π θ
x 1 0 tan  tan  tan θ / 2
2 2 2
Apply L – Hospital Rule,
4a


 3x  
1  x 2  x.2 x   π


cos     3  Hence, D is correct.
 1 x2 
 
2


1 x2 2  tan x  sin x   x 3
Lim 14. Lim
x 1  sin x x 0 x5



Use series expansion,
1  x  1  x    3x  


Lim  3    cos     0     
1  x 
x 1 sin x 2
 1 x2   x3 2 5 1 x5
2   2  x   x  .....    x  x 3   ....    x 3
  3 15   6 120  
Lim 
1


x 0 5
x 4
Hence, A is correct.

Hence, A is correct.
cos  2 x  4   33
10. Lim  16 1  cos x  2 sin x  sin 3 x  x 2  3x 4

x  2 2 15. Lim
tan 3 x  6 sin 2 x  x  5x 3

x 0

x 4x  8
2
4x x  22
ps Apply L – Hospital Rule
Lim  Lim  16

x 2 
x2 x 2 
x2
sin x  2 cos x  3 sin 2 x cos x  2 x  12 x 3
el
2

Lim
3 tan 2 x sec2 x  12 sin x cos x  1  15x 2
eh

x 0
Applying Sandwich Theorem,


Hence, B is correct.
je

Lim f  x   16

x  2 log cos x 0
iit

16. Lim p  form



q
x 0 cos x  cos x

Hence, C is correct. 0
@

11. Lim (1 + x ) { 2 /x
} Lim
x →0
1
cos x
× ( − sin x ) × 1
1
x →0
(cos x ) P −1 ( − sin x )

1
P
As Lim (1 + x )
2 /x
→ e 2 & e 2 ≈ 7.29 1
− (cos x ) q ( − sin x )
x →0
1 −1

q
So, Lim (1 + x )
2 /x
= e2 − 7 pq
x →0 ⇒

q−p
Hence, A is correct.


Hence, C is correct.

 3 −1 −1 
 x − 4x   x + 2x 2    1  1  1   1 
12. Lim  3  −  x−2 −
x → 2  x − 8 
 17. Lim 1   1  2  1  4  ............ 1  2 
 x − 2  n   5  5  5 
n

 5 


 
 1
multiply & divide by 1  
Lim 

  x  2  x 2  4  2x

 x  2  x  2  1 
   5
x 2  x  x  2  x  2 x  x  2  2  x  2  2  
 
1 − 1 
( )

 2
 
n
Hence, A is correct. 52 5

Lim ⇒
 ax  n →∞ 4/5 4
13. Lim a 2  x 2 cot  

x a 2 ax



  Hence, B is correct.

Put x = a cosθ
( )
1/x
 x ln (2 −1) x x 
 e − 2 x − 1 sin x 
π  18. Lim   ⇒ 1∞ form
Lim a sin θ cot  tan θ / 2 e x lnx

x →0
θ→ 0 +
2   

FREE BOOKS FOR JEE & NEET =>(@iitjeeadv)
1.142  
Differential Calculus for JEE Main and Advanced

Lim
 e x ln  2x 1  2x 1 x sin x  e x lnx 

x 0 
 
xe x lnx

 Lim
eay  e by
 Lim
eay  1
 Lim
e by  1    
e   y 0 y y 0 y y 0 y


  2x 1x  2x 1x sin x  x x 
Lim    ab
x 0
 x .x x 


e  Hence, B is correct.



  2x 1  x 
  1 
3
1  tan 1 3x  3 1  sin 1 3x
 2x 1  sin x 
x

  x 

21. Lim
Lim  


x 0 


x  x  x 

x 0
1  sin 1 2 x  1  tan 1 2 x
 
e Rationalise



 ln 2 
ln  
tan −1 3x + sin −1 3x
e
ln ln 2 1 ln 2
( )
Lim
 e  e   x →0 − tan −1 2 x + sin −1 2 x
e
( 1 − sin )

−1
Hence, B is correct. 2 x + 1 + tan −1 2xx

×
(1 + tan ) ( ) (( )( ))
1/n
 n!  2 /3 2 /3 1/3
−1
+ 1 − sin −1 3x + 1 + tan −1 3x 1 + sin −13x
1/n
1  n!  3x
19. Lim   = Lim  n 
( ) 
n →∞  mn n  →∞ n 

m n
6 2
  1
n 43
 n! 
Let y  Lim  n 

n   n  Hence, B is correct.
ps

 ln 1  x   ln 2  3  4x 1  x 

1 n r
el
 ln y  Lim  log  
n
Lim
n  n  7  x 1/ 3  1  3x 1/2  sin  x  1
eh

r 1 22. x 1
 


1
je

  log xdx Put x  1  h


iit

 ln  2  h   ln 2   4h  1  h  
0

 

@

 x log x  x 1
 1  lim x log x 3 lim
0
x 0 h 0  8  h 1/ 3   4  3h 1/ 2  sinh
 

log x
= −1 − lim

x →0 1/x 
ln 1  h
2 4 h
1 h 

 1  lim   x  3 lim
 8  h  1/ 2 
h 0
  4  3h 
1/ 3
x 0 sinh
 

= −1

3    ln 4  1
1
1
So, y = 2
e 1 2 / 3 1
8  3   4 1/ 2

1/n
 n!  1
3 2

Lim   =
( ) 
n →∞  mn n  3 ln 4 / e 9 e
me  ln
2  1 3 4 4
 12  4 

Hence, A is correct.  

 x ln 1  
 ay   by 
x ln 1 

 e  x 
e  x   Hence, B is correct.
20. Lim  Lim 

y 0  x  

y 1 2
  x 5 tan 3 x 7
23. lim x 2
3

x x x  x 7 x 8
 ay   by 
1  x   1  x 
Lim Lim     1
Put x  
x  y 0 y y

FREE BOOKS FOR JEE & NEET =>(@iitjeeadv)
Limits 1.143

1 y2 3 1 + 1/ x 1
− tan + 2 +7 = lim ⇒
y 5
π y x →∞ 2
lim x+ x
y→0 1 7 1+ +1
+ +8 x
y3 y


So, K = 1


y2 Hence, A is correct.
7 y  3y  tan
5 3


lim 
y 0 8 y5  7 y 4  y 2  ln x 10 
26. lim x 4
 ln x  10
 lim  form
4




x  x  x
y2 /π
7 y3 + 3y − tan Apply L – Hospital Rule,
y2 1


lim =−
8 y3 + 7 y 2 + 1 π 10  ln x 
y→0 9
lim

Hence, C is correct. x  4x 4

 




cos 2 1  cos 2 1  cos 2 ..........cos 2 x Applying L-Hospital again and again, will reduce the power
of ln x to zero while power of x remains 4.
24. xlim
0   x  4  2  Hence, finally, the limit tends to zero.

sin     
  x   Hence, B is correct.

   tan  x  1 log ex


x 1


cos 2 1  cos 2 1  cos 2 ..........cos 2 x
ps
27. lim
x 1 3
x 1

lim
   x  4  2  x  4  2 
el
x 0
sin   Put x = 1 + h
eh

x  x  4  2

 
   tanh  h  log 1  h  
je

lim

 
3

h 0

 
h
iit

cos 1  cos 1  cos ..........cos x


2 2 2 2

RHL ⇒ lim tanh .log 1  h   1
 2
@

lim
x 0   
sin  

h 0 h h
 x4 2
 tanh h log 1  h 

Hence, B is correct. LHL ⇒ lim    1

  
h 0  h h

h
a 2 x 2  ax  1  a 2 x 2  1  a 2 x 2  ax  1  a 2 x 2  1
Hence, D is correct.
 
25. lim

x 

a x  ax  1  a x  1
2 2 2 2

1 1 1
ax
28. lim  2
 1    x ln 1  a1/ x
x 
 

x 0 x
 lim  x
x 
a 2 x 2  ax  1  a 2 x 2  1
1+ x2 −1
( )

a 1 ⇒ lim + lim x ln 1 + a1/x
 lim  x →0 x 2 x →0
x  a 1 1 2

a2   2  a2  2
x x x
⇒ lim
x2
+ lim
(
ln 1 + a1/ x )
( 1 + x − 1)

x →0 x →0 1/x
    x2 2
 x  x  x  x  x  x  x  x 
Now, lim    

x  
 x x x  x
 If a   0,1  lim x ln 1  a1/ x  0
x 0
 
 
1
x x Hence, Ans is

 lim 2
x 
x x x  x If a  1,  

FREE BOOKS FOR JEE & NEET =>(@iitjeeadv)
1.144  
Differential Calculus for JEE Main and Advanced


ln 1  a1/ x    form Apply L – Hospital Rule


1 x 3 3x
lim 2 m lim cos  sin
x 0 1/ x  e x 0
m m m m

1 a1/ x ln a  1 / x 2  e2
 lim 


x 0 1  a1/ x 1 / x 2  1 
lim  x  x 2  3x cos 

33.
ln a x   x 


= lim = ln a 
x →0 a −1/x + 1
1

1 Put x  
Hence, Ans is + ln a y

2


 1 1 3 
 n 4n  1 
n
lim     cos y 
29. lim  2  n 0 y 0  y y 2
y 
n   n  2 2 1   

 


Hence, C is correct. 1  3y cos y  1
lim

 
g x  y 0 y
30. lim log sin x x

3y cos y

x 0 3
lim 
As x  0  g  x   0
 y 0 y  1  3y cos y  1  2


 
0
Hence, B is correct.
Hence lim log sin x x 1
ps

x 0 

 1 
el
34. lim  x  x 2  x 2 sin 
Hence, B is correct. 

x  x
 

eh

31. lim n 2  a    a  


1 1
n n 1

  1
Put x  
je

n 
y

iit

 1/n  1 
lim n 2 a1/n 1 a n 1  1  1 1 1 
@

n 
  lim     sin y 
y 0  y y 2
y2 
 

 1 

1
n  n 1
a n 1  n 2  a  1 1  sin y  1
  lim
  1 y 0 y
lim
n  n  1

n  1
 sin y 1
n  n  1 lim 
y 0 y  1  sin y  1  2

 1 

1
 a n  n 1  1  1 Hence, D is correct.

lim a n 1   
n   1 / (n  1)  1  1 / n x 2 n sin n x
35. lim
  x 2 n  sin 2 n x

x 0

⇒ ln a x 2 n sin n x
lim
x  

Hence, A is correct.
x 0 n
 sin n x x n  sin n x


2m
sin n x/ x n
32. lim  sin  x   cos 3x  x lim
m x 0 
 sin x    x  sin x 

n
x 0    m n n
1     
  x    x . x n n

 x 3x  2 m
sin  cos 1  
x 0 
lim
 m m  x

e
For limit to exist,


 x 3x 
 sin m  cos m 1 x n − sin n x
2 m lim   lim should exist and non zero.
e x 0 x x →0 x 2n


FREE BOOKS FOR JEE & NEET =>(@iitjeeadv)
Limits 1.145

n sin x 2
 1 x5  lim =
x n   x  x 3   .......  x→
π  −13 
cos sin x  π
 6 120  2
lim 2 n  π 
x 0 x ⇒  Denominator will be exactly while numerator will tend to 1
 2 

For n = 2, this will be non zero.

Hence, A is correct.
Hence, B is correct.



cot 1 a 9 log a x 

ax 40. lim
lim 1
a a


36. x  x  sec x
log x
x  b2 x 2  x

1 log a x
Put x   As lim a
 0 & lim a x log x a  
x  x x 

y


a
1
Thus lim

cot 1 x  a log a x   / 2 1
lim
y 0
y
 lim
y 0
ay  1 x  sec 1
a x
log x a  / 2
1 b2 1 1  b2  y

  Hence, A is correct.
y y2 y
n
 p1/n  q1/n 


1 1 41. nlim    1 form
  as b  0   2
 

1 b 1 b ps

p n 1 q n 1
1 1
Hence, B & C are correct. lim
el

1/n
n 
2 1
n
 m n e
37. lim   r 
eh
n  ln p  ln q

 r 1 
 pq
je

e 2

 

1/n
iit

 lim 1n  2n  .............  m n Hence, B is correct.



 
n 
cot 1 x 1  x
@

1/n 42. lim
  1   2  n  n
  2x  1  

x  x
 lim  m n        .......1  sec 1   
n    m   m     x  1  
    

 

  1 n  2 n 
1/n
As lim cot 1 x 1  x

x 
 m lim        .......1
n    m  m 
   1 
 lim cot 1  

=m x   x 1  x 

Hence, A is correct. 
 and

 2
k  
n n
k  

38. lim   cos    cos    0
n    4   6     2 x + 1 x  
−1  2 + 1/x 
x


 = lim sec −1   =
  x →∞
lim sec    
x →∞   x − 1    1 − 1/x  2
kπ kπ
This will always hold true if cos or cos is not equal

to ±1. 4 6
lim
cot 1  x 1  x  1
Hence K ≠ 4 K1 or 6K1   2x  1  
Hence, x 
x
So, K is not divisible by 4 & 6. But K can be a multiple of 24. sec 1    

 
Hence, D is correct.   x 1  
Hence, A is correct.
sin x

39.
lim
 1  tan x  1 sin x
x cos 1   3 sin x  sin 3x   43. lim
x x  1

2 x 0

 4 

FREE BOOKS FOR JEE & NEET =>(@iitjeeadv)


1.146  
Differential Calculus for JEE Main and Advanced

tan  x  1 sinx 1 r


6 1
 tan 1  x
6
RHL = lim
x 0 x  x  1 
n  n 
dx

12
24 lim  24 0

tan 1  x  1 sin 1  x 
n  n 3
 n  1  2n  1
3
2 7
LHL = lim  sin 1
1  x  1  x  1
7
x 0 n


Hence, D is correct.
Hence, D is correct.



  
x a b c
44. lim ln x   x  cos  cos cos
r r r  0 form
x 0


48. lim
r  b c


sin sin 0
lim  ln 1  x  1 
1 x
r r
x 0 
Apply L – Hospital Rule,



lim  ln  2  x  
1 x a a b b c c b c
 ln 2 2
sin  2 sin cos  2 cos sin
x 0 
lim r r r r r r r r

r  b b c c b c
Hence, C is correct.  2 cos sin  2 sin cos

r r r r r r
 100 x   sin x 
45. lim   99
x 0  sin x  x 
sin a /r sin b /r b sin c /r
  a2  b2 cos c /r  c2 cos

lim a/r b /r r c/r
sin x x r  b sin c /r c sin b /r
 bc cos  bc cos
=100 + 98 = 198 (As 1 & 1)
ps
r c/r r b/r

x sin x
el
Hence, B is correct.
eh

n
 1 

46. lim  1  2 n  b2  c2  a 2
je

n    

n 0 2
2bc
iit

 1
Multiple & divided by 1   Hence, C is correct.
@


 2

49. 100 1  P  x  1
lim

 1  1  1   1  x 0 x
1  2  1  2  1  2  .................. 1  2n 
lim 
n
  2 
 1
 2 
1  P  x    11/100 1/100


Px
1   lim
x 0 Px
 2 x


2 1
 1  ⇒
1   2n  100
2  2

lim
n  1 Hence, B is correct.
1

2 2    2  2  2  1    1
 0 0

50.
Hence, B is correct. 3    3


16  26  36  ..........  n 6
47. nlim 6 x 2 cot x cos ec2 x

 
12  22  ............  n 2 13  23  ..........  n 3  lim

51. x 0     
sec  cos x   tan    1

1  2  3  ..........  n
6 6 6 6
  4 sec x  
lim
n  n  1  2n  1  n  n  1 
n  2
x x
  . 6
6 2 tan sin 2x  3.
  lim
x 0     


sec  cos x   tan    1
n
r6  4 sec x  
lim 24 
n 3  n  1  2n  1
n  3
r 1 Hence, D is correct.


FREE BOOKS FOR JEE & NEET =>(@iitjeeadv)
Limits 1.147

 b t 1  a t 1 
1/t
Then, lim f  x   1  lim  x  1
x 0 x 0
52. lim  
 ba 


t 0

RHL = lim  x − 1 = −1
  b t 1  b   a t 1  a   1 x →0 +

lim   


t 0  
e 
t t  b a
LHL = lim  x − 1 = −1
x →0 −

 b  b t 1 a  a t 1  1


lim 
t 0 
 
t t  b a Hence, B is correct.
e  


1 cos  x 1

1
 x 3  2 x 2  x  1   x 1
1

b ln b  a ln a  bb  b a 2

ln  a  b b  b a lim  
e b a  e a   a  x 1 
a   x  2x  3 
2
  56.


1/22
5

Hence, D is correct.  

53. Put x = a tanθ 6

 2x  Hence, D is correct.
f  x   cot 1  1
  cot tan 2

 1 x2  57. f  x  
1


1 x
  tan 1 tan 2
2 1 1 x x 1
fof  x    
ps 1 x x
 1 x2  1
g  x   cos 1    cos  cos 2 
1
1 x
el
 1 x2
 

eh

1
fofof  x   x

  
 tan 1 tan 2     1
 x  1
f  x   f a 
je

lim  lim
2 2 2 x
x a g  x   g  a 
iit


 cos cos 2  /2

 1 

x  
e tan x  e e tan x  e x
fofof x
4
@

So, lim  lim


x 0 tan x  fofof  x 

  x 0 tan x  x
 tan 1 tan 2   2 
lim 2
 
1
 lim 2
 x  
 1  
e x e tan x  x  1  1
x     lim
4 cos cos 2
2 4 2
2 x 0  tan x  x 


Hence, B is correct. Hence, B is correct.

   ln 1  x  x 

ln 1  x  x 2 2
54. lim 7 29 5n  2 n 
58. s  lim   2  .......... 

x 0 sec x  cos x n   10 10 10n 



lim
  
ln 1  x  x  ln 1  x 2  x
2
  cos x 5r  2 r 1 1
r r
x 0 1  cos x 2 Tr      
10r 2 5


 

2

ln  1  x 2  x 2  r r
   cos x
n
1 1 n
1 n
1
lim S  lim        lim  r  lim  r
x 0 1  cos 2 x n 
r 1  
2  
5 n 
r 1 2
n 
r 1 5

   x 1  x   cos x

ln 1  x 4  x 2 2 2
1
 lim 1/2
x 0 x4  x2 1  cos 2 x   5
1 1  1/2 1  1/5
1 5

Hence, A is correct.  1 

4 4
2x
55. f  x   lim tan 1 nx  x



n  Hence, C is correct.

FREE BOOKS FOR JEE & NEET =>(@iitjeeadv)
1.148  
Differential Calculus for JEE Main and Advanced


1  t

x22 
3  1 9  1  27  1  ln 3 ln 9 ln 27  6  ln 3
x x x

 1  t 2  dt
tan 3
59. lim
  0 x 0 x 3

lim x
 form


x  sin 2 x 0 Hence, C is correct.


Apply L – Hospital Rule.
sin x   sin x 

sin x
 x4  −1  x  1 62. lim
lim  tan −1  2 x − tan  1 + x  × ............. I  1  sin x ln sin x


x
x →∞  1+ x4  2 x 2

 x  Apply L – Hospital Rule


tan −1 


 1 + x  x cos x   sin x  cos x  cos x ln sin x   0
sin x
As lim × → 0 ...................... II
x →∞ x (1 + x ) x lim
x
   cos x  cos x ln sin x 
1+ x 2



Using II in I Hence, B is correct.



 x 4  1  x  1
x
63. lim x 2  x
n
tan 1   2 x  tan  


 1 x
4
  1  x  2 x x 
n 0 n
lim 0
x  2 cos 2 x 2
As e x  1  x  x  ...............


Hence, A is correct. 1 2

 
2 1/x
r3  r 2  1

ps  x xn 
   e
1/x
60. lim 2 
r  

x 
el
r 1
4
 r2 1 r2  r  n 0 n 
eh
 Hence, B is correct.
r 4  r 2  1  r3  r 2


2  n  1
r  
je

r 1
4
 r2 1 r2  r 64. As,Sn 1  Sn  t n 1 
3n  4
iit



1 r
 r2  r  r4  r2 1 2  n  1
@

2
r 1 l  lim n  
n 
 3n  4   n  n  1 3


1 1 r
 r  r 1  2 2 2

r 1 r 1  r   Now,

L  l  2l2  3l3  ...............


1 1 1 1 1 
 r  r  1  2  r 2  1  r  r 2  1  r  lL  l2  2l3  ...............

r 1
1  l  L  l  l2  l3  ...............


1 1  1  1 1 
     2 2  2 

r 1    r 1       l
r r 1 r 1 r r 1 r
1  l  L 
1 l

1 3
 1  1 

2 2 l 2/3
L  6
1  l 

2
Hence, A is correct. 1/9

61. lim 
729    243   81  9 x  3x  1
x x x

Hence, D is correct.

x 0 x3

n
1.n    n  1 1  2    n  2  1  2  3  ..........1 r
  9  1  3
x
 243 x
 3 1x
9 x x x
1 65. lim r 1
lim

x 0 3 n  n4
x
 r   r  1
(3 − 1) ((243)
n 1

lim
x x
− 9 x 3x + 1 + 1
 ( )  n  r
r 0 2
lim
x →0 x3 n  n 4

3  1  243


  27   9  1
x x
n  r 2  n  r   r3   r 2
x x

lim  1
lim
x 0 x3 2 n  n4


FREE BOOKS FOR JEE & NEET =>(@iitjeeadv)
Limits 1.149

n ( n ) ( n − 1) ( 2n − 1) n ( n − 1)( n ) n 2 ( n − 1)  
2 2
2 2
+ − lim C0  C1    Cn  2  ..................
6 2 4 n  
 3 3 
n ( n − 1) ( 2n − 1)



1 6  2  n  1
n
lim 4 lim 1     lim    0
2 n →∞ n n   3   n   3 


2n 2 ( n − 1) ( 2n − 1) + 6n 2 ( n − 1) − 3n 2 ( n − 1)
2


Hence, A is correct.
− 2n ( n − 1) ( 2n − 1)


1
lim 69. As x  0 ,  x   0  lim f  x   0
2 × 2 × 6 n →∞ n 4


x 0

  1 1 1 1   As x  0 ,  x   1  lim f  x   sin 1




 2 1 −   2 −  + 6  − 2  
x 0
 n n n n
lim  
1 Hence, D is correct.
 1


2
24 n →∞  1  1 1 
 −3  1 −  − 2  −   2 −   1
  n   n n2   n   70. If lim 2n
0


n 
3 1 
5

  tan 2 x 
× ( 4 − 3) =
1 1  
24 24
3 3

Hence, B is correct. Then tan 1 2 x  1 or tan 1 2 x  1
 


x2
ps
 sin x
2
dx  
tan 1 2 x  or tan 1 2 x  
el
66. 0
0


lim form 3 3
eh

x  n
x 0

Apply L – Hospital Rule,    
 tan 1 2 x  or   tan 1 2 x  
je

sin x  2x 4
2 3 2 3
iit

lim
nx n 1
x 
  2 x  3 or    2 x   3
@

For limit to exist & non zero
 2x  3
x n1  x 5

Hence, A is correct.
 n 1  5

n6 2 n 4 1
 n !  5n 5
1

c/x 71. I  lim  n   00 form
1 t  n   n 

67. t 0   1  a sin bx 
lim ln dx
t0 

2 n 4 1 n !
lim ln n
Ie n  5 n 1
5
n
t

 1  a sin bx 

c/x
dx 2 n 4 1 n  r 
0  ln  n 
n  5 n 1 r 1
 form
lim
e
0 5

Now , lim
t 0 t 0


1  2 1/n 4  n  r 
Apply L – Hospital Rule. lim  ln
n  5 1/n 5  r 1  n 

n 

e
lim 1  a sin bt   1 form
c/t
t 0

21
5 0

ln xdx
lim  a sin bt  c/t  e
e t 0
 eabc


2
 t c/x   1 1
1  5 
Hence, lim ln  1  a sin bx  dx   abc
 e 2/5
t 0  t  e
 0 

Hence, C is correct.

Hence, A is correct.
 2 2  2 2  2 2 
1  1 x + 1  +  2 x + 2  + 3 x + 3 

 2 2
2
n2
n 
68. lim Cn    Cn 1    Cn  2  ........  1   C0  72. lim
n →∞ n 3  
+ ......... +  n 2 x + n 2 

n   3 3 3 

  

FREE BOOKS FOR JEE & NEET =>(@iitjeeadv)


1.150  
Differential Calculus for JEE Main and Advanced

12 x  +  22 x  + .............  n 2 x    1  
I = lim       74. lim  f  x   ln 1    ln f  x     0
 x


x 0
 
f
n →∞ n 3
  e 

I1   
lim f  x   ln e    1  f  x   ln f  x    0
x 0
f x


1 + 2 + 3 + .......... + n


2 2 2 2
+ lim − − − − − −1  ef  x   1 
n →∞ n3 lim ln  0
x 0  f  x  
↓   A


I2 Hence, lim f  x   0
x 0

Using Sandwich Theorem,
So, f  0   0

12.x − 1 + 22 x − 1 + 32 x − 1 + ............. + n 2 x − 1 E


F
lim 3
≤ I1 ≤ Hence, A is correct.
n →∞


n 75. If AC = a units

lim
(1 2
+ 2 + ......... + n
2 2
)x a1 = EC =
a
units C

n →∞ n3 2 B
D

x 1 A = Areas of ∆ Removed =
I1  & I2 
3 3 3 2 3 2 2 3 2
a1  3  a2  3 a 3  ............
x 1
ps
So, I   4 4 4
el
3 3
Where a2 is side of ∆ formed when process is repeated two

eh

Hence, C is correct. times and so on



1 1 1  a2 2 2 
  ................. a a
je

73. nlim 3
 na na  1 na  n  b  a  A  lim   3    32    ...........

 4 4 8 
iit

n  4
1  b a 1 

lim  
@

 3 a 2 /4 3 2
n  n
 r 0 a  r /n    a  1
4 1  3/4 4

b a
1
dx  ln  a  x 

b a
 ax
0 Now Area of ∆ABC = 1 sq unit
0

 ln  b/a 

3 2
 a  1 Then A = 1

4

Hence, B is correct. Hence, B is correct.


Multiple Correct Answer Type

76. A, B, D
2x 2  3

1 C) lim sgn x
 
1/4
x  x2  x  5


x sin
x  lim sin 1/ x  1  0
A) lim 2x 2  3
x  1 x  x1/4 2


 x 1/ x lim 2 2
x 
x  x
lim
x  
x 9 5
0
D)
x 3 x 2  9


1  sin x 


0
B) lim 1  sin x  tan x  lim  form 77. B, D
  cot x 0

x x


2 2 A) lim f  x  may or may not exist.
x a


Use L – Hospital Rule
g (x ) ≤ M ⇒ −M ≤ g (x ) ≤ M

B)
 cos x


 lim 0
x
  cos ec 2 x Hence, lim f  x  .g  x   0 as lim g  x   finite value
x c x c

2

FREE BOOKS FOR JEE & NEET =>(@iitjeeadv)
Limits 1.151

80. B, C, D
C) If lim f  x   L


x c


A) It may or may not exist



Then, lim f  x  may or may not be L only B) It is correct

1 1



f x  & g x  2 
x c

D) If f  x   g  x  x  0 C) It is correct eg x x



as x  0


Then, lim f  x   L, Then lim g  x  will also be L. D) It is correct



x 0

x c 81. As


78. A, B, D 1 1
lim x cos x = lim | x | sin x = lim x cos = lim x sin = 0

x→0 x→0 x→0 x x→0 x
If lim f  x   a , Then y = a is an horizontal asymptote.
x  1
, sin x all lie between [ −1,1]


Now cos x, cos


x x
A) lim  1, Hence y = 1 is an asymptote
x  x 1 so,



x ⇒ − x < x cos x ≤ x
 1, Hence y =  1 is an asymptote


lim
x  x 1 Similarly − | x |≤| x | sin x ≤| x |

2x 1
B) lim 2 & − x ≤ x cos ≤x


x 
x 12 x
2x 1
lim  2 for y = x sin
ps
x 
x 12 x
el
sin x 1
C) lim 0 y = x sin ≤| x |
eh

x 1
2 x


x 

je

lim
sin x
0 Hence, all graphs will lie between y = |x| & −|x| and will

x  x2 1 have a limiting value at x →0.
iit

lim cot 1  2 x  1  0 Hence, A, B, C, D are correct.


@

D)

82. A, B, C, D


x 

lim cot 1  2 x  1  

x 
A) It will depend whether P(x) = 0 has 1 as a root or not

B) A rational function may or may not have a vertical
79. C, D
asymptote.

For vertical asymptote at x = a
C) For vertical asymptote, there is no condition on value

lim f  x   

x a
of f(x) at that point.
D) For a function, it can have at max two horizontal

x2 1 asymptote.
A) lim 2
83. A, B, C


x 1 x 1

2
x 2  6x  7  n 1 
B) lim  8
A) lim   1
n   n  1 


x 1


x 1
B) & D) as (−1)n is not defined when n →∞
x2 1
lim 
x 1 
x 1 n2 1
x2 1  C) lim 


lim n  n
C) x 1 x  1 
x2 1 84. A, B, C, D


 

lim
x 1 x  1 
sin x
A) lim 0
So, x  1 is a vertical asymptote


x  x
sin  x  2 
lim  B)  
lim a cos 2 x  b sin 2 x  Does not exist


x 
x 1 x 1
sin  x  2  


D) lim C) lim x sin x  Does not exist


x 1 x 1  x 




sin  x  2 
lim   D) lim tan  x   Does not exist
x 1 x 1 x 


FREE BOOKS FOR JEE & NEET =>(@iitjeeadv)
1.152  
Differential Calculus for JEE Main and Advanced

85. B, C, D
Lim f  x   cot 1

B) x 0



sin 1
lim  sin 1
sin  x  RHL 
 
x 1 1
2
A) lim C) cot 1 lim f  x   cot 1 cot1  1
x



x 1 LHL  x 0


sin 0
lim  not defined 88. B, C
x 1


0
2
 en 
1/n RHL 
B) lim    lim
e
0 A) lim  x    2 x  1
x   
x 3 LHL 



  n


x 
  2

C) lim sin sin x 


1 0
RHL 
x 1 B) lim  x   x




As RHL will not exist & x 1 LHL 
1

LHL  lim sin sin 1
x  0
x 1    1
RHL 
lim x   x

2 2
C)
lim sin 1 sin x   1

  x 0 LHL 
D) x 1


2

86. A, B tan 1
tan  sgn x  RHL 
ps


0 D) lim


LHL 
el
x 0 sgn x
     RHL 
l1  lim cos 1 sec  x    tan 1
eh

x

  4   LHL 
0
4
89. A, B, D
je


/2 1
iit

     RHL  2 x 2 n sin x
1
l2  lim sin cos ec  x    f  x   lim x
@

x

  4   LHL  n  1  x 2n
4 /2
1 1
2 sin  2 n 1

/4 x x
1      RHL   lim
l3  lim tan cot  x   
n 
1  2n
1
x

  4   LHL  x
4 0

lim xf  x 

/2 A)
x 


     RHL 
l4  lim cot 1  tan  x   
x

  4   LHL  1 1
 2n
4 3 /4 2 x sin
 lim lim x x 2

87. B, C x  n  1
1  2n


x
 tan 2 x

 ,x  0
 x2  x

2
  Lim f  x   lim lim
1 1
 2 n 1
x x
2 sin

f x   1 ,x  0 B) x 1 x 1 n  1
1  2n



 x cot x
x
,x  0
RHL = 2sin1

 LHL = 1

A) RHL  Lim f  x   1 2 x 2 n sin
1
 x RHL  0


x 0
C) lim f  x   lim lim x
LHL  Lim f  x   cot 1 1  x 2n LHL 


x 0 x 0 n 
x 0 
0

FREE BOOKS FOR JEE & NEET =>(@iitjeeadv)


Limits 1.153

1 1 If m = n − 1
2 sin  2 n 1


D) lim f  x   lim lim x x 0 2m 1  1 2  1
x  x  n  1 I 


1  2n  n
x


If m  
90. B, C, D I0


 ax  1 
x
93. B, C, D
L  lim f  x   lim 
 


x 

x   bx  2  lim x 2  x  1  ax  b
x x 
 a  1/x 


 lim   x 2  x  1   ax  b 
2
x   b  2 /x 
lim
x 
x 2  x  1   ax  b 

L = 0 if b > a




L = ∞ if b < a
If a = b lim
1  a  x 2 2

 1  2ab  x  1  b 2  0
L = (1)∞ form
x 
x  x  1   ax  b 
2


 ax 1 
1 x  a 2  1 & 2ab  1  0
x  
lim
L e  bx  2  1 1
 a  1 & b   or a  1 & b 

 ax 1 ax  2 
ps 2 2
x   x
lim
 e  ax  2  1
a =  1 & b = is invalid as limit will not be zero
el

1 2
eh

e
lim
x 
a  2/x  e-1/a or e-1/b Hence, a  1, b  1/2


je

91. B, C lim sec 2 n  k !b   lim sec 2 n  Multiple of  


C) n  n 

iit


mx 2  n , x  0 1

@

f  x    nx  m, 0  x  1 lim sec 2 n  k !a   lim sec 2 n  Multiple of  


 3 x 1 D) n  n 
 nx  m,


1

A) lim f  x   m 94. A, B, C, D


n 0

lim f  x    x  cos 2 x
n 0 l1  lim 1
lim f  x  will exist only if m =  x  x  sin x
n 0

1
lim f  x   m  
hdx
n 1 
l2  lim
h 0  h2  x2
1
lim f  x   m  

C) n 1 1 x



 lim h  tan 1   1
lim f  x  will exist for alll values of m & n. h 0 h h
1
n 1

92. A 1  1
 lim tan 1    tan 1   

h 0 h  h

2x

 sin
m
tdt 

I  lim x
xn
x 0 95. A, B, D

Applying L-Hospital Rule 1
lim x x 1

x 1
2 sin m 2 x  sin m x A)
I  lim


1
 x 1
nx n 1
lim
x 0
e x 1 e
x 1

FREE BOOKS FOR JEE & NEET =>(@iitjeeadv)
1.154  
Differential Calculus for JEE Main and Advanced

x 2
 1 lim
B)
lim 1 +
x →∞ 

x x 
 (1  1 / x) 2/3
 (1  1 / x ) 2 / 3  (1  1 / x 2 )1/ 3 )


 1


 x 
x →∞ 
lim x 2
⇒e =e =
x 3
3
 x4


lim  
x   x  2 
 1 
As lim  x   3 x 3  x 2   0 ................................. (I)
n   



3
lim  x  4 
C)  e   1  x  3
x 

  


x 2 ( x  1 / 3)3  ( 3 x 3  x 2 )3
 lim
2  x  3 x   2 
1
   1
lim
e  e2
2/3
  x    x3  x 2   x   3 x3  x 2 
x 

x2  3  3 
 
lim 1  f  x  
1/f(x)


x  1 / 27  x / 3
1
lim 0
f  x .
2

 
x 
D) lim
f x  1 2/3
x  3  x  x  ( x  1 / 3) x  x
x  3 2 3 3 2


e
 
e
 1 3 
Similarly, lim  x   x 3  x 2   0 .................... (II)
Assertion-Reason Type x   


ps 3
x {x} Subtracting, (I) from (II), we get
96. As f ( x ) = = 1+
el
2
lim 3 x 3  x 2  3 x 3  x 2 

[x] [x]
x  3
eh

 {x} 
lim f ( x ) = lim 1 + =1

x →∞ x →∞  [ x ]  Hence, A is correct.
je

And lim [ x ] does not exist . 99. As lim x x  1
iit

x 0
x →∞

 
@

 lim x x  x x  1
x
Hence, D is correct.
x 0 


n+3

1  2 1 
97. As∑ =∑ − & lim x x ( x  1)  1
(
2 n + 3n + 2
n 2
) 

n =1 2n n + 1 n + 2
 x 0


Hence, B is correct.
 ∞ 

1 1
= ∑  n −1 − n  n (n  1)
n =1  2 (n + 1) 2 (n + 2)  100. Number of circles = 1 + 2 +-------+ n 
2
1 1   1 1   1 1  Let side of triangle is ‘a’
= −  + − 2  +  2 − 3  + ...
 2 2 ⋅ (3)   2⋅3 2 ⋅ 4   2 ⋅ 4 2 ⋅ 5

1 1  a  2(n  1)r  2r cot  2r (n  1  3 ) ............... (I)
= − n


6
2 2 (n + 2)
n 3 1
Now lim  
n  n
n 2
 3n  2  2

2

Hence, A is correct.

98. lim  3
x3  x 2  3 x3  x 2 

x 

lim
 2x  2

 3
   x  x   
n  2/3 2/3 1/ 3 
 x x  x6  x4

2 3 2

  r

FREE BOOKS FOR JEE & NEET =>(@iitjeeadv)


Limits 1.155

n (n  1)  a 
2
 e1/ x  1 
103. RHL  lim [ x ]  1/ x 0
   x 0  e  1 

Area of circle

2  2( n  1  3 )   
Area of triangle 3 2  e1/ x  1 
a LHL  lim [ x ]  1/ x   1
4 
x 0
 e 1 


n (n  1)


  e1/ x  1 
2 3 (n  1  3 )2 RHL  lim  1/ x  1
x 0  e  1 




x (n )(n  1)  3  e1/ x  1 
lim   LHL  lim  1/ x
2 3 (n  3  1) 2   1
x 0  e  1 
n  2 3 6 




Hence, C is correct. Hence, B is correct.


 
2

f ( x )e x /2
sin cot 2 x
101. Let f ( x ) =
x2 / 2 104. lim

e  (  2x ) 2


x
2
x2 / 2 
f ( x )  x f ( x )e x2 / 2
lim f ( x )  lim
e
xe x
2
/2 
sin tan 2 h 1

x  x 
lim 2
n 0 4h 4
/ 2 ′′ /2 ′
f ( x ) + xe x f (x) + (x f ′ (x)
2 2

ex

Hence, D is correct.
ps

+ f ( x ))e x + x 2 f ( x )e x
2 2
/2 /2
= lim RHL  lim cos 1[cot x ]  
el
+ ex
2 2
x →∞ x 2ex /2 /2 105. 
x

2

eh

f ( x )  2 xf  ( x )  f ( x )  x 2 f ( x )


 lim 0 LHL → lim cos −1[cot x ] = 0


x−
je

x  x 12
x→
2

iit

 lim f ( x )  0

x 
So, lim f ( x ) does not exist
π

@

x→

2
Hence, A is correct.
Hence, D is correct.

102. Let side of ∆ABC = a units

Comprehension Type

1 3 a
r1 = × a= Comprehension 1:
3 2 2 3
f ( x )  x[ x[ x ]], x  [1, 3]
 
2
Now,  r1  r2    r1  r2   3  r1  r2 
2 2

x = − 1, f (x ) =1

 r1  r2  2  r1  r2 
A

For x  [1, 0), f ( x )  0

3r2 =r1 x  [0,1), f ( x )  0

r1 x  [1, 2), f ( x )  x
  r2 r1
3 r2
 5
x   2,  , f ( x )  4 x

C B
Sum of areas of all circles  2


 r12  r22   5 
x   , 2  , f ( x )  5x
2 

  1 2  1 2  x  3, f ( x )  9 x
=      ... 
 3   3 3  


  = =
106. As f ( x ) 13 is possible when x
13

5
/3 3
= 
11 / 9 8  a  b  18


Hence, D is correct. Hence, B is correct.


FREE BOOKS FOR JEE & NEET =>(@iitjeeadv)
1.156  
Differential Calculus for JEE Main and Advanced

107. Limit of f(x) does not exist at x  1,1, 2, 5, 3 113. As f f ( x )  0


lim f (2 sin x ) = lim f (+2 cos h ) = 2 ⇒ Hence, C is correct.


108. π− h → 0−
x→ 114. xlim g ( x )  lim sgn f ( x )  0

2 0 x 0


Hence, A is correct. Hence, A is correct.


Comprehension 2: Comprehension-4:
x 12
x 1
2
109. A : lim  2, lim  2 x
t 2 dt
| x 1 | | x 1 | 

x 1 x 1

a  t 
1/ p
115. r
Approx f(x) does not exist. 0


lim
2{x}  4 2{x}  4 x 0 bx  sin x
B : lim  4, lim 1
x 2 [ x ]  3

x 2 [ x ]  3 

x2 1
 lim 

Approx f(x) does not exist.
a  x  b  cos x
x 0 1/ p
r
1 1 1
 0, 


C : lim lim
x 0 2  21/ x

x 0 2  21/ x 2 
For limit to exist and non zero  b  1


Approx f(x) exists.
Hence, D is correct.
Hence, C is correct. x2 1
116. xlim  1

   1 (1  cos x ) a  x r
 
0 1/ 3
x e[ x ]|x|  2

110. lim
ps

x 0 
[ x ] | x |
el
 a1/ 3  2
x e [ x ]| x |
  1 x

eh

2 x e 2 a 8
lim   lim 0

x 0 [ x ] | x | x 0 1  x
je

 

Hence, A is correct.


iit

Approx f ( x )  1  x 2   1 1/ 2 2
117. lim    3
x 0  1  cos x   9  x r 

@


Hence, A is correct.  

111. xlim f ( x )  0, lim f ( x )  a Hence, B is correct.
0 
x 0 


Comprehension-5:
For approx f(x) to exist [| a |]  1
 0 | a | 2 p a
1
x
1
 p 2 a 2x  p n a nx 1
118. lim F( x )  lim x
e


 2  a  2 x 0 x  0

 a  (2, 2)  p1  a1x 1 p 2  a 2x 1  a nx 1 
   p n

lim  x x x 
 
Hence, B is correct.  e
x  0

Comprehension-3:

 lim e
p ln a  p ln a 2 p n ln a n )
 sin x 
1 1 2

112. f ( x )    x 0 

 x 

π 2π

0

= a1 a 2 … a n
p p 1 p 2 n

+
At x = 0 , f (x) = 0

Hence, C is correct.


In Ist quad, f ( x ) = 0
 
1/ x
119. F( x )  p1a1x  p 2 a 2x 


In II nd quad, f ( x ) = 0
log F( x ) 
1

log p1a1x  p 2 a 2x  

at x  n, f ( x )  0 x

 

In III rd & IV th quad, f ( x ) = −1 log p1a1x  p 2 a 2x 
lim log F( x )  lim

x  x  x
Hence, D is correct.


FREE BOOKS FOR JEE & NEET =>(@iitjeeadv)
Limits 1.157

p1a1x ln a1 + P2 a 2x ln a 2 + ... 120. Put, x  1/ y


= lim


x →∞ p1a1x + p 2 a 2x + ...
lim log F(1 / y)  lim
 
log p1a11/ y  p 2 a 21/ y 

p1 ln a1 + p 2 (a 2 / a1 ) ln a 2 + ... 1 / y
x
y 0 y 0
= lim


p1 + p 2 (a 2 / a1 ) + ...
x →∞ x
p1a11/ y ln a1  p 2 a 21/ y ln a 2  ...  p n a n1/ y ln a n

= ln a1 p1a11/ y  p 2 a 21/ y   p n a n 1/ y


 lim F( x )  a1 ⇒ lim F( x ) = a n
x  x →−∞



Hence, C is correct. Hence, D is correct.



Match the Column for JEE Advanced

  1 2 1/ 3   x a 
 1 x
121. A) lim cos  n 1   2  
2
D) lim e x  a 
n    n n  

x 
 
2 ax
e a  e 2a  e
  1 2 
ps lim x
x 
lim cos  nπ 1 +
2
+ 2 + ...
  3n 3n 

el
n →∞
1
a
    
eh

  2  2
lim cos 2  n    

A → Q, B → R, C → P, D → P .
je

n   3 3n 


iit

1
cos 2  122. A) lim x 2  x  1  ax  b
x 


3 4
@
    
n 

B) lim n sin 2 1  n 2  lim
x 
x 2  x  1  (ax  b) 2
x 2  x  1  ax  b


  1  
1/ 2
 lim n sin  2n 1  2   1  a  x 2 2

 (1  2ab) x  1  b 2 
n    n  
lim
 x 
x 2  x  1  ax  b
    

  1  For limit to be zero; a 2  1, 2ab  1  0
 lim n sin  2n 1  2   

n    2n  1
 a  1, b   or a  1, b  1 / 2
    


2
 lim n sin 
n  n (a , 2b)  (1, 1) which lies on 3 x  2 y  5  0

    
n 
 
  
C) lim (1) n sin  n 2  0  5n  1 sin   
 2 2n  B) lim
1  a   8e3 1/ x

1  1  b  e


x  3 1/ x

 1 1    
lim (1) sin n 1 
n
 2  ...  cos  
n   4 n 2 n   2n  9  a3
2
    
       2  b3
lim (1) n sin  n    ...  cos  

n   4 2n   2n  ⇒ 5 + a 3 + 2b3 = 0 .......................(I)

    
=
1
2
   5
 a , b3 can be (1, 3),  0,  
 2
    

FREE BOOKS FOR JEE & NEET =>(@iitjeeadv)
1.158  
Differential Calculus for JEE Main and Advanced

For 15 x  2 y  11  0
n a sin 2 n sin 2 n n a 1  

C) lim  lim 0
 15a  2b3  11  0 ........................................ (II) n 1 11/ n



n  n 


Solving (I) and (II)  a 3  15a  6  0 D) log a (2a  x )   log a x  has only one solution at x  a
3



Which will have a real solution A → S, B → R , C → P, D → Q

& b3 ≠ 2


( )

x+ x − x− x
 
124. A) lim
x 4  x 2  1  ax 2  b



x →∞
C) xlim



 
x 4  x 2  1  a 2 x 4  b 2  2abx 2  lim 
2 x

lim x   
x 
x 4  x 2  1  ax 2  b   x x  x x 



lim
1  a 2
 x  (1  2ab)x  1  b   0
4 2 2
 lim 

x  
2 
 1

x  x  1   ax  b 
x  4 2 2  1  1 / x  1  1 / x 


sin 2 x  2 tan x

B) lim
1 x 0

ln 1  x 3 


 a  1  a  1, b  
2
2
 

2
& (1  2ab  0) 2 sin x sin x x3
ps
 lim 

 (a , 2b) x 0 cos x x3 ln 1  x 3  
el


eh

=(1,1) =−2

( )

je

x 7  ( a ) 7 C) lim ln sin 3 x − ln x 3 ( x + e)
D) lim  7( a ) 6  7 x → 0+


iit

x  a x  ( a )



 sin 3 x   1
(as a < 0) = lim ln  3  = ln  e  = −1
@

a  1  a  1 x →0  x ( x + e) 
+


so, ( −1, 2) lies only on y = 2
5

123. A) f ( x ) | x  a |  | x  10 |  | x  a  10 |
D) 2 | sin  | 2 | cos  |


2


At x  a , f ( x ) | a  10 |  | 10 | 10 (Min value)
5
⇒ | sin θ | + | cos θ | =

At x  10, f ( x ) | 10  a |  | a | 10 (Min value)

4

x
At x = a + 10, f ( x ) =| 10 | + | a |= 10 (Min value) 5
f (x)   

4
x (1  cos 2 x ) 2  a (sin x  tan x ) 2

B) lim   4 x 
x 0 tan 5 x  a sin 8 x


lim  2      0
  5  
x  
 x (2 sin 2 x / 2) 2 
× −

 4 a  A → S, B → P, C → Q, D → R
sin 5 x  sin x cos 2 x sin 3 x 
= lim
( )

x → 0 sin 5 x sin 5 x sec5 x + a sin 3 x 1  cos 2 x
125. A) lim

ex  ex  x
2
x 0


 
 
 x 
sec 
4
 lim  4  
4a sin x / 2 2 sin 2 x 4 cos 2 x
  5
x  a sin 3 x lim  lim 4
x 0   sin x  3
  2 xe  e  1 x2
4 x e  2e x  e x
2 x2 2
x x x 0 x x 0
 cos 2 x  2 sin cos  
  2  

2
1/ x  3+ x  1
B) lim  3 + x 
 −1 ×
=4 = lim e 3− x  x
x →0  3 − x 


+
x → 0+

FREE BOOKS FOR JEE & NEET =>(@iitjeeadv)


Limits 1.159

2
 x  2 sin x 
 lim e 3 x  e 2 / 3 D) xlim  
x 0 0  


 x  2 sin x  1  sin x  x  1 
2 2




 pq 5
( x  2 sin x )  x 2  2 sin x  1  sin 2 x  x  1 

C) lim tan x  tan x
3 3
lim
x 0
x 2
 x  2 sin x  sin x
2



x 0 x5

 


   
3 2 sin x
x3 2 5 x 9 2 15 (1  ) x 2  2 sin x  1  sin 2 x  x  1
 x   x      x 3   x   
lim x
3 15 3 15
 lim     x 0  2 sin x sin 2 x 
x 0 5
1  x   
x 
x
 x

3
 


x3 2 5 =2
 1 (As coeff of x in  x   x   5
 


 3 15  A → R , B → S, C → P, D → Q


Review Exercises for JEE Advanced

tan(a  2h )  2 tan(a  h )  tan a ps  (1  cos x ) 2 4x  1


1. (i) lim  lim  2  
 


h 0 h2 x 0  cos x sin x3
 5  8 sin x cos x
sin x 3 8

el
tan(h )(1 + tan(a + 2h ) tan(a + h )) − tan(h )(1 + tan a tan(a + h ))

lim
eh

h →0 h2 4


tan h tan(2h ) 5
je

lim [tan(a  h )] [1  tan a tan(a  2h )]



h 0 h h
iit

(i) lim 1  1  x cos x


2

2 sin a 3.


 2 tan a sec a  2
@

x 0 tan 4 x
cos3 a
 x2 x4 

3
1  tan 1 3x  3 1  sin 1 3x 1  1  x 2 1   
(ii) lim
 2 4 
x 0
1  sin 1 2 x  1  tan 1 2 x


 lim 4
x 0
 tan −1 3x + sin −1 3x   x3 2 5 
lim −  
 x   x   
x → 0  tan −1 2 x + sin −1 2 x   3 15 

 
 1 − sin −1 2 x + 1 + tan −1 2 x 
 x2 x4   x2 x4 
1  1     1     

( ) ( ) ( ) ( )
1/ 3 
 1 + tan −1 3x
2/3
+ 1 − sin −1 3x
2/3
+ 1 + tan −1 3x
1/ 3
1 − sin −1 3x  2 8 2  4
 lim    (I)


x 0 4
 x 3
2 
3 2
    1  x   x 5   
 3 15 
2 3

1  cos x cos 2 x cos 3x 1
2. (i) lim Coeff of x 4 in Numerator =
3
x 0 sin 2 2 x Hence, from (I),


sin x cos 2 x cos 3x  2 sin 2 x cos x cos 3x  3 sin 3x cos x cos 2 x 1 4
 lim x  ax 6  bx 8  
x 0 2  2 sin 2 x cos 2 x 3
lim 4
1 4  9 7 x 0
 x3 2 5 
   x   x   
8 4 3 15
 


(sin x  tan x ) 2  4 x sin 4 x 1
(ii) lim
x 0

sin 5 x 5  8 sin 3 x cos8 x  =


3

FREE BOOKS FOR JEE & NEET =>(@iitjeeadv)
1.160  
Differential Calculus for JEE Main and Advanced

(ii) lim 1  x sin x  cos 2 x  3x  2a   3 sin x x x


2 cos   4 sin 3  3 sin 
2   2


x 0 tan 2 x / 2  lim  2 2
x 0 3
(1  cos 2 x  x sin x ) x
 lim 


x 0 tan 2 x / 2  3x  2a  sin x / 2
3

1  lim  8 cos     cos a


x 0  2  x3
1  x sin x  cos 2 x


   
(1  x ) 1  x 2  1  x 2 n

sin x ( x  2 sin x )
 lim  6. lim
 (1  x) 1  x 1  x 
x 1 2


x 0 2 x 2 n
tan
2
1
Divide Nr &Dr by (1 − x ) 2 n
( 1  x sin x  cos 2 x )

sin x  sin x   1  x 2   1  x 3   1  x 2n 
= lim × 1 + 2 ×     
x →0 x  x   1  x   1  x   1  x 
 lim
x2 1 x 1
  1  x 2   1  x3   1  x n 
2
×    
( 1 + x sin x + cos 2 x )  1  x   1  x   1  x  
2
tan x / 2
    

=6

2  3  4 2n 2n

1  x2 x2 x2 x2   
ps
4. (i) lim 1  cos  cos  cos cos  (2  3   n ) 2
(n ) 2


x 0 x 8  2 4 2 4 

el

n
    1  1  
eh

1  cos
x2
 1  cos
x2
  1   1   
 n  2 n 
2 4 lim  
 lim    7.
je

n    1   1 

x 0 x 8
 1  2  1  n 1  
iit

  2 n   2  n 

2 sin 2 x 2 / 4 2 sin 2 x 2 /8
@

= lim × n

x →0 x4 x4  1   1   1   1  
1 1 1 2 1 n 1  1
 lim e  n   2 n   2 n   2 n  

1 11
 4   n 
16 64 28

 1 1 1  

( )

1  2  n 1   higher power of n 
(ii) sin sin tan x 2 / 2  lim e  2 2 2  
lim n 


x →0 ln cos 3x

sin sin tan x 2 / 2 sin tan x 2 / 2 = e2
= lim × ×

x →0 sin tan x 2 / 2 tan x 2 / 2 y  lim  log sin x sin 2 x   1 form
logsin 2 x
8.
x 0

tan x 2 / 2 x 2 × (cos 3x − 1)
×  (logsin 2 x  logsin x ) 
x2 /2 2 ln(1 + cos 3x − 1)(cos 3x − 1) lim   logsin 2 x ...............................� �(I)
y e  logsin x 
x 0




1

9 (log sin 2 x  log sin x )
 log sin 2 x

sin (3x + a ) − sin a Now, lim
x 0 log sin x
5. −3(sin(2x + a ) − sin( x + a ))

lim
log sin 2 x
x →0 x3  lim  log 2  log cos x  
x 0 log sin x
 3x + 2a 
2 cos  sin 3x / 2 − 6 cos
 2 

 log sin 2 x 
 3x + 2a   As xlim  1
= log 2   log sin x 
 2 
sin( x / 2) 0

= lim

x →0 x3 =
From (I), y e=
log 2
2 Ans


FREE BOOKS FOR JEE & NEET =>(@iitjeeadv)
Limits 1.161

(tan x  sin x )   n 2 (n + 1) 2 (2n + 3)n (n + 1)(2n + 1) 


1  n  − 
9. tan x  sin x    1
Sn = ∑ Tr = 
4 6 
lim 2  n (n + 1) 2 (n + 2) 

x 0 r =1
1  x 3  x 3    + 
 2 


Let y  t  t  t  n −1
2 n
1∑ r + 2∑ r +  + n ⋅1

r =1 r =1
 y2  y  t  0 Now lim

n →∞ n4


1  1  4t 1  1  4t
 y y  3n 2 (n + 1) 2 − 2n (n + 1)(2n + 1)(2n + 3) 
2 2  

1  +6(n )(n + 1) 2 (n + 2) 
Put t  tan x  sin x & x 3 & use eq (1) = lim  4 
n →∞ 24 n

 
1  1  4(tan x  sin x )  
 
lim


x 0
1  1  4 x 3 1 1
= [3 − 8 + 6] =

 1  4x  1

3

24 24
4(tan x  sin x )
 lim  r (r + 1)(2r + 1) × 4 2(2r + 1) 2  1 1 
x 0 4x 3  
1  4(tan x  sin x )  1 12. Tr =
6 ⋅ r (r + 1)
2 2
= =  +
3r (r + 1) 3  r r + 1 

ps

tan x  sin x 1  lim s n = −T1 + T2 − T3 + T4 +  ∞

el
1 x →∞
  As xlim  

 x3 2
eh

2 0
2  1   1 1  1 1   1 1 
=  −1 −  +  +  −  +  +  +  +  ∞ 

3 
je

1  x 02 1 xr 2 2 3 3 4 4 5 
10. lim where x r 1 

iit

x1x 2  x n

n  2 2
=−
@

Let x 0  cos  3


⇒ x1  cos  / 2 13. lim ln x 2
+1 x2 + 1 − x
x →∞


x +1
1  x 02 sin 
 lim
n  x1x 2  x n
 lim
n      
 cos 2 cos 2  cos n  = − lim ln
( x2 + 1 + x )
 2 2  x →∞
( )
ln x 2 + 1 − ln( x + 1)

sin  sin  / 2n

 lim 2 n
  x + x2 + 1
n  sin  1
 ×

sin  / 2n  x + x2 + 1  x2 + 1
 lim      cos 1 x 0 = − lim
n 
  / 2n  x →∞ 2x

1
x2 + 1 x + 1

( )

r (n − (r − 1))(n − (r − 2)) ( x + 1) x 2 + 1
11. Tr =
= − lim
( )(

2
r ((n + 1) − r )((n + 2) − r )
x →∞
x 2 + 1 x 2 + 2x − 1 )
=
(1 + 1 / x ) (1 + 1 / x )

2 2

(

r (n + 1)(n + 2) − r (2n + 3) + r 2 ) = − lim = −1
=
x →∞
1 + 1 / x (1 + 2 / x − 1 / x )
2 2
2


1 3 2  x3 + 1 
= r − r (2n + 3) + r (n + 1)(n + 2)  14. lim  − x2 
2 x →±∞  

x

FREE BOOKS FOR JEE & NEET =>(@iitjeeadv)
1.162  
Differential Calculus for JEE Main and Advanced

 1 5 5 2
= lim   = 0 = × 6=
x →±∞  x  3 3


which means at x tends to infinity, value of both the functions are
tending towards same finite value. Hence, they are asymptotic 1+ x + x2 −1 x + x2 1
= lim =
( 1 + x + x + 1)
18. lim
to each other. x →0 x x →0 2 2


x
 x +1  x 
tan −1  − tan −1 
 x + 2   x + 2 
15. lim 2 − 2 cos x 2 − 2 cos( π / 4 + h )
& limπ = lim

x →∞ 1/ x
 π  h →0 sin h


x→ sin  x − 
 
4
 4 
 
1 1 1 2  2 − 2 cos h + 2 sinh
lim  × − × × −x 2 = lim
x →∞  ( x + 1) 2 ( x + 2) 2 x2 ( x + 2) 2  h →0
 1+ 1+ sin h

 ( x + 2) 2 ( x + 2) 2 


sin h / 2 [sin h / 2 + cos h / 2]

lim
( ) (
−(( x + 2) 2 + x 2 + 2 ( x + 2) 2 + ( x + 1) 2 ) )x 2
= 2 2 lim
h →0 2 sin h / 2 cos h / 2
= 2


x →∞ (( x + 2) 2 + ( x + 1) 2 )(( x + 2) 2 + x 2 ) 1
Now LHS = + 2 = 1.914



( x + 2) 2 + 2( x + 1) 2 − x 2 ) x 2 2
lim 
( )
x →∞  ( x + 2) 2 + ( x + 1) 2 (( x + 2) 2 + x 2 ) RHS = log 0.005 = −2.301

ps
  
⇒ LHS > RHS

el
2 1
= =

ex 11 2
eh

4 2 (1 + x )1/ x − e + − ex

19. lim 2 24
(sin x − tan x ) 2 + (1 − cos 2 x ) 4 + x 5
je

x →−0 x3

16. lim

x →0 7 tan x + sin x + 2 sin x
7 6 5
iit

 ex 11 2 7 x 3e  ex 11 2
sin x  2
x
2
 e − + ex − +  − e + − ex
@

2 sin 2  + 24 sin 8 x + x 5  2 24 16  2 24
2  = lim

cos x  2
= lim x →0 x3
x →0 7 tan 7 x + sin 6 x + 2 sin 5 x
7

4 sec 2 x sin 4 x / 2 =− e
3
+ 24 sin 3 x + ( x / sin x )5 16
sin x 1

= lim =
x →0 7 sin 2 x cos7 x + sin x + 2 2 20. For r = 1,


| x −1| 2
+ x | x − 1 | +2 − d1 − 2 2+ x −2
2 x lim = lim
17. A) xlim x→4 x − 4 x→4 x−4
→1/ 2 1



x −2+ x −2
x = lim
1 1 
=  + + 2 − 4 / 1/ 2
4 4 
x→4
( x − 4) ( 2+ x +2 )

1 1

= lim =
)( )
3
=−
2
x→4
( x +2 2+ x +2 16


| x −1| 2 For r = 3
+ x | x − 1 | +2 −

2 x
B) lim
x →3/ 2 d 3 = 2 + d 2 = 2 + 2 + d1 = 2 + 2 + 2 + x


1
x −2+

x
1 3 4 2 1 2+ 2+ 2+ x −2
=  + +2−  − lim
4 4 3 3 2 x→4 x−4


FREE BOOKS FOR JEE & NEET =>(@iitjeeadv)
Limits 1.163

Apply L-Hospital Rule,


1 1 1 1

x 20 1 − 2
lim f n ( x ) = lim  n +
−n
( ) 
lim × × ×
x→4 n →∞ n →∞  2 1 
2 2+ 2+ x 2 2+ 2+ x 2 2+ x 2 x 


1 = 20
= = 2 −8


4 4 24. If 0 < a < 1,



1/ x
x n f ( x ) + g( x )  ax −1 
21. φ n ( x ) = lim   = ( 0) 0 = 1
x n +1 x →∞  x (a − 1) 

Case 1: | x |< 1


If a > 1

x f ( x ) + g( x )
n
 ax −1 
1/ x
lim = g( x )
n →∞ x +1n y = lim  
x →∞  x (a − 1) 

Case 2: | x |> 1



f ( x ) + x − n g( x ) 1  ax −1 
= f (x) ln y = lim ln  

lim
n →∞ 1 + x −n x →∞ x  x (a − 1) 

Case 3: x = 1
x (a − 1) (
x (a − 1)a x ln a − a x − 1 (a − 1) )

= lim ×
x f ( x ) + g( x )
n
f ( x ) + g( x ) x →∞ a −1x
x (a − 1)
2 2
=
lim
ps
n →∞ x +1
n

2
xa x ln a − a x + 1
el

= lim
Case 4: x = –1 x →∞
(
x ax −1 )
eh

x f ( x ) + g( x )
n

= Not defined
je

lim  x 
xn + 1

n →∞ a ln a 1
= lim  − 
iit

 a sin x   b tan x   (
x →∞  a x − 1 x
 )
@

22. lim  +
x →0  x   x 


 ln a 1
= lim  −  = ln a
sin x tan x x →∞  1 − a − x x
As <1 & >1 when x → 0


x x ⇒y=a

( ( ))

a sin x b tan x
⇒ <a & >b
25. y = 1 − lim lim cos ( m !πx )
2n
x x m →∞ n →∞


 a sin x   b tan x  When x is rational, m !πx → Integral multiple of π
⇒ lim  + =a − 1 + b
x   x 

x →0 
⇒ cos m !πx = ±1

x + 20

x
23. As f1 ( x ) = + 10 = ⇒ lim (cos m !πx )
2n
=1

2 2 n →∞

x + 20 ⇒y=0
+ 20

2 + 20 When x is irrational, m !πx → Irrational multiple of π
2 + 20

2 cos m !πx = ( −1,1)
fn (x) = ∞
+

⇒ lim lim cos ( m !πx )
2n
=0

x + 20 + 40 + 80 + 160 +  m →∞ n →∞
fn (x) =

2n ⇒ y =1

( )


x 20 2n − 1 2
fn (x) = + x1/ 3 (1 − x ) 2 / 3 1 3
2n 2n 26. lim = lim  − 1 = +1 = m
→∞ x 

x →∞ x x

FREE BOOKS FOR JEE & NEET =>(@iitjeeadv)
1.164  
Differential Calculus for JEE Main and Advanced

b = lim x1/ 3 (1 − x ) 2 / 3 − x  π
x →∞ 3  nπ +  −1
 4
=3

lim
x (1 − x ) 2 − x 3 n →∞ π
= lim nπ + + 1
x →∞ x 2 / 3 (1 − x ) 4 / 3 + x 2 + x ⋅ x1/ 3 (1 − x ) 2 / 3 4



x (1 − 2 x ) −2 f (x) x 2 sin(1 / x )
= lim = 30. lim = lim
 1  4 / 3 x → 0 φ( x ) x →0
1  


x →∞ 2/3 3 tan x
x 2  − 1 + 1 +  − 1 
  x   x sin(1 / x )
 x  = lim =0
x →0  tan x 

Now as b = xlim (f ( x ) − mx )  
→∞ x 


⇒ lim (f ( x ) − mx − b) = 0 As b is independent of x. Whereas,


x →∞

f ′ ( x ) = − cos(1 / x ) + 2 x sin(1 / x )
1
27. y = lim φ′ ( x ) = sec 2 x
n →∞ 1 + n sin 2
πx


when x is not an integer f ′ (x) 2 x sin 1 / x − cos 1 / x
lim = lim

1/n x →0 φ′ ( x ) x →0 sec 2 x
y  lim 0
n  1/n  sin 2 x  1 1
= lim  2 x cos 2 x sin − cos 2 x cos 

when x is an integer x →0  x x


= Not defined
sin 2 πx will be equal to zero
ps

⇒ n sin πx = 02
el
f ( x ) + nφ( x ) sin 2 πx
31. y = lim

eh

1 + n sin 2 πx

1 n →∞
y = lim =1
n →∞ 1 + n sin 2 πx Similar to Q.28
je


( )( ) ( ) 
1/ n
 3 3 3
28. y = lim ϕ( x ) + nφ( x ) sin πx
2
iit

 n + 1 n + 23  n 3 + n 3
32. y = lim 

1 + n sin 2 πx 
( )
n →∞
@


n →∞ n
When x is an integer  n3 

ϕ( x ) + nφ( x ) sin 2 πx 1 n   r 
3
y = lim ln y = lim ∑ ln 1 +   
n →∞ 1 + n sin 2 πx  n 

n →∞ n
r =1 

= ϕ( x )
( )

1
When x is not an integer = ∫ ln 1 + x 3 dx

0
ϕ( x )

+ φ( x ) sin 2 πx
= x ln (1 + x ) − ∫
3
3x 3
y = lim n = φ( x ) dx
n →∞ 1 / n + sin 2 πx 1+ x 3

( )

dx
x tan x + 2 x − 1 = x ln 1 + x 3 − 3x + 3∫
29. lim 1 + x3
x +1

x →∞

( )
1
1 1 
 3 ln( x + 1) − 6 ln x − x + 1 
3 2
x 2
As tan x = x + + x5 + …
(
= x ln 1 + x 3 ) − 3x + 3  

3 15  1 −1  2 x − 1 
⇒ Power of x in numerator is larger than the denominator, + tan 
  
then limit is not defined.  3 3 0

π  π  π
Now, if x tends to nπ + =  ln 2 − 3 + ln 2 + 3  −  − 3 
4  6  6
 π 

π  π π
  nπ + 4  + 2  nπ + 4  − 1 = 2 ln 2 − 3 + = ln 4 − 3 +
lim lim  = 3 3
n →∞ π
n → nπ + 
π 

nx + + 1 π
4   −3
 4 ⇒ y = 4e 3


FREE BOOKS FOR JEE & NEET =>(@iitjeeadv)
Limits 1.165

x3 1 1 2 −1
4 ⋅ (9) −1/ 2 − 27 ⋅ ⋅ (27) −2 / 3 −1
33. lim  3x + 1 + x
2 1
= 2 3 = 3 = 3
 

x →∞  3x 2 − 1 1 −2 / 3 1 −4 / 5 1 22 80 − 22 × 12
(8) − 22 ⋅ (32) −
 3 x 2 +1  x 3 3 5 12 80 12 × 80
 3 x 2 −1 −1 (1+ x )
= lim e +1 12 × 80 + 40
x →∞ = × =

3 184 23
2x 3


= lim = e2 / 3 = k
x →∞
(
(1 + x ) 3x − 1 2
) ⇒ 230b = 400



⇒ 6 ln k = 6 ln e 2 / 3 = 4 ln e = 4 cos 4 x + a cos 2 x + b
35. lim = finite quantity


⇒ [6 ln k ] = 4
x →0 x4

a + b + 1 = 0 .............................................................. (1)
x 2 + 5 − 3 2 x 3 + 3x + 5



34. lim −4 sin 4 x − 2a sin 2 x

x→2 3
x + 6 − 5 4x 2 + 6x + 4 lim
x →0 4x 3
(x ) 
( ) 

1/ 2 1/ 3
2
+5 − (9)1/ 2 2 x 3 + 3x + 5 − (27)1/ 3 −2 sin 2 x (4 cos 2 x + a )
−   lim
x−2  x−2  x →0 4x 3
 

⇒ lim
⇒ For limit to exist, a = −4
x→2 
( 1/ 5 
)
1/ 5
ps
( x + 6)1/ 3 − (8)1/ 3  4 x + 6 x + 4 − (32) 
2

− ⇒b=3
x−2  x−2 
el
 
⇒ a b +1 = ( −4) 4 = 44 = 256
eh


 
 ( x + 2)
(( x 2 + 5)1/ 2 − (9)1/ 2 )
− 2 x 2 + 4 x + 11  ( )
je

 x +5−9
2
( ) 
iit

 
(
  2 x 3 + 3x + 5 1/ 3 − (27)1/ 3 
) 
@

   
 
= lim

3
(
2 x + 3x + 5 − 27 ) 
x→2  
( 
)
1/ 5
 4x + 6x + 4 − (32)1/ 5  
2
 ( x + 6)1/ 3 − 81/ 3
 ( x + 6 − 8) − 2(2 x + 7)  
 4 x 2 + 6 x + 4 − 32
   

Target Exercises for JEE Advanced

1. lim 
 x x +1 + x + 1 ∞
 = 1 form.
x

( )
 x 2 − 2ax 2 − a 2 x + 2a 3 + 2a ( x + a )( x − 2a ) 

x →∞  x x +1   −2 x ( x + a )( x − 2a ) 

⇒ lim  
x →a

2
(
x x − a ( x − 2a )
2
) 

 x x +1 + x +1− x x +1 
lim
x →∞  x
 x x +1   
= e  

( x + 1)  x 2 − 2ax 2 − a 2 x + 2a 3 + 2ax 2 − 4a 3 
= lim =1
xx  

x →∞
e
 − 2a 2 x − 2 x 3 + 4a 2 x + 2ax 
 2  ⇒ lim  
( x − 2ax 2 − a 2 x + 2a 3 )( x − 2a ) −1 + 2a ( x + a ) 2  x →a x ( x − a )( x + a )( x − 2a )
lim 

2. −  
x →a  x x2 − a2 ( ) x−a  

   

FREE BOOKS FOR JEE & NEET =>(@iitjeeadv)


1.166  
Differential Calculus for JEE Main and Advanced

−2 x 3 + x 2 + a 2 x + 2ax − 2a 3 ⇒ 1+
1 3
=
⇒ lim
x ( x − a )( x + a )( x − 2a ) 2 2

x →a


As Lim x→a, Numerator tends to finite value & denominator (3 + ax )5 / 2 − b ln x − c(sin( x − 1)
5. lim
tends to zero. Hence, Limit does not exist. x →1− ( x − 1) 2


1
For limit to exist, a = − 3
(
 a x −a − 1 x p − a p )( )  x −a


3. form
lim   = 1∞ Now,
( )( )


x → a  ( x − a ) 2 ln a p a p −1 
 5 b
−35 / 2 (1 − x )3/ 2 − − c cos( x − 1)
lim 2 x
 (a x − a −1)( x p − a p ) − ( x − a ) 2 ln a p (a p −1 )  1
⇒ lim  × x →1− 2( x − 1)
x →a
 ( x − a ) 2 ln a p (a p −1 )  x −a


e
For limit to exist ⇒ b+c = 0


(a x −a
−1)px p −1 + ( x p − a p )(a x − a ln a ) − 2 ( x − a )ln a p (a p −1 )
Now,
⇒ lim
3( x − a ) 2 (ln a p )(a p −1 )


x →a

e 15 b
+35 / 2 (1 − x )1/ 2 + 2 + c sin( x − 1)
(a x −a −1)p ( p −1) x p−2 + (a x −a px p−1 ln a )+ p( x p−1 )(a x −a ln a )+ ( x p − a p )a x −a (ln a )2 − 2 ln a p (a p−1 ) 4 x
⇒ lim
x →a
6 ( x − a )ln a p (a p −1 )
lim
e
x →1− 2
lim p ln a ( p −1)(a p − 2 )+ p⋅a p −1 (ln a ) 2 + 2(ln a p )(a ( p −1 ) ln a + ( p −1) a p − 2 ) ⇒ b=4

x →0

⇒ c = −4
p −1
6 p ln aa

e
ps
( p −1) a p − 2 + a p −1 ln a + 2(a ( p −1 ) ln a + ( p −1) a p − 2 ) a 2 + b 2 + c 2 = 41
el


6 a p −1

e
eh

1  1 1 + ax 
( p −1) + a ln a lim 3 

+ bx 
6.
⇒ e x →0  1+ x

1
je

2a x

iit

  1  1− x   (1 + bx ) − (1 + ax ) 1 + x 
⇒ lim  
lim x ln  e 1 +  
@

4. x →0  x 3 (1 + bx ) 1 + x 

x →∞   x 

  1 1 2 1 1 3 
1 × x x+ ×
Put x =   1 2 2 2 2 2 
1 + bx − (1 + ax ) 1 + 2 x − + x + ....
3

y 2 6
  
 
1  1−  ⇒ lim
1

lim ln  e(1 + y) y  x →0 x 3 (1 + bx ) 1 + x
y→0 y  
 1 1 a 2 
  b − a − 2  x +  8 − 2  x + ...

 1 ⇒ lim
1 + 1 −  ln(1 + y)
 y x →0 x 3 (1 + bx ) 1 + x
lim

y→0 y 1
For limit to exist, b − a − =0&


y + ( y − 1) ln(1 + y) 2
⇒ ylim 1 a
→0 y2 − =0

8 2
y −1

1+ + ln(1 + y) 1 3
⇒ y +1 a= ,b =
lim 4 4

y→0 2y

1 1 1 1
l  
ln(1 + y) 16 8 4 16
1+
⇒ 2 y + ln(1 + y)

2y 1 2 3
= lim ⇒ − + = 4 − 32 + 4 = −24


lim
y→0 2 y( y + 1) y→0 y +1

a l b

FREE BOOKS FOR JEE & NEET =>(@iitjeeadv)


Limits 1.167

 1  1  1
+1 −
7. lim x 2  sin x sin 3 −  x tan 3  sgn( x )
 x    xy

x →∞ x ⇒ lim
x → y (a1 + x ) ( b1 + y ) ( x + y)
1
Put x =


xy − 1

y
⇒ lim
1  1 3 1  x→ y 1 1 
sin sin y − tan 3 y  xy  + x   + y ( x + y)
2 
lim
y→0 y  y y  x y 



sin 2 y  1 1 sin y  x2 −1
lim sin y ⋅ sin −  ⇒
(x + 1) (2x )
2 2
y→0 y
 y y cos3 y  2

= −1


1

nπ nπ 10. Let x 4 e x = y
2

n cos 2 + sin 2


lim 2 2 As x → 0, y → ∞
8.
y = n →∞  2 nπ 2 nπ 



n  cos + n sin 
 2 2 π
+y

If n is even integer, Then


If n is odd, ∫ 0
4 cos n t dt is finite

n Hence the limit is zero.


y = lim =1

n →∞ n
ps ln x  x 
− ln 
 1 + x 

If n is odd integer, then (1 + x ) 2
el
11.
( )
lim

eh

1 x →0 ln 1 − x 2 + 2 x
+

y = lim 2 = 0
n →∞ n
je

If n is Real no, then ln x − (ln x )(1 + x ) 2 + (ln(1 + x ))(1 + x ) 2
iit

lim
nπ sin 2 nπ / 2
+
x → 0+  x ln 1 − x 2
(1 + x ) ⋅ x ⋅ 
( 

)
@

2
cos 2
+ 2
y = lim 2 n  x2 
n →∞ 2 nπ nπ
cos + n sin 2

2 2
ln(1 + x )(1 + x ) 2
(ln x )( − x − 2) +

= Does not exist x
lim
( )

x → 0+  ln 1 − x 2 
a n +1 = (a n + x )
−1 1
9. = 2
(1 + x ) x +2 
an + x  

x2
1

b n +1 = =∞
bn + y

1− | x − 1 |, | x − 1 | ≤ 1

bn − a n 12. f ( x − 1) = 
a n +1 − b n +1 = + (y − x) | x − 1 |, | x − 1 | > 1
(a n + x ) ( b n + y )

0 ≤ x ≤1

Put n = 0  x,
2 − x, 1 < x ≤ 2

y−x 
a1 − b1 = ............................. (I) =
xy  x − 1, x>2
1 − x , x<0

For n = 2


a 2 − b2  x + 2, −2 ≤ x ≤ −1
lim  − x,
x → y ( x − y)( x + y) −1 < x ≤ 0

f ( x + 1) = 

b1 − a1 + y − x  x + 1, x>0
= lim  − x − 1,
x→ y (a1 + x ) (b1 + y) (x − y)(x + y) x < −2


FREE BOOKS FOR JEE & NEET =>(@iitjeeadv)
1.168  
Differential Calculus for JEE Main and Advanced

2 x + 1, 0 < x ≤1 15. α, β are roots of ax 3 + x − 1 − a = 0


0 x=0
 α + β +1 = 0
3, 1≤ x ≤ 2
 αβ + α + β = 1 / a
g( x )= 2x x>2
1+ a
1 − 2 x −1 < x < 0 αβ =
 a
3 −2 ≤ x ≤ −1


 ⇒
1
is a root of (a + 1) x 3 − x 2 − a = 0
 −2 x x < −2
α

Now, lim g (sin x ) = 3


x→
π & (a + 1) x 3 − x 2 − a = ( x − 1 / α ) ( x − 1 / β ) ( x − 1)(a + 1)
2


( )×

 r 4 − r 2 + 1  2r  ln 1 + (a + 1) x 3 − x 2 − a ((a + 1) x 3 − x 2 − a )
13. cot −1   = tan −1  4 2 
(e ) × (1 − αx)(x − 1)
lim
   r − r + 1 1− αxx

2r x→
1 ((a + 1) x 3 − x 2 − a ) −1
α
  1 − αx
2r
= tan −1  
(
 1 + r 2 r 2 − 1  ) =
1  1 1
− (a + 1)
− α  α β 

 
= tan −1 
2r
 a  β
=− (β − α ) = a 1 − 
( )( )
 1 + r 2 + r r 2 − r 
ps
α  α

el
( ) ( )

Hence Proved
= tan −1 r 2 + r − tan −1 r 2 − r
eh

  x4 −1  
−1
4 1 − 3x + x 2 

∞  r − r + 1 16. lim  2 −  + 3  3 

je

4 2
x →1   x − x −1  x − x −1  
∑ cot −1  1 − x3 

Now,   
iit

r =1 2r 
 2 − 

@

1
= tan −1 (2) − tan −1 0 + tan −1 6 − tan −1 2 +  4 x + 1 − 3 x + x
= lim   + 3 x 
x →1   x3 − 1  

= tan −1 ∞ − tan −1 0  

π lim( x − 1 + 3x ) = 3
= x →1
2

( )

17. 2f (sin x ) + 2f (cos x ) = tan x
(1 − cos x ) e x − cos x

14. lim π
Put x = − x

x →0 xn 2
case - 1 n = 3

2f (cos x ) + 2f (sin x ) = cot x

 1 − cos x   e − cos x 
x

lim 
x →0 
   ⇒ 2f (s in x ) = 2 tan x − 2 cot x
x2   x 

cot x

1 ⇒ f (sin x ) = tan x −
= 2
2


case - 2 n > 3 x 1 − x2
⇒ f (x) = −

 1 − cos x   e − cos x  1
x
1 − x2 2x
lim 
x →0 
  n −3 = ∞
x2  

x x Now, lim 1 − x f ( x )

x →1−
case - 3 n < 3


 1 − cos x   e − cos x  3− n
x  x (1 − x ) 1 + x  1
lim   =0 = lim  − =
x →0 
x x →1  1 + x
x2   

x  2x 2


FREE BOOKS FOR JEE & NEET =>(@iitjeeadv)
Limits 1.169

 −1 
−8 y = 1 − x − x − 1 + x = −x

18. lim   a − x − 1/ 4 


4 4
2 4 ax  and
− 2 log 2 a 
x →a  a + x
( )( ) 



 x1/ 4 − a1/ 4 x1/ 2 + a1/ 2 
   y = 1 − x − lim cos 2 n +1 2πx = − x
n →∞

)( )
8


  1/ 4 1/ 4 1/ 2 1/ 2   Hence both functions are identical.
 x − a x +a  1/ 4 
lim 
x →a  −a 
( ) 21. f ( x + y) = f ( x ) + f ( y) ∀x , y ∈ R , f ( x ) = 1 .
2
  − x1/ 4 − a1/ 4 − 2(ax )1/ 4  


   Solving functional eqn

( )( )  − a f(x) = x
8
  x1/ 4 − a1/ 4 x1/ 2 + a1/ 2 
 1/ 4 
lim  −   2sin x  2 tan x − cos x − 1
x →a
  x1/ 2 + a1/ 2   2 tan x − 2sin x
    Then, xlim = lim
→0 x 2 sin x x →0 sin x


x2 ⋅x

= −a{ 1/ 4 8
} = a2 x
1

= ln 2
19. lim ax 2 + 2bx + c − (αx + β) = 0 2
x →∞


lim
(ax 2
)
+ 2bx + c − (αx + β) 2 22. lim

n
∑ n k (k + 3)
nc k

n →∞
x →∞ k =0
ax 2 + 2bx + c + αx + β
Now
(a − α ) x ( )=0
ps


2 2
+ (2b − 2αβ) x + c − β 2
lim (1 + x ) n = c0 + c1x + c 2 x 2 + … c n x n
el
x →∞
ax + 2bx + c + αx + β

2
Multiply by x2 & integrate
eh

⇒a=α 2
& 2b = 2αβ 1/ n c0 x 3 c1x 4
∫0
je

x 2 (1 + x ) n dx = + + ...

b 3 4
⇒ α = ± a, β=
iit

± a 1 n nc
∑ n k (k + 3) = ∫0
1/ n
@

⇒ x 2 (1 + x ) n dx

k

b n3
⇒ α = a, β= k =0
a n nc
∑ n k (k + 3) = nl→∞
1/ n
im n 3 ∫
( ) ⇒ lim x 2 (1 + x ) n dx ...... (I)

k

x c − β2


n →∞ 0
k =0
Now, lim
x →∞
ax + 2bx + c + αx + β
2
1/ n x 2 (1 + x ) n +1 2 x (1 + x ) n +1
I=∫ x 2 (1 + x ) n dx = −∫ dx

c  2 0 n +1 n +1


a  x 2 (1 + x ) n +1 2 x (1 + x ) n + 2 2(1 + x ) n + 3 1/n
 b  2 =
n +1
− +
(n + 1)(n + 2) (n + 1)(n + 2)(n + 3) ∫0
 c    b 2  ac
 

a 
  (1 + n ) n 2(n + 1) n +1 2(n + 1) n + 2
= − +
(n ) (n + 2)(n + 3)
2 a 2a a n +3 n +3
n n ( n + 2) n +3

Hence Proved.

20. Case I → If x ≠ I 2

y = 1 − x − [ x ] − [1 − x ] −
⇒ (n + 1)(n + 2)(n + 3)

y = (1 − x ) − [1 − x ] − [ x ]

From (I)

y = {1 − x} − [ x ] = 1 − {x} − [ x ]

 1
n
 1
n +1
n

y = 1+ x = lim 1 +  − 2 1 +  ⋅
n →∞   n n n+2


and
( )


y = 1 − x − lim cos 2 n +1 2πx = 1 − x 2(1 + 1 / n ) n + 2 n 2
x →∞ +
(n + 2)(n + 3)

Case II ⇒ If x = I


FREE BOOKS FOR JEE & NEET =>(@iitjeeadv)
1.170  
Differential Calculus for JEE Main and Advanced

2n 3  1 n
  θ 
−  = lim sin θ / 2∑  sin  rθ +  + sin( −θ / 2)
(n + 1)(n + 2)(n + 3)  n →∞ 2n
r =1 
 2  



= e − 2e + 2e − 2 = e – 2 1 1 1 
= lim  ∑ (sin − θ / 2) + ∑ sin(rθ + θ / 2)
1 a  2 sin θ / 2 n →∞  n n 
23. x n +1 =  xn + 
2 xn 



 θ 1 sin(nθ / 2) 
As n → ∞, x n +1 → x n  − sin 2 + nlim
→∞ n sin θ / 2 
 

=
1   θ   = −1
1 a   θ + + nθ + θ / 2  2
⇒ xn = xn +  2 sin θ / 2  2 
2  xn  sin 
2  
   
   

xn a
⇒ =


2 2x n c1 + c 2 + . . . + c n
⇒ lim =

n →∞ n
⇒ x 2n = a ⇒ x n = ± a
n ⋅1 / 2 + d1 + d 2 + . . . + d n
=0

lim
As x, is a negative ? x n = − a n →∞ n


as n → ∞ Similarly, s n = sinθ + sin 2θ + ... + sin nθ

ps

24. U n = m
cn x n nθ
el
sin
2 sin  θ + nθ 

Now, if m is smaller than n, then one of the bracket among =  
eh

sin θ / 2 2 
m(m − 1)(m − 2)(m − (n − 1))

je

becomes zero S1 + S2 + …+ Sn 1 n sin rθ / 2  θ + rθ 


∑ sin θ/ 2 sin  2 
iit

lim lim
⇒ Un = 0 n →∞ n n →∞ n
r =1
@



Else,
∑ cos (θ/ 2) − cos (rθ + θ/ 2)
1 1
lim
U n +1 = m
c n +1x n +1
& Un = m
cn x n θ n →∞ n
2 sin
2

( )
c n +1 x n +1

m
U n +1
For n → ∞, lim =  θ 1 
lim  cos − ∑ cos ( rθ + θ / 2)
m
1
n →∞ U n cn x n
2 sin θ / 2 n →∞  2 n 


!m !n !m − n cot θ/2
= lim × x =
x →∞ ! n + 1 ! m − n − 1 !m 2


m−n  m / n − 1
= lim ⋅ x = lim   x = −x Hence Proved.
n →∞ n + 1 n →∞  1 + 1 / n 

πx

Hence, lim m
c n x n = 0 for all values of m x 2 n sin + x2
26. 2
n →∞ y = lim

n →∞ x 2n + 1

25. Let d n = cos θ + cos 2θ +  + cos nθ
If x = 1, or − 1

sin n θ / 2  θ + nθ 

= cos 
sin θ / 2  2  πx
x 2 n sin + x2

y = lim 2 =1
 θ + rθ 
sin rθ / 2 cos  n →∞ x 2n + 1
d +  + dn 1 n
 2 
= lim ∑

lim 1
n →∞ n n →∞ n sin θ / 2 If | x |< 1
r =1


FREE BOOKS FOR JEE & NEET =>(@iitjeeadv)
Limits 1.171

y = lim
x 2 n sin
πx
2
+ x2
= x2
( ( (
cos 2 1 − cos 2 1 − cos 2  cos 2 x )))
29. xlim 
n →∞ x 2n + 1 →0 x + 4 − 2


π⋅ 

If | x |> 1  x 
Now, as


2− 2n
sin πx / 2 + x πx x+4 −2 ( x + 4) − 4 1
y = lim −2 n
= sin = =
n →∞ 1+ x 2 x x ( x + 4 + 2) x+4+2


 x+4 −2 1
 1, x = 1 or − 1 lim =
x →0 x 4

( ( ( ))) = 1


⇒ y =  x2 , −1 < x < 1
 πx & lim cos 2 1 − cos 2 1 − cos 2  cos 2 x
x →0
sin , x < −1 or x > 1
cos (1 − cos (1 − cos ( cos x )))


2 2 2 2 2

4
⇒ lim =
x n φ( x ) + x − n ϕ ( x ) x →0  x + 4 − 2 π
27. y = lim π
x +x
n −n 

n →∞
 x 
case -1 : x = 1 of x = −1
φ( x ) + ϕ ( x ) 30. As x n = a + a + a +  ∞
ps

y=
2 ⇒ xn = a + xn
el

⇒ x 2n − x n − a = 0
eh

case-2 : when | x |< 1
1 ± 1 + 4a
je

⇒ xn =

2
( x ) 2 n φ( x ) + ϕ ( x )
iit

y = lim = ϕ( x ) As x n is positive ⇒ x n = 1 + 1 + 4a
n →∞ x 2n + 1
@



1 1 1
case-3 : when | x |> 1 31. [ x ] + [2 x ] + [3x ] + …+ [nx ]

2 3 n

φ( x ) + x −2 n ϕ( x )  1 1 
y = lim = φ( x ) = nx −  {x} + {2 x} +  {nx}
n →∞ 1+ x −2 n  2 n 


Hence, y is composed of two parts of graphs ϕ( x ) & φ( x ), Now,

1 1 1
together with two isolated points at x = ±1 . [ x ] + [2x ] + [3x ] +  + [nx ]
⇒ lim 2 3 n
n →∞ n (n + 1)(2n + 1)
28. y = lim lim
2
(
arctan n sin 2 m !πx ) 6

m →∞ n →∞ n

 1 1 
6  {x} + {2 x} +  + {nx}
Case 1: When x is rational, sin 2 m !πx be equal to integral  6nx   2 n 
⇒ lim   − nlim
multiple of π & is equal to zero. n →∞  n ( n + 1)( 2n + 1)  →∞ n (n + 1)(2n + 1)

(
⇒ tan −1 n sin 2 m !πx ) = 0 =0

32. let y = sin x + 2 sin 2 x + 3 sin 3 x +  ∞


⇒ y=0
xy = sin 2 x + 2 sin 3 x +  ∞


Case 2: When x is irrational, sin 2 m !πx is always positive & (1 − sin x ) y = sin x + sin 2 x + sin 3 x +  ∞
π
a fraction, then, tan −1 n sin 2 m !πx tends to

sin x
2 y=
(1 − sin x )2
⇒ y =1


FREE BOOKS FOR JEE & NEET =>(@iitjeeadv)
1.172  
Differential Calculus for JEE Main and Advanced

( )
1 1/ 2
Now, lim (1 − sin x ) 2 f ( x ) sin x −1 ⇒ lim U n <
π n →∞ 1 − 1/ 2

x→


2
⇒ lim U n < 1 < 2
= lim (sin x )1/sin x −1 n →∞
π


x→ Hence proved.
2



34. lim 1 + 2 +  + n
k k k
1
(sin x −1) ×
(sin x −1)


= lim e =e n →∞ n ⋅ nk
π
x→ 1
x k +1
2 k
1 n  r
= lim ∑   = ∫ x k dx =
1

33. v1 =
1
<2 n →∞ n  
r =1 n
0 k +1
0

2


1
v2 =
1
+
1 1 1
< + <2 =
2 2 ⋅ 4 2 22 k +1

( ) ( )

Now, coefficient of x n − 2 in ( x − 1k ) x − 2k  x − n k is F(n )

1 1 1
Now, U n = + + +
2 2⋅ 4 2⋅ 4⋅6
⇒ F(n ) = 1k 2k + 1k 3k +  + 2k 3k + 2k 4k +  + (n − 1) k m k

1 1 1
< + + +
2 22 23 = ∑ (ij) k
1≤ i < j≤ n

As n → ∞
ps

∑ ∑ ik jk − ∑ (ij)k

el
1 1 1 1≤ i ≤ n i ≤ j≤ n i= j
lim U n < + 2 + 3 +  ∞ =
n →∞ 2 2 2
eh

2


je

Previous Year's Questions (JEE Advanced)


iit

x4
@

1.
x 0
  
  
lim g (f ( x ))  gf 0  g sin 0  g 0  1 4. lim 
 x6

x   x  1 


lim g (f ( x ))  g (f (0 ))  g  sin 0   g  0   1
  
 x 6 
x 
x4

 e5
x 0 
lim e x 1 
⇒ Hence, lim gf ( x )  1 x 
x 0

1/ x 2
 1 + 5x 2 
 sin(1 / x ) 1  5. lim 
 4 1
+ 2  1 +
x 
x → 0  1 + 3x 2 

 x sin x 
2. lim  x =  x  = −1

( )
lim    1+ 5 x 2  1

x →−∞
 1+ | x | →−∞ 1
3
 x
 −1  −1
 1+ 3 x 2  x 2
   x 3
 = lim e
x →0
A
3. BD  r  (h  r )  2rh  h
2 2 2
=e 2

x2
BC  2 2rh  h 2
∫ cos t
2
dt
r

r O
Area of ∆ABC = h 2rh − h 2
6. lim 0

r x →0

x sin x
A h 2rh − h 3
Also, lim = lim (cos x 4 )2 x
( 2rh − h ) = lim

B D C h →0 P3 h →0 3
+ 2hr x → 0 x cos x + sin x
2
8

2r − h 2 cos x 4
= lim = lim =1
( )
h →0 3 x →0 sin x
8 2r − h + 2r cos x +
x

1
=
128r

FREE BOOKS FOR JEE & NEET =>(@iitjeeadv)
Limits 1.173

ln(1 + 2h ) − 2 ln(1 + h ) 1 2n r
7. lim 15. lim ∑ 2 2

h →0 2
h n →∞ n


r =1 n + r
 1 + 2h 
ln   1 2n r/n
lim  2
(1 + h ) 2  = lim ∑
( n)
n →∞ n 2
r =1
h →0 h 1+ r
 −h 2 
ln 1 + 
2
x
 1 + h + 2h 
2
1 =∫ dx
lim ×− 1+ x2
h →0 −h 2
1 + 2h + h 2 0
2
1 + h + 2h
2 1
= × 2 1+ x2 = 5 −1
= −1 2 0
Hence, B is correct.
Consider f ( x )  | x  a | , g( x )  x  a


8.
x a | x a | x tan 2 x  2 x tan x

16. lim


x 0 4 sin 4 x
Then, lim f ( x )  g ( x ) exist but lim f ( x ) or lim g ( x )
x a x →a x →a  2 tan x 

x
lim  2 tan x 
does not exist. x 0 4 sin 4 x  1  tan x
2


Hence, it is a false statement. 1 x 1 1 1
1  sin x lim   
x 1 2 x  sin x cos3 x 1  tan 2 x 2
9. lim f ( x )  lim

x  x  2
1  cos x Hence, C is correct.

x
ps x  x 3 
x

1
Hence, C is correct.  x 3  
 lim e x  2   e 5

17. lim  
10. lim  1 n 
el
2 x   x  2 
  .....  x 



n   1 n2 1 n2 1 n2 
eh

Hence, C is correct.
(1  2  3  .....  n )

lim
1  n  
sin  cos 2 x   lim sin   sin x    sin
je

2
n  2 2
x
18. lim x
iit

 sin x

x 0 2 x 0 2 2
n (n  1) 1 x x
lim 
  Hence, B is correct.
@

n  2 1 n2 2

Hence, B is correct. 19. L  lim

(cos x  1) cos x  e x 

 sin[ x ]

x 0 n
 , x  (, 0)  [1, ) x
11. f ( x )   [ x ]
 cos x  e   x

0 , x  [0,1) (cos x  1) 1
  lim 
n 0 2 n 3
sin[h ] x x x
lim f ( x )  lim  sin 1 For L to be finite non zero, n-3 = 0
x 0 
h 0 [  h ]
⇒n = 3
lim f ( x )  lim 0  0

x 0 h 0 Hence, C is correct.

∴Limit does not exist
((a  n )nx  tan x ) sin nx

Hence, D is correct. 20. lim 0

x 0 x2

xn
12. lim 0 sin nx  tan x 
x ex  n lim  (a  n ) n  x   0

x 0 nx  
⇒For n is any whole no, it will hold true
 (a  n ) n  1  0

Hence, B is correct.

 x  2x 1 1
a n
13. lim    xlim 
 1
 1
  n

x 0 tan 2 x  0 2 tan 2 x 2
Hence, D is correct.

Hence, B is correct. sin x 



1/ x  1
  21. lim (sin x )1/ x +   
14. lim  tan   x    x

x →0 

4  

x 0 

 1+ tan x  1 sin x
 −1 × 2 tan x
×
1
 1
lim e 1− tan x  x
= lim e x 1− tan x = e2 = lim(sin x )1/ x + lim  
x →0 x →0 x →0 x →0  x 

FREE BOOKS FOR JEE & NEET =>(@iitjeeadv)
1.174  
Differential Calculus for JEE Main and Advanced

 β3 x 2 β5 x 4 
ln x
lim −
= 0+e
limsin
x →0
x ln(1/ x )
=e x →0
cosec x x3  β − + 

 3! 5! 
lim
sin x
tan x ∴ lim =1
=e xx →0
= e0 = 1 x →0  1 x2 
x  − 
3
Hence, C is correct.  3! 5 ! 

sec 2 x
⇒ 6β = 1
∫ f ( t )dt
22. lim 2 6(α + β) = 6α + 6β

x→π / 4 π2 = 6 +1 ⇒ 7
x2 −
16
( )
Ans is 7
f sec 2 x 2 sec 2 x tan x
 ecos(a ) − e  −e


2f (2) 8f (2) n

lim = =
x→
π 2x π/4 π 27. Given, lim  =
 α
α →0  m
 2

{ } ( )
Hence, A is correct.
e e ( ) − 1 cos α n − 1 −e
cos a −1

n

x
1 t ln(1  t )
23. lim  dt ⇒ lim ⋅ =
x3 t4  4
( )

x 0
0 α → 0 cos α n − 1 αm 2
x ln(1  x ) 1
 lim   cos(α )−1 
x 
n
x 0 4
 4 3x 2 12 e − 1 −2 sin 2 α n / 2 −e
⇒ lim e   ⋅ =
( )
lim
Hence, B is correct. α →0
 cos α − 1  α →0
n αm 2

e    2b sin 2 
ps
( )⋅ α
ln 1 b 2

24. sin 2 α n / 2 2n
−e

⇒ e× 1 × ( −2) lim =
el
1 b  1 b  2 2
or sin 2    1 as   1 α
α →0 4a 2n m
2

eh

2b  2b  4

α 2n − m −e
je


2 ⇒ e × 1 × ( −2) × 1 × lim =
iit

α →0 4 2

Hence, D is correct. For this to exists, 2n − m = 0
@

 x2  x 1  m
25. xlim   ax  b   4 ⇒ =2
  x 1 
 

2
(1  a ) x 2  (1  a  b) x  (1  b) Hence ans is 2
lim 4

x  x 1 1 / 2(1  cos 2 x )  sin x
 1 a  0 & 1 a  b  4 28. lim  lim  1
x 0 x 0

x x
 a  1 & b  4 Hence, B is correct.

Hence, B is correct. 1  cos 2( x  1) | sin( x  1) |
 2 lim

29. lim
x sin(βx )
2 x 1 x 1 x 1 x 1

26. Here, lim =1 ⇒Limit does not exist
x →0 αx − sin x


Hence, C is correct.
 (βx )3 (βx )5 

x 2  βx − + −  a  a2  x2 
x2
 3! 5!  4
⇒ lim =1 30. L  lim
  x 0 x4

x →0 x3 x5
αx −  x − + −  1 1
 3! 5!   lim 
x 0
x (a  a  x )
2 2 2 4x 2

 β3 x 2 β5 x 4  (4  a )  a 2  x 2
 lim
x β − + 
 
3
x 0
 3! 5!  4x 2 a  a 2  x 2
⇒ lim 3 5
=1
x →0 x x
+  (α − 1) x + For Numerator → 0, if a = 2

3! 5! 1
Limit exiists only, when α − 1 = 0 Then, L =

64
α =1 Hence, A, C is correct.


FREE BOOKS FOR JEE & NEET =>(@iitjeeadv)
Limits 1.175

1 − x (1+ | 1 − x |)  1    h h
f (x) = cos  a 2  2 cos  a +  sin 
 1 − x 
31.
|1− x |   2 2

lim + lim 2a sin(a + h )
1 − x (1 + 1 − x )  1  h →0 h h →0
Now, lim f ( x ) = lim cos  2
x →1−
x →1 −
1− x  1 − x  2
 1  = a 2 cos a + 2a sin a
= lim (1 − x ) cos  =0


x →1 −
 1 − x  2x  1
34. lim

1 − x (1 − 1 + x )  1  x 0 (1  x )1/ 2  1


and lim f ( x ) = lim cos 
x →1+ x →1+ x +1  1 − x  2x  1 x
lim 
 1  x 0 x (1  x )1/ 2  1
= lim ( x + 1) cos
 , which doesn’t exists.
x →1  x + 1

+
ln 2
  2 ln 2
B, D are correct options. 1/ 2

32. lim
a  2 x  3x 35. lim 2 (n + 1) cos −1  1  − n
n →∞ π  n


3a  x  2 x

x a

2  1  2  1
Rationalising both Numerator & Denominator lim  n cos −1   − n  + lim cos −1  

n →∞  π  n  n →∞ π  n
a  2 x  3x 3a  x  2 x
lim  2  −1  1  π 
x a 3a  x  4 x a  2 x  3x n cos   −  + 1
ps lim
n →∞ π   n 2
1 4 9 2
  
el
3 2 3 a 3 3  −1  1  
 sin  n   2
eh

− lim   +1
(a + h ) 2 sin(a + h ) − a 2 sin a n →∞  1 π
je

33. lim
 

h →0 h n
iit

(
a 2 sin(a + b − sin a ) + 2ah sin(a + h ) + h 2 sin(a + h ) 2
@

lim 1−
h →0 h π

FREE BOOKS FOR JEE & NEET =>(@iitjeeadv)


1.176  
Differential Calculus for JEE Main and Advanced

Previous Year's Questions (AIEEE/JEE MAINS)

(1  cos 2 x )(3  cos x ) e x  cos x


2

1. lim 4. lim

x 0 x tan 4 x x 0 sin 2 x


2 sin x (3  cos x )
2
Apply L-Hospital Rule,
 lim


x 0 x  tan 4 x e x  2 x  sin x
2

 4x lim
4xx x 0 2 sin x cos x
2 sin 2 x (3  cos x ) 1  sin x  sin x 3
 lim  lim   lim  2e x   cos x 
2

x 0 x 2 x 0 4 tan 4 x x 0  x   2
x 2
lim
x 0 4 x


Hence, B is correct.


4
 2  1 (1  cos 2 x )(3  cos x )
5. lim
4 x 0


x tan 4 x

 sin  2 sin x (3  cos x )
2
 lim  1 and  lim
 0  x 0 x  tan 4 x
 4x
tan   4xx
lim  1
0   2 sin 2 x (3  cos x ) 1
 lim  lim 
=2
x 0 x2 x 0 4 tan 4 x

lim
ps x 0 4 x

Hence, the correct option is C.
4  sin  tan  

el
 2   1  lim  1 and lim  1

sin  cos 2 x  4  0  0  
eh

2. lim 2

x 0 2
x
je

 
Hence, the correct option is C
sin  1  sin 2 x

 
iit

1/ 2 x
 lim 6. P  lim 1  tan 2 x 1 form 
x 0 x 2
x 0  


@

 lim

sin    sin 2 x   lim sin   sin x 
2
e
lim
x 0
 tan 2
x 11  21x
x 0 x2 x 0 x2

lim tan 2 x
 sin(  )  sin  e x 0

2x

sin  sin 2 x  sin 2 x  1  tan x 
2
1  tan x 
2

 lim  ( )  2  lim 
2 x 0 

x 
lim 
2 x 0 

x 
x 0  sin 2 x  x   e  e
 sin  e 1/ 2
  lim 1
 0   logg P  log e1/ 2

Hence, D is correct. P  1/ 2


tan( x  2)  x 2  (k  2) x  2k  Hence, option C is correct.
  5

3. lim  a 4 
2x

x 2 x  4x  4
2
7. lim 1   2   e3
 
x   x x 

tan( x  2) x  (k  2) x  2k
2

 lim  lim  a 4 
x  2 ( x  2)) x2  e   2   2x x 

 x x 
tan( x  2) ( x  2)( x  k ) lim  4
 lim   2k  5  e  a    2  e 2 a  e3
x 2  x  2 
x 

( x  2)  x
k3 3
a

Hence, D is correct. 2

Hence, B is correct.

FREE BOOKS FOR JEE & NEET =>(@iitjeeadv)
Limits 1.177

(1 − cos 2 x ) 2 1 sin h (1 − cos h )


8. L = lim = lim
x → 0 2 x tan x − x tan 2 x 8 h →0 cos h ⋅ h 3

= lim
4 sin 4 x
=
1 (
sin h 2 sin 2 h / 2 )
x →0  1  lim
2 x tan x 1 − 8 h →0 cos h ⋅ h 3
 1 − tan 2 x 
1 sin h ⋅ sin 2 (h / 2)
= lim
x →0
2 sin 4 x
x tan 3 x
(tan 2
x −1 ) = lim
4 h →0 h 3 cos h
2
= −2 1  sin h   sin h / 2  1 1 1 1 1
= lim     ⋅ ⋅ ⇒ × =
Hence, C is correct 4 h →0  h   h / 2  cos h 4 4 4 16


3x  3 Hence, C is correct
9. lim


x 3 2x  4  2 1 2  15  

12. lim x            
( 3x  3)( 3x  3)( 2 x  4  2 ) x 0 x x

 x 


lim
x 3 ( 2 x  4  2 )( 2 x  4  2 )( 3x  3)  15  8   1  2  3 15 
       ...  
 
lim x 
3( x  3) 2x  4  2 x 0 

 x  x
x
x
x

lim
x 3 2( x  3)
 3x  3  120
Hence, C is correct

1

ps x tan 2 x  2 x tan x
2 13. lim
x 0 (1  cos 2 x ) 2
el
Hence, B is correct
 
eh

1
1a  2a    n a 1 2 x tan x   1
10. lim   1  tan x 
2
je

n  [ n  1]  lim
a 1
[(na  1)  (na  2)    (na  n )] 60

x 0 4ssin 4 x
iit

1 n
 r 1  r / n  2 x tan 3 x
a
1
lim 
 
@

lim n x0 4 sin 4 x 1  tan 2 x


a 1 2
n 
 1 1 n  r 
 n   n  r 1  a  n  
1 
     Hence, C is correct

1 (27  x )1/ 3  3
 x dx
a 1 14. lim
0 x 0 9  ( 27  x ) 2 / 3

 1
a 2 1
1 0
 
1 ( a  1)( 2a  1) 60 (27  x )1/ 3  3
 (a  x )dx a  2  0 lim
x 0
3  (27  x)  3  (27  x) 
1/ 3 1/ 3
0
 (a  1)(2a  1)  120 1

a7 6
Hence, C is correct


Hence, A is correct 1 + 1 + y4 − 2

cot x − cos x 15. lim
y→0 y4

11. lim
x → π / 2 ( π − 2 x )3

1 cos x (1 − sin x ) 1  1  y4  2 1  1  y4  2
= lim  lim 
x→π / 2 8
π 
3 y 0 y4 1  1  y4  2
sin x  − x 
2 

1 cos( π / 2 − h )[1 − sin(π / 2 − h )] 1  1  y4  2
= lim ⋅  lim
h →0 8
π π π 
3 y 0  
sin  − h   − + h  y4  1  1  y4  2 
2 2 2   

FREE BOOKS FOR JEE & NEET =>(@iitjeeadv)
1.178  
Differential Calculus for JEE Main and Advanced

 lim
1  y4  1

1  y4  1  x  0 
 [ x ]  1 
y 0 


y4  1  1  y4  2 

1 y 1
4

 lim

tan  sin x 2
   sin 2
x 
  1 
2
sin x 

x 0  sin 2 x x2  x 
1  y4  1


 lim ∴ LHL≠RHL
y 0 4  
 


 1  y4  2  1  y4  1
y  1 So, Limit does not exist
 


Hence, D is correct


1
 lim x cot(4 x )
y 0  
 1 1 y  2  1 y 1
4 4
  19. lim
x 0 sin 2 x cot 2 ( 2 x )


 
1 1 x tan 2 2 x
  lim
   tan 4 x sin 2 x
x 0
1 1 0  2 1 0 1 4 2
4x x2 tan 2 2 x
lim  2  .4  1

Hence, A is correct x 0 4 tan 4 x sin x 4x 2

x ([ x ] | x |) sin[ x ] Hence, D is correct

16. lim
x 0 
cot 3 x  tan x

|x|
20. lim
x (1  x ) sin(1) x   / 4 cos( x   / 4)

 lim
x 0 
x
ps Applying L-Hospital,

 3 cot 

 lim [ x ] = −1 and lim | x |= − x  2
x cosec 2 x  sec 2 x
el
 x →0 −
x →0  −
lim
  
eh

x  sin  x  

  sin1 4
 4
je

Therefore the correct option is A

Hence, C is correct
iit




(1 | x |  sin | 1  x |) sin  [1  x ]    2 sin 1 x
@

2  21. lim
17. lim x 1 1 x

x 1 
| 1  x | [1  x ]

(1  x )  sin( x  1)     2 sin 1 x   2 sin 1 x
 lim  sin  (1)   lim 
x 1 ( x  1)(1) 2  x 1
1 x   2 sin 1 x
 sin( x  1)   
 lim 1   (1) 2   sin 1 x 
x 1  
( x  1)   2 
 lim
 (1  1)(1)
0
x 1
1 x    2 sin 1 x 
2 cos 1 x

1
Hence, D is correct  lim 

  1 x 2 
x 1

tan  sin 2 x  | x |  sin( x[ x ]) 


2

2 1
18. lim
x 0 x2  lim  Putting x  cos 

0 2 sin( / 2) 2 

tan  sin 2 x  x 2 
 x  0 


RHL  lim  [x]  0 4 2
 
x 0  2
x
2 2 

tan  sin 2 x    sin 2

x
 lim 1   1 Hence, C is correct.

x 0  sin 2 x x2

Now,

LHL  lim
 
tan  sin 2 x  ( x  sin x ) 2
x 0 
x2

FREE BOOKS FOR JEE & NEET =>(@iitjeeadv)
2
CHAPTER

Continuity of Functions

Definition A function f is continuous at x = a if the following


2.1 Definition of Continuity


three conditions are met:

Earlier we came across continuous functions and widely (i) f(x) is defined at x = a.

used their properties when constructing the graphs of simple (ii) lim f(x) exists.
ps


x →a
functions, though the term “continuous function” was not
(iii) lim f(x) = f(a).
el
used that time since the definition of this notion was not x →a


given then.
eh

In other words, function f(x) is said to be continuous at x = a,


In the first stage the graphs of functions, say, y = ax + b, y = if lim f(x) = f(a).
x →a
je

ax2 or y = ax3 were plotted point by point. The procedure was In terms of one-sided limits, f(x) is continuous at x = a if
iit

as follows: we first tabulated the values of a given function L.H.L. = f(a) = R.H.L.

for certain values of the argument and then we constructed i.e. lim f(x) = f(a) = lim f (x)
@

the points whose coordinates were put down in the table x a  x a 

and then joined the plotted points with a “continuous curve”. i.e. lim f(a – h) = f(a) = lim f(a + h).
h→0 h→0
Thus, we obtained the graph of the given functions. We
There is another way to discuss about continuity. Let there be
did not notice then that the graph of such functions were
a function y = f(x), x ∈ (a, b), and let x0 be a certain value
continuous.
of the argument from the interval (a, b). Then, if x ∈ (a, b) is
The geometrical concept of continuity for a function which another value of the argument, the difference x – x0 is called
possesses a graph is that the function is continuous if its graph the increment of the argument and is denoted by ∆x, and the
is an unbroken curve. A point at which there is a sudden break difference
in the curve is thus a point of discontinuity.
f(x) – f(x0) = f(x0 + ∆x) – f(x0)

The notion of continuity is a direct consequence of the concept is termed as the increment of the function f at the point x0 and
of limit. The special class of functions known as continuous is denoted as ∆f or ∆y.
functions possesses many important properties which will be
If the function f is continuous at the point x0, then, by definition,
investigated in this chapter.
lim f ( x )  f ( x 0 ) and
x x0
Continuity at a Point consequently, lim f ( x )  f ( x 0 ) = 0,
x x0
The question for our consideration is as follows: given any
function f(x) defined in the neighbourhood of a, is the function which means lim f = 0.
x 0
f(x) continuous or not at x = a? It follows from the last relation that if f(x) is continuous at
The graph of y = f(x) is said to be continuous at x = a if it can the point x0, then to a small increment of the argument there
be traced with a continuous motion – without any jump – of corresponds a small increment of the function or, the increment
the pen from left to right at x = a. of the function f is an infinitely small quantity as ∆x → 0.

FREE BOOKS FOR JEE & NEET =>(@iitjeeadv)


2.2 Differential Calculus for JEE Main and Advanced

(v)


(vi)



Formally, the function f(x) is continuous when x = a, if given ε,
a number δ can be found, such that, whenever | x – a | ≤ δ, we
have | f(x) – f(a) | < ε.
The function shown in figures (i) to (v) are discontinuous at
Points of Discontinuity
x = a while that in (vi) is continuous at x = a. Thus, a function f
If a function f(x) is continuous at a point x = a, then the point a can be discontinuous due to any of the following three reasons:
is called the point of continuity of the function f(x). Otherwise, (i) lim f(x) does not exist

x →a


when the limit of the function f(x) at the point a does not
exist, or exists but is not equal to f(a), the function is said to i.e. lim f(x) ≠ lim f (x) [figures (i) and (iv)]
x a  x a 


be discontinuous at the point x = a, the latter being called the
point of discontinuity of the function f(x). (ii) f (x) is not defined at x = c

[figures (ii) and (v)]


In particular, if f(x) is defined for all points of the interval (iii) lim f(x) ≠ f (c)
ps [figure (iii)]


x →a
(a – δ, a + δ) except for the point a, then x = a is also a point
of discontinuity of the function f(x) in the interval. It follows Geometrically, the graph of the function will exhibit a break
el
at x= c .
eh

from the aforegoing that a function is discontinuous at a given


point if either (i) the given point fails to be in the domain of Note: It should be noted that continuity of a function is


je

the function, or (ii) the function fails to be have a limit at the the property of interval and is meaningful at x = a only if the
function has a graph in the immediate neighbourhood of x = a.
iit

given point, or (iii) the limit of the function is unequal to the


function's value at the given point. For example, the discussion of continuity of
@

(i) 1
f(x) = at x = 1 is meaningful, but continuity of f(x) =


x −1
ln x at x = –2 is meaningless.
Similarly, if f(x) has a graph as shown in the figure below, then
continuity at x = 0 is meaningless since we can’t approach 0
from either side of the point. Such a point is called an isolated
(ii) point.

(iii) Consider the following examples:



1
(i) The function f(x) = is discontinuous at x = 1.
(1 − x ) 2

This function is not defined at the point x = 1.
sin x
(ii) The function f(x) = has a discontinuity at

|x|
(iv)
sin x

x = 0, since f(0+) = lim  1 and


x 0 x
sin x
f(0–) = lim  1 .
x 0 x



FREE BOOKS FOR JEE & NEET =>(@iitjeeadv)


Continuity of Functions 2.3

 x2  a2   Solution: Given, f(0) = 0


 when 0  x  a ,


 xa e1/ x 0
 L.H.L. = lim f ( x )  lim  0
(iii) Let f ( x )   a when x  a , x  0 x  0 1  e1/ x 1 0



 2a when x  a.
 1

 since lim     
x 0  x  


e1/ x
R.H.L. = lim f ( x )  lim
x 0 
x 0 
1  e1/ x
1 1
lim  1
x  0 1
1 0 1
e1/ x
Here lim f ( x )  2a , f(a) = a, lim f ( x )  2a ,


1
x a _ x a  since lim    
x 0  x 

and there is a discontinuity at x = a due to the isolated point P.


To understand explicitly the reasons of discontinuity, consider Since lim f ( x )  lim f ( x ) ,
x  0 x  0
the graph of the following function y = f(x).
f(x) is discontinuous at x = 0.
Example 3: Find whether f(x) is continuous or not at


 x
sin , x 1
x = 1, if f(x) =  2 ,
 [ x ]
ps , x 1
where [ ] denotes the greatest integer function.
el
  Solution: For continuity at x = 1, we determine, f(1),
eh


lim f(x) and lim f(x).
je

x→1− x1
Let us comment on the continuity of the function.
Now, f(1) = [1] = 1.

iit

(i) f is continuous at x = 0 and x = 4


πx π

(ii) f is discontinuous at x = 1 as limit does not exist lim f(x) = lim sin = 1.
@

= sin

(iii) f is discontinuous at x = 2 as f (2) is not defined although

x→1− 2 x→1−
2
the limit exists. and lim f(x) = lim [x] = 1.

x1 x1
(iv) f is discontinuous at x = 3 as lim f (x) ≠ f(3)
So f(1) = lim f(x) = lim f(x).

x→3

x→1− x1
(v) f is discontinuous at x = 5 as neither the limit exist nor f
∴ f(x) is continuous at x = 1.

is defined at x = 5.
 
 
Example 1: Discuss the continuity of the function [cos x] One-Sided Continuity


π A function f defined in some neighbourhood of a point
at x = , where [ ] denotes the greatest integer function.
2 a for x ≤ a is said to be continuous at a from the left if
  Solution: L.H.L = lim  cos x   0 . lim f(x) = f(a).


 x a 
x
2 A function f defined in some neighbourhood of a point
R.H.L = lim  cos x   1 .

a for x ≥ a is said to be continuous at a from the right if


x
2
lim f(x) = f(a).
x a 
   One-sided continuity is a collective term for functions
f = cos 2  = 0. continuous from the left or from the right.
2  
  
Since, L.H.L R.H.L the limit does not exist. If the function f is continuous at a, then it is continuous at a
π from the left and from the right . Conversely, if the function f
So, the function is discontinuous at x = . is continuous at a from the left and from the right, then lim
2 x →a
Example 2: Test the continuity of the function f(x) at f(x) exists, and lim f(x) = f(a).
x →a


e1/ x
x = 0, where f ( x )  , when x ≠ 0 and f(0) = 0. The function y = sgn x is neither left continuous nor right
1  e1/ x continuous at x = 0.

FREE BOOKS FOR JEE & NEET =>(@iitjeeadv)


2.4 Differential Calculus for JEE Main and Advanced

The function y = sin–1 x is left continuous at x = 1. We cannot of definition of the function f(x) and if at that new point the
discuss the right continuity here as the function is not defined value of the function is put equal to the common value of the
in the right neighbourhood of x = 1. left hand and right hand limits, the (new) function F(x) thus
The function y = {x} is right continuous at x = 0 but left obtained, is continuous at the point a. Then, we can define the
discontinuous there. new function F(x) by the formula
At each integer n, the function f(x) = [x] is continuous from f ( x ) if x is in the domain of f
F( x )  
the right but discontinuous from the left because L if x  a
lim f ( x )  lim [ x ]  n  f (n )


x n  x n  The function F is continuous at x = a. It is called the continuous
extension of f to x = a.

but lim f ( x ) = lim [ x ] = n − 1 ≠ f (n ) . sin x
x→n − x→n − is not defined at the point

For example, the function y =
x

 x2 if x2 sin x
Example 4: Let f(x) =  x = 0. But since lim = 1 we can introduce a new function,

 x  1 if x2


x →0 x
Show that f is continuous from the left at 2, but not from the defined for all the values of x and coinciding with the old one
right. for x ≠ 0, which is everywhere continuous:
  Solution: f (2) = 2 + 1 = 3  sin x
 for x  0

lim f(x) = lim (x + 1) = 3 and F(x) =  x .




x 2 x 2
 1 for x  0
lim f(x) = lim x2 = 4 ps

x 2 x 2
Example 6: Let f(x) = ln(1  ax )  ln(1  bx ) . Find the


Since lim f(x) = f(2), f is continuous from the left at 2 and
el
x 2  x
lim f(x) ≠ f(2), f is not continuous from the right at 2.
value which should be assigned to f at x = 0, so that it is
eh

x 2 continuous at x = 0.
je

Continuity at End Points   Solution:  lim f(x)


iit

x→0
Let a function y = f(x) be defined on [a, b].
 ln (1  ax )   ln (1  bx ) 
@

Then the function f(x) is said to be continuous at the left end = lim a    b  ( bx ) 
 ax   

x→0
point x = a if, f(a) = lim f(x),
x a 

= a. 1 + b. 1 = a + b .

and f(x) is said to be continuous at the right end point x = b
if, f(b) = lim f(x).  log (1  x ) 
∵ lim  1
x b

 x 0 x 
For example, consider the function f(x) = {x}, 0 ≤ x ≤ 1. It is

So, f(0) = a + b, if f is continuous at x = 0.
continuous at x = 0 and discontinuous at x = 1.
x −a Example 7: A function y = f (x) is defined as


Example 5: Discuss the continuity of f(x) =
b−x  ( x  3)


 k sin for x  2
where a < b, at x = a and x = b.  6
f(x) = 
  Solution: We notice that the domain of the function is  3  11  x

for x  2

(a, b). At the left end point x = a, we have lim f(x) = 0 = f(a). 
 x2
x a 
Hence f is continuous at x = a. If f (x) is continuous at x = 2, then find the value of k.

At the right end point x = b, f(b) is undefined and lim f(x) = ∞.   Solution: Since f (x) is continuous at x = 2
x b

Hence f is discontinuous at x = b. k
lim f(x) = f (2) ⇒ lim f(x) = .

Continuous Extension to a Point x→2 x→2 2
A function may have a limit at a point where it is undefined.  5  k
Now, f (2–) = k sin  = and
Then we can extend the definition of the function at that point  6  2


to make it continuous there. Suppose f(a) is not defined, but 3  11  x
f (2+) = lim
lim f(x) = L exists. If the point a is added to the domain x2

x 2
x →a

FREE BOOKS FOR JEE & NEET =>(@iitjeeadv)
Continuity of Functions 2.5

( x  2) 1
= lim .
 
=
( x  2) 3  11  x

x 2 6
k 1 1
∴ = ⇒ k= .



2 6 3
ln(1  x )1 x  x
Example 8: Let f(x) = , then find the
x2


value of f(0) so that the function f is continuous at x = 0. It follows that the function is continuous at x = 0, since the
(1  x ) ln(1  x )  x  0  value of the function and the value of the limit are the same
  Solution: lim f ( x ) = lim  0 form 
x →0 x 0 x2   at 0. This shows that the behaviour of a function can be very

ln(1  x )  1  1
complex in the vincinity of x = a, even though the function is
= lim (by L’Hospital’s Rule) continuous at a.
x 0


2x
1 ln(1  x ) 1 Example 11: Test the continuity of f(x) at x = 0 if




=lim  .   1 1
2 x 0 2  

x 2 
  |x| x  , x  0
f(x) =  ( x 1) .
For continuity, we must have f(0) = lim f ( x ) .
x →0  , x0
 0
1
Hence, f(0) = .
2   Solution: L.H.L. = lim f(x) = lim f(0 – h)
ps

x0 h→0
Example 9: Examine the continuity of the function
 1 1 


el
2   
 x2 1  |0  h | ( 0  h ) 
 2 , x 1 = lim (0  h  1)
eh

 x  2 | x  1 | 1 h→0
f(x) =  , at x = 1.  1
2  
1 

1
je

(0  h  1)  |0  h | ( 0  h ) 
 , x 1 = hlim
2


0
iit

= lim (1 – h) = (1 – 0) = 1.
2 2
x2 1
@


  Solution: R.H.L = f(1+) = lim h→0
x 1 x 2  2 | x  1 | 1

R.H.L = lim f(x) = lim f(0 + h)
x0 

h→0
x2 1
= xlim
1 x 2  2( x  1)  1

 1 1
2  

= lim (h  1)  |h| h 
h 0

( x  1)
= lim =∞ 2
2
x 1 ( x  1)  2


= lim (h  1) h

h 0
There is no need to find the L.H.L. at x = 1 since R.H.L. is
non-existent.
2

= lim(h + 1) h = e −2
Thus, f(x) is discontinuous at x = 1. h →0

f(0) = 0.
Example 10: Check the continuity of the function
∴ L.H.L. ≠ R.H.L.


 x sin(1 / x ), x ≠ 0 Hence, f(x) is discontinuous at x = 0.
f(x) =  at x = 0 .
x=0

0, Example 12: Discuss the continuity of the function
1


  Solution: Consider the limit lim xsin    a 2[ x ]{x}  1
x  , x0

x→0
f(x) =  2[ x ]  {x} (a ≠ 1)
If x > 0, – x ≤ x sin (1/x) ≤ x,  log a

 , x0
and if x < 0, x ≤ x sin (1/x) ≤ – x. e

Thus, for x ≠ 0, – |x| ≤ x sin (1/x) ≤ |x|. at x = 0, where [x] and {x} are the greatest integer part and
fractional part of x respectively.
Since both |x| → 0 and – |x| → 0 as x → 0, the Sandwich
theorem applies and we can conclude that x sin (1/x) → 0 as   Solution: f(0) = loge a

x → 0. This is illustrated in the following figure. L.H.L. = lim f(x) = lim f(0 – h)
x0 

h→0

FREE BOOKS FOR JEE & NEET =>(@iitjeeadv)


2.6 Differential Calculus for JEE Main and Advanced

a 2[ 0  h ]{0  h}  1 2 cos x − 1
= lim Example 14: Let f(x) =
h 0 2[0  h ]  {0  h} cot x − 1



a 2[ 0  h ]{1 (1 h )}  1    
 x   0, except at x = . Define f   so that f(x) may
= lim
h 0 2[0  h ]  {1  (1  h )}  2   


a 2  (1 h ])  1 a 1  1 1 be continuous at x = .
= lim   1 . 
h 0 2(1  h ) 1 a


R.H.L. = lim f(x) = lim f(0 + h)   Solution: f(x) will be continuous at x = , if




x0  h→0

a 2[  )]{0  h}1 lim f ( x )  f  
= lim
h 0 2[0  h ]  {0  h}
x  / 4 

 2 cos x  1
a 0 h  1 ∴ f  = lim
= lim  x  / 4 cot x 1



h 0 0  h

ah 1 ( 2 cos x  1) sin x
= lim = loge a = lim
h 0 h x  / 4 cos x  sin x

We find that 1 – 1/a = logea only when a = 1, which is not ( 2 cos x  1) ( 2 cos x  1) (cos x  sin x ) sin x
= lim
acceptable. Since L.H.L. ≠ R.H.L. = f(0), f(x) is discontinuous x / 4 ( 2 cos x  1) (cos x  sin x ) (cos x  sin x )
ps

at x = 0.
 
2 cos 2 x  1  cos x  sin x  sin x
el
lim
 
Example 13: If the function = x   / 4 (cos 2 x  sin 2 x ) 2 cos x  1


eh

tan(tan x )  sin(sin x )
f(x)= ( x  0) is continuous at x = 0,
tan x  sin x sin x (cos x  sin x )
je

= lim
then find the value of f (0). x  / 4 ( 2 cos x  1)

iit

tan(tan x )  sin(sin x )
  Solution: f(0) = lim 1  1 1 
@

tan x  sin x   

x0
2 2 2 1
tan(tan x )  sin(sin x ) =  .
1 2
= lim 2. 1
tan x  1  cos x  3

x0
2
x  x 2 
x
Example 15: Let
tan(tan x )  sin(sin x )


= 2 lim 3
e tan x − e x + ln (sec x + tan x ) − x

x0 x f(x) = be a continuous
tan x − x
1  tan 3 x 2 
function at x = 0. Find the value of f (0).
lim 3   tan x 
= 2 x  tan 5 x  ...... 
  

 3 15 


0 x
  Solution: For continuity of f at x = 0, we have

 sin 3 x sin 5 x  f (0) = lim f ( x )
  sin x    .........   x→0
 3 ! 5 ! 
 
e tan x  e x ln(sec x  tan x )  x

  tan 3 x sin 3 x   = lim  lim
x 0 tan x  x x 0  tan x  x  3

      x
  tan x  sin x   3 3!    x3 
= 2 lim  
x0 
  .... 
 x 3
 x 3
 e x (e tan x  x  1) ln(sec x  tan x )  x
  = lim  3 lim
  x 0 tan x  x x 0 x3

  tan x   1  cos x  1 1  sec x  1
= 2 lim    3  6 = 1 + 3 lim (by L’Hospital’s Rule)
x   3x 2


x0   x2  

x0

1 1 1 3
= 2    = 2. =1+ = .
2 2

2 2

FREE BOOKS FOR JEE & NEET =>(@iitjeeadv)
Continuity of Functions 2.7

Example 16: Let a function f(x) be defined in the h


h 2  4 sin 2


neighborhood of as 2
­
= lim
ln(2  cos 2 x ) h4


h0
 2 for x  0
 ln (1  sin 3x ) h  2 sin
h
h  2 sin
h
 2 . lim 2
f (x) =  = lim
 h 0 h h 0 h3


 e
sin 2 x
1
 for x  0 2 t  2 sin t
ln(1  tan 9 x ) = 2 · lim where h = 2t
8t 3



t0
Find whether it is possible to define f (0) so that f may be
t  sin t 1

continuous at x = 0. = lim  .

3
12


t0 2t
  Solution: lim f (0 + h)
sin h  ln(e h cos h )

h→0
lim f (0  h )  lim
esin 2 h  1 sin 2h .(2h )
h 0 h 0 6h 2


= lim
h  0 ln(1  tan 9h ) sin 2h .( 2h ) sin h  h  ln cos h

= lim
6h 2

h0
esin 2 h  1 sin 2h 2h
= lim . lim lim 1
h  0 sin 2h h  0 2h h  0 ln(1  tan 9h ) sin h − h

1
− ln (cos h ) h
2

= 2
6

2h 6h
= 1.1. lim
ps
h 0 tan 9h ln(1  tan 9h ) cot 9h sin h  h 1 1
= lim · h + lim  ln  cos h  h
el
2

h3

2h 2 h0 h 0 6
= lim  . lim f (0 – h)
eh

h 0 tan 9h 9
Lim (cos h −1)
h→0 1
1 1
= 0 − ln e
h →0 2
h
je

ln(1  2 sin 2 h ) 2 sin 2 h =



= lim . lim 6 12
iit

h0 2 sin 2 h h0 ln 2 (1  sin 3h )



1
Hence, lim f (0  h )  lim f (0  h ) = f(0) =− = p.
@

2 2
2 sin h h h 0 h 0 12
= lim . lim
h0 h2 h0 ln 2 (1  sin 3h )

sin 2 x + A sin x + B cos x
Example 18: If f(x) = is
2 · h 2 (  sin 3h ) 2
2


= lim = x3
h 0 sin 3h ·ln (1  sin 3h )
3 2
9 continuous at x = 0, find the values of A and B. Also find f(0).

2   Solution: As f(x) is continuous at x = 0,

Since f (0+) = f(0–) = the limit exists. sin 2 x  A sin x  B cos x
9 f(0) = lim .
x 0 x3


Therefore, it is possible to define f(0) such that f is continuous
at x = 0. As denominator → 0, when x → 0, numerator should also
2 approach 0, which is possible only if
∴ f (0) = .
sin 2(0) + A sin (0) + B cos(0) = 0 ⇒ B = 0

9

Example 17: Determine the value of p, so that the sin 2 x + A sin x


f(0) = lim
 x  2 cos x  2 2 x3

x→0
 for x  0
 x4  sin x   2 cos x  A 
 = lim   
function f ( x )   p for x  0 x→0  x   x2 


 sin x  n (e cos x ) for x  0
x
 2 cos x  A 

 6x 2 = lim  
 x2 

x→0
is continuous at x = 0. Again we can see that denominator → 0 as x → 0.
  Solution: lim f (0  h ) = lim
h 2  2(1  cos h) ∴ Numerator should also approach 0 as x → 0
h4 ⇒ 2 + A = 0 ⇒ A = – 2.

h0 h0


FREE BOOKS FOR JEE & NEET =>(@iitjeeadv)
2.8 Differential Calculus for JEE Main and Advanced

 4 sin 2 x / 2  Now, f(x) is continuous at x = 0.
2 cos x  2 
⇒ f(0) = lim    xlim   ∴ L.H.L. = R.H.L. = f(0)
  0 

2
x→0 x  x2  5−a
= ed = 3
  sin 2 x / 2 


2
= xlim    1 ∴ a = 1, d = ln 3, c = 0, b = –6.
0  2
 x /4 

So, we get A = – 2, B = 0 and f(0) = – 1. Example 20: Let f (x)



1  a cos 2x  b cos 4x

Example 19: Let  if x  0
=  x 2 sin 2 x ,



 c if x  0
 a (1  x sin x )  b cos x  5 
 ,x0 be continuous at x = 0, then find the values of a, b and c.
 x2


f(x) =  3 ,x0 1  a cos 2 x  b cos 4 x
  Solution: lim

 x4


1/ x x0
   cx  dx 3  
 1     ,x0 As x → 0, Denominator → 0 and


Numerator → 1 + a + b
2
   x  


If f is continuous at x = 0, then find the values of a, b, c and d. For existence of limit, a + b + 1 = 0 ...(1)



a cos 2 x  b cos 4 x  (a  b)
  Solution: f(0) = 3 ∴ c = lim ...(2)

x4



x0
R.H.L. = lim f(x) = lim f(0 + h) = lim f(h)
x0 h→0 h→0 a (1  cos 2 x ) b(1  cos 4 x )

  ch  dh 3
1/ h
ps

2
  = – lim x x2
= hlim 1     2

el
0

  h2 
x0 x

Since f is continuous at x = 0, R.H.L. exists. 1 1
eh

The limit of numerator = 4a   + 16b  


For existence of R.H.L., c must be 0.   2


2
je

∴ R.H.L. = lim {1 + dh)1/h (form 1∞) ⇒ 2a + 8b = 0 ⇒ a = – 4b ...(3)


iit





h→0
From (1) and (2) – 4b + b = – 1
1 dh 1)


lim(
@

= e h 0
= ed 1 4
⇒ b=

and a=–
L.H.L. = lim f(x) = lim f(0 – h) 3 3



x0 x→0
4(1  cos 2 x )  (1  cos 4 x )
a (1  ( h ) sin( h ))  b cos( h )  5 From (2), c = lim
= hlim x0 3x 2
0 (h )2

a(1 − hsin h) + bcos h + 5 8 sin 2 x − 2 sin 2 2 x
= lim
= lim 3x 4

x→0
h →0 h2

For finite value of L.H.L. the numerator must tend to 0 as h → 0. 8 sin 2 x − 8 sin 2 x cos 2 x
∴ a + b + 5 = 0. = lim
3x 4

x→0
a − ahsin h − (5 + a)cos h + 5
L.H.L. = lim 8 sin 2 x sin 2 x 8
h →0 h2 = lim · · = .
2 2
(5 + a)(1 − cos h) − a hsin h 3 3

x→0 x x
= lim
h →0 h2

Example 21: Discuss the continuity of the function,


 (a + 5)(1 − cos h)(1+ cos h) a hsin
 
h ln(2  x )  x 2 n sin x
= hlim  −  f(x) = lim at x = 1.
→0 
 h 2 (1+ cosh) h2 
 1  x 2n

n 


 (a + 5)sin h asin h
2
   Solution: We have f(1) = (ln3 – sin1)/2
 2 − 

= hlim  0,
→ 0  h (1+ cos h)  if x 2  1
and lim x 2 n  
 h 

 , if x 2  1
n 
(a + 5)(1) 2 
= –a.1
1+1 ∴ For x2 < 1, we have


a +5  5a  ln(2  x )  x 2 n sin x
= –a=  . f(x) = lim = – ln (2 + x)
n  1  x 2n

 2 

2

FREE BOOKS FOR JEE & NEET =>(@iitjeeadv)


Continuity of Functions 2.9

Again for x2 > 1, we have we have f(0) = f(0). f(0) ⇒ f(0) = 0 or 1.



1 If f(0) = 0 then putting y = 0 in (1)
ln(2  x )  sin x
2n we get f(x) = f(x) . f(0)
f(x) = lim x = – sin(x)
1 ⇒ f(x) = 0 for all x.

n 
1  2n


x Hence f is continuous for all x. Here the continuity of f at
Here, as x → 1 x = 1 is not required as a condition.
lim f(x) = ln3 and lim f(x) = – sin1 If f(0) = 1 then we have the following:

x1 x1 Putting x = 1, y = –1 in (1)
So, lim f(x) ≠ lim f(x). we have f(0) = f(1). f(–1) ⇒ f(1). f(–1) = 1



Hence f(1) is non-zero. ...(2)

x1 x1

Therefore, f(x) is discontinuous at x = 1. As the function is continuous at x = 1,


Example 22: Find the value of f(1) if the function we have lim f(1 + h) = f(1)


h→0


x m 1  (m  1) x  m (using f(x + y) = f(x) . f(y))
f(x) = lim , x≠1
( x  1) 2
⇒ lim f(1) . f(h) = f(1)

n 


h→0
is continuous at x = 1. ⇒ lim f(h) = 1 using (2) ...(3)



  Solution: We have
h→0

Now, we consider any arbitrary point x.



x ( x m  1)  m( x  1)
lim f ( x )  lim , lim f(x + h) = lim f(x) . f(h)
x 0 x 0 ( x  1) 2

h→0 h→0

m 1
ps = f(x) lim f(h)
x ( x  1)(1  x  x  ....  x
2
)  m( x  1)

h→0
= lim
el
x 0 ( x  1) 2

= f(x) using (3)


Hence, at any arbitrary point x, limit = function's value.
eh

( x  x  x  ....  x )  (1  1  ...m times)


2 3 m
= xlim Therefore, the function is continuous for all values of x.
0 x 1
je

( x  1)  ( x 2  1)  ( x 3  1)  .....  ( x m  1) Example 24: Let f be a function satisfying
iit


= lim 6 − f ( y) = f(x)f(y) and lim f(x) = 6.
x 0 x 1 f(x + y) +

@


x→0
 x 1 x 12 3
x 1 m
Discuss the continuity of f.
= lim 1    .....  
x 0  x 1 x 1 x  1    Solution: lim f(x + h)= lim f ( x )f (h )  6  f (h ) 


h 0  

h→0
m(m +1)
= 1 + 2 + 3 + .... + m = = f ( x ) lim f (h )  lim 6  f (h )
2 h 0 h 0

Hence, for f to be continuous at x = 1, we should have
= f(x).6 – 0 = 6f(x) ≠ f(x)
m(m +1)
f(1) = . Putting x = 0, y = 0 in the given relation, we get
2
f(0) + 6 − f (0) = f2(0). ⇒ f(0) ≠ 0
Example 23: A function f(x) satisfies f(x + y) = f(x).

∴ f(x) = 0 for all x is not possible.


f(y) for all x and y ∈ R. Show that the function is continuous
for all values of x if it is continuous at x = 1. Thus, lim f(x + h) ≠ f(x).
h→0
  Solution: We have f(x + y) = f(x) . f(y) ...(1)
Hence, f is discontinuous at all x.


Putting x = 0, y = 0 in (1)

Concept Problems A

1. Check the continuity of
x , x 3

 tan(sin t ) 
 , sin t  0 2. If f (x) =  4 , x  3 is continuous at x = 3 then find
at t = 0. 3x  5 , x  3

f(t) =  sin t
 1 

 , sin t  0
the value of λ.

FREE BOOKS FOR JEE & NEET =>(@iitjeeadv)


2.10  
Differential Calculus for JEE Main and Advanced

6. Use continuity to evaluate the limits :


2 x4


3. If f(x) = , x ≠ 0 is continuous function at x = 0, 5 x
sin( 2 x )

(i) lim
x 4 5  x



then find the value of f(0).
 x 2  (a  2) x  2a  x2  4 
4.

If the function f(x) =  x2
, x2
is (ii) lim tan 1  
 3x  6 x 
x 1 2





 2 , x2
7. For the function φ(x) = x.lnsin2x when x > 0, and φ(0) = 0,
continuous at x = 2 then find the value of a.


discuss the right continuity at x = 0.
5. If f and g are continuous functions with f(3) = 5 and lim 8. The function f(x) = (x + 1)cotx is undefined at x = 0. Find

x→3




[2f(x) – g(x)] = 4, find g(3). the value which should be assigned to f at x = 0, so that
it is continuous at x = 0.

Practice Problems A

9. If possible find value of λ for which f(x) is continuous at 1
π 14. If f(x) = (1 + ax ) x x<0
x= .



2
=b x=0



1  sin x  1
f(x) = , x ( x  c) 3  1
1  cos 2 x



2 = x>0
 x



ps
= λ, x find the values of a, b, c, f(x) is continuous at x = 0.


2
el
1 ln(sec 2 x )
2x    15. If f (x) = cos(x cos ) and g (x) = for
eh

x

= , x x sin x
4  2x    2


2
x ≠ 0 and they are both continuous at x = 0 then show that
je

10. Find the value of f(0) so that the function f (0) = g (0) = 1.
iit

16. The function f(x) = a [x + 1] + b [x –1], where [.] is the
1− x − 3 1− x

greatest integer function then find the condition for which
@

f(x) = is continuous at x = 0.
x f(x) is continuous at x = 1.
11. Show that the function f defined on R by setting f(x) = 17. Let f (x)

1  a
| x |m sin , when x ≠ 0, f(0) = 0, is continuous at x = 0 (1 | sin x |) |sin x| for    x  0
x  6
whenever m > 0. 
= b for x  0
 tan 2 x

1
12. The function f(x) = 1–xsin is meaningless for x = 0. 
e tan 3x

x
 for 0  x 
How should one choose the value f(0) so that f(x) is  6
continuous for x = 0? Find ‘a’ and ‘b’ if f is continuous at x = 0.
13. Find the values of a and b such that the function
 1

f(x) = x + a 2 sin x, 0x 18. If f(x) is continuous in [0, 1] and f   = 1 then
2



4
   n 
= 2x cot x + b, x find lim f   .


4 2 n 
 2 n 1 

= a cos 2x – b sin x, x sin(e x− 2 − 1)
, x ≠ 2.


2 19. Let f(x) =
log( x − 1)

π π
is continuous at and . If f(x) is continuous at x = 2 find f(2).

4 2

2.2 Continuity In An Interval Roughly speaking, a function is said to be continuous on


an interval if its graph has no breaks, jumps, or holes in

We can extend the concept of continuity and say that a function that interval. Continuity is important because, as we shall
f(x) is continuous in an interval if it is continuous at every see, function with this property have many other desirable
point in the interval. properties.

FREE BOOKS FOR JEE & NEET =>(@iitjeeadv)


Continuity of Functions 2.11

Continuity in an Open Interval


A function f is said to be continuous in an open interval (a , b)
if f is continuous at each and every point lying in the interval
(a , b) .

Continuity in a Closed Interval


A function f is said to be continuous in a closed interval [a, b] if : Thus, the domain of continuity of each of the functions sin
(i) f is continuous in the open interval (a , b) x, cos x, tan x, cot x, sec x, and cosec x coincides with the


(ii) f is right continuous at ‘a’ corresponding domain of definition.


i.e. lim f(x) = f(a). 
x a  f(x) = tan x is continuous at all points except x = (2n + 1) ,

(iii) f is left continuous at ‘b’ n ∈ I.


i.e. lim f(x) = f(b). Note: f is a continuous function if it is continuous at each
x b



number a in its domain.
For instance, f(x) =  x 2  3x  2 , where 1 ≤ x ≤ 2, is a The examination of the graph of the function y = 1/x in the
function continuous on this interval, since it is continuous at vicinity of the point x = 0 clearly shows that it "splits" into
every point of the interval (1, 2), continuous on the right at two separate curves at that point.
the point x = 1, and continuous on the left at the point x = 2.
However, the function f(x) = 1/x, whose domain consists of
1
The function y = is continuous in the open interval the intervals (–∞, 0) and (0, ∞) is continuous. Although this
1− x2 function explodes at 0, this does not prevent it from being a
(–1, 1). It is discontinuous at the points x = – 1 and x = 1. continuous function. The key to being continuous is that the
ps
Similarly, f is continuous on the half-open interval (a, b] if it is function is continuous at each number in its domain. The
el
continuous at each number between a and b and is continuous number 0 is not in the domain of f(x) = 1/x.
eh

from the left at the endpoint b. However, f(x)=1/x is discontinuous in the interval (–∞, ∞)
In the case where f is continuous on (–∞, ∞), we will say that f Theorem Every elementary function is contin-uous at every
je

is continuous everywhere. The general procedure for showing



point in its domain.
iit

that a function is continuous everywhere is to show that it is The polynomial function f(x) = a 0x n + a 1x n – 1 + ..... +
continuous at an arbitrary real number. an – 1 x + an is continuous over R .
@

Continuity of Elementary Functions It follows from the preceding theorem and the fact that constant
function and the identity function are continuous over R.
All basic elementary functions are continuous in the intervals
where they are defined. If f(x) is continuous for any particular value of x, then any
polynomial in f(x), such as
The constant function and the identity function are continuous
over R. a0{f(x)}n + a1{f(x)}n – 1 + ..... + an – 1 {f(x)} + a n is also
We know that y = sin x and y = cos x are continuous for every continuous

value of x. a 0 x n  a1x n 1  .....  a n
The rational function f(x) =
From the graph we see that f(x) = sin x is continuous in its b0 x m  b1x m 1  .....  b m
is continuous at every value of x except at those points where
entire domain. the denominator becomes zero.

3 x
For instance, the function f(x) = is continuous
We see that the functions 4x  7
sin x cos x 7
y = tan x = , y = cot x = throughout the entire number line except for the point x = − ,
cos x sin x

4
1 1 at which the denominator of the fraction vanishes. And the
y = sec x = , y = cosec x =
cos x sin x x 3  4x 2  x  1

function f(x) =  is continuous everywhere
are continuous for all those values of x for which they are x2  x 1
defined. The discontinuities of these four functions arise only on R, since the denominator never vanishes.
when the denominators become zero and for such values of x, Now, each of the irrational functions ( x − a )(b − x ),
these functions themselves cease to be defined.
 x a 
f(x) = tan x is continuous at all points except 3 ( x − a )(b − x ) and 3
 b  x  are continuous at each
  
x = (2n + 1) , n ∈ I.
 point in their domain, since these functions are elementary.

FREE BOOKS FOR JEE & NEET =>(@iitjeeadv)


2.12  
Differential Calculus for JEE Main and Advanced

All trigonometric functions, exponential and logarithmic y = ln x + tan–1 x is continuous on (0, ∞). The denominator,
functions are continuous in their domain. y = x2 – 1, is a polynomial, so it is continuous everywhere.
Example 1: Test the following functions for continuity Therefore, f is continuous at all positive numbers x except where
x2 – 1 = 0. So f is continuous on the intervals (0, 1) and (1, ∞).


2 x 5  8x 2  11
(a) f(x) =
x 4  4 x 3  8x 2  8x  4
We need to examine the continuity of non-elementary functions


carefully.
3 sin 3 x  cos 2 x  1


(b) f(x) = For instance, the greatest integer function f(x) = [x], is continuous
4 cos x  2



everywhere except for the integral values of x.
  Solution:

(a) A function representing a ratio of two continuous functions

(polynomials in this case) is discontinuous only at points
for which the denominator is zero.
But in this case (x4 + 4x3 + 8x2 + 8x + 4)
= (x2 + 2x + 2)2 = [(x + 1)2 + 1]2 > 0 (non-zero).

Hence f(x) is continuous everywhere.
(b) The function f(x) suffers discontinuities only at points for
which the denominator becomes zero i.e. at the roots of y = [x] is discontinuous at x ∈ I.


the equation Also, the fractional part function f(x) = {x}, is continuous
4 cos x – 2 = 0 ⇒ cos x = 1/2. everywhere except for the integral values of x.

⇒ x = 2nπ ± π/3, n ∈ I. Hence, the continuity of functions {f(x)} and [f(x)] should be

checked at all points where f(x) becomes an integer.
Thus the function f(x) is continuous for all real x, except at the
ps
points 2nπ ± π/3, n ∈ I. |x|
el
The function y = sgn x (or y = ) is discontinuous at the
x  kx  1 . Find all possible
2 x
point x = 0.
eh

Example 2: Let f (x) =




x2  k Hence, the continuity of the function sgn (f(x)) should be checked
je

values of k for which f is continuous for every x ∈ R. at the points where f(x) = 0. (Note that if f(x) is constanly equal
 
 
iit

to 0 when x → a then x = a may not be a point of discontinuity).


x 2  kx  1
  Solution: f (x) =
@

Usually there are only a few points in the domain of a given


x2  k

function f where a discontinuity can occur.
For f to be continuous ∀ x ∈ R To find the points of discontinuity, we collect all the doubtful
x2 + kx + 1 ≥ 0 and x2 – k must not have any real root. points and examine them for continuity.
∴ k2 – 4 ≤ 0 and k < 0 We use the term suspicious point for a number a where


⇒ k ∈ [–2, 2] and k < 0
(i) The definition of the function changes or domain of f

From above, k ∈ [–2, 0).

splits, or
Example 3: Where is the function F(x) = ln(1 + cos x) (ii) substitution of x = a causes division by 0 in the function.



continuous? A function may be discontinuous at a suspicious point which
  Solution: We know that f(x) = ln x is continuous and can be found using the test of continuity.

g(x) = 1 + cos x is continuous (because both y = 1 and y = x2 −1
For example, y = 2 has two suspicious points x = ± 2
cos x are continuous). Therefore, F(x) = f(g(x)) is continuous x −4
wherever it is defined. (where the denominator becomes 0).
Now ln(1 + cos x) is defined when 1 + cos x > 0. So it is 1
y = x sin has one suspicious point x = 0.
undefined when cos x = – 1, and this happens when x = ± π, x
± 3π, .... Thus, F has discontinuities when x is an odd multiple For the function y = |x2 – 4| we have
of π and is continuous on the intervals between these values. y = x2 – 4 when x2 – 4 ≥ 0

and y = 4 – x2 when x2 – 4 < 0.
Example 4: Where is the function

This means the definition of the function changes when x2 – 4 = 0,


ln x + tan −1 x i.e. x = ± 2. Thus, the function has two suspicious points x = ± 2.
f(x) = continuous?
x2 −1

Note: There is no chance of continuity at points where


  Solution: We know that the function y = ln x is con- the function is not defined. We know that the function cannot

tinuous for x > 0 and y = tan–1x is continuous on R. Thus, be continuous at such points.

FREE BOOKS FOR JEE & NEET =>(@iitjeeadv)


Continuity of Functions 2.13

Example 5: If f(x) = [sin πx], 0 ≤ x < 1 L.H.L. = lim (2 sin x ) = 2






 2  5 x
=  x   sgn  x   , 1 ≤ x ≤ 2, 2
 3  4

R.H.L. = lim A sin x + B = – A + B
where { . } represents the fractional function then find the x



suspicious points for continuity of function in the interval [0, 2]. 2
So B – A = 2 ...(1)
  Solution:



At x = π/2

(i) Continuity should be checked at the end-points of intervals L.H.L. = lim A sin x + B = A + B

of each definition i.e. x = 0, 1, 2. x


(ii) For [sin πx], continuity should be checked at all values of


2

R.H.L. = lim cos x = 0



x at which sin πx ∈ Ι i.e. x = 0, 1/2 

x
 2  5 2
(iii) For  x   sgn  x   , continuity should be checked
 3  4 So A + B = 0 ...(2)


when x – 5/4 = 0 i.e. x = 5/4 and when x – 2/3 ∈ Ι i.e. Solving (1) and (2) we get A = – 1 and B = 1.




x = 5/3.
Example 8: Find the intervals on which f and g are
Finally, continuity should be checked at the suspicious points



x = 0, 1/2, 1, 5/4, 5/3 and 2 .  3  x if 5  x  2
continuous where f(x) = 
 x  2 if 2  x  5
Example 6: Show that the function


2 x  3, 3  x  2  2  x if 5  x  2
 and g(x) =  .
 x  2 if 2  x  5
ps
f(x) =  x  1, 2  x  0
 x  2,

0  x 1   Solution: The domain for both function is [–5, 5). Both
el
  
functions are continuous except possibly at the suspicious
eh

is discontinuous at x = 0 and continuous at other points in the


point x= 2. Examining f, we see
interval [–3, 1].
je

lim f(x) = lim (3 – x) = 1


  Solution: The graph of

x 2 x 2
iit

2 x  3, 3  x  2
 and lim f(x) = lim (x – 2) = 0
f(x) =  x  1, 2  x  0
@


x 2 x→ 2+
 x  2,

 0  x 1 so lim f(x) does not exist and f is discontinuous at x = 2. Thus,
x→2
is plotted below : f is continuous for –5 ≤ x < 2 and for 2 < x < 5.
For g, we have g(2) = 0
and lim g(x) = lim (2 – x) = 0
x 2 x 2

and lim g(x) = lim (x – 2) = 0.


x 2 x 2

Therefore, lim g(x) = 0 = g(2), and g is continuous at x = 2.


x→2
Here, we observe from the graph that at x = 0, Hence, g is continuous throughout the interval [–5, 5).
lim f(x) = 1 and lim f(x) = 2, which shows that the function
x0 x0 Example 9: Find the points of discontinuity of the


is discontinuous at x = 0 and continuous at every other point following functions for x ∈ R.
in [–3, 1]. 1 1
(i) f(x) = (ii) f(x) = 2
2 sin x − 1 x  3 | x | 2


Example 7: Let


 2 sin x if x   / 2 1 1
  (iii) f(x) = (iv) f(x) =
 x + x +1
4 2 x −1


f(x) = A sin x  B if   x   / 2 . 1 − e x −2

 2
(v) f(x) = [[x]] – [x – 2], where [.] represents the greatest

 cos x if x  /2
integer function.
Find A and B so as to make the function continuous.   Solution: A function is discontinuous at all such points

  Solution: At x = – π/2 where it is undefined.


FREE BOOKS FOR JEE & NEET =>(@iitjeeadv)
2.14  
Differential Calculus for JEE Main and Advanced

1 π 2π 4π 5π
(i) f(x) = when 2 cosx = ± 1 ⇒ x = , , , ;
2 sin x − 1 3 3

3 3
f(x) is discontinuous when 2 sinx – 1 = 0 when 2 cosx = ± 2 ⇒ x = 0, π, 2π.

1 π 5π Clearly from the graph given above, f(x) is discontinuous at
i.e. sinx = ⇒ x = 2nπ + , 2nπ + ,n∈I π π 3π 2π 4π 5π

2 6 6 x = 0, , , , , , , 2π.
3 2 2 3 3 3
1
(ii) f(x) =
x  3 | x | 2
2

Example 11: Draw the graph and discuss the continuity



f(x) is discontinuous when x – 3|x| + 2 = 0 2 of f(x) = [sinx + cosx], x ∈ [0, 2π], where [.] represents the
greatest integer function.


|x|2 – 3|x| + 2 = 0



(|x| – 1) (|x| – 2) = 0 Solution: f(x) = [sinx + cosx]






|x| = 1, 2  
2 sin  x  



x = ±1, ±2 Let g(x) = sinx + cosx =
 4


1 1
(iii) f(x) = 4
x + x2 +1
= 2 The range of g(x) is [ − 2 , 2 ].
 2 1

3
  
2 
x The suspicious points occur when g(x) = 0 , ± 1
 4
2 ⇒ x = 0, π , 3π , π, 3π , 7 π , 2π
 1

3 2 4 2 2
Now, x4 + x2 + 1 =  x 2    ≥1∀x∈R
 2

4 ps
⇒ f(x) is continuous ∀ x ∈ R

el
1
(iv) f(x) =
eh

x −1

1 − e x −2
je

f(x) is discontinuous when x – 2 = 0 and also



iit

x −1 x −1
x −1
when 1 – e x − 2 = 0 ⇒ e x − 2 = 1⇒ = 0 ⇒ x = 1.
@

x−2

Thus f(x) is discontinuous at x = 1, 2. Clearly from the graph given above f(x) is discontinuous at
(v) f(x) = [[x]] – [x – 2] = [x] – ([x] – 2) = 2 π 3π 3π 7 π
⇒ f(x) is continuous ∀ x ∈ R x = 0, , , π, , , 2π.


2 4 2 2
Example 10: Draw the graph and find the points of
Example 12: Discuss the continuity of f(x) in [0,2] where


discontinuity for f(x) = [2cos x], x ∈ [0, 2π], (where [.]


represents the greatest integer function)  [cos x ], x 1
f(x) = 
  Solution: | 2 x  3 | [ x  2], x  1
and [.] denotes the greatest integer function.

Solution: For x ∈ [0, 1], f(x) = [cos πx]



Since [x] is discontinuous at x ∈ I, we must check continuity
of [cos πx] at points where cos π x is an
1
integer. This happens at x = 0, , 1.
2
1
When x ∈ [0, 1], the suspicious points are x = 0, , 1.

2
Now for x ∈ (1, 2) , –1 < x – 2 < 0

∴ [x – 2] = –1 Thus, f(x) = –| 2x – 3 |
Since | x | is a continuous function, f(x) is continuous in (1, 2).
The suspicious points are: The right end point x = 2 is a suspicious point.
π 3π 1 3
when 2 cosx = 0 ⇒ x = , ; Hence, the suspicious points are 0, , 1, , 2.
2 2 2 2

FREE BOOKS FOR JEE & NEET =>(@iitjeeadv)


Continuity of Functions 2.15

1 , x0 lim f ( x )  lim {x2} = 0.25 = f(3.5).


 x 3.5 x 3.5 


0 1
, 0 x  (0  cos  x  1 )


2 Hence, f(x) is discontinuous at x = 3, 10 , 11, 12 .
1
 1  x 1 (1  cos  x  0)
Example 14: Let f(x) = maximum (sin t, 0 ≤ t ≤ x), 0 ≤ x ≤
,
 2
Now f(x) = 



2x  3 3 π


, 1 x 
2
(| 2 x  3 | 3  2 x ) 2π. Discuss the continuity of this function at x = .

2
3
3  2 x x2 (| 2 x  3 |  2 x  3)

,
2 Solution: f(x) = maximum (sin t, 0 ≤ t ≤ x), 0 ≤ x ≤ 2π



0 x2 ([ x  2]  0)
 ,
Graph of f(x)

 
If x ∈ 0,  , sin t is an increasing function
 2


Hence if t ∈ [0, x], sin t will attain its maximum value at t = x.
1 
It is clear from the graph that f(x) is discontinuous at x = 0,
2
, ∴
f(x) = sin x if x ∈ 0, 
 2
2 and continuous at all other points in [0, 2].
 
If x ∈  , 2  and t ∈ [0, x]
ps
sgn( x  2)  [ln x ], 1  x  3
 2 
Example 13: If f(x) = {x 2 } π
el

 3  x  3.5
.


then sin t will attain its maximum value when t =
where [.] denotes the greatest integer function and {.} 2
eh

represents the fractional part function, find the point where the π  
⇒ f(x) = sin = 1 if x ∈  , 2 
je

2 

continuity of f(x) should be checked. Hence, find the points 2
of discontinuity.
iit

  
sin x if x   0, 
Solution: Continuity should be checked at the end- ,  2
@





points of intervals of each definition, i.e., x = 1, 3, 3.5. ∴ f(x) = 

{x2} is discontinuous for those values of x where x2 is an integer. 
 1  
Hence, continuity should be checked when x2 = 10, 11, 12 or , if x   , 2 

 2 
x = 10 , 11, 12 .

sgn(x–2) should be checked when x –2 = 0 or x = 2. Now, f   = 1
[lnx] should be checked when lnx = 1 or x = e. 2
Hence, continuity must be checked at x =1, 2, e, 3, lim f(x) = lim sin x = 1





10 , 11, 12 , 3.5. x
2
x
2
Now, f(1) = 0 and
lim lim
lim f ( x )  lim sgn(x–2) × [lnx] = 0 

f(x) = 

1 = 1.
x 1 
x 1 
x


x 2
Hence f(x) is continuous at x = 1. 2

lim f ( x )  lim sgn(x–2) × [lnx] = (–1) × 0 = 0 π π


x  2 x  2
Since f( ) = L.H.L.= R.H.L., f(x) is continuous at x = .

2 2
lim f ( x )  lim sgn(x–2) × [lnx] = (1) × 0 = 0
x  2 x 2  Example 15: If the function




Hence, f(x) is continuous at x = 2. 2  ( 256  7 x )1/ 8
lim f ( x )  lim sgn(x – 2) × [lnx] = 1 f(x) = , x ≠ 0 is continuous
(5x  32)1/ 5  2

x 3 x 3

everywhere then find the value of f(0).
lim f ( x )  lim {x2} = 0
x 3 x 3 

  Solution:  f(x) is continuous at all points except at the



Hence, f(x) is discontinuous at x = 3. point where (5x + 32)1/5 = 2 i.e. x = 0.
Also f(x) is discontinuous at For continuity at x = 0,
x = 10 , 11, 12 because of {x2}. f(0) = lim f(x) = lim f(0 + h)


x→0 h→0

FREE BOOKS FOR JEE & NEET =>(@iitjeeadv)


2.16  
Differential Calculus for JEE Main and Advanced

[ note that here h assumes both positive and negative values ⇒ y = 0 for all x < 0.
close to 0] 2x 2 ,
 x0
Hence, f(x) =  which is clearly continuous
(256  7 h ) 1/ 8
 (256) 1/ 8

 0, x0
= – lim
h 0 (5h  32)1/ 5  (32)1/ 5 for all x as shown graphically below.


(256  7 h )  (256) 1/ 8 1/ 8
.(7 h )
(256  7 h )  256
= – lim
(5h  32)1/ 5  (32))1/ 5

h 0
.(5h )
(5h  32)  32
(256  7 h )1/ 8  (256)1/ 8
7 (256  7 h )  256
= lim


5 h 0 (5h  32)1/ 5  (32)1/ 5


(5h  32)  32 Example 18: Let f(x) = x3 – 3x2 + 6 ∀ x ∈ R, and



1  max f ( t ), x  1  t  x  2,  3  x  0
.(256)1/ 8−1   g(x) = 
7 8 xn  an 1  x , for x0
= . ∵ lim  na n 1 
5 1 .(32)1/ 5−1  x a x  a  Test continuity of g (x) for x ∈ [–3, 1].

5
Solution: Since f(x) = x3 – 3x2 + 6
7 (2) −7 7 1



= ⋅ −4 = ⋅ 3 =
7
. f ′(x) = 3x2 – 6x = 3x (x – 2)
ps

8 (2) 8 2 64

For maxima and minima, f ′(x) = 0
7
⇒ x = 0, 2
el
Hence, f(0) = .

64
f ″ (x) = 6x – 6
eh
Example 16: If f(x) is a continuous function for all real
f ″ (0) = –6 < 0 (local maxima at x = 0)


je

values of x and satisfies x2 + (f(x) – 2) x + 2 3 – 3 – 3 .



f ″ (2) = 6 > 0 (local minima at x = 2)
iit

f(x) = 0, ∀ x ∈ R, then find the value of f( 3 ).



Also, f(0) = 6.
@

Solution: As f(x) is continuous for all x ∈ R. Now graph of f(x) is :




lim f(x) = f( 3 ) where
x→ 3


x 2  2x  2 3  3
f(x) = ,x≠ 3
3x

lim f(x) = lim x  2 x  2 3  3
2

x→ 3
3x


x 3

(2  3  x )( 3  x )
= lim  2(1  3 )
x 3 ( 3  x)

∴ f( 3 ) = 2(1 – 3 ).


Example 17: Let y = f(x) be defined parametrically as
Clearly f(x) is increasing in (– ∞, 0) and (2, ∞) and decreasing


y = t2 + t | t |, x = 2t – | t |, t ∈ R. Then examine the continuity
of f(x) at x = 0. in (0, 2).
Consider x + 2 < 0 ⇒ x < – 2

Solution: y = t2 + t | t | and x = 2t – | t |





When t ≥ 0, For –3 ≤ x < – 2,

x = 2t – t = t, y = t2 + t2 = 2t2 –2 ≤ x + 1 < –1 and –1 ≤ x + 2 < 0

⇒ x = t and y = 2t2

Since f(x) increases, the maximum value of g(x) is f(x +2).




⇒ y = 2x2 ∀ x ≥ 0
∴ g(x) = f(x + 2) if –3 ≤ x < – 2.

When t < 0,

⇒ x = 2t + t = 3t and y = t2 – t2 = 0. Now consider x + 1 < 0 and 0 ≤ x + 2 < 2

FREE BOOKS FOR JEE & NEET =>(@iitjeeadv)
Continuity of Functions 2.17

For – 2 ≤ x < –1, g(x) = f(0). n


 (x r  x r )2 , x  1

Example 21: Given f (x) = and
Now for x + 1 ≥ 0 and x + 2 < 2



r 1
i.e. –1 ≤ x < 0, g(x) = f(x + 1).
 f ( x  2) , 3  x  2 
g(x) =  n  

 lim f ( x )  2n ) x 2 n  2 (1  x 2 ) for x  1 
 f ( 0) , 2  x  1 
1, for x  1

Hence, g(x) =  show that g(x) is continuous for all x in the domain.
 f ( x  1) , 1  x  0
 1  x , x0 n
 1 
Solution: f (x) =   x 2r  x 2r  2 



Now, we can check that g(x) is continuous in the interval r 1
[–3, 1].
n n
1
Example 19: Consider the function =  x 2 r   x 2 r  2n



 1 1 r 1 r 1
f (x) =x  +
 (1 + x ) (1 + x ) (1 + 2 x ) = (x +x +...+ x2n) +
2 4



1   1 1 1 

(1  2 x ) (1  3 x )
 ....   for x ≠ 0.  2  4  .....  2 n  + 2n
 x x 

x
Find f (0) if f (x) is continuous at x = 0 .
 1 


1 − 2n 
Solution: We have the sum upto n terms as x 2 (1 − x 2 n ) 1  x 
ps
+ 2 + 2n


=
(1  2 x )  (1  x ) (1  3 x )  (1  2 x ) (1 − x 2 ) 1

1 x 1− 2
el
 
1 x (1  x ) (1  2 x ) (1  2 x ) (1  3 x ) x
eh

(
(1  n x )  1  n  1 x ) x 2 (1  x 2 n ) 1  x  2n
je

+ ...... + Thus, f (x) =   2n


(1  n 1 x) (1  n x ) 1 x2 (1  x 2 ) x 2 n
iit
@

2 1 (1  x 2 n )  2 1 
=  upto n terms when x¹ 0. f (x)  x  2 n   2n , x ≠ ± 1
1 x 1 nx 2 



1 x  x 

Now, f(x) = 1 2 x if x  0 and n   . 2n  2 1 



(f(x) – 2n) (1 – x2) = (1  x )  x  2 n 
 x 


Thus, for continuity of f at x = 0, f(0) should be equal to the Now consider
limiting value of f which is 2.
 

g (x) = nlim f ( x )  2n ) x 2 n  2 (1  x 2 ) for x  1
Example 20: Examine the continuity of the function 



sin x
f (x) = lim in (0, π). 2n  2 1  1
n 1 + (2 sin x ) 2 n = lim (1  x )  x  2 n  · 2 n  2 , x  1

n   x  x

sin x
Solution: f (x) = lim (1  x 2 n )( x 2 n  2  1)
n 1 + (2 sin x ) 2 n



= nlim
 ( x 2 n )( x 2 n  2 )

sin x , 0  x   or 5   x  
 6 6  1  1 
 = lim  1  2 n  1  2 n  2 
1  5 n   x  

= 4 , x or x
6 6

  1 if | x | 1
0,   x  5 
 Now g (x) =  undefined if | x | 1
 6 6
 1 if x  1
We can see from the simplified definition of f that it is
π The domain of g(x)is x ∈ (–∞, –1] ∪ [1, ∞).
discontinuous at x = and 5 π .
6 6 Now clearly g is continuous for all x in the domain.

FREE BOOKS FOR JEE & NEET =>(@iitjeeadv)


2.18  
Differential Calculus for JEE Main and Advanced

Example 22: Let f : R → R be a function which satisfies x if xQ




f(x + y3) = f(x) . (f(y))3 ∀ x, y ∈ R. If f is continuous at x = 0, For example, f(x) =  is continuous only at
 x if xQ

prove that f is continuous everywhere.
x = 0. The limit of the function does not exist anywhere except
Solution: To prove lim f(x + h) = f(x). at the point x = 0.


h→0

Put x = y = 0 in the given relation x if x  Q


f(x) =  is continuous only at x = 0.

f(0) = f(0) + (f(0))3 ⇒ f(0) = 0 0 if x  Q

Since f is continuous at x = 0, lim f(h) = f(0) = 0. x if x  Q
h→0
f(x) =  is continuous only at x = 1/2.
Now, lim f(x + h) = lim f(x) + (f(h))3 1x if x  Q
h→0 h→0
= f(x) + lim (f(h))3 = f(x) + 0 = f(x). 
 x 2 if x  Q
f (x) =  is continuous only at x = 1 or – 1.


h→0
1
 if x  Q


Hence f is continuous for all x ∈ R.
Example 23: If f(x + y) = f(x) . f(y) for all x, y ∈ R and Example 24: Find the values of a and b if the function





f(x) = 1 + g(x) . G(x) where lim g(x) = 0 and lim G(x) exists, f(x) = x2 + ax + 1, if x is rational
x→0 x→0
= ax2 + bx + 1, if x is irrational
prove that f(x) is continuous at all x ∈ R.
is continuous at x = 1 and 2.
Solution: lim g(x) = 0 ⇒ lim g(h) = 0 ...(1)
Solution: Continuity at x = 1 implies




x→0 h→0


lim G(x) exists ⇒ lim G(0 + h) = finite ...(2)
ps a+2=a+b+1


x→0 h→0

Now, lim f(x + h) = lim f(x) . f(h) = f(x) lim f(h) which gives b = 1.
el

h→0 h→0 h→0
[∵ f(x + y) = f(x) . f(y)] Continuity at x = 2 implies
eh

= f(x) . lim {1 + g(h) G(h)}, 2a + 5 = 4a + 2b + 1
je

h→0
which gives a = 1.
[using the given relation]
iit



= f(x) . {1 + lim g(h) . lim G(h)} Note: Point functions are treated as continuous. If the
@




h→0 h→0
domain of the function contains a countable number of points
= f(x) . {1 + 0 . finite}, using (1) and (2) then it is continuous at all of these points.
= f(x)

∴ lim f(x + h) = f(x). For example, f(x) = 1  x  x  1 is continuous at x = 1.



h→0
∴ f(x) is continuous everywhere. Here the function is defined only at x = 1.
1
Single Point Continuity Also, f(x) = is defined only for integers. Hence,
{x}  { x}  1
There are some functions which are continuous only at one point
it is continuous at all points in the domain.
in an interval, though they are defined everywhere in the interval.

Concept Problems B

1. At what points are the tangent and cotangent functions 4. For what value of ‘ k ’ is the function ,


continuous?  sin 5 x
2. Where are the following functions discontinuous?  , x0
f (x) =  3 x continuous ?


1  x2  k , x0
(i) sec x (ii)
1 x 2




5. Is the function

3. Prove that the function 2 x  1 if 3  x  2

 x + 1 , −1 ≤ x ≤ 0, 
f(x) =  is discontinuous. f(x) =  x  1 if 2  x  0

 −x , 0 < x ≤1  x  2 if

 0  x 1
continuous everywhere in (–3, 1) ?

FREE BOOKS FOR JEE & NEET =>(@iitjeeadv)
Continuity of Functions 2.19

6. If b and c are given, find all values of a for which 11. Find the numbers at which f is discontinuous. At which
 2 cos x if x  c, of these numbers is f continuous from the right, from the

f(x) =  is continuous. left, or neither ?
ax  b if x  c
2


1  x 2 if x  0

(i) f(x) = 2  x if 0  x  2
7. Find constants a and b so that the given function will be

continuous for all x in the domain. 




( x  2) if x  2
2
 tan ax
 bx if x0
x  2 if x  0
  x
(i) f(x) =  4 if x0 (ii) f(x) = e if 0  x  1


 ax  b



if x0 2  x if x  1
 

12. Choose parameters, entering into the definition of


2 sin(a cos x ) if 1
1  x  0 the function such that the function f(x) becomes

 continuous:
(ii) f(x) =  3 if x0
 21/( x 1) , x  1



 ax  b if x0 (i) f (x)  

 ax  bx  1, x  1


2

8. For what (if any) values of x are the following functions

 x 2  x  1, x   1

discontinous: (ii) f ( x )  
sin ( x  a )), x   1.



(i) y = [x2], (ii) y = [ x ],
ps


13. Locate the discontinuities of the following functions:
(iii) y = ( x − [ x ]) , (iv) y = [2x],

el
1


(v) y = [x] + [–x] ? (i) y = (ii) y=ln(tan2x)
1 + e1/ x
eh



9. (i) For x ≠ 0, let f(x) = [1/x], where [.] denotes the
14. Let f(x) = 2−1/ x for x ≠ 0.
2
je


greatest integer function. Sketch the graph of f over

iit

1 1 (i) Find lim f(x) (ii) Find lim f(x)


the intervals [–2, – ] and [ , 2].




x x→0
5 5
@

What happens to f(x) as x → 0 through positive values? (iii) Is it possible to define f(0) in such a way that f is
and through negative values? continuous for all real x ?
15. Determine the set of all points where the function f(x) =
Can you define f(0) so that f becomes continuous

x3
at 0? is continuous.
4+ | x |
(ii) Answer the same when f(x) = (–1)[1/x] for x ≠ 0.
16. Examine the continuity of the function over x ∈ R.


(iii) Answer the same when f(x) = x(–1)[1/x] for x ≠ 0.


1 x 1


(i) f(x) = tan (ii) f(x) =
10. If f(x) is continuous on (–∞, ∞), then it has no vertical x ( x  1)( x 2  4)




x

asymptotes. True or false. (iii) f(x) = [x] + [–x].

Practice Problems B

 1  cos( 4 x )  1 
 , x0 18. If f(x) = sgn   x    [ln x], 1 < x ≤ 3
 2 

 x2
 3 < x ≤ 3.5
17. Let f(x) =  a, x0 = {x2},



 Find the point where the continuity of f(x) should be

 x checked.
, x0

 (16  x )  4
1/ 2
19. The function f(x) is defined as
then find the value of ‘a’ for which f(x) is continuous at  x 2 cos e1/ x , x  0

f(x) =  .

x = 0.
 x 0

 1,
Show that f(x) is discontinuous at x = 0.

FREE BOOKS FOR JEE & NEET =>(@iitjeeadv)


2.20  
Differential Calculus for JEE Main and Advanced

20. If  the integer nearest to x if x is not




x for rational values of x in [0 , 1]  midway between two conssecutive integers

f(x) =  f(x) =
1  2 x

for irrational values of x in [0 , 1] 


 x − 2 if x is midway between two
1

Show that f(x) is continuous only at x = 1/3. 


consecutive integers


(i) Does lim f(x) exist ? If so, evaluate it.
x 2 − 2x




x3.5
21. Find the points of continuity of f(x) = .
x −1 (ii) Does lim f(x) exist ? If so, evaluate it.




x→3.5
22. If the function f given by
(iii) Is f continuous at 3.5 ?

 tan 2ax (iv) Where is f not continuous ?
 3bx if x  0 26. Test the continuity of the function



x
c if x  0 y  lim .
f(x) =  n  1  (2 cos x ) 2 n


| x |  x 1
3

if 0  x  1 27. If f(x) = x + {– x} + [x] ,where [ ] denotes the greatest
 x x 1
 integer function. and {.} denotes the fractional part
 b if x  1


function. Discuss the continuity of f in [–2 , 2] .
is everywhere continuous, find a, b, and c. 28. Discuss the continuity of the function f(x) = [[x]] – [x – 1],

23. Let f : [0, ∞) → [0, ∞), where [.] denotes the greatest integer function.

29. Find all possible values of a and b so that f (x) is continuous
x + x + x + ..... Is f right continuous at 0?

f(x) = for all x ∈ R if
ps


24. What are the points of discontinuity of the function | ax  3 | , if x  1
el

cos x if x  Q | 3x  a | , if  1  x  0
eh

f(x) =  ? f(x) = .
sin x if x  Q  2b , if 0  x  

b sin 2 x

je

x
25. Let f be the “nearest integer, with rounding down”
cos 2 x  3 , if x  

iit

function. That is,


30. Examine the continuity at x= 0 of the function
@


x x x
f(x) = x + 1 + ( x + 1) (2x + 1) + (2x + 1) (3x + 1) +....∞.

2.3 Classification of Discontinuity lim f(x) = 1 i.e. the one sided limits
the right hand limit is x 
0

exists.
Discontinuity of First and Second Kind
Let a function f be defined in the neighbourhood of a point a,
except perhaps at a itself. Also let both the one-sided limits
lim f(x) and lim f(x) exist, but the conditions of continuity
x a  x a 
are not satisfied. Then the function f(x) is said to have a
discontinuity of the first kind at the point a.
 x
 , x0  x 2  1 for x  0,
1

For instance, the function f(x) = 1  2 x has a The function y=  5 for x  0,



x0  x x  0,
 1, 
for
discontinuity of the first kind at x = 0, since both the one-sided
limits are 0 but this is not equal to f(0) which is 1.
x
The function f(x) = has a discontinuity of the first kind
|x|
at x = 0 since the left hand limit is lim f(x) = –1, while
x0

FREE BOOKS FOR JEE & NEET =>(@iitjeeadv)


Continuity of Functions 2.21

has a discontinuity of the first kind at x = 0 because the one-


sided limits exist.
A function f(x) having a finite number of discontinuities of
first kind in a given interval is called sectionally or piecewise
continuous function.
Sometimes, a function f is discontinuous at x = a because
lim f(x) does not exist as one or both of the one-sided limits
x →a
do not exist. Removable Discontinuity
The function f(x) is said to have discontinuity of the second A function f is said to have a removable discontinuity at x = a,
kind at x = a, if atleast one of the one-sided limits (L.H.L. or if lim f(x) exists but is not equal to f(a). In this case we can



x→a
R.H.L.) at the point x = a does not exist or equals to infinity.
redefine the function such that lim f(x) = f(a) and make it
x→a
Let us show that for the function f(x) =
1
, the point x = 3 continuous at x = a.
3− x
is a point of discontinuity of the second kind. For example, f(x) = [x] + [–x] has a removable discontinuity
at x = 1 since lim f(x) = – 1, but it is not equal to f(1) which
Consider the limits on the left and on the right at the point x→1
is 0.
x = 3:
1 1 Removable discontinuity can be further classified as:
lim = ∞ , lim = –∞
x 3 3  x

x →3 3 − x
+

(i) Missing Point Discontinuity


ps
Thus, the one-sided limits of the function f(x) at the point A function f is said to have a missing point discontinuity
el
x = 3 are infinite, and according to the definition, this means at x = a if lim f(x) exists and the function is undefined at
x→a


eh

that the function f(x) has a point of discontinuity of the second x = a.


kind at this point.
(1 − x )(9 − x 2 )
je

1 For example, f(x) = has a missing point


(1 − x )
iit

The function y = has no one-sided limits at


( x − 2)( x − 3)
discontinuity at x = 1 since lim f(x) = 8 but the function is
@

x = 2 and x = 3. Therefore x = 2 and x = 3 are discontinuities x→1

of the second kind. We can see the graph of the function undefined at x = 1.
below.

(ii) Isolated Point Discontinuity
A function f is said to have an isolated point discontinuity at
The function y = ln |x| at the point x = 0 has the limit x = a if lim f(x) exists and the function is defined at x = a,
x→a

lim ln |x| = –∞. Consequently, lim f(x) (and also the one- but they are unequal.
x→0 x→0

sided limits) does not exist. Hence, x = 0 is a discontinuity of  x2 − 4
 , x ∈ R − {2}
For example, f(x) =  x − 2 has an isolated point
the second kind.
 1 , x=2

It is not true that discontinuities of the second kind only arise
when lim f(x) = ∞. The situation can be different. The function discontinuity at x = 2 since lim f(x)
x→ 2
x →a
exists and is equal to 4, but it does not match with the function's
y = sin (1/x), does not have the one-sided limits as x → 0 since
the values of the function sin (1/x) do not approach a certain value at x = 2, which is 1.
number, but oscillate an infinite number of times within the 0 if x  I
Also the function f (x) = [x]+[–x] = 
interval from –1 to 1 as x → 0. 1 if x  I
It has a discontinuity of the second kind at x = 0. The graph has isolated point discontinuities at all integral x as shown in
is shown below. the figure

FREE BOOKS FOR JEE & NEET =>(@iitjeeadv)
2.22  
Differential Calculus for JEE Main and Advanced

x2  9 =
lim lim (x + 3) = 6,


x 3 
x 3 x3 0

that is, the one-sided limits are equal. Hence, lim f(x) = 6.

x→3
The function f(x) = sgn(cos2x–2sinx+3) However, the point x=3 is a discontinuity in the function since
= sgn( 2(2 + sin x)(1 – sinx) ) the limit is not equal to the function's value.

  Let us see how we can make the function continuous at
 0 if x  2n 
 2 x = 3. Consider the function F(x) defined as
= 
1 if x  2n  


 2  x2  9
π  for x3
has an isolated point discontinuity as x=2nπ + . F(x) =  x  3
2  x 3
 6 for

The values of F(x) coincide with those of the function f(x) =



(x2 – 9) / (x – 3) everywhere except at x = 3, where the “old”
function was defined as 4, while at x = 3 the “new” function
has a value of 6.
Continuous Extension of a Function The function F(x) fulfills the equality
ps
A function F is called a continuous extension of a given function lim F(x) = F(3) = 6,

x→3
f, at a removable discontinuity a of f, if the values of F and f
el
agree at every point of the domain of f and the value of F at a which means that F(x) is continuous at x = 3. Thus, the
is the limit of f at a.
eh

discontinuity of f(x) has been removed.



For example, let f(x) = (x4 – 4)/(x – 2), x ∈ R – {2}. Then
je

Note that we were able to get a continuous function F(x) from



x = 2 is a missing point discontinuity. the discontinuous function f(x) because the one-sided limits
iit

were equal.
@

Irremovable Discontinuity
A function f is said to have an irremovable discontinuity at
x = a, if lim f(x) does not exist. In this case we cannot redefine
x→a
But since lim x  4  lim ( x  2)  4 ,
2
the function such that lim f(x) = f(a) and make it continuous
x→a
x 2 x  2 x 2
at x = a.
we see that x = 2 is a removable discontinuity.
Irremovable discontinuities can be further classified as :
Here F is defined as

 x2 − 4 (i) Finite Discontinuity


 , x ∈ R − {2} A function f is said to have a finite or jump discontinuity at
F(x) =  x − 2


 , x=2
x = a if lim f(x) does not exist since the left hand limit and
 4 x→a
the right hand limit are unequal, but the one-sided limits
It is the continuous extension of f at x = 2. do exist.

 x2 − 9 For instance, f(x) = [x] has a finite or jump discontinuity at

 , x ∈ R − {3} all integral x.
The function f(x) =  x − 3 is defined for all


 4 , x=2
 Jump of a Function at a Point
x ∈ R. At x = 3 it has the following left hand and right hand If x = a is a point of finite discontinuity of the function f(x), then

limits: the graph of this function undergoes a jump at x = a.
x2  9 lim (x + 3) = 6, The difference R.H.L. – L.H.L. i.e. f(a+) – f(a–) is called the
lim = x 3 0
x 3 jump in the function at x = a.

x 3 

FREE BOOKS FOR JEE & NEET =>(@iitjeeadv)


Continuity of Functions 2.23

  
f 0
 2 
 
π
(ii) f(x) = 2tanx at x =


2
  
f
   
The graph of the function f(x) = sgn(x) makes a jump of 2 units 2 
at the point x = 0 since
1 f (0 + ) = ∞
f(0 ) – f(0 ) = 1 – (–1) = 2.
+ – (iii) f(x) = at x = 0
x2


f (0 − ) = ∞

The difference between the greatest and least of the
three numbers f(a+), f(a–), f(a) is the saltus or measure of
 x 2 , x≤0
discontinuity of the function at the point a. (iv) f(x) = 
 ln x , x>0


f (0  ) 
1 2
(i) lim tan 1   ; jump = π has an infinite discontinuity at x = 0 since the right hand
x 


f (0  )   limit is infinite (note that the left hand limit is 0.
x0


2
f ( )  1 Pole Discontinuities
| sin x | The concept of pole discontinuity is related with infinite limit.
(ii) lim ; jump = –1


x→ π x − π

 For a point x = a to qualify as a pole of a function f, we must
f ( )  0
1
ps
have lim =0.
f (2+ ) = 1 x →a f ( x )
el
1 What this means roughly is that "f(x) becomes numerically big
(iii) lim [ x ]
eh

; jump =
and stays big as x gets close to a".

x − 1 2
x→2
f (2 ) =
je

2
1
has pole at x = 2 and x = 3.
iit

1 The function y =
(iv) f ( x )  and f(0) = e . (x − 2)(x − 3)
1  e 1/ x

@

 x 2 , x≤0
Note that f(x) = 

f (0 )  lim
1
 lim
1 ln x , x>0
h 0 1  e 1/(0  h ) h 0 1  e 1/ h

does not have a pole discontinuity at x = 0, even if it has
an infinite discontinuity at x = 0 since the left hand limit
1 1

  1 is finite.
1  e  1 0

Again, the reciprocal of a polynomial of degree n has atmost
1 1 n poles : in fact, the real zeros of a polynomial are the poles

and f (0 )  lim 1/( h  h )
 lim of its reciprocal.
h 0 1 e h 0 1  e1/ h


In particular, the reciprocal of the quadratic polynomial
1 1 with negative discriminant has no poles. The poles of a
 
  0. rational function in "lowest terms" are the zeros of the
1 e 1 

denominator.
Hence f is discontinuous at x = 0, the discontinuity being of
The zeros of sin x and cos x are respectively the poles of
the first kind and irremovable. This function has a jump of one
cosec x and sec x.
unit at 0 since f(0+) – f(0–) = 1. The saltus is e.

Consider the following graph to understand the nature of
(ii) Infinite Discontinuity discontinuity.
A function is said to have an infinite discontinuity at
x = a if atleast one of the one-sided limits is infinite. For

instance, the following functions have infinite discontinuity:
f (1 )   
x
(i) f(x) = at x = 1

1− x
f (1 )  

FREE BOOKS FOR JEE & NEET =>(@iitjeeadv)


2.24  
Differential Calculus for JEE Main and Advanced

From the above graph note that lim f(x) = lim π[x] – 1 = π – 1.






(i) f is continuous at x = – 1 x x


2 2
(ii) f has non removable (finite type) discontinuity at x= 0



(iii) f has isolated discontinuity at x = 1 The jump of discontinuity = R.H.L. – L.H.L.


π π


(iv) f has missing point discontinuity at x = 2 =π–1– = – 1.



(v) at x = 3 non removable infinite type discontinuity. 2 2


2 cos x  sin 2 x
(iii) Oscillatory Discontinuity Example 3: Let f (x) = ,
(  2x )2




A function is said to have an oscillatory discontinuity at
x = a if atleast one of the one-sided limits does not exist because e  cos x  1 f ( x ) , x   / 2
of too much oscillation in the values of the function. g (x) = , and h (x) = 
1 8x  4 g ( x ) , x   / 2
For example, f(x) = sin
x
has an oscillatory discontinuity then show that h has an irremovable discontinuity at x = π/2.
at x = 0 .
 1   2 cos x  sin 2 x 
The function f(x) = 1 + sin(ln | x |)  oscillates between 0  , x
 (  2x )
2
2
 3 
and 1 at x = 0 and hence has a oscillatory discontinuity at x = 0. Solution: h (x) = 
  cos x



e 1 
Note: In all the cases of irremovable discontinuity the  8x  4 , x


2
value of the function at x = a (point of discontinuity) i.e. f(a)
may or maynot be defined. L.H.L. at x = π/2

 x 1 2 sin h  sin 2h
 x,
Example 1: If f(x) = 
ps = lim
4h 2

h0

2
x 1


x ,
el
then find the type of discontinuity at x = 1. 2 sin h (1  cos h )
= lim = 0.
eh

4h 2

h0
 x , x 1
Solution: f(x) =  2 esin h  1
je

 x , x 1 R.H.L. = hlim


  (  2)  h   4 
iit

0

lim f(x) = lim x = 1

@

x→1− x→1−
esin h  1 sin h 1
= lim · = .
lim f(x) = lim x2 = 1 h 0

and 8h sin h 8

x1 x1
      1
∴ lim f(x) = lim f(x) = 1 Thus, h   = 0 and h   =

x1 x1 2  2  8
but f(1) is not defined.      

So, f(x) has a missing point removable discontinuity at x = 1. We have h   ≠ h   .
The discontinuity is of first kind. 2  2 
π Since L.H.L. ≠ R.H.L., h(x) has an irremovable discontinuity
Example 2: Let f(x) = cos–1 {cot x}, x <
2 at x = π/2.


= π[x] – 1,  Example 4: State the number of point of discontinuity
x


and discuss the nature of discontinuity


2
π 1
Find jump of discontinuity at x = . for the function f (x) = and also sketch its graph.
ln | x |

2
 −1 π  1
cos {cot x} if x<  ln x if x  0, x  1
2 
Solution: f(x) =  Solution: f (x) = 
π  1


 π[x] − 1


if x≥ if x  0, x  1
 2 
 ln ( x )
lim f(x) = lim cos–1 {cot x} The function is obviously discontinuous at x = 0, 1, –1. as it


 


x
2
x
2
is not defined.
π lim f ( x )  0 
= cos –1 0 = . x 0 
 Limit exists at x = 0.
lim f ( x )  0 

2

x 0 


FREE BOOKS FOR JEE & NEET =>(@iitjeeadv)


Continuity of Functions 2.25

Hence there is a removable discontinuity (missing point) at x x


x = 0. ∴ f (x) = lim  .
11 n


n 2
lim f ( x )   
x 1 
 Limit dne. π 5π
lim f ( x )   

x 1 
 For <x< , 4 sin2x > 1
6 6
Hence there is a nonremovable discontinuity (infinite type)
at x = 1. ∴ f (x) = lim x
 0 .



n
1  (greater than 1) n
 
 
lim f ( x )   
x 1  π 5π
lim f ( x )   
 Limit dne. For 0 ≤ x < or < x < π , 4 sin2x < 1


6 6
x 1 

x
Hence there is a non removable discontinuity (infinite type) ∴ f (x) = lim  x .
1  (less than 1) n


n
at x = –1.
 
 
Note that f (x) is even ⇒ the graph is symmetric about y axis. x for 0  x 

or
5
 x 
The graph of f (x) is as follows.  6 6


Y  x  5
Hence f (x) =  2 for x  or
6 6


 5
X  0 for  x 
6 6
ps
1 
, then find f  
el
Example 5: Let f(x) = lim
1 + n sin 2 x 4


n
eh

and also comment on the continuity at x = 0.


je

1
lim
iit

Solution: Let f(x) = n 1 + n sin 2 x




@

 1
f   = lim
 4  n 

1  n . sin 2
4
1
lim
= n 1 =0
1 n  

2
The graph of f(x) is as shown above.
From the graph it is clear that f (x) is continuous everywhere
Now, π
in (0,π) except at and 5π and where it has removable

1 1
f(0) = lim = =1 6 6
n n . sin 2 (0) + 1 1+ 0
discontinuity of finite type.
 1   1  }= Example 7: Consider f(x)
lim f(x) = lim  lim  {form  0


 n  1  n 
 1  
2
sin x 

x→0 x→0  ba
1 a sin 2 x
2
Here sin x is a very small quantity but not zero. a  b tan 2 x
= ba  
2 ,
∴ ba

Thus f(x) is discontinuous at x = 0. The type of discon- 1  a sin x 

tinuity is isolated point removable discontinuity.  

Example 6: Examine the function for b > a > 0 and g (x) = sgn (f(x)). Find whether g (x) is


x continuous at x = 0 or not and state the nature of discontinuity,
f(x) = lim 2 n
for continuity in (0, π). Plot its graph
n  if discontinuous.
1  (4 sin x )
and state the nature of discontinuity. 1 b  a ·sin 2 x· a  b tan 2 x
·
π 5π Solution: f(x) = ba a  (b  a ) sin 2 x


Solution: For x = or , 4sin2x = 1 a·
6


6 a

FREE BOOKS FOR JEE & NEET =>(@iitjeeadv)


2.26  
Differential Calculus for JEE Main and Advanced

Example 8: Let
sin 2 x · a + b tan 2 x



=  ex  1
 if x  0

a cos x + b sin x
2 2
 (1  x 2 )  1  x 2

f(x) =  ,
sin 2 x · a cos 2 x + b sin 2 x 


=  2
if x  0

| cos x | a cos 2 x + b sin 2 x 
 3
sin 2x then find whether f(x) is continuous at x = 0 or not and state
Hence, f (x) = the nature of discontinuity, if discontinuous.
| cos x |
 sin 2x  Solution:
Now, g (x) = sgn  



 | cos x |   ex  1 
 x  · x· (1  x )  1  x
2 2

We have g (0) = 0 Limit = lim  



x 0
 sin 2h  (1  x 2 )  1  x 2 · (1  x 2 )  1  x 2
g (0 ) = lim  lim sgn  2 sin h  = 1.
 = h
+
h0  cos h  0

2·x 2x
lim
= x lim
= x

0
(1  x )  (1  x )
2 2 2 0
x  3x 2
4
 2 sin 2h 
g (0 ) = lim sgn 
– lim sgn  2 sin h  = – 1
 = h 0
h0  cos h  Now R.H.L. =
2
and L.H.L. = –
2


3 3
Hence, g (x) = sgn (f(x)) is discontinuous at x = 0. The nature
ps
of discontinuity is irremovable discontinuity of finite type. Hence, f(x) is discontinuous at x = 0. The nature of discontinuity
el
is irremovable discontinuity of finite type.
eh

Concept Problems C
je

iit

1. Let = x, x < 1 f(x) 6. Test the discontinuity of the following function at x = a


@





= x2, x > 1 and specify the type of discontinuity.



= 2, x = 1 1 1
f (x)  cos ec , x ≠ a and f(a) = 0.




x a xa

Find the type of discontinuity at x = 1.


2. Let f(x) = x, x < 1 1
7. The function f(x) = tan –1 is meaningless for x = 2.




= 2x, x ≥ 1 x−2

Is it possible to define the value of f(x) in such a way that



Find the type of discontinuity at x = 1.
the redefined function becomes continuous at x = 2 ?


3. Prove that the function h(x) = 2 has infinite discontinuity tanx

at x = π/2. 8. Which of the following functions f has a removable

discontinuity at a? If the discontinuity is removable, find
x+2
4. If f(x) = x + find the points of discontinuity and a function g that agrees with f for x ≠ a and is continuous
|x+2|

on R.
determine the jumps of the function at these points. x 2 − 2x − 8
(i) f(x) = , a = −2
x+2


1 1
5. If f ( x )  sin find f(a+) and f(a–). Is the function
xa xa x 7

(ii) f(x) = ,a7
| x 7|


continuous at x = a?

Practice Problems C

9. Test the function f(x) for continuity and indicate the kind (iii) f(x) = {x},



of discontinuity, if any. 
x 2 if x is an irrational number

(i) f(x) = x sin (1/x) (iv) f(x) = 

1


if x is a rational number


(ii) f(x) = sin(1/x),


FREE BOOKS FOR JEE & NEET =>(@iitjeeadv)
Continuity of Functions 2.27

11. Can f(x) = x(x2 – 1)/|x2 – 1| be extended to be continuous


x2 + 2


(v) f(x) = , at x = 1 or –1?
x3 + 1


(vi) f(x) = tan–1(1/x), 12. Show that the only discontinuities a rational function


can have are either removable or infinite. That is, if


1
(vii) f(x) = x /(1− x ) ,
r(x) is a rational function that is not continuous at
1− e


a, show that either a is a removable discontinuity or
(viii) f(x)=ln
x2 lim |r(x)| = ∞.
x →a
( x  1)( x  3)

13. Let f(x) = lim (1 + x)n. Comment on the continuity of
 x 2 for 0  x  1,


n
f(x) at 0.
(ix) f(x) = 


2  x for 1  x  2,


14. Investigate the following functions for continuity:


cos(x/2) , |x|  1, 7 x 3
(x) f(x) =  (i) y



|x-1| , |x|  1 x2  4


10. What is the nature of the discontinuities at x = 0 of the 1

functions: (ii) y = (1 + x) tan –1
1− x2



sin x
(i) y= (ii) y = [x] + [–x] 1




x
(iii) y = 1
1


(iii) y = cosec x (iv) y = x, 1  e1 x
 




15. Investigate the following functions for continuity:
ps

1 1 1
(v) y = 3
x (vi) y = cosec
(i) y = lim (x ≥ 0)
 
el




x n  1  x n



eh

sin(1 / x )
(vii) y = ? (ii) y = nlim ( x tan 1 nx )
sin(1 / x ) 




je
iit
@

lim h(x) = lim [f(x) + g(x)]


2.4 Algebra of Continuous Functions x →a x →a


It is easily deduced from the theorems on limits that the sum, = lim f(x) + lim g(x) = f(a) + g(a) = h(a).
x →a x →a
product, difference or quotient of two functions which are Thus, the sum h(x) = f(x) + g(x) is a continuous function.
continuous at a certain point are themselves continuous at that Note, as a corollary, that the theorem holds true for any finite
point (except that, in the case of the quotient, the denominator number of terms.
must not vanish at the point in question).
Theorem The sum of a finite number of functions continuous

Further it is true that composition of a continuous function with at a point is a continuous function at the point.
a continuous funcion is a continuous function.
Proof Suppose we are given a definite number of functions

1. If f(x) and g(x) are continuous at x = a, then the following f1, f2, ...,fn continuous at a point x = a. We have to prove that

functions are also continuous at x = a. their sum g = f1+ f2 + ... + fn is a continuous function at that
(i) cf(x) is continuous at x = a, where c is any constant. point. The functions f1, f2, ...,fn being continuous, we have


(ii) f(x) ± g(x) is continuous at x = a.
lim f1 ( x )  f1 (a ), lim f 2 ( x )  f 2 (a ), .......


(iii) f(x). g(x) is continuous at x = a. x a x a



(iv) f(x)/g(x) is continuous at x = a, provided lim f n ( x )  f n (a ),


g(a) ≠ 0. x a

Theorem If the functions f(x) and g(x) are continuous at a By the theorem on the limit of a sum we write

point x = a, then the sum h(x) = f(x) + g(x) is also continuous lim g ( x )  lim (f1  f 2  ...  f n )
x a x a

at the point x = a.
Proof Since f(x) and g(x) are continuous, we can write = xlim f1 ( x )  lim f 2 ( x )  .....  lim f n ( x )
a x a x a


lim f(x) = f(a) and lim g(x) = g(a) = f1(a) + f2(a) + .... + fn(a) = g(a).
x →a


x →a

By theorem on limits, we can write Thus, lim g ( x )  g (a ) which is what we wished to prove.
x a

FREE BOOKS FOR JEE & NEET =>(@iitjeeadv)


2.28  
Differential Calculus for JEE Main and Advanced

By the above theorem we at once recognize that the functions Here f(x) is continuous at x = 0 and g(x) is


y = x2 + 3x + √x, discontinuous at x = 0. But the product function
y = sin3x – x sin x – (x4 – 1) cos x, and x3 , x0


y = sin x – 2x/(x2 + 1) are continuous at every point in a f(x)g(x) = 0, x  0 is continuous at x= 0


domain common to all functions involved.  3
 x , x0
We can show that a polynomial is a continuous function.The
As another example, the product of the functions
polynomial function P : R → R is given by P(x) = a0 + a1x +


.... + anxn, where a0, a1,...., an ∈ R. The functions f1, f2, ...., fn :  1
sin x0
R→ R defined by f(x) = x and g ( x )   x


f1(x) = x, f2(x) = x2, ..., fn(x) = xn are continuous.  0 x0
Hence, the functions a1f1, a2f2, ..., anfn are also continuous is continuous at x = 0 even when f(x) is continuous


functions. Therefore, the function at x = 0 and g(x) is discontinuous at x = 0.
P(x) = a0 + a1f1 + .... + anfn However, the product of the functions f(x) = x and


is also continuous as the sum of continuous functions is a g(x) = [x] is discontinuous at x = 1.
continuous function. (iii) f(x)/g(x) is not necessarily discontinuous at x = a.



Theorem The product of a finite number of functions Here also we need to work on the function f(x)/g(x)

continuous at a point is a continuous function at that point. to get the result.
Proof Let g = f1. f2 ..... fn retaining the notation of the above  x  1, x  0
Let f(x) = x(x2 –1) and g(x) =  .

theorem and using the theorem on the limit of a product, we get  x  1, x  0

lim g ( x )  lim (f1. f 2 .... f n ) ps Here f(x) is continuous at x = 0 and g(x) is
x a x →a


discontinuous at x = 0. We can check that f(x)/g(x)
= xlim f1 ( x ). lim f 2 ( x ). ..... . lim f n ( x )
el
→a x →a x →a is continuous at x = 0.

eh

= f1(a) . f2(a) ...... fn(a) = g(a) Example 1: Discuss the continuity of f(x) = [x] + | x – 1 |.




which is what we set out to prove. Solution: Let us draw the graphs of the functions y = [x]
je


Theorem The quotient of two functions continuous at a point and y = | x – 1 |
iit

x = a is a continuous function at the point x = a provided that the Y
denominator does not turn into zero at the point.
@

f1 y = |x – 1|
Proof If g = , then by the theorem on the limit of a
f2

1 y = [x]
quotient, we have
lim f1 ( x )
f1 ( x ) x →a f1 (a ) –1
lim g ( x )  lim = lim f ( x ) = = g (a ) O 1
X
x a x →a f 2 ( x ) f 2 (a ) 2

2
x →a

f1 –1
if lim f2(x) = f2(a) ≠ 0. Therefore g = is a continuous function
x →a f2 It is a clear from the figure that f(x) = [x] is discontinuous at
at the point x = a. all integral points and g(x) = |x – 1| is continuous for all x ∈ R.
2. If f(x) is continuous at x = a and g(x) is discontinuous at The sum of a discontinuous and a continuous function is

x = a, then we have the following results. discontinuous. Hence f(x) + g(x) is discontinuous at all integral
(i) Both the functions f(x) + g(x) and f(x) – g(x) are points.


discontinuous at x = a. 3. If f(x) and g(x) both are discontinuous at x = a, then we

For example, consider, f(x) = x and g(x) = {x}. have the following results.

Here f(x) is continuous at x = 0 and g(x) is (i) The functions f(x) + g(x) and f(x) – g(x) are not



discontinuous at x = 0. Both the sum function x + {x} necessarily discontinuous at x = a. However, atmost
and the difference function x – {x} are discontinuous one of f(x) + g(x) or f(x) – g(x) can be continuous
at x = 0. at x = a. That is, both of them cannot be continuous
(ii) f(x). g(x) is not necessarily discontinuous at x = a. simultaneously at x = a. We have the following reason.


We need to find the result by getting the limit of the Let us assume that both f(x) + g(x) and f(x) – g(x) are

product f(x). g(x) and comparing it with f(a). g(a). continuous. Then the sum of functions (f(x) + g(x))
For example, consider, f(x) = x3 and g(x) = sgn(x). + (f(x) – g(x)) = 2f(x) must be continuous at x = a,

FREE BOOKS FOR JEE & NEET =>(@iitjeeadv)
Continuity of Functions 2.29

which is wrong as it is given that f(x) is discontinuous Continuity of Composite Functions


at x = a. Hence our assumption is wrong. So, both Theorem A function composed of a finite number of


the functions cannot be continuous simultaneously continuous functions is a continuous function.
at x = a.
It is sufficient to prove this assertion for a composite function
For example, consider, f(x) = [x] and g(x) = {x}. formed by two continuous functions because after that it can be

Here both f(x) and g(x) are discontinuous at x=0 extended, consecutively, to an arbitary number of constituent

The sum function [x] + {x} being equal to x is functions.

continuous at x = 0. The difference function [x] – {x} Theorem If f(x) is continuous at x = a and g(x) is continuous


however is discontinuous at x = 0. at x = f(a) then the composite function (gof)(x) is continuous
But this does not mean that one of the functions f(x) + at x = a.


g(x) or f(x) – g(x) must be continuous. We can have Proof Let y = g(u), u = f(x) and y = g(f(x)) = F(x)


both the functions discontinuous. For example, where f(x) is continuous for x = a and g(u) is continuous for
if f(x) = 2[x] and g(x) = {x}, then both the functions u = b = f(a). We have to prove that

f(x) + g(x) or f(x) – g(x) are discontinuous y = F(x)) is continuous at the point a.
simultaneously at x = 0 .
Indeed, let x → a. The continuity of the function
(ii) f(x) . g(x) is not necessarily discontinuous at x = a. u = f(x) implies that lim f(x) = f(a) = b,


We need to find the result by getting the limit of the x →a
product f(x). g(x) and comparing it with f(a). g(a). that is u → b. The function g(u) being continuous at the point b.
For example, consider, f(x) = [x] and g(x) = [–x]. we have lim g(u) = g(b).
u →b

Here both f(x) and g(x) are discontinuous at x = 0
ps

Now, since u = f(x), we can rewrite the last relation in the form
but, the product function [x].[–x] is continuous at
el
x = 0. lim g(f(x)) = g(f(b)) or, equivalently,
eh

Further, f(x) = [x] and g(x) = {x} are both x →a



discontinuous at x = 0 and, the product function [x]. lim F(x) = F(a) which is what we wished to prove.
je

{x} is discontinuous at x = 0. x →a
iit

Hence, we cannot comment in advance, when both This theorem is also named as the chain rule for continuity.
π

the functions are discontinuous at x = a.
@

For example, f(x) = sin x is continuous at x = and


(iii) f(x)/g(x) is not necessarily discontinuous at x = a. Here 2

 x 2  1, x 1 π


also we need to work on the function f(x)/g(x) to get g(x) =  is continuous at x = f( ) = 1. Hence
the result.  x  1,
 x 1 2
π
 x 2 −1,
 x≥0  x +1, x ≥ 0 the composite function (gof)(x) is continuous at x = .
Let f(x) =  x +1, x<0
and g(x) =  x − 1, x < 0 . 2

  Note:



Here both f(x) and g(x) are discontinuous at x = 0. 1. Let a function f(x) be continuous at all points in the interval

But we find that f(x)/g(x) is continuous at x = 0.

[a, b], and let its range be the interval [A, B] and further
Example 2: If f(x) = [sin(x–1)] – {sin(x–1)} then the function g(x) be continuous in the interval [A, B],
π


then the composite function (gof)(x) is continuous in the
comment on continuity of f(x) at x = + 1.
interval [a, b].

2
2. If the function f is continuous everywhere and the function
Solution: f(x) = [sin (x – 1)] – {sin (x – 1)}Let g(x) =

g is continuous everywhere, then the composition gof is


[sin (x – 1)] + {sin (x – 1} = sin (x – 1)
π continuous everywhere.
which is obviously continuous at x = +1 3. All polynomials, trigonometric functions, inverse
2

Here [sin (x – 1)] and { sin (x – 1)} are both discontinuous at trigonometric functions, exponential and logarithmic
π functions are continuous at all points in their domains.
x= + 1. 4. If f(x) is continuous, then | f(x) | is also continuous.
2

π x sin x
∴ Atmost one of f(x) or g(x) can be continuous at x = + 1. For example, f(x) = and g(x) = x are continuous
x2 + 2

2
π for all x. Hence, the composite function (gof)(x) =
As g(x) is continuous at x = + 1, therefore, f(x) must be
2 x sin x
is also continuous for all x.
discontinuous. x2 + 2

FREE BOOKS FOR JEE & NEET =>(@iitjeeadv)


2.30  
Differential Calculus for JEE Main and Advanced

Example 3: Check the continuity of cosine are continuous at every point, while root functions are


continuous on their proper domains.
x 3 cos x + x 2 sin x
f(x) = . Example 5: Show that the function
cos(1 / sin x )



2/3
Solution: The numerator is continuous for all x. As far  x7 
f(x) =   is continuous everywhere.


as the denominator is concerned, according to the theorem on  x 2  2x  2 
continuity of a composite function, it is continuous at points
where the function u = 1/sin x is continuous, since the function Solution: Note first that the denominator



cos u is continuous everywhere. x2 + 2x + 2 = (x + 1)2 + 1is never zero.


Hence the rational function
Hence the denominator is continuous everywhere, except at
x 7
the point x = kπ (k an integer). Besides, we must exclude r(x) = 2
x  2x  2


the points at which cos (1/sin x) = 0. i.e. the points at which
1/sinx = (2p + 1) π/2 (p ∈ I), or sin x = 2/[2p + 1)π]. Thus, is defined and continuous everywhere. It then follows from
the function f(x) is continuous everywhere except at the points the continuity of the cube root function that f(x) = [r(x)]2/3 =
2 3
[r ( x )]2 is continuous everywhere.
x = kπ and x = (–1) sin (2p  1)  n , n –1
As was mentioned earlier, all the basic elementary functions
where k, p, n ∈ I. are continuous in the intervals where they are defined
and therefore the theorems proved here imply that every
Example 4: What can you say about the continuity of the elementary function is continuous in those intervals where


function f(x) = 9 − x2 ? it is defined.
ps
An elementary function can only be discontinuous at those
Solution: Because the natural domain of this function is the
el
points where some of the constituent functions it is formed of


closed interval [–3, 3], we will need to investigate the continuity
are not defined or where the denominators of some fractions
eh

of f on the open interval (–3, 3) and at the two endpoints. If c is any


involved vanish.
je

x
number in the interval (–3, 3), then lim f ( x ) = lim 9 − x 2 For instance, the function y = is discontinuous at the
x2 − 4

x →c x →c
iit

= lim(9 − x 2 ) = 9 − c 2 = f (c) points x = ± 2 and continuous at all the other points.


@

x →c

The function y = x 2 tan x is discontinuous at the points
π
which proves f is continuous at each number in the interval x = (2k + 1) , k ∈ I.
(–3, 3). The function f is also continuous at the endpoints since 2

Example 6: Find the points of discontinuity of


lim f ( x ) = lim 9 − x2 = lim (9 − x 2 ) = 0 = f (3) ;


x →3− x →3− x →3−
1 1
y= , where u = .
lim f ( x ) = lim 9−x u u2
2
x −1

2
x →−3+ x →−3+

1
Solution: The function u = f(x) = is discontinuous
= lim (9 − x ) = 0 = f ( −3) . x −1


2
x →−3+

at the point x = 1. ...(1)

Thus, f is continuous on the closed interval [–3, 3]. 1 1
The function y = g(u) =  is
Note: The nth-root function f(x) = n
x is continuous u2  u  2 (u  2)(u  1)


everywhere if n is odd and it is continuous for x ≥ 0 if n is even. discontinuous at u = –2 and u = 1.
We may combine this result with the previous theorem. Then we 1
see that a root of a continuous function is continuous, wherever When u = –2, = –2
x −1
it is defined. That is, the composition
1
h(x) = n g ( x ) = [g(x)]1/n ⇒ x–1=– ⇒ x = 1/2 ...(2)
2





n
of f(x) = x and the function g(x) is continuous at a assuming 1
that g(a) ≥ 0 if n is even (so that n g (a ) is defined). When u = 1,
x −1
=1
For example, the above theorem tells us that composite ⇒ x–1=1 ⇒ x=2 ...(3)




functions like sin x , x , ( x + 1) , cos x + 3 , 3 2 Hence, the composite function y = g(f(x)) is discontinuous at
are continuous at all points at which the functions are defined, three points x =
1
, 1, 2.
because polynomials and trigonometric functions like sine and 2

FREE BOOKS FOR JEE & NEET =>(@iitjeeadv)


Continuity of Functions 2.31

x 1 1 1 x −1
Example 7: If f(x) = and g(x) = , then discuss u = f(f(x)) = 1 − 1 / (1 − x ) = .
x 1 x−2 x




the continuity of f(x), g(x) and (fog)(x). Hence, the point x = 0 is a discontinuity of the function
x 1 u = f(f(x)). If x ≠ 0, x ≠ 1, then
Solution: f(x) =
x 1


1
f(x) is a rational function it must be continuous in its domain. y = f(f(f(x))) = = x is continuous everywhere.
1 − ( x − 1) / x
f is not defined at x = 1
∴ f is discontinuous at x = 1 Thus, the points of discontinuity of this composite function are

1 x = 0, x = 1, both of them being removable.
g(x) =
x−2 Example 9: If f(x) = sgn(2sinx + a) is continuous for all



g(x) is also a rational function. It must be continuous in its x, then find the possible values of a.
domain and it is not defined at x = 2
Solution: Since f(x) = sgn(2sinx + a) is continuous for
∴ g is discontinuous at x = 2



all x, we should have 2sinx + a ≠ 0 for any real x.

Now fog(x) may be discontinuous at
(i) x = 2 (point of discontinuity of g(x))
⇒ sin x ≠ –a/2


⇒ |a/2| > 1

(ii) g(x) = 1 (when g(x) = point of discontinuity of f(x)).


⇒ a < –2 or a > 2.

1

If g(x) = 1 then =1 ⇒ x = 3.
x−2


Continuity of the Inverse of a Continuous
∴ The discontinuity of fog(x) should be checked at x = 2 Invertible Function

and x = 3
ps
1 Theorem If the function y = f(x) is defined, continuous
1

el
x2 and strictly monotonic on the interval I, then there exist the
At x = 2, fog (x) = 1 inverse function y = f–1(x) defined, continuous and also strictly
eh

1 monotonic in the range of the function y = f(x).
x2
je

We see that fog (2) is not defined. Theorem Assume f is strictly increasing and continuous

iit

1 on an interval [a, b]. Let c = f(a) and d = f(b) and let g be the
1 1 x  2 inverse of f. Then
lim fog (x) = lim x  2
@

= lim = 1. (i) g is strictly increasing on [c, d], and


x→2 x→2 1 x→2 1  x  2


1 (ii) g is continuous on [c, d]
x2


There is a coresponding theorem for decreasing functions. That
∴ fog (x) is discontinuous at x = 2 and it has a removable
is, the inverse of a strictly decreasing continuous function f is

discontinuity at x = 2.
strictly decreasing and continuous.
At x = 3, fog (3) is not defined.
3
1 For example, we prove that the function y = x is continuous
1
lim fog (x) = lim x  2 =∞ ∀ x ∈ R, considering it as the inverse of y = x3. The function

y = x3 is continuous ∀ x ∈ R, and its range is y ∈ R. Also, it
 
x3 x3 1
1
x2 is strictly increasing and hence invertible. Hence, its inverse
1 function,
1
lim fog (x) = lim x2 =–∞ y= 3
x , x ∈ R, is continuous ∀ x ∈ R.

x3 x3

1
1 Continuity of Integrals
x2
Assume f is integrable on [a, x] for every x in [a, b] and let
∴ fog (x) is discontinuous at x = 3 and it has an infinite x
A(x) = ∫ f ( t )dt . Then the integral A is continuous at each point

irremovable discontinuity there.
a
Example 8: Given the function f(x) = 1/(1 – x). of [a, b]. (At each endpoint we have one-sided continuity)


Find the points of discontinuity of the composite function
y = f(f(f(x))). Proof We choose a point c in [a, b]. Now we prove that

A(x) → A(c) as x → c. We have
Solution: The point x = 1 is a discontinuity of the function


x

∫ f (t )dt
1
v = f(x) = . If x ≠ 1, then A(x) – A(c) =

1− x
c

FREE BOOKS FOR JEE & NEET =>(@iitjeeadv)


2.32  
Differential Calculus for JEE Main and Advanced

We estimate the size of this integral. Since f is bounded on the theorem. If c is an endpoint of [a, b], we must let x → c
[a, b], there is a constant M > 0 such that –M(x – c) ≤ A(x) – from inside the interval, so the limits are one-sided.
A(c) ≤ M(x – c). Note: If f is continuous on [a, b], then it is integrable



If x < c, we obtain the same inequalities with x – c replaced by on the interval. Hence, the integral of a continuous function
c – x. Therefore, in either case we can let x → c and apply is continuous. For example, ∫1
x sint
dt is continuous in the
the Sandwitch theorem to find that A(x) → A(c). This proves interval [1, ∞).
t

Concept Problems D

(ii) Both functions f(x) and g(x) are discontinuous at
1 x0



x = x0?
1. Let f(x) =  and
1 x  0

4. Prove that if the function f(x) is continuous and non-


1 x  0 negative in the interval (a, b), then the function F(x) = f ( x )
g(x) =  is likewise continuous in this interval.
1 x0

Show that f + g is continuous at x = 0 even though f and 5. Prove that f(x) = 1 / x 4 + 7 x 2 + 1 is continuous


everywhere.
g are both discontinuous there.
6. Suppose that the function f is continuous everywhere and
2. Will the sum of two functions f(x) + g(x) be necessarily

that the composition f(g(x)) is continuous at x = a. Does

discontinuous at a given point x0 if :
ps it follow that g(x) is continuous at a ?
(i) The function f(x) is continuous and the function g(x)
7. Can one assert that the square of a discontinuous function
el



is discontinuous at x = x0, is also a discontinuous function? Give an example of
eh

(ii) Both functions are discontinuous and the function at a function discontinuous everywhere whose square is a


x = x0? continuous function.
je

3. Is the product of the two functions f(x) g(x) necessarily 8. Give an example of a function f such that f is not continuous
iit


discontinuous at a given point x0 if : but |f| is continuous. Show that f 2 can be continuous when
f is not.
@

(i) The function f(x) is continuous and the function g(x)




is discontinuous at this point;

Practice Problems D

x≠0 12. Let f and g be two functions defined as follows:
 x if

9. Let f(x) =  and 
x=0 1  x x0
3
,
 2 if

f (x)   ;


x  1 , x0
2
x≠0 
3x if
g(x) =   1
 −2 if x=0

 ( x  1) 3 , x0
g(x )  
Show that f + g is continuous at x = 0 even though f and 
1

( x  1) x0
g are both disontinuous there. 2 ,
10. Let f(x) = x2 and Comment the continuity of gof(x)


 cos 
x x0 13. Let f and g be two functions defined as follows :
g(x) = 

x0 1 if | x |  1

0 2  x 2
 if | x |  2
Show that f.g is continuous at x = 0. f(x) = 0 if | x |  1 , g(x) = 
if | x |  2
 2



11. Let f and g be two functions defined as follows: Find a formula for the composite function h(x) = f[g(x)].


x+ | x |  x for x  0
 For what values of x is h continuous?
f(x) = for all x, g(x) =  2

x for x  0

2 14. Discuss the continuity of the composite function h(x) =

Find a formula for the composite function h(x) = f[g(x)]. 1
f(g(x)) where f(x) = , g(x) = x2 + 5.

For what values of x is h continuous? ( x − 6)

FREE BOOKS FOR JEE & NEET =>(@iitjeeadv)


Continuity of Functions 2.33

1 17. If f(x) = – 1 + | x – 1|, 1  x  3 and g(x) = 2 – | x + 1|,


15. If f(x) = . Find the points of discontinuity of the
1− x 2  x  2 , then discuss the continuity of f(g(x)).

function y = f (f(f(x))).
 x  1,
 1  x  0
1  x , 0  x  2 18. Let f(x) =  and g(x) = sin x.
16. Let f(x) =  2
 x , 0  x 1


. 
3  x , 2  x  3

Further let h(x) = f ( | g(x) | ) + | f (g(x))|. Discuss the
Determine the form of g(x) = f(f(x)) and hence find the


continuity of h(x) in [–1, 1].

point of discontinuity of g , if any.

Weierstrass Theorem (Extreme Value Theorem)


2.5 Properties of Functions

If a function is continuous in a closed interval there exists
Continuous on a Closed Interval atleast one point at which the function assumes the greatest
Functions continuous on a closed interval possess d in the value and atleast one point at which it assumes the least value
following theorems. on that interval.
If a function f is continuous on a closed interval [a, b],
Boundedness Theorem then there exist its minimum and maximum values on [a, b],
If a function f(x) is continuous at every point of a closed interval i.e. there exist points α, β, ∈ [a, b] such that f(α) ≤ f (x) ≤ f(β)
[a, b], then the function f(x) is bounded on this interval. for all x ∈ [a, b]. In other words,
ps
Note that the boundedness of a function on the interval [a, b] min f(x) = f(α), max f(x) = f(β).
el
x∈[ a , b ] x∈[ a , b ]


means that there is a number K > 0 such that |f(x)| < K for
all x ∈ [a, b]. In other words, if m = min{f(x) : a ≤ x ≤ b} and M = max{f(x):
eh

Y a ≤ x ≤ b}, then there are two points α, β ∈ [a, b] such that


je

K f(α) = m and f(β) = M. Note that these points α and β need


iit

not be unique.
The continuous function y = f(x) represented in Figure 1 attains
@

0 a b X its minimum on [a, b] at the point x = α and maximum at the


point x = β. In this case both points α and β belong to the
–K
interval (a, b).
The continuous function y = f(x) represented in Figure 2
Represented in the figure is the graph of a continuous function reaches its minimum on [a, b] at its left end point and maximum
f on a closed interval [a, b]. Obviously, there exists a number at a certain interior point β of this interval.
K > 0 such that the graph is located between the straight lines Y
y = K and y = –K.
Note that if a function is continuous on an open interval
(a, b) or on a half-open interval [a, b) or (a, b], then it is
not necessarily bounded on such an interval. For instance, the α β X
function y = 1/x is continuous but not bounded in the interval Figure 1
(0, 1]. There is no contradiction with the theorem since the
function in question is not continuous on a closed interval but
only in a half-open one.
If the continuity condition is not take into account, then the
assertion that the function f(x) is bounded may not be true. For
instance, the function
1 / x for 0  x 1
y= 
 0 for x0

The value (α) and f(β), whose existence is affirmed by the
is defined on the closed interval [0, 1], but is not bounded on theorem, are respectively the absolute minimum and absolute
this interval since it is discontinuous at x = 0. maximum of the function on the interval [a, b].

FREE BOOKS FOR JEE & NEET =>(@iitjeeadv)


2.34  
Differential Calculus for JEE Main and Advanced

The extreme value theorem guarantees that the maximum and the x ∈ (–1,1), shows that the function is continuous throughout
minimum exists, but does not tell us how to find them. The problem the open interval (–1, 1), but there is no number c in (–1,1)
of finding them is discussed in the chapter of maxima – minima. at which f has a maximum value. However, f has a minimum
If the function f(x) is continuous in the open interval (a, b) or value, f(0) = 1.
in the half-open interval (a, b], then the function may not attain
its least or greatest value. Further, if f(x) is a discontinuous Sign Preserving Property of Continuous Functions
function, then the theorem may not hold true. If f is continuous at c and f(c) ≠ 0, then there exists an interval
To see that continuity is necessary for the extreme value (c – δ, c + δ) around c such that f(x) has the sign of f(c) for
theorem to be true refer the graph shown below. every value of x in this interval.
Y Its truth is obvious if we understand that a continuous function
does not undergo sudden changes so that if f(c) is positive for
the value c of x and also f is continuous at c, it cannot suddenly
become negative or zero and must, therefore, remain positive
for values of x in a certain neighbourhood of c.
X If lim f(x) = b and b > 0, then there exists a deleted
x →c
There is a discontinuity at x = c in the interval. The function
has a minimum value at the left end point x = a and f has no neighbourhood D of c such that f(x) > 0 for every x in D.
maximum value. Similarly, if b < 0 there exists a D such that f(x) < 0 for every
Also, the function x in D.
x for 0  x  1, Proof Suppose f(c) > 0. By continuity, for every

f(x) =  ∈ > 0 there is a δ > 0 such that
ps
0 for x 1

f(c) – ∈ < f(x) < f(c) + ∈ ...(1)
el
which is continuous on an open interval [0, 1) has no absolute


whenever c – δ < x < c + δ.
maximum. As x → 1 from the left , f(x) → 1, but f(x) does not
eh

If we take the δ corresponding to ∈ = f(c)/2


attain the value 1. Incidentally, f takes on its minimum value 0
(this ∈ is positive), then (1) becomes
je

on the interval [0,1] at two places, at x = 0 and at x = 1


1 3
f ( c ) < f ( x ) < f ( c ) , c – δ < x < c + δ.
iit

If we consider the function y = x in the interval 0 < x < 1, there is no



least and no greatest values among them. There is no extreme 2 2
@

left point, since no matter what point x = x1 we take there will Y


3
x 2 f(c)
be a point to the left of it, for instance, the point 1 . Likewise
2 f(c)
there is no extreme right point; consequently, there is no least 1
2 f(c)
and no greatest value of the function y = x, 0 < x < 1.
Consider the function f(x) = tan–1x for x ≥ 0. c X
It is obvious that lim tan x = π/2. But there is no x for
–1
Here f(x) > 0 for x near c because f(c) > 0.
x
which the function tan–1x takes on the value π/2, and it does Therefore f(x) > 0 in this interval , and hence f(x) and f(c)
have the same sign. If f(c) < 0, we take the δ corresponding to
not attain maximum on x ≥ 0. In this case the conditions of
the theorem are not fulfilled : here the domain of the function 1
∈ = f (c) and arrive at the same conclusion.
[0, ∞) is unbounded. 2
If a function f is discontinuous, then it may have both a maximum Corollary. If f(x) is continuous at x = c, and f(x) vanishes for

and a minimum value, but this is not always true. values of x as near as we please, or assumes, for values of x
as near to c as we please, both positive and negative values,
 x, 0x2
For example, the function f(x) =  then f(c) = 0.
3  x , 2x4
This is an obvious corollary to sign preserving property. If
has a maximum at x = 2 and a minimum at x = 4, even if it is f(c) is not zero, it must be positive or negative; and if it were,
discontinuous of x = 2. This can be concluded from the graph for example, positive, f(x) would be positive for all values of
of y = f(x).
x sufficiently near to c, which contradicts the hypotheses of
Example 1: Let f(x) = 1/(1–x2) in the open interval (–1, 1). the theorem.


Show that f does not have a maximum value in this interval. Note: If there is one-sided continuity at c, then there is


Solution: For x near 1, f(x) gets arbitrarily large since a corresponding one-sided interval [c, c + δ) or (c – δ, c] in


the denominator 1–x 2 is close to 0. The graph of f, for which f has the same sign as f(c).

FREE BOOKS FOR JEE & NEET =>(@iitjeeadv)


Continuity of Functions 2.35

Bolzano's Theorem Hence, if a function is discontinuous in the interval [a, b]


If a function f is continuous on a closed interval [a, b] and the and f(a) and f(b) have opposite signs then f(x) may or maynot
numbers f(a) and f(b) are different from zero and have opposite have a root in (a, b).
signs, then there is atleast one point c on the open interval (a, b) Example 2: The equation x3– x – 2 = 0 may have a



such that f(c) = 0. solution somewhere between x = 1 and x = 2. Apply the
Y Bolzano's theorem to show that this is true.
Solution: The function f(x) = x3– x – 2 is continuous



on [1, 2] because it is a polynomial. f(1) = –2 and f(2) = 4.
0 a c Since, they have opposite signs, Bolzano's theorem implies
b X
that the function vanishes at some point in (1, 2).
In particular, there exists atleast one c in (1, 2) such that
The function whose graph is depicted in the above figure f(c) = c3 – c – 2 = 0.
satisfies the conditions of Bolzano's theorem. It is continuous So x = c is a solution of the equation x3 – x – 2 = 0 which lies
on [a, b] and f(a) < 0, f (b) > 0. Geometrically, it is obvious in (1, 2).
that the graph must intersect the x-axis atleast at one point
c ∈ (a, b). This is just what is stated by the theorem. Example 3: What can be said about the roots of f(x) =



In other words, if f is continuous in [a, b] and f(a) and f(b) have cosx – 2x2 in the interval 0 ≤ x ≤ 1 ?
opposite signs, then there is atleast one value of x for which Solution: f(x) = cosx – 2x2 in the interval 0 ≤ x ≤ 1.


f(x) vanishes in the interval (a, b). Since f(x) is a difference of the two continuous functions, it
Proof To fix the ideas, suppose that f(a) < 0 and f(b) > 0. is continuous. We find values of f at different values of x as
ps

Since f(x) is continuous, it will be negative in the neighbourhood shown in the table.
Y
el
of a and positive in the neighbourhood of b. The set of values of 1
x between a and b which make f(x) positive is bounded below by
eh

a, and hence possesses an exact lower bound k: clearly a < k < b.


0.2 0.4 0.6 0.8 1X
From the definition of the lower bound, the values of f(x)
je

must be negative or zero in a ≤ x < k. Since f(x) is continuous


iit

–1

when x = k, Roots occur where the graph of


@

a function crosses the x-axis


  
lim f ( x )  f (k ).
x k _ We conclude that f(x) has atleast one root in the interval 0.6 < x
< 0.8, since f(x) changes sign from positive to negative on that
Hence f(k) is also negative or zero. We shall show that f(k)
interval. The graph of f(x) suggests that there is only one root
cannot be negative; for if f(k)= – c, where c is positive, then
in the interval 0 ≤ x ≤ 1, but we cannot find the root from the
there exists a positive number δ such that
graph or the table of values.
|f(x) – f(k)| < c when |x – k| ≤ δ,
since f(x) is continuous when x = k. The function f(x) would Example 4: Given that a > b > c > d then prove that the


when be negative for whose values of x in (a, b) which lie equation (x – a) (x – c) + 2(x – b) (x – d) = 0 will have two
between k and k + δ, which contradicts the fact that k is the real and distinct roots.
lower bound of the set of values of x between a and b which Solution: (x – a) (x – c) + 2 (x – b) (x – d) = 0



make f(x) positive. It follows that f(k) = 0, and the theorem is Let f(x) = (x – a) (x – c) + 2 (x – b) (x – d)

therefore proved. f(a) = (a – a) (a – c) + 2 (a – b) (a – d) = + ve

In the following figure, f (a) and f (b) are of opposite signs but f(b) = (b – a) (b – c) + 0 = –ve

 
 
f (x) has no root in (a, b) as f is discontinuous. f(c) = 0 + 2 (c – b) (c – d) = –ve

f(d) = (d – a) (d – c) + 0 = +ve

Y

f(b) c b a
d
a
0 b X
f(a) Hence (x – a) (x – c) + 2(x – b) (x – d) = 0 has atleast one real
root in (d, c) and atleast one real root in (b, a). Since, a quadratic
equation can have atmost two real roots, exactly one real root

FREE BOOKS FOR JEE & NEET =>(@iitjeeadv)


2.36  
Differential Calculus for JEE Main and Advanced

lies in each of the two intervals. Thus the roots are real and g (1) = f (2) – f (1) = f (2) – f (1) ...(2)



distinct. Thus, g (0) and g (1) are of opposite signs.
Example 5: Show that e–x sin x = ln x has atleast one Hence by Bolzano's theorem there exists some c ∈ (0, 1)


solution on the interval [1, 2]. where g (c) = 0,
Solution: Notice that the function i.e. f (c + 1) = f (c) {c + 1 ∈ (1, 2) as c ∈ (0, 1)}


c = x1 ; c + 1 = x2, obviously x1, x2 ∈ (0, 2) we


f(x) = e–x sin x – ln x is continuous on [1, 2]. Putting


We find that have f (x2) = f (x1) where x2 – x1 = 1.
f(1) = e–1 sin 1 – ln 1 ≈ 0.31 > 0 and Example 9: Prove that the function



f(2) = e–2 sin 2 – ln 2 ≈ –0.57 < 0
f (x) = a x −1 + b 2 x − 1 – 2 x 2  3x  1


Therefore, by Bolzano theorem there is atleast one number c
on (1, 2) for which f(c) = 0, and it follows that e–c sin c = ln c. where a + 2b = 2 and a, b ∈ R always has a root in (1, 5)
∀ b ∈ R.
Example 6: Given a function on the interval [–2, 2]
Solution: There are three cases:




 x 2 + 2 if − 2 ≤ x < 0, Case I: Let b > 0, then f(1) = b > 0
f(x) = 
 − ( x + 2) if 0 ≤ x ≤ 2
2 and f(5) = 2a + 3b – 6 = 2(a + 2b) – b – 6
= 4 – b – 6 = – (2 + b) < 0

is there a point on this closed interval at which f(x) = 0?
Hence by Bolzano's theorem, there exists some c ∈ (1, 5) such
Solution: At the endpoints of the interval [–2, 2] the that f (c) = 0.


given function has different signs : Case II: If b = 0 then a = 2.
ps


f(–2) = 6; f(2) = – 6.
But it is easy to notice that it does not become zero at any f(x) = 2 x − 1 – 2 x 2  3x  1 = 0

el
point of the interval [–2, 2]. Indeed, x2 + 2 > 0 and – (x2 + 2) ⇒ 4(x – 1) = 2x2 – 3x + 1 = (2x – 1) (x – 1)

eh

< 0 at any x; this is due to the fact that f(x) has a discontinuity 5
at the point x = 0. ⇒ (x – 1) (2x – 5) = 0 ⇒ x =
je

2
Example 7: Let f be a continuous function defined from
iit

5


[0, 1] to [0, 1] with range [0, 1]. Show that is some 'c' in [0, 1] Hence f (x) = 0 if x = which lies in (1, 5).
2
@

such that f (c) = 1 – c.


Case III: If b < 0 , f(1) = b < 0 and
Solution: Consider g (x) = f (x) – 1 + x


f(2) = a + b 3 – 3
g (0) = f (0) – 1 ≤ 0 (as f (0) ≤ 1)



g (1) = f (1) ≥ 0 (as f(1) ∈ [0, 1] ) = (a + 2b) +  3 2 b– 3



There are three cases:
Case I: g(0) = 0 = (2 – 3 ) – (2 – 3)b

This happens when f(0) = 1. In such a case c = 0, which lies
in [0, 1]. = (2 – 3 ) (1 – b) > 0 (as b < 0)

Case II: g(1) = 0 Thus, f(1) as f(2) have opposite signs
This happens when f(1) = 0. In such a case c = 1, which lies Hence there exists some c ∈ (1, 2) ⊂ (1, 5) for which f(c) = 0.
in [0, 1].
Case III: g (0) and g (1) are of opposite signs Isolation of Roots
By Bolzano's theorem there exists atleast one c ∈ (0, 1) such A real number x1 is said to be a real root of the equation
that g (c) = 0. f(x) = 0 if f(x1) = 0. We say that a real root of an equation
⇒ g (c) = f (c) – 1 + c = 0 ⇒ f (c) = 1 – c. has been isolated if we exhibit an interval [a, b] containing

this root.
Combining the three cases, there exists some c in [0, 1] such
that f (c) = 1 – c. If f is a continuous function in an interval I and f(a) f(b) < 0 for
some a, b ∈ I, then by Bolzano’s theorem there is a point c

Example 8: Let f : [0, 2] → R be continuous and between a and b for which f(c) = 0. This is often used to locate


f(0) = f (2). Prove that there exists x1 and x2 in (0, 2) such that the roots of equations of the form f(x) = 0.
x2 – x1 = 1 and f (x2) = f (x1) For example, consider the equation x3 + x – 1 = 0.
Solution: Consider the continuous function g on Let f(x) = x3 + x – 1.


[0, 2] defined as g (x) = f (x + 1) – f (x) (x2 = x1 + 1) Note that f(0) = –1 whereas f(1) = 1. This shows that the above
Now,g (0) = f (1) – f (0) = f (1) – f (2) ...(1) equation has a root between 0 and 1.


FREE BOOKS FOR JEE & NEET =>(@iitjeeadv)
Continuity of Functions 2.37

Now we try with 0.5, f(0.5) = –0.375. So there must be a root of Now if we take the approximate root to be .6875, then we are
the equation between 1 and 0.5. We again try 0.75, f(0.75) > 0, away from the exact root by atmost a distance of .0625. If we
which means that the root is between 0.5 and 0.75. continue this process further, we shall get better and better
So we may try 0.625, f(0.625) < 0. So the root is between 0.75 approximations to the root of the equation.
and 0.625.

Concept Problems E

1. The boundedness theorem tells us that every continuous 4. Let f(x) = tanx. Although f(π/4) = 1 and f(3π/4) = –1,


real-function on the closed interval [0, 1] is bounded and there is no x in the interval ( π /4, 3 π /4) such that
attains its bounds. For each of the following intervals I f(x) = 0. Explain why this does not contradict Bolzano’s
give an example of a continuous function f from I to R theorem.
which is unbounded, and an example of a function g from 5. With the aid of Bolzano’s theorem, isolate the real roots
I to R which is continuous and bounded on I, but such that


of each of the following equations (each has four real
g does not have a maximum value on I.
roots).
(i) I = [0, 1); (ii) I = (1, ∞) ; (iii) I = [0, ∞).
(i) 2x4 – 14x2 + 14x – 1 = 0




2. Prove that the equations have a solution in the given (ii) x4 + 4x3 + x2 – 6x + 2 = 0

intervals

(i) x4 + 2x – 1 = 0 on [0, 1] ; 6.
Suppose that f is continuous on the interval [0, 1], that


(ii) x5 – 5x3+ 3 = 0 on [–3, –2] f(0) = 2, and that f has no zeros in the interval. Prove that


3. Prove that the equation sinx – x cosx = 0 has a root
ps f(x) > 0 for all x in [0, 1].

3
between π and π .
el
2
eh

Practice Problems E

je

7. A function f from R to R is said to be periodic, and a 1


iit

(ii) = x2 – x – 1 on [1, 2]
real number T > 0 is said to be a period of f it, for all x +1


@

x ∈ R, f(x + T) = f(x). Suppose that f is a continuous


function from R to R and that f is periodic. Prove that f (iii) + 9x2/3 = 29 on [0, 8]


is bounded. (iv) cos x = x2 – 1 on [0, π]


8. Does the function 2x – x3 + x5 have (a) a maximum value 12. Prove that the equation tan x = x has infinite number of

real roots.

for x in [–3, 10] ? (b) a minimum value for x in [–3, 10] ?
x 2 n  2  cos x
9. Prove that the function 13. Let ( x )  , show that φ(0) and φ(2) differ

x 2n  1

 x + 1, −1 ≤ x ≤ 0,
f(x) =  in sign but φ(x) does not vanish in [0, 2].
 −x , 0 < x ≤1

14. (i) Prove that the only polynomial function (with real


is discontinuous at x = 0 and still has the maximum and coefficient) p such that, for all x > 0, p(x)/x2 ∈ [2, 3]

minimum value on [–1, 1]. are those of the form p(x) = Ax2 for some constant
10. Show that there is a number x between π/2 and π such A ∈ [2, 3].

that tanx = –x. (ii) Give an example of a continuous function f from


11. Show that the given equation has atleast one solution on (0, ∞) to R which is not a polynomial but such that,

the indicated interval. f (x)
for all x > 0, 2 ≤ 2 ≤ 3 .
(i) 3 x = x2 + 2x – 1 on [0, 1] x


2.6 Intermediate Value Theorem (I.V.T.) In other words, a continuous function defined on

[a, b] takes on all values between f(a) and f(b). Pictorially, it
Theorem If f is continuous on the closed interval [a, b] and asserts that a horizontal line of height k must meet the graph of

k is any number between f(a) and f(b), then there is atleast one f atleast once if k is between f(a) and f(b), as shown in the figure.
number c in (a, b) such that f(c) = k. That is, when you move a pencil along the graph of a continuous

FREE BOOKS FOR JEE & NEET =>(@iitjeeadv)


2.38  
Differential Calculus for JEE Main and Advanced

function from one height to another, the pencil passes through all the interval [a, b]. Therefore, by Bolzano theorem , there is a
intermediate heights. This is a way of saying that the graph has point c lying inside (a, b) such that g(c) = 0 or f(c) – k = 0, that
no gaps or jumps, suggesting that the idea of being able to trace is f(c) = k. This is what we wished to prove.
such a graph without lifting the pen from the paper is accurate. Since k is any value between f(a) and f(b), it follows that f takes
The following graphs of functions continuous in a closed interval all values between f(a) and f(b) atleast once. In other words,
illustrate the theorem. a continuous function cannot pass from one value to another
without assuming once (atleast) every intermediate value.
Alternative proof: For definiteness, let f(a) < 0 and f(b) > 0.
Divide the interval [a, b] into n equal parts and consider the
values of the function with their proper sign at the points of
division, say x1, x2, .... , xn–1. Let xp+1 be the first of these points
for which f(x) is positive. Then at x = xp, the function is either
negative or zero. If it is zero, the theorem is at once established.
If not, we rename the interval [xp, xp+1] as [a1, b1].
Y
Divide it into n equal parts and consider the values of f(x) at
the points of division again. Suppose [a2, b2] is the first of the
k new sub-intervals such that f(b2) > 0 and f(a2) ≤ 0.
We take f(a2) ≠ 0, for otherwise the theorem is established.
Continuing this process indefinitely, we shall get a sequence
c1 c2 c3 X of intervals
[a1, b1], [a2, b2], [a3, b3], .....
ps
such that [a1, b1) ⊃ [a2, b2] ⊃ [a3, b3] ⊃ .....
Y

el
f(a) Also bk – ak = (b – a) / nk → 0 as k → ∞.
eh

This sequence of intervals will therefore define a limiting point


k x0. We will now show that f(x0) = 0. Since f is continuous at x = x0,
je

we have |f(x) – f(x0)| < ∈ for all x in the interval (x0 – δ, x0 + δ).
f(b)
iit

Now we can choose m so large that the interval [am, bm] lies within
(x0 – δ, x0 + δ), such that f(am) < 0 and f(bm) > 0. Also by the
@

a
sign preserving property if f(x0) ≠ 0, f(x) must have the same
bX
Any one of these sign as f(x0) in the interval (x0 – δ, x0 + δ). Hence f(am) and
three numbers
serves as c.
f(bm) cannot differ in sign unless f(x0) is zero, thus establishing
the theorem.
Note that continuity through the interval [a, b] is essential for
As a simple example of this theorem, consider a person’s
the validity of this theorem. This is illustrated by the following height. Suppose that a girl is 5 feet tall on her thirteenth
figure birthday and 5 feet 7 inches tall on her fourteenth
Y
birthday. Then, for any height h between 5 feet and
7 inches, there must have been a time t when her height was
k exactly h. This seems reasonable because human growth is
continuous and a person’s height does not abruptly change
from one value to another.
Example 1: Show that the function


f(x) = x – 1 – cos πx, x ∈ [1, 2] takes on the value 1/2.
X
Even though the theorem guarantees the existence of c, it does not Solution: The function f(x) = x – 1– cos πx, x ∈ [1, 2] is


tell how to find it. Such theorems are called existence theorems. continuous everywhere in the given interval.
To find c, we must solve the equation, namely f(c) = k. Also, we have f(1) = –1 and f(2) = 1.

Proof Let us define a new function g(x) = f(x) – k, where k Since –1 < 1/2 < 1, hence by I.V.T. there exists atleast one point

is a constant number lying between f(a) and f(b). Since f is a x = c in [1, 2] such that f(c) = 1/2.
continuous function on [a, b], so is the function g.
Without loss of generality let us assume that f(a) < k < f(b). Example 2: Does the function f(x) = x3/4 – sin πx + 3 take


Now g(a) = f(a) – k < 0 and g(b) = f(b) – k > 0. The function g, 1
on the value 2 within the interval [–2, 2]?
obviously, attains values of opposite signs at the end points of 3

FREE BOOKS FOR JEE & NEET =>(@iitjeeadv)


Continuity of Functions 2.39

Solution: The function f(x) = x3/4 – sin πx + 3 is Y




3
continuous within the interval [–2, 2]. Furthermore, at the end f( x ) = x + 2 x - 1
2 (1, 2)
points of this interval it attains the values f(–2) = 1; f(2) = 5.
1 1 (c, 0)
Since 1 < 2 < 5, then, by I.V.T. within the interval [–2, 2] there
3
1
exists at least one point x such that f(x) = 2 . –1 1 X
3
–1 (0, –1)
Example 3: Show that the function


a+b (Fig. 3)
f ( x ) = ( x – a ) 2(x–b)2 + x takes the value for some
value of x ∈ [a, b]. 2 f is continuous on [0, 1] with f(0) < 0 and f(1) > 0.

Solution: f is continuous in [a, b] and f(a) = a , f(b) = b. We The converse of I.V.T. is however not true. Consider, for


know that the average value of a and b example, the function f defined as follows :
a+b
i.e. ∈ [a, b]. f(x) = sin(1/x), x ≠ 0 and f(0) = 0.


2
In the interval [–2/π, 2/π], this function takes all values
By Intermediate Value Theorem, there exists atleast one
between f(–2/ π ) and f(2/ π ), that is, between –1 and 1
a+b
c ∈ (a, b) such that f(c) = . and infinite number of times as x varies from –2/ π to
2
2/π but the function is not continuous in this interval, being
Example 4: Use Intermediate Value Theorem to show discontinuous at x = 0.


that the equation 2x3 + x2– x + 1 = 5 has a solution in the ps
interval [1, 2]. Example 5: Let


3  x 2  2x 2  x  0
el
Solution: Let P(x) = 2x3 + x2 – x + 1. Then


f(x) = x 2 3
0x2
eh

P(1) = 2 . 13 + 12 – 1 + 1 = 3


P(2) = 2 . 23 + 22 – 2 + 1 = 19 1 x 2x5

je

Since P is continuous and 5 is between P(1) = 3 and P(2) = 19, Show that f(x) attains all intermediate values between f(–2)
iit

we may apply the Intermediate Value Theorem to P in the and f(5) even if f is discontinuous in the interval.
case a = 1, b = 2, and k = 5. Thus there is atleast one number c
@

between 1 and 2 such that P(c) = 6. This completes the answer. Solution: This is self evident from the graph of the


To get a more accurate estimate for a number c such function shown below. Howevever, this could not be concluded
that P(c) = 5, we can find a shorter interval for which the from I.V.T. since f(x) is discontinuous at x = 0 and x = 2.
Y
Intermediate Value Theorem can be applied. For instance,
P(1.2) ≈ 4.7 and P(1.3) ≈ 5.8. By the intermediate value
theorem, there is a number c in [1.2, 1.3] such that P(c) = 5.
Note: The Intermediate Value Theorem guarantees the


existence of atleast one number c in the closed interval [a, b].
There may, of course, be more than one number c such that
f(c) = k, as shown in Figure 1. A function that is not continuous
does not necessarily possess the intermediate value property.
For example, the graph of the function shown in Figure 2 jumps
over the horizontal line given by y = k and for this function
there is no value of c in [a, b] such that f(c) = k. X

Corollary. The I.V.T can also be reformulated in the following



way : a function continuous on a closed interval [a, b] assumes
all the intermediate values lying between its least and greatest
values on [a, b] (which exist by virtue of Extreme Value
Theorem).

FREE BOOKS FOR JEE & NEET =>(@iitjeeadv)


2.40  
Differential Calculus for JEE Main and Advanced

In other words, let f be continuous on [a, b] an let If x0 = a or x0 = b, then we prove the one-sided continuity of
k ∈ [m, M] where m = absolute minimum value of f and the function f in a similar way.
M = absolute maximum value of f on [a, b]. Then there exists Let f(x) be a function defined and bounded in the interval [a, b],
c ∈ [a, b] such that f(c) = k. then, if M and m are the bounds of f(x), the number M – m is
Proof By extreme value theorem, there exist x1, x2 ∈ called the span or oscillation of the function f(x) in the interval.

[a, b] such that m = f(x1) and M = f(x2). If x1 = x2, then f is constant Theorem A continuous function attains its bounds. If f(x)


on [a, b] and the result follows. Let x1 < x2. Since, f is continuous is continuous in [a, b] and M and m are its maximum and
on [x1, x2] ⊂ [a, b], by intermediate value theorem there exists minimum values, there are atleast two points x1 and x2 in (a, b)
c ∈[x1, x2] ⊂ [a, b] such that f(c) = k. such that
Y M f(x1) = M, f(x2) = m.
Proof Suppose that M is not attained; then M – f(x) does not
k


vanish at any point of [a, b]. Hence
1
is a continuous function, and therefore bounded. If
m M − f (x)
0 a x1 c x2 b X
G > 0 be its upper bound, we have
Similarly the result can be proved when x1 > x2.
1
≤ G,
Note: A continuous function whose domain is closed M − f (x)



must have a range in a closed interval but it is not necessary 1
that if the domain is open then range is also open (range can So that
M – f(x) ≥ ,
G
ps
be closed).
1
f(x) ≤ M –
el
Theorem Let f be a continuous strictly increasing function that is,


G
on a closed interval [a, b] and let α = f(a), β = f(b). Then (i) the
eh

range of f for the closed interval [a, b] is the closed interval [α, but this contradicts the fact that M is the maximum value of
f(x) in (a, b). Hence M must be attained. Similarly it may be
je

β], (ii) there exists a function x = g(y), the inverse of f, which


proved that m is attained.
is one-valued, strictly increasing and continuous on [α, β].
iit

Consider the following examples:


For example, on the closed interval [–π/2, π/2] the function
@

(i) If f(x) = 1/x except when x = 0 and f(0) = 0 , then f(x) has

y = sin x is continuous and strictly increasing; it has a neither an upper nor a lower bound in any interval which
continuous inverse which is, as we know, designated as includes x = 0 in its interior, such as the interval (–1, 1).
x = sin–1y (–1 ≤ y ≤ 1). (ii) If f(x) = 1/x2 except when x = 0, and f(0) = 0, then f(x)


A strictly decreasing function f(x), continuous on has the lower bound 0, but no upper bound, in the interval
[a, b] has an inverse, which is a strictly decreasing continuous (–1, 1).
function on [β, α] where α = f(a), β = f(b). (iii) Let f(x) = sin(1/x) except when x = 0 and f(0) = 0, then

Theorem Let a strictly increasing function y = f(x) map f(x) is discontinuous for x = 0. In any interval (–δ, δ) the

the closed interval [a, b] onto the closed interval [α, β], i.e. lower bound is –1 and the upper bound 1 and each of these
values is assumed by f(x) an infinity of times.
f([a, b]) = [α, β]. Then f is continuous on [a, b].
(iv) Let f(x) = x – [x]. This function is discontinous for all
Proof Let us be given an arbitrary point x0 belonging for the

integral values of x. In the interval (0, 1) its lower bound

time being to the open interval (a, b). By virtue of the fact that
in 0 and its upper bound 1. It is equal to 0. when x = 0 or
f is strictly increasing, the corresponding point y0 = f (x0) will
x = 1, but it is never equal to 1. Thus f(x) never assumes
belong to the interval (α, β) (α < y0 < β).
a value equal to its upper bound.
Let us take ε > 0 so small that α < y0 – ε < y0 < y0 + ε < β. (v) Let f(x) = 0 when x is irrational, and f(x) = q when x is
By the hypothesis, there exist points x1, x2 ∈ (a, b), (x1 < x0

a rational fraction p/q. Then f(x) has the lower bound 0,
< x2) such that y0 – ε = f(x1), y0 + ε = f(x2). but no upper bound, in any interval (a, b).
The interval (x1, x2) can be regarded as a neighbourhood of the But if f(x) = (–1)p q when x = p/q then f(x) has neither an

point x0 (x0 ∈ (x1, x2)). upper nor a lower bound in any interval.
Since the function f increases, for x ∈ (x1, x2) we shall have Corollary. If f(x) is continuous in the interval [a, b], then (a, b)

y0 – ε < f(x) < y0 + ε or |f(x) – y0| < ε, i.e. | f(x) – f(x0) | < ε, and can be subdivided into a finite number of sub-intervals in each
we have proved the continuity of the function f at the point x0. of which the span or oscillation of f(x) is less than any given ε.

FREE BOOKS FOR JEE & NEET =>(@iitjeeadv)


Continuity of Functions 2.41

Existence of Solutions of Equations Here f is continuous everywhere in (a, b], excluding the end
The Intermediate Value Theorem can be often used to locate the point a. Although f(a) is negative and f(b) is positive, there is
zeros of a function (i.e.solutions of equations written in the form no x in (a, b) for which f(x) = 0.
f(x) = 0) that is continuous on a closed interval. Specifically, Example 7: If n is a positive integer and if a > 0, then
if f is continuous on [a, b] and f(a) and f(b) differ in sign, then



prove that there is exactly one positive b such that bn = a.
the Intermediate Value Theorem guarantees the existence of
atleast one zero of f in the open interval (a, b). Solution: Choose c > 1 such that 0 < a < c, and consider



the function f defined on the interval [0, c] by the equation
Let f(x) = x2 – 2 = 0
f(x) = xn. This function is continuous on [0, c], and at the end
f(1) = –1 < 0, whereas f(2) = 2 > 0 points we have f(0) = 0, f(c) = cn.
We note that the function f is continuous on [1, 2] and that Since 0 < a < c < cn, the given number a lies between the
k = 0 is an intermediate value of f on the interval [1, 2]. function values f(0) and f(c). Therefore, by the Intermediate
Therefore, it follows from I.V.T. that f(c) = c2 – 2 = 0 for some
Value Theorem, we have f(x) = a for some x in (0, c) say
number c in (1, 2).
for x = b. This proves the existence of atleast one positive
i.e. c2 = 2 b such that bn = a. There cannot be more than one such b
This number c is the desired square root of 2. Thus it is the because f is strictly increasing on [0, c]. This completes the
intermediate value property of continuous functions that proof.
guarantees the existence of the number 2 .
Consider another example. Prove that the equation  
Example 8: Prove that the equation



x – cos x = 0 possesses a root in the interval (0, π). 1
The function f(x) = x – cos x is continuous on the closed interval
ps x −5 = has atleast one real root.

x +3
[0, π] and assumes f(0) = –1, and f(π) = π + 1 values having
el
opposite signs at its end points. Since – 1 < 0 < π + 1, by I.V.T. Solution: Let f(x) = x -5 –
1
f(x) assumes the value 0 at some x between 0 and π. Thus, the


x +3
eh

equation x – cos x = 0 possesses a root in the interval (0, π). The function is continuous on [5, 6].
je

1 1
Example 6: Show that the equation x5 –2x2 + x + 11= 0 Also f (5) = 0 – =– <0
5+3
iit


8


has atleast one real root.
1 8
@

Solution: For x large and positive the polynomial f (6) = 1 – = >0





9

P(x) = x5 –2x2 + x + 11 is positive 9
(since limx→∞ P(x) = ∞ ). Hence by intermediate value theorem there exists atleast one
Thus, there is a number b such that P(b) > 0. Similarly, for x value of c ∈ (5, 6) for which f (c) = 0
negative and of large absolute value P(x) is negative (since
lim P(x) = −∞ ). Let us now select a number a such that
x
P(a) < 0.
The number 0 is between P(a) and P(b). Since P is continuous

on the interval [a, b]. There is a number c in [a, b] such that
1
P(c) = 0. This number c is a real solution to the equation ∴ c−5 – c+3 = 0


x5 – 2x2 + x + 11 = 0. 1
Note that the argument in this example applies to any Thus, c is root of the equation x −5 = where
x+3
polynomial of odd degree. c ∈ (5, 6).
Note: In both Bolzano’s theorem and the intermediate
Example 9: Let f : R → R be a continuous onto function


value theorem, it is assumed that f is continuous at each point


satisfying f(x) + f(– x) = 0,  x  R . If f(–3) = 2 and f(5) = 4
of [a, b], including the endpoints a and b. To understand why
continuity at both endpoints is necessary, we refer to the curve in [– 5, 5], then prove that the equation f(x) = 0 has atleast
in the figure below. three real roots.
Y Solution: f(x) + f(– x) = 0 ⇒ f(x) is an odd function.


Since points (– 3, 2) and (5, 4) lie on the curve, points (3, – 2)
and (– 5, – 4) will also lie on the curve.
0 a b X
For minimum number of roots, the graph of a continuous
function f (x) can be drawn as follows.

FREE BOOKS FOR JEE & NEET =>(@iitjeeadv)


2.42  
Differential Calculus for JEE Main and Advanced

Y Example 13: Suppose that f is a continuous function of



[0, 1] and that f(0) = f(1). Let n be a positive integer. Prove
that there is some number x ∈ [0, 1] such that f(x) = f(x + 1/n).
[Universal Chord Theorem]
X Solution: Consider g(x) = f(x) – f(x + 1/n), which is clearly



continuous. If g is never zero in [0, 1] then g must be either
strictly positive or strictly negative. But then
 1 
   1 2 
0 = f(0) – f(1) =  f (0)  f  n   +  f    f   
     n   n 


From the above graph of f(x), it is clear that equation f(x) = 0
has atleast three real roots.  3    n 1   n 
 
2  

+  f    f    + ... +  f    f  n  .
 n  n    n   


Example 10: Using Intermediate Value Theorem, prove


that there exists a number x such that
The sum of each parenthesis on the right is strictly positive or
1 strictly negative and hence never 0, a contradiction.
x200 + = 200.
1 + sin 2 x

Example 14: A function f is continuous in the interval



[0,1] and assumes only rational values in the entire interval. If
Solution: Let f(x) = x200 + (1 + sin2x)–1.
1 1 1


f is continuous and f(0) = 1 < 200 and f(2) > 2200, which is f( ) = , prove that f(x) = everywhere.
2 2 2
much greater than 200. Hence, from the Intermediate Value
ps
Theorem there exists a number c in (0, 2) such that f(c) = 200. Solution: Suppose, if possible, that there exists a point


el
Example 11: Let f : R → R satisfy 1 1
c ∈ [0,1] such that f(c) ≠ . It is obvious that c ≠ . By the


eh

f(x) – f(y) = ex – y – 1 ∀ x, y ∈ R. 2 2

Prove that f is a continuous function. Also prove that the 1 1
je

law of trichotomy, either c < or < c. Without loss of


function f(x) has atleast one zero if f(0)< 1. 2 2
iit

1
Solution: lim f(x + h) – f(x) = lim (ex + h – x – 1) = 0 generality, let us assume that c < . The function f is continuous
@

2


h→0 h→0
Hence f is continuous everywhere. 1
in [c, ]. and therefore by I.V.T. it must every value lying
Putting y = 0, we have f(x) = f(0) + ex – 1. 2
Also lim f(x) = ∞ and lim f(x) = f(0) – 1 < 0 1
x x between f(c) and f   . But this is not possible, because f(c)
2
Since f(x) is positive for large positive x and negative of large
negative x, by Intermediate Value Theorem f(x) = 0 has atleast  
1
and f   are two distinct rational numbers between which
one root. 2
there lie infinitely many irrational numbers and f(x) does not
Example 12: If f(x) be a continuous function in [0, 2π] take any irrational value. The contradiction shows that there


and f(0) = f(2π) then prove that there exists point c ∈ [0, π] does not exist any c ∈ [0, 1] such that f(c) is different from
such that f(c) = f(c + π). 1 1
. Hence f(x) = everywhere.
Solution: Let g(x) = f(x) – f(x + π) ....(1) 2 2



at x = π; g(π) = f(π) – f(2π) ....(2) Example 15: Let f and g be continuous functions on an



g(0) = f(0) – f(π)


at x = 0, ...(3) interval I, let f(x) ≠ 0 for any x ∈ I and let (f(x))2 = (g(x))2



Adding (2) and (3), g(0) + g(π) = f(0) – f(2π) for all x ∈ I. Prove that either f(x) = g(x) for all x ∈ I or

⇒ g(0) + g(π) = 0 [given f(0) = f(2π)] f(x) = –g(x) for all x ∈ I.

⇒ g(0) = –g(π) Solution: Suppose, if possible, that there exist x1 ∈ I and

⇒ g(0) and g(π) are opposite in sign.


x2 ∈ I, such that f(x1) = g(x1) and f(x2) = –g(x2).

⇒ There exists a point c between 0 and π such that g(c) = 0. Since f is continuous on I and f(x) ≠ 0 anywhere on I, therefore

From (1) putting x = c, g(c) = f(c) – f(c + π) = 0 f(x1) and f(x2) must be of the same sign. Consequently, g(x1)
Hence, f(c) = f(c + π). and g(x2) must be of opposite signs.

FREE BOOKS FOR JEE & NEET =>(@iitjeeadv)
Continuity of Functions 2.43

Now g is continuous on I and g(x1), g(x2) are of opposite signs. Then f(a) = f(a . 1) = af(1) and
Therefore, by Intermediate Value Theorem, there exists x0
lying between x1 and x2, such that g(x0) = 0. Combining this  a a
f(a) = f  b  = bf   by the result we proved for integers
with (f(x))2 = (g(x))2 for all x ∈ I, we have f(x0) = 0, which is  b b
not possible. and hence
Therefore either f(x) = g(x) for all x ∈ I or f(x) = –g(x) for a
af(1) = bf   ⇒ f   = f(1)   .
a a
all x ∈ I. b b b
Example 16: Let f be a continuous function defined over We have established that for all rational numbers x ∈ Q,


the real numbers that satisfies the Cauchy functional equation f(x) = xf(1).
f(x + y) = f(x) + f(y), ∀ x, y ∈ R, We have not used the fact that the function is continuous so
Then prove that f is linear, that is, there is a constant c such far. Since the rational are dense in the reals the extension of
that f(x) = cx. the result to all real numbers now follows.

Solution: We first prove the assertion for positive integers Example 17: f is a continuous real-valued function such





n using induction. We then extend our result to negative that f(x + y) = f(x) f(y) for all real x, y. If f(2) = 5, find f(5).
integers. Then we extend the result to reciprocals of integers Solution: More generally, since f(nx) = f(x)n for all



and after that to rational numbers. Finally we extend the result integers n, f(1) = c = f(1/n)n for some constant c and all integers
to all real numbers. n. Thus f(k/n) = f(1/n)k = f(1)k/n = ck/n for all rational numbers
We prove by induction that for integer n ≥ 0, f(nx) = nf(x). k/n. By continuity, it follows that f(x) = cx for all real numbers
Using the functional equation, x. Since f(2) = 5, c = 5 , so f(5) = 25 5 .
ps
f(0 . x) = f(0 . x + 0 . x) = f(0 . x) + f(0 . x)

Example 18: Let f(x) be a polynomial with real
el
⇒ f(0 . x) = 0 f(x),


coefficients for which the equation f(x) = x has no real

and the assertion follows for n = 0. Assume n ≥ 1 is an integer
eh

solution. Prove that the equation f(f(x)) = x has no real


and that f((n – 1)x) = (n – 1)f(x). solution either.
je

Then
iit

f(nx) = f((n – 1)x + x) = f((n – 1)x) + f(x) Solution: Let g(x) = f(x) – x. Then, g(x) is a polynomial


that never vanishes. We argue that it must always have the
@

= (n – 1) f(x) + f(x) = nf(x),



same sign. Suppose if possible that g(a) < 0 < g(b) for some
proving the assertion for all strictly positive integers.
reals a and b. Since g(x), being a polynomial, is continuous,
Let m < 0 be an integer. Then –m > 0 is a strictly positive the Intermediate Value Theorem applies and there must be
integer, for which the result proved above holds, and thus, a number c between a and b for which g(c) = 0, yielding a
f(–mx) = –mf(x). Now. contradiction.
0 = f(0) ⇒ 0 = f(mx + (–mx)) = f(mx) + f(–mx) Thus, either g(x) > 0 for all x or else g(x) < 0 for all x. Then,
⇒ f(mx) = –f(–mx) = –(mf(x)) = mf(x), f(f(x)) – x = f(f(x)) – f(x) + f(x) – x


and the assertion follows for negative integers. = g(f(x)) + g(x)

We have thus proved the theorem for all integers. for all real x. Since g never changes sign, both g(x) and g(f(x))
a
Assume now that x = , with a ∈ I and b ∈ I–{0}. have the same sign (either positive or negative) and so their
b sum cannot vanish. Hence f(f(x)) ≠ x for any real x.

Concept Problems F

1. Use the Intermediate Value Theorem to show that there is 3. Use the Intermediate Value Theorem to show that the


a number c in (1, 2] such that 4 – c = 2c. equation has atleast one root :
2. Verify that the Intermediate Value Theorem applies in the (i) 2x3 + x2 – x = 4 (ii) x5 – 2x2 + x + 11 = 0





indicated interval and find the value of c guaranteed by 4. Prove that the equation 2x3 + 5x2 – 5x – 3 = 0 has a root

the theorem. between –∞ and –1, another between –1 and 0 and a third
(i) f(x) = x2 + x – 1, [0, 5], f(c) = 11 between 1 and 2.


(ii) f(x) = x3 – x2 + x – 2, [0, 3], f(c) = 4 5. Show that the equation x3 – 3x + 1 = 0 has a real root in



x2  x  5  the interval (1, 2). Approximate this root.
, 4 , f(c) = 6
x  1  2 
(iii) f(x) = ,


FREE BOOKS FOR JEE & NEET =>(@iitjeeadv)
2.44  
Differential Calculus for JEE Main and Advanced

6. Show that the equation x3 – 3x2 + 1 = 0 has three distinct 14. Let f be continuous on [a, b] and let f(x) be always rational.



roots by calculating the values of the function at x = –3, What can we say about f ?
–2, –1, 0, 1, 2 and 3 and then applying the Intermediate 15. Suppose that f and g are two functions both continuous


Value Theorem of continuous functions on appropriate on the interval [a, b], and such that f(a) = g(b) = p and
closed intervals. f(b) = g(a) = q where p ≠ q. Apply Intermediate Value
7. Prove that x3 + x2 – 3x – 3 = 0 has a root between 1 and 2, Theorem to the function h(x) = f(x) – g(x) to show that

between 1.5 and 1.75, between 1.625 and 1.75, etc. Show f(c) = g(c) at some point c of (a, b).
that if we continue this procedure, we can approximate 16. Suppose that today you leave your home at 1 P.M. and


the root of the equation as closely as we want. drive to school, arriving at 2 P.M. Tomorrow you leave your
8. If f(x) = x3 – x2 + x, show that there is a number c such school at 1 P.M. and retrace the same route, arriving home

that f(c) = 10. at 2 P.M. Show that at some instant between 1 and 2 P.M.
9. Use the Intermediate Value Theorem to show that there is you are at precisely the same point on the road both days.

a root of the given equation in the specific interval : 17. Let f : [1, e] → [0,1] be continuous then prove that


(i) 3
x = −1 − x , (0, 1) (ii) ln x = e , (1, 2)
–x f(x) = ln x has atleast one solution in [1, e].




10. Is there a number that is exactly 1 more than its cube? 18. Let f : (1, 10) → [2,11] be a continuous function, then



prove that it cannot be an invertible function.
11. Apply the Intermediate Value Theorem to prove that every
19. Let f be continuous on [a, b] and suppose that f(x) = 0 for

real number has a cube root.

every rational x in [a, b]. Prove that f(x) = 0 for all x in
12. Apply the Intermediate Value Theorem to prove that the [a, b].

equation x5 + x = 1 has a solution.
20. Let f, g be continuous functions from R to R. Suppose that
ps
13. Apply the Intermediate Value Theorem to prove that the
f(x) = g(x) for all x ∈ Q. Prove that f(x) = g(x) for all x

equation x3 – 4x2 + 1 = 0 has three solutions.
∈ R, i.e. f = g.
el
eh

Practice Problems F
je

iit

21. Show that the equation g(1) prove that there is a number c in (0, 1) such that

@

1 (f(c))3 = g(c)
x + sin x =
x +3

30. Prove that x  3  e x has atleast one real root.
has atleast one solution on the interval [0, π].


n
 ck x k ,
22. Show that the equation x5 + 3x4 + x – 2 = 0 has atleast one

root in the interval [0, 1]. 31. Let f be a polynomial of degree n, say f(x) =

k 0
23. Show that the equation x5 – 2x3 + x2 – 3x + 1 = 0 has atleast such that the first and last

one root in the interval [1, 2].
coefficients c0 and cn have opposite signs. Prove that
24. Find roughly the situations of the roots of 2x3 – 3x2 –


f(x) = 0 for atleast one positive x.
36x + 10 = 0.
25. Apply the Intermediate Value Theorem to show that every 32. Given a real valued function f which is continuous on the

closed interval [a, b]. Assume that f(a) ≤ a and that f(b) ≥

positive number a has a square root. That is, given a > 0,
prove that there exists a number r such that r2 = a. b. Prove that f has a fixed point in [a, b].
26. Suppose that a < b < c. If the function f is continuous on the 33. Prove that if f is continuous and has no zeros in [a, b],


closed interval [a, b] and on the closed interval [b, c], does then either f(x) > 0 for all x in [a, b] or f(x) < 0 for all x
it follow that f is continuous on [a, c] ? If f is continuous in [a, b].
on the closed interval [n, n + 1] for every integer n, does 34. A rational function can have infinitely many x-values at

it follow that f is continuous on the entire real line? which it is discontinuous. True or false.
27. Let f be a continuous function on R and periodic with 35. Let f1(x) and f2(x) be continuous on the closed interval


fundamental period 1 i.e. f(x + 1) = f(x), then prove that there [a,b]. If f1(a) < f2(a) and f2(b) > f2(b), prove that there exists
will be a real number x0, such that f(x0 + π) = f(x0). c between a and b such that f1(c) = f2(c).
28. Use the Intermediate Value Theorem to show that there 36. Show that the equation x4 + 5x3 + 5x – 1 = 0 has atleast


is a square with a diagonal length that is between r and 2r two solutions in the interval [–6, 2].
and an area that is half the area of a circle of radius r. 37. If f(x) is a continuous function in [2, 3] which takes only

29. Let f : [0, 1] → [0, 1] and g : [0, 1] → [0, 1] be con- irrational values for all x ∈ [2, 3] and f(2.5) = 5 then

tinuous fractions. Given that f(0) < g(0) and (f(1))3 > find f(2.8).

FREE BOOKS FOR JEE & NEET =>(@iitjeeadv)


Continuity of Functions 2.45

38. Find an interval in which the given equation has atleast there are no other functions that satisfy the stated

one solution. Note that the interval is not unique. conditions]
(i) ln x = (x – 2)2 (ii) e–x = x3 (b) Assume that f is a continuous function such that f(x)






(iii) cos x – sin x = x (iv) tan x = 2x2 – 1 > 0 for all x and f(x + y) = f(x) f(y) for all x and y.




39. Let f be a continuous function from R to R. Suppose that Let f(1) = c.

(i) f(1) = 1 and that (ii) f(x + y) = f(x) + f(y) for all real (i) Show that f(n) = cn for any positive integer n.



x and y. (ii) Show that f(0) = 1.



(a) Prove that f(x)= x for every positive rational x. (iii) Show that f(n) = cn for any negative integer n.





n
(b) Prove that f(0)=0 and that f(–x)=–f(x) for all real x. (iv) Show that f(1/n)= c for any positive integer n.





n m
(c) Prove that f(x) = x for all real x. (v) Show that f(m/n) = ( c ) for any integer m and




40. Let f be a continuous function such that f(x) positive integer n.

f(f(x)) = 1 and f(2006) = 2005, then consider the (vi) By (v), f(x) = cx for any rational number x.



1
Assuming that f is continuous and that the
following assertion : f(1000) = . If the reason is : as exponential function cx is continuous, deduce
1000

that f(x) = cx for all real numbers x.
1 1
f(f(x)) = we have f ( t ) = 44. Let f be a continuous function whose domain is the x axis


f (x) t and which has the property that f(x + y) = f(x) + f(y) for
all numbers x and y. This question shows that f must be of
1
⇒ f(1000) = the form f(x) = cx for some constant c.
1000


(a) Let f(1) = c. Show that f(2) = 2c.


Prove that the assersion is correct but the reason is false.
ps
(b) Show that f(0) = 0.

41. Find all functions f : R → R, continuous at x = 1 such that


(c) Show that f(–1) = –c.

el
∀ x ∈ R, f(x) = –f(x2).


(d) Show that for any positive integer n, f(n) = cn.
eh

42. Let f and g be continuous functions defined for all x. (e) Show that for any negative integer n, f(n) = cn.



Assume that f(x) = g(x) for all rational x. Deduce that 1
(f) Show that f ( 2 ) = c/2.
je


f(x) = g(x) for all real numbers x. (g) Show that for any nonzero integer n, f(1/n) = c/n.
iit


43. Let us determine all continuous functions f such that (h) Show that for nay integer m and positive integer n,



f(x + y) = f(x)f(y) for all real numbers x and y and such
@

f(m/n) = c(m/n).
that the values of f are always positive.
(i) Show that for any irrational number x, f(x) = cx. (This
(a) Let b be a fixed positive number. Let f(x) = b x.


is where the continuity of f enters).


Check that f(x + y) = f(x)f(y). [Part(b) will show that

Target Problems for JEE Advanced

Problem 1: Find the values of a and b if f is continuous tan(3  6 h )  tan 6 h




at x = π/2, where = lim    6  cot 5
6 tan(5 / 25h ) =
lim  

tan 6 x h 0  5  h 0  5 
 6  tan 5 x 
f (x)    , 0x  tan 6 h tan 5 h

0
5 2 6 6
 lim   = lim    1
π h 0  5  h0  5 

= a + 2, x=
2

b|tan(  / 2  h )|
b|tan x|      a
 lim f   h   lim 1  cot   h  
= (1 | cot x x

|) a , h 0  2  h 0  2 

2
 bcoth
Solution: We have f    a  2 = lim (1+ tan h)
2
a = e b/a .



h →0
tan 6 (  / 2  h )
   6  tan(5 / 25h ) Since f is continuous at x = π/2, we have
lim f   h   lim   a + 2 = 1 = eb/a

h 0  2  h 0  5 

which gives a = –1, b = 0

FREE BOOKS FOR JEE & NEET =>(@iitjeeadv)
2.46  
Differential Calculus for JEE Main and Advanced

Problem 2: Obtain a relation in a and b, if possible, so


lim x sin x − 2 (1 − cos x )


that the function x→0
x4


x n (a  sin( x n ))  (b  sin( x n )) x x
5 lim 2 x sin 2 cos 2 − 4 sin 2
2 x
f(x) = lim
(1  x n ) sec(tan 1 ( x n  x  n ))
n  =

2 x→0 x4


is continuous at x = 1. 2 sin x x cos x − 2 cos x
x n (a  sin( x n ))  (b  sin( x n )) 5 2 lim 2 2
Solution: f(x) = lim = xlim x→0
1 n n
(1  x ) sec(tan ( x  x 2 →0 x3


n
n 


)) x
For continuity at x = 1 Let x = 2 θ


lim f(x) must exist and equals f (1) 5 lim 2θ cos θ − 2 sin θ
=

x→1
16   0 θ3


1n (a  sin 1n )  b  sin(1n ) (  tan )
f(1) = nlim lim 2 cos θ


(1  1n )·sec tan 1 (1n  1 n )  =

0 3


a  sin 1  b  sin 1 a+b 5 lim   tan   1 5
= = = = 5   = − .
 
 8  3


1 8 0 3 24

sec(tan 2) 2 5
5
Now for x > 1 in the immediate neighbourhood Since f is continuous f(0) = − .
b  sin x n 24
a  sin( x n ) 
xn Problem 4: Let S denotes the sum of an infinite
f(x) = lim


n   1  geometric progression whose first term is the value of the

1 n n
1  n  sec(tan ( x  x ))
ps
function
 x 
el
a  (some quantity between 1 and  1)  0 sin( x  ( / 6))
= =0 f(x) = at x = π/6, if f(x) is continuous at
1·sec(tan 1 )
eh

3  2 cos x

Similarly for x < 1 in the immediate neighbourhood of 1 x = π/6 and whose common ratio is the limiting value of the
je

b sin( x )1/ 3 ln(1  3x )


iit

f(x) = =0 function g (x) =


( tan 1 x ) 2 (e5 x  1)
1/ 3

1· sec(tan 1 )

@

Hence f (x) = 0 for x ≠ 1 as x → 0. Find the value of S.


∴ lim f (x) = 0 = a + b.
x→1
 sin  x  ( 6) 
lim
A cos x  B x sin x  5 Solution: a = f   = x
3  2 cos x
, (x ≠ 0)  
6


Problem 3: If f (x) = 6
x4


sin  x  ( 6) 
is continuous at x = 0, then find the value of A and B. Also lim
 6 2  cos(  6)  cos x 
= x
find f (0).

Solution: For continuity lim f (x) = f (0) 2 sin(( x/2)  (/12)) cos(( x/2)  (/12))
lim


x→0 = x 6 4 sin((/12)  ( x/2)) sin((/12)  ( x/2))

A cos x  B x sin x  5
Now for lim to exist 2
x→0 x4 == 1.

2
as x → 0, Numerator → A − 5 and
Hence, a = 1
Denominator → 0 . Hence A − 5 = 0 ⇒ A = 5

Hence lim B x sin x − 5 (1 − cos x )


sin( x )1/ 3 .ln(1  3x )  5x  1/ 3

r = xlim
x→0
x4 0
 tan 1 x 
2

e  1
 5x  1/ 3

5.x1/ 3
 
sin x
B. x  5 sin 2 x
 x 
1  cos x x 2
= lim
x→0 3 ln(1  3x )1/ 3x 3

x2 = lim  .
x 0 5 5

5
as x → 0, Numerator → B − and
2 a 1 5
Sum S =  
Denominator → 0 ⇒ B =
5 1 r 1 3 2


2 5

FREE BOOKS FOR JEE & NEET =>(@iitjeeadv)


Continuity of Functions 2.47

Problem 5: Let f (x) a h sin ah  b h sin bh




n
 b sin x  sin bx 
a x 1 = hlim
 n   x0 a h . b h [tan ah  tan bh ]
0


 x sin x  cos x  cos bx 
 (1  h ln a.....)(ah....)  (1  h ln b)(bh...)
=  = hlim
0 (ah...)  (bh....)




 x (a  b)h  ......
 a sin bx  b sin ax
x
x0 = hlim
0 (a  h ) h  ......
=1
n bx  tan ax


tan
be continuous at x = 0 (a, b > 0 ,b ≠ 1, a ≠ b). Obtain f(0) and ∴ If f (x) is continuous at x = 0, then f (0) = 1.
a relation between a, b and n. Also the continuity relationship between a, b and n should be
n
b
Solution: ln a ·    1 .
3


n
a h  1  b sinh  sin b h
f(0+) lim f (0  h )  lim Problem 6: If f (x)
h 0 sinh . h n  
 cos h  cos b h 



h 0

sin 3x + A sin 2x + B sin x
ah 1 h = (x ≠ 0)
=  lim


x5

h 0 h sin h
n is continuous at x = 0, then find f (0).
 b sin( h )  sin(bh)  h2 
  cos h  cos bh 
 Solution: We have
 h .h2  


  1  (3x )3 (3x )5 
lim
f(0)= x  3x 
5 
  ......
= ln a · l n (say) 0 x   3! 5! 


ps  
b sin h  sin bh h2 ( 2 x )3 ( 2 x )5
lim ·  A  2x    .....

where l = h  
el
0 h3 cos h  cos bh 3! 5!
eh

 b(sin h  h )  (sin bh  bh )  x3 x5 
lim   B  x    ......  ...(1)
= h 0  h3   3! 5!  


je



1 Now f (x) is continuous at x = 0, so we must have
iit

 
.  b 2  1  cos bh    1  cosh   2A + 3 + B = 0 ...(2)
    

@




2 2
 b h 
2
 h 
27 8A B
and + + =0
  sin h  h   sin bh  bh )  3  6 6 6


= lim  b     b  ⇒ 8A + B = – 27 ...(3)
  h 
3
h0   b3 h 3  



∴ On solving (2) and (3), we get
 2 1 

b 1 A = – 4, B = 5
.    

 2 2  
5
3 A.25 B
  Hence, f (0) = + + = 1.
5! 5! 5!

  1  3  1  2
= b     b    . 2 Alternative:
  6   6   (b  1)


We have f (0)
sin x  x 1 3 sin x  4 sin 3 x  2A sin x cos x  B sin x
[using lim  ] lim
= x 0 x5

x3

h 0 6
For limit to exist 3 + 2A + B = 0 ...(4)

b (b 2  1) 2 b
= · 2   3 sin x  4 sin x  2A sin x cos x  (3  2A ) sin x
3
6 (b  1)  3  lim
= x

0 x5
n
b sin x  3  4 sin 2 x  2A cos x  3  2A 
Hence f (0+) = ln a ·   lim
= x  
3 0 x  x4 

Now f (0 ) = lim f (0  h )

h0
2A (1  cos x )  4 sin 2 x
lim
= x 0 x4


a  h sin( bh )  b  h sin(ah )
 lim x
h 0 tan( bh )  tan(ah ) 4A sin 2  4 sin 2 x
lim 2

(multiply Dr & Nr by ah · b h) = x 0 x4


FREE BOOKS FOR JEE & NEET =>(@iitjeeadv)
2.48  
Differential Calculus for JEE Main and Advanced

x x x Solution: f(x) = cosec 2x + cosec 22x + .... + cosec 2n x


−4A sin 2
− 16 sin 2 cos 2



2 2 2 1 sin(2 x − x )
= lim Now cosec 2x = = = cot x – cot 2x

x →0 x4 sin 2 x sin x sin 2 x
x  x Similarly cosec 22 x = cot 2x – cot 22 x
sin 2  A  4 cos 2 


cosec 23 x = cot 22 x – cot 23 x
=  lim 2 2


  
x 0 x4  x2




cosec 2n x = cot 2 n – 1 x – cot 2n x
4  


⇒ f (x) = cot x – cot 2n x
∴ A+4=0 ⇒ A= –4 ⇒ B=5


∴ g (x) = f (x) + cot 2n x = cot x



Also f (0) = 1.


lim H (0 + h) = lim ((cos h)cot h + (sec h)cosech )
Now h→0 h→0
Problem 7: The function


e 2 x  1  x (e 2 x  1) = elim coth (cos h 1) + elim cosech (sec h 1)
h 0 h 0


f (x) = is not defined at x = 0 .
x3 =1+1=2 ...(1)



What should be the value of f (x) so that f (x) is continuous
at x = 0 ? e  h  e h  2 cos h
H (0 – h) = lim

h→0 h sec h
e 2 x  1  x (e 2 x  1)
Solution:  = lim
x→0
x3  h 
 e  e  2 2(1  cos h ) 
h


lim 
= h→0 2
 2  = 2 ...(2)
 

e6 t  1  3 t (e6 t  1)  h h 
Put x = 3 t = tlim From (1) and (2) H (x) will be continuous if p = 2.
→0 3
ps

27 t
Problem 9: Check continuity of the function
el
(e 2 t  1)3  3 e 2 t (e2 t  1)  3 t (e6 t  1)


= tlim cos x  x 2 n sin( x  1)
eh

→0
27 t 3 f(x) = lim
n  1  x 2 n 1  x 2 n

je

(e 2 t 1)3 3 e 2 t[e 2 t 1t (e 2 t 1)]3 t[e 2 t (e 2 t 1)  e6 t  1]


= tlim Solution: We have
iit

→0


27 t 3


@

(e 2 t − 1)3 1 
= tlim + lim e2t ×   cos x  0 , | x | 1 ∵lim x 2 n 0 if | x |  1
→0 27 t 3 t→0  n  

9  1 x  0

 cos x  sin( x  1)
1 f(x) =  , | x | 1
− tlim
→ 0 9 t 2 (e − 1) (e − 1)
2t 4t
 111

 cos x
  sin( x  1)
2n  sin( x  1)
8  2t
 1  e4 t  1   lim x  , | x | 1
⇒ =
8

8 lim  e  ×    n  1
 x 1 x 1
t→0  
 4t  
2t  2 n

9 27 9  x

1 2 cos x , | x | 1
⇒  = −1 ⇒  = − .  1  sin 2, x  1

3 3 

2 i.e. f(x) = 1, x 1
The value of f(0) =  = – 

3   sin( x  1) , | x | 1

 x 1
Problem 8: Let f (x) = cosec 2x + cosec 2 2 x + cosec
The above function may have discontinuities only at


23 x + .......... cosec 2n x, x ∈ (0, π/2)
x = ±1.
and g (x) = f (x) + cot 2n x
At x = –1, we have

If
lim f (1  h )  lim cos ( 1  h )  1 .

(cos x )g ( x )  (sec x )cos ec x if x  0 h 0 h 0


p if x  0 f(–1) = –1 + sin2.
H (x) =  x  x

 e  e  2 cos x This implies that f(x) is discontinuous at x = –1.

 if x  0
 x sin x At x = 1, we have
find the value of p, if possible to make the function H (x)  sin(1  h  1)  sinh
lim f (1  h )  lim  lim  1 .
continuous at x = 0. h 0 h 0 1 h 1 h  0 h

FREE BOOKS FOR JEE & NEET =>(@iitjeeadv)


Continuity of Functions 2.49

lim f (1  h )  lim cos (1  h )  1 . We discuss continuity of g(u) for u = a ∈ N.


h 0 h 0

LHL at u = a:
f(1) = –1 lim g(u) = lim [a – h] + a − h − [a − h ]

u a  h→0


Thus, f(x) is discontinuous at x = 1.
Hence, f(x) is continuous on x ∈ R − {−1}. lim (a – 1) +
= h→0 a − h − (a − 1 )


x f (1)  h ( x )  1
m
=a–1+1=a


Problem 10: If g(x) = lim Now RHL at u = a :
m  2x  3x  3
m


lim lim a  h  [a  h ]
is continuous at x = 1 and g(1) = lim{ln(ex )}
2 /ln x
, then find u a g(u) = h→0 [a + h] +



x →1
the value of 2g(1) + 2f(1) – h(1); assume that f(x) and h(x) are lim a +
= h→0 a h a



continuous at x = 1.
= a,


Solution: Here, and g(a) = [a] + a − [a ] = a as a ∈ N.




g(1) = lim {lne + lnx }
2 /ln x
So, g(u) is continuous ∀ a ∈ N, now g(u) is clearly continuous

x→1
in (a – 1, a) ∀ a ∈ N.
{1 + lnx }
2 /ln x
= lim
Hence g(u) is continuous in [0, ∞).

x→1

= lim
x →1
ln x
2
Now u(x) = tan x is continuous in [0, π/2).
e ln x

So, f(x) = g{u(x)} is continuous in [0, π/2).
2
g(1) = e ...(1)


x x
Problem 12: If f(x) = +
 x m f (1) + h ( x ) + 1 (1 + x ) (1 + x )(1 + 2 x )
ps


Now, lim lim lim
g(x) = x 1 m   
 2 x + 3x + 3 

x1  m x
el
+
+ .....
(1 + 2 x )(1 + 3x )

h(1) + 1
eh

= {since x < 1 ⇒ m lim xm = 0}


to infinity, then examine the continuity of f at x = 0.
3+3


je

h(1) + 1 Solution: For x ≠ 0, sum of n terms of the series


∴ lim g(x) = ...(2)


iit




x1 6
 1   1 1 
1  1  x  +  1  x  1  2x 
@

 x m f (1)  h ( x )  1 
 
And, lim g(x) = lim lim     


 2 x  3x  3 
m
  



x 1 m +
x→1  1 1   1 1 
+  1  2x  1  3x  + ... +   
f (1)  h ( x ) / x  1 / x    1  ( n  1) x 1  nx 

m m
f (1)
= lim lim = 1
m 1
23/ x 3/ x

x 1 m   m 2 =1– .

f (1) 1+ nx
∴ lim g(x) = ...(3)  1 
∴ f(x) = lim 1 



x1 2 = 1 – 0 = 1.
n   1  nx 

As g(x) is continuous at x = 1, from (1), (2) and (3)
h (1)  1 f (1) and f(0) = 0
e2 =   1, x  0
∴ f(x) = 

6 2
⇒  x, x  0

2
h(1) = 6e – 1 and f(1) = 2e2



⇒ 2g(1) + 2f(1) – h(1) = 2e + 4e – 6e2 + 1 = 1
2 2 Clearly f(x) is discontinuous at x = 0.

∴ 2g(1) + 2f(1) – h(1) = 1. Problem 13: Show that the function f : R→R defined



1
Problem 11: Prove that by f ( x )  1/(sin n ! x ) can be made discontinuous at any
1 e


f(x) = [tan x] + tan x − [tan x ] . (where [.] denotes greatest
rational point in the interval [0, 1] by a proper choice of n.
 
integer function) is continuous in 0,  . Solution: Let x be rational say p/q, where p and q are
 2


integers prime to each other. Then taking n = q, we see that
Solution: f(x) = [tan x] + tan x − [tan x ] n! πx = q! . π . (p/q)



Let g(u) = [u] + u − [u ] , is an integral multiple of π and therefore sin n! πx = 0 but cos n!
πx = 1 or –1 according as n! x is an even or an odd integer. Now

Then f(x) = g(u(x)) where u(x) = tan x ≥ 0

FREE BOOKS FOR JEE & NEET =>(@iitjeeadv)


2.50  
Differential Calculus for JEE Main and Advanced

sin n! π(x±h) = sin n! πx cos n! πh ± cos(n! πx) sin(n! πh) and if x is an odd integer, then
= ± sin n! πh or ∓ sin(n! πh) ...(1) f(x) = 0, f(x+) = –1 and f(x–) = 1.


Hence f has discontinuities of the first kind at


according as n! x is even or odd. x = 0, 1, 2, .... , n , .... ,
(i) Let n! x be an even integer.



Problem 15: Discuss the nature of the discontinuity of
1




Then f ( x )  lim the function f defined by
h  0 1  e1/sin{n ! ( x  h )}

log(2  x )  x 2 n sin x
f(x) = nlim at x = 1.
1 1  1  x 2n


 lim   0 [using (1)]
h 0 1  e1/sin( n ! h ) 1  e Show that f(0) and f(π/2) differ in sign and explain still why

1 f does not vanish in [0, π/2].

and f ( x )  lim
h 0 1 e 1/sin{n ! ( x  h )}
Solution: We shall first of all obtain an expression for f




in [0, π/2] in a form free from limits.
1
 lim 1/sin( n !h ) [using (1)] If 0 ≤ x < 1, then x2n → 0 as n → ∞ , and if x > 1, then x–2n →
h 0 1 e

0 as n → ∞ , therefore we have:
1 1 log(2  x )  x 2 n sin x
 
 1 If 0 ≤ x < 1, then f(x) = lim
1 e 1 0


n  1  x 2n
(ii) Let n! x be an odd integer. Then = log (2 + x) ...(1)




log(3)  1 sin 1 2n
1
ps
f(x+) = lim 1/sin( n !h )
1 If x = 1, then f(x) = lim
h 0 1 e 12 n

n 


el
1
= (log 3 − sin 1) ...(2)
eh

 1
and f ( x )  hlim  0.



2
1/sin( n !h )
0 1 e


x 2 n log(2  x )  sin x
je

If x > 1, then f(x) = nlim


Hence f(x ) ≠ f(x ) at any rational point x.
+ –  x 2 n  1

iit

= –sin x ...(3)


Problem 14: Show that the function f : R → R defined by From (1), (2) and (3), we have
@


(1  sin x ) t  1 log(2  x ) if 0  x  1
f ( x )  lim 1
t  (1  sin x ) t  1 f(x) =  2 (log 3  sin 1), if x  1 ...(4)




has discontinuity of first kind at the points  sin x if x 1
x = 0, 1, 2, ...., n .... .
f(1–) = lim f (1  h ) = lim log(3  h ) = log 3

h 0 h 0
Solution: At x = 0, 1, 2, 3, ..... , n .... , we have
f(1+) = lim f (1  h ) = hlim  sin(1  h ) = –sin1.


0 h 0


(1  0) t  1
sin πx = 0, so that f ( x )  lim 0 We find that f(1 ) and f(1 ) both exist but are unequal; also
– +
t (1  0) t  1

neither of them is equal to f(1). Therefore f has a discontinuity
at these values.
at x = 1.
Now, if 2m < x < 2m + 1 (m being an integer), then sin πx is
From (4), we find that
positive. Hence for such values of x, we have
f(0) = log 2 > 0, f(π/2) = –sin π/2 = –1,
1  1 / (1  sin x ) t 1  (1 / ) so that f(0) and f(π/2) are of opposite signs.
f ( x )  lim  1
t  1  1 / (1  sin x ) t
1  (1 / ) Again, from (4) it is clear that f does not vanish anywhere in
And if 2m + 1 < x < 2m + 2. sin πx is negative and so [0, π/2].

lim (1 + sin πx)t = 0. The function f is not continuous in [0, π/2], the point x = 1
being a point of discontinuity. This explains as to why f does

t 
0 1 not vanish anywhere in [0, π/2] even though f(0) and f(π/2)
∴ = –1 for these values of x.
f(x) =
0 1

are of opposite signs.
Hence if x an even integer, then The hypothesis as well as the conclusion of the Intermediate
f(x) = 0, f(x+) = 1 and f(x–) = –1, Value Theorem are not satisfied for the function f in [0, π/2].

FREE BOOKS FOR JEE & NEET =>(@iitjeeadv)
Continuity of Functions 2.51

Things to Remember
1. A function f(x) is said to be continuous redefine the function such that lim f(x) = f(a) and make

x→a
at x = a , if lim f(x) = f(a). it continuous at x = a.
x →a

14. A function is said to have a finite or jump discontinuity at
A function f(x) is said to be continuous at the left end point


2.
x = a if lim f(x) does not exist since the left hand limit

x = a if, f(a) = lim f(x) and f(x) is said to be continuous x→a
x a 
at the right end point x = b if, f(b) = lim f(x) and the right hand limit are unequal, but the one-sided
x b limits do exist.

f ( x ) if x is in the domain of f 15. If x = a is a point of finite discontinuity of the function
3. F( x )  


if x  a f(x), then the graph of this function undergoes a jump at

L
The function F is continuous is called the continuous x = a. The difference R.H.L. – L.H.L. i.e. f(a+) – f(a–) is
called the jump in the function at x = a.

extension of f to x = a, provided lim f(x) = L exists.
x →a
4. A function f is said to be continuous in an open interval 16. The difference between the greatest and least of the


three numbers f(a+), f(a–), f(a) is the saltus or measure of

(a , b) if f is continuous at each and every point lying in
the interval (a, b). discontinuity of the function at the point a.
5. A function f is said to be continuous in a closed interval 17. A function is said to have an infinite discontinuity at

x = a if atleast one of the one-sided limits is infinite.

[a, b] if :
(i) f is continuous in the open interval (a , b) 18. The concept of pole discontinuity is related with infinite



(ii) f is right continuous at ‘a’ i.e. lim f(x) = f(a). limit. For a point x = a to qualify as a pole of a function
x a 
ps


1
(iii) f is left continuous at ‘b’ i.e. lim f(x) = f(b). f, we must have lim 0.
el
x b x a f ( x )



A function is said to have an oscillatory discontinuity
eh

6. We use the term suspicious point for a number c where 19.




(i) The definition of the function changes or domain of at x = a if atleast one of the one-sided limits does not
je


f splits, exist because of too much oscillation in the values of the
(ii) Substitution of x = c causes division by 0 in the
iit

function.


function.
If f(x) and g(x) are continuous at x = a, then the following
@

20.
7. If both the one-sided limits lim f(x) and lim f(x) exist,

x a  functions are also continuous at x = a.

x a 
but the conditions of continuity are not satisfied. Then the (i) cf(x) is continuous at x = a, where c is any constant.
function f(x) is said to have a discontinuity of the first kind


(ii) f(x) ± g(x) is continuous at x = a.
at the point a.


(iii) f(x). g(x) is continuous at x = a.
8. A function f(x) having a finite number of discontinuities


(iv) f(x)/g(x) is continuous at x = a, provided

of first kind in a given interval is called sectionally or


piecewise continuous function. g(a) ≠ 0.
9. The function f(x) is said to have discontinuity of the second 21. If f(x) is continuous at x = a and g(x) is discontinuous at

x = a, then we have the following results.

kind at x = a, if atleast one of the one-sided limits (L.H.L.
or R.H.L.) at the point x = a does not exist or equals to (i) Both the functions f(x) + g(x) and f(x) – g(x) are


infinity. discontinuous at x = a.
10. A function is said to have a removable discontinuity at (ii) f(x). g(x) is not necessarily discontinuous at x = a.



x = a, if lim f(x) exists but is not equal to f(a). In this case (iii) f(x)/g(x) is not necessarily discontinuous at


x→a
x = a.
we can redefine the function such that lim f(x) = f(a) and
x→a 22. If f(x) and g(x) both are discontinuous at x = a, then we
make it continuous at x = a.

have the following results.
11. A function is said to have a missing point discontinuity (i) The functions f(x) + g(x) and f(x) – g(x) are not

at x = a if lim f(x) exists while the function is undefined


necessarily discontinuous at x = a. However, atmost
at x = a. x → a
one of f(x) + g(x) or f(x) – g(x) can be continuous
12. A function is said to have an isolated point discontinuity at x = a. That is, both of them cannot be continuous

at x = a if lim f(x) exists and the function is defined at simultaneously at x = a.
x→a
x = a, but they are unequal. (ii) f(x). g(x) is not necessarily discontinuous at x = a.


13. A function is said to have an irremovable discontinuity at (iii) f(x)/g(x) is not necessarily discontinuous at



x = a, if lim f(x) does not exist. In this case we cannot x = a.
x→a

FREE BOOKS FOR JEE & NEET =>(@iitjeeadv)


2.52  
Differential Calculus for JEE Main and Advanced

23. If f(x) is continuous at x = a and g(x) is continuous at value and atleast one point at which it assumes the least

x = f(a) then the composite function (gof)(x) is continuous value on that interval.
at x = a. 33. If f is continuous at c and f(c) ≠ 0, then there exists an


24. Let a function f(x) be continuous at all points in the interval [a, interval (c - δ, c + δ) around c such that f(x) has the sign

b], and let its range be the interval [A, B] and further a function of f(c) for every value of x in this interval.
g(x) is continuous in the interval [A, B], then the composite 34. If a function f is continuous on a closed interval [a, b] and
function (gof)(x) is continuous in the interval [a, b].


the numbers f(a) and f(b) are different from zero and have
25. If the function f is continuous everywhere and the function opposite signs, then there is atleast one point c on the open

g is continuous everywhere, then the composition gof is interval (a, b) such that f(c) = 0.
continuous everywhere. 35. If f is continuous on the closed interval [a, b] and k is any


26. All polynomials, trigonometric functions, inverse number between f(a) and f(b), then there is atleast one

trigonometric functions, exponential and logarithmic number c in [a, b] such that f(c) = k.
functions are continuous at all points in their domains. 36. A function continuous on a closed interval [a, b] assumes


27. If f(x) is continuous, then | f(x) | is also continuous. all the intermediate values lying between its least and
greatest values on [a, b].

28. A root of a continuous function is continuous, wherever it is

defined. That is, the composition h(x) = n g ( x ) = [g(x)]1/n 37. A continuous function whose domain is closed must have


of f(x) = n x and the function g(x) is continuous at a if g is, a range in a closed interval but it is not necessary that
if the domain is open then range is also open (range can
assuming that g(a) ≥ 0 if n is even (so that n g (a ) is defined).
be closed).
29. If the function y = f(x) is defined, continuous and strictly
38. Let f be a continuous strictly increasing function on

monotonic on the interval I, then there exist a single valued

ps
a closed interval [a, b] and let α = f(a), β = f(b). Then
inverse function x = g(y) defined, continuous and also
(i) the range of f for the closed interval [a, b] is the closed
strictly monotonic in the range of the function y = f(x).
el
interval [α,β], (ii) there exists a function x = g(y), the
30. Assume f is integrable on [a, x] for every x in
eh

inverse of f, which is one-valued, strictly increasing and



continuous on [α, β].
x

∫ f ( t ) dt . Then the integral A is


je

[a, b] and let A(x) =


A strictly decreasing function f(x), continuous

iit

a

continuous at each point of [a, b]. (At each endpoint we on [a, b] has an inverse, which is a strictly decreasing
continuous function on [β, α] where α = f(a), β = f(b).
@

have one-sided continuity)
31. If a function f(x) is continuous at every point of a closed 39. Let a strictly increasing function y = f(x) map the closed

interval [a, b] onto the closed interval [α, β], i.e. f([a, b])

interval [a, b], then the function f(x) is bounded on this
interval. Note that the boundedness of a function on the = [α, β]. Then f is continuous on [a, b].
interval [a, b] means that there is a number K > 0 such 40. Let f(x) be a function defined and bounded in the interval
that |f(x)| < K for all x ∈ [a, b].

[a, b], then, if M and m are the bounds of f(x), the number
32. If a function is continuous in a closed interval there exists M – m is called the span or oscillation of the function f(x)

atleast one point at which the function assumes the greatest in the interval.

Objective Exercises
Single Correct Answer Type

1. The function 5

1 − cos x (cos 2 x )1/ 2 (cos 3x )1/ 3 (A) f(0) = (B) [f(0)] = –2




f(x) = is not defined at x = 0. 2
x2

(C) {f(0)} = –0.5 (D) [f(0)]. {f(0)} = –1.5




If f(x) is continuous at x = 0 then f(0) equals
(A) 1 (B) 3 
| x 2  1 | 1 , x 1
If f(x) = 




(C) 6 (D) –6 3. , then it is continuous
| 2 x  3 |  | x  2 | , x  1






x  e x  cos 2 x for
2. If f(x) = 2 ,x 0 is continuous at x = 0, (A) R (B) R – {0}

x




then (C) R – {1} (D) none of these




FREE BOOKS FOR JEE & NEET =>(@iitjeeadv)
Continuity of Functions 2.53

 1  2  x + x 2 n sin x 
4. If f(x) = [x] +  x     x   then number of points 12. If g(x) = lim 
 3  3  then

n →∞  1 + x 4n 


of discontinuity of f(x) in [–1, 1] is (A) g(x) is continuous at x = 1
(A) 5 (B) 4



(B) g(x) is discontinuous at x = 1




(C) 7 (D) none



(C) limit does not exist at x = 0






 sin{x}
(D) none of these



 , {x}  0
5. If f(x) =  {x} , where [.] denotes fractional 3
13. Let f(x) = x – a sin πx + 3, –4 ≤ x ≤ 4. The value of f(x)

K, {x}  0



4
part function, then f(x) will be continuous 1999
(A) if K = 0 (B) if K = sin 1 is for some x ∈ [–4, 4] This statement is




(C) if K = 1 (D) for no value of K 199
(A) true (B) false




6. If f(x) is a continuous function from R → R and attains only





(C) true only if a ≥ 0 (D) true only if a ∈ [–4, 4]

100
 f (r) is equal to





irrational values, then 14. The ordered pair (a, b) such that


r 1
99 100 
(A)  f (r ) (B)  f ( 2r )  be x  cos x  x
 , x0




r 1 r 1  x2
100 f(x) =  a , x0

 f ( 2r )  1

(C) (D) none of these   
ps 1 x
  tan (e )  4 




r 1
2   x0
If f(x) = sgn (cos 2x – 2 sin x + 3), then f(x) ,
el
7.  x

(A) is continuous over its domain
eh


(B) has a missing point discontinuity becomes continuous at x = 0 is



(C) has isolated point discontinuity 1 
je

(A) (1, 1) (B)  ,1




(D) irremovable discontinuity 2 




iit


1 1 1
Let f(x) = (sin x )  2 x , x ≠ π/2 if f(x) is continuous at (D) not possible
@

8. (C)  , 
2 2





x = π/2 then f(π/2) is,
(A) e (B) 1  2 x  1 , x Q




(C) 0 (D) none 15. The function f(x) =  2 is
 x  2 x  5 , x  Q





x +1
9. If f(x) = , the number of points of discontinuity of (A) continuous no where

x


the composite function. y = f (f(f(x))) are (B) continuous at every rational point


(A) 0 (B) 1 (C) continuous at irrational points only






(C) 2 (D) 3 (D) continuous exactly at one point






 a | x2  x  2 |
16. Let [x] denotes the greatest integer less than or equal to

 , x2 x. If f(x) = [x cos x], the f(x) is
 2  x  x2
 (A) continuous at x = 0
10. Let f(x) =  b, x2



 (B) continuous in (–1, 0)

x  [x]


 , x2 (C) discontinuous at x = 1
 x2



 (D) continuous in (–1, 1)


If f(x) is continuous at x = 2 (where [.] denotes greatest

integer function) then (a, b) is 1 1
sin 4    sin 2    1
(A) (1, 1) (B) (1, 2) x x




(C) (2, 1) (D) (2, 2) 17. If f(x) = is to be made continuous
 1  1





11. The function f(x) = | 2 sgn 2x | + 2 has cos 4    cos 2    1
x x

(A) jump discontinuity
at x = 0, then f(0) should be equal to


(B) removable discontinuity


(C) infinite discontinuity (A) 0 (B) 1






(D) no discontinuity (C) 1/3 (D) 1/2






FREE BOOKS FOR JEE & NEET =>(@iitjeeadv)
2.54  
Differential Calculus for JEE Main and Advanced

lim (1 – sin2 x)n, n ∈ N, then (A) 0 (B) 2


18. If f(x) = n





(C) 4 (D) none






(A) f   is nearly zero 23. If graph of the function y = f(x) is continuous and passes
4


 



n (3f ( x )  2)
 through point (3, 1) then xlim
3 2(1  f ( x ))
is equal
1
(B) f   is equal to
4  


2 3 1
(C) f(x) is discontinuous at infinite number of points (A) (B)





2 2


(D) f(x) is a periodic


3 1
 a sin 2 n x (C) – (D) –
for x  0 and n  





2 2
19. Let f(x) =  then
 b cos 2 m x  1 for x  0 and m  

24. Let f(x) = the highest power of ( u x + u2 + 2u + 3). Then at
2


(A) f(0–) ≠ f(0+) (B) f(0+) ≠ f(0) x = 2 , f(x) is




(C) f(0 ) - f(0)

(D) f is continuous at x = 0 (A) continuous (B) lim f(x) = –2









x→ 2
20. f is a continuous function in [a, b]; g is a continuous (C) discontinuous (D) none of these





function in [b, c]
25. Let f(x) = [sin x + cos x], 0 < x < 2π, (where [.] denotes
A function h(x) is defined as


the greatest integer function) Then the number of points

h(x) = f(x) for x ∈ [a, b)
of discontinuity of f(x) is



= g(x) for x ∈ (b, c]
(A) 6 (B) 5


if f(b) = g(b), then




(C) 4 (D) 3



(A) h(x) has a removable discontinuity at x = b.
ps




26. If α, β(α < β) are the points of discontinuity of the



(B) h(x) may or may not be continuous in [a, b]
el

(C) h(b–) = g(b+) and h(b+) = f(b–) 1
function f(f(f(x))) where f(x) = , then the set of



eh

(D) h(b+) = g(b–) and h(b–) = f(b+) 1− x



21. If graph of |y| = f(x) and y = |f(x)| is as shown below values of ‘a’ for which the points (α, β) and (a, a2) lie on
je

the same side of the line x + 2y – 3 = 0, is



iit

 3 
(A)   3 ,1 (B)   ,1
@

 2 




 2 

(C) [1, ∞) (D)  ,  3 






 2
27. Let f : (0, ∞ ) → R be a continuous function such

f (x)
2≤ ≤ 3 for all x > 0. Then with certainity f(c) = c
x2
for atleast one point ‘c’ belonging to the interval (s)
1  1 1
(A)  , 1 (B)  , 
2  3 2



1 1
(D)  , 1
1
(C)  , 
8 4



4 
28. If f(x) = sgn(sin2x – sinx – 1) has exactly four points of

Then number of points of discontinuity of f(x) in [a, d] is discontinuity for x ∈ (0, nπ), n ∈ N then

(A) the minimum value of n is 5

(A) 1 (B) 2


(B) the maximum value of n is 6




(C) 3 (D) 4


(C) there are exactly two possible values of n




22. If x ∈ R+ and n ∈ N, we can uniquely write x = mn


(D) none of these

+ r where m ∈ W and 0 ≤ r < n. We define x mod


n = r. For example 10.3 mod 3 = 1.3. The number  1
29. f ( x )  lim sin 2 n (x )   x   , where [.] denotes the
 2

n
of points of discontinuity of the function f(x) =
(x mod 2)2 + (x mod 4) in the interval 0 < x < 9 is greatest integer function is

FREE BOOKS FOR JEE & NEET =>(@iitjeeadv)


Continuity of Functions 2.55

3 sin 2 x , x rational
(A) Continuous at x = 1 but discontinous at x =
2 35. Let f(x) =  sin 2 x , x irrational





3
(B) Continuous at x = 1 and x = Then set of points where f(x) is continuous -
2



3   
(C) Discontinuous at x = 1 and x = (A) (2n  1) , n  I 
 



2


2
3 (B) null set



(D) Discontinous at x = 1 but continuous at x = (C) {nπ, n ∈ I}


2



(D) set of all rational numbers
30. If f(x) is a continuous function ∀ x ∈ R and the



36. If the graph of the continuous function y = f(x) passes

 f (x) 


range of f(x) is (2, 26 ) and g(x) =  through (a, 0), then
 is
 a 

lim ln(1  6f ( x )  3f ( x ) is equal to
2
continuous ∀ x ∈ R (where [.] denotes the greatest integer x →a


3f ( x )

function), then the least positive integral value of a is
(A) 1 (B) 0
(A) 2 (B) 3





(C) – 1 (D) None




(C) 6 (D) 5




 1 




 axe x  b sin x 37. Let f be a continuous function on R such that f  n  =
 , x0
4 
31. Let f(x) =  x3 , where {.} represents n 2
 c cos {x} (sin en) e − n + 2 . Then f(0) is equal to

2

 , x  0 n +1
ps
fractional part function, if f(x) is continuous at x = 0, then (A) 0 (B) 1
el




the value of c is: (C) 2 (D) none




eh

  
(A) 4 (B) 2 38. Let f(x) = [tan x[cot x]], x ∈  ,  (where [.] represents
 12 2 





je

3 3
iit

1 the greatest integer function) then the number of points,


(C) (D) – 1 where f(x) is not continuous is




@

3
(A) one (B) zero
32. Let f R → R such that f(x) is continuous and




(C) three (D) infinite

attains only rational values at all real x and




39. If f: R → R and g : R → R such that

4
f(3) =4. If a1, a2, a3, a4, a5 are in H.P., then  a r a r 1 is  0 if x rational  0 if x irrational
f(x)= 1 if x irrational ; g(x) =  1 if x rational then

n 1
(A) f(3). a1a5 (B) f(3). a4a5  

(A) f + g is discontinuous



(C) f(3). a1a2

(D) f(2). a1a3






(B) f + g is continuous at rational only
2 x  1 , 2  x  0


(C) f + g is continuous everywhere
33. If f(x) = 


and (D) f + g is continuous for irrationals only
x2 , 0 x  2



1/ x
40. Let f(x) = g(x) e  e
1/ x
 [ x ] , 4  x  2 and x ≠ 0, where g is a
e1/ x  e 1/ x

g(x) =  then
 x  2 , 2  x  4

continuous function. Then lim f(x) exists if
x→0
(A) lim f(g(x)) = 2
 (A) g(x) is any polynomial


x2


(B) g(x) = x + 4
(B) f(g(x)) is discontinuous at x = –2


(C) g(x) = x2


(C) f(g(x)) is not defined at x = 2


(D) g(x) = 2 + 3x + 4x2




(D) none of these 41. The point of discontinuity of the function


34. The function defined by f(x) = [x2 + e1/(2–x)]–1 when x > 2

 2 sin x 2n

and f(x) = k when x = 2 is continuous in interval [2, ∞). f(x) = lim is
3n   2 cos x 
2n

x
Than k is equal to
(A) 0 (B) 1/4 (A) nπ ± π/3; n ∈ I (B) nπ ± π/6; n ∈ I








(C) –1/4 (D) None (C) 2nπ∈; n ∈ I (D) none of these








FREE BOOKS FOR JEE & NEET =>(@iitjeeadv)
2.56  
Differential Calculus for JEE Main and Advanced

42. The set of all points of discontinuity of the function  −1 , x < 0



 tan x log x  
48. Let f(x) =  0 , x = 0 and g(x) = sin x + cos x, then points
f(x) =   contains  1 , x>0


 1  cos 4 x  
n of discontinuity of f{g(x)} in (0, 2π) is
(A)  ; n  I  (B)  n ; n  Q 
 2 




 2 
(A)   , 3  (B)  3 , 7 





 n  2 4  4 4 
(C) (–∞, 0] ∪  ; n  N
 


2
 2 5 
(D) none of these (C)  ,  (D)  5 , 7 
 3 3 





 4 3 


43. Let f(x) = sin x if x is rational and f(x) = 1 – 2 cos x if x
2x 3 ,

is irrational then  x2
49. The value of ‘a’ for which the function f ( x )  
(A) f(x) is no where continuous x2
2
 ax ,





(B) f(x) is continuous at one point only  sin x  x2  a2
, x0


(C) f(x) is injective 4  , xa
g( x )   x and h ( x )   x a


(D) f(x) is continuous at infinite no. of points  a  2 x , x  0  xa


 8,
44. The function f(x) is defined by are all continuous is


 3 (A) 2 (B) 4
log ( 4 x 3) ( x  2 x  5) if  x  1 & x  1
2




f(x) =  4 (C) no value of a exists (D) none of these






4 if x  1 50. The set of all points of discontinuity of the
ps

(A) is continuous at x = 1  tan x log x 
el
function f(x) =  1  cos 4 x  contains


(B) is discontinuous at x = 1 since f(1+) does not exist  



eh

though f(1–) exists


 n 
(C) is discontinuous at x = 1 since f(1–) does not exist (A)  n , n  I  (B)  , n  Q
je


 2 




though f(1+) exists  2 
iit

(D) is discontinuous since neither f(1–) nor f(1+) exists.  n 


(C) (–∞, 0] ∪  , n  N


@

45. The function f defined by  2 





(D) none of these
 (1  sin x )  1 


t
f(x) = lim   is 0, if x is rational
t   (1  sin x ) t  1 
  

(A) everywhere continuous 51. Let f (x) = 
5x , if x is irrational




(B) discontinuous at all integer values of x


(C) continuous at x = 0 Then



(D) none of these (A) f (x) is continuous at every rational number.




46. The function f : R /{0} → R given by (B) f (x) is continuous at every irrational number.


(C) f (x) is discontinuous everywhere.

1 2


f(x) = − 2x can be made continuous at (D) f (x) is continuous only at x = 0.
x e −1



x = 0 by defining f(0) as x 3  2 x 2  9 x  18
52. Let R ( x ) 

(A) 2 (B) –1 x4  4




­
(C) 0 (D) 1
Which of the following statements describes the graph of





1 | x | y = R(x) :
 , x  1
47. If f(x) =  1  x , then f([2x]) is where (A) The graph has two vertical asymptotes


 1 (B) The graph has two holes in it

, x  1


(C) The graph has one hole and one vertical asymptote


[ ] represent greatest integer function (D) The graph has neither holes nor asymptotes

(A) continuous at x = –1




(B) continuous at x = 0 n sin x
x
53. Given that  cos


(C) discontinuous at x = 1/2 = n  x .
2n 2 sin  n 



(D) all of these n 1
2 


FREE BOOKS FOR JEE & NEET =>(@iitjeeadv)
Continuity of Functions 2.57

 n
1  x    4 − x2
 lim n tan  n  , x (0, )    58. The function f(x) = is:
  2  2 4x − x 3


n 1 2
n
Let f (x) = 

 2 (A) discontinuous at only one point
 , x



2 (B) discontinuous at exactly two points



Then which one of the following alternative is true? (C) discontinuous at exactly three points



(A) f(x) has non-removable discontinuity of finite

(D) none




π
type at x = .  tan kx
2  for x  0
π 59. Let f(x) =  x . If f (x) is continuous at
(B) f(x) has missing point discontinuity at x =


. 3x  2k 2 for x  0


2
π x = 0 then the number of values of k is
(C) f(x) is continuous at x = .
(A) more than 2 (B) 1


2





(D) f(x) has non-removable discontinuity of infinite type (C) 2 (D) none







π 60. If f(x) = lim x tan–1 (nx) ; then f(x) is
at x = .


2 n

2 4 x  16
2
(A) continuous at x ∈ I



54. The value of f(0) such that f(x) = is (B) discontinuous at x ∈ I
cos 2 x  1



continuous at x = 0 is : (C) continuous at all x ∈ R


(D) none of these
(A) 1/64 (B) − 1/64
ps






1
(C) 1/32 (D) − 1/32  1 
el
61. If f (x) = 3 + 1  71 x




55. Consider  then

 
eh

 

 2 sin x  sin 3 x  sin x  sin 3 x   
  (A) lim f ( x )  4
je

f(x) = 
   , x1


 2 sin x  sin x  sin x  sin x
3 3


iit

(B) lim f ( x )  3
x1


π
@

x≠ for x ∈ (0, π) (C) lim f ( x )  5



2 x1


f(π/2) = 3 where [.] denotes the greatest integer function (D) f has irremovable discontinuity at x = 1



then, x 1
 0,
(A) f is continuous & differentiable at x = π/2 62. Let f(x) =  . The number of solutions of
 2 x  2, x 1



(B) f is continuous but not differentiable at x = π/2


(C) f is neither continuous nor differentiable at x = π/2 the equation f(f(f(f(x)))) = x is
(A) 2 (B) 4


(D) none of these






(C) 5 (D) None
56. Given f(x) = b ([x]2 + [x]) + 1 for x ≥ −1







= [sin (π (x+a) ) ] for x < −1 (3x  1) 2
is not defined for



where [x] denotes the integral part of x, then for what values 63. The function
sin x . n (1  x )


of a, b the function is continuous at x = −1?
x = 0 for the function to be continuous at the point x = 0,
(A) a = 2n + (3/2) ; b ∈ R ; n ∈ I

f(0) must have the value :



(B) a = 4n + 2 ; b ∈ R ; n ∈ I
(A) e3 (B) 1


(C) a = 4n + (3/2) ; b ∈ R+ ; n ∈ I




(C) (ln 3)2 (D) none



(D) a = 4n + 1 ; b ∈ R+ ; n ∈ I




64. The value of f(0) so that the function

[ x ]  x  [ x ] for x  0

57. The function f(x) =  is 4  2x  x 2  4  2x  x 2
sin x for x  0

f(x) =
2x  2x

(A) continuous only for all non-negative integers


(B) continuous only for all positive integers is continuous at x = 0 is:

(A) 2 (B) 1


(C) discontinuous only for all negative integers






(D) cont. for all real numbers. (C) − 2 (D) −2 2






FREE BOOKS FOR JEE & NEET =>(@iitjeeadv)
2.58  
Differential Calculus for JEE Main and Advanced

log sin|x| cos3 x  [ x ] , −4 ≤ x < −2


π g(x) =  then
65. If f(x) = x , |x| < , x ≠ 0 = 4 , x = 0 then  x + 2 , −2 ≤ x ≤ 4


3

log sin|3x| cos
2
(A) lim f {g ( x )} = 2
the number of points of discontinuity is x →−2+




(A) 0 (B) 1 (B) f(g(x)) is discontinuous at x = – 2




(C) 2 (D) 4



(C) f(g(x)) is not defined at x = 2






66. Which one of the following functions defined below are (D) None of these




discontinuous at the origin? 68. The number of points of discontinuity of


 x cos 1 if x  0 f(x) = [4x] + {3x} in x ∈ [0, 5] is


(A) f (x) =  x (A) 20 points (B) 25 points
0 if x  0





(C) 30 points (D) 35 points






2 2
 x8  x 4  2 x 2 69. Let f(x) = [tan x][cot x] where [.] denotes greatest integer
if x  0


 function then number of points at which function f(x) is
(B) g (x) =  sin x
discontinuous in (0, 2π)


 0 if x  0
(A) 0 (B) 3





 sin x . cos 1 if x0 (C) 4 (D) 7




(C) h (x) =  x
70. h(x) is not a constant function and xlim  h (x )
0 if x0 c



exists finitely for all values of c ∈ R (where [.] denotes
 x 4  x8  2 x

if x  0 greatest integer function), then which of the following
(D) (x) =  tan x
ps statement is true ?


 0 if x  0 (A) lim  h ( x )   [lim h ( x )], ∀ c ∈ R
el
x c x c


(B) h(x) can not take any integral value
eh

2 x − 1 , −2 ≤ x < 0


67. If f(x) =  and (C) h(x) can take maximum two integral values
x+2 , 0≤ x ≤2



je

(D) None of these




iit
@

Multiple Correct Answer Type for JEE Advanced

71. If f(x) = [x2] + [x]2, then (where [.] denotes greatest integer (C) f (x) has a non removable discontinuity at x = 2.



function (D) f (x) is discontinuous at all positive integers.


(A) f(x) is discontinuous at x = 2 x 2 n 1  ax 2  bx


74. Let f (x) = lim . If f (x) is continuous
(B) f(x) is continuous at x = 51/4 x 2n  1

n 


(C) f(x) is continuous at x = 31/5
for all x ∈ R then


(D) f(x) is discontinuous at x = 0
(A) a = 0 (B) b = 0


1




72. Given the function f(x) = , the points of (C) a = 1 (D) b = 1
(1 − x )





75. Which one the following function(s) is/are continuous

discontinuity of the composite function y = f 3n(x), where ∀x∈R


f n(x) = fof... of (n times) are
ex + 1
(A) 0 (B) 1 (A) 2sin x + 3 (B)
ex + 3








(C) 3n (D) 2




5/ 7
 2x  sgn x +1
 1
x  x[ x ] if x  0 (C)  2  1  (D)
 3x 
73. Let f (x) =   x  5




where [x] 2

 0 if x  0 76. Which the following equations have roots ?

denotes the greatest integer function, then the correct (A) cos x – x + 1 = 0



statements are (B) x5 – 18 x + 2 = 0, x ∈ [–1, 1]
(A) Limit exists for x = – 1.


(C) x2x = 1, x ∈ (0, 1)


(B) f (x) has a removable discontinuity at x = 1.


(D) x3 – 3x + 1 = 0, x ∈ [1, 2]




FREE BOOKS FOR JEE & NEET =>(@iitjeeadv)
Continuity of Functions 2.59

77. f(x) = x4 – 14x3 + px2 + qx – 105 is continuous ∀ x ∈ R then which of the following hold


g(x) = x4 + ax3 + bx2 + cx + 105 good?

the smallest root of f(x) = 0 is α and remaining root are (A) d = 4c (B) a ≠ b






in A.P. If the smallest root is increased by 2 then equation 5
(C) a + b + d = –3 (D) a + b + c + d = –





f (x) 2
becomes g(x) = 0 function F(x) = then 82. The function defined as


g( x )
(A) Domain of F(x) is R – {1, 3, 5, 7} and range is  2n 
 cos x if x<0 


R – {1, 2, 3 , 3 } n 
 1+ x 0 ≤ x ≤ 1
n
2 4 lim if
(B) Function F(x) is bijective f (x) = n  


1
 x >1 


(C) Function F(x) has removable discontinuities at x = 3, if
 1 + x n 


5, 7
(D) Function F(x) has irremovable discontinuity at x = 1 Which of the following does not hold good?



78. Let f(x) and g(x) be defined by f(x) = [x] and (A) continuous at x = 0 but discontinuous at x = 1




x  I , where [ . ] denotes greatest (B) continuous at x = 1 but discontinuous at x = 0
g(x) =  0 ,



 2 (C) continuous both at x = 1 and x = 0
 x , other wise


(D) discontinuous both at x = 1 and x = 0


integer function. Then
83. Which of the following functions defined below are
(A) gof is continuous for all x ∈ R
continuous for every x ∈ R ?


(B) lim . fog(x) = 3
 sin x


x→2
(C) fog is continuous for all x ∈ R
ps if x  0



(D) lim . fog(x) = 5 (A) f (x) =  x
el


 x  1 if x  0


x→5
  
eh

79. If f(x)=cos   cos  2 ( x  1)  where [x] is the greatest  2


 
x   1
 x sin x if x  0

je

integer function of x, then f(x) is continuous at (B) g (x) = 




iit

(A) x = 0 (B) x = 1  0 if x  0





@

(C) x = 2 (D) none of these (C) h (x) = sin | x |







80. Which of the following statement(s) is/are correct? (D) k (x) = |1 – x + | x ||


(A) Let f and g be defined on R and c be any real 84. Which of the following functions is/are continuous



number. If lim f ( x ) = b and g (x) is ∀x∈R?
x →c
(A) f (x) = cos(x2 – 2)

continuous at x = b then xlim


→c
g(f(x)) = g (b).

x
(B) There exist a function f : [0, 1] → R which is (B) f (x) =
1 − sin 2 x




discontinuous at every point in [0, 1] and |f(x)| is
continuous at every point in [0, 1].  sin 2 x
(C) If f (x) and g (x) are two continuous function defined  , if x  0
(C) f (x) =  x


from R → R such that f (r) = g (r) for all rational 0


 if x  0
numbers 'r' then f (x) = g (x) ∀ x ∈ R.
(D) If f (a) and f (b) possesses opposite signs then there  sin 2 x
 , if x  0


must exist atleast one solution of the equation f (x) = 0
(D) f (x) =  sin 4 x
in (a, b) provided f is continuous in [a, b]. 


2 if x  0
 sin ax
 bx , x0 85. Which of the statement(s) is/are incorrect?

 (A) If f + g is continuous at x = a, then f and g are
ax  1 , 0  x 1


continuous at x = a.
 2
cx  2 , 1 x  2 (B) If lim (f g ) exists, then lim f and lim g both exists.


81. If f (x) =  x →a x →a x →a
 d( x  4) ,
2
2x4 (C) Discontinuity at x = a ⇒ non existence of limit



 x (D) All functions defined on a closed interval attain a



12 , x4 maximum or a minimum value on that interval.

FREE BOOKS FOR JEE & NEET =>(@iitjeeadv)


2.60  
Differential Calculus for JEE Main and Advanced

(C) A is true, R is false.


 tan −1 (1 x) if x ≠ 0


86. Let f (x) =  (D) A is false, R is true.
if x = 0


1

 1 / x  * x  0
then which of the following do/does not hold good? 91. Let g(x) =  , where <r>* is the distance
 0 x0



(A) f is continuous on (– ∞, 0) ∪ (0, ∞).


(B) f has a non removable discontinuity of finite type at from x to the integer nearest to x then


x = 0. Assertion (A) : g is discontinuous at x = 0


(C) f has a non removable discontinuity of oscillatory 2
Reason (R) : Let xn = 1/n and x'n = ;


type at x = 0. 2n + 1


(D) f has a non removable discontinuity of infinite type 2n + 1 *


at x = 0. g(xn) = <n>* = 0, g(x'n) =
2


87. Let f(x) and g(x) be defined by f(x) = [x] and
= <n + 1/2>* = 1/2

 0 , xI


 x{x}  1 0  x  1
g(x) =  2 , where [ . ] denotes 92. Consider the function f(x) = 
 x , other wise  2  {x} 1  x  2


greatest integer function. Then where {x} denotes the fractional part function.


(A) gof is continuous for all x ∈ R Assertion (A) : f(x) is continuous in [0, 2]



(B) lim . fog(x) = 3
Reason (R) : lim f(x) exists


x→2

x→1
(C) fog is continuous for all x ∈ R 93. Assertion (A) : The function | ln x | and ln x are both



(D) lim . fog(x) = 5 continuous for all x > 0.
ps


x→5
Reason (R) : Continuity of | f (x) | ⇒ Continuity of f (x).
88. The function

el

f(x) = max ({x}, {–x}), x ∈ (–∞, ∞) is 94. Assertion (A) : If f(x) is a continuous function such that

f(0) = 1 and f(x) ≠ x, ∀ x ∈ R, then f(f(x)) > x.
eh

(A) continuous every where
Reason (R) : If f : R → R, f(x) is an onto function then


(B) continuous at x = 0
je

f(x) = 0 has atleast one solution.


(C) continuous for all x = n + h where n ∈ Ι, h = 1/2
iit


(D) continuous every where except all x ∈ Ι  1
95. Let f (x) = cos  x cos 


 
@


89. f(x) is continuous at x = 0, then which of the following x

are always true ? Assertion (A) : f (x) is discontinuous at x = 0.

(A) lim f(x) = 0 Reason (R) : lim f ( x ) does not exist.




x→0 x0
(B) f (x) is non continuous at x = 1 96. Assertion (A) : Let f : [0, 1] → R be a continuous function.




(C) g(x) = x2 f(x) is continuous at x = 0 There cannot exist such a function f which crosses the x


(D) lim (f(x) – f(0)) = 0 axis infinitely often.



x0 
1
 e[ x ]  e[ x ]  x sin , x  0
 x
, x0 Reason (R) : The function f(x) =  x crosses

 e  0, x0
90. The function f(x) =  sin{x} , the x-axis infinitely often.
 {tan x} , x  0

 97. Assertion (A) : Let f(x) = x2 + x4 + x6 + x8 + ....., for all

 2 , x0 real x such that the sum converges. The number of fixed
where [ . ] and { . } represent greatest integer and points of the function is two.
fractional part functions respectively, is

x2
(A) continuous at x = 0 Reason (R) : Since f(x) = , f(x) = x
1− x 2




(B) discontinuous at x = 0 ⇒ x2 + x – 1 = 0. This gives two values of x.
π



(C) continuous at x = 98. Consider the functions



6 f(x) = sgn (x – 1) and g(x) = cot–1[x – 1]
π

(D) discontinuous at x = where [ . ] denotes the greatest integer function.
4



Assertion (A) : The function F(x) = f(x) . g(x) is

Assertion (A) and Reason (R) discontinuous at x = 1.
(A) Both A and R are true and R is the correct explanation of A. Reason (R) : If f(x) is discontinuous at x = a and g(x) is


(B) Both A and R are true but R is not the correct also discontinuous at x = a then the product function f(x).

explanation of A. g(x) is discontinuous at x = a.

FREE BOOKS FOR JEE & NEET =>(@iitjeeadv)


Continuity of Functions 2.61

107. The fundamental period of the function f(x) is


 x  1 0x2


99. Let f (x) = sgn x and g (x) =  , then (A) 2 (B) 4
1  x 2x4





2

(C) 8 (D) None





Assertion (A) : The function (fog)(x) is continuous at
108. xlim f(x) is equal to

x = 2. 1 


Reason (R) : If (fog) (x) is continuous at x = a then g (x) 1
(A) (B) 3

is continuous at x = a and f (x) is continuous at x = g(a).





3 4
100. Assertion (A) : f (x) = [1 + cos x] where [x] denotes

1
greatest integer function is discontinuous at x = π (C) (D) 5





Reason (R) : f (x) = [x] where [x] denotes greatest integer 2 12

function is discontinuous at all integers. 109. The number of points of discontinuity in the function


y = f(x) over the interval [0, 4] is:
Comprehension - 1 (A) 0 (B) 1





(C) 2 (D) 3
 1 





If f(x) = maximum  cos x , ,{sin x}  , 0 ≤ x ≤ 2π, where { . }
 2  Comprehension - 4
represents the fractional part function, then Consider a ∈ R+, f(x) = | x – a |
101. The number of points where f(x) is equal to 1/2 is
  x 

(A) 1 (B) 2 for x  a
 f ( x ) sec  




(C) 4 (D) infinite   2a 
g (x) = 




102. The number of points of discontinuity of f(x) is  cot  x  cos ec(f ( x )) for x  a
ps
 2a 

(A) 1 (B) 2   




el
(C) 3 (D) 4




eh

103. The number of points where f(x) has non-removable h (x) = lim
n

discontinuity is
je

(A) 0 (B) 1 110. The value of g(a) so that g is continuous at x = a







iit

(C) 2 (D) 3 (A) is –


2a
(B) is
a




π





@

Comprehension - 2 (C) is 1 (D) can not be determined






A strictly monotonic polynomial function f : (0, ∞) → (0, ∞) 111. The number of possible ordered pairs (a, g ( a ) ) is
 x2 

(A) 0 (B) 1
is such that f   ≡ x.




 f (x)  (C) 2 (D) more than 2




6
104. If f(1) = 2 then the value of f(2) is
112. If lim (cos( x − a )) ( x −a )sin( 2 x − 2a ) = e–kh([a]) then k equals

(A) 8 (B) 4

x →a




(C) 16 (D) None (where [.] denotes greatest integer function)





105. The number of fixed points of f (i.e. number of solution (A) 0 (B) 2/3




(C) 3/2 (D) 6

of f(x) = x) is




(A) 3 (B) 2 Comprehension - 5




(C) 1 (D) None 1 , x  0
Let the Heaviside step function be defined as H(x) = 




ln f ( x )  ln f (1) 0 , x  0
106. If lim exists with nonzero value, then
sin n x 113. The solutions of the equation x2 – x H(x) – 2 = 0 are

x 1

the value of n is (A) {–1, 2} (B) {– 2 , –1, 2}




(A) 1 (B) 2
(C) {± 2 , – 1, 2} (D) {– 2 , 2}




(C) 3 (D) None




114. The solution set of the inequality H(1 – x2) > |sin π/2 x| is





Comprehension - 3      
Define f : R → R by f(x) = x2 + 1 for –1 < x ≤ 1 and (A)   ,  (B)   , 
 2 2  2 2




1
f(x + 2) = for all x ∈ R.   
f (x) (C) R   ,  (D) None of these
 2 2




FREE BOOKS FOR JEE & NEET =>(@iitjeeadv)
2.62  
Differential Calculus for JEE Main and Advanced

Match the Columns for JEE Advanced

115. Column - I Column - II




 1  3 cos x
 , x0
 x2
(A) If f(x) =  is continuous at x = 0, (P) |a + b| = 0
 b tan    , x  0




  
 [ x  3] 
then (where [ . ] denotes the greatest integer function)

 
 2 sin x ,   x   2

  
(B) If f(x) = a sin x  b,   x  is continuous in [– π, π],then (Q) |a – b| = 2




 2 2
 
 cos x , 2  x  

  3 (cos 3x )/(cot 2 x ) 
   , 0x
 2 2
 
ps
(C) If f(x) =  b3 , x (R) [a – 2b] = –2
 2




el
  a |tan x | 
(1 | cos x |) b  
eh

, x
 2
je

π
iit

is continuous at x = , then
2

@

 sin[ x ]
 a , x0
 x
(D) If f(x) =  2, x0 (S) |a + 2b| = 4




b   sin x  x  , x  0
 
  x 
3

(where [.] denotes the greatest integer function), (T) |a – b| = 1





is continuous at x = 0, then b is equal to


116. Column – I Column – II



1 1
(A) Given f(u) = 2 , where u = , then f(x) is (P) continuous at x = 0
u u2 x −1




(B) If f(x) = sgn x (1 – x ), then f(x) is
2
(Q) discontinuous at x = 1, 1/2, 2




f (x)
(C) If lim exists and f(0) = 0, then f(x) is (R) discontinuous function
x →0




x
nx  sgn x
(D) The function f(x) = lim is (S) discontinuous at x = 2
1 n



n 

117. Column - I Column - II




(A) The number of natural numbers less than the (P) 1




fundamental period of sin2x + sec2x – tan2 x is

(B) The number of points of discontinuity of the function (Q) 2




f(x) = [x] + {2x} + [3x] for x ∈ [0, 1], where [ . ] and

{ . } represent greatest integer and fractional part functions is

FREE BOOKS FOR JEE & NEET =>(@iitjeeadv)
Continuity of Functions 2.63

 sin x (1  cos x ) 
(C)  xlim  , where [ . ] represents (R) 3
  




0 x cos x
greatest integer function, is equal to

(D) The number of solutions of the equation (S) 4




–1 –1
sin x – 2 cos (1 + x) = 0 is

118. Column-I Column-II


(A) In a ∆ABC maximum value of cos2A + cos2B + cos2C, is (P) 3/4




(B) If a, b are c are positive and 9a + 3b + c = 90 then


the maximum value of (log a + log b + log c) is (Q) 2



(base of the logarithm is 10)

(C) tan x tan x  sin x sin x equals (R) 3
lim




x0 x3 · x
1 ln(sec 2 x )
(D) If f (x) = cos(x cos ) and g (x) = are (S) non existent




x x sin x
both continuous at x = 0 then f (0) + g (0) equals

119. Column I Column II


1
ps
(A) If f(x) = nlim , then (P) f(x) is continuous ∀ x ∈ R
 1  n sin 2 x




el
x
eh

lim
(B) If f(x) = n  1  ( 4 sin 2 x ) n , then (Q) f(x) is discontinuous at x = 1




je

 x 2 n 1  ax 2  bx 
iit

(C) If h(x) = nlim   is continuous


  x 2n  1
(R) f(x) is discontinuous at
 
@




π
∀ x ∈ R, and f(x) = [tan(a + b – 1)x], then x = 2nπ +


6

(D) If f(x + y) = f(x). f(y), ∀x, y ∈ R and (S) f(x) is continuous at x =




6
f(x) = 1 + g(x).G(x); lim g ( x )  0 and

x 0

lim G ( x ) is finite real number, then


x →0

Review Exercises for JEE Advanced
1. Test the continuity of the function 4. If f(x) be defined as


 1  sin x
 x0
f(x) =  x   [ x ] , in the interval –2 ≤ x ≤ 2. Also
,
 2 f(x) =  x

1  x cos x , x  0

draw the graph of y = f(x), where [.] denotes greatest
  3 

integer function. then discuss the continuity y = [f(x)] in   ,  where
 2 2

2. Discuss the continuity of the function f(x) = [[x]] – [x – 1], [.] denotes G.I.F.

2
where [·] denotes the greatest integer function. x
5. Let f(x) = + 1. 0 ≤ x < 1
Let f(x) be defined in the interval [–2, 2] such that

3. 2


f(x) = 1, –2 ≤ x ≤ 0 3
= 2x2 – 3x + , 1 ≤ x ≤ 2.
= x – 1, 0 < x ≤ 2

2

and g(x) = f(|x|) + |f(x)|.Test the continuity of g(x) in [–2, 2]. Discuss the continuity of g(x) = f(x) + f(x – 1).



FREE BOOKS FOR JEE & NEET =>(@iitjeeadv)
2.64  
Differential Calculus for JEE Main and Advanced

6. Discuss the continuity of the function 16. Prove that, if f(x) is continuous on (a, b) and x 1, x2,


(1  cos x ) n  5nx ......... xn are some values of x from this interval, then we
f(x) = nlim
 2  (1  cos x ) n
can find x = c, c ∈ (a, b) such that f(c) = (1/n) [f(x1) +

f(x2) + .............. + f(xn)].
n
x
7. Let fn(x) = cosn x and g(x) = lim  f k  4  . If g(x) is | x  1 | , x  0
17. Let f(x) = 

x
k 0 ;
 x , x0


continuous is (0, c), then find the largest value of c.
8. Examine the function f defined on R by setting | x | 1 , x 1
g(x) =  .

 | x  2 | , x  1


e1/ x sin(1 / x )
f(x) = , when x ≠ 0, f(0) = 0 Discuss the continuity of f + g.
1 + e1/ x


2 x − 1, − 2 ≤ x < 0
for points of discontinuity, if any. 18. Let f(x) =  and
 x + 2, 0 ≤ x ≤ 2


9. Examine each of the following functions for points of

discontinuity and the nature of discontinuity : [ x ], − 4 ≤ x < −2
(i) f(x) = (x – [x])2, for all x ≥ 0 g(x) = 
 x + 2, − 2 ≤ x ≤ 4




(ii) f(x) = [x] + (x – [x])2, for all x ≥ 0
Discuss the continuity of fog(x) over its domain.


10. If the function
19. Discuss the continuity of f : R+ → R defined as :

 ( x  2)3 
f(x) =  a  sin(x–2) + a cos(x–2) where [.]  1  p2 
1/ 2
  f(x) = x when x is irrational, f ( x )  


ps 

1  q
2
denotes the greater integer function, is continuous in [4, 6], 

then find the values of a. when x is rational number of the form p/q.
el

11. Find the value of a & b for which 20. Use the Intermediate Value Theorem to show that there is
eh


 a right circular cylinder of height h and radius less than r
je

(sin x  cos x )cosec x ,  1 / 2  x  0 whose volume is equal to that of a right circular cone of

iit

height h and radius r.


f(x) =  a , x0
21. Use Intermediate Value Theorem to locate all
@

 1/ x

e  e  e

2/ x 3 /| x |
x
, 0  x  1/ 2 discontinuties of the function f(x) = 3 .
 ae3/ x  be3/|x| x  3x  1

is continuous at x = 0. 22. Prove that if a and b are positive, then the equation


2x, − 1 ≤ x ≤ 0 a

b
 0 has atleast one solution in the
 x 1 x  3
12. Prove that the function f(x) =  2 x + 1

 2 , 0 < x ≤ 1 interval (1, 3).


is discontinuous at x = 0 but still has both maximum and 23. Let f(x) = x(1–x2), x ∈ R and


minimum values on [–1, 1]. | x |
 , x0
 n e1/ x g(x) =  x
x , x0 

 0 , x0
 1  e
1/ x

, x  0 . Find the smallest Discuss the continuity of (fog) x and (gof) x.


13. Let f(x) = 0

 24. Show that any continuous function defined for all real x

1
 x n sin , x0

and satisfying the equation f(x) = f(2x + 1) for all x must
 x
be a constant function.
n ∈ W such that f(x) is continuous.
25. Let f(x) be a continuous function in [–1, 1] and satisfies

 x 2  ax  1 , x  Q

f(2x2 – 1) = 2x f(x) ∀ x ∈ [–1, 1]. Prove that f(x) is
14. If f(x) =  2 is continuous at x = 1 & e
ax  2 x  b, x  Q identically zero ∀ x ∈ [–1, 1].

then a & b. [f ( x )] , x  (0,  / 2)  ( / 2, )
26. If g(x) =  and
x2  3 , x  /2

2 ln x
15. Let f(x) = , g(x) = . Prove that there exist a
ex

x 2(sin x  sin n x ) | sin x  sin n x |
f(x) = , n∈R
point ‘c’ between 1 and e such that f(c) = g(c). 2(sin x  sin n x ) | sin x  sin n x |

FREE BOOKS FOR JEE & NEET =>(@iitjeeadv)
Continuity of Functions 2.65

where [.] denotes the greatest integer function. Prove that that f is continuous at x0 = 0. Is f discontinuous at any

g(x) is continuous at x = π/2 when n > 1. point in R?

27. Prove that the function f(x) = (−1) , where [.]


[x ] 3
32. Consider the function f : R → R defined by



log(2  x )  x 2 n sin x
denotes the greatest integer function, is discontinuous for f ( x )  lim

x = n1/3, n ∈ I. 1  x 2n
n 


28. Let f be a continuous function in [0, 4] and f(0) = f(4). in the interval [0, π/2] and explain why the function does


not vanish anywhere in this interval, although

Prove that these exists point x = c ∈ [0, 2] such that
f(c) = f(c + 2). 1 
f(0) and f    differ in sign.
2 


29. Let f be a function such that f(xy) = f(x) f(y3) for all x and y.
33. A continuous function f : R → R satisfiesf(x + f(x)) = f(x)

If f(x) is continuous at x = 1, show that f(x) is continuous


at all x ≠ 0. ∀ x ∈ [0, 1]. Prove that f is constant.
| x  1 |; x  0 34. Let f be a non-zero function whose domain is the set of


30. If f(x) =  all real numbers satisfying
 x; x  0 and
f(x + h) = A f(x) f(h) (A ≠ 0). If f(0) ≠ 0, and f


| x | 1; x 1 is continuous at x = 0 then show that f is a continuous
g(x) = 
function.
 | x  2 |; x 1

Draw the graph of f + g and discuss its continuity. 35. Let f be a continuous function on R. If f(1/3n) = (cos en)


31. Let f(x) = x, if x is not the reciprocal of a positive integer; n2
3 n  then find f(0).
ps 2

but f(x) = x2 if x = 1/n for some positive integer n. Show n2  n 1

el

Target Exercises for Jee Advanced


eh
je

1 , | x | 1 5. Let f(x) = x when x is rational



iit

 1
 1 1 = 1–x when x is irrational.
| x | < , n = 2, 3,..... prove that

1. Let f(x) =  2 , Show that f(x) assumes every value between 0 and 1
@

n n −1

n


0 , x  0 once and once only as x increases from 0 to 1, but is
f(x) is discontinuous at infinitely many points. 1
discontinuous for every value of x except x = .
 (1  x ) 1/ x  (1  x )1/ x 2
 , 1  x  0 6. Let f be a continuous functions on [–1, 1] such that (f(x))2 +
 x

2. Let f (x) = e, x0 x2 = 1, for all , x ∈ [–1, 1]. Show that either f(x) = 1 − x 2

 for all x in [–1, 1] or f(x) = – 1 − x 2 for all x in [–1, 1].
(logsin x sin 2 x )
ln sin 2 x
, 0  x 1
 x2 x2 x2
Examine the continuity of f(x) at x = 0. 7. Let yn(x) = x2+ + +....+
1+ x 2 (1 + x )2 2
(1  x 2 ) n 1


3. Let f(x) = x – 9x + 15x + 7, and
3 2

and y(x) = lim yn(x)
(min f ( t ) : 0 ≤ t ≤ x ), 0 ≤ x ≤ 6

n
g(x) =  Discuss the continuity of yn(x)(n = 1, 2, 3,.....n) and y(x)
 x − 24 , x>6


at x = 0
Draw the graph of g(x) and discuss the continuity of g(x).
8 Show that the function


4. Discuss the continuity of the function 1 1 1
f(x) = – x + [2x]– [1 – 2x] assumes every

 2( x  x 3 ) | x  x 3 |

2 2 2
 , x0 value between 0 and 1 once and once only as
 2( x  x )  | x  x |
3 3

x increases from 0 to 1, but is discontinuous for
 1 1
, x0 x = 0, x = and x = 1.
f(x) = 

3 at x = 0,1. 2


 3 , x 1 9. Let I be a closed and bounded interval on the line and let

 f be continuous on I. Suppose that for each x ∈ I, there
 exists a y ∈ I such that

FREE BOOKS FOR JEE & NEET =>(@iitjeeadv)


2.66  
Differential Calculus for JEE Main and Advanced

1 (b − a) 2
|f(y)| ≤
b
2
|f(x)|. ∫a f(x)dx − (b − a)f(a) ≤

2
Prove the existence of a t ∈ I such that f(t) = 0. (iii) More generally, prove that for any c in [a, b], we have

10. Let f : R → R be a continuous function such that



(b − a) 2

lim f(x) = 0 = lim f(x). Prove that if f is strictly b
∫a f(x)dx − (b − a)f(c) ≤

x x
2
negative somewhere on R then f attains a finite absolute
20. Let f be continuous and strictly monotonic on the positive

minimum on R, and that if f is strictly positive somewhere


on R then f attains a finite absolute maximum on R. real axis and let g denote the inverse of f. If a1 < a2 < ... <
11. Let f : [0, 1] → [0, 2] be continuous. Show that there exists an are n given positive real numbers, we define their mean
value (or average) with respect to a f to be the number Mf

a point x ∈ [0, 1] such that f(x) = 2x.
defined as follows:
12. Let f : [0, 1] → R be a continuous function such that f(0)
1 n 

= f(1), then prove that there is a solution Mf = g   f ( a i ) 
n 


 1 n 1  i 1 
of the equation f(x) – f  x   = 0, in  0, for
   n 

n 1 n
every natural number n . Prove that (i) f(Mf) =  f (a i ) (ii) a1 < Mf < an
n i 1

13. How many continuous functions are there on R which
If h(x) = af(x) + b, a ≠ 0, then show that Mh = Mf.

satisfy (f(x))2 = x2 for all x ∈ R ? 21. If f is a function, then by a chord of f we shall mean a line

14. Let f be continuous on [a,b], let f(x) = 0 for exactly one segment whose ends are on the graph of f. Now let f be
ps
continuous throughout [0,1] and let f(0) = f(1) = 0.

c in [a, b] and let f(x) > 0 for some x such that c < x ≤ b
(i) Explain why there is a horizontal chord of f of
el
and let f(x) < 0 for some x such that a ≤ x < c. What can


we say about f for all x in [a,b].
eh

length 1 .
2
15. Let f be a function that satisfies the conclusion of the
(ii) Explain why there is a horizontal chord of f of
je

intermediate value theorem on a closed interval I and let f


length 1/n, where n = 1, 2, 3, 4,....
iit

be injective on I then prove that f must be continuous on I.


2
16. A function f is said to satisfy a Lipschitz condition on a (iii) Must there exist a horizontal chord of length
@

?



3

given interval if there is a positive constant M such that
(iv) What is the answer to (iii) if it is given that f(x) ≥
|f(x) – f(y)| < M | x – y |.


0 for all x in [0, 1]
for all x and y in the interval (with x ≠ y). Suppose f
22. Classify the points of discontinuity of the function f(x) =

satisfied a Lipschitz condition on an interval and let c be

a fixed number chosen arbitrarily from the interval. Use pnx, where pnx denotes the positive or negative excess of
limit to prove that f is continuous at c. x over the nearest integer; when x exceeds an integer by
1/2 let pnx = 0.
17. Find the points of discontinuity of the function
23. Discuss the continuity at x = 1 of the functions :

f : (0, ∞) → R where

x  x n sin x
 1 p (i) f(x) = lim ;
if x Q  (0, ), x = in lowest terms n  1 xn



f(x) =  p  q q
 x  xn

0 if x  (0, )  Q (ii) φ(x) = lim .
n  1  ax n


18. Let f, g be continuous function from [0, 1] to n

[2rx ]
[0, 1] such that 24. If f(x) = lim
n2

, discuss the continuity of f(x) where
n 

f(g(x)) = g(f(x)) ∀ x ∈ [0, 1]. r 1

Prove that f and g have a common fixed point in [0, 1]. [.] denotes the greatest integer function.

25. Consider the functions y(x) = sinx and zk(x) = ke–x. Now
19. Let f be a function such that |f(u) – f(v)| ≤ |u – v| for all u

define a new function f by f(k) = {smallest positive

and v in an interval [a, b].
solution of y(x) = z k(x)} Explain why the function
(i) Prove that f is continuous at each point of [a, b].
f(k) is not continuous on the interval 0 ≤ k ≤ 10.


(ii) Assume that f is integrable on [a, b] prove that


FREE BOOKS FOR JEE & NEET =>(@iitjeeadv)
Continuity of Functions 2.67

26. Let f and g be continuous on an interval I, and let (f(x))2 Consider another function g(x); such that


(g(x))2 = 1 for all x ∈ I. Prove that either f(x) = 1/g(x) for g(x) = f(x) for x ≥ 0



all x ∈ I or f(x) = –1/g(x) for all x ∈ I. = 2 2 f(x) for x < 0


27. A function f is defined on [0, 1] as follows : Discuss the continuity of other functions f(x) & g(x) at



1 1 1 x = 0.
f(x) = n when n 1  x  n (n ∈ W), f(0) = 0
31. Let f be the function defined on [0,1] by setting

2 2 2


2 3
1 1 1 f(x) = 2rx, when 1 1
 x  , r = 1, 2, 3, ....
Show that f is discontinuous at the points
2  2   2 
, , ,....
r 1



r
and examine the nature of discontinuity. f(0) = 0, f(1) = 1.


Examine for continuity the function f at the points


a sin x − a tan x 1 1 1
28. f(x) = for > 0 1, , , ......., ,......., and 0 .
tan x − sin x



2 3 r
ln(1  x  x 2 )  ln(1  x  x 2 ) 32. Let f be defined on R by setting
= for x < 0.


sec x  cos x

t, if 0  t  12
If f is continuous at x = 0, find ‘a’. 

f(t) = 0, if t  12
 x
Now if g(x) = ln  2   . cot (x – a) for x a, a > 0. If


 t  1, if 2  t  1
1
 a

g is continuous at x = a then show that g(e–1) = –e.
ps
and f(n+t) = f(t), when n is any integer.
29. Given the function g(x) = 6 − 2 x and
el

Determine the points of discontinuity of f.

h(x) = 2x2 – 3x + a. Then

eh

33. Sketch the graph of the function y = f(x), where
(i) evaluate h(g(2))

je


g ( x ), x  1 ln(2 + x ) − x 2 n sin x
(ii) If f(x) =  , find ‘a’ so that f is continuous.
iit

f(x) = lim
h ( x ), x  1 1 + x 2n

n →∞


@

30. Let 1

 
in the interval 0 ≤ x ≤ π and explain why the function
2  2

1 1
  2  sin (1  {x} )   sin (1  {x})
   for x  0 does not vanish anywhere in this interval, although f(0)
f(x) =  2 ({x}  {x}3 ) 1 
 and f    differ in sign.

  2 
for x  0
 2

where {x} is the fractional part of x.



Previous Year's Questions (JEE Advanced)

A. Fill in the blanks: 2. A discontinuous function y = f(x) satisfying x2 + y2 = 4 is



given by f(x) =.......... [IIT - 1982]

 ( x 3  x 2  16 x  20)

 , if x  2
1. Let f(x) =  ( x  2) 2   
3. Let f(x) = [x] sin   , where [ . ] denotes the greatest


 [ x  1] 

 k, if x  2
If f(x) is continuous for all x, then k =....... integer function. The domain of f is ....... and the points of
[IIT - 1996]

[IIT - 1981] discontinuity of f in the domain are.......


FREE BOOKS FOR JEE & NEET =>(@iitjeeadv)
2.68  
Differential Calculus for JEE Main and Advanced

4. Let f(x) be a continuous function defined for 1 ≤ x ≤ 3. π


(A) f(x) is discontinuous at x =

If f(x) takes rational values for all x and f(2) = 10, then



2
f(1.5) = ............... [IIT - 1997] π

(B) f(x) is continuous x =
B. Multiple Choice Questions with ONE correct



2

answer: (C) f(x) is continuous at x = 0



(D) None of these
ln(1  ax )  ln(1  bx )



5. The function f(x) = is not defined 11. For every integer n, let an and bn be real numbers. Let

x


at x = 0. The value which should be assigned to f at x = 0 function f :  →  be given by
so that it is continuous at x = 0, is 
[IIT - 1983]
(A) a – b (B) a + b a  sin  x , for x  [2n , 2n  1]
f (x)   n ,




(C) ln a – ln b (D) none of these
b n  cos  x , for x  [2n  1, 2n ]






 2x  1  for all integers n?
6. The function f(x) = [x] cos   π, [ . ] denotes the


 2 

(A) an–1 – bn–1 = 0 (B) an – bn = 1





greatest integer function, is discontinuous at
(C) an – bn + 1 = 1 (D) an–1–bn = –1
[IIT - 1995]





[IIT - 2012]

(A) All x (B) All integer points
12. For every pair of continuous functions f, g:[0, 1] → R such




(C) No x (D) x which is not an integer





7. The function f(x) = [x] – [x2] (where [y] is the greatest
2 that
max {f(x):x ∈ [0, 1]} = max {g(x):x ∈ [0, 1]}, the correct

integer less than or equal to y), is discontinuous at


[IIT - 1999]
ps
statement(s) is(are):

(A) all integers (A) (f(c))2 + 3f(c) = (g(c))2 + 3g(c) for some c ∈ [0, 1]
el



(B) all integers except 0 and 1 (B) (f(c))2 + f(c) = (g(c))2 + 3g(c) for some c ∈ [0, 1]

eh

(C) all integers except 0 (C) (f(c))2 + 3f(c) = (g(c))2 + g(c) for some c ∈ [0, 1]

(D) all integers except 1


je

(D) (f(c))2 = (g(c))2 for some c ∈ [0, 1] [2014]






C. Multiple Choice Questions with ONE or
iit

13. Let [x] be the greatest integer less than or equal to x. Then,

MORE THAN ONE correct answer :

at which of the following point(s) the function f(x) = x
@

8. If f (x) = (x – 1)/2, then on the interval [0, π] cos(π(x + [x])) is discontinuous?




[IIT - 1989] (A) x = –1 (B) x = 1





(A) tan (f(x)) and 1/f(x) are both continuous (C) x = 0 (D) x = 2 [2017]


(B) tan (f(x)) and 1/f(x) are both discontinuous







(C) tan (f(x)) and f–1(x) are both continuous D. Subjective Problems:


(D) tan (f(x)) is continuous but 1/f(x) is not.

14. Let f(x + y) = f(x) + f(y) for all x and y. If the function f(x)


The following function are continuous on (0, π)

9.
is continuous at x = 0, then show that f(x) is continuous at


[IIT - 1991]
all x [IIT - 1981]

(A) tan x



1 15. Determine the values a, b, c for which the function
(B)
x
∫0 t sin t dt



 sin(a  1) x  sin x
 , x0
 3 
1 , 0x 

x
(C)  4  f(x) =  c , x0

2 sin 2 x , 3  x    



 ( x  bx )  x
2 1/ 2 1/ 2

 9 4 
 , x0

 bx 3/ 2
 x sin x , 0  x   / 2
 is continuous at x = 0 [IIT - 1982]
(D)   


sin(   x ) ;  x  



2 2 1  x , 0  x  2
16. Let f(x) =  ;
 x sin x , when 0  x   / 2 3  x , 2  x  3


10. If f(x) =    , then Determine the form of g(x) = f[f(x)] and hence find the
 sin(   x ) ; when  x  



2 2 points of discontinuity of g, If any.
[IIT - 1991] [IIT - 1983]


FREE BOOKS FOR JEE & NEET =>(@iitjeeadv)
Continuity of Functions 2.69

17. Let f(x) be a continuous and g(x) be discontinuous function, =a x=0






prove that f(x) + g(x) is discontinuous function. x
= x>0



[IIT - 1987] 16  x  4



18. Find the value of a and b so that the function
Determine the value of a, if possible, so that the function



[IIT - 1989]



is continuous at x = 0.
 x  a 2 sin x , 0 x  /4
  
{1 | sin x |} ;  x0
a /|sin x |
f(x) =  2 x cot x  b /4 x  /2
6

a cos 2 x  b sin x /2 x   
 20. Let f(x) =  b ; x0



is continuous for 0 ≤ x ≤ π  


 e tan 2 x / tan 3x ; 0x

1 − cos 4 x  6
19. Let f(x) = x< 0
x2 [IIT - 1990] [IIT - 1994]





Determine a and b such that f(x) is continuous at x = 0.


Previous Year's Questions (JEE Main Papers)
1. If the function the interval [a, ∞), then an ordered pair (a, b) is


 2 + cos x − 1 (a) ( − 2 , 1 − 3 ) (b) ( 2 , − 1 + 3 )
, x≠π
ps





f(x) =  (π − x ) 2 (c) ( 2 , 1 − 3 ) (d) ( − 2 , 1 + 3 )
el






 k , x=π [2016, online]
eh
is continuous at x = π then k equals
1 k −1

If the function f defined as f(x) = − 2x , x ≠ 0,
je

(a) 2 (b) 1/4 5.


x e −1





iit

(c) 1/2 (d) 0 [2014, online] is continuous at x = 0, then the ordered pair (k, f(0)) is





equal to
If f(x) is continuous and f   = , then
@

9 2
2.
 2 9 (a) (2, 1) (b) (3, 1)





(c) (3, 2) (d) (1/3, 2) [2018, online]
 1 − cos 3x  is equal to





lim f 
  6. Let f : R → R be a function defined as

x →0 x2

(a) 9/2 (b) 2/9 5 if x ≤ 1
a + bx if 1 < x < 3




(c) 0 (d) 8/9
f(x) = 
[2014, online]






Let k be a non-zero real number. If b + 5x if 3 ≤ x < 5

3.

Ï (e x - 1)2 30 if x ≥ 5
Ô , xπ0
Ô Ê xˆ Ê xˆ then f is
f(x) = Ì sin Á ˜ log Á 1 + ˜

Ô Ë k¯ Ë 4¯

(a) continuous at a = 5 and b = 5


ÓÔ 12 , x=0 (b) continuous at a = –5 and b = 10


is a continuous function, then the value of k is (c) continuous at a = 0 and b = 5



(a) 1 (b) 2 (d) not continuous for any values of a and b [2019]







(c) 3 (d) 4 [2015]





4. Let a, b ∈ R (a ≠ 0). If the function f defined as

 2x 2
 , 0 ≤ x <1
 a
f(x) =  a , 1 ≤ x < 2 is continuous in

 2
 2b − 4b , 2 ≤x<8
 x
3

FREE BOOKS FOR JEE & NEET =>(@iitjeeadv)


2.70  
Differential Calculus for JEE Main and Advanced

ANSWERS
Concept Problems—A 11. (i) discontinuous at 0, left continuous



1 (ii) discontinuous at 0, right continuous; discontinuous at



2. 1 3. – 1; left continuous.




8
12. (i) a + b + 1 = 0 (ii) a = 3/2 + 2n.





4. a=0 5. 6
13. (i) 0 (ii) nπ/2, n ∈ I.



(i) 7/3 (ii) π/4





6. 14. (i) 1 (ii) 0






7. continuous (iii) yes, define f(0) = 0



8. f(0) = e 15. R



2 2
Practice Problems—A 16. (i) Discontinuities at x = 0 and x =   ,  3 ,.....,



2
9. discontinuous 10.
5  ,...... n ∈ I.
(2n  1)



6
π 
(ii) Discontinuities at x = –2, –1, 0, 2


12. 1 13. a = , b = (iii) f(x) is discontinuous at x ∈ I .



6 12


14. a = –In 3, b =
1
, c = 1 16. a + b = 0 Practice Problems—B
3



ps
17. a = 8
17. a = 2/3, b = e2/3. 18. 1




3 5
el
19. 1 18. {1, , , e, 3 , 10 , 11 , 12 , 3.5 }


2 2
eh

21. x ∈ [0, 1) ∪ [2, ∞)


Concept Problems—B

22. 12, 4, 2
je

The tangent is continuous everywhere except at



1.
iit

23. No


π

1
x=   n , where n is any integer; 24. all x ∈ R except x = nπ + , n ∈ I
@

2 4

the cotangent is continuous everywhere except at x = nπ, 25. (i) 3 (ii) dne




(iv) at x = n + 1/2, n ∈ Ι.

where n is any integer. (iii) no




π
 26. Discontinuous at x = nπ ± , n ∈ I.
2. (i) x  (2 n  1) , n  I (ii) ±1 . 3





2 27. Discontinuous at all integral values in [–2, 2]

5 28. Continuous on R.
4. 5. No




3 29. a=0,b=1

6. a = (2cosc – b)/c2 if c ≠ 0; if c = 0 there is no solution 30. discontinuous at x = 0.


unless b = 2, in which case any a will do. Concept Problems—C
1. isolated point removable discontinuity of first kind
(i) a = 16 b = 4

7.
2. irremovable finite discontinuity of first kind



2 4. x = – 2 is a discontinuity of the first kind (the jump being
(ii) a = 2n  (1)
n
, nI, b = 3

equal to 2)
3


5. No, it has a discontinuity of second kind.

8. (i) x =  n , n  N , (ii) x = n , n ∈ N ,
2 6. Infinite discontinuity of second kind.





7. No
(iii) x = n , n ∈ I , (iv) x = n / 2, n ∈ I ,

8. (i) g(x) = x – 4,




(v) x ∈ I


(ii) irremovable discontinuity




9. (i) No (ii) No Practice Problems—C




(iii) f(x) → 0 as x → 0, define f(0) = 0 for continuity at 0 9. (i) x = 0 is a point of removable discontinuity,




10. True (ii) x = 0 is a point of discontinuity of the second kind



FREE BOOKS FOR JEE & NEET =>(@iitjeeadv)
Continuity of Functions 2.71

(iii) x = k (k ∈ I) are points of discontinuity of the first kind,


 1 if x is rational


(iv) at the points x = 1 and x = –1 the function is 7. No. Example: f(x) = 
 −1 if x is irrational



continuous and the other points are points of
discontinuity of the second kind,
(v) x = –1 is a point of discontinuity of the second kind,  x  1, x  0
8. f (x)  


(vi) x = 0 is a point of discontinuity of the first kind  x  1, x  0




(vii) x = 1 is a point of discontinuity of the first kind
Practice Problems—D


(viii) x = –1 and x = 3 are points of discontinuity of the


second kind, 11. x2 if x ≥ 0; 0 if x < 0; h is continuous everywhere.


(ix) x = 1 is a point of removable discontinuity, 12. gof(x) is discontinous at x = – 1, 0, 1


(x) x = –1 is a point of discontinuity of the first kind.


13. 1 if 1 ≤ |x| ≤ 3 ; 0 otherwise. h is continuous everywhere



10. (i) missing point discontinuity, except at x = ±1, ± 3 .



(ii) misolated point discontinuity, 14. discontinuous at 1 and –1.




(iii) infinite discontinuity, 15. {0, 1}


(iv) infinite discontinuity,


 x  2, 0  x  1


(v) infinite discontinuity,



(vi) infinite discontinuity, 16. g ( x )   2  x , 1  x  2 and g(x) is discontinuous at



(vii) irremovable discontinuity 4  x, 2  x  3



11. No
x = 1 and x = 2

13. Irremovable discontinuity
ps
17. continuous everywhere


el
14. (i) x = – 2 is a discontinuity of second kind; x = 2 is a 18. h(x) is discontinuous at x = 0



removable discontinuity.
Concept Problems—E
eh

(ii) x = –1 is a removable discontinuity; x = 1 is a




discontinuity of first kind.
je

1
1. (i) f(x) = ; g(x) = x
(iii) x = 1 is a discontinuity of first kind. x −1


iit


1
15. (i) x = 1 is a discontinuity of first kind. (ii) f(x) = x ; g(x) = 2
x +1
@





(ii) continuous. (iii) f(x) = x2 ; g(x) = tan–1x




(i) (–4, –3), (–2, –1), (–1, 0), (0, 1),
Concept Problems—D 5.


(ii) (–3, –2.5), (–2.5, –2), (0, 0.5), (0.5, 1),
Yes, Hint. If the function ϕ(x) = f(x) + g(x) is


2. (i)
Practice Problems—E


continuous at the point x = x0, then the function
g(x) = ϕ(x) – f(x) is also continuous at this point; 8. (a) yes (b) yes
(ii) No. Example: f(x) = – g(x) = sgn x; both function




14. (ii) f(x) = x2(sin2x + 2)




are discontinuous at the point x = 0, and their sum is
identically equal to zero, and is, hence continuous. Concept Problems—F
3. (i) No. Example f(x) = x is continuous everywhere, 2. (i) c = 3 (ii) c = 2




(iii) c = 3


π
for x ≠ 0,


and g(x) = sin
x 5. 1.53 10. Yes



g(0) = 0 being discontinuous at the point x = 0. The
14. f must be a constant function.

product of these function is a function continuous

π Practice Problems—F
at x = 0 since lim x sin = 0;
24. (– 4, – 3), (0, 1), (4, 5) 26. Yes

x→0 x



 1 for x ≥ 0 30. 0.6 approx. 34. False
(ii) No. Example: f(x) = – g(x) = 



 −1 for x < 0 37. 5



both functions are discontinuous at the point 38. (i) [1, 2] (ii) [0, 1]






x = 0, their product f(x) g(x) ≡ – 1 being continuous
  
everywhere. (iii) 0,  (iv)   , 0 
 2




6. No  4 

41. f(x) = 0

FREE BOOKS FOR JEE & NEET =>(@iitjeeadv)
2.72  
Differential Calculus for JEE Main and Advanced

OBJECTIVE EXERCISES REVIEW EXERCISES for JEE ADVANCED


1. B 2. D 3. D 1. discontinuous at x = –1, 0, 1, 2.








4. C 5. D 6. B 2. continuous in R








7. C 8. B 9. D 3. discontinuous at x = 0.








10. A 11. B 12. B 4. discontinuous at x = 0, π.








13. A 14. A 15. D 5. g is continuous on (1, 2].








16. B 17. B 18. C 6. discontinuous at all positive odd multiples of π/2








19. A 20. C 21. A 7. 4π







22. C 23. C 24. A 8. discontinuity of the second kind at x = 0.







25. B 26. A 27. D 9. (i) discontinuity of the first kind from left at x = 1, 2, 3, ....;








28. C 29. A 30. B (ii) continuous for all x ≥ 0.









31. B 32. A 33. B 10. a > 64








1
34. B 35. C 36. C 11. a = e, b = - e







e
37. B 38. C 39. C






40. C 41. B 42. C 13. n = 1







43. D 44. D 45. B 1
14. a = 1– ,b=0







46. D 47. D 48. B ps e






17. Discontinuous at 0, 1
49. B 50. C 51. D







18. continuous in domain
A 54. A
el
52. 53. C

Discontinuous at all positive rational number except 1,






A 56. A 19.
eh

55. 57. D

continuous otherwise.






58. C 59. C 60. C
discontinuties lie one in each interval : (–2, –1), (0, 1),
je

21.






62. A

61. D 63. C (1, 2)
iit






64. C 65. C 66. D 23. (fog) x is continuous everywhere and (gof) x is






@

A

67. 68. C 69. C discontinuous at x = 0, ± 1.






70. D 71. ABCD 72. AB 30. discontinuous at x = 0.







73. ABCD 74. AD 75. BC 35. 1







76. ABCD 77. ABCD 78. AB






BC 80. ABCD 81. ACD
79.
TARGET EXERCISES for JEE ADVANCED






82. ABC 83. ABCD 84. AC






85. ABCD 86. BCD 87. AB 2. discontinuous







88. CD 89. CD 90. BCD 3. continuous everywhere.







91. A 92. D 93. C 4. discontinuous at x = 0, 1.







94. B 95. C 96. D 7. y n(x) is continuous at x = 0 for all n and y(x) is







97. C 98. C 99. C discontinuous at x = 0






100. D 101. D 102. B 8. f(x) = 0 when x = 0,







103. B 104. B 105. D 1 1
− x when 0 < x < ,






106. A 107. B 108. C =
2

2






109. C 110. C 111. B
1 1






112. C 113. D 114. A = when x = ,
2







2
115. (A)–(R); (B)–(PQ); (C)–(QS); (D)–(T)


116. (A)–(QRS); (B)–(R); (C)–(P); (D)–(P) 3 1
− x when
= < x < 1,


(A)–(R); (B)–(S); (C)–(Q); (D)–(P)

117. 2 2


118. (A)–(S); (B)–(R); (C)–(P); (D)–(Q) = 1 when x = 1,



119. (A)–(Q); (B)–(R); (C)–(PS); (D)–(PS) 13. 4




FREE BOOKS FOR JEE & NEET =>(@iitjeeadv)
Continuity of Functions 2.73

14. f(x) > 0 for c < x ≤ b, and f(x) < 0 for a ≤ x < c. 4. 10 5. B





17. f is continuous at every irrational in (0, ∞ ) and 6. C 7. D





discontinuous at every rational in (0, ∞) 8. CD 9. BCD




21. (iii) No (iv) yes 10. A 11. B, D




22. finite discontinuity at x = n + 1/2, n ∈ I.




12. A, B 13. A, B

23. both are discontinuous; φ(x) is continuous if a = 1.




3 1

24. continuous everywhere. 14. a = – , b ∈ R+, c =


2 2
27. Jump discontinuity.

29. (i) 4 – 3 2 + a, (ii) a = 3 2  x, 0  x  1





π π 16. g(x) =  2  x , 1  x  2
 4  x. 2 x  x  3


30. f(0 + ) = ; f(0 – ) = f is discontinuous
2 

4 2
at x = 0 ; g(0+) = g(0–) = g(0) = π/2 ⇒ g is cont. discontinuity at x = 1, 2


at x = 0.
31. discontinuity of the first kind at 1/r, r = 2, 3, ....;
π π
17. a = ,b=–

discontinuity of the first kind from left at x = 1;


6 12
discontinuity of the first kind from right at x = 0. 18. a = 8
32. n + 1/2, n ∈ I.

2
19. a = , b = e2/3
PREVIOUS YEAR'S QUESTIONS

3
ps
(JEE ADVANCED)
el
QUESTIONS FROM PREVIOUS
eh

1. k=7 YEAR'S (AIEEE/JEE MAIN PAPERS)



je

2. f(x) = 4 − x 2 , –2 ≤ x ≤ 0 = – 4 − x2 , 0 ≤ x ≤ 2
1. B 2. B

iit

3. D
3. (–∞, –1) ∪ [0, ∞), I – {0} where I is the set of integer





5. B

4. C 6. D
@

except n = –1





FREE BOOKS FOR JEE & NEET =>(@iitjeeadv)
HINTS & SOLUTIONS

Objective Exercises
Single Correct Answer Type

7. f (x) = sgn (cos 2x − 2sin x + 3)


( )


1 − cos x(cos 2x) 1/2 1/3
cos(3x) = sgn 4 − 2sin 2 x − 2sin x
1. Correction → f (x) =

x2
Sol using L-Hospital’s Rule
( (
= sgn −2 sin 2 x + sin x − 2 ))

1/2
lim(sin x)(cos 2x) (cos 3x) 1/3
+ cos x(cos 3x)
1/3
x →0
  1 9
2
= sgn  −2   sin x +  − 
sin 2x sin 3x
+ cos x(cos 2x)1/3
  2  4 
 
cos 2x (cos 3x ) 2/3
lim f (x) =
x →0 2
π
1 3
= lim  + 1 +  = 3 For x ≠ (4n + 1) ,


x →0  2 2 2

Hence, option B is correct. f (x) = 1


x − e + cos 2x
x
π
2. lim For x = (4n + 1)
x →0 x2


2
1 − e x − 2sin 2x −1 f (x) = 0
5
ps
lim = −2= −

x →0 2x 2 2
el
Hence, option C is correct.


Hence, D is correct.
eh

1

 x 2 − 1 − 1, π
x ≤1 8. f (x) = (sin x) π− 2x , x≠
f (x) = 

3.
je

2

| 2x − 3 | − | x − 2 |, x > 1
iit

1
 π
f   = lim(sin x)
π− 2x
x − 2
2
x ≤ −1
@

  2 x→
π

 −x 2 −1 < x ≤ 1
2

 sin x −1
=  1− x 1< x ≤ 3/ 2 limπ
π− 2x
 =e
x→
2

 3x − 5 3/ 2 < x < 2

 x − 1 =1
x≥2

Hence, B is correct.
  

Hence, C is correct.
x +1

 1  2 9. f (x) = , x≠0
4. f (x) = [x] +  x +  +  x +  x

 3  3

In [ −1,1], f (x) is discontinuous at 7 points i.e., x +1
+1
2x + 1

1 2 f  f (x) = x = , x ≠ −1
0, ± , ± , ± 1 x +1 x +1
3 3
x
Hence, C is correct.

2x + 1

 sin{x} +1
5. 
f (x) =  {x}
, {x} ≠ 0 f  f  f (x) = x + 1
2x + 1


 {x} = 0
k x +1
f (x) can never be continuous for any value of K. 3x + 2 −1
= , x ≠

Hence, D is correct. 2x + 1 2


6. If f (x) take only irrational values, then, f (x) is a constant So, there are three points of discontinuity


function. Hence, D is correct.

100 100
⇒ ∑ f (r) = ∑ f (2r) x − [x] {x}
10. lim = lim =1
x → 2+ x−2 x → 2 {x} +

r =1 r =1

Hence, B is correct. So, b = 1


FREE BOOKS FOR JEE & NEET =>(@iitjeeadv)
Continuity of Functions 2.75

a x2 − x − 2 (
a 2 + x − x2 ) 16. f (x) = [x cos x]


lim = lim
x → 2− 2 + x − x2 x → 2− 2 + x − x2 lim f (x) = 0, lim f (x) = −1
x →0+ x →0−

=a



So, a =1 lim f (x) = 0, lim f (x) = 0, f (1) = 0
x →1+ x →1−

Hence, A is correct.




11. f (x) =| 2sgn 2x | + 2. Hence, B is correct.



f (0) = 2  1  1
sin 4   − sin 2   + 1
f (0− ) = 4 17. lim  x   x
f (0+ ) = 4


x →0  1  1
cos 4   − cos 2   + 1

Hence, B is correct.  x  x

x + x 2n sin x  1  1
12. lim g(x) = lim lim − sin 2   cos 2   + 1
1+ x  x  x

x →1+
x →1 n →∞
+ 4n
lim =1
−4n +1
+ x −2n sin x
x →0  
1  1
x − cos 2   sin 2   + 1
= lim lim  x  x
x →1+ n →∞ x −4n + 1

=0 Hence, B is correct.


( )
n
x + x 2n sin x 18. f (x) = lim 1 − sin 2 x = lim cos 2n x
lim g(x) = lim lim =1
ps

n →∞ n →∞
x →1− x →1− n →∞ 1 + x 4n

 π
el
Hence, B is correct. f  =0
 4
eh

x3

13. f (x) = − a sin πx + 3, f (4) = 19, f ( −4) = −13 & f(x) discontinuous is at x = nπ.
je

4

iit

1999 Hence, A or C is correct.


As f (x) is continuous & − 13 < < 19.

19. f (0− ) = lim b cos 2m x − 1 = −1
@

199
x →0−

1999
Using IVT, f(x) is for some x ∈[ −4, 4]
f (0+ ) = lim a sin 2n x = 0

199
Hence, C is correct. x →0+


Hence, A is correct.
be x − cos x − x

14. lim 20. As f(b) = g(b) & both are
x → 0+ x2


Continuous function, then
For limit to exist, b = 1


e x − cos x − x h(b − ) = g(b + ) & h (b + ) = f (b − )
lim =1= a

x →0+ x2 Hence, C is correct.


 π 21. f (x) is discontinous at x = b.
2  tan + e x − 

lim 
4 Hence, A is correct.
=1

x →0 −
x
22. f (x) = (x mod 2) + (x mod 4)
2

Hence, A is correct.


15. For f(x) to be continuous, Points of discontinuity are x = 2, 4, 6, 8


Hence, C is correct.
2x + 1 = x 2 − 2x + 5

ln(3f (x) − 2) ln(1 + 3(f (x) − 1))
⇒ x 2 − 4x + 4 = 0 23. lim = lim
2(1 − f (x))

x →3 x →3 2
3(1 − f ( x)
⇒ ( x − 2) = 0
2
3
⇒ x = 2. −3
=

Hence, f(x) is continuous at only x = 2. 2

     

Hence, D is correct. Hence, C is correct.


FREE BOOKS FOR JEE & NEET =>(@iitjeeadv)
2.76  
Differential Calculus for JEE Main and Advanced

24. f (x) = highest power of (u x + u 2 + 2u + 3)


2
1
If x =

4
lim f (x) = 2
+
f ( 2) = 2 1  1 3
x→ 2 → ≤f  ≤
lim f ( x ) = 2 8  4  16


x→ 2 1
If x =

Hence, A is correct. 8

25. f (x) = [sin x + cos x] 1  1 3
→ ≤f  ≤

 π  32  8  64
=  2 sin  + x  


  4  Hence, D is correct.
( )


π 3π 3π 7 π

So, f(x) is discontinuous at x = , π , , , 28. f (x) = sgn sin 2 x − sin x − 1



2 4 2 4
 1
2
5
Hence, B is correct. = sgn   sin x −  − 
 2

4
1
26. f (x) = , x ≠1


1− x For f(x) to be discontinuous,


2
 1 5
1 1− x ⇒  sin x −  =
f  f (x) = = ,x≠0  2 4

1 −x
1−
1− x
ps 1 5
sin x − =±

1 2 2
el
f  f  f (x) = =x

1− x 1± 5
1+ sin x =
eh

x 2


je

So, α = 0, β = 1 1− 5
⇒ sin x =
iit

a + 2a 2 − 3 < 0 2

@

Hence, C is correct.


⇒ 2a 2 + a − 3 < 0
 1
⇒ 2a 2 + 3a − 2a − 3 < 0 29. f (x) = lim sin 2n (πx) +  x + 
n →∞  2

⇒ (2a + 3)(a − 1) < 0
f (1) = 1
 3 

a ∈  − ,1 lim f (x) = 1, lim f (x) = 1
 2  x →1+ x →1−


Hence, A is correct.  3
f  =3

f (x)  2
27. 2 ≤ 2 ≤ 3

lim f (x) = 2, lim f (x) = 1

x 3+ 3−
x→ x→
2x ≤ f (x) ≤ 3x 2
2
2 2

1 Hence, A is correct.
If x =

2 30. For g(x) to be continuous, then

1  1 3 a > range of f(x)
→ ≤f  ≤

2  2 4 ⇒ a > 26

If x = 1

least positive integral value of a = 3

→ 2 ≤ f (1) ≤ 3 Hence, B is correct.


1 31. As f(x) is continuous at x = 0
If x =

3 f(0) = c

2  1 1 axe x + b sin x
→ ≤f  ≤ LHL at x = 0 = lim
9  3 3 x →0 x3


FREE BOOKS FOR JEE & NEET =>(@iitjeeadv)
Continuity of Functions 2.77

Applying L-Hospital Rule,


sin 2 x = − sin 2 x

ae + axe + b cos x .
x x
lim ⇒ 2sin 2 x = 0

x →0 3x 2
⇒ x = nπ
For limit to exist, a + b = 0



2ae x + axe x − b sin x Hence, C is correct.
Applying L-Hospital again, lim


x →0
( )

6x ln 1 + 6f 2 (x) − 3f (x)
32. If f(x) attains only rational values for all real x, then f(x) =4. 36. lim

x →a


3f (x)
a1 , a 2 , a 3 , a 4 , a 5 are in H.P.
( ) × (6f

1 1 ln 1 + 6f 2 (x) − 3f (x) 2
(x) − 3f (x))
Then, a r ⋅ a r +1 = ⋅
(6f )
lim
a + (r − 1)d (a + rd)

x →0 2
(x) − 3f (x) 3f (x)
1 1 1 


=  −  = −1
d  a + (r − 1)d a + rd 


Hence, C is correct.
    
1  1 1   1 1 


4
∑ a r a r +1 =  −  + − 
r =1 d  a a + d   a + d a + 2d   1
( n2
37. f  n  = sin e n e − n + 2
4  n +1
) 2



 1 1 
+ −
 a + 3d a + 4d    n2 
ps (
f (0) = lim  sin e n e − n + 2  ) 2
  
1 1 1 
n →∞  n + 1
= −
d  a a + 4d 


el
=1
   
eh
4 Hence, B is correct.
= = 4a1a 5

a ⋅ a + 4d  π π
je

38. f (x) = [tan x[cot x]], x ∈  , 


   
 12 2 
iit

Hence, A is correct.

2x − 1, −2≤ x < 0
@

π 3 +1
33. f (x) =  As cot = ≈ 3.7
 x + 2, 0≤x≤2 3 −1


12
[x], − 4 ≤ x < −2 π
g(x) =  cot =0
 x + 2, − 2 ≤ x ≤ 4. 2

[cot x] = 0,1, 2,3

lim f (g(x)) = 2

x →−2+ Hence, f(x) is discontinuous whenever


lim f (g(x) = Not defined [cot x] = 1, 2,3
x →−2−


Hence, C is correct.
Hence, A & B is correct.

39. f (x) + g(x) = {1, ∀x ∈ R

 1  −1

  x 2 + e 2 − x  , x > 2 Hence, C is correct.
34. f (x) =   



e1/ x − e −1/ x
k , x = 2. 40. lim g(x) ⋅
e1/ x + e −1/ x

x →0
−1
 1 
1 lim f (x) = lim g (x)
lim  x 2 + e 2 − x  = x → 0+ x → 0+
x→2  +
 4


lim f (x) = lim g (x)

Hence B is correct. x → 0− x → 0−


sin 2 x, x is rational Hence, C is correct.

35. f (x) = 
 − sin x, x is irrational (2sin x) 2n

2
41. f (x) = lim
n →∞ 3n − (2 cos x) 2n

If f(x) is continuous, Then,

FREE BOOKS FOR JEE & NEET =>(@iitjeeadv)
2.78  
Differential Calculus for JEE Main and Advanced

⇒ lim
(2sin x) 2n
f (x) = lim
((1 + sin πx) − 1) t

((1 + sin πx) + 1)


n →∞ 2n 45.
 2 

t →∞


t
1−  cos x 
 3 
Case-I If x is integer


f(x) is discontinuous whenever
LHL of f(x) = lim f (x) → 1

t →∞


2
2sin x = ±1 & cos x = ±1
3 RHL of f(x) = lim f (x) → −1
t →∞


π
⇒ x = nπ ± Hence, B is correct.


6
1 2

Hence, B is correct. 46. f (x) = −
x e 2x − 1



tan x log x
42. f (x) = 1 2  e 2x − 1 − 2x
1 − cos 4x f (0) = lim  − 2x  = lim

Firstly, x ≠ ( −∞, 0]
x →0  x e − 1 x →0 x e 2x − 1 ( )


& 1 − cos 4x ≠ 0 e 2x − 1 − 2x x
= lim ×

cos 4x ≠ 1

x →0 x 2
e 2x
−1
=1

⇒ 4x ≠ 2nπ

Hence, D is correct.



ps
x≠ 1− | x |
2  , x ≠ −1
47. f (x) =  1 + x
el

1,

Hence, C is correct. x = −1
eh

sin x, x ∈Rational
43. f (x) =  1− | [2x] |
je

1 − 2 cos x, x ∈Irrational  [2x] ≠ −1



,
f ([2x]) =  1 + [2x]
iit

For continuity, 1 [2x] = −1




@

sin x = 1 − 2 cos x


sin x + 2 cos x = 1 1− | [2x] |  −1 
 1 + [2x] x ∈ R −  , 0
2 

1 2 1 =
sin x + cos x =
5 5 5 1 1
− ≤x<0


1 1 2
sin(x + α ) = , where cos α =


5 5 At x = –1

There are infinite values of ‘x’ which satisfies the eqn. −2
LHL = = 1, RHL = 1, f ( −1) = 1

Hence, D is correct. −2



log
44. f (x) =  (4x −3)
x 2 − 2x + 5 ,
3
4
(
< x < 1& x > 1 ) At x = 0
LHL = RHL = f (0) = 1

4, x =1

( )
1
log x 2 − 2x + 5 At x =
2
lim
x →1+ log(4x − 3)  1
LHL = 1, RHL = 0, f   = 0
( )  2

log (1 + h) 2 − 2(1 + h) + 5 4h

lim × Hence, D is correct.
h →0 log(1 + 4h) 4h


= Does not exist  −1, x < 0


Similarly, lim f (x) = Does not exist 48. f (x) = 0, x = 0 , g(x) = sin x + cos x

x →1− 

Hence, D is correct. 1, x > 0

FREE BOOKS FOR JEE & NEET =>(@iitjeeadv)
Continuity of Functions 2.79

 −1, sin x + cos x < 0 n


 x sin x

f  g(x) = 0, sin x + cos x = 0
53. ∏ cos  2n  =  x


n =1 2n sin  n 
1 sin x + cos x > 0 2 

Taking log & differentiate w.rt. x.

 π 
( )


−1 sin  + x  < 0 n
 4  n
1 x 1 cos x / 2
 π 
∑ 2n tan  2n  = 2n  x
− cot x.
= 0 sin  + x  = 0 n =1 sin  n 
  2 


 4
 π   x
n
1 sin  + x  > 0 now, lim ∑
1 1
tan  n  = − cot x.
 4  n →∞ 2 
n


n =1 2
x

Hence, B is correct.
 x 2
n
1
lim lim ∑

49. For continuity of f (x) → tan  n  =
π n →∞
n =1 2
2  π n

x→
2
4a = 16

Hence, C is correct.
a=4
2 − 4 x 2 + 16

For g(x) ⇒ 4 = a 54. lim

x → 0 cos 2x − 1

For h(x) ⇒ a = 4 4 − 2 x 2 + 16
ps
(2 + )

lim
x →0
Hence, B is correct. 4
x 2 + 16 (cos 2x − 1)
el

eh

tan x log x

50. f (x) = −x2
1 − cos 4x

(2 + )( )
lim
je

x →0
tan x log x
4
x 2 + 16 4 + 2 x 2 + 16 (cos 2x − 1)
=
iit

2sin 2 2x

1
@

=

As logx is not defined for ( −∞, 0] & tanx is not defined 64

π

for (2n + 1) Hence, A is correct.

( )
2
 2 sin x − sin 3 x + sin x − sin 3 x 
1 π  
( )
55. lim
& is not defined for n π  2 sin x − sin 3 x − sin x + sin 3 x 
+

x→

sin 2x 2 2  
Hence, C is correct.  3 (sin x − sin x )  3

51. f (x) is continuous only if 0 = 5x lim  =3
 (sin x − sin x ) 
π+ 3

n→
⇒x=0  2 
 2 (sin x − sin x ) + sin x − sin

x

3 3
Hence, D is correct.
lim  

 2 (sin x − sin x ) − sin x + sin
π+ 3 3 
x
x 3 − 2x 2 − 9x + 18 x→
 
52. R(x) = 2
x4 − 4


=3

For vertical asymptote, lim R(x) → ±∞ π
So, f(x) is continuous at x =

x →a

2

lim R(x) = lim


( )
(x − 2) x 2 − 9 LHD = RHD = 0 (By first principle)

x →a x →a
(x − 2)(x + 2)
2 2 Hence, A is correct.

56. (
lim b [x]2 + [x] + 1 )

x →−1+

So, at x = ± 2 , there are two vertical asymptotes

Hence, A is correct. ⇒ 1 ⇒ b ∈R


FREE BOOKS FOR JEE & NEET =>(@iitjeeadv)
2.80  
Differential Calculus for JEE Main and Advanced

lim [sin π(x + a)] = 1 tan kx


x →−1− 59. lim f ( x ) = lim =k


x → 0− x → 0− x

⇒ π(a − 1) = (4n + 1)
π
2 x → 0+ x → 0+
(
lim f ( x ) = lim 3 + 2k 2 = 2k 2 )
2
For continuity, k = 2k

1


⇒ a − 1 = 2n + ⇒ K = 0 or 1
2


2

3 Hence, C is correct.
a = 2n +


2 60. f ( x ) = lim x tan −1 nx


x →∞

Hence, A is correct.
 π

[x] + x − [x]  2 x, x > 0
 , x≥0 
57. f ( x ) =  sin x  0, x=0

sin x , x<0  −π
  x, x < 0
If x is +ve integer = k  2

f (x) = k Hence, C is correct.


−1
 1 
lim f ( x ) = k , lim f ( x ) = (k − 1) + 1 = k 61. f ( x ) = 3 + 1 + 71− x 
x→k +
x→k


 

If x is -ve integer = K f (1) = Not defined

f(x) = sin k
  1  
−1

LHL = RHL=sin K
ps

lim f ( x ) = lim+ 3 + 1 + 7   = 4
1− x

x →1 
If x= 0 x →1+
   
el

f(0) = 0 = RHL, LHL = sin 0 = 0
eh

−1

If x is any + real no, = K  1 
lim f ( x ) = 3 + lim 1 + 71− x 

je

f ( x ) = [k ] + {k} x →1 −
x →1
 

iit

RHL = [k ] + {k}, LHL = [K ] + {k} =3


Hence, option D is correct.
@


If x is any – Real no = K 62. fofofof(x) = x


f(x) = sin K = RHL = LHL

Hence, D is correct.

58.
4 − x2

f (x) = Y = f(x) ⇒ 2x–2

| 4x − x 3 |
 −(4 − x 2 ) 1
 , x>2
 (4x − x )
3

 4 − x2
 , 0<x<2

=  4x − x
3

 4−x
2
, −2 < x < 0 Now, from the graph, roots of fofofof(x) = x is only


 4x − x
3 possible at x = 0 & 2.
 −(4 − x 2 ) Hence, A is correct.

x < −2

(3 − 1)
, 2
 4x − x 3
x
63. lim
 1 (sin x ) ln(1 + x )

x →0
 − x , x > 2 or − 2 < x < 0
(3 − 1)
2
= x
x x
 1 , 0 < x < 2 < x < −2 = lim × ×
 x
x →0 x 2
s i n x ln(1 + x )
= (ln 3) 2

So, f(x) is discontinuous at x = 0, +2, -2.

Hence, C is correct. Hence, C is correct.


FREE BOOKS FOR JEE & NEET =>(@iitjeeadv)
Continuity of Functions 2.81

64. lim
4 − 2x + x 2 − 4 + 2x + x 2 67. lim f (g ( x )) = lim f 0+ = 2
x →−2+ x →−2+
( )


x →0 2+ x − 2−x

Hence, A is correct.


Rationalize,
68. f(x) = [4x] + {3x} of (4x)
( −4x ) ( )


2+ x + 2−x
1 1 3 5 3 7 9
( 4 − 2x + x )
lim pt of discontinuous are , , ,1, , , , 2,
x →0
(2x ) 2
+ 4 + 2x + x 2


4 2 4 4 2 4 4
5 11 13 7 15 17 9
⇒− 2 , , 3, , , , 4, ,
2 4 4 2 4 4 2
Hence, C is correct



Pd of discontinuous for {3x} are


log sin x| cos3 x  π π 1 2 4 5 7 8 10 11 13 14
65. f ( x ) = , x ∈ − ,  , x ≠ 0
 3 3 , ,1, , , 2, , , 3, , , 4, ,

x
log sin|3x| cos 3 3 3 3 3 3 3 3 3 3
2


3 log(1 + cos x − 1) log(1 + sin | 3x | −1) There are 30pts in total.
= ×


 x  log(1 + sin | x | −1) Hence, C is correct.
log 1 + cos − 1


 2 
69. f ( x ) =  tan x  cot x 
2 2


π π π 3π 5π 3π 7 π
f(x) is discontinuous at x = ± f ( x ) is discontinous at , , , π,, , ,

6 4 2 4 4 2 4
Hence, C is correct.

Hence, D is correct.
x 4 + x8 + 2x

66. l( x ) =  
ps
70. lim[h ( x )] ≠  lim h ( x ) 
 x →c 

tan x x →c

el
lim l( x ) = 2 ≠ l(0)
x →0 & h(x) can take any number of integral values.
eh
Hence, D is correct. Hence, D is correct.


je
iit

Multi Correct Answer Type for JEE Advanced


@

71. f (x) = [x ] + [x]


2 2 lim f (x) = 1, lim f (x) = 1, f (1) = 2
x →1+ x →1−

As [x] is discontinuous whenever x is an integer.

lim f (x) = 4, lim f (x) = 2

Hence, f(x) will be discontinuous at x = 2 & 0. x → 2+ x → 2−


Hence, A, B, C, D are correct. Hence, A, B, C, D are correct.


1 x 2n −1 + ax 2 + bx
72. f (x) = 74. f (x) = lim
x 2n + 1

1− x n →∞

1 1− x lim f (x) = 1
f 2 (x) = = x →1+

1 −x
1− lim f (x) = a + b
1− x x →1−


1 a + b +1
f 3 (x) = =x f (1) =
1− x 2
1+

x ⇒ a + b = 1.


So, pattern will repeat after an interval of three 57
 22x + 1 
ex + 1

f 3n
(x ) = x. ∀x except {0,1} 75. f (x) = 2sin x + 3, x ,  3x  are
e + 3  2 + 5


Hence, A & B are correct. continuous for x ∈ R.

 1 and
 x   + x[x], x ≠ 0

73. f (x) =   x 
f (x) = sgn(x) + 1 is not continuous at x = 0.

0, x=0


 Hence, A, B, C are correct.

lim f (x) = 3, lim f (x) = 3
x →−1+ x →−1−

FREE BOOKS FOR JEE & NEET =>(@iitjeeadv)
2.82  
Differential Calculus for JEE Main and Advanced

76. D) y = x 3 − 3x + 1 ∀ x ∈[1, 2] (
d x2 − 4 )= −

lim 6d, lim 12 = 12
dy
(
= 3x 2 − 3 = 3 x 2 − 1 > 0 ) x → 4− x x → 4+


dx −3 −3 1
a= ,b= , c = , d = −2.


f (1) = −1, f ( −1) = 3 2 2 2



⇒ it will have a root. Hence, A, C, D are correct.



Hence, D is correct. 82. lim lim cos 2n x = 0
n →∞ x → 0−



77. f (x) = x 4 − 14x 3 + px 2 + qx − 105 lim lim n 1 + x n = 1
n →∞ x → 0+

Sum of roots are 14 & product is −105. lim lim n 1 + x n = 1

n →∞ x →1−
Hence, roots are −1, 3, 5, 7.

1
lim lim =0
g(x) = (x − 1)(x – 3)(x – 5)(x – 7) n →∞ x →1+ 1+ xn

Hence, A, B, C are correct.
(x + 1)(x − 3)(x − 5)(x − 7)


F(x) = 83. All are continuous ∀x ∈ R
(x − 1)(x − 3)(x − 5)(x − 7)


Hence, A, B, C, D are correct.


x +1 84. B & D are discontinuous function
= ∀x ≠ 1, 3, 5, 7
x −1 Hence, A & C are correct.


Hence, ABCD are correct. 85. Conceptual question
ps


0, x ∈ I Hence, A, B, C, D are correct.

el
78. f (x) = [x] & g(x) =  2  −1 1
 x , otherwise x≠0

 tan ,
eh

86. f (x) =  x
g  f (x) = 0, x ∈R

1, x=0

je

lim f  g(x) = 4, lim f  g(x) = 3 f  (g(2) = 0
π π
iit

x → 2+ x → 2−
lim f (x) = , lim f (x) = , f (0) = 1

lim f  g(x) = 25, lim f  g(x) = 24 x → 0+ 2 x →0 −
2
@

x → 5+ x → 5−

Hence, B, C, D are correct.


Hence, A is correct.
0, x ∈I

π π  87. f (x) = [x] & g(x) =  2
79. f (x) = cos   cos  (x − 1)
 x ,

x 2  otherwise

LHL = RHL = f (0) = 0 g  f (x) = 0, x ∈R
x → 0− x → 0+

0, x ∈I

lim f (x) = cos 3, lim f (x) = cos 3, f (1) = cos 3 f  g(x) =  2
x →1+ x →1−
[ x ] otherwise

lim f (x) = 0, lim f (x) = 0, f (2) = 0

x → 2+ x → 2− Hence C, D are correct.


Hence, A, B, C are correct. 88. f(x) = max({x}, – {–x})


80. Conceptual question {x}

Hence, A, B, C, D are correct. -2 -1 0 1 2 3

sin ax a
81. lim = {-x}
x → 0−

bx b
-2 -1 0 1 2 3

lim (ax + 1) = 1
x → 0+
Hence, C, D are correct.


lim (ax + 1) = a + 1 lim (cx − 2) = 4c − 2 2
89. Conceptual problem.

x →1− x → 2−
Hence, C, D are correct.


d(x 2 − 4) e x − e[x]
lim (cx − 2) = c − 2
2
lim =0 90. lim = 1− e .
x →1 +
x→2 +
x ex

x → 0−

FREE BOOKS FOR JEE & NEET =>(@iitjeeadv)
Continuity of Functions 2.83

sin{x} π
lim =1 =
x →0 +
{tan x} 2




sin{x} sin π / 6 3 lim F(x) = lim sgn(x − 1) cot −1[x − 1]
x →1− x →1−
lim = =
π


π {tan x}
+
tan / 6 2
x→ 3π
6 =−

4
sin{x} 3 ,  π



3
lim = f  = Hence, C is correct.
 6

π −
{tan x} 2


x→ 2
6 99. lim f  g(x) = 1 , lim f  g(x) = 1, f  g(2) = 1
x → 2+ x → 2−


sin{x} sin π / 4
lim = =∞ A is true statement, but R is not correct.
π {tan x}
+
π


x→
4 tan − 1 Hence, C is correct.
4



Hence, B, C, D are correct. 100. lim [1 + cos x] = 0 , lim [1 + cos x] = 0 , f (π) = 0
x →π + x →π −



91. Both A & R are correct statements.
Hence, D is correct.


Hence, A is correct.

Comprehension 1
92. lim f (x) = 1, lim f (x) = 2
x → 0+ x →1−

 1 
f (x) = max  cos x, ,{sin x} , 0 ≤ x ≤ 2π
lim 2 − {x} = 2, lim f (x) = 1, f (2) = 2.  
ps

2
x →1+ x → 2−

el
Hence, D is correct.

eh

93. y = ln x y = ln x (0,1)


(0,1 )
je

Hence, C is correct. 2

iit

(0,0) π π 3π 2π
2 0 2
@

(1,0) (1,0)
 π 11π
cos x, 0≤x< , ≤ x ≤ 2π
4 6

sin x, π π π 5π
≤x< , <x≤
94. Conceptual  4 2 2 6
f (x)= 

Hence, B is correct. 1 , π 7π 11π
x = , π, <x<

2 2 6 6
95. lim cos  x cos  = 1
1
f(0) is not defined.  5π 7π
 x


x →0 1 + sin x, <x≤
+

 6 6
 1

lim cos  x cos  = 1 101. Hence, D is correct.
 x

x → 0−
102. Hence, B is correct.


Hence, C is correct. 103. Hence, B is correct.


96. Conceptual. Comprehension 2

Hence, D is correct. 104. Put x = 2


(
x 2 x 2n − 1 )  4 
f =2
97. f (x) =  f (2) 
x2 −1

4
98. lim F(x) = lim sgn(x − 1) cot −1[x − 1] ⇒ = 1 ⇒ f (2) = 4
x →1+ x →1+
f(2)

Hence, B are correct.

FREE BOOKS FOR JEE & NEET =>(@iitjeeadv)
2.84  
Differential Calculus for JEE Main and Advanced

 x2 
105. f (x) = x & f  111. For g(x) to be continuous, 2a = π ⇒ 2a = π
 =x π


 f (x)  2a

Hence, B is correct.


x2 6
⇒x= ⇒ x(f (x) − x) = 0
f (x) 112. lim(cos( x − a )) (x − a ) 2 sin( x − a )cos( x − a )


x →a
(x −a ) 6
−xlim 2 sin 2 ×
( x − a )sin( x − a )cos( x − a )
⇒ x=0 or f (x)=x ⇒e
→a
2

Hence, D is correct. ⇒ e −3/ 2



ln(fx) − ln f (1) x
106. lim sin
x →1 sin n zx

Now, h ( x ) = lim 2n × x = x
Comprehension 3 x


n →∞
1
f(x) = x + 1, -1 < x ≤ 1& f(x + 2) = 2n
f (x )
2

h (a ) = a
107. As f(x) is symmetric about origin in [-1,1]. Now, value of 3
⇒k=

f(x) in (1,2) is reciprocal of values of f(x) is (0,1). Values of 2

f(x) in (2,4) is reciprocal of values of f(x) is (0, 2) Hence,
Hence, C is correct.
fundamental period is 4.

Hence, B is correct. Comprehension 5

1, x ≥ 0
1 1
ps
108. lim f ( x ) = lim 2 = H( x ) = 
x →1 x →−1 x + 1 0, x < 0

+
2
el
Hence, C is correct. 113. If x ≥ 0
eh


109. No of discontinuity are x = 1 & 3. x2 − x + 1 − 2 = 0

je

Hence, C is correct.
x2 − x −1 = 0

iit

Comprehension 4
1+ 5
x = 1± 5 ⇒ x =
@

 πx
(a − x ) sec 2a , x<a 2
As g( x ) =  If x < 0
 − cot πx cosec( x − a ), x > a
 2a x2 − x − 2 = 0
( x − 2)( x + 1) = 0
110. lim g( x ) = lim (a − x )
 πx  x = 2 or − 1

x →a −
x →a −

cos  
 2a  so, x = −1

h 2a No options matches.
= lim ⇒

h →0  π h π 1, x ∈[ −1,1]
sin 
 2 a  (
114. H 1 − x 2 =  )
0, x ≠ [ −1,1]

 π π  
lim g ( x ) = lim  − cot  +
 2 2a  ( ) π
h cosec( h )
x →a + h →0   Now, H 1 − x 2 > sin x
2
 π 
tan  h 
 2a  h π π
= lim + × × =
h →0 sin h π 2a 2a
h
2a
Hence, D is correct.

FREE BOOKS FOR JEE & NEET =>(@iitjeeadv)
Continuity of Functions 2.85

Match the Columns for JEE Advanced

115. 116.

1 + a cos x 1
 , x<0 A) f (x) =
x2 1 1



 + −2
A) f (x) =  (x − 1) 2
x −1
b tan  π  ,


x≥0
  [x + 3]  (x − 1) 2 −(x − 1) 2
= =
1 + a cos x 1 + x − 1 − 2(x − 1) 2 2x 2 − 5x + 2
lim
x →0 −
x2 −(x − 1) 2
=

Exists if a = −1 . Then limit is
1
. (x − 2)(2x − 1)

2 So, Q, R, S are correct.


π 1 1
lim b + a = ⇒ b = ⇒ [a − 2b] = −2 B) f (x) = sgn x(1 − x 2 )
x →0



+
3 2 2 3
So, R is correct as it is discontinuous at


Hence, R is correct. x = 0, ±1

lim ( −2sin x) = 2

B) So, R is correct.
π−



x →−
2 f (x)
ps
C) lim exist & f (0) = 0
lim (a sin x + b) = + b − a


x →0 x
el
π+
x →−
2 ⇒ f (x) is continuous at x = 0
eh


lim (a sin x + b) = a + b So, P is correct.
π−

je

x→
2 sgn x
iit

lim cos x = 0 D) f (x) = lim x + n = x
π+ n →∞ 1


x→
1+
@

2
n

Hence P, Q is correct. So, P is correct.


cos3x 117.
 3  cot 2x A) Fundamental period = π = 3.14
C) lim   =1


π  2


x→ Hence, R is correct.

2
a|sin x|
B) f (x)=[x]+{2 x}+[3 x]


a|tan x| lim+ a
π b 1 1
lim (1+ | cos x =e = eb f(x) is discontinuous at x = 0,
x→
|) b 2
, ,1
π+

x→ 2 3
2 So, S is correct.


 π π
f   = +3
 2 2  sin x(1 + cos x)    sin x   1 
C)  lim  =  lim    1+  = 2

⇒ a=0 
x →0   x →0  x   cos x  


x cos x

sin[x]
D) lim a + = a. So, Q is correct.
x → 0+



x D) Sin−1x=+2 cos−1 (1+x)


 sin x − x  Domain −1 ≤ x ≤ 0
lim b +   = b −1

x →0 −
 x3  LHS ≤ 0, RHS ≥ 0

It holds only if both


f (0) = 2 sides are simultaneously


zero, which happens
⇒ a = 2, b = 3


for x=0.


So, T is correct. So, P is correct.


FREE BOOKS FOR JEE & NEET =>(@iitjeeadv)
2.86  
Differential Calculus for JEE Main and Advanced

118. Using AM – GM inequality 119.




A) Cos A + Cos B + Cos C ≤ 1 + 1 + 1
2 2 2 1
A) f (x) = lim


In a triangle this is positive if A → 0, B → 0 and n →∞ 1 + n sin 2 πx




C→ π . It is discontinuous at x =1


Hence the maximum value of 3 is not attained. So, Q is correct.


Hence Cos 2 A + Cos 2 B + Cos 2 C < 3
x

So, P is correct. B) f (x) = lim
( )
n →∞ n



9a + 3b + c 3 1 + 4sin 2 x
B) ≥ 27abc


3 1
abc ≤ (10)3 It is discontinuous when sin x = ±


2

Now, ⇒ log a + log b + log c = log abc So, R is correct.




≤ log(10)3 = 3 x 2n −1 + ax 2 + bx
C) h(x) = lim
x 2n + 1



n →∞

So, R is correct.

tan x tan x − sin x sin x 1 + a / x 2n − 3 + b / x 2n − 2
C) lim lim lim = 1.
x →1+ n →∞ 1
x →0 x3 x x + 2n −1


x
sinx sin x (1 − (cos x)3/2 )

lim x 2n −1 + ax 2 + bx
x →0 (cos x)3/2 ⋅ x 3/2 × x 2
ps lim lim =a+b
x →1 n →∞

x 2n + 1

el
3

= ⇒ a + b = 1.
eh

4

f (x) = [tan(a + b − 1)x] = 0

So, P is correct.
je


 1 So, P, S is correct.
iit

D) lim cos  x cos  = 1



x →0  x D) f (x + y) = f (x) ⋅ f (y)


@



lim
(
ln sec 2 x ) = lim − ln (1 − sin 2
x)
=1
⇒ lim f (x) = 1 + lim g(x) ⋅ lim a(x)
x →0 x →0 x →0

x →0 x →0 sin x ⇒ f (0) lim f (h) = 1 & f (0) = 1

x sin x x2 h →0
x

So, f (0) + g(0) = 2 lim f (h) = 0
h →0


So, Q is correct. So, P, S are correct.


Review Exercises Solution
 1 5
1. f (x) =  x +  [x] ∀x ∈[ −2, 2]
 2

4
(–2,3)
1 1 3
lim f (x) = 3, lim f (x) = 1, lim f (x) = , f ( −1) =
x →−2+ x →−1− x →1+ 2 2 2

1
lim f (x) = , lim 0, f (0) = 0, lim f (x) = 0 1
x →0 −
2 x →0 x →1 + −

X

3 3 5 –1 0 1 2
lim f (x) = , f (1) = , lim f (x) = , lim f (x) = 5
x →1 +
2 2 x→2 −
2 x→2 +

f (2) = 5
Y

f (x) is discontinous at x = −1, 0, 1, 2

FREE BOOKS FOR JEE & NEET =>(@iitjeeadv)
Continuity of Functions 2.87

2. f (x) = [[x]] − [x] + 1 g(x) is discontinuous at x = 2 only



f (x) = 1 (1 + cos x) n + 5ln x
6. f (x) = lim

n →∞ 2 + (1 + cos x) n


So, f (x) is continous ∀ x ∈R

1, −2≤ x ≤ 0 
3. f (x) =   5ln x
− 0<x≤2  π 3π 

 x 1,  , x ∈ , 
 2 2 2
 1, −2 ≤ | x | ≤ 0  5ln x + 1
f (| x |) =   π 3π
| x | −1, 0 < | x | ≤ 2 f (x) =  , x = or
 3 2 2

1, x=0 1, x ∈[0, π / 2)
= 
| x | −1; x ∈[ −2, 2] ~ {0}   3π 
or x ∈  , 2π

 1, −2 ≤ x ≤ 0   2 


| f (x) | = 1 − x, 0 < x ≤ 1
 x − 1, 1 < x ≤ 2 Hence, f(x) is discontinuous at all positive odd



Multiples of π

g(x) = f (| x |) + | f (x) | 2

 | x |, −2 ≤ x < 0  x  x
 7. g(x) = 1 + cos   + cos 2   + ……∞
 x=0  4  4
2,
ps

=
 | x | − x, 0 < x ≤1
el
1
 x + | x | −2, 1 < x ≤ 2 =
1 − cos x / 4
eh

So, g(x) is discontinuous at x = 0.
If g(x) is continuous from (0, c)
je


  sin x 
x>0
iit

 ,
y =   x 
x
4. ⇒ = π (∴common ratio of Infinite GP becomes −1 )
@



1 − [x cos x], x ≤ 0 4

⇒ x = 4π Hence c = 4π .
0, 0<x≤π

 −1, π < x ≤ 3π / 2
 e1/ x sin(1 / x)
8. lim =0
= 1, x=0 x →0 1 + e1/ x


2 π
− ≤x<0 sin(1 / x)
 lim = Not defined

2 x →0 1 + e −1/ x

+

So, discontinuous at x = 0 & π


Hence, f(x) is discontinuous at x = 0.


x 2
 + 1, 0 ≤ x < 1 9.
5. f (x) =  2
i) f (x) = (x − [x]) 2 = {x}2

2x 2 − 3x + 3 , 1 ≤ x ≤ 2


 2 Discontinuity of first kind for all x ∈ I

 (x − 1) 2
 + 1, 1 ≤ x < 2 ii) f (x) = [x] + (x − [x]) 2
f (x − 1) =  2


2(x − 1) 2 − 3(x − 1) + 3 , 2 ≤ x ≤ 3 = [x] + {x}2
 2
   
For x = k , k ∈ I

 2 3 (x − 1) 2

2x − 3x + + + 1, 1≤ x < 2
g(x) =  2 2 lim f (x) = k, lim f (x) = k − 1 + 1 = k, f (x) = k
x→k + h x→k −
2x 2 − 3x + 3 9
+ 2(x − 1) 2 − 3x + , x = 2

 2 2 So, f(x) is continuous for all x ≥ 0 .


FREE BOOKS FOR JEE & NEET =>(@iitjeeadv)
2.88  
Differential Calculus for JEE Main and Advanced

 (x − 2)3  15. Assume a h(x) = f (x) − g(x)


10. f (x) = 



 a 

x2 2 ln x
sin(x − 2) + a cos(x − 2) = x

e x


1
Max value of (x − 2)3 ∀ x ∈ [4, 6] is 64 h(1) = >0

e
Hence, a > 64 for f(x) to be continuous.


2
h(e) = e 2 − e − = e 2 − e − 2e −1 < 0

e
lim (sin x + cos x)cosecx


11.
x → 0−

as h(x) is continuous between (1, e) & it has opposite signs,


(sin x + cos x −1) Then, there will be atleast one ‘c’ where c ∈ (1, e) where
lim− =e=a
e x →0 si n x h(c) = 0

⇒ f (c) = g(c)
e1/ x + e 2/ x + e3/ x


lim =
x → 0+ e1+ 3/ x + be3/ x 16. Use intermediate value Theorem.


e −2/ x + e −1/ x + 1 1  −(x + 1), x ≤ −1
lim = 
x →0 +
e+b b+e 17. f (x) =  x + 1,

−1 < x ≤ 0
 x,

1  x>0
b= −e
e
ps
1 − x x≤0

x + 1
el
 0 < x ≤1
1 g(x) = 
2x+
eh

y= 2 x − 2 1< x < 2
2 − x x≥2
je

 −2x, x ≤ −1
iit

12.


−1 < x ≤ 0
@

2
–1 1 
2x f + g =  2x + 1 0 < x ≤1
y
= 2x − 2 1< x < 2

 2 x≥2

f (x) is discontinuous at x = 0 but has maximum value of So, f +g is discontinuous at x = 0, 1


2g(x) − 1, −2 ≤ g(x) < 0
3 at x = 1 & minimum value as 18. f  g(x) = 
g(x) + 2, 0 ≤ g(x) ≤ 2

2
−2 at x = −1 . = x+4 −2≤ x ≤ 0

13. f (x) is continuous for all n ∈N f  g(x) is continuous over its domain.


Smallest value of ‘n’ is 1. 19. for f (x) to be continuous.


1/2
14. If f (x) is continuous at x = 1 p  1+ p 2 
=
q  1 + q 2 

a+2 = a+b+2
( ) ( )

b=0 ⇒ p2 1 + q 2 = q 2 1 + p2

If f (x) is continuous at x = e

⇒ p =q 2 2


e + ae + 1 = ae + 2e + b
2 2
⇒ p = q, x > 0

e − 2e + 1
2
e −1 ⇒ x =1
⇒a = =

e −e 2
e Hence, f (x) is continuous only at x = 1 .


FREE BOOKS FOR JEE & NEET =>(@iitjeeadv)
Continuity of Functions 2.89

1 2 24. As f ( x ) = f (2 x + 1)
20. Volume of cone = πr h.


x   x 1 

3
⇒ f ( x ) = f  − 1 = f  2 − − 1
2  2 2 
Volume of right cylinder = πr 2 h = f (r)
 x 1 1 

f (0) = 0 = f  − 2 − − 1
 23 2 2 

f (r) = πr 2 h As n → ∞
 x 

1 1
Using Intermediate value theorem, there exist c ∈ (0, r) f ( x ) = f  n − n −1 − n − 2 1
2 

2 2
1
where f (c) = πr 2 h. f ( x ) = f ( −2)
3


Hence, f(x) is a constant function.
x
( )


21. f (x) = is discontinuous wherever
25. As f 2 x − 1 = 2 xf ( x )
2
x 3 − 3x + 1


Put x = c os θ
x 3 − 3x + 1 = 0.
f (cos 2θ) = 2 cos θ f (cos θ)

Now, g(x) = x 3 − 3x + 1 θ  θ
= 22 cos θ cos f  cos 

2  2
g ′ (x) = 3x 2 − 3 = 0 ⇒ x = ±1 ps θ θ  θ
= 23 cos θ cos cos f  cos 

2 4  4
g( −2) = −1, g( −1) = 3, g(0) = 1, g(1) = −1, g(z) = 3
el
θ θ  θ  θ
= 2n +1 cos θ cos cos ....cos  n  f  cos n 

eh

2 4 2   2 
Using IVT, f (x) is discontinuous in ( −2, −1 ), (0, 1) &

je

( k , 2). sin 2θ  θ
As n → ∞, f (cos 2θ) = f  cos n 
iit

a b  θ  2 
22. f (x) = +  sin 

@

x −1 x − 3

f (cos θ) = 0 For all values of θ
( )
f (1+ ) = ∞, f 3− = −∞

( )
 2 sin x − sin n x + sin x − sin n x 

& f(x) is continuous in (1, 3). 26. lim  
( )

π  2 sin x − sin n x − sin x + sin n x 

x→
Using IVT, f (x) will be zero for atleast one value of 2  

x ∈ (1,3) .
 3 (sin x − sin x ) 
n

lim  =3
23. f  g(x) = g(x) 1 − g 2 (x) ( x→
 (sin x − sin x ) 
π−

n


2

0 x > 0  π
 lim [f (x)] = 3 & g   = 3.
= 0 x < 0 π +
 2
x→
0 x = 0
 2

  
π
f  g ( x ) is always continuous. Hence, g(x) is continuous at x = for n > 1


2
 −1 x >1  x 3 
1 27. f (x) = ( −1) 
0 < x <1


g  f (x) =  −1 −1 < x < 0 lim f (x) = ( −1) n , lim f (x) = ( −1) n −1
1 x < −1
x → n1/3 + x → n1/3 −



0 x = 0, ± 1 Now if n ∈ I, LHL ≠ RHL.

g  f (x) is discontinuous at x = 0, ± 1. So, f (x) is discontinuous for x = n1/3 .


FREE BOOKS FOR JEE & NEET =>(@iitjeeadv)
2.90  
Differential Calculus for JEE Main and Advanced

28. g(x) = f (x) – f (x + 2). Hence, f (x) is discontinuous at all positive integer


except 1.
g (0) = f (0) – f (2)
log(2 + x), x < 1
g(2) = f (2) − f (4) = − (f (0) − f (2) )

 log 3 − sin1


g(0) = – g(2) 32. f ( x) =  x =1


 2

⇒ Using IVT, there exist point x = c ∈ (0, 2)  − sin x x >1

Where f (c) = f (c + 2)
f (x) is not continuous in  0, π  .

29. f ( xy) = f ( x )f y3 ( )  2



Put x = 1, y = 1 ⇒ f (1) = 1 33. f ( x + f ( x )) = f ( x )


As f ( x ) is continous at x = 1 Let f ( x ) = y
lim f (1) = f (1) = 1 f (x +y) = y
x →1
Now, for any x = x 0 As y → 0

( )
lim f ( yx 0 ) = f (xx )f y3
x 0 →1
lim f ( x + y) = 0
y→0

⇒ lim f( x )=f ( x 0 )
As f ( x ) is a continous for all x
x→x0 ⇒ f (x ) = lim f ( x + y) = 0
y →0
So, it is continous for all x ≠ 0 .
ps
Hence, f ( x ) is a constant function.
30. Similar to Ques 17.
el

 1 34. f ( x + h ) = Af ( x )f (h )
 x, x ≠ n
eh

Put x = h = 0
31. f (x) = 
je

x 2 , x = 1 1
⇒ f ( 0) = as f (0) ≠ 0

iit

n A
As f ( x ) is continous at x = 0
lim f (x ) = 0
@

x → 0+ lim f ( x + h ) = f (h)
x →0

lim f (x) = 0 f (0) = 0 ⇒ f ( x ) is continous for all x
x →0 −

( )

  
Hence, f (x) is continuous at x = 0  1 cos e n n2
 n  = +

35. As f
n2 + n + 1
2

At integral points ⇒ x = x 3n

2 3

As n → ∞
x(x − 1) = 0
x = 0 or 1  1  cos e n n2 
m n + 2
lim f  n  = lim 
n →∞  3  n →∞  3 n + n + 1
2
      

⇒ f (0) = 1. As f ( x ) is continous on x ∈ R

FREE BOOKS FOR JEE & NEET =>(@iitjeeadv)


Continuity of Functions 2.91

Target Exercises Solution

 1, | x |≥ 1  7, 0≤x<α
1  3
 1 1 g ( x) =  x − 9 x + 15x + 7, α ≤ x < 5
2
1. f (x) =  2 , ≤| x |< , n = 2, 3,…
n n n −1  −18, 5≤ x<6


 0, x=0
9 − 21
1 Where α 3 − 9α 2 + 15α + 7 = 0 ⇒α=
lim f ( x ) = 2
x→
1 (n )2


n Hence, g(x) is always continuous.


lim f ( x ) =
1
(n − 1) 2
4. ( )2 x − x3 + x − x3


2 (x − x ) − x − x
1 lim
x→ − x → 0+ 3 3
n

3(x − x )
1
So, f ( x ) is not continous at x = . 3
n lim =3
x → 0+ x − x3
 (1 − x )

−1/ x
− (1 + x ) 1/ x
, −1 < x < 0 ( )
2 x − x3 + x − x3
=
1
2 (x − x ) − x − x
lim
 x x → 0− 3 3 3
2. f (x) =  e , x=0
So, discontinuous at x = 0


ps
 (log sin x sin 2 x )

( )
ln sin 2 x
, 0 < x <1 2 x − x3 + x − x3 1
el
 =
2 (x − x ) − x − x
lim
x →1+ 3 3 3
eh

lim (log sin x sin 2 x )


ln sin 2 x
=2
x → 0+ 2 (x − x ) + x − x
3 3
je

f ( 0) = e =3
2 (x − x ) − x − x
lim
x →1− 3 3
iit

Hence, f(x) is discontinuous at x = 0.




@

So, discontinuous at x = 1.

3. f ( x ) = x 3 − 9 x 2 + 15x + 7
 x , when x is rational

f ′ ( x ) = 3x 2 − 18x + 15 5. As f ( x ) = 
1 − x , when x is irrational

(
= 3 x 2 − 6x + 5 ) 1
For limit to exist, x = 1 − x ⇒ x =
= 3( x − 5)( x − 1) 2

1
Graph of f(x) So, f(x) is continuous at x = only.


2
6.
(f ( x )) 2 + x 2 = 1 ∀x ∈[ −1,1]

⇒ (f ( x )) 2 = 1 − x 2
⇒ f (x) = ± 1 − x 2
0
As f ( x ) is continous function in [ −1,1]
1 5 6
⇒ f ( x ) is either 1 − x 2 or − 1 − x 2
x2 x2
7. yn (x) = x 2 + + +
1+ x
( )
2 2

1+ x2
As n → ∞
As f (0) = 7

yn(x) is infinite G.P with |r| < 1

f (1) = 14 x2
⇒ lim y n ( x ) = = 1+ x2
f (5) = −18 n →∞ 1
1−
f (6) = −11 1+ x2


FREE BOOKS FOR JEE & NEET =>(@iitjeeadv)
2.92  
Differential Calculus for JEE Main and Advanced

⇒ y( x ) = 1 + x 2  1  2  3
⇒ f (0) < f   < f   < f   =  < f (1)
 n  n  n

So, y(x) is continuous everywhere except x = 0.



x2 x2 But f(0) = f (1)
yn (x) = x 2 + + +


1+ x2
(1 + x ) Hence, it is a contradiction.
2
2


Assume g(x) is always -ve.
y n (0) = 0 LHL = RHL = 0


 1  2
⇒ f (0) > f   > f    > f (1)

So, yn(x) is continuous at x = 0.  n  n


1 1 1 1
8. f (x) = − x + [2x ] − − [ −2x ] Which is also a contradiction


2 2 2 2
 1
1 ⇒ g(x) should be equal to zero for x ∈0,1 − 
= − x + ([2 x ] − [ −2 x ])  


n
2
For x = 0, f(0) = 0  1  1
⇒ f ( x ) = f  x +  for some value of x ∈0,1 + 

 n  n


1
RHL = , Hence, discontinuous at x = 0. Hence proved.

2


1  1 1 13. If f(x) is continuous, then


For x = , f   = f(x) = ±x or ±|x|.
2  2 2
So, there are 4 continuous function possible.


1 14. If f(x) = 0 exist for only one C ∈ [a,b] & f(x) > 0 ∀ some
RK = 1. Hence, discontinuous at x = .

x ∈ (c,b]

2
ps
At x = 1, f(1) = 1 ⇒ f(x) is always +ve for x ∈ (c,b]

& If f(x) < 0 ∀ some x ∈ [a,c)

el
3

RHL = . So, discontinuous at x = 1. ⇒ f(x) is always -ve for x ∈ [a,c)
eh


2
15. For an injective function, following IVT Theorem,
Now as f(0) = 0 & f(1) = 1,

je

⇒ f(x) is a monotonic function ∀ x ∈ I



Using IVT, f(x) will assume all values between (0,1).

⇒ f(x) needs to be a continuous function in x ∈ I
iit


1
9. As for each x ∈ I, | f ( y) | ≤ | f (x) | 16. As | f ( x ) − f ( y) |< M | x − y |
@

2

1 1 Put x = c + h , y = c, h → 0

⇒ | f ( y1 ) | ≤ | f ( y) | ≤ | f ( x ) | lim f (c + h ) − f (c) ≤ 0
2 4 h →0


Repeating this process n times, ⇒ lim f (c + h ) = f (c) .............................. I

h →0


f ( yn ) ≤
1
f (x) Put x = c - h, y = c, h → 0
2n

lim |f (c − h ) − f (c) |≤ 0
As n → ∞, f ( y n ) ≤ 0 h →0

⇒ f ( yn ) = 0

⇒ lim |f (c − h ) − f (c)| .............................. II
h →0



⇒ there is some values of x for which f(x) = 0.
From 1 & II ⇒ lim |f (c + h ) = lim f (c − h ) = f (c) .

h →0 h →0

10. If f(x) is strictly negative somewhere on R & f(x) is

continuous on R → R, then, f’(x) = 0 at some point and p
it will be a finite absolute point of minima. Similarly, if 17. If x is rational =

q
f(x) is strictly positive somewhere on R. Then, it will have
finite absolute point of Maxima. p 1
For x + h = + where h → 0, ⇒ n → ∞

11. As 0 ≤ f(0) ≤ 2 & 0 ≤ f(1) ≤ 2. q n

As x ∈ [0,1], y = f(x) will cut y = 2x atleast at one point  1  np + q  1
RHL = lim f  x +  = f   = →0

since f(x) is continuous function n →∞  n  nq  np + q + nq
⇒ f(x) = 2x atleast at one point in [0,1]  1  np − q  1
LHL = lim f  x −  = f  = →0

 1 n →∞  n  nq  np − q + nq
12. Let g ( x ) = f  x +  − f ( x )
 n


Hence, f(x) is discontinuous for rational numbers.

Assume g(x) is always +ve, If x is irrational, then if x ± h = rational ⇒ q ⇒ ∞


FREE BOOKS FOR JEE & NEET =>(@iitjeeadv)
Continuity of Functions 2.93


1
=0  1  1
21. Let us assume g ( x )  f  x    f ( x ) in 0,1  
p+q    


k k

Hence, f(x) is continuous for irrational numbers.

18. As f(g(x)) = gf(x) ⇒ For a point (a,b) lying on f(x), there For a horizontal chord of length 1 , g(x) should be zero


is a point (b,a) lying on g(x).  1 k
Now Let h(x) = f(x) – g(x) in 0,1   .
 k

h(a) = f(a) – g(a) = b – g(a) = b – b = 0

Hence, f(x) & g(x) have a common point. Now Let us assume g(x) is always +ve,



19. (i) 1 2 k
As | f (u ) − f ( v) |≤| u − v |  f (0)  b    b         f (1)
k k k
Put u = x + h , v = x , h → 0


⇒ lim | f ( x + h ) − f ( x ) |≤ 0 Which is a contradiction


h →0
& Similarly, if g(x) is always -ve,
⇒ lim f ( x + h ) = f ( x )


h →0
1 2 k

Similarly,  f (0)  b    b         f (1)
k k k

u = x − h, v = x, h → 0

lim f ( x − h ) − f ( x ) ≤ 0 Which is a contradiction. Hence, g(x) should be zero

h →0 somewhere.
⇒ lim f ( x − h ) = f ( x ) 1
⇒ there is a horizontal chord of length ‘ ’
h →0
ps

k

Hence, f(x) is continuous in x ∈ [a,b]. (iii) No
el



20. (i) g(x) is inverse of f(x) ⇒ f(o) g(x) = x or g(o) f(x) = x.
eh

1 
Now, As M f  g   f  a i   y = f (x)
je

n 
iit

 1 
 f  Mf   f  g   f  a i  
 n 
@

0
1
  f ai 
n

Hence proved.

f  a1     f  a n 
(ii) As f  M f  

n (iv) If f(x) ≥ 0

Using Averages, ai < Mf < an.



Hence proved. Then, g(x) will have horizontal length from 0 to 1.


(iii) If h ( x )  af ( x )  b 22. As f(x) = pnx, excess of x over nearest integer


1 1
Mf  g   f  a i  At x = x + , n being integer
n

2
1 1 a a 
M h  g   (h (i))  g  b   b   1
f (x) =
i

n n 
 n 
2
1 

 g   b  a i   bn   lim f ( x ) = 0
n  1
+

x→n +
 f ai  b  2
 ga   1
 n lim f ( x ) =
 
1− 2
n→n +
1 
 g   f / ai  2

n 
 Mf 1
Hence, f(x) is discontinuous at x = n + .

2
   
Hence proved.

FREE BOOKS FOR JEE & NEET =>(@iitjeeadv)
2.94  
Differential Calculus for JEE Main and Advanced

23. Taking square root,


x + x si n x n


(i) f ( x ) = lim 1 −1
1+ xn f (x) = or f ( x ) =


h →∞
g( x ) g( x )
1+ s i n1


f (1) = 27. f ( x ) =
1
,
1
<x≤
1
2 n n +1
2n


2 2
 x   1
x n  n + s i n x 1
As f  n  = n
x  2  2


RHL = lim = s i n1
n  1
n →∞
x 1 + n   1  1
RHL = f  n + n  = n −1
 x  2  2
Hence, f(x) is discontinuous at x = 1.  1  1
LHL = f  n − n  = n −1

(ii) 2  2

x + xn


φ( x ) = lim 1
h →∞ 1 + ax n Hence, f(x) is discontinuous at x = n & it is a jump


2 discontinuity. 2
φ(1) =
1+ a
 x  28. Now, lim
(ln (1 + x + x ) + ln (1 − x + x )) cos x
2 2

(1 − cos x )
x n  n + 1

x  1 x + xn x → 0− 2
RHL = lim = , LHL = lim =1
n →∞ n  1  a n →∞ 1 + ax n
ps
x  n + a
x  (
+ x2 + x4
ln 1+ ) cos x
el
= lim
(1 − cos x )

x →0 − 2
Hence, φ(x) is discontinuous at x = 1, but φ(x) can be
eh

continuous if a = 1.
je

n
[2rx ] a sin x − a tan x
24. f ( x ) = lim ∑ Now, lim
iit

n →∞ n2 x → 0+ tan x − s i n x

r =1
(a )
@

tan x − sin x
2rx {2rx} −1
= lim ∑ 2
−∑ 2 = lim − a sin x
nn →∞ n x → 0+ (tan x − sin x )
2 x (n )(n + 1) {2rx} = − ln a = 1
= lim 2
−∑ 2
n →∞ 2n n ⇒ a = e −1
= x−0 = x

Hence, f(x) is continuous everywhere.  x
Now, g ( x ) = ln  2 −  cot ( x − a ), x ≠ a .

25. Graph of zx(x) = ke-x  a


As g ( x ) is continous at x  a
(o,k)  x
ln  2  
 lim 
a
x a tan( x  a )

  x 
1  1   
 a  (a  x )
 lim ln 


If k = 0, ZK(x) = 0 x a  x  a (tan( x  a)
1  a 

f(0) = z
 

f(0+) = 0+
1

So, discontinuous at x = 0    e

Hence, f(x) is discontinuous is 0 ≤ x ≤ 10. a
 
 g e 1  e

1
26. As f ( x ) = 2
2


g (x) Hence proved.

FREE BOOKS FOR JEE & NEET =>(@iitjeeadv)
Continuity of Functions 2.95

29.  1

t, 0t
(i) h(g (2))  h( 2 )  4  3 2  a 2

(ii) lim f ( x)  2  1
x 1 32. f ( t )  0, t


lim f ( x)  a  1  2
x 1  1
 t  1,  t 1

For f(x) to be continuous  2

& f(n + t) = f(t), where n is integer.
 a 1  2


1  1 1  1  1
a 3 f    0, f   , f  
2  2  2
  2   2

30.
lim f ( x ) = lim
( )
cos −1 1 − x 2 sin −1 (1 − x )
1

f ( t ) is disconttinuous at t 
x → 0+ x → 0+
(
2x 1 − x 2 ) 2


( 2x − x )
sin −1 (1 − x ) 1
= lim sin −1 2 4 As f(x+t) = f(t) ⇒ f(x) is discontinuous for all x  n  ,
( )

x →0 +
2x 1 − x 2 2
where n is an integer.
π 33.
=
2 ln(2  x )  x 2 n sin x
f ( x )  lim
Hence, f(x) is discontinuous at x = 0. n  1  x 2n

f ( x ), x0 f (1) 
ln(2  x )  sin x
As g ( x )  
ps 2
2 2f ( x ), x  0 lim f ( x )   sin x
el
x 1

 lim f ( x )  ln(2  x )
g(0) 
eh

x 1
2

For all x ∈ [0,1), f(x) = ln(2 + x)

je

 


lim g ( x )  , lim g ( x )   
iit

x 0 
2 x 0 
2 For all x  1,  , f(x) = -sin x.
 2


@

Hence, g(x) is continuous at x = 0. 



1  ln(2  x ), 0  x 1
31. f    2(r  1) x 
 ln(2  x )  sin x
r

 f (x)   , x 1
lim f ( x )  2(r  1) x  2
1

 
x
r   sin x , 1 x 
2
lim f ( x )  2rx

1

As f(x) is discontinuous at x = 1 ⇒ IVT is not applicable
x

r  
1 for f(x) where x  0,  ⇒ f(x) doesn’t vanishes in
Hence, f(x) is discontinuous at x = .  2

r  
x  0, 
 2.

Previous Year's Questions (JEE Advanced)

( x 3 + x 2 − 16 x + 20) 2. y2 = 4 − x 2

1. lim
x→2 ( x − 2) 2

y = ± 4 − x2
Apply L-hospital rule
 4 − x2 ,

 −2 ≤ x ≤ 0
3x 2 + 2 x − 16 6x + 2 ⇒ f (x) = 
lim = lim =7
2( x − 2)  − 4 − x , 0≤x≤2
2
x→2 x→2 2
⇒k=7

FREE BOOKS FOR JEE & NEET =>(@iitjeeadv)
2.96  
Differential Calculus for JEE Main and Advanced

 π  x
 1 t2  1 t  1
3. f ( x ) = [ x ]sin 
 [ x ] + 1 ∫  t
t sin   dt = sin   + cos  
 t 2  t

0
2

   

Domain ⇒ [ x ] ≠ −1 It is continuous for x ∈(0, π)

   


x ≠ [ −1, 0) 2x
C) lim 2 sin =1

⇒ x ∈ ( −∞, −1) ∪ [0, ∞) 3π



x→ 9
4

Discontinuity → at all integers in domain except 0 Hence, it is continuous for x ∈(0, π)

   


4. If f(2) = 10 & f(x) is continuous & take all rational values π
lim x sin x =

for all x. D)
π−



x→ 2
This is true only if f(x) = constant function 2

f(1.5) = 10 π π

ln(1 + ax ) − ln(1 − bx ) lim sin(π − x ) =
π+
2 2
5. lim x→
x →0

x 2

a ln(1 + ax ) b ln(1 − bx ) It is continuous for x ∈(0, π)


= lim +
   

x →0 ax − bx Hence, B C D are correct.
   

⇒ a + b Hence, B is correct. π
10. lim x sin x =

 2 x − 1
x→
π− 2
f ( x ) = [ x ]cos  π
 2 
6. 2

π π
ps
(2 x − 1)π lim sin(π + x ) = −
π+ 2 2
At all integral points, cos is always zero. x→
el

2 2
eh

Hence, f(x) will always be continuous. Hence, A is correct.




lim (a n + sin πx ) = a n
Hence, C is correct.
je

11.


7. f ( x ) = [ x ]2 −  x 2  x→2n +

lim ( b n + cos πx ) = b n + 1
iit

lim f ( x ) = 0, lim f ( x ) = 1, f (0) = 0 x→2n −
@

x →0 +
x →0 −

a n = 1 + bn ⇒ a n − bn = 1
lim f ( x ) = 0, lim f ( x ) = 0, f (1) = 0
x →1 +
x →1−
Similarly, a n −1 + 1 = + b n

Hence, D is correct. ⇒ a n −1 − b n = −1


 x − 1 Hence, B & D are correct.
tan f ( x ) = tan 
 2 

8.
12. We take two cases.


π Case 1: Let f and g attain their common
As tan x is discontinuous at odd multiple of

2 maaximum value at p
x −1  −1 π − 1 
in [0, π], will vary from  , ⇒ f (P) = g (P)
2 2 

2
where p ∈[0,1]
So, tan f(x) is continuous.

Case 2 : Let f and g attain their common maximum
f −1 ( x ) = 2 x + 1 is also continuous.
value at different point.

1
=
2
is discontinuous at x=1 ⇒ f (a ) = M and g (b) = M
f (x) x − 1

f (a ) − g (a ) > 0 and f (b) − g (b) < 0
Hence, C, D are correct.

⇒ f (c) − g (c) = 0 for some c ∈[0,1] as f and g

π are continous functtions.
9. A) tan x is discontinuous at


2 ⇒ f (c) − g (c) = 0 for some c ∈[0,1] for all cases.
x
 1 t2

1 1 1
B) ∫  t 
t sin dt = sin + ∫ cos dt Option A ⇒ f 2 (c) − g 2 (c) + 3[f (c) − g (c)] = 0


0
2 t 2 t
Which is true for Eq I
1

Put t = and use IBP Option D ⇒ f 2 (c) − g 2 (c) = 0 which is true
   

y

FREE BOOKS FOR JEE & NEET =>(@iitjeeadv)
Continuity of Functions 2.97

Now, if we take f ( x ) = 1 and g ( x ) = 1 ∀ x ∈[0,1] 17. Let h ( x ) = f ( x ) + g ( x )


options b an c does not hold, Hence. lim h ( x ) = f ( t + h ) + g ( t + h ) = LHL
x→t+
A and D are correct.
lim h ( x ) = f ( t − h ) + g ( t − h ) = RHL

x→t−
13. f ( x ) = x cos( π( x + [ x ]))
h ( t ) = f ( t ) + g( t )

At x = 0


As, f(x) is continuous LHL ≠ RHL for h(x)
lim f ( x ) = lim x cos(π ( x + x )) = 0


x →0 x →0 Hence, h(x) is a discontinuous function.


and f ( x ) = 0
π
∴ Its continous at x = 0 & clearly discontinous at 18. lim f ( x ) = a +
π−


x→ 4
other integers 4

Hence ans is → A, B, D π
lim f ( x ) = +b
π +

x→ 2
14. At x = t 4

( )

lim f ( t ) + f (h ) = f ( t ) + f 0+ lim f ( x ) = b
x→t+ π−
x→

lim f ( t ) + f ( − h ) = f ( t ) + f (0 )
2

x→t−
lim f ( x ) = −a − b
π+
x→
f ( t ) = f ( t ) + f (0) 2

If f(x) is continuous at x=0 =>f(x) is continuous for all x.
ps Now,


sin(a + 1) x + sin x π
el
15. lim = (a + 1) + 1 = a + 2 a= + b & a + 2b = 0
4
eh

x →0 −

x
π π
(x + bx )
1/ 2
2
− x1/ 2 b=− , a=+
(1 + bx )1/ 2 − 11/ 2
je

lim = lim 12 6

iit

x → 0+ bx 3/ 2 x →0 bx
19. lim f ( x ) = 8
x → 0−

1
@

=
2 ⇒a=8
1 3
⇒ c = ,a = − , b = R+ lim− |sin x |×
a
2 2 20. lim f ( x ) = e x →0 |sin a |
= ea

x → 0−
1 + x , 0 ≤ x < 2
16. f (x) =  lim f ( x ) = e 2 / 3
3 − x , 2 ≤ x < 3 x → 0+

1 , 0 ≤ x < 1 ⇒ e = b = e2 / 3
a

 2 ,1 ≤ x < 2 2
 ⇒ a = & b = e2 / 3
[f ( x )] =  3
1 , x = 2
0 , 2 < x < 3
2 ,0 ≤ x < 1
1 ,1 ≤ x < 2

g ( x ) = f [f ( x )] = 
2 ,x = 2
1 ,2 < x < 3
Hence, g(x) is discontinuous at x = 1, 2

FREE BOOKS FOR JEE & NEET =>(@iitjeeadv)
2.98  
Differential Calculus for JEE Main and Advanced

Previous Year's Questions (AIEEE/JEE MAINS)

 2 + cos x − 1 2
 , x≠π lim f ( x ) = , lim f ( x ) = a ⇒ a 2 = 2 ⇒ a = ± 2
f ( x ) =  (π − x )2 x →1 a x →1
− +
1.

k, x=π lim f ( x ) = a
 x→ 2

2 + cos x − 1 ( 2 + cos x + 1) 2b 2 − 4b 2b 2 − 4b
lim × lim = = 2
x→π (π − x )2
( 2 + cos x + 1) x→ 2
+
x3 2 2
1 + cos x


1
lim × ⇒ b 2 − 2b = 2 ⇒ b 2 − 2b − 2 = 0
x → π (π − x )2 ( 2 + cos x + 1)


 1 − cos h  1 1 2 ± 12
lim 
h →0 
 × = ⇒b= = 1± 3
h 2
( 2 − cos h + 1) 4 2
1 Hence , c is correct
For f ( x ) to be continous, k =


4 1 k −1
Hence, B is correct. 5. f (x) = − ,x ≠ 0
x e2 x − 1


f (1 − cos 3x ) e 2 x − 1 − (k − 1) x
2. lim lim f ( x ) = lim
( )

x →0 x2
x →0 x →0 x e2 x − 1
3x
ps
2 sin 2
1 − cos 3x 2 ×9= 9 Appply L-Hospital Rule,
= lim
el
As lim 2
x →0 x x → 0 9 2 4 2 2e 2 x − (k − 1)
x
eh

(e )
4 lim
x →0 2x
− 1 + 2 xe 2 x
 9  1 − cos 3x  2
je

As f ( x ) is continuous, f   = lim f  =
 2  x →0  x2  9 For limit to exist, k = 3
iit

Hence, B is correct
(
2 e2 x − 1 )
@

= 1 = f ( 0)
(e )
lim

( ) − 1 + 2 xe 2 x
2 x →0 2x

 ex − 1
 , x≠0 Hence, B is correct
3. f ( x ) =  sin  x  log 1 + x 
   

  k 4

5 if x ≤ 1
12, x=0 a + bx ,
  1< x < 3
6. f (x) = 
 b + 5x , 3≤ x <5
2
 e x − 1

  30, x≥5
 x 
lim = 4k lim f ( x ) = a + b, lim f ( x ) = a + 3b
x →0  x  x
sin    1 +  x →1+ x →3−
 k 4
log lim f ( x ) = b + 15, lim f ( x ) = b + 25, lim f ( x ) = 30
x x x →3+ x → 5− x → 5+

As f ( x ) is continous, 4k = 12 As f ( x ) is continous, b + 25 = 30
⇒k=3 ⇒b=5
Hence, c is correct Now, b + 15 = a + 3b
⇒a=5

 2x 2
 , 0 < x <1 but it will be discontinous at x = 1
 a Hence , D is correct.
4. f ( x ) = a , 1≤ x < 2



 2b − 4b ,
2
2 ≤x<8
 x3

FREE BOOKS FOR JEE & NEET =>(@iitjeeadv)


3
CHAPTER

Differentiability

3.1 Introduction Note: ∆x is a single symbol and does not mean delta


times x. Do not forget that as ∆x → 0, ∆x is getting close to 0,

Our first application of the concept of limit will be to the problem but is not equal to 0.
of determining the instantaneous rate of change of a function. ps
Geometrically, this problem is equivalent to that of finding a Consequently, the derivative of a given function
tangent line to the graph of the function. Both of these problems y = f(x) with respect to the argument x is the limit of the ratio
el
are solved by finding the derivative of the given function, as will of the increment in the function ∆y to the increment in the
eh

be described in this chapter. argument ∆x, when the latter approaches zero, provided the
The Definition of Derivative
je

limit exists.
Symbolically, the above equation is written as
iit

Let there be a function y = f(x) defined in a certain interval.


The function y = f(x) has a definite value for each value of the dy df ( x )
@

argument x in this interval. Let the argument x receive a certain = = f ′(x)




dx dx
increment ∆x (it is immaterial whether it is positive or negative).
where ∆ x represents finite change and dx represents
Then the function y will receive a certain increment ∆y.
infinitesimally small change in x.
Thus, for the value of the argument x we will have
It is convenient to write ∆x as h while calculating the derivative.
y = f(x),
With such notation the derivative of the function f with respect

for the value of the argument x + ∆x we will have to the variable x is given by
y + ∆y = f(x + ∆x).
f (x  h)  f (x)

Let us find the increment of the function ∆y f ′(x) = lim ...(4)


h 0 h
∆y = f(x + ∆x) – f(x) ...(1) provided the limit exists.


Forming the ratio of the increment of the function to the increment
Usually, when we say that a function f has a derivative f ′(x)
of the argument, we get the difference quotient
at a point x, it is implied that this derivative is finite, i.e. the
y f ( x  x )  f ( x ) above limit is finite. But it may occur that there exists an
 ...(2)
x x infinite limit equal to ∞ or –∞. In such cases it is useful to say


that the function f has at point x an infinite derivative (equal
We then find the limit of this ratio as ∆x → 0. If this limit exists,
to ∞ or –∞).
it is called the derivative or differential coefficient of the
df
given function f(x) and is denoted f ′(x). Thus, by definition, It should be emphasized that although the notation suggest
dx
dy y a ratio, the derivative as we have defined it, is not a ratio – even
= lim
x 0 x

dx though it is the limit of one. df is simply an abbreviation of f ′.
dx
f ( x  x )  f ( x ) It will be noted that in the general case, the derivative f ′(x)
f ′(x) = lim ...(3)
x 0 x has a definite value for each value of x. If we associate with



FREE BOOKS FOR JEE & NEET =>(@iitjeeadv)
3.2 Differential Calculus for JEE Main and Advanced

each value of x the value of the derivative of f(x), if it exists,
we obtain a new function of x, called simply the derivative of
f(x) with respect to x. This is denoted by f ′(x).
The domain of f ′, the set of points in the domain of f for which
the limit (3) exists, may be smaller than the domain of f. If
f ′(x) exists, we say that f has a derivative (or f is differentiable)
at x. f ′(x) often has the same domain of definition as f(x), but
not always.
The operation of finding the derivative of a function f(x)
is called differentiation of the function. If we can find a
mathematical expression for f ′(x), we can then find the value
of the derivative for a particular value of x by mere substitution.
Here, we considered the derivative of a function f at a general
1 x point x. If we change our point of view and let the number x be a
Example 1: Find f ′ if f(x) = .
2x fixed number a then we replace x in Equation (4) by a. We obtain



f (a  h )  f (a )
f (x + h) − f (x) f ′(a) = lim ...(5)
Solution: f ′(x) = lim


h 0 h
h →0


h
Note that we can also define
1 − (x + h) 1 − x
− f ′(a) = lim f ( x ) − f (a ) , provided the limit exists.
2 + (x + h) 2 + x x→a

= lim
ps x −a
h →0 Thus, the derivative of a function f is the function f ′ defined by

h
f ( x )  f (a )
el
(1 − x − h ) (2 + x ) − (1 − x ) (2 + x + h ) f ′(a) = lim .
x a

= lim x a
eh

h →0 h (2 + x + h ) (2 + x ) Instantaneous Rate of Change – The Physical



Meaning of Derivative
je

(2 − x − 2h − x 2 − xh ) − (2 − x + h − x 2 − xh )
iit

= hlim Suppose y is a function of x, say y = f(x). Corresponding to


→0 h (2 + x + h ) (2 + x )

a change from x to x + ∆x, the variable y changes from f(x)
@

−3h to f(x + ∆x).


= hlim
→0 h (2 + x + h ) (2 + x ) The change in y is ∆y = f(x + ∆x) – f(x), and the average rate of

change of y with respect to x over the interval from x to x + ∆x is
−3 3
= hlim =− . Average rate of change
→0 (2 + x + h ) (2 + x ) (2 + x )2

change in y y f ( x  x )  f ( x )
=  
change in x x x

Example 2: If f(x) = x3 – 1, plot the graph of the derived


function f ′. For example, let y = f(x) = x2 + 2x. Starting at x0 = 1, change
x to 1.5. Then ∆x = 0.5. The corresponding change in y is
Solution: For any real number x, ∆y = f(1.5) – f(1) = 5.25 – 3 = 2.25. Hence the average rate of


f (x + h) − f (x) change of y on the interval between
f ′(x) = lim . y 2.25
x = 1 and x = 1.5 is   4.5 .

t→0 h
x 0.5
We have f(x + h) – f(x) = ((x + h)3 – 1) – (x3 – 1) As the interval over which we are averaging becomes shorter
(that is, as ∆x → 0), the average rate of change approaches
= 3x2h + 3xh2 + h3,
what we would intuitively call the instantaneous rate of change

f (x + h) − f (x) of y with respect to x, and the difference quotient approaches
and so = 3x2 + 3xh + h2
h dy
the derivative .
dx
Consequently, f ′(x) = h→0
lim (3x2 + 3xh + h2) = 3x2.
Thus, the instantaneous rate of change at x
The graph of the function f ′(x) = 3x is the parabola shown in
2 dy y f ( x  x )  f ( x )
= lim  lim = f ′(x)
figure, on which the graph of the original function f(x) = x3 – 1 dx x 0 x x 0 x

has also been drawn. provided that the limit exists.

FREE BOOKS FOR JEE & NEET =>(@iitjeeadv)


Differentiability 3.3

It is conventional to use the word instantaneous even when Example 4: Suppose that a ball is dropped from the top



x does not represent time. The word is, however, frequently of a tower, 450 m above the ground.
omitted. When we say rate of change, we mean instantaneous (a) What is the velocity of the ball after 5 seconds?


rate of change. (b) How fast is the ball travelling when it hits the ground?


If the dependence upon time t of the distance s of a moving Solution: We first use the equation of motion



point is expressed by the formula 1
s = f(t) s = f(t) = (9.8) t2 = 4.9t2


2

The velocity v at time t is expressed by the formula to find the velocity v(a) after a seconds :
s f ( t  t )  f ( t ) f (a  h )  f (a ) 4.9(a  h ) 2  4.9a 2
v = lim  lim v(a) = lim  lim
t 0 t t 0 t h 0 h h 0 h

Hence v = st ′ = f ′(t) 4.9(a 2  2ah  h 2  a 2 ) 4.9(2ah  h 2 )
= lim  lim
or, the velocity is equal to the derivative of the distance with h 0 h h 0 h
respect to the time. 4.9(2a  h )  9.8a
= hlim
0
In general, suppose an object moves along a straight line
according to an equation of motion s = f(t), where s is the (a) The velocity after 5s is v(5) = (9.8)(5) = 49 m/s.


displacement (directed distance) of the object from the origin (b) Since the height of tower is 450 m above the ground, the

ball will hit the ground at the time a when
at time t. The function f that describes the motion is called the s(t1) = 450, that is, 4.9a2 = 450

position function of the object. In the time interval from t = a
450 450
to t = a + h the change in position is f(a + h) – f(a). The average This gives a2 = and a = ≈ 9.6 s.
ps 4.9 4.9
velocity over this time interval is
The velocity of the ball as it hits the ground is therefore
el
displacement f (a  h )  f (a )
average velocity =  450
≈ 94 m/s.
eh

time h v(a) = 9.8a = 9.8


4.9
je

Now suppose we compute the average velocities over shorter


Slope of Tangent – The Geometrical Meaning
and shorter time intervals [a, a + h]. In other words, we let h
of Derivative
iit

approach 0. We define the velocity (or instantaneous velocity)


@

v(a) at time t = a to be the limit of these average velocities : The concept of the line tangent to a curve at a point is an
important one in geometry. We shall now consider the problem
f (a  h )  f (a ) of defining the tangent line to the graph of a function f at a point.
v(a) = lim
h 0 We will soon find that the concept of tangent can be expressed

h
Example 3: The position of a particle is given by the in purely analytic terms involving the function f. In fact, the
problem leads directly to the definition of the derivative of a


equation of motion s = f(t) = 1/(1 + t), where t is measured in
seconds and s in metres. Find the velocity and the speed after function, the central idea in differential calculus.
2 seconds. What does it mean to say that a line is tangent to a curve at
a point ? School geometry defines the tangent to a circle as a
Solution: The derivative of f when t = 2 is straight line lying in the plane containing the circle and having a


1 1 single point in common with it. In other words, the tangent line
− to a circle at point P is the line that is perpendicular to the radial
f ( 2 + h ) − f ( 2) 1 + (2 + h ) 1 + 2 line at point P, as shown in the first figure below. However, it
f ′(2) = lim = lim
h →0 h →0 is not so simple an idea as it may first appear.

h h
1 1 3  (3  h ) For example, how would one define the tangent lines shown
− in the other figures? We might say that a line is tangent to a
3 (3  h )
= lim 3 + h 3 = lim curve at point P if it touches, but does not cross, the curve at
h →0 h 0

h h point P. This definition would work for the first curve shown
−h −1 1 in the figure, but not for the second. Or we might say that a
= hlim = hlim =− . line is tangent to a curve if the line touches or intersects the
→0 3 (3 + h ) h → 0 3 (3 + h ) 9

curve at exactly one point – this definition would work for a
1 circle but not for more general curves, as the third curve in the
Thus, the velocity after 2 second is f ′(2) = – m/s, and the figure shows. Here, the tangent "cuts" the curve at the point of
9
1 tangency instead of "touching" it. In the fourth curve, the tangent
speed is | f ′(2) | = − 9 = 9 m/s.
1
"touches" the curve at P, but also "cuts" it again on the y-axis.

FREE BOOKS FOR JEE & NEET =>(@iitjeeadv)


3.4 Differential Calculus for JEE Main and Advanced

The slope of the secant line joining the points (1, – 2) and
(1.2, – 0.56) is 7.2.
Essentially, the problem of finding the tangent line at a point P
boils down to the problem of finding the slope of the tangent
line at P. We can approximate this slope using a secant line
through the point of tangency and a second point on the curve.
This method of approximation leads to the following definition.
Definition The tangent to the curve at the point P is the limit


of a secant line through P and another point Q on the curve as
Q approaches P.
Let P(a, f(a)) and Q (a + h, f(a + h)) be two points very
close to each other on the curve y = f(a). Draw PM and QN
perpendiculars from P and Q on x-axis, and draw PL as
perpendicular from P on QN. Let the chord PQ produced meet
the x-axis at R and ∠QPL = ∠QRN = α. Now in right angled
As another example, both x and y-axis have one
triangle QLP
common point each with the parabola y = x 2, but the
x-axis is tangent to the parabola, while the y-axis is not. QL NQ − NL NQ − MP
tanα = = =
Thus, we see that for more complicated curves, more careful PL MN ON − OM

definitions must be made. f (a + h ) − f (a ) f (a + h ) − f (a )
= = ...(1)
If we have a simple continuous curve, our intuition and experience
ps (a + h ) − a


h
tell us that a point which moves along this curve constantly
el
changes its direction of motion, but that at each point P of the
eh

curve there is a definite straight line which gives the direction


of motion and closely approximates the curve near P. The line is
je

called the tangent at P and may be approximated in drawing by


iit

turning a ruler about P until it appears to have the proper position.


In the figure below, Q, Q′, Q′′, Q′′′ are several successive
@

positions of the point Q as it approaches P. Evidently the


secant PQ rotates about P and approaches a limiting position
PT, which is the tangent.
When h → 0, the point Q moving along the curve tends to P,
Note: A secant line is a line that intersects a curve in two i.e., Q → P. The chord PQ approaches the tangent line PT at


or more points.
the point P and then α → θ. Now applying
f (a + h ) − f (a )
lim tan α = lim

h →0 h →0 h
f (a + h ) − f (a )
tan θ = lim = m = f ′(a)
h →0


h
If the limit exists, then the function f(x) is said to have a finite
Example 5: Find the slope of the secant line joining the derivative at the point x = a and the line passing through


points (1, – 2) and (1.2, – 0.56) of the parabola (a, f(a)) with slope m is the tangent line to the graph of f at
y = x2 + 5x – 8. the point (a, f(a)).

Solution: Given y = f(x) = x2 + 5x – 8, we find ∆y and The slope of the tangent line to the graph of f at the point (a, f(a))



∆y/∆x as x changes from x0 = 1 to is also called the slope of the graph of f at x = a.

x1 = x0 + ∆x = 1.2 If we use the point slope form of the equation of a line, we

∆x = x1 – x0 = 1.2 – 1 = 0.2 and can write an equation of the tangent line to the curve y = f(x)

∆y = f(x0 + ∆x) – f(x0) = f(1.2) – f(1) at the point (a, f(a)) as :
y – f(a) = f ′(a) (x – a)

= – 0.56 – (–2) = 1.44.


y 1.44 Further, if lim f ( x ) does not exist in the ordinary sense, but
x a
So   7.2
x lim | f ( x ) | = ∞ then the secant line gets steeper and steeper as

0.2
x a

FREE BOOKS FOR JEE & NEET =>(@iitjeeadv)


Differentiability 3.5

x approaches a. In this case the line tangent to the graph of


the function y = f(x) at the point (a, f(a)) is perpendicular to
the x-axis and we call such a line as a vertical tangent. The
equation of the vertical tangent is x = a. Here, the function f(x)
is said to have an infinite derivative at the point x = a.
Note that not every curve has a tangent at any of its points
because the existence of tangent depends on the existence of
the above limit.
Definition The tangent line to the graph of f at the point

P(a,f(a)) is
(i) the line y – f(a) = f ′(a)(x – a) through P with slope f ′(a),

if f ′(a) exists;
f ( x )  f (a )
(ii) the line x = a if lim = ∞.
x a x a

If neither (i) nor (ii) holds, then the graph of f does not have a
tangent line at the point P(a, f(a)).
Example 6: Find the equation of the tangent line to the
f (3 + h ) − f (3)


parabola y = x2 – 8x + 9 at the point (3, – 6). (ii) f ′(3) = lim
h →0

ps h
f (a + h ) − f (a )
Solution: We have f ′(a) = lim [(3 + h ) 2 − 4(3 + h ) + 7] − [32 − 8.3 + 7]
h →0
el
h = lim



h →0 h
eh

[(a + h ) 2 − 8(a + h ) + 9] − [a 2 − 8a + 9]
= lim 2h  h 2
h →0 = lim= 2.

h
je

h 0 h
a 2 + 2ah + h 2 − 8a − 8h + 9 − a 2 + 8a − 9
iit

The derivative of the function at x = 3 is 2.


= lim
h →0 h Thus, the instantaneous rate of change of f at x = 3 is f ′(3) = 2.
@

2ah  h 2  8h The tangent line at x = 3 has slope 2, as shown in the figure.
= lim = lim (2a + h – 8) = 2a – 8.
h 0

h h→0 Example 8: Find equation of the tangent line to the


2
Since the derivative of f(x) = x – 8x + 9 at the number a is graph of the function f(x) = 3 x at the origin.
f ′(a) = 2a – 8, the slope of the tangent line at (3, – 6) is Solution: We have
f ′(3) = 2(3) – 8 = – 2.


f ( x )  f ( 0) 3
x 1
Thus, the equation of the tangent line is lim  lim  lim 2 / 3   .
x 0

x 0 x 0 x x 0 x
y – (– 6) = (– 2) (x – 3) or y = – 2x.

Therefore, the tangent is the vertical line x = 0.
Example 7: Let f(x) = x2 – 4x + 7.


(i) Find the average rate of change of f with respect to x Example 9: Show that the following functions do not


have finite derivative at the indicated points :

between x = 3 and 5.
(ii) Find the instantaneous rate of change of f at x = 3. Interpret (a) y= 5
x 2 at x = 0 (b) y =
5
x −1 at x = 1


the result geometrically. 5
(x ) 2 1
Solution: Solution: (a) f ′(0) = lim = lim .
x 0 x x 0 5
(x )3




(i) The (average) rate of change from x = 3 to x = 5 is found

by dividing the change in f by the change in x. The change Now the left hand limit is –∞ and right hand limit is ∞.
in f from x = 3 to x = 5 is Hence, the limit does not exists and there is no derivative at
f(5) – f(3) = [52 – 4(5) + 7] – [32 – 4(3) + 7] = 8. x = 0.

Thus, the average rate of change is 5
1 + ∆x − 1 1
(b) f ′(1) = lim = lim = ∞.
∆x → 0 ∆x  0
( x ) 4

f (5)  f (3) 8 x 5
 = 4.
53 Essentially, both the functions donot have a finite derivative

2
The slope of the secant line is 4, as shown in the figure. at the given points.

FREE BOOKS FOR JEE & NEET =>(@iitjeeadv)


3.6 Differential Calculus for JEE Main and Advanced

Meaning of Sign of Derivative One-Sided Derivatives
Remember that a positive derivative signals an increasing For the derivative of f at a point x to exist, it is necessary
quantity and that a negative derivative signals a decreasing that the function f be defined in a certain neighbourhood of
quantity. the point x including the point x itself. Then the expression
dy
This is obvious also from the geometrical interpretation of . y f ( x  x )  f ( x )
dx  is defined for all sufficiently small
dy x x
For, if x and y are increasing together dx is the tangent of an ∆x different from zero.
acute angle and therefore positive, while if, as x increases y f ( x  x )  f ( x )
Now, the limit f ′(x) = lim does not
decreases,
dy
represents the tangent of an obtuse angle and x 0 x
dx always exist for any function f defined in a neighbourhood
is negative. of the point x.
This aspect will be studied in detail in the chapter of The one-sided limit form of the derivative is useful in
monotonicity. investigating the existence of derivative.
If in the above limit it is assumed that ∆x tends to zero attaining
3.2 Differentiability only positive values (∆x > 0), then the corresponding limit
(wherever it exists) is called the right hand derivative of the

The derivative of the function f with respect to the variable x function f at a point x. We shall denote it as f ′(x+).
at x = a is given by Analogously, the limit, when ∆x tends to zero running through
f (a  h )  f (a ) negative values (∆x < 0), is termed as the left hand derivative
f ′(a) = lim
ps
h 0 of f at x (denoted as f ′(x–).

h
Of course, to compute f ′(x+) (or f ′(x–)) it is only necessary that
el
provided the limit exists.
the function f be defined at the point x and on the right of it in
eh

If the derivative of f(x) exists for x = a, we say that f(x) a certain neighbourhood (or at x and on the left of x). Consider
is differentiable (derivable) for x = a, otherwise, it is non-
je

the existence of derivative of y = f(x) at the point A(a, f(a)).


differentiable.
iit

If a function f (x) is differentiable at x = a, the graph of f (x) will


be such that there is a tangent with finite slope to the graph at
@

the corresponding point. But if f (x) is non-differentiable at


x = a, there will be either vertical tangent or no tangent with
finite slope at the corresponding point of the graph.
If f(x) is differentiable for every value of x in an interval I, we
say that f(x) is differentiable in the interval I.
If f is differentiable at every point in its domain, it is simply
called a differentiable function.
If f is differentiable on (– ∞, ∞ ) we will say that f is
differentiable everywhere.
Example 1: If g(x) = 1/x, find the derivative g ′(x). Also
Let us choose a point B to the right of A. If the slope of tangent


find the domain of g ′(x).
at A is evaluated by allowing B to approach A along the curve,
1 1
 then we have
Solution: g ′(x) = lim x  h x
h 0 f (a  h )  f (a )


h
Slope of right tangent = hlim  f '(a  )
0 h
x  (x  h) 1 which is called the Right Hand Derivative (R.H.D.) of f at
= hlim = hlim
0 hx ( x  h ) 0 x(x  h) x, provided the limit exists..

1 Let us choose a point C to the left of A. If the slope of tangent
= − at A is evaluated by allowing C to approach P along the curve,
x2

Evidently, the domain of g ′(x) is the set of all non-zero then we have
numbers, just as is the domain of g(x). That is, g is differentiable f (a )  f (a  h )
Slope of left tangent = hlim  f '(a  )
for all x except x = 0. 0 h

FREE BOOKS FOR JEE & NEET =>(@iitjeeadv)


Differentiability 3.7

which is called the Left Hand Derivative (L.H.D.) of f at x,


R.H.D. = f (1+) = lim f (1  h )  f (1)
provided the limit exists..


h→0 h
Sometimes, we prefer to write the left hand derivative as
f (a − h ) − f (a ) ln 2 (1  h )  0
f ′(a ) = lim

. = lim = 0.


h 0
h→0 −h h

Differentiability at an Interior Point Since L.H.D.= R.H.D. f(x) is differentiable at x = 1 and


f ′(1) = 0.
A function f(x) is said to be differentiable at some interior point
x = a in its domain if both L.H.D. and R.H.D. exist at that point, Example 3: Examine the differentiability of the function



and are equal, that is, if f ′ (a+) = f ′(a– ) = a finite quantity. f(x) = | ln x | at x = 1.
ln x if ln x  0
This geometrically means that a unique tangent with finite slope Solution: f (x) = | ln x | = 
can be drawn at x=a as shown in the figure.  ln x if ln x  0



There are cases of failure of existence of f ′(a) and even of ln x if x  1
= 
f '(a − ) and f '(a + ) at the point x = a, i.e. when the graph of the  ln x if 0  x  1


function has neither a right, nor a left tangent at the given point.
It may happen that f has at x = a, left and right derivatives L.H.D. = f ′(1–) = lim f (1 − h ) − f (1)
h→0


different from each other : i.e. f ′(a–) ≠ f ′(a+). −h
 ln(1  h )  0
= lim = – 1.
h


h0
f (1  h )  f (1)
ps
R.H.D. = f (1+) = hlim →0 h

el
ln(1  h )  0

eh

= lim = 1.

h0 h
je

Since L.H.D. ≠ R.H.D., f(x) is not differentiable at x = 1.


iit

Then such a point is called a sharp corner. In this case the  sin x 2
 if x  0
@

tangent line does not exist at the point, but we say that there Example 4: Let f (x) =  x
exist right and left tangents with different slopes at that point. 0


 if x  0
Differentiability at Endpoints Find the slope of tangent at x = 0, if it exists.
At the left endpoint of domain, the existence of R.H.D. is a f ( 0  h )  f ( 0)
Solution: L.H.D. = f ′(0–) = lim
sufficient condition for differentiability and at the right endpoint h



h0
of domain, the existence of L.H.D. is a sufficient condition for
sin h 2
differentiability. = lim  0 = 1.
h 2

h0
If x = a is the left endpoint of the domain of the function f, then
f is differentiable at x = a if the right hand derivative f ( 0  h )  f ( 0)
R.H.D. = f ′(0+) = lim
f (a  h )  f (a )
h0 h
lim exists.
sin h 2

h 0 h = lim  0 = 1.
h2

h0
If x = b is the right endpoint of the domain of the function f,
then f is differentiable at x = b if the left hand derivative Since L.H.D.= R.H.D., f(x) is differentiable at x = 0 and the
f ( b  h )  f ( b) slope of tangent at x = 0 is 1.
lim exists. Example 5: Comment on the differentiability of
h

h 0



x , x 1
Example 2: Examine the differentiability of the function f(x) =  at x = 1.


f(x) = ln2x at x = 1.
2
x ,
 x 1
Solution: L.H.D. = f ′(1–) = lim f (1 − h ) − f (1)
Solution: L.H.D. = f ′(1–) = lim f (1 − h ) − f (1)


h→0 −h h→0


−h
ln 2 (1  h )  0 1 − h − 1
= lim = 0. = hlim
→0 = 1.
h −h

h 0

FREE BOOKS FOR JEE & NEET =>(@iitjeeadv)
3.8 Differential Calculus for JEE Main and Advanced

f (1  h )  f (1) Example 8: A function f(x) is such that
R.H.D. = f ′(1+) = hlim



→0
h    

fx    – |x| ∀ x. Find f ′   , if its exists.
1  h  2h  1   2


2 2 2
(1  h )  1 2
= hlim
→0
= hlim
→0

h h   
Solution: Given that f  x    – |x|
= lim (h + 2) = 2.  2 2







h→0
Since L.H.D. ≠ R.H.D., f(x) is not differentiable at x = 1.     
f   hf   |h|
f ′     lim  2   2   2

2  1 .
Example 6: If f(x)
= A + Bx2 , x < 1  


h 0 h h
 2 



= 3Ax – B + 2, x ≥ 1
    


then find A and B so that f(x) becomes differentiable at x = 1. h f  | h | 
f ′     lim  2   2   2
f
  2  1.


Solution: L.H.D. = f ′(1–) = lim f (1 − h ) − f (1)  2  h 0 h h
h→0



−h

⇒ f ′   does not exist.
A  B(1  h ) 2  3A  B  2 2


= hlim
→0
h

Example 9: Show that the function f(x) = |cos x| does not
(2A  2B  2)  Bh 2  2Bh


= hlim 2k + 1
→0 have derivative at x = π, k ∈ I.
h

2
Hence, for this limit to exist – 2A + 2B – 2 = 0  2k  1 
  x 
i.e., B = A + 1. Then,
ps cos 
 2k  1  
  = lim
 2 
f ′(1–) = lim (Bh – 2B) = 2B Solution: f ′ 
el
 2  x 0 x



h→0

eh

R.H.D. = f ′(1+) = lim f (1  h )  f (1) sin ∆x


h→0
= lim = −1
∆x
je

h ∆x → 0 −

 2k  1   sin x
iit

3A (1  h )  B  2  3A  B  2 f′   = lim
= hlim
→0  1.
 2  x 0 x


@


h
3Ah Since L.H.D. ≠ R.H.D., f(x) is not derivable at
= hlim
→0 = 3A.
2k + 1

h
x= π, k ∈ I.
Since f is differentiable at x = 1, we have f ′(1–) = f(1–)

2
⇒ 3A = 2B ⇒ 3A = 2(A + 1) Example 10: Examine the differentiability of the function



⇒ A = 2, B = 3. π
f(x) = cosx + | cos x | at x = .

os x ] x  1 2
Example 7: Let f(x) =  . Comment on the
 2{x}  1 x  1
Solution:





 
derivability at x = 1. 2 cos x if 0  x  2
f (x) = cos x + | cos x | = 
Solution: f(1–) = lim f (1 − h ) − f (1) 

h→0 0 if  x  


−h  2
cos( π − πh) +1 1 cos h
= hlim
→0 = hlim
→0
−h h

h
2 sin 2

= hlim
→0
2 = 0.
−h

f(1 ) = lim f (1  h )  f (1)
+
 π− 2 sin h  0

h→0
h f ′   = lim =–2
  h0
2

h
2{1  h}  1  1 lim
= hlim = h → 0 2h = 2.   
→0 00

h
f ′   = lim 0.
Since L.H.D. ≠ R.H.D., f(x) is not derivable at x = 1.  2  h0 h


FREE BOOKS FOR JEE & NEET =>(@iitjeeadv)
Differentiability 3.9

Since the left and right derivatives are different from each other,  7 62 2 
π x3    x  ... 
the graph of the function has a corner point at x = and f(x) is lim  3 15 
2 = x→0
 1  2
not differentiable here. (a  1  3) x  9  x 3  (242) x 5  ....
3  15
Example 11: Given f (x) = [x] tan (π x) where [.] denotes
limit b can be non - zero if a + 1 – 3 = 0


greatest integer function, find the LHD and RHD at x = k,


where k ∈ I. 7

Solution: f (x) = [x] tan (πx)  7   3 
∴ a = 2 and hence b = 3 =    


f (k  h )  f (k ) 1
−9  3   26 
f ′ (k+) = lim 3


h0 h
[k  h ]·tan(k  h )  0 7 b 7
= lim = ⇒ = .
26



h0 h a 52
k tan(h ) ax 2 − bx + 2 if
= lim = kπ x<3
h Example 13: Let f(x) =  .

h0
 bx − 3 x≥3
2
if



f (k − h ) − f (k )
f ′ (k–) = lim
−h Find the values of a and b so that f(x) is differentiable

h→ 0
everywhere.
[k  h ]·tan(k  h )  0
= lim Solution: f ′(3–) and f ′(3+) exist finitely.
h



h0
ps f (3 − h ) − f (3)
(k  1) tan(h ) ⇒ f ′(3–) = lim
= lim h →0 −h
el
h

h0
a (3 − h ) 2 − b(3 − h ) + 2 − (9b − 3)
eh

= π (k – 1). = lim

Hence R.H.D. = f (k+) = kπ and L.H.D. = f ′ (k–) = (k – 1)π. h →0 −h

je

Example 12: ah 2 + h (b − 6a ) + 9a − 12b + 5


iit

= lim ...(1)
(−h)


h →0


 x  x tan x  x tan 2 x
2
@

 , x0 ∵ f ′(3 –) exists, the numerator must tend to zero as h → 0


 ax  tan x  tan 3x

Let g(x) =  . ∴ 9a – 12b + 5 = 0 ...(2)


 Now from (1) and (2)
 0, x0
ah 2 + h (b − 6a )
If g′ (0) exists and is equal to a non-zero value b, then find the f ′(3–) = lim = 6a – b,
h →0 −h

b
value of . f (3  h )  f (3)
a and f ′(3+) = lim
h 0

h
Solution: g ′(0) = b = lim x  x tan x  x tan 2 x
2
b(3  h ) 2  3  (9b  3)


x→0
x (ax  tan x  tan 3x ) = lim

h 0 h
lim x  tan x  tan 2 x
= x→0 (3 + h ) 2 − 9
ax  tan x  tan 3x = b lim = 6b . ...(3)

x →0

h
 x3 2 5  Given f ′(3–) = f ′(3+)
x   x   x  ........  6a
 3 15  ⇒ 6a – b = 6b ⇒ b = . ...(4)
7



 8x 2 3  From (2) and (4),
  2 x   · 32 x 5 ....... 
 3 15  9a
lim
= x→0 9a – 72a + 5 = 0 ⇒– +5=0


  7 7


x3 2 5
ax   x   x  .....  35 10
 3 15  ∴ a= ,b= .
3

9
 27 x 3 2 
  3x   243x 5  .....  Example 14: A function f is defined by f(x2) = x3 for all


 3 15  x > 0. Show that f is differentiable at 4.

FREE BOOKS FOR JEE & NEET =>(@iitjeeadv)


3.10  
Differential Calculus for JEE Main and Advanced

Solution:  f (a  h )  f (a ) 1 


= (exp.)  lim 
   
·
f ( 4  h ) 2  f 22


f ( 4  h )  f ( 4)  h0 h f (a ) 
f (4)  lim  lim
h 0 h h 0 h  f '(a )  f '( a )
= (exp.)   = f (a ) .
(4  h ) 3/ 2
8 8 [(1  h / 4) 3/ 2
 1]  f (a )  e


 lim  lim
h 0 h h 0 h
Example 16: Let f : R → R satisfying |f(x)| ≤ x2, x∈R

3 3  h2  




8 1 
3h 
 ....  1 8  h     ... then show f(x) is differentiable at x = 0.
8 8  16  
   lim 
24
 lim Solution: Since, |f(x)| ≤ x2 , ∀ x ∈ R
h 0 h h 0 h



= 3. at x = 0, |f(0)| ≤ 0 ⇒ f(0) = 0 ...(1)




Hence f is differentiable at 4. f ( h )  f ( 0) f (h )
∴ f ′(0) = lim = lim ....(2)
Example 15: Let f be differentiable at x = a and let


h 0 h h →0 h


n
 f (a  1 n ) 
f(a) ≠ 0. Evaluate lim   .
{f(0) = 0 from (1)}


n   f (a ) 
n
Now, f (h ) ≤ | h | ⇒ – | h | ≤ f (h ) ≤ | h |


 f (a  1 n )  h h
Solution: l = nlim   (1∞ form)
  f (a ) 



f (h )
  f (a  1 n )  f (a )  
⇒ =0 lim ...(3)


h h →0
l = (exp.)  lim n   
ps
 

 n f (a )
{using Sandwitch theorem}

el
put n =
1 ∴ from (2) and (3) , we get f ′(0) = 0.


i.e. f(x) is differentiable at x = 0.
eh

h

je

Concept Problems A
iit

@

1. Examine the differentiability of the function f(x) = e– | x | (c) Show that y=x2/3 has a vertical tangent line at(0, 0).



at x = 0. (d) Illustrate part (c) by graphing y = x2/3.


sin x if x   5. Compute the difference quotient for the function
2. Let f (x) = 

where m and n are  sin x
 mx  n if x  

 if x  0
constants. Determine m and n such that f is derivable defined by f(x) =  x

 1

at x = π. if x  0
3. Discuss the differentiability of f at x = 1 Do you think f(x) is differentiable at x = 0? If so, what is


f(x) = 3x, – 1 ≤ x ≤ 1 the equation of the tangent line at x = 0 ?


= 4 – x, 1 < x < 4  x tan −1 1 / x , x ≠ 0
6. Let f(x) = 


.
4. (a) If g(x) = x2/3, show that g ′(0) does not exist.  0 , x=0



(b) If a ≠ 0, find g ′(a). Comment on the derivative of f(x) at x = 0.



Practice Problems A

7. Find the slope of the secant to the parabola y = 2x – x2,
a2  x2

if the abscissas of the points of intersection are equal to: (b) f(x) = sin–1
a2  x2


x1 = 1, x2 = 1 + h. To what limit does the slope of the secant
x
tend if h → 0 ? ,
(c) f(x) = 1
x ≠ 0; f(0) = 0
8. Find f ′(0+) and f ′(0–) if 1+ ex



1
(a) f(x) = sin( x 2 ) (d) f(x) = x2 sin , x ≠ 0; f(0) = 0.


x


FREE BOOKS FOR JEE & NEET =>(@iitjeeadv)
Differentiability 3.11

9. Discuss the differentiability of the function 12. Prove that if f(x) and ϕ(x) are equal to zero for x = 0 and


x (3e1/ x  4) have derivatives at x = 0, ϕ ′(0) being not
f(x) = , x≠0
2  e1/ x



f ( x ) f '(0)
equal to zero, then lim = .
=0, x=0 ϕ( x ) ϕ '(0)


x→0


at x = 0.
13. Prove that if f(x) has a derivative at x = a, then

10. Prove that the function



 3/ 2 1 lim xf (a ) − af ( x ) = f (a ) − af '(a ) .
| x | sin if x  0, x  R , x →a
f(x) =  x−a


x
0

if x  0, 14. Let f(x) be an even function defined for x ∈ R. If f ′(0)


is differentiable at the point x = 0, and f ′(0) = 0. exists, find its value.

11. It is known that f(0) = 0 and there exists a limit of
x g( x )

f (x) 15. Let f(x) = , g(0) = g’(0) = 0 and f(x) be continuous
the expression as x → 0. Prove that the limit


|x|
x

is equal to f ′(0). at x = 0. Find f’(0), if it exists.

The point x = 0 is called a corner and the function y = |x| is
3.3 Reasons of Non-Differentiability not differentiable at 0.

We now find the reasons because of which a function does not
ps
have a derivative at a point.
el
A function has a derivative at a point x = a if the slopes of the
secant lines through P(a, f(a)) and a nearby point Q on the graph
eh

approach a limit, as Q approaches P. However, a function will


je

fail to have a derivative at a point where the graph has


(i) a corner, where the one-sided derivatives differ.
iit

In general, if the graph of a function f has a “corner” or “kink”



(ii) an oscillation point, where the one-sided derivative(s) does
@

in it, then the graph of f has no tangent at this point and f is not
(do) not exist.
(iii) a vertical tangent, where the absolute value of slope of PQ differentiable there. This can be verified by computing the left

approaches ∞. and right hand derivatives at the point.
(iv) a discontinuity.

Such slopeless points may be roughly classified as "sharp Caution
points", "oscillation points", "steep points", and "points of
discontinuity". But one should be careful about predicting a corner at a point.
For instance, one may feel that the function f(x) = |x3| has a
Case I: Sharp Corner or Kink corner at x = 0 and hence non-differentiable there. This is
If it happens that in a continuous function f at x = a, the left found to be wrong.
hand and right hand derivatives are different from each other
i.e. f ′(a–) ≠ f ′(a+), then such a point is called a sharp corner
or kink.

On computing, we find that f ′(a–) = 0 = f ′(a+).


So, f is differentiable at x = 0.

Case II : Oscillation Point


If in a continuous function f, either the left hand derivative or
right hand derivative does not exist at x = a, because of too much
For example, we see that the function y = |x| as shown in the oscillation of the graph in the neighbourhood of the point, then
figure has a graph which changes its direction abruptly at x = 0. such a point is called an oscillation point.

FREE BOOKS FOR JEE & NEET =>(@iitjeeadv)


3.12  
Differential Calculus for JEE Main and Advanced

 1 The graph of a continuous function f having a vertical tangent


 x sin , x  0 at (x0, f(x0)) can have
For example, the function f(x) =  x
0, (a) f ′(x) approaching either ∞ or –∞ as x → x0– and as x → x0+,
x 0


i.e. where the slope of the secant line approaches ∞ from
has an oscillation point at x = 0. both sides or approaches – ∞ from both sides, or,
This is illustrated in the following figure. (b) f ′(x) approaching ∞ from one side and –∞ from the other


side. In such a case, the function f is said to have a cusp
at x0.

Case III : Vertical Tangent


A continuous function may also fail to be differentiable at
x = x0 if the absolute value of its difference quotient approaches
Figure (a) Figure (b)
infinity. In this case, the function is said to have a vertical

    
tangent at x = x0. In figure (a), the slope of PQ approaches –∞ from both sides.

ps
In figure (b), the slope of PQ approaches ∞ from left side and
–∞ from right side.
el
Thus, the graph of the continuous function f(x) = x2/3 has
eh

a vertical tangent line at the origin, even though f is not


differentiable at x = 0, since, the function f(x) = x2/3 is continuous
je

at x = 0, and has infinite derivative at x = 0:


iit

L.H.D. = lim (0 − h ) − 0 = – ∞
2/3
@

h→0
−h
R.H.D = lim (0  h )  0 = ∞.
2/3

Definition The curve y = f(x) has a vertical tangent line h→0


h

at the point (x0, f(x0)) provided that f is continuous at a and Further, since the one-sided derivatives are infinity opposite
|f ′(x)| → ∞ as x → x0 ...(1) signs, we say that its graph has a cusp at x = 0.


Note that the requirement that f be continuous at x = x0 implies The graph of the function f(x) = 1– 5 x 2 , has a
that f(x0) must be defined. Thus, it would be pointless to ask cusp"(rather than a corner) at the point (0,1) because
about a line (vertical or not) tangent to the curve y = 1/x where
2 −3/ 5
x = 0. the derivative f ′(x) = − 5 x approaches ∞ as x → 0–
If f is defined on only one side of x = x0, we mean by (1) that
and approaches –∞ as x → 0+. The curve also has a vertical
|f ′(x)| → ∞ as x approaches x0 from that side.
tangent at the point.
For example, the function f(x) = x 1/3 is continuous at
x = 0, and Note: A function cannot have a vertical tangent at a


L.H.D. = hlim (0 − h ) 1/ 3
−0 = ∞ point of discontinuity. For example, y = sgn (x) does not
→0 have a vertical tangent at x = 0 even if both the one- sided

−h derivatives at x = 0 are infinite.
The figures give below show four curve elements that are
R.H.D = lim (0  h )  0 = ∞
1/ 3
commonly found in graphs of functions that involve radicals

h→0
h or fractional exponents. In all four cases, the function is not
Since both L.H.D. and R.H.D are infinite, the function has a differentiable at x0 because the secant line through (x0, f(x0))
vertical tangent at x = 0. and (x, f(x)) approaches a vertical position as x approaches
∴ The y-axis i.e. x = 0 is the vertical tangent to f(x) = x1/3 at x0 from either side. Thus, in each case, the curve has vertical

origin. Note that f is not differentiable at x = 0. tangent line at (x0, f(x0)).

FREE BOOKS FOR JEE & NEET =>(@iitjeeadv)


Differentiability 3.13

Now consider f(x) = 6x1/3 + 3x4/3.


2(2 x + 1)
f ′(x) = 2x–2/3 + 4x1/3 = 2x–2/3(1 + 2x) = .
x 2/3
There is point of vertical tangency at x = 0, since
2(2 x  1)
lim f ′(x) = lim =∞


x 0 x 2/3
 
x0

2(2 x  1)
lim f ′(x) = lim = ∞.
lim f ′(x) = ∞, lim f ′(x) = ∞ x 2/3


x0 x 0



x→x0 −
x→x0 +

It does not have a cusp at x = 0.

Case IV: Discontinuity


There is another way for a function not to have a
derivative. We shall prove in the next section that if f
is discontinuous at x = a, then f is not differentiable at
x = a. So, at any discontinuity, f fails to be differentiable.

lim f ′(x)= – ∞, lim f ′(x)= – ∞


x → x 0−


x → x 0+
ps
el
eh

The following figures of discontinuous functions suggest that


je

the secant lines donot approach a finite slope from atleast one
iit

side.
@

lim f ′(x) = ∞, lim f ′(x) = – ∞





x → x 0− x → x 0+

lim f ′(x)= – ∞, lim f ′(x) = ∞


x → x 0−



x → x 0+

Note that the third and fourth figures represent a cusp.


Consider f(x) = (x – 4)2/3.
2 2
f ′(x) = 3 (x – 4)1/3 = 3.4 Relation between Continuity and
3( x − 4)1/ 3


Differentiability
There is a vertical tangent and cusp at x = 4
since f(x) = (x – 4)2/3 is continuous at x = 4 and Theorem If a function f is differentiable at x = a then it

must be continuous at x = a.
2
lim f ′(x) = lim =∞ Proof Since f is differentiable at x = a,
3( x  4)1/ 3

 


x 4 x 4
2 f (a  h )  f (a )
lim f ′(x) = lim = –∞. f ′(a) = lim exists. ...(1)
x  4 3( x  4)1/ 3

 
h 0

x 4 h

FREE BOOKS FOR JEE & NEET =>(@iitjeeadv)


3.14  
Differential Calculus for JEE Main and Advanced

To prove that f is continuous at x = a, we must prove that For example, the functions f(x) = x and
lim  f (a  h )  f (a )   0 . 1
h 0 g(x) = x sin if x ≠ 0; g(0) = 0 are


x
Now lim  f (a  h )  f (a )   lim
 f (a  h )  f (a )  continuous at x = 0 but not differentiable at x = 0.
.h
h 0 h 0 h Since f is continuous at x = a, we have
 f (a  h )  f (a )  lim  f (a  h )  f (a )   0 . ...(1)
 lim . lim h  f '(a ). 0  0



h 0

h 0 h h 0
We need to check whether
{using (1)} f (a  h )  f (a )
f ′(a) = lim

exists. ...(2)
⇒ lim f (a  h )  f (a ) ⇒ f is continuous at x = a.



h 0 h
h 0

Further, if f is differentiable at every point of its domain, then 0


Now (1) suggests that the limit (2) is in   form, but
it is continuous in that domain. 0
we know that only some of these limits exist and hence we are
Example 1: Examine the differentiability of the
not certain about the existence of the derivative.


 cos x  1
 ,x0 In fact, continuity of a function at a point is a necessary
function f(x) =  x at x = 0 and comment condition for its differentiability at this point. However, it
0, x0 should be noted that the continuity of a function at a point is
on continuity at that point. not a sufficient condition for the derivative of this function to
exist at that point, i.e. the continuity of a function at a point
Solution: We have
cos x − 1 does not necessarily mean its differentiability at that point.
ps


f ( x ) − f ( 0) −0
f ′(0) = lim = x Thus, differentiability of a function is a stronger condition than
el
x →0 x−0 lim
x →0 x−0 continuity alone.  p 1
, x0
eh

 x sin
cos x − 1 − sin 2 x Example 3: Let f(x) =  x .
= lim = lim


 0
je

x →0 x2 x →0 x 2 (cos x + 1) , x 0

Find the values of p for which (i) f is continuous but not differ-
iit

  sin x  
2
 
= –  lim 
1 1 entiable at x = 0 (ii) f is continuous and differentiable at x = 0.
 
@

 xlim  =– 2.
 x 0  x    0 cos x  1 

Solution: f(0) = 0


Since, f is differentiable at x = 0, and using the above theorem, For continuity, lim f(x) = 0
x→0
it is also continuous at x = 0.
1
Example 2: Given f (x) = x2 . sgn (x), examine the =0.
lim xp sin


x


x→0
continuity and derivability at x = 0 .
This is possible only when p > 0. ...(1)

 x 2 if x0 For differentiability,


Solution: f (x) =  0 if x0 1
h p sin  0
f ( 0  h )  f ( 0)



 2 if x0 f (0)  lim  lim h should
 x h 0 h h 0 h
h2 − 0 1
f ′(0+) = lim =0 exist i.e. lim h p 1 sin should exist.

h→0 h h 0 h
Thus, f ′(0) will exist only when p > 1
h2 − 0 f(x) will not be differentiable if p 1
f ′(0 −) = lim − =0 ...(2)




h→0 h From (1) and (2),
⇒ f is derivable at x = 0. (i) f is continuous but not differentiable at x = 0 if

⇒ f is continuous at x = 0. 0 < p 1.
(ii) f is continuous and differentiable at x = 0 if p > 1.
The Converse of the Above Theorem Is Not

Necessarily True ax  b for
 for x  1
Example 4: If f (x) = 
ax  x  2b x  1
3



If f is continuous at x = a, then f may or may not be differentiable
at x = a. is differentiable for at x = – 1, find a and b.

FREE BOOKS FOR JEE & NEET =>(@iitjeeadv)


Differentiability 3.15

Solution: For differentiability, continuity is a necessary 1 


2
 b 
 a  4  2   bh  h
2


condition. Hence L.H.L. = R.H.L. at x = – 1. a  b h   2
⇒ – a + 1 = – a – 1 + 2b = lim 2  = lim  
h0 h h0 h
⇒–a–1+b+a=0 ⇒ b = 1.

f ( 1  h )  f (1) b
Now, f ′(–1–) = lim For existence of this limit a 2 ....(1)
h


h0 4
a (1  h )  b  (b  a ) ⇒ f must be continuous at x = ± 1/2.
= lim  1 
h

h0
Then, f '   = b.
a  ah  a 2 
= lim =a
h

h0
 1   1 
Now f '   = f '  
[a (1  h )  (1  h )  2b]  [b  a ]
3
2 
and, f ′(–1+) = lim   2 
h0 h
⇒ b = – 4.


= lim
ah (h 2  3h  3)  h  b  1
{ using b = 1} Using (1), a – 1 = 2 ⇒ a = 3.

h0 h Hence, a = 3 and b = – 4.
= 3a + 1. Theorem If a function f is discontinuous at x = a then it


Hence f ′ (–1+) = f ′ (–1–) ⇒ 3a + 1 = a ⇒ a = –
1
. is non-differentiable at x = a.
1 2 Proof If a function f is discontinuous at x = a then the
Thus, a = – , b = 1.

equation lim  f (a  h )  f (a )   0 does not hold true.
ps
2
h 0
Example 5: A function f is defined as ,
el
For f to differentiable at x = a,


1
eh

if | x |  1 f (a  h )  f (a )
| x | 2 f ′(a) = lim must exist.
f (x) =  h 0
je

. h
a  b x if | x |  1 0

2
 We know that this limit can only exist in   form,
iit

2 0
If f (x) is derivable at x = 1/2 find the values of a and b.
@

which is not possible because the numerator does not approach


 1 1 1 to 0 because the condition
 | x | , x   2 , x  2
Solution: f (x) =  lim  f (a  h )  f (a )   0 does not hold true.
h 0




 a  bx 2 ,  1  x  1 Hence, f is not differentiable at x = a.
 2 2
 1 1 Example 6: Comment on the differentiability of


 , x  x , x 1
x 2 f(x) = 
 at x = 1.
 x  2 , x  1

1 1
=  , x
 x 2 Solution: L.H.D. = f ′(1–) = lim f (1 − h ) − f (1)

h→0


 1 1 −h
a  bx ,   x  − −
2
1 h 3
 2 2 lim
= h→0 = ∞.
−h

1
1  1 2
f   h f   1  R.H.D. = f ′(1+) = lim f (1  h )  f (1)
 h→0

2 h 
lim   2
= 2 = h
h lim  
h0
h0 h (1  h  2)  3 h
1  = hlim
→0 = hlim
→0 = 1.
1 2  h   2h h

h
 2  lim
= lim = h0 h  1  h  = – 4. Since L.H.D. ≠ R.H.D. f(x) is not differentiable at x = 1.
1  2 

h0
h  h   If we use the previous theorem, we see that
 2 
L.H.L. = f(1–) = 1 and R.H.L. = f(1+) = 3
 1  1  1 f(1) = 3. Thus, the function is discontinuous and hence non-
f   h  f 
f '  =  2  2 differentiable. We need not find L.H.D. and R.H.D. here to
 2  lim
  h0 h find differentiability.

FREE BOOKS FOR JEE & NEET =>(@iitjeeadv)


3.16  
Differential Calculus for JEE Main and Advanced

Theorem If a function f is non-differentiable at x = a but xe1/ x



both the one-sided derivatives exist (though being unequal), 0
f ( x )  f ( 0) 1  e1/ x
then f is continuous at x = a. f ′(0 ) = lim
+
= lim
x 0


x 0 
x 0 x 

Proof To prove that f is continuous at x = a, we must prove



that lim  f (a  h )  f (a )   0 ...(1) e1/ x 1
= lim = lim
= 1. 1/ x

h 0 1 e 1/ x
1


x 0 e x 0

 f (a  h )  f (a )   0 Since f ′(0– ) ≠ f ′(0+), f(x) is non-differentiable at x = 0.


and hlim
0
...(2)
To check continuity, we use the previous theorem.

Since L.H.D. exists at x = a, Since both the one-sided derivatives exist (though they are
f (a  h )  f (a ) unequal), f is continuous at x = 0.
f ′(a–) = lim exists. ...(3)
h 0 h We verify this result below :


 f (a  h )  f (a )  xe1/ x
Now lim  f (a  h )  f (a )   lim .(h ) We have f(0–) = lim =0
h 0 h 0 h
x 0 1  e1/ x

 f (a  h )  f (a )  xe1/ x x
 f ′(0+) = lim = lim =0
 lim . lim( h )  f '(a ). 0  0 . 1 e x 0 1  e 1/ x


x 0  1/ x
h 0 h h 0
Also f(0) = 0
{using (3)}

∴ f(0–) = f(0+) = f(0) ⇒ f is continuous at x = 0.

Since R.H.D. exists at x = a,

f (a  h )  f (a ) Example 8: Show that the function f : R → R defined by
f ′(a+) = lim exists.
ps


h 0  1 2 


h
f(x) = x 1  3 sin(ln x )  , x ≠ 0 and f(0) = 0 is everywhere
...(4)
el

 f (a  h )  f (a )   
Now lim  f (a  h )  f (a )   lim
eh

.h continuous, but has no differential coefficient at the origin.


h 0 h 0 h
Solution: Let us first check the differentiability
 f (a  h )  f (a ) 
je


 lim . lim h  f '(a ). 0  0 f (0 − h ) − f (0)
iit

h 0 h h 0 f ′(0) = lim
h →0 −h

{using (4)}
@

Thus, both the conditions (1) and (2) are established. Hence,  1 
(0 − h ) 1 + sin (ln h 2 ) − 0
the function is continuous at x = a. =
lim  3 

Note that if any of the one-sided derivatives at x = a does not h →0 (−h)
exist then, f may or maynot be continuous at x = a.
 1 
lim 
= h→0 1  sin(ln h 2 ) 
Study Tip  

3
= does not exist.
Differentiable ⇒ Continuous

Hence f(x) is not differentiable at x = 0. We cannot say anything


Continuous 
 Differentiable about continuity immediately.

Discontinuous ⇒ Non-differentiable
Now we check the continuity at x = 0 :

Both one-sided derivatives exist ⇒ Continuous Non-
differentiable 
 Discontinuous lim f(x) = lim f(0 + h)


x→0 h→0
1/ x
xe  1 
Example 7: If f(x) = , x ≠ 0, f(0) = 0, examine lim 
= h→0 1  sin(ln h 2 )  = 0
1 + e1/ x


 

3
the continuity and differentiability at x = 0. Finally f(x) is continuous at x = 0 but not differentiable at x = 0.
Solution: We check the differentiability first.


 x 2 sgn[ x ] + {x} ,
 0≤x<2
xe1/ x Example 9: If f(x) =  ,
0 sin x + | x − 3 | ,
 2≤x<4


f ( x )  f ( 0)
= lim 1  e
1/ x
f ′(0–) = lim comment on the continuity and differentiability of f(x) at
x 0

x 0 
x 0  0 x
x = 1, 2.
e1/ x Solution: Looking at the kind of functions involved we
= lim = 0.


x 0  0 1  e1/ x check the continuity first.

FREE BOOKS FOR JEE & NEET =>(@iitjeeadv)
Differentiability 3.17

Continuity at x = 1 mathematicians when, Weierstrass gave an example of such


lim f(x) = lim (x2 sgn [x] + {x}) a function.
x  1

x  1
Consider one of his functions
=1 sgn (0) + 1 =1
 n


2
lim f(x) = lim (x 2
sgn[x] + {x}) f(x) =   3  cos (9n x ),

x  1 x  1


n 0
=1+0=1 a formula that expresses f as an infinite sum of cosines with


f(1) = 1. increasingly higher frequencies. By adding wiggles to wiggles

Since, L.H.L = R.H.L = f(1), f(x) is continuous at x = 1. infinitely many times, the formula produces a graph that is too
Now, differentiability at x = 1, bumpy in the limit to have a tangent anywhere.
f ′(1–) = lim f (1 − h ) − f (1) Another example of such a function defined on R is

h0 

−h 
1
= hlim
(1  h ) sgn[1  h ]  1  h  1
2
=1
f(x) =  2n
cos(3n x), for all x ∈ R,


0 
h n1

f ′(1+) = lim f (1  h )  f (1) It can be shown that f is continuous everywhere but is not
h0



h derivable anywhere. The proof of these assertions is, however,


(1  h ) 2
sgn[1  h ]  {1  h}  1 beyond the scope of the present book.
= hlim
0 
Remark In the definition of derivative we are regarding the

h


derivative f ′(x0) as a real number associated with the difference
(1  h ) 2  h  1 lim h + 3h = 3.
2
quotient (f(x) – f(x0))/(x – x0) considered as a function which
= hlim
0  = h0 
ps
h h

has values for each x in the domain of f with the exception of x0.
Here, f ′(1+) ≠ f ′(1–). In fact, the definition merely equates the ideas of derivability of
el
Hence f(x) is non-differentiable at x = 1. f at x0 with the notion of continuous extension of the difference
eh

Now at x = 2, quotient to x0.


lim f(x) = lim x2 sgn [x] + {x} = 4 . 0 + 1 = 1 For if we write Q(x) = (f(x) – f(x0))/(x – x0), the domain of Q(x)
je

x 2 x 2
is Df – {x0} so that x0 is a removable discontinuity for Q(x), and
iit

lim f(x) = lim (sinx + |x – 3|) = 1 + sin 2 we have the continuous extension Q*(x) defined to be Q(x)

x 2 x 2
for x ≠ x0 with Q*(x0) = f ′(x0). In symbols,
@

Since, L.H.L ≠ R.H.L, f(x) is discontinuous at x = 2 and hence


it is non-differentiable at x = 2.  f (x) − f (x 0 )
Note: It can be proved that a function f is continuous from  , x ∈ Df − {x 0 }
Q*(x) =  x − x 0


the left at those points where it is differentiable from the left,  f '( x )

 0 , x = x0
and f is continuous from the right at those points where it is
differentiable from the right. Thus, for instance, if f is continuous on its domain, then Q(x)
Now, consider the function, is continuous on the same domain minus x0, but the continuous
extension Q*(x) would be continuous on the whole domain of
f(x) = 2x x 3 − 1 + 5 x 1 − x 4 + 7x2 x − 1 + 3x f. The latter idea is expressed geometrically by rotating secant
Here the domain of the function is x ∈ {1} line through a given point on a curve continuously to a tangent
Since, it is a point function, there is no question of continuity line as a limiting line.
and hence differentiability at x = 1. Theorem Suppose f is differentiable at x = c. Then

∴ f (x) is neither continuous nor differentiable at x = 1. f(x) = f(c) + f ′(c) (x – c) + e(x)(x – c),
Suppose that the function f is continuous everywhere. At how where e(x) → 0 as x → c and e(c) = 0.
many points do you suspect that f can fail to be differentiable ? Conversely, let f be defined on an interval D and let
One might think that if a function is continuous on an interval, c ∈ D. Suppose there is a constant K such that
then it might fail to be derivable at finitely many points at the f(x) = f(c) + K(x – c) + e(x)(x – c), where e(x) → 0 as x → c.
most. This, however, is far from the truth. Then f is differentiable at c and f ′(c) = K.
What ′s the worst such function you can think of ? Can a f (x)  f (x)
continuous function fail to have a derivative at every point? Proof If lim  f '(c) exists, then
x c xc

The answer, surprisingly enough, is yes.
There exist functions which are continuous on R but which f ( x ) − f (c)
e(x) = − f '(c) → 0 as x → c
are not derivable at any point whatsoever. It did give a jolt to x−c

FREE BOOKS FOR JEE & NEET =>(@iitjeeadv)
3.18  
Differential Calculus for JEE Main and Advanced

Conversely, given f(x) = f(c) + K(x – c) + e(x)(x – c), where x +1 2

e(x) → 0, then = e x ,
Example 11: Let f (x) x<0





= 0, x=0
f ( x ) − f (c)



= K + e(x) → K = x2, x>0
x −c



Discuss continuity and differentiability of f (x) at x = 0.

Hence the derivative exists and equals K.
Solution: At x = 0



Example 10: Check the continuity and differentiability
f ′(0–) = lim f (0 − h ) − f (0)


of f(x) at x = 1, where f(x) = |x – 1| ([x] – {x}). h→0
−h
f (1  h )  f (1)  1
  h 
Solution: f ′(1+) = lim 1
h 0 e  h 0 −


h lim lim e– h · e h
= h→0 = h→0
h


| h | ([1  h ]  {1  h})  0 h (1  h ) −h
= lim = lim  1.
h 0 h 0

h h lim 1
= h→0 put t = – we get


f (1)  f (1  h ) h
f ′(1–) = lim t
h 0 lim t et = lim t = lim

h = t t − t t
−e − t


e
0 | h | ([1  h ]  {1  h})
 lim (applying L’ Hospital rule)
h 0

h
   
= 0.

h[0  (1  h )] f ′(0+) = lim f (h ) − f (0) = lim h − 0
2
= lim  1
ps

h 0 h→0 h→0

h h h
Since f ′(1+) and f ′(1–) are not equal, hence f is not differentiable lim
el
= h→0 h = 0.
at x = 1. Since, the one-sided derivatives exist, f is continuous

eh

at x = 1. Hence, function is continuous and differentiable at x = 0.


je

Concept Problems B
iit

@

1. Consider the graph of a function f. 3. Consider the graph of a function f.





(a) For which numbers a does lim f(x) exist, but f is not
x →a


continuous at a ?
(b) For which numbers a is f continuous at a, but not
(a) For which numbers a does lim f(x) exist, but f is not


differentiable at a ? x →a


continuous at a ?
2. Consider the graph of a function f. (b) For which numbers a is f continuous at a, but not



differentiable at a ?
4. The graph of f is shown. State, with reasons, the number

at which f is not differentiable.

(a) For which numbers a does lim f(x) exist, but f is not


x →a
continuous at a ?
(b) For which numbers a is f continuous at a, but not


differentiable at a ?

FREE BOOKS FOR JEE & NEET =>(@iitjeeadv)
Differentiability 3.19

9. A function f is defined as follows :


2  x if x  1


5. Let f(x) =  2 Is f differentiable at 1? 
 x  2 x  2 if x  1 x0

1 for
Sketch the graphs of f and f ′. 
f(x) =  1  sin x for 0  x  2 .



 −2 x if x < 1  
2
2  x  2 for 
x
6. Let f(x) =  . Show that f is continuous 2
 x − 3 if x ≥ 1

Discuss the continuity and differentiability at x = 0 and
but not differentiable at x = 1.


x = π/2 .
 ax 2  1, x  1
 e x  x , x  0
2

7. If f ( x )   2 is differentiable at x = 1,
10. If f ( x )   is differentiable at x = 0, then
 x  ax  b, x  1

ax  b, x  0


then find a and b.
find a and b.
1
 sin x , x  0
2
8. If f(x) =  x , discuss the continuity and [2x]  x , x  1
0, 11. If f(x) =  comment on the continuity

x0  {x}  1 , x  1


differentiability of f(x) at x = 0. and differentiable at x = 1.

Practice Problems B

ps
12. Check the continuity and differentiability of 16. Examine for continuity and differentiability at the
el

 − x, x < 0 points x = 1 and x = 2, the function f defined by
eh

 2
f(x) =  x , 0 ≤ x ≤ 1 at x = 0 and x = 1. x [x] , 0 x 2
je

f(x) = 

 3
 ( x  1) [ x ] , 2  x  3

 x − x + 1, x > 1
iit

13. Test the continuity and differentiability of the function where [x] = greatest integer less than or equal to x.
@


 x (21x − 2 −1x )
 1 ,x≠0 17. Show that the function

−1x
f(x) =  (2 + 2 )
x
 sin x.cos(1 / x ) at x  0,


0 ,x=0 f(x) = 
 x  0.

0 at
at x = 0.


14. Find the value of ‘a’ for which is continuous at the point x = 0, but does not have even


sgn(cos x ) , x   / 2 one sided derivatives.
f(x) =  is differentiable at
 a  sin x , x   / 2  x

 sin , x 1
x = π/2. 18. If f ( x )   2 , where [.] denotes the

[2 x  3]x , x  1


15. Check the differentiability of the function

 2 cos x 
f(x) = sin–1   at x = 0, π. greatest integer function, then find whether it is
 1  cos 2 x 

differentiable at x = 1.

zero running through negative values (∆x < 0) only. This is
3.5 Derivability at End Points
termed as the left hand derivative of f at x (denoted as f ′(x–)).

If this exists, then it is considered as the derivative of the
f ( x  x )  f ( x ) function at x.
Till now, the derivative f ′(x) = lim was
x
x 0
Similarly, if a function is defined only in the right neigh-
calculated when the function f was defined in a neighbourhood bourhood of x, then the above limit can be calculated when
of the point x, where ∆x tends to zero attaining both positive ∆x tends to zero running through positive values (∆x > 0)
and negative values. only. This is termed as the right hand derivative of f at x
Suppose that a function is defined only in the left neighbourhood (denoted as f ′(x+)). If this exists, then it is considered as the
of x, then the above limit can be calculated when ∆x tends to derivative of the function at x.

FREE BOOKS FOR JEE & NEET =>(@iitjeeadv)


3.20  
Differential Calculus for JEE Main and Advanced

Definition If a function f is defined only in the left Example 1: Discuss the differentiability of the function



neighbourhood of a point x = a (for x ≤ a), it is said to be
differentiable at a if
f(x) = x ( x − ( x + 1) ) at x = 0.
Solution: Since domain of f(x) is [0, ∞), we find the
L.H.D. = f ′(a–) = lim f (a − h ) − f (a ) exists.



h→0
−h
R.H.D.
f ( 0  h )  f ( 0)
In such a case f ′(a) = f ′(a–). ∴ f ′(0+) = lim
h 0 h


Similarly, if a function f is defined only in the right
neighbourhood of a point x = a (for x ≥ a), it is said to be lim h ( h − h + 1) − 0
= h→0
differentiable at a if


h
R.H.D. = f ′(a+) = lim f (a  h )  f (a ) exists.
lim ( h − h +1) = – 1

h→0
h = h→0
In such a case f ′(a) = f ′(a+).


For example, f(x) = x3/2 is differentiable at x = 0 but g(x) = x Hence, f is differentiable at x = 0.
is not differentiable at x = 0.
Since, f and g are defined only in the right neighbourhood of 3.6 Differentiability over an Interval
a point x = 0, we calculate only R.H.D.


f (x) is said to be differentiable in an interval if it is differentiable
h 3/ 2  0
f ′(0 ) = lim
+
 lim h1/ 2  0 exists. at each and every point of the interval.
h 0

h→0 h
Hence f ′(0) = 0. Differentiability Over an Open Interval
ps
h1/ 2  0 A function f(x) is said to be differentiable in an open interval
g ′(0+) = lim  lim h 1/ 2  
el
h 0 (a, b) if it is differentiable at all interior points in (a, b).

h→0 h
For example, f(x) = | x – 1| is differentiable in (0, 1).
eh

does not exist.


Hence, x is not differentiable at x = 0. y = sin–1x is differentiable in (–1, 1).
je

Note: The four inverse trigonometric functions sin–1x, Similarly, a function f(x) is said to be differentiable in the open
iit

interval (a, ∞) or (–∞, a) or (– ∞, ∞) if it is differentiable at




cos–1x, cosec–1x, and sec–1x are not differentiable at the points
@

x = ± 1. all interior points in the interval.


Consider f(x) = sin–1x. For example, f(x) = x is differentiable in (0, ∞).
At x = 1, f is defined only in the left neighbourhood. Differentiability Over a Closed Interval
f (1  h )  f (0) A function f(x) is said to be differentiable in a closed interval
Hence, f ′(1–) = lim
h 0 h [a, b] if
sin 1 (1  h )  sin 1 1 (i) it is differentiable at all interior points in (a, b).

= lim (ii) R.H.D. = f ′(a+) exists at the left endpoint a.
h

h 0

(iii) L.H.D. = f ′(a–) exists at the right endpoint b.

1 1
= lim sin (1  h )   / 2  lim  cos (1  h ) If a function is defined in a closed interval then, we by
h h

h 0 h 0 derivatives at the end points of the interval, we always mean
1 the one-sided derivatives : the right hand derivative at the left
= lim sin 2h  h 2
 end point and the left hand derivative at the right end point.

h 0 h This interpretation of differentiability at the end points of the
Thus, f(x) = sin–1x is not differentiable at x = 1. interval of definition is analogous to that of continuity.
For example, f(x) = x3/2 is differentiable in [0, 1].
f(x) = sin x is differentiable in [0, 2π].
It should be noted that not all the elementary functions are
differentiable everywhere in their domain.
Note:


1. All polynomial, exponential, logarithmic and trigonometric

functions (inverse trigonometric not included) are
There is a vertical tangent to the graph of f at x = 1. differentiable at each point in their domain.

FREE BOOKS FOR JEE & NEET =>(@iitjeeadv)


Differentiability 3.21

2. Modulus function and signum function are non differen- The suspicious points for continuity and differentiability are

tiable at x = 0. Hence, y = | f(x) | and y = sgn(f(x)) should obtained when:
be checked at points where f(x) = 0. 1
3. The inverse trigonometric functions y = sin–1x, cos–1x, (i) x + becomes an integer for 0  x  1
3



cosec–1x, and sec–1x are not differentiable at the points 2
x = ± 1. Hence, y = sin–1(f(x)), cos–1(f(x)), cosec–1(f(x)), and ⇒ x= .
3



sec–1(f(x)) should be checked at points where f(x) = ± 1.
4. Greatest integer function and fractional part functions are (ii) sin πx becomes an integer for 0  x  1



non differentiable at all integral x. Hence, y = [f(x)] and 1
⇒ x = 0,
y = {f(x)} should be checked at points where f(x) = n, n ∈ I.



2
5. Further, a function should be checked at all those points (iii) The point of change in definition is x = 1.


where discontinuity may arise.
x (iv) 2x becomes an integer for 1 ≤ x ≤ 2


Example 1: Show that f(x) = 1+ | x | is differentiable 3
⇒ x = 1, , 2.




for all x ∈ R. 2
 x 4 4
1  x , x  0 (v) x − becomes zero for 1 ≤ x ≤ 2 ⇒ x = .
3


3

, x0, (vi) The end points of closed domain [0, 2] ⇒ x = 0, 2.
Solution: Since f(x) =  0

 x Hence, the points where we should check the continuity



 , x0 and differentiability are
1  x ps 1 2 4 3
x = 0, , , 1, , , 2.
clearly f is differentiable for all x ∈ R – {0}. We need to check

2 3 3 2
We check continuity at x = 1.
el
at x = 0.

h
eh

−0  1
f ( 0  h )  f ( 0 ) 1 + h lim f(x) = lim  x   [sin πx] = 0
f ′(0+) = lim = lim x1  3

 
x1
je

h 0 h →0

h h
iit

1
1  1
= lim
h 0 1  h
lim f(x) = lim [2x] sgn  x   = 2 {– 1} = – 2

 3

@

 
x1 x1
−h
−0 Since, L.H.L ≠ R.H.L. f(x) is discontinuous at x = 1 and
f ( 0 − h ) − f ( 0 ) = lim + h

f ′(0 ) = lim 1

hence it is non-differentiable at x = 1.
−h h →0 −h

h→0
Example 3: Check differentiability of f(x) in [–2, 2], if
1


= lim 1  
h 0 1  h
cos 2 (| x | {x}) x 1

⇒ f ′(0+) = f ′(0–). Hence f is differentiable at x = 0. f(x) = 
 4 x 2  12 x  9{x}

Finally, f is differentiable for all x ∈ R.  x 1
Example 2: where {.} denotes the fractional part function.


 1 Solution: We have
  x   [sin x ] 0  x  2


 3  
Let f(x) =  cos 2 (| x | {x}) x 1
[2 x ]sgn  x  4  1  x  2 f(x) = 

  4 x 2  12 x  9{x}
 3 

  x 1
where [.] and {.} denote the greatest integer function and  
fractional part function respectively. Find the points at which cos (| x | {x}) x 1
continuity and differentiability should be checked. Also check i.e. f(x) =  2
| 2 x  3 | {x}

the continuity and differentiability of f(x) at x = 1. x 1
Now, we have
 1
  x   [sin x ] 0  x  1 |x| – {x} = |x| – x + [x] = –2x – 2, –2 ≤ x < –1
 3


Solution: f (x) =  = –2x – 1, –1 ≤ x < 0



[2 x ]sgn  x  4  1  x  2 0≤x<1


= 0,
  3 


 and |2x – 3|{x} = |2x–3|(x – [x])

FREE BOOKS FOR JEE & NEET =>(@iitjeeadv)
3.22  
Differential Calculus for JEE Main and Advanced

= (3 – 2x)(x – 1), 1 ≤ x < 3/2 At x = 3/2, we have




= (2x – 3)(x – 1), 3/2 ≤ x < 2 3  3
f   hf  


 3 
f '    lim   2
= 0, x=2 2


 2  h 0


Thus, we have h
 
  3 
cos 2 (2 x  2) ,  2  x  1 2h   h  1   1
  lim  
2
1
cos  (2 x  1) ,


1  x  0 h 0 h
 2  3 3 
f(x) = cos 0, 0  x 1 f   − f  − h
 3−   2 2 

 f '   = lim
(3  2 x )( x  1), 1 x  3/ 2


 2  h →0 h
(2 x  3)( x  1), 3/ 2  x  2
 3 
0  2h   h  1 
0, x2
 2   1
 lim


We need to check differentiability of f(x) at h 0 h
x = –2, –1, 0, 1, 3/2, 2. We first check continuity. ⇒ non-differentiable at x = 3/2


At x = 2, we have Hence, f is differentiable on [–2,2] except at

f(2) = 0 and f(2–) = (2 . 2 – 3) (2 – 1) = 1. x = –1, 0, 1, 3/2, 2.

⇒ discontinuity at x = 2.

Example 4: If x + 4 y = 6y, then find whether y as a

At x = 3/2, we have


function of x is continuous and derivable.

 3 
ps
3 Solution: We have x + 4 y = 6y
f    f    0


2
el
 2 

1
 x , x  0
eh

 3   3  3  ⇒ y = f(x) =  2
and f  2    3  2.    1  0

 1 x , x 0
je

   2  2 
10
iit

⇒ continuity at x = 3/2.
lim f(x) = lim f(0 + h) = lim 1 h = 0

@

At x = 1, we have f(1+) = f(1) = 0 and f(1–) = cos 0 = 1 h→0 h→0



x  0 2

⇒ discontinuity at x = 1.
lim f(x) = lim f(0 − h) = lim 1 (0 − h) = 0

At x = 0, we have
x0 h→0 h → 0 10




f(0+) = f(0) = cos 0 = 1 and f(0–) = cos  0 ⇒ f is continuous at x = 0 and hence for all x ∈ R.
2


⇒ discontinuity at x = 0.
f ′(0+) = lim f  0  h   f (0)

At x = –1, we have h→0

h


f(–1+) = f(–1) = cos  0 and f(–1–) = cos0 = 1 h −0
1

2 2
⇒ discontinuity at x = –1. = hlim
→0 =

h 2

f ′ (0–) = lim f  0  h   f (0)
At x = –2, we have f(–2+) = f(–2)
⇒ continuity at x = –2. h→0

h
Hence, we need to check differentiability only at
x = –2, 3/2. − h −0 1
10

At x = –2 which is the left endpoint, we have = hlim
→0 −h = 10


f (2  h )  f (2)
f ′(–2+) = lim ∴ f is not derivable at x = 0.

h 0

h ⇒ f is derivable at all x except x = 0.

 
cos (4  2h  2)  cos (4  2) Example 5:


= lim 2 2
 1  x , x  1

h 0 h  2
| x  1 |, 1  x  0
cos (1  h )  1  sin(  h ) If f(x) =  ,
= lim  lim 0  k ( x  1), 0  x 1

h 0 h 0

h 1
| x  1 |, x 1
⇒ f is differentiable at x = –2. 

FREE BOOKS FOR JEE & NEET =>(@iitjeeadv)
Differentiability 3.23

then find the value of k so that f(x) becomes continuous at


x = 0. Hence, find all the points where the functions is non-
differentiable.
Solution: The only possible value of k is 1 which is


obtained by equating the one-sided limits at x = 0.
Now, we draw the graph of y = f(x).
From the graph, it is clear that the function is non-differentiable
at x = 0, ±1 because at each of these points the graph is having
a corner.
Graph of y = Min {|x|, |x – 2|, 2 – |x – 1| :

Example 6: If f(x) = |x + 1| (|x| + |x – 1|), then draw the




graph of f(x) in the interval [–2, 2] and discuss the continuity
ps
and differentiability in [–2, 2] It is clear from the figure that f(x) is continuous ∀ x ∈ R and
5
el
1
Solution: Here, f(x) = |x + 1| ( |x| + |x – 1| ) non-differentiable at x = – , 0, 1, 2, .
2 2
eh


 ( x  1)(2 x  1), 2  x  1 Example 8: Consider the function
je

( x  1)(2 x  1), 1  x  0




 f(x) = min(|x|, 1 − x 2 ), –1 ≤ x ≤ 1
= 
iit

( x  1), 0  x 1




= [|x|], 1 < |x| < 2
@


 ( x  1)(2 x  1), 1 x  2


Plot the curve and discuss its continuity and differentiability.
The graph of f(x) is

Solution: y = [|x|], x ∈ (–2, –1) ∪ (1, 2)




⇒ y = 1.

The curve shown by thick lines represents the curve. From the
graph, we can see that f(x) is
which is clearly, continuous for x ∈ R. continuous on x ∈ (–2, 2) – 1 and
The graph has corners at x = –1, 0, 1.
Hence, f(x) is differentiable for x ∈ R – {–1, 0, 1}.  1 
differentiable on x ∈ (–2, 2) – 1, , 0 .
Example 7: If f(x) = Min {|x|, |x – 2|, 2 – |x – 1|}, then  2 


draw the graph of f(x) and also discuss its continuity and Example 9: Let f (x) = x3 – x2 + x + 1 and


differentiability.
max{f ( t ); 0  t  x}, 0  x  1
Solution: f(x) = Min {|x|, |x – 2|, 2 – |x – 1|} g(x) =  .
 3  x; 1 x  2



We compare the graphs of
y = |x|, y = |x – 2| and y = 2 – |x – 1| and Discuss the continuity and differentiability of the function g(x)
choose the least value at each x. in the interval (0, 2).

FREE BOOKS FOR JEE & NEET =>(@iitjeeadv)


3.24  
Differential Calculus for JEE Main and Advanced

Solution: f(t) = t3 – t2 + t + 1 1 if x  0



f ( t )  3t  2 t  1
2
Solution: Since f (x) = 0 if x  0



1 if x  0

Its discriminant = (– 2)2 – 4.3.1 = – 8 < 0 
and coefficient of t2 = 3 > 0
1 if g ( x )  0

Hence f ( t )  0 for all real t. 
⇒ f(t) is strictly increasing f(g(x)) = 0 if g ( x )  0


1 if g( x )  0

Thus f(t) is maximum when t is maximum and tmax = x 
max f(t) = f(x)

1 if x  (, 1)  (0,1)
 x 3  x 2  x  1, 0  x  1 
⇒ g(x) =  = 0 if x  0,1,  1
3  x , 1 x  2

1 if x  (1, 0)  (1, )



Now it can be easily seen that f(x) is continuous in (0, 2)
and differentiable in (0, 2) except at x = 1 because at x = 1. The graph of y = f(g(x)) is shown below
LHD > 0 while RHD = – 1 < 0.
Example 10: Check the differentiability of the function


f (x) = max {sin–1 |sin x|, cos–1 |sin x|}.
Solution: sin–1 |sin x| is periodic with period π


ps
 
 x , n  x  n 
el
2
⇒ sin–1 |sin x| = 


  x , n   x  n  
eh

 2 From the graph we can easily conclude that f(g(x)) is


je

discontinuous and non derivable at – 1, 0, 1.


π Now, g(f(x)) = f (x) ( 1 – f 2(x) )
iit

Also cos–1 |sin x| = – sin–1 |sin x|


2 = sgn x (1 – sgn2 x) = 0 for all x,
@



 
Hence g (f(x)) is continuous and derivable for all x.
  
 x,  x , n  x  n 
 2 2 Example 12: Find the interval of values of k for which
f (x)= max 
 



   x, x    the function f(x) = |x2 + (k – 1) |x| – k| is non differentiable at

, n   x  n   five points.
 2 2
Solution:
 


 2  x, n   x  n 
4
f(x) = |x2 + (k – 1) |x| – k| = |(|x| – 1) (|x| + k)|


 x,   Also f(x) is an even function and f(x) is not differentiable at
n   n   five points. So |(x – 1) (x + k)| is non differentiable for two
 4 2
⇒ f (x) =  positive values of x.
 3

  x, n   x  n 
 2 4
  3
x  , n   x  n  
 2 4

⇒ f (x) is not differentiable at x = 0, π , π , 3π , π...



4 2 4


⇒ f (x) is not differentiable at x = .
4

Example 11: Let f (x) = sgn x and g (x) = x (1 – x2).


Investigate the composite functions f(g(x)) and g(f(x)) for ⇒ Both the roots of (x – 1) (x + k) = 0 are positive.

continuity and differentiability. ⇒ k < 0 ⇒ k ∈ (–∞, 0).


FREE BOOKS FOR JEE & NEET =>(@iitjeeadv)
Differentiability 3.25

Concept Problems C


1. Indicate the points at which the following functions have  12
no finite derivative :  if x  3,
(5  x ) 2
x +1 1 (ii) f(x) = 
(b) y = sin–1  2



(a) y = cos–1
2 x  x  3x  3 if x  3


2. Show that for any positive integer n, the root function f(x) 6. Discuss the continuity and differentiability of the function
= x1/n is not derivable at the origin. Is it continuous at the f(x) = sinx + sin | x |, x ∈R. Draw a rough sketch of the
origin ? graph of f(x) .
3. (a) Find f ′(0–) and f ′(0+) given f(x) = |x|. 7. (a) Sketch the graph of the function f(x) = x|x|.

(b) The function f(x) = |3x – 10| is differentiable except (b) For what value of x is f differentiable?
at a single point. What is this point, and what are the (c) Find a formula for f ′(x).
values of its left hand and right hand derivatives of 8. If f(x) = [x] + [1 – x] = – 1 ≤ x ≤ 3, Draw its graph and
f there ?


comment on the continuity and differentiability of f(x).
4. Sketch the graph of the function f(x) = x + |x|. Then 9. Show that the Dirichlet function is nowhere derivable.

investigate its differentiability. Find the derivative f ′(x) 10. Examine the differentiability of
where it exists. Also find the one-sided derivatives at the

points where f ′(x) does not exist.  xe|x| , x  1
5. Investigate the differentiability of the functions: f(x) =  2 .
ex , x  1
ps

(i) f(x) = |x3|

el

Practice Problems C
eh

je

x 1 1
iit

11. Let f(x) = for x ≠ 0, f(0) = 0. Is the function



x
@

f(x) continuous and differentiable at x = 0?


12. Investigate the differentiability of the function

11  6 x  x 2 if x  3
f(x) =  2 .
 x  6 x  29 if x  3

min{x , x 2 }, x ≥ 0 1  x , ( 0  x  1 )

13. If f(x) =  then find the 17. f(x) =  x  2 , (1  x  2 ) .
max{2 x , x − 3}, x < 0
2




4  x , ( 2  x  4 )
points of non differentiability of f(x).
Discuss the continuity and differentiability of y = f (f(x))
14. Find a and b, given that the derivative of f(x) exists for 0 ≤ x ≤ 4.

everywhere, when

18. Draw the graph of f(x) = max.{sin x, cos x, 1 – cos x} and

ax 2 − bx + 2, x < 3 find the number of points belonging to (0, 2π) where f(x)
f(x) =  2 . is non differentiable.
 bx − 3, x ≥ 3

0 if x is an integer

19. Let f(x) = [x] and g(x) =  then define
15. Check differentiability of f(x) at x = 0, 2
 x otherwise



 1  g o f and f o g and examine their differentiability.
where f(x) = x 1  sin(ln x 2 )  , x ≠ 0
 3  20. Let f(x) = cos x and

= 0, x=0  minimum {f ( t ) ; 0  t  x}, x  [0, ]


where [.] represents the greatest integer. g(x )   .
 sin x 1, x

16. Copy the graph of the function. Then sketch a graph of its
derivative directly beneath. Draw the graph of g(x) and comment on differentiability.

FREE BOOKS FOR JEE & NEET =>(@iitjeeadv)
3.26  
Differential Calculus for JEE Main and Advanced

3.7 Alternative Limit Form of



the Derivative
The following is an alternative limit form of the derivative.
The derivative of f at x = a is
f (a  h )  f (a ) f ( x )  f (a )
f '(a )  lim = lim
 x a

h 0 h x c

f ( x ) − f (a )
f ′(a) = lim .
x−a

x →a

The use of this formula simplifies some derivative calculations. Caution


Y y = f(x) secant slope is f (a  h )  f (a  h )
f(x) – f(a) The quotient may have a limit as h → 0
2h
x–a
Q(x), f(x)) when f has no derivative at x = a.
For instance, if f(x) = |x| then


f(x) – f(a)
| 0  h | (0  h |
P(a, f(a)) lim = 0.

h 02h
As we can see, the limit exists even though f(x) = |x| has no
ps
h=x–a derivative at x = 0.
el
f (a  h )  f (a  h )
In fact, lim
eh

a x X h 0 2h
 f (a  h )  f (a ) f (a  h )  f (a ) 
je

The one-sided derivatives of f at a are given by 1


= lim  
h 
  
iit

2 h 0 h
f ( x )  f (a ) f ( x )  f (a )
f ′(a+) = lim and f ′(a–) = lim .
x a x a
@

x a  x a 

1
(f ′(a+) + f ′(a–)).
=
f ′(a) exists if they are equal.

2
We notice that it is equal to f ′(a), if f is differentiable at x = a.
Example 1: If f is a differentiable function with f ′(2) = 3 Hence, if f ′(a) exists, then


f ( x )  f (2) f (a  h )  f (a  h ) = f ′(a).
then find the value of lim . lim
x 2


x 2
h 0 2h
f ( x )  f (2) x+ 2 Example 2: If f ′ (5) = 7 then find the value of



Solution: lim ·
x 2 lim f (5  t )  f (5  t ) .


x 2 x+ 2 t→0

2t
= lim
f ( x )  f ( 2)
x2
·  x 2  Solution: lim f (5  t )  f (5  t )


x 2


t→0
2t
= f ′(2) · 2 2 = 6 2 . = 7 . lim f (5  t )  f (5  t )

t→0

2t
Centred Difference Quotient
 f (5  t )  f (5) f (5  t )  f (5) 
The centred difference quotient = lim   
t→0  2t 2t 

f (a  h )  f (a  h ) f  (5 ) f  (5 )
= + 2 = f ′ (5) = 7.

2h 2

is used to approximate f ′(a) in numerical work because (i) its Note:
limit as h → 0 equals f ′(a) when f ′(a) exists, and (ii) it usually


(i) If f ′(a) exists and ψ(h) → 0 as h → 0, then
gives a better approximation of f ′(a) for a given value of h than

f (a  h )  f (a ) f (a   (h ))  f (a )
Fermat's difference quotient lim = f ′(a).
(h )

h h 0

FREE BOOKS FOR JEE & NEET =>(@iitjeeadv)


Differentiability 3.27

(ii) If f ′(a) exists, and ψ(h) → 0 and φ(h) → 0 as h → 0, 2h 1 1


= f ′(2) × lim  =6×2×

= 3.
f (a   (h ))  f (a  (h )) h 0 1  h f (1) 4
then lim = f ′(a).
 (h )  (h )

h 0
Example 6: If f ′(2) = 4 then find the value of



lim f (2  h )  f (1  cos h) .
2
Example 3: If f ′(a) = 3, evaluate the limit


h→0
h2


lim f (a  2h )  f (a  3h ) .
Solution: lim f (2  h )  f (1  cos h)
h→0 2

h




h→0
Solution: lim f (a  2h )  f (a  3h ) h2


h→0
lim f (2  h )  f (2  cos h  1). h  (cos h  1)
2 2
h = h→0 2
h  (cos h  1) 2


= 5 . lim f (a  2h )  f (a  3h )
h
now as h → 0, h → 0 and (cos h – 1)→ 0
2

h→0
5h


= f ′(a) × 5 = 3 × 5 = 15. = f ′(2) × lim h − (cos h − 1)
2


h→0
Example 4: If f is a differentiable function and h2

 




f (  h )3  f (3 ) 3
lim = π, = 4× = 6.

2

h0 h
then find the value of f ′(π3). Differentiability of Parametric Functions

Let the function y = f(x) be defined by
Solution: π = lim 
f (  h )3  f (3 ) ...(1)
psx= x(t) and y= y(t), where t is the parameter.



h0 h Suppose we are interested in the derivative of y = f(x) and the
el

f   (3h   3 h  h 3 )  f (3 )
3 2 2
 possibility of its existence.
eh

lim
= h For parametric functions, the derivative is defined as
h

0
je

dy
where ( 3h 2   32 h  h 3 )→ 0 as h → 0. dy dt
iit

f ′(x) = =
dx dx

f((  h )3 )  f (3 ) (3h 2   32 h  h 3 )
@

= hlim  lim dt
0 (3h   3 h  h ) h 0
2 2
h 3

The issue of differentiability is investigated using the existence


= f ′ (π3) · 3 π2.
of the limit :

1
⇒ π = f ′ (π3) · 3 π2 ⇒ f ′ (π3) = . y ( t  )  y ( t )

3π y ( t  )  y ( t )
Alternative: dy
 lim   lim
Using L ′Hospital ′s rule in (1), we get dx 0 x ( t  )  x ( t ) 0 x ( t  )  x ( t )


lim f ' (  h )3 · 3(   h ) 2 = π 
The one-sided derivatives of f at x = x(t) are given by

h0

⇒ f ′ (π ) · 3 π = π 3 2
y ( t  )  y ( t )
π 1
f ′ (π3) = = . 
3π 2 3π L.H.D. = f ′(x ) = lim


  0 x ( t  )  x ( t )

Example 5: Given f′(2) = 6 and f ′(1) = 4, evaluate
y ( t  )  y ( t )




f ( 2 h  2  h 2 )  f ( 2) = lim ...(1)
lim .  0 x ( t  )  x ( t )


h 0 f (h  h 2  1)  f (1)
y ( t  )  y ( t )
f ( 2 h  2  h 2 )  f ( 2)
Solution: lim 
f (h  h 2  1)  f (1) R.H.D. = f ′(x+) = lim



h 0
 0 x ( t  )  x ( t )

f ( 2  2 h  h 2 )  f ( 2) 
= lim
2h  h 2 y ( t  )  y ( t )

h 0
= lim ...(2)
0 x ( t  )  x ( t )


h (2  h ) (1  h  h )  1 2
 
h (1  h ) f (1  h  h 2 )  f (1) provided x = x(t) increases with increase in t.


FREE BOOKS FOR JEE & NEET =>(@iitjeeadv)
3.28  
Differential Calculus for JEE Main and Advanced

If x = x(t) decreases with increase in t, then (1) evaluates the Example 9: The function f is defined by y = f(x) where
R.H.D. and (2) evaluates the L.H.D.



x = 2t – |t|, y = t2 + t |t|, t ∈ R. Discuss the continuity and
As before, f ′(x) exists when both (1) and (2) exist and are equal. differentiability at x = 0.
Another way is to eliminate t to find the function y in terms of
x and then proceed as usual. Solution: Here, we eliminate t and get the function



y = f(x) first.
Example 7: The function f is defined by y = f(x) where If t ≥ 0 we have x = t, y = 2t2,


x = 2t – |t – 1|, y = 2t2 – t |t|, t ∈ R. Discuss the differentiability hence y = 2x2, x ≥ 0
of y = f(x) at x = 2. and for t < 0, x = 3t, y = 0, x < 0
Solution: We solve the equation x = 2 to get the ∴ The function is defined as


corresponding value of t.
2 x 2 , if 0 ≤ x ≤ 1
2t – |t – 1| = 2 ⇒ t = 1. f(x) = 
 0 , if − 1 ≤ x < 0



We notice that at t = 1, x = x(t) increases with increase in t.
Hence, f ′(0–) = 0


y(1  )  y(1) 2h 2  0
L.H.D. = f ′(2–) = lim
0 x (1  )  x (1) f ′(0+) = lim  0.


h 0 h
It is clear that f(x) is differentiable and hence continuous at
2(1  ) 2  (1  ) | 1   | 1
= lim x = 0.
0 2(1  ) | 1    1 | 2

Differentiability Using Derivative of Functions
2  2 2
Consider the function f(x) = x4/3ex. Let us find f ′(0) from the
= lim = .
ps
0 3

3
formula of its derivative. We differentiate f(x) using product
el
y(1  )  y(1) rule (to be discussed later).
R.H.D. = f ′(2+) = lim
eh

0 x (1  )  x (1)

4 1/3 x
f ′(x) = x4/3ex + x e ...(1)
je

2(1  )  (1  ) | 1   | 1
2 3


= lim
iit

0 2(1  ) | 1    1 | 2 To find f ′(0), even when we donot know whether it exists ,

we put x = 0 into the formula (1). We get f ′(0) = 0. This
@

2   2 provides the result in a simple way


= lim = 2.
 Now, if f(x) = x1/3 sin x ,

0

∴ f ′ (2–) ≠ f ′ (2+) and hence f ′ (2) does not exist. 1
f ′(x) = x1/3 cos x + sin x · x –2/3
Example 8: The function f is defined by y = f(x) where

3


x = 1 – t3, y = [t]t3, t ∈ R. Discuss the differentiability of y = sin x
f(x) at x = 1. = x1/3 cos x + ...(2)
3x 2 3

Solution: The corresponding value of t is, Here, f ′(x) is not defined at x = 0. Do we conclude that f (x)



1 – t3 = 1 ⇒ t = 0 is not differentiable at x = 0 ? The answer is no.

We notice that at t = 0, x = 1 – t3 decreases with increase in t. In such cases, we should try to find the derivative using its basic
Hence, definition (i.e. first principles). The derivative may exist even
y(0  )  y(0) when the formula f ′(x) is undefined.
R.H.D. = f ′(1+) = lim
 0 x ( 0   )  x ( 0) f (0 + h ) − f (0)
Now, f ′(0) = = lim
h →0 h
[]()3  0
= lim 1
0 1  (  ) 3  1 h1/ 3 sinh − 0

= lim = 0.

h →0 h
y ( 0   )  y ( 0)
L.H.D. = f ′(1–) = lim ∴ f ′(0) = 0. The function f is differentiable at x = 0.
0 x ( 0   )  x ( 0)

There is another point to be noted. We find the limit of
[]3  0 f ′(x) given by (2) as x → 0,
= lim  0.
  0 1  3
1

sin x
lim f ′(x) = lim ( x1/3 cos x + ) = 0.
Hence, f is non-differentiable at x = 1. 3x 2 3

x→0 x→0

FREE BOOKS FOR JEE & NEET =>(@iitjeeadv)


Differentiability 3.29

lim Thus, at x = 0, L.H.D. = R.H.D. = 0,


We are now tempted to take the limit of the derivative ( x→0


so we have f ′ (0) = 0.
f ′(x)) as the value of the derivative (f ′(0)). Both are equal to
Now, we extend the definition of f ′ (x) at x = 0.
0 in this case.
 2 x x0
Example 10: Suppose that the real valued function f is Hence, f ′(x) = 
 2x x0


continuous on R and differentiable on R–{0}, and that
lim f ′(x) = L. ⇒ f ′ (x) = 2 x.

x→0
Prove that f is differentiable at 0 and that f ′(0) = L. Note:



Solution: We are required to prove that If we consider the function


f ( x ) − f ( 0)   x 2 x0
lim exists and equals L.
x−0 g(x) =  2

x →0
 x  1 x0


Let F(x) = f(x) – f(0) and G(x) = x.
and proceed as above, we get
Then, since f is continuous, lim F(x) = 0, and of course
x→0  2 x x0
lim G(x) = 0 too. g ′ (x) = 
 2x x0



x→0
However, F′(x) = f ′(x) and G ′(x) = 1,
Here, also we get lim g ′(x) = 0 and lim g ′(x) = 0.
x0 x0
so lim F '( x ) exists and equals L.
Since the function is discontinuous at x = 0, there is no question

x→0 G '( x )
ps
of differentiability. However, if the above limits are considered
By L′ Hospital′s rule, lim F '( x ) = L too, as the L.H.D. and R.H.D. then it would be misleading.
el


x→0 G '( x )
In fact when we calculate L.H.D. and R.H.D. using first
eh

f ( x ) − f ( 0) principles, we get L.H.D. = ∞ and R.H.D. = 0.


i.e. lim = L, as required.
je

x−0 Hence, we should check the continuity of the function in



x →0
advance.
iit

In general, the limit of the derivative may not exist, but when it

Using the following example, we now show that the derivative
exists then it is equal to the value of the derivative.
@

of a continuous function is not always a continuous function.


In such a case, we say that the derived function f ′(x) is
continuous, or the function f(x) is continuously differentiable. Example 12: If f(x) = x2 sin (1/x) when x ≠ 0 and


In most of the differentiable functions used in routine, the limit f(0) = 0, show that f is differentiable for every value of x but
of the derivative exists and this gives us an opportunity to find the derivative is not continuous for x = 0.
the value of the derivative using the limit of the derivative. Solution: For x ≠ 0,


But, it must be checked before differentiating the function that 1  1  1 
f ′(x) = 2x sin + x2 cos    − 2 
it is continuous. x  x  x 

Example 11: If f(x)=xx, then find its derivative. = 2x sin
1 1
– cos .


x x
Solution: f (x) = x x At x = 0, we have



 x 2 x  0 x 2 sin
1
f (x) =  2 ...(1) f ′(0) = lim f ( x ) − f (0) = lim x
 x x0

x−0


x→0 x→0
x
We see that f is continuous. 1
= lim x sin
= 0.
 2 x x  0

x
...(2)
x→0
f ′ (x) = 
 2x x0 Thus the function possesses a derivative for every value of x



given by
Note that the equality sign at x = 0 has been removed because 1 1
f ′(x) = 2x sin – cos when x ≠ 0, f ′(0) = 0.
the existence of f ′ (0) is in doubt.

x x

We now show that f ′ is not continuous for x = 0.
We can see that lim f '( x )  lim (2 x )  0
x 0 x 0
 1 1
and lim f '( x )  lim 2 x  0 . Here lim f ′(x), i.e., lim  2 x sin − cos  does not exist
x 0 
x 0 
x→0 x x

x→0

FREE BOOKS FOR JEE & NEET =>(@iitjeeadv)


3.30  
Differential Calculus for JEE Main and Advanced

Similarly, f ′(0+) = – 1
since lim  2 x sin 1   0 exists but lim  cos  does not
1
x→0   Hence, f ′(0) = – 1.
 x x→0  x
exist. (b) If lim f ′(x) ≠ lim f ′(x), then f(x) is non-differentiable
x a  x a 


Since, lim f ′(x) does not exist, f′ is discontinuous at x = 0. and its derivative f ′(x) is discontinuous at x = a.

x→0
In other words, f(x) is differentiable but not continuously  2x 
Example 14: Let f(x) = cos–1   . Define f ′(x)
differentiable.  1 x2 



If the limit of the derivative does not exist, does it mean that stating clearly the points where f(x) is not differentiable.


the function f(x) is not differentiable ? The answer is no. This  2x 
is proved by the above example.
Solution: f(x) = cos–1  , x ∈ R
 1 x2 



So, the question arises that in what circumstances can we use
1 2(1  x 2 )  4 x 2
limit of the derivative in deciding the differentiability of the f ′(x) = – ·
function ? (1  x 2 ) 2


2
 2x 
1  
Assuming that the function f(x) is continuous at x = a, two
 1 x2 
cases arise:
2(1  x 2 )
(i) If lim f ′(x) and lim f ′(x) both exist or are infinite then f ′(x) = –
x a  x a  | 1  x 2 | (1  x 2 )


lim f ′(x) = f ′(a–)
x a  lim f ′(x) = – 1 and lim f ′(x) = 1

x1 x1
i.e L.H.L. of f ′(x) = L.H.D. of f(x) at x = a
ps

and lim f ′(x) = f ′(a+) Since lim f ′(x) and lim f ′(x) are unequal, f ′(1) does not exist.
x a 


x1 x1
el
i.e R.H.L. of f ′(x) = R.H.D. of f(x) at x = a. Similarly f ′(–1) does not exist.
eh

(a) If lim f ′(x) = lim f ′(x), then f(x) is differentiable and  2
x a  x a 

− 1 + x 2 if − 1 < x < 1
je

its derivative f ′(x) is continuous at x = a. 


iit

Example 13: Hence, f ′(x) = non existent if x = ± 1


 2
@


 x tan 1 x  sec 1 1 x  , x (1,1)  {0}  if x > 1 or x < −1
If f(x) =  , then find 1 + x 2
 2 if x  0
(c) If lim f ′(x) and lim f ′(x) are infinite, then f(x) has an
f ′(0), if it exists.

x a  x a 
infinite derivative at x = a and hence it is non-differentiable
Solution: there and f ′(x) is discontinuous at x = a.


1
x  1  | x |x , x ≠ 0
f ′(x) = + tan x +–1 1 1  2  Example 15: Suppose f(x) =  , then find
1+ x2 − 1  x  1, x=0



2
x x
whether f(x) is differentiable at x = 0.
x | x |2  1 
= + tan–1x +  2  Solution:
1+ x 2
 x 



2
1− x For x > 0, f (x) = xx
lim f′(x) = – 1 and lim f ′(x) = – 1 and f ′(x) = xx(1 + ln x) ...(1)


For x < 0, f (x) = (–x)x = ex ln (–x)

x0 x0

Hence f ′(0) = – 1 and f ′(x) = (–x)x[1 + ln (–x)] ...(2)



It can be shown using L'Hospital's rule that
Alternative:
Using first principles: lim x x = 1 and =1
x →0

f ′(0+) = lim
1
h tan (h )  sec 1
1 h    2 Now, lim f ′(x) = – ∞ and f ′(x) = – ∞
x0

h0 h
lim f '( x ) = – ∞
cos 1 (h )   2  sin 1 (h )

= lim = lim =–1 x→0
Hence f is not differentiable at x = 0.

h0 h h0 h

FREE BOOKS FOR JEE & NEET =>(@iitjeeadv)


Differentiability 3.31

(ii) If any of the limits xlim f ′(x) or lim f ′(x) does not 1 −1
a x a f ′(x) = . –2x
 


.
exist, then we cannot conclude anything about the 2 1− 1− x2 2 1− x2


differentiability of the function. In such a case, we should x
try to find the derivative using its basic definition (i.e. first = 
2 1 x 1 1 x2


principles).
In the previous example, we have seen this situation : The formula of f ′(x) is undefined at x = – 1, 0, 1.


f(x) = x2 sin (1/x) when x ≠ 0 and f(0) = 0, The differentiability of the function should be checked at

x = – 1, 0, 1.
1 1
For x ≠ 0, f ′(x) = 2x sin – cos . We can see that
x x
 1 1 lim f '( x )   and lim f '( x )   .
Here lim f ′(x) = lim  2 x sin − cos  does not exist. x 1 x 1 


x→0 x→0 x x
Hence, f has an infinite derivative at x = – 1 and 1.
However, using first principles, we have


To check the existence of f ′(x) at x = 0, we find whether
1
h 2 sin lim f '( x ) exists or not.
f ′(0) = lim f (h ) − f (0) = lim h x →0

h→0 h h→0
h
x x
1 lim  lim
= lim h sin = 0. We have x 0 x 0
1 x 2
1 1 x 2
1 1 x2

h→0 h
Hence, the function f is differentiable at x = 0, even if the limit ps
= lim x 1  1  x
2
of its derivative does not exist.

Example 16: Calculate the derivative of the function x 0 x2
el


eh

2 x  2 as x  0

f(x) = x 2  2 x  1 on the interval [0, 2].
 

lim
= x 0 | x | 
je

Solution: Since the expression under the radical sign is  2 as x  0




a perfect square, we can, according to the definition of the
iit

= does not exist.



modulus, represent the given function in the following form : Hence, f is non-differentiable at x = 0.
@

1  x , 0  x  1, Thus, the function f is differentiable in its domain except at


f(x) = |x – 1| =  x  1, 1x 2 x = –1, 0, 1.

Alternative approach at x = 0 using first principles.
Differentiating f(x) separately on the intervals [0, 1) and
(1, 2], we obtain  f ( 0 − h ) − f ( 0) 
f ′(0–) = lim  
1, 0  x  1, h →0  −h 

f ′(x) = 
1, 1x 2

 
 1 − (1 − h ) − 0 
2
lim f ′(x) = – 1 and lim f ′(x) = 1
= h →0 
lim 

 
x1 x1
 −h 

Since the left and right derivatives of f (x) at the point x = 1  
do not coincide, the derivative does not exist at x = 1. We take
the values of the left hand derivative of function at the point 2 1 − (1 − h 2 ) 1 + (1 − h 2 )
and of its right hand derivative at the point 0 as the values of = lim ×
h →0 −h 1+ 1 − h2

f ′(x) at the endpoints of the interval [0, 2].
 1, x  [0,1), h
Thus f ′(x) =  lim 1
1, x  (1, 2] = h →0
( − h ) 1 + (1 − h 2 ) = –

2
Example 17: Check the differentiability of the function f (0 + h ) − f (0)


f(x) = 1 − 1 − x 2 . and f ′(0+) = lim
h →0

h
Solution: The function f(x) = 1 − 1 − x 2 is defined
1 − (1 − h 2 ) − 0




for x ∈ [–1, 1]. It is continuous in its domain. = lim

Differentiating w.r.t. x, we have

h →0 h

FREE BOOKS FOR JEE & NEET =>(@iitjeeadv)


3.32  
Differential Calculus for JEE Main and Advanced

Solution: Since 1 ≤ x < 2 we have0 ≤ x – 1 < 1,


1 − (1 − h 2 ) 1 + (1 − h 2 )



× and [x2 – 2x] = [(x – 1)2 – 1] = [(x – 1)2] – 1
= hlim
→0 h
1+ 1 − h2 = 0 – 1 = –1


 1
1  4 x , 0  x  2
2
h 1
lim =
= h →0 2 
h 1 + (1 − h 2 ) ∴ f(x) =  2 1

4x  1 , 2  x  1



f ′(0 ) ≠ f ′(0 ), f(x) is not differentiable at x = 0.
– +



 1 , 1 x  2
A  Bx 2 , x 1
Example 18: If f(x) =  find the Hence, the graph of f(x) is:
   x 1


3Ax B 2 ,


value of A and B so that f(x) is differentiable at x = 1.
Solution: Since f(x) will be differentiable at x = 1, it


must be continuous,
∴ 3A – B + 2 = A + B


∴ A – B + 1 = 0 ...(1)

 2Bx , x  1
Now, f ′(x) = 
3A , x  1
It is clear from the graph that f(x) is discontinuous at
lim f ′(x) = 2B and lim f ′(x) = 3A
ps 1
x = 1 and not differentiable at x = and x = 1. To verify these

 
x1 x1
2
el
∴ 3A = 2B ...(2) details we calculate the derivative of f:

eh

2B
From (1) and (2), –B+1=0 8x , 0  x  1 / 2
3 
je

B f ′(x) =  8x , 1 / 2  x  1
⇒ – =–1 ⇒ B=3

 0,
iit

1 x  2


3 
⇒ A = 2.
@


| 1  4x | , 0  x  1
2
This shows that f(x) is not differentiable
Example 19: If f(x) =  2
[ x  2 x ] , 1  x  2 at x = 1/2 (as RHD = 4 and LHD = –4)



and x = 1 (as RHD = 0 and LHD = 8)
where [.] denotes the greatest integer function.

Discuss the continuity and differentiability of f(x) in [0, 2). Therefore, f(x) is differentiable x ∈ [0, 2) – {1/2, 1}.

Concept Problems D

sin(3 + x ) 2 − sin 9 6. If f ′(2) = 4 then find the value of
1. Evaluate lim

.
x →0 x f (2 + h ) − f (2 + sin h)
2. Find a function f and a number a such that lim .
h. sin h .tan h

h→0
(2 + h )6 − 64
lim = f '(a ) .
h →0 Discuss the differentiability of the function y = f(x)

h 7.
3. Each of the following limit represents the derivative of

defined as x = 2 t + | t | and y = [ t] t2 at x = 0.

some function f at some number a. State such an f and a
in each case.  3x 2 , x  1
2 x  32 cos (  h )  1 8. Let f(x) =  . Find the values of a and b so
ax  b, x  1

(i) lim (ii) lim
h 0 x  5 h 0




h
 x 2 that f will be differentiable at x = 1.
x0
4. Let f(x) =  2 .  x 2 ,
 x x0 x0

9. Let f(x) =  2 . Show that
Prove that f is differentiable. Is f ′ is continuous?  x  1, x  0



1 f (a  2 h 2 )  f (a  2 h 2 )
5. If f ′(a) = , find lim . lim f ′(x) = lim f ′(x) but that f ′(0) does not exist.
4 h2


h0 x0 x0

FREE BOOKS FOR JEE & NEET =>(@iitjeeadv)


Differentiability 3.33

3.8 Derivatives of Higher order Solution: Let us find f ′(x) first. We differentiate f(x)



using product rule.

Because the derivative of a function is a function, differentiation 4 1/3
can be applied over and over, as long as the derivative itself is f ′(x) = x4/3cos x + x sin x ...(1)



3
a differentiable function.
f '(0 + h ) − f '(0)
Let a function y = f(x) be defined on an open interval (a, b) . Now, f ′′(0) = lim



h →0 h
Its derivative, if it exists on (a, b) is a certain function f ′(x) [or


(dy/dx) or y ] and is called the first derivative of y w. r. t. x. 4
h 4 / 3 cos h + h1/ 3 sin h − 0









If it happens that the first derivative has a derivative on (a, b) = lim 3


then this derivative is called the second derivative of y w. r. t. x h →0 h




and is denoted by f ′′(x) or (d2y/dx2) or y′′.  1/ 3 4h1/ 3 sin h 


Once we have found the derivative f ′ of any function f, we can = hlim  h cos h   = 0.
0 3h
 


go on and find the derivative of f ′.
∴ f ′′(0) = 0.
f (a  h )  f (a )


f ′′(a) = lim . Hence, the function f is twice differentiable at x = 0.
h 0 h

Example 2: Find whether
Similarly, the third order derivative of y w. r. t. x, if it exists, is





 x 2  x
 x0
d  d2 y
d3 y  f (x) = 
defined by    . It is also denoted by f ′′′(x) or y ′′′.
d x 3 dx  d x 2 sin x  x x0
2



is twice differentiable and find its second derivative.
The third derivative, written f ′′′, is the derivative of the second
ps
derivative, and, in principle, we can go on forever and form 
 x x x0
2
el
derivatives of higher order. We adopt the alternative notation Solution: f (x) =  ..(1)
eh

sin x  x x0
2



f(n) for the nth derivative of f. Notice also that for derivatives 
higher than the third, the parentheses distinguish a derivative We see that f is continuous.
je

from a power. For example, f 4 ≠ f (4).  2x  1 x0


iit

f ′(x) =  ...(2)
Twice Differentiability cos x  2 x x0


@

Suppose that a function y = f(x) defined in an interval has the Note that the equality sign at x = 0 has been removed because
derivative f ′(x) with respect to the independent variable x. If the the existence of f ′ (0) is in doubt.
function f ′(x) is further differentiable, its derivative is called
the second derivative (or the derivative of the second order) We can see that lim f '( x )  lim f '( x )  1 .
x 0 x 0
of the original function f(x) and is denoted f ′′(x).
So we have f ′(0) = 1.
A function f(x) is twice differentiable at x = a if its derivative
Now, we extend the definition of f ′(x) at x = 0.
f ′(x) is differentiable at x = a i.e.
 2x  1 x0

f (a  h )  f (a ) Hence, f ′(x) = 
the limit f ′′(a) = lim exists. cos x  2 x x0

h 0 h
f '( x ) − f '(a )  2 x0
Alternatively, f ′′(a) = lim . Now f ′′(x) = 
x →a x−a  sin x  2 x0
The value f ′′(x) of the second derivative at a point x
characterizes the rate of change of f ′(x) at that point, that is We find that lim f ''( x )  lim f ''( x )  2 .
x 0 x 0
the rate of change of the rate of change of f(x). By analogy
with mechanics, we can say that f "(x) is the acceleration of the ∴ f ′′(0) = 2 and the function f is twice differentiable at
x = 0. We can easily see that f is twice differentiable at

change of the function f(x) at the given point x.
The geometrical meaning of the second derivative will be other values of x.
discussed later. Example 3: Let f be defined in a neighbourhood of
Further, the nth derivative of f(x) at x = a exists if


x and f ′′(x) exists.
f ( n −1) ( x ) − f ( n −1) (a ) f ( x  h )  f ( x  h )  2f ( x )
f (n)(a) = [f(n–1)]′ (a) = lim exists. Show that lim  f "( x )
x →a x−a h 0 h2
Example 1: Find whether the function f(x) = x4/3 sinx is Show by an example that the limit may exist ever if f ′′(x)


twice differentiable at x = 0. does not exist.

FREE BOOKS FOR JEE & NEET =>(@iitjeeadv)


3.34  
Differential Calculus for JEE Main and Advanced

f '( x + h ) − f '( x )  x 3 if x  1
Solution: f ′′(x) = lim Example 5: Let f(x) =  .


h →0 h



ax  bx  c if x  1
2

 f (x + h + k) − f (x + h) f (x + k) − f (x)  If f ′′ (1) exist then find the value of a2 + b2 + c2.


lim  − 
 


lim
= h→0 k →0 k k
Solution: For continuity at x = 1 we must have

h



Let k = – h f (1+) = f (1–)


⇒ f ′′(x) = – lim f ( x ) − f ( x + h ) − 2( x − h ) + f ( x ) ⇒ a+b+c=1 ...(1)




h→0 2
h
3x 2 if x  1
f ′(x) = 
lim f ( x + h ) + f ( x − h ) − 2f ( x ) 2ax  b if x  1


= h→0
h2 For continuity of f ′(x) at x = 1



f ′(1+) = f ′ (1–) 2a + b


Note: The above limit cannot be used as the test for twice ⇒ 2a + b = 3 ...(2)




differentiability of f. One can verify this by using f(x) = x| x |. 6 x if x  1

Let us calculate the limit first: f ′′(x) = 
2a if x  1

lim f (0 + h ) + f (0 − h ) − 2f (0) f ′′(1–) = 6, f ′′(1+) = 2a

h→0
h2

Hence, 2a = 6 ⇒ a=3


lim h | h | + ( − h ) | − h | − 0 = lim 0 = 0. From (1) and (2), b = – 3, c = 1
= h→0

h2 h→0
h2 Hence, the value of a2 + b2 + c2 = 19.

ps

In the previous section we had calculated  2 x 2 sin  x
 x 1
el
Example 6: Let f (x) =  3
 2 x x  0 x  a x  b x  1

2


eh

f ′(x) = 
 2x x0 be a differentiable function . Examine whether it is twice

je

from where f ′′(0 –) = – 2 and f ′′(0 +) = 2. differentiable in R .


iit

Hence f ′′(0) does not exist. However, the given limit exists. Solution: For continuity at x = 1 we must have


a+b+1=0
@

 xe x
x≤0
Example 4: If f(x)=  then examine 
 4 x sin  x  2x cos  x
2
x 1
 x + x − x x>0


2 3
f ′(x) = 
3x  2a x
2
x 1



whether f ′(x) is continuous and differentiable at x = 0.
For differentiability of f :

 xe x x≤0

 f ′(1+) = 3 + 2a, f ′(1–) = –2 π should be equal.
Solution: f(x) = 

x + x − x x>0
2 3



2  3 2  3
f(x) is continuous at x = 0. ⇒ a = − and b =
2 2



 xe x + e x , x<0
f ′(x) =  
 4 x sin  x  2  x cos  x , x  1
2

1 + 2 x − 3x , x > 0
2 Thus f ′(x)=  and

3 x  (3  2 ) x

2 ,x  1

 (4  2 2 x 2 ) sin  x  8  x cos  x
lim f '( x ) = 1 = lim f '( x ) , x 1
x0 x0 

f ′′ (x) = 

Hence f(x) is differentiable at x = 0 and thus 6 x  3  2  , x 1
 xe x + e x , x≤0 f ′′(1 −) = − 8 π and f ′′(1 +) = 3 − 2 π
f ′(x) = 

is continuous. ⇒ f ′′(x) does not exist at x = 1.
1 + 2 x − 3x , x > 0
2


However, f (x) is twice differentiable for x other than x = 1


e x + ( x + 1)e x , x < 0
Now, f ′′(x) =  Example 7: Find
2 − 6 x , x>0



 x 3 sin(1 / x ) for x  0,
lim f ''( x )  lim f ''( x ) = 2 f ′′(x) if f ( x )  
 for x  0

x 0 
x 0 
0

⇒ f ′(x) is also differentiable at x = 0. and find whether f ′′(x) is continuous at the point x = 0.

FREE BOOKS FOR JEE & NEET =>(@iitjeeadv)
Differentiability 3.35

Solution: For x ≠ 0 the derivative f ′(x) can be found by Given that g(0) = g ′(0) = 0 and that g"(0) = 6, determine f(0)
and f ′(0).


differentiating the function x3 sin (1/x) according to the rule of
differentiation of a product. This yields Solution: By the definition of g ′(0), we have
f ′(x) = 3x2 sin (1/x) – x cos (1/x) (x ≠ 0).



g ( x ) − g ( 0)
0 = g ′(0) = lim

lim f ′(x) = 0 = lim f ′(x) x →0 x−0

x0 x0
This means that f ′(0) exists g( x )
Thus f ′(x) exists at all points: = lim (since g(0) = g ′(0) = 0)


x  x
3x 2 sin (1 / x ) − x cos (1 / x ) for x ≠ 0, g(x )
f ′(x) =  = lim f(x) (since f(x) = for x ≠ 0),
 0 for x = 0.


x→0 x
Now, f ′′(x) = 6x sin (1/x) – 4 cos (1/x) = f(0) (since f is continuous)



– (1/x) sin (1/x) (x ≠ 0). So f(0) = 0. To find f ′(0), note that

lim f ′′(x) does not exist.
f ( x ) − f (0) lim g ( x )

x→0
To test whether f′′(x) exists at x = 0, we apply first principles f ′(0) = lim =
x−0 x  x 2


x →0
on f ′(x)
g(x )

f ′′(x) = lim f '(h ) − f '(0) (since f(0) = 0 and f(x) = for x ≠ 0).

x

h→0 h
1 1 To determine this latter limit, we can use L′Hospital ′s rule.
3h 2 sin − h cos − 0 Certainly lim g(x) = 0 and lim x2 = 0.
lim
= h→0 h h
ps x→0 x→0

h
g '( x )
So we look at (differentiating top and bottom).
el
lim  3h sin 1  cos 1 
= h→0 2x
 
eh

h h

g '( x )
which does not exist. But g′(0) = 0, so lim g '( x ) = g"(0) = 6, and so lim = 3.
je

x→0 x x→0 2 x
Thus f is twice differentiable for all x except x = 0.
iit

Further, f′′(x) is discontinuous at the point x = 0. Thus by L ′Hospital ′s rule,



@

Example 8: Let f, g be twice differentiable functions lim g ( x ) = 3 too, and this gives f ′(0) = 3.



from R to R, and suppose that f(x) = g(x)/x for all x ≠ 0. x2
x→0

Concept Problems E

3. Let f(x) = x4/3
 x 2 , x  0

1. Let f(x) =  3 Show that f ′(0) exists but f ′′(0) (a) Compute f ′.
 x , x  0 (b) Is f a differentiable function?

does not. (c) Show that f"(0) does not exist, and compute
f ′′(x) for x ≠ 0.
2. Let f be the function defined by
4. Let the function g be defined by
 x2
 if x ≥ 0,  x 2 x ≤ 0,
 2 g(x) = 

f(x) =
− x
2
 2 x − 1 x > 1.

if x ≤ 0.
 2 (a) Compute g ′ and g".
(a) Compute f ′. (b) Are g and g ′ differentiable functions ?
(b) Is f a differentiable function?
(c) Show that f ′′(0) does not exist, and compute sin(a  2 x )  2 sin(a  x )  sin a
5. Evaluate lim .
f ′′(x) for x ≠ 0. x 0 x2

FREE BOOKS FOR JEE & NEET =>(@iitjeeadv)


3.36  
Differential Calculus for JEE Main and Advanced

Practice Problems D


6. Find y" for the following functions: tan(a  2 x )  2 tan(a  x )  tan a
(a) y = |x3| 8. Evaluate lim .
x 0 x2
 x sin(1 / x ), x  0,
2
9. (a) Show that f(x) = x7/3 is twice differentiable at 0, but
(b) y = 
 0, x0 not three times differentiable at 0.
Also find y" (0) if it exists. (b) Find an exponent k such that f(x) = x k is
(n – 1) times differentiable at 0, but not n times
7. A function f is defined as
differentiable at 0.
 1
 (b 2 − a 2 ) for 0 ≤ x ≤ a
 2
 1 x2 a2
f (x) =  b2 − − for a < x ≤ b
2 6 3

 1 b −a 
3 3
   for x > b
 2  x 
Prove that f and f ′ are continuous but f ′′ is discontinuous. ps
el
3.9 Algebra of Differentiable Functions Note: The sum of a finite number of functions differentiable
eh



at a point is a differentiable function at the point.
We can deduce from the theorems on limits that the sum,
Theorem If f and g are differentiable at x = a, then so is
je

product, difference or quotient of two functions which are



their product f. g.
differentiable at a certain point are themselves differentiable
iit

Proof Let F(x) = f(x). g(x)


at that point (except that, in the case of the quotient, the

@

denominator must not vanish at the point in question). Further f (a + h ) g (a + h ) − f (a )g (a )


F′(a) = lim
it is true that composition of a differentiable function with a h →0 h
differentiable function is a differentiable function. f (a + h ) g (a + h ) − f (a + h )g (a ) + f (a + h )g (a ) − f (a )g (a )

1. If f(x) and g(x) are differentiable at x = a, then the = lim
h →0 h

following functions are also differentiable at x = a.
 g (a + h ) − g (a ) f (a + h ) − f (a ) 
(i) cf(x) is differentiable at x = a, where c is any constant. = lim f (a + h ) + g (a ) 
h →0  h h 
(ii) f(x) ± g(x) is differentiable at x = a.
+ −


(iii) f(x). g(x) is differentiable at x = a. = lim f (a  h ) . lim g ( a h ) g ( a )
(iv) f(x)/g(x) is differentiable at x = a, provided g(a) ≠ 0. h 0 h →0 h
Here, we prove some of these results: f (a  h )  f (a )
+ g(a) . lim
Theorem If f and g are differentiable at x = a, then so is h 0 h

their product f + g. As h approaches zero, f(a+h) approaches f(a) because f, being
Proof We apply the definition of derivative to F(x) = differentiable at a, is continuous at a.

f(x) + g(x) as follows: = f(a) g ′(a) + f ′(a) g(a).
(f (a + h ) + g (a + h )) − (f (a ) + g (a )) Therefore, f(x). g(x) is differentiable at x = a.
F′(a) = lim
h →0 h Note: The product of a finite number of functions


 f (a + h ) − f (a ) g (a + h ) − g (a )  differentiable at a point is a differentiable function at that point.
= lim  +

h →0   By the above theorems we recognize that the functions


h h
y = x2 + 3x + 2
f (a + h ) − f (a ) g (a + h ) − g (a )

= lim + lim y = sin3x – x sin x – (x4 – 1) cos x, and


h →0 h h →0 h y = (sin x – 2x)/(x2 + 1)
= f ′(a) + g′ (a). are differentiable at every point in a domain common to all

Therefore, f(x) + g(x) is differentiable at x = a. functions involved.

FREE BOOKS FOR JEE & NEET =>(@iitjeeadv)


Differentiability 3.37

2. If f(x) is differentiable at x = a and g(x) is non-differentiable differentiable function is non-differentiable. Hence f(x) + g(x)

at x = a, then we have the following results: is non-differentiable at all integral points and x = 1/2.
(i) Both the functions f(x) + g(x) and f(x) – g(x) are 3. If f(x) and g(x) both are non-differentiable at x = a, then




non-differentiable at x = a. we have the following results.
For example, consider, f(x) = x and g(x) = | x |. (i) The functions f(x) + g(x) and f(x) – g(x) are not





Here f(x) is differentiable at x = 0 and g(x) is non- necessarily non-differentiable at x = a. However,
differentiable at x = 0. Both the sum function x + | x | and atmost one of f(x) + g(x) or f(x) – g(x) can be
differentiable at x = a. That is, both of them cannot


the difference function x – | x | are non-differentiable
be differentiable simultaneously at x = a.

at x = 0.
(ii) f(x). g(x) is not necessarily non-differentiable at x = a. Let us assume that both f(x) + g(x) and f(x) – g(x) are


differentiable. Then the sum of functions (f(x) + g(x)) +
We need to find the result by first principles.
(f(x) – g(x)) = 2f(x) must be differentiable at x = a, which
For example, consider, f(x) = x3 and g(x) = sgn(x).


is wrong as it is given that f(x) is non-differentiable at
Here f(x) is differentiable at x = 0 and g(x) is non-
x = a. Hence our assumption is wrong. So, both the func
differentiable at x = 0. But the product function

­
tions cannot be differentiable simultaneously at x = a.

x3 , For example, consider, f(x) = [x] and g(x) = {x}.
x0
 Here both f(x) and g(x) are non-differentiable at x = 0.
f(x)g(x) = 0, x0 The sum function [x] + {x} being equal to x is
 3


 x , x0 differentiable at x = 0. The difference function [x] – {x}


however is non-differentiable at x = 0.
But this does not mean that one of the functions
is differentiable at x = 0.
ps f(x) + g(x) or f(x) – g(x) must be differentiable. We
As another example, the product of the functions


can have both the functions non-differentiable. For
el
 1 example, if f(x) = 2[x] and g(x) = {x}, then both
eh

x0

f(x) = x and g ( x )   the functions f(x) + g(x) or f(x) – g(x) are non-
x sin
x


differentiable simultaneously at x = 0 .

 0
je

x0
(ii) f(x). g(x) is not necessarily non-differentiable at x = a.
iit

We need to find the result by applying first principles


is differentiable at x = 0 even when f(x) is on the product f(x). g(x).
@

differentiable at x = 0 and g(x) is non-differentiable For example, consider, f(x) = [x] and g(x) = [–x].
at x = 0. Here both f(x) and g(x) are non-differentiable at x = 0
However, the product of the functions f(x) = x and but, the product function [x].[–x] is differentiable at x = 0.
g(x) = [x] is non-differentiable at x = 1.

Further, f(x) = [x] and g(x) = {x} are both non-
(iii) f(x)/g(x) is not necessarily non-differentiable at x = a. differentiable at x = 0 and, the product function [x].
Here also we need to work on the function f(x)/g(x) {x} is non-differentiable at x = 0.
to get the result. Hence, we cannot comment in advance when both the
 x  1, x  0 functions are non-differentiable at x = a.
Let f(x) = x2(x2 –1) and g(x) =  .
(iii) f(x)/g(x) is not necessarily non-differentiable at x = a.
 x  1, x  0

Here also we need to work on the function f(x)/g(x)
Here f(x) is differentiable at x = 0 and g(x) is non- to get the result.
differentiable at x = 0.
 x 2  1, x  0
 x 2 ( x  1), x  0 Let f(x) =  and
 x  1, x  0


f(x)/g(x) =  2
 x ( x  1), x  0  x  1, x  0

g(x) =  .
We can check that f(x)/g(x) is differentiable at x = 0.  x  1, x  0
Here both f(x) and g(x) are non-differentiable at x = 0.
Example 1: Discuss the differentiability of
But we find that f(x)/g(x) is differentiable at x = 0.


f(x) = [x] + | 2x – 1 |.
Solution: Let us study the functions y = [x] and Differentiability of Composite Functions


y = | 2x – 1 |. It is a clear that f(x) = [x] is non-differentiable Theorem If f(x) is differentiable at x = a and g(x) is

at all integral points and g(x) = |2x – 1| is differentiable for differentiable at x = f(a) then the composite function (gof)(x)
all x ∈ R – {1/2}. The sum of a non-differentiable and a is differentiable at x = a.

FREE BOOKS FOR JEE & NEET =>(@iitjeeadv)


3.38  
Differential Calculus for JEE Main and Advanced

Note: A function of a function composed of a finite number


6h  h 2


of differentiable functions is a differentiable function.  lim  


π h 0 h
For example, f(x) = sin x is differentiable at x = and
2
 x  1, x  1
 2 π 9 − ( −3 + h ) 2 − 0
g(x) =  is differentiable at x = f( ) = 1. Hence f '( −3+ ) = lim


2 x  2, x  1
 2 h →0 h
π
the composite function (gof)(x) is differentiable at x = .
2 6h  h 2
 lim 


Note: h 0 h


1. Let a function f(x) be differentiable at all points in the Thus, f is differentiable on the open interval (–3, 3).

interval [a, b], and let its range be the interval [A, B] and


further a function g(x) is differentiable in the interval [A, B], The nth-root function f(x) = n x is non-
Note:



then the composite function (gof)(x) is differentiable in differentiable at x = 0.
the interval [a, b]. We may combine this result with the previous theorem. Then
2. If the function f is differentiable everywhere and we see that a root of a differentiable function is differentiable
the function g is differentiable everywhere, then the except possibly at the points where the given function is zero.
composition gof is differentiable everywhere. That is, the composition
3. If f(x) is differentiable, then | (f(x))p |, p > 1 is also h(x) = n g ( x ) = [g(x)]1/n
differentiable.


For example, f(x) = 2
sin x
and g(x) = x  x  are of f(x) = n x and the differentiable function g(x) is
x +1 differentiable at x = a if g(a) ≠ 0.
ps


differentiable for all x. Hence, the composite function Example 4: Show that the function
el



sin x sin x
eh

2/3
(gof)(x) = 2 is also differentiable for all x.  x 1 
x +1 x2 +1 f(x) =  2 

 x  2x  2 

je

Example 2: Check the differentiability of is differentiable everywhere except at one point.


iit


x3 Solution: Note first that the denominator
@

f(x) =


sin(cos x ) x2 + 2x + 2 = (x + 1)2 + 1 is never zero.

Hence the rational function
Solution: The numerator is differentiable for all x. As far
x +1


as the denominator is concerned, according to the theorem on r(x) =
differentiability of a composite function, it is differentiable at x + 2x + 2
2

points where the function u = cos x is differentiable, since the is defined and differentiable everywhere. It then follows from
function sin u is differentiable everywhere. the theorem and the differentiability of the cube root function
We must exclude the points at which sin(cos x) = 0. i.e. the that f(x) = [r(x)]2/3 = 3
[r ( x )]2 is differentiable everywhere
points at which cos x = kπ (k ∈ I), or cos x = 0. Thus, the
except when r(x) = 0.
function f(x) is differentiable everywhere except at the points
Here r(x) = 0 at x = – 1.

x = (2n + 1)π/2 (n ∈ I).
2/3
Example 3: What can you say about the differentiability  h 
 2  0 = − ∞


h 1 
9 − x2 ? f '(1 )  lim 

of the function f(x) =
h 0 h
Solution: Because the natural domain of this function


is the closed interval [–3, 3], we will need to investigate the
2/3
 h 
 2  0 = ∞
differentiability of f on the open interval (–3, 3) and at the two h 1 
f '(1 )  lim 


endpoints. If c is any number in the interval (–3, 3), then f is h 0 h
differentiable at c. The function f is non-differentiable at the
Hence, f is differentiable everywhere except at x = – 1.
endpoints since
Example 5: Let f(x) = [sin x] + [cos x], x ∈ [0, 2π],
9 − (3 − h ) 2 − 0



f '(3 ) = lim where [.] denotes the greatest integer function.

h →0 −h Find the number of points where f(x) is non-differentiable.

FREE BOOKS FOR JEE & NEET =>(@iitjeeadv)


Differentiability 3.39

π Continuity of g(x):
Solution: [sin x] is non-differentiable at x = , π, 2π
2 g(x) is continuous at all those points where both f(|x|) and |f(x)|


π 3π
and [cos x] is non-differentiable at x = 0, , , 2π. are continuous.
2 2 At x = 0, f(|x|) is continuous but |f(x)| is discontinuous.
Thus, f(x) is definitely non-differentiable at

∴ g(x) is discontinuous at x = 0 and hence non-differentiable.


x = 0, π, . At x = 1, f(|x|) is differentiable but |f(x)| is non-differentiable

2 π because of corner. Hence, g(x) is non-differentiable at x = 1.
We need to check at x = and 2π, since both the functions


2 Similarly g(x) is non differentiable at x = 2.
are non-differentiable at these points. Finally, g(x) is discontinuous at x = 0 and it is non differentiable
   at x = 0, 1, 2.
f    1, f   0  0 Theorem Let f(x) and g(x) be defined on an open interval
2 2 


containing the point x = a where f is differentiable at a, f(a) = 0
and, f(2π) = 1, f(2π – 0) = – 1.
and g is continuous at a, then the product f.g is differentiable at a.
π
Thus, f(x) is discontinuous at x = and 2π and hence non- f (a + h ) g (a + h ) − f (a )g (a )
differentiable at these points. 2 (fg)′(a) = lim
h →0 h
π 3π
Finally, f is non-differentiable at x = 0, , π, , 2π. f (a + h ) g (a + h ) − 0.g (a )
2 2 = lim [since f(a) = 0]
i.e. at 5 points.
h →0 h
 x  3 x0
Example 6: If f(x) =  2 f (a  h )  f (a )
 x  3x  2 x  0 = lim .g (a  h )



h 0 h
and g(x) = f(|x|) + |f(x)|, then comment on the continuity and
ps
differentiability of g(x) by drawing the graph of f(|x|) and, |f(x)|. f (a  h )  f (a )
. lim g (a  h ) = f ′(a). g(a)
el
= lim

h 0 h h 0
Solution: Graph of y = f(x)
eh

As h approaches zero, g(a+h) approaches g(a) because g is




continuous at a.
je

Therefore, f(x). g(x) is differentiable at x = a.


iit

For example, f(x) = sin x.(|x – π| + 1) is differentiable at x = π,


@

since sin x is differentiable at x = π, sin π = 0 and (|x – π| + 1)


is continuous at x = π.
Corollary. If f(x) is differentiable at x = a, then the product
f(x).|f(x)| is also differentiable at x = a.
For example, f(x) = sin x|sin x| is differentiable at x = 0, since
sin x is differentiable at x = 0.
Example 7: Let f(x) = e(x – 1) – ax2 + b and



e x1 , x  1
g(x) =  2 with f ′(1) = 2. Find the values
 x , x  1
of a and b so that the function h(x) = f (x) · g(x), is differentiable

at x = 1.
Solution: Note that g is continuous at x = 1, but g′(1)


does not exist.
Also f ′(x) = ex – 1 – 2ax
1
f ′(1) = 1 – 2a = 2 ⇒ a  


2
In such a situation, for h (x) = f (x)· g (x) to be differentiable at
x = 1, f(1) should be 0, using the theorem given above.
f (1) = 0 ⇒ 1 – a + b = 0

⇒ b3.

2

FREE BOOKS FOR JEE & NEET =>(@iitjeeadv)


3.40  
Differential Calculus for JEE Main and Advanced

Concept Problems F


1. If it reasonable to assert that the sum F(x) = f(x) + g(x) (ii) f(x) = |x|, g(x) = |x|. Is it reasonable to assert that the

has no derivative at the point x = x0 if : product F(x) = f(x) g(x) has no derivative at the point
(a) The function f(x) has a derivative at the point x0, and x = x0 if :
the function g(x) has no derivative at this point? (a) The function f(x) has a derivative at the point x0, and
(b) Neither function has a derivative at the point x0? the function g(x) has no derivative at this point ?
2. Suppose that f(x) = x2 and g(x) = |x|. Then the composite (b) Neither function has a derivative at the point x0?
functions (f o g) (x) = |x|2 = x2 and (g o f) (x) = |x2| = x2 4. Find the one-sided derivatives of the function
are both differentiable at x = 0 even though g itself is not f(x) = |x – x0| g(x) at the point x0, where g(x) is a function
differentiable at x = 0. Does this contradict the chain rule? continuous at the point x0. Does the function f(x) possess
Explain. a derivative at the point x0?
3. Consider the functions: 5. If f(x) is differentiable at x = a, then prove that

(i) f(x) = x, g(x) = |x|; F(x) = (f(x) – f(a)).|x – a| is also differentiable at x = a.

Practice Problems E

6. If f(x) = tan πx and g(x) = x – 1. Is the product 1  x 2 , x  0



function F(x) = f(x) . g(x) differentiable at x = 1 ? 
10. Let f(x) = 2 x  1 , 0  x  1 without finding (fof) (x)




ps
 2

 x 3 sin 1x if x  0 x  2 , x  1
el
1
7. If f(x) = =  0 if x  0 and g(x) = x . Is the
explicitly, find whether fof is differentiable at x = 0.




eh

1  x 0  x  1
product function F(x) = f(x) . g(x) differentiable for all 
11. Let f(x) =  x  2 1  x  2 , Discuss the continuity and
je

real x ? 4  x 2  x  4


iit

8. Let g(x) be derivable on R and g(x0) ≠ 0. differentiability of fof.


@

 x − 1 , − 1 ≤ x < 0
(a) Show that if f(x) = (x – x0)2 g(x), then
12. Let f(x) =  2 , Discuss the continuity and
 x , 0 ≤ x ≤ 1

f(x0) = f ′(x0) = 0 and f"(x0) ≠ 0.
differentiability of h(x) = f(|sin x|) + |f(sin x)| in [0, 2π].

(b What can you say about f(x) = (x – x0)3 g(x)?
(c) Generalize to f(x) = (x – x0)n g(x). ea / x  e  a / x
13. Let f(x) = xp when x ≠ 0 and f(0) = 0, with p,
ea / x  e  a / x

9. Suppose u = g(x) is differentiable at x = – 5, a > 0. Find all possible values of p so that f(x) is

y = f(u) is differentiable at u = g(–5), and (f o g)′ differentiable at x = 0.
(–5) is negative. What, if anything, can be said about the 14. Discuss the differentiability of the functionf(x) = (x2–a)

values of g ′(–5) and f ′(g(–5))? |x2–5x+6|+(sin x) |sin x| for all a ∈ R.

(iii) Then we integrate f ′ (x) to get f (x) as a function of x


3.10 Functional Equations and a constant of integration. In some cases a differential

We follow the following steps to determine the functions which equation in formed which can be solved to get f (x).

are differentiable( or which can be proved to be differentiable) (iv) Finally we apply the boundary conditions to determine
and satisfying a given functional rule : the value of the constant of integration.
(i) First we write down the expression for f ′(x): Example 1: If f (x + y) = f (x) · f (y), ∀ x , y ∈ R and



f (x  h)  f (x) f (x) is a differentiable function, then find f (x).
f ′(x) = lim
f (x  h)  f (x)

h→0 h
(ii) We manipulate f (x + h) – f (x) in such a way that the given Solution: f ′(x) = lim


h0 h
functional rule is applicable. Now we apply the functional
rule and simplify the R.H.S. to get f ′ (x) as a function of x.

FREE BOOKS FOR JEE & NEET =>(@iitjeeadv)


Differentiability 3.41

f ( x )·f ( h )  f ( x ) f (x  h)  f (x)
= lim We have f ′(x) = lim
h 0

h0 h h
= lim f ( x )
f (h )  1 ...(1)  xh
f
x  [from (1)]
lim 


h0 h =



Put x = 0, y = 0; f (0) = f (0) 2 h 0 h
⇒ f (0) = 0 or f (0) = 1  h
f 1  



If f (0) = 0 then put y = 0 in the rule f 1   
= lim . 
1 x 1


f (x) = f (x) · f (0) = 0 = . lim
x 0 


h 0 x h

⇒ f (x) = 0. f (x) x

⇒ f (x) = 0. h

If f(0) = 1, then we proceed from (1) as follows : [∵ → 0 as x → 0 ∀ x ∈ (0, ∞) ]


x
f ′(x) = f(x)
f (h )  f (0) = f(x) · f ′(0) 3 f (1  x )
= [from lim  3]


h
x 0




[Let k = f ′(0)] x x
f '( x ) To find f(x), write the above equation as
⇒ =k df 3
f (x) =


⇒ ln | f (x)| = k x + c

dx x
On integrating both sides w.r.t. x we get

| f (x)| = ekx + c = ekx ·e c
f(x) = 3 ln x + c, where c is a constant.


f(x) = ± ekx ·e c
ps

Now, using the condition f(1) = 0, we have

f(x) = a.ekx where a = ± ec f(1) = c which gives c = 0
el


If x = 0, f (0) = 1 ⇒ a = 1


Hence, we have f(x) = 3 ln x.
eh


⇒ f (x) = ekx where k = f ′(0).
Example 4: A differentiable function satisfies the relation

Hence, the functions are f(x) = 0 and f(x) = ekx.
je


f (x + y) = f (x) + f (y) + 2xy – 1 ∀ x, y ∈ R.
iit

Example 2: If f (x + y) = f (x) + f (y) ∀ x, y ∈ R and f


If f ′(0) = 3  a  a 2 , find f (x) and prove that f (x) > 0


(x) is a differentiable function, then prove that
@


∀ x ∈ R.


f(kx) = k f (x) for ∀ k, x ∈ R.
Solution: Given f (x + y) = f (x) + f (y) f (x  h)  f (x)
Solution: f ′(x) = lim




f (x  h)  f (x) h0 h
f ′(x) = lim
f ( x )  f (h )  2 xh  1  f ( x )

h0 h = lim

f ( x )  f (h )  f ( x ) h0 h
= lim
f (h )  1

h0 h = 2x + lim [Put x = 0, y = 0 to get f (0) = 1]
f (0  h )  f (0) h0 h
= lim = f ′(0). [as f (0) = 0] f ( h )  f ( 0)
= 2x + lim

h0 h
⇒ f ′(x) = f ′(0) h0 h
⇒ f ′(x) = 2x + f ′(0)

∴ f (x) = f ′(0)x + c


If x = 0, f (0) = 0 ⇒ c = 0 Integrating, f (x) = x2 + f ′(0)x + c

∴ f (x) = f ′(0)x If x = 0 ; f (0) = 1 ⇒ c = 1

 




f (x) = ax, where a = f ′(0) ∴ f (x) = x2 + 3  a  a2 x 1

f (kx) = a kx = k ax = kf (x)

Now D = 3 + a – a2 – 4 = – (a2 – a + 1) < 0

Hence, f (kx) = kf (x).

⇒ f (x) > 0 ∀ x ∈ R.
Example 3: A function f : (0, ∞) → R satisfies the



equation f(x/y) = f(x) – f(y). If f(x) is differentiable on (0, ∞ ) Example 5: Given a function g which has derivative


f (1  x ) g′(x) for all x satisfying g ′(0) = 2 and g(x + y) = ey g(x) + ex
 3 , then determine f(x).

and lim
x 0
g(y) for all x, y ∈ R, g(5) = 32. Find the value of g ′(5) – 2e5 .
x

Solution: We have f(x/y) = f(x) – f(y) ...(1) Solution: Putting x = y = 0 in






Putting x = 1 and y = 1 in equation (1), we have g(x + y) = ey g(x) + ex g(y) we get g(0) = 2g(0)
f(1) = f(1) – f(1) ⇒ f(1) = 0. ⇒ g(0) = 0


FREE BOOKS FOR JEE & NEET =>(@iitjeeadv)
3.42  
Differential Calculus for JEE Main and Advanced

g ( h )  g ( 0) g(h )  f ( 2 x ) + f ( 2h ) 
So 2 = g ′(0) = lim  lim  − f (x) 
h h 0 h 0 h 2
= lim  
h →0 h
g( x  h )  g( x )  


Also g′(x) = lim  
h 0 h
 f ( 2 x ) + f ( 2 h ) − 2f ( x ) 
= lim e g (h )  e g ( x )  g ( x )
x h
= lim  
h →0  2h 



h 0 h
 g(h ) eh  1   f ( 2 h ) − f ( 0) 
= lim  e x  g( x )  = lim   (from (1))
 h →0  2h 



h 0


 h h 
g(h ) = f ′(0)



= ex lim  1. g ( x ) = – 1 ∀ x ∈ R (given)
h 0 h


= g(x) + 2e .
x Integrating, we get f(x) = – x + c
Thus, g ′(5) – 2e5 = g(5) = 32. Putting x = 0, then f(0) = 0 + c = 1 (given)
Example 6: A differentiable function f(x) satisfies the ∴ c = 1 then f(x) = 1 – x




condition f(x + y) = f(x) + f(y) + xy for all x, y ∈ R and ∴ f(2) = 1 – 2 = – 1.

1 Alternative 1:
lim f (h )  3 then find the least value of f(x).
 x  y  f ( x )  f ( y)

h 0 h
∵ f 
f (x  h)  f (x)  2  2

Solution: f ′(x) = lim
h 0
ps
Differentiating both sides w.r.t x treating y as constant.


h
f ( x )  f (h )  hx  f ( x )  x  y  1 f '( x )  0 xy
el
= lim ∴ f′ · 2  ⇒ f '  = f ′(x).
h 0  2  2  2 

eh

h
f (h ) Replacing x by 0 and y by 2x,
je

= lim +x=3+x
h →0 h then f ′(x) = f ′(0) = – 1 (given)

iit

x2 Integrating, we have f(x) = – x + c.


Integrating f(x) = 3x + +k ...(1) Putting x = 0, f(x) = 0 + c = 1 (given)
@

2
∴ c=1
Putting x = 0, y = 0 in the given relation

Hence, f(x) = – x + 1 then f(2) = – 2 + 1 = – 1.
f(0) = f(0) + f(0) + 0 ⇒ f(0) = 0
Alternative 2:


Now from (1) we have f(0) = 0 + k ⇒ k = 0 Suppose A(x, f(x)) and B(y,f(y)) be any two points on the

x2 curve y = f(x).
∴ f(x) = 3x + .
If M is the mid point of AB, then coordinates of M are

2
It is a quadratic function whose least value occurs at  x  y f ( x )  f ( y) 
9  2 . 
 

x = – 3. The least value is − . 2
2 According to the graph, coordinates of P are
 x  y  f ( x )  f ( y)  x  y  x  y 
Example 7: Let f   for all real x and  .f 
 2   2 

2  2


y. If f ′(0) = – 1 and f(0) = 1, then find f(2).
xy f ( x ) + f ( y)
and PL > ML ⇒   >
 x  y  f ( x )  f ( y)  2  2
Solution: Given f  
 2 


2
f (2 x ) + f (0)
Replacing x by 2x and y by 0, then f(x) =
2
⇒ f(2x) + f(0) = 2f(x) ⇒ f(2x) – 2f(x) = – f(0) ...(1)

f (x + h) − f (x)
f ′(x) = lim
h →0 h

 2 x + 2h 
f
  − f (x)
2 
= lim

h →0 h

FREE BOOKS FOR JEE & NEET =>(@iitjeeadv)


Differentiability 3.43

 x  y  f ( x )  f ( y) f ′(x) = f ′(0) = c (say)


But given f   which is possible when


At x = 2,
 2  2
f ′(2) = c = 2 (given)
P → M, i.e., P lies on AB. Hence, y = f(x) must be a linear


∴ f ′(x) = 2
function.


On integrating we get
Let f(x) = ax + b ⇒ f(0) = 0 + b = 1 (given) f(x) = 2x + a
and f ′(x) = a ⇒ f ′(0) = a = – 1 (given) Putting x = 0, then f(0) = 0 + a = 2 {from (1)}
∴ f(x) = – x + 1 ∴ f(x) = 2x + 2.

∴ f(2) = – 2 + 1 = – 1. Example 9: Let f(xy) = xf(y) + yf(x) for all



 x  y  2  f ( x )  f ( y) x, y ∈ R+ and f(x) be differentiable in (0, ∞) then determine f(x).
Example 8: If f   for all real
 3  3 Solution: Given f(xy)= xf(y) + yf(x)


x and y and f ′(2) = 2 then determine y = f(x).



Replacing x by 1 and y by x then we get x f(1) = 0
 x  y  2  f ( x )  f ( y) ∴ f(1) = 0, x ≠ 0 (∵ x, y, ∈ R+)
Solution: f  



...(1)
 3  f (x  h)  f (x)



3
Now, f ′(x) = lim
Replacing x by 3x and y by 0 then h 0


h
2 + f (3x ) + f (0)
f(x) =   h 
f  x 1     f ( x )

3 =  x 
⇒ f(3x) – 3f(x) + 2 = – f(0) ...(2) lim 

h 0 h

Putting x = 0 and y = 0 in (1),
we get f(0) = 2 ...(3)  h  h
ps
xf 1    1   f ( x )  f ( x )

lim 
f (x + h) − f (x)
= x  x
el


Now, f ′(x) = hlim
→0
h 0 h
h
eh

 3x + 3h   h h
f − f (x) xf 1    f ( x )
 3 
je

lim 
= x x

= lim
iit

h →0 h 0 h

h
2 + f (3x ) + f (3h )  h
@

− f (x) f 1  
= lim 3  x
 lim
f (x)
= lim
h

h →0 h 0 h 0 x


h
x
f (3x ) − 3f ( x ) + f (3h ) + 2  
= lim

h →0 3h f (x)
= f ′(1) +
f (3h )  f (0)

= lim {from 2)} x

h 0 3h
f (x)
= f ′(0) = c (say)
⇒ f ′(x) – = f ′(1)


x

∴ f ′(x) = c
xf ( x )  f ( x ) f (1)

At x = 2, f ′(2) = c = 2 (given) ⇒ 
x2


x
∴ f ′(x) = 2

Integrating both sides, we get f(x) = 2x + a f ( x )  f ( x )  f (1)
⇒  
Putting x = 0 then x  x  x

f(0) = 0 + a = 2 {from (2)} On integrating w.r.t.x and taking limit 1 to x,

∴ a = 2 f ( x ) f (1)
then f(x) = 2x + 2. we have − = f ′(1) (ln x – ln 1)
x 1
Alternative:
f (x)
We have f  x  y   2  f ( x )  f ( y) ⇒ – 0 = f ′(1) ln x (∵ f(1) = 0)

x
 3  3
∴ f(x) = f ′(1) (x ln x).
Differentiating both sides w.r.t. x treating y as constant,

Alternative:
 x  y   1  2  f '( x )  0 Given f(xy) = xf(y) + yf(x)
we get f '    

 3  3  3 Differentiating both sides w.r.t. x treating y as constant,
Now replacing x by 0 and y by 3x, then f ′(xy) . y = f(y) + y f ′(x)

FREE BOOKS FOR JEE & NEET =>(@iitjeeadv)
3.44  
Differential Calculus for JEE Main and Advanced

Putting y = x and x = 1, then


 f (x) 


f ′(xy). x = f(x) + xf ′(x)  using xlim = 2


→0 x
⇒ xf ( x )  f ( x )  f (1) 1 2


x2 x ⇒ f ′(x) = 2 × ⇒ f ′(x) =
1+ x 1+ x2 2



⇒ d  f ( x )   f (1) Integrating both sides, we get

dx  x  x
f(x) = 2 tan–1(x) + ck, where f(0) = 0 ⇒ c = 0
Integrating both sides w.r.t. x taking limit 1 to x, Thus, f(x) = 2 tan–1 x.
f (x) f (1)
= f ′(1){ln x – ln 1}  1   
– Hence, f   = 2  , and

x 1  3  6 3
f (x)
(∵ f(1) = 0)
2 2
⇒ – 0 = f ′(1) ln x f ′(1) =  1.
1  12

x


2
Hence, f(x) = f ′(1)(x ln x).
Example 11: If e –xyf(xy) = e –xf(x) + e –yf(y) ∀ x,



 xy  y ∈ R+, and f ′(1) = e, determine f(x).
Example 10: If f(x) + f(y) = f   for all x, y ∈ R
 1  xy  Solution: Given e–xy f(xy) = e–xf(x) e–yf(y) ...(1)





Putting x = y = 1 in (1), we get, f(1) = 0 ...(2)
f (x)  1 
(xy < 1) and lim =2. Find f   and f ′(1). f (x  h)  f (x)
x →0 x  3 Now, f ′(x) = lim
h 0


ps h
 xy    h 
Solution: f(x) + f(y) = f   ...(1) f  x  1     f ( x.1)
 1  xy 
el
 x 
lim 
=


 

Putting x = y = 0, we get f(0) = 0.
eh

h 0 h
Putting y = – x, we get f(x) + f(–x) = f(0) 
 1
h
 
h 
e x  h e  x f ( x )  e x f 1     2 x (e  x f ( x )  e 1f (1))
je

⇒ f(–x) = – f(x) ...(2) = 


  x 


iit

lim
h 0 h
f (x)
Also, lim =2
@

h
x →0 x
x  h 1  h x 1
eh f ( x )  e xf
1  x   f ( x )  e f (1)
=  
f (x + h) − f (x)

lim
Now, (x) = lim ...(3) h 0 h

h→0 h h
h  h
1  x 
e xf
lim f ( x + h ) − f ( x )
= h→0 (using (2))  eh  1   
= f(x) lim   + e
(x – 1)
lim


h h 0


 h  h 0 h
x.
 x+h−x  x
 1 − ( x + h )( − x )  (∵ f(1) = 0)

⇒ f ′(x) = lim h (using (1))
f ′(1)


h→0
= f(x) . 1 + ex – 1.
x


  h 
f  
⇒ f ′(x) = lim   1 x ( x  h  e x−1.e
h→0 
= f(x) + (∵ f ′(1) = e)




h x
 
 
ex 1
f ′(x) = f(x) + ⇒ e–xf ′(x) – e–x f(x) =
  x




h x
f 2 
⇒ f ′(x) = lim  1  xh  x    1 

d –x 1
  (e f(x)) =

h→0    1  xh  x 2  x


h dx
 2 
 1  xh  x  On integrating we have
e–xf(x) = ln x + c at x = 1, c = 0
 

f
h ∴ f(x) = ex ln x.
2 

⇒ f ′(x) = lim  1  xh  x   lim 1    
Example 12: A function f : (–1, 1) →  ,  satisfies


h→0
 h  h 0 1  xh  x 2  2 2


 2 
 1  xh  x  the equation

FREE BOOKS FOR JEE & NEET =>(@iitjeeadv)


Differentiability 3.45

gives f(x) = sin–1x + c (c is a constant).


f(x) + f(y) = f(x 1 − y 2 + y 1 − x 2 ).
Using the condition f(0) = 0 gives c = 0, and hence we

(i) Show that f(x) is odd. have f(x) = sin–1x.
(ii) If f(x) is differentiable on (–1, 1) and f ′(0) = 1, then show
1 Example 13: A differentiable function f satisfies the
that f ′(x) =



1− x2 relation f(x + y) – 2f(x – y) + f(x) –2f(y) = y – 2
(iii) Hence, determine f(x). ∀ x , y ∈ R. Find f (x).
Solution: f(x + y) – 2f(x – y) + f(x) –2f(y) = y – 2
Solution:



...(1)


(i) f(x) + f(y) = f(x 1 − y 2 + y 1 − x 2 ).


...(1) Put x = y = 0 in (1)


Putting y = –x in equation (1), we have ⇒ f(0) = 1


f(x) + f(–x) = f(x 1 − x 2 – x 1 − x 2 ) Put y = h in (1)

⇒ f(x) + f(–x) = f(0) f(x + h) –2f(x – h) + f(x) – 2f(x) = h – 2


Putting x = 0 and y = 0 in equation (1), we have ⇒f(x + h) – f(x) – 2[(f(x – h) – f(x)] – 2(f(h) – 1) = h
f(0) + f(0) = f(0) ⇒ f(0) = 0. Dividing by h on both sides and applying limit h→ 0

Hence, f(x) + f(–x) = 0 ...(2) f ( x + h ) − f ( x ) 2(f ( x − h ) − f ( x ))
+

Thus f(x) is odd. lim

h →0 h −h
(ii) Now, f ′(0) = 1
 2f (h )  1 
f ( )  f  0    =1
⇒ =1 [using f(0) = 0]  

lim h
0 
ps



⇒ f ′(x) + 2f ′(x) – 2f ′(0) = 1 [∵f is differentiable]
f ( )

el
⇒ lim =1 ⇒ 3f ′(x) = 1 + 2f ′(0) ...(2)
 0 


eh


Putting x = 0 in (2) we get f ′(0) = 1.
f (x  h)  f (x) ⇒ f ′(x) = 1
je

Now f ′(x) = lim



h 0 h ∴ f(x) = x + c
iit

f ( x  h )  f ( x ) Since f(0) = 1, c = 1, we have f(x) = x + 1.
@

= lim [using(2)]
h 0 Example 14: A differentiable function f satisfies


h


f(x + y) + f(x – y) – (y + 2) f(x) + y(x2 – 2y) = 0 ∀ x , y ∈ R.
f {x  h ) 1  x 2  x 1  ( x  h ) 2 } Find f (x).
= lim

h 0 h
Solution: Put y = h
[using (1)]


f(x + h) – f(x) + f(x – h) – f(x) – hf(x) + h(x2 – 2h) = 0


(x  h ) 1  x 2  x 1  (x  h )2 Dividing by h on both sides
= lim f () . lim
 f (x + h) − f (x) f (x − h) − f (x)

0 h 0 h
+ – f(x) + x2–2h= 0 and

assuming δ = ( x  h ) 1  x 2  x 1  ( x  h ) 2 h h
applying limit h→ 0 on both sides, we get
(x  h ) 1  x 2  x 1  (x  h )2 f ′(x) – f ′(x) – f(x) + x2 = 0

= lim ⇒ f(x) = x2.

h 0 h

Example 15: A twice differentiable function f satisfies
= lim x ( 1  x  x 1  ( x  h ) )  h 1  x
2 2 2


the relation f(x2 + y2) = f(x2 – y2) + f(2xy) ∀ x, y ∈ R. If

h 0 h h f(0) = 0 and f ′′(0) = 2, find f(x).
x (1  x  1  ( x  h ) )
2 2

= 1  x  hlim
2 Solution: f(x2 + y2) = f(x2 – y2) + f(2xy)


0
h( 1  x  1  (x  h) )
2 2

Put y = h
f(x2 + h2) = f(x2 – h2) + f(2xh)
x2

1
= 1 x2   Dividing by h2 on both sides
1 x 1 x2

2
f (x 2  h 2 )  f (x 2 ) f (x 2  h 2 )  f (x 2 )


1 h2 h2
Hence, f ′(x) = ...(3)
1− x 2


f (2 xh )
+ ...(1)
(iii) Integrating both sides of (3) h2



FREE BOOKS FOR JEE & NEET =>(@iitjeeadv)
3.46  
Differential Calculus for JEE Main and Advanced

f (2 xh )
f '(2 xh ).2 x
Now lim 2
 lim ⇒ f (x) = x2 + 2 ( as f (x) > 0 )
h 0 h 0



h 2h
(using L′Hospital ′s rule) Example 17: function f : (0, ∞) → R satisfies the equation



xf "(2 xh ).2 x x
= lim f(xy) = 2f(x) – f   .If f is differentiable on R and f(1) = 0,
+

y

h →0 1
= 2x2f"(0). f ′(1) = 1, then show that
Applying limit h → 0 on both sides of (1), we get  1 x
f ′(x2) = – f ′(x2) + 2x2f"(0) (i) f(y) = – f   (ii) f(x) + f(y) = f  
 y





y

2f ′(x2) = 2x2f"(0) and hence determine f(x).
Replacing x2 by t x
Solution: We have f(xy) = 2f(x) – f   ...(1)
2f ′(t) = 2t f"(0) ⇒ f ′(t) = 2t




y

Integrating both sides w.r.t. t (i) Putting x = 1 in (1), we have
 1
∫ f '(t) dt = ∫ 2t dt 2f(1) = f(y) –  

 y
f(t) = t2 + c
 1

Since f(0) = 0, c = 0. ⇒ f(y) = –f   [since f(1) = 0] ...(2)

∴ f(t) = t2 or, f(x) = x2.  y





Example 16: f(x) is a differentiable function satisfy the (ii) In (1) we exchange x and y to get


relationship f 2(x) + f 2(y) + 2 (xy –1) = f 2(x + y)  y
f(xy) = 2f(y) – f  

∀ x, y ∈ R. Also f (x) > 0 ∀ x ∈ R , and f ( 2) = 2.  x
ps

Determine f (x).
x
⇒ f(xy) = 2f(y) + f   [using (2)] ...(3)
el
Solution: Put x = 0 and y = 0 ⇒ f 2(0) = 2


y


eh

f ′(x) = lim f ( x  h )  f ( x ) Now we subtract (1) from (3)



x
h→0
je

h
0 = 2f(y) – 2f(x) + 2 f  
iit


lim f ( x  h )  f ( x )
2 2 y
= h→0 x
[f ( x  h )  f ( x )]. h
@

⇒ f(x) – f(y) = f  

...(4)


y
lim f (h )  2( xh  1)
2
We have
= h→0  x + h

2f ( x ) h f
f (x + h) − f (x)  x 
  f ′(x)= lim = lim [using (4)]
lim  2xh  f (h )  2 
2
1 h →0 h h →0 h

= h→0
 h h 

2 f (x)  h
f 1  
1  f ( h )  f ( 0)  2 2  x  f '(1)
=  2 x  lim  = lim
h 0 h
=

2 f ( x )   h  x

h 0
x.
x
1  f ( h )  f ( 0) 
2 x  lim .  f ( h )  f ( 0)  
2 f ( x ) 
= 1
h0 h  i.e. f ′(x) =


x
1 This gives f(x) = ln x + c
f ′(x) = [2 x  2 f (0). f '(0)]
2 f ( x ) Now, using the condition f(1) = 0 gives c = 0.

f (x) . f ′ (x) = x + f (0). f ′(0) Hence, we have f(x) = ln x.


⇒ f (x) . f ′ (x) = x + λ, where λ = f (0) · f ′(0) Example 18: A differentiable function f satisfies the


Integrating both sides,


relation f(x + y) + f(xy – 1) = f(x) + f(y) + f(xy) ∀ x, y ∈ R.

f 2 (x) x2 If f(1) = 2, f ′(0) = 1 and f ′(– 1) = – 1, find f(x).
= + λx + c
2

2 Solution: Putting x = 0, y = 0 in the given rule,
f 2(x) = x2 + 2λx + c


f(0) + f(–1) = 3f(0)

at x = 0 , f 2 (0) = 2 ⇒ c=2

⇒ f(–1) = 2f(0) ...(1)


at x = 2 , f ( 2 ) = 4 ⇒ λ = 0
2

Now, putting y = h

f 2(x) = x2 + 2 f(x + h) + f(xh – 1) = f(x) + f(h) + f(xh)


FREE BOOKS FOR JEE & NEET =>(@iitjeeadv)
Differentiability 3.47

⇒ f(x + h) – f(x) + f(– 1+ xh ) – f(– 1) f ( x )  f ( y)


= f ′(x), we see from (2) that

Since lim
= f(h) + f(xh) – f(– 1) xy yx



⇒ f(x + h) – f(x) + f(– 1+ xh ) – f(– 1) |f ′(x)| ≤ 0 ⇒ f ′(x) = 0

= f(h) – f(0) + f(xh) – f(0) [using(1)]
⇒ f (x) = c.

Dividing both sides by h,
Hence f(x) is a constant function.
f ( x  h )  f ( x ) f (1  xh )  f (1)
 .x Example 20: Suppose
h xh



p(x) = a0 + a1x + a2x2 + ... + anxn.

f (h )  f (0) f ( xh )  f (0)


=  .x If | p(x) | ≤ | ex–1 – 1 | for all x ≥ 0, prove that



h xh | a1 + 2a2 + ... + nan | ≤ 1.
Now, applying limit h → 0 on both sides , we get


Solution: Given p(x) = a0 + a1x + a2x2 + ... + anxn
f ′(x) + xf ′(–1) = f ′(0) + x f ′(0)



∴ p′(x) = 0 + a1 + 2a2 x + ... + nanxn–1

Given that f ′(0) = 1, f ′(–1) = –1, we have


f ′(x) = 2x + 1 ⇒ p′(1) = a1 + 2a2 + ... + nan


Now, | p(1) | ≤ | e1–1 – 1| = |e0 – 1| = 0

f(x) = x2 + x + c

f(1) = 2 ⇒ c = 0 ⇒ | p(1) | ≤ 0 ⇒ p(1) = 0




∴ f(x) = x2 + x. As | p(x) | ≤ | ex–1 – 1 |


Example 19: Let f : R → R be such that for all x and we get | p(1 + h) | ≤ | eh – 1| ∀ h > – 1, h ≠ 0
⇒ | p(1 + h) – p(1) | ≤ |eh – 1|


y in R, |f(x) – f(y) | ≤ |x – y|3 . Prove that f(x) is a constant ( p(1) = 0)


function. p(1 + h ) − p(1) p h −1
ps
⇒ h

Solution: We are given that h

el


| f(x) – f(y) | ≤ |x – y|3 ...(1)
Taking limit as h → 0 on both sides, then


eh

Let x be any real number and let y be chosen arbitrarily close


to x but not equal to x. Then writing (1) as p(1 + h ) − p(1) eh − 1
⇒ ≤ lim
je

lim
h →0 h →0

f ( x ) − f ( y) h h
≤ |x – y|2
iit

x−y

p(1 + h ) − p(1) eh − 1
⇒ ≤ lim
@

lim
and letting y → x, we get h →0 h h →0 h

f ( x )  f ( y) 2
⇒ | p′(1)| ≤ 1
lim  lim x  y ...(2)
xy

yx yx


⇒ | a1+ 2a2 + ... + nan| ≤ 1 {from (1)}


Practice Problems F

1. A function f : R → R+ satisfies f(x + y) = f(x) .  xy  f ( x )f ( y )

f(y) ∀ x ∈ R. If f ′(0) = 2, then show that f ′(x) = 2f(x). 6. Let f   = for all real x and y. If
 2  6



2. Let f(xy) = f(x) f(y) ∀ x, y ∈ R and f(1) ≠ 0, f ′(1) = 1, f(1) = f ′(1) = 3, then prove that one of the functions

f satisfies f(x) + f(1 – x) = 3 for all non-zero real x.

prove that f is differentiable for all x ≠ 0. Hence determine
f(x). x
7. If 2f(x) = f(xy) + f   ∀ x, y ∈ R+, f(1) = 0 and
 x  y  f ( x )  f ( y) y

3. Let f   for real x and y. If f ′(0)
f ′(1) = 1 then find f ′(2) and f(2).
 2 

2

exists and equals –1 and f(0) = 1 then find the value of f(2) 8. Let f : R → R such that f(x + y)

4. If f(x) satisfies f(1 – x) = f(x) ∀ x ∈ R and f ′(1) = 0 then = f(x) + f(y) + ex + y (x + y) – xex – yey + 2xy ∀ x, y ∈ R

then find f(x) given that f ′(0) = 1.

find f ′(0) if it exists.


xy f ( x ) + f ( y)  2 x  3y  2f ( x ) + 3f ( y)
5. Let f   = ∀ x, y ∈ R, n > 2 and 9. If f   = ∀ x, y ∈ R, f(0) = 1,
 n   

n 5 5

f ′(0) = 2 then find f(x). f ′(0) = – 1, prove that

FREE BOOKS FOR JEE & NEET =>(@iitjeeadv)
3.48  
Differential Calculus for JEE Main and Advanced

n 1 10. A differentiable function f satisfies the relation f(x + y)


n 2 (n + 1) 2
 (f (r))3


=– . [f(x) – f(y)] = f(x –y)[f(x) + f(y)], where f(1) = 2. Find f(x).

r 2 4

Target Problems for JEE Advanced


Problem 1: Check the differentiability of f(x) at x = 0, h ( )
⇒ 2 lim =0 ...(1)


where   0




 2e | x |{x} 
f(x) = x  x≠0 Now consider f ′(x) at x = 0.
 | x | {x} 
,


  f (  )  f ( 0) h () cos(1 / )
R.H.D. = lim  lim
1 0    0 
, x=0 =
h ( )


2 . lim cos(1 / )
= lim
where { } represents the fractional part function. 0  0


Solution: =0 [ using (1)] ...(2)





 2e |h |{h}  1 f ( 0)  f (   )  h () cos(1 / )
h    L.H.D.= lim  lim
  | h | {h}  2 0  0 
 lim 
f ( 0 h ) f ( 0 )
f ′(0+) = lim
h 0 h h 0 h
 h ( )
 2  e2h  1 = lim . lim cos(1 / )
ps 0  0


h   
 2h  2
el
= lim = 0 [ using (1)] ...(3)



h 0 h Since R.H.D. = L.H.D. = 0, we have f ′(0) = 0.


eh

[∵ |h| + {h} = h + h = 2h for small h > 0]   1 


1 

Problem 3: If f ( x )   xe  x

 x 
je

, x 0,
1  e2h


a ,
iit

= lim = –ln e = –1.  x 0



h 0 2h
find the value of ‘a’ such that f(x) is differentiable at x= 0.
@

f ( 0)  f ( 0  h )
f ′(0 ) = lim

Solution:
h 0



h 1 1 
  
h h
1  2e | h |{ h}  f ( h )  f ( 0) h e   a
 ( h )   f ′(0) = lim = lim
 |  h | { h} h0

2 h0
 h h
= lim
h 0 1 1

h   
h e h h a
1
 h ( 2  e) f ′(0 +) = lim

h0 h
= lim 2
h 0 h . e 2 / h  a

h a
= lim = 0 − lim = 0 , provided a = 0.
[∵ |–h| + {–h} = h + 1– h = 1 for small h > 0] h0 h→0 h

h

 1 1
1    

= lim h . e  h h a
h 0 2h Also f ′(0 –) = lim

h0 h

Hence, f is non-differentiable at x = 0.
a
Problem 2: Find f ′(0) if f(x) = h(x)cos(1/x), x ≠ 0, = 1 + lim
= 1, provided a =0.
h→0 h



f(0) = 0, where h(x) is an even function differentiable at x = 0
Hence, a = 0.
and h(0) = 0.  x  3 x, x 1

Solution: The existence of h′(0) implies that Problem 4: Let f(x) =  x
  x 1


 sin ,


h (  )  h ( 0) h ( 0)  h (   )  2
lim  lim where [.] denotes the greatest integer function.
0  0 
Find whether f(x) is

h ( )  h ( ) (i) is continuous at x = 0,
⇒ lim  lim


0  0  (ii) is differentiable at x = 0,


FREE BOOKS FOR JEE & NEET =>(@iitjeeadv)
Differentiability 3.49

(iii) is continuous but not differentiable at x = 1,


2 h  3  h  0

(iv) is continuous but not differentiable at x = 3/2. 2  2h .1
= lim = lim =2

h→0 h→0


Solution: h h


  h  3
(i) lim f(x) = lim f(0 − h) = lim sin   =0 ∴ f(x) is not differentiable at x = but it is continuous
x0  2 


h→0 h→0


2
πh there.
lim f(x) = lim f(0 + h) = lim sin =0
x0 2


h→0 h→0 Problem 5:



f(0) = 0  1

∴ f(x) is continuous at x = 0.  (1  {x}) 1  {x}
 {x} 

f 0  h   f 0   , x  Integer
(ii) f ′(0–) = Lim 

h Let f(x) =  e  .

h→0
 


h πh 
 sin 0 π sin 2 π 
2
= lim = lim π = 2 , x  Integer
h→0 h h→0 2 h 2 e

2 
f 0  h   f 0 Discuss the continuity and differentiability of f(x) at any integral
f ′(0+) = lim point, where {.} denotes the fractional part.

h→0 h
πh Solution: Let x = I0 be any arbitrary integers.
π sin 2 π


= lim = 2
h→0 2 π h f(I0) = .

2
ps

2 e
f ′(0–) = f ′ (0+)
el
f(I0 – ) = lim f(I0 – h)


∴ f(x) is differentiable at x = 0.

h →0
eh

f 1  h   f 1  1 
(iii) f ′(1–) = lim  (1  {I  h}) {I  h}   (1  1)1 
1
je

h
0
2

h→0 = lim  0
    
iit

h 0    e 

e e
  1  h    
sin  1 h  
h  1
@

  cos
= lim = lim 2 1/ h
h→0 h h→0 h  (1  h )1/ h 

f(I0 + h) = lim (I0 + h) = lim  
 h h →0 h 0 e

 
4 .  h = lim   h  = 0
2 sin 2 2 2
= lim  
h→0  h 
2 h→0  8 

 (1 h )1/ h  1  (1 h )1/ h 
1/ h
16  
 4  = h 0 
lim  lim  1
  e  e   e h e 
h 0

f (1  h )  f (1) 1  h  1
lim lim 1  (1 h )  (1 h )1/ h  e
1/ h

f ′(1 ) =
+ h→0 h = h→0 h lim 
h e
1 lim

h 0
 e


h 0
eh
1  h  1 1− h −1 e
= lim = lim =−1

1/h
Now (1 + h)

h→0 h h→0 h


f ′(1 ) ≠ f (1 ) – ′ + 1
ln(1 h )
1  h2 h3 
 h   .... 
 1 h h2 
 h    ..... 
=

h   
Hence f(x) is not differentiable at x = 1 but it is continuous e h e 2 3
 e.e 2 3 4

there.  1 h h2 

   
 h    .... 
2 3 4 

f 3 h f 3 e  
1
2 2 ⇒ f(I0 +) = lim
(iv) f ′(3/2–) = lim h 0

e h
h

h→0
 − h  12 − h3 + h42 −....   1 h h 2
3  2 h  3  3  h  0 e   
−1  − + −.... ( −1)
2     2 3 4 
= lim
h→0 h

lim 1
h →0
 1 h h2  −
2h .1 = e − h  − + −....
2 3 4  =e 2
= lim =−2
h→0 −h


Since f(I0 + ) ≠ f(I0 –)
f ′(3/2 ) = lim
+
f 3 h f 3
2 2     ⇒ f(x) is discontinuous at all integral points and hence non-

differentiable.

h→0 h

FREE BOOKS FOR JEE & NEET =>(@iitjeeadv)


3.50  
Differential Calculus for JEE Main and Advanced

Problem 6: Let 1 1
Thus, since ≠ 0 and → 0 as n → ∞,



   {x}{ x}   n n
   x  
  5  1
 n  e [ x ]  [ x ] 
x

2e
for x  0 f 
   n
f ′(0) = lim  1   0,
1
  3 e |x|  n   


f(x) =     n 


 0 for x  0  

 1  e| x | {x} 1
because f    0 for all n ∈ N.
x  for x  0  


n
 | x | {x}
where [ ], { } represents integral and fractional part functions Problem 8: If f(x) = 3x10 – 7x8 + 5x6 – 21x3 + 3x2 – 7 then



respectively. Compute the one-sided derivatives at x = 0 and f (1  h )  f (1)
find the value of lim .
comment on the continuity and differentiability at x = 0. x1 h 3  3h 

 1  eh  h  Solution: f (x) = 3x10 – 7x8 + 5x6 – 21x3 + 3x2 – 7


h    0



f (1  h )  f (1) h
hh l = lim ·
Solution: f ′(0+) = lim   h h (h 2  3)


h0
h →0


h
1 1
(1 − e 2 h ) l = lim  f '(1)· f '(1) =– ...(1)
= lim h =–1 h 3 3 2


h0

h →0 ( 2h ) h
Now, f ′(x) = 30x9 – 56x7 + 30x5 – 63x2 + 6x
 1 h  h 
ps
f ′(1) = 30 – 56 + 30 – 63 + 6 = – 53
 2 e | h |  5 
 h  n  e  1  0   

 1 53
∴ From (1), l = – (– 53) =
el
1
   .
 3 e h
 3 3
eh

f ′(0 ) = lim

h →0 h Problem 9: If f (x + y) = f (x) + f (y) + | x | y + xy2,


je

 1  ∀ x, y ∈ R and f ′(0) = 0, then prove that f is differentiable for


 2eh  5 
all x ∈ R, but it is not twice differentiable at x = 0.
iit

h   1 
 3  eh  Solution:
 
@

= lim


f ′(x) = lim f ( x  h )  f ( x )  lim f (h ) | x | h  xh
2

h →0 h


1 h 0 h h 0 h
2  5e h ∵ f(0) = 0, we have
= lim =2

1

h →0 
3e 1 h  f ( h )  f ( 0) 
f ′(x) = lim   | x |  xh 
h0  h 

Hence f is continuous but not derivable at x = 0.
f ′(x) = f ′ (0) + | x | = | x |.
Problem 7: Let f be a continuous function from R to R

Hence, f is differentiable for all x ∈ R.


such that f is differentiable at 0. Suppose that f(1/n) = 0 for
all n ∈ N. It is easily seen that f ′ (x) = | x | is not differentiable at
(i) Prove that f(0) = 0. (This fact does not requires the x = 0. So, f is not twice differentiable at x = 0.


differentiability of f) Problem 10: If the function f(x) = |x – a|.g(x), where g(x)


(ii) Prove that f ′(0) = 0. is a continuous function, is differentiable at x = a then find g(a).

Solution: Solution: We have


1


Since f is continuous at 0 and lim    0 , f (a  h )  f (a ) hg (a  h )  0
(i)
n   n 
f ′(a+)= lim  lim = g(a).

h 0 h h 0 h
1 [∵ g is continuous at x = a we have
we must have f (0)  lim f    0 ,

n  n   lim g (a  h )  lim g (a  h ) = g(a)]

h 0 h 0
1 f (a )  f (a  h ) 0  hg (a  h )
since we are given that f    0 for all n ∈ N. f ′(a–) = lim  lim = –g(a)
n h 0 h h 0 h
(ii) By part (i) we have f(0) = 0 and so Since, f is differentiable at x = a, therefore we have

 f ( x ) − f (0)   f (x)  g(a) = – g(a)
f ′(0) = lim   = lim  

x →0  x−0  x   x  ⇒ g(a) = 0.


FREE BOOKS FOR JEE & NEET =>(@iitjeeadv)
Differentiability 3.51

2 2  3  1  2  1
 3 (a  b ), 0  x  b
2
( x  x ) cos  2    3   sin 2 (3x  4 x ) x  0
4 2 3
 =   x  x  x
2 4 2b3 
Problem 11: If f(x) =  a 2  x 2  , b  x  a, 0 if x  0


3 9 9x
 2 3 and lim f ( x )  (0)  sin 1(3  4)   sin 1 .
 9 x (a  b ), x  a
3
x1

Thus, f ′(x) is continuous in [0, 1] and hence it is also bounded.
then find whether f ′(a) exists.

Solution: We have,  x  a if x  0
Problem 13: f (x) =  and


 x  1 | if x  0





 0, 0  x  b
  x  1 if x  0
 8 2b3 g (x) = 
f ′(x) =  x  2 , b  x  a , ( x  1)  b if x  0
2



 9 9x
where a and b are non negative real numbers.
 2 3
 2 (a  b ), x  a
3
Determine the composite function gof. If (gof) (x) is continuous
 9 x for all real x, determine the values of a and b. Further, for
 2 (a 3  b3 )  these values of a and b, is gof differentiable at x = 0 ? Justify
∴ f ′(a+) = lim f '  a  h  = lim  ·  your answer.
h  0  9 (a  h ) 2 

h 0  
 x  a if x  0
2a 3  2b3 Solution: f(x) =  and
 x  1 | if x  0




=
.
9a 2

ps
x  1
 if x  0
and f ′(a–) = lim f '(a  h ) g(x) = 
( x  1)  b if x  0
2

el

h 0

eh

 8 2b3  (0, a)
  f(x)
= hlim (a  h ) 
0  9 9 a  h  
2

je

  x+a (0, 1)
x–1
iit

−8a 2b3 8a 3  2b3 1–x


= + = . (–a, 0)
@

9a 2 9a 2

9 O 1 X
∴ f ′(a+) ≠ f ′ (a–) and hence f ′(a) does not exist.
Problem 12: Let f : [0, 1] → R be defined as f ( x )  1
 f (x)  0
G(x) = g(f(x)) = 


(f ( x )  1)  b f ( x )  0
2
 3  1  
 x (1  x ) sin  2  if 0  x  1
f(x) =  x  . x  a  1 if f ( x )  0 or x  a


0 if x  0  ( x  a  1)  b if f ( x )  0 or  a  x  0
2


 (1  x  1)  b if f ( x )  0 or 0  x  1
2
Then prove that (i) f(x) is differentiable in [0, 1]
(i) f(x) is bounded in [0, 1] =    x b
2



(ii) f ′(x) is bounded in [0, 1].  ( x  1  1) 2  b if f ( x )  0 or x  1


1 
   ( x  2)  b
2
h 3 (1  h ) sin 2  0
Solution: f ′(0+) = lim h =0 (gof) (x) is continuous :


h0 h
Continuity at x = – a
1
(1  h )3 ( h ) sin 0 lim G (a  h ) = lim (a  h )  a  1 = 1
(1  h ) 2

h0 h0
f ′(1 ) = lim

h lim G (a  h ) = lim (a  h  a  1) 2  b

h0


h0 h0
1
= lim  (1  h )3 sin = – sin 1 =b+1
(1  h ) 2




h0
⇒ b + 1 = 1 ⇒ b = 0.

Hence f is derivable in [0, 1]. Continuity at x = 0
Obviously f is continuous in a closed interval [0, 1] and hence lim G (0  h ) = (a – 1)2 (as b = 0)


h0
f is bounded.
Now f ′(x) lim G (0  h ) = 0 ⇒ a – 1 = 0 ⇒ a = 1


h0

FREE BOOKS FOR JEE & NEET =>(@iitjeeadv)


3.52  
Differential Calculus for JEE Main and Advanced

x  2 x  1 ⇒ 5 = 5(f(1) – f(1) – 5 + 2 [∵ f(2) = 5 ]





∴ G (x) =  x 2 1  x  1 ⇒ f(1) = 2.


 ( x  2) x 1
2
Now (2) reduces to
Y  h
f 1    f (1)
f (x)  1 f (x)  1
. lim 
x
f ′(x) =  .f '(1)
h


x h 0 x
X x
To find f(x), write the above equation as
⇒ G is differentiable at x = 0.

df dx
Problem 14: Let f : R → R be a real valued function ∀  f '(1)
f 1 x


x, y ∈ R such that |f(x) – f(y)| < sin4(x – y). Prove that f(x) is


Integrating both sides w.r.t. x
a constant function.
we get ln |f–1| = f ′(1).ln|x| + lnc.
Solution: Since |f(x) – f(y)| < sin4(x – y) , Let f ′(1) = a


x ≠ y|f(x) – f(y)| < | sin4(x – y) | ln|f–1| = a.ln|x|c


Dividing by | x – y | on both sides, Now, using the condition f(1) = 2, we have
f ( x ) − f ( y) sin 4 ( x − y) ln |2 – 1| = a.ln c ⇒ c = 1
∴ <
x−y x−y

Now, f – 1 = ± |x|a
Taking lim y → x, we get using domination law of limits that ⇒ f(x) = 1 ± |x|a
f ( x ) − f ( y) sin 4 ( x − y)
lim ≤ lim
ps
Using the condition f(2) = 5, we have
y→ x x−y y→ x x−y 5 = 1 ± 2a
el


f ( x ) − f ( y) sin 4 ( x − y)
This gives a = 2, using + sign.
eh

⇒ lim ≤ lim Hence, we have f(x) = 1 + |x|2


y→ x x−y y→ x x−y

which can also be written as f(x) = 1 + x2.
je

⇒ |f ′(x)| ≤ 0
 x  x 2  x 3  ....  x n 

⇒ |f ′(x)| = 0 ( |f ′(x)| ≥ 0, in general)
iit

Problem 16: Let f  1 




 


∴ f ′(x) = 0 n
@

⇒ f(x) = c (constant)  f ( x1 )  f ( x 2 )  f ( x 3 )  ....  f ( x n ) 

=  
Problem 15: A function f : R → [1, ∞) satisfies the  n 



equation f(xy) = f(x)f(y) – f(x) – f(y) + 2. If f is differentiable
where xi are any real numbers and n ∈ N. If f(x) is differentiable
on R and f(2) = 5, then show that
and f ′(0) = a and f(0) = b, find f(x).
f ( x ) −1
f ′(x) = .f '(1) . Hence, determine f(x).  x  y  f ( x )  f ( y)


x Solution: We have, f 
 2 



2
Solution: We have


f(xy) = f(x)f(y) – f(x) – f(y) + 2 ...(1) {taking x1 = x, x2 = y, n = 2}

 
This holds for any real x, y. So y is independent of x, i.e.,
h 
f  x 1     f ( x ) dy
f (x  h)  f (x) =0
= lim  
x 
Now, f ′(x) = lim dx .
h 0 h h 0 h
Differentiating w.r.t x,
 h  h
f ( x )f  1    f ( x )  f  1    2  f ( x ) xy 1 dy  1  dy 
 x  x f   ·  1  dx   2 f ( x )  f ( y) dx 
= lim  2  2   
h 0

h

1 xy 1
[using (1)] ∴ f  f ( x )
2  2 


2
 h
f 1    2 Taking x = 0 and x in place of y we get
f (x)  1
= lim 
x
. ...(2) 1  0x  1
h 0 h


x  f '(0)
2  2  2
f'
x

Putting x = 1 and y = 2 in equation (1), we have x
∴ f '    f '(0)  a (given ).
2

f(2) = f(1) f(2) – f(1) – f(2) + 2

FREE BOOKS FOR JEE & NEET =>(@iitjeeadv)


Differentiability 3.53

x Problem 18: Let f be a function such that


Integrating w.r.t. x , f   = a . x + c



2 2 2 f(x + f(y)) = f(f(x)) + f(y) ∀ x, y ∈ R



When x = 0, f(0) = 0 + c, i.e., b = c and f(h) = h for 0 < h < ε where ε is a small positive quantity.


[ f(0) = b (given)] Determine f”(x) and f(x).

x ax Solution: Given f(x + f(y)) = f(f(x) + f(y))...(1)
+b



Thus, f   = ...(1) Putting x = y = 0 in (1),
2

2
f(0 + f(0))=f(f(0)) + f(0) ⇒ f(f(0))=f(f(0)) + f(0)
 x  y  f ( x )  f ( y)


Again, putting y = 0 in f   we get ∴ f(0) = 0 ...(2)
 2 



2 f (x  h)  f (x)
Now f ′(x) = lim (for 0 < h < ε)
x f ( x )  f (0) f ( x )  b h 0 h
f  
2 = 2 2 putting x as h and y as x in (1)



f (x) + b ax f (f (h )) f (h )
∴ From (1), = +b = lim
h →0
= lim
h →0 h
( f(h) = h)



h

2 2
∴ f(x) = ax + b. h
= lim = 1.

h →0 h


Problem 17: Find a function continuous and derivable
Integrating both sides with limits 0 to x


for all x and satisfying the functional relation, f (x + y) .
f (x − y) = f 2 (x) , where x and y are independent variables f(x) = x



∴ f ′ ′(x) = 0.

and f (0) ≠ 0 .

Problem 19: Let f(x) be a thrice differentiable function
Solution: Put y = x and x = 0 to get
ps




f (x) . f (− x) = f 2 (0) ...(1) satisfying f(x + y) = f(x – y) + y[f ′(x + y) + f ′(x – y)]
el
where f(0) = 0, f ′(0) = 1, and f(1) = 2. Find f(x).




f (x  h)  f (x)
Now f ′ (x) = lim
eh

h→0 h Solution: Put y = x




f(2x) = f(0) + x[f ′(2x) + f ′(0)]
je

f 2 (0)

f (x  h)  f(2x) = xf ′(2x) + xf ′(0) + f(0)
f ( x )
iit

= lim Differentiate both sides w.r.t. x
h→0 h

f ′(2x). 2 = f ′(2x) + xf ′′(2x).2 + f ′(0)
@


1 f ( x  h ) . f (  x )  f 2 ( 0) f ′(2x) = 2xf ′′(2x) + f ′(0)

= . lim
f (− x ) h → 0 h Replace 2x by x,

f ′(x) = xf ′ ′(x) + f ′(0)
h h   h  h 

f     x   . f     x    f 2 ( 0) Again differentiate both sides w.r.t. x
1 2 2   2  2 
=
f (− x )
lim
h f ′′(x) = xf ′′′(x) + f ′′(x)
h→0


∴ xf ′′(x) = 0
h

f 2    f 2 ( 0) ⇒ f ′′′(x) = 0

=
1 2 ∴ f(x) = ax2 + bx + c
lim
f (− x )


h→0 h Since f(0) = 0, c = 0
 h  h  f ′(0) = 1 ⇒ b = 1 and f(1) = 2 ⇒ a = 1.
f  2   f (0)  f  2   f (0) 

1        Hence f(x) = x2 + x.
= lim
2 f (− x ) h → 0 h Problem 20: Let f(x) be a positive differentiable



2 function satisfying
f (x) f ′ ( 0)
= . 2 f (0) . f ′ (0) = f(x) xy
f   f ( x ).f ( y) ∀ x , y ∈ R, where f ′(0) = 2.
2
f ( 0)

2 f (0)
 2 
f ′ (x) f ′ ( 0) Find f(x).
⇒ = = k (say)

f (x) f ( 0) Solution: Put x = y = 0, f(0) = f (0) 2


⇒ ln | f (x)| = k x + c ⇒ f(0) = 0, 1


⇒ | f (x)| = ekx + c = ekx ·e c Since f is a positive function f(0) = 1.
f  x  h   f (x)

⇒ f(x) = ± ekx ·e c
f ′(x) = lim

⇒ h 0

f(x) = a.ekx where a and k are arbitrary constants. h

FREE BOOKS FOR JEE & NEET =>(@iitjeeadv)
3.54  
Differential Calculus for JEE Main and Advanced

f (2 x ).f (2h ) − f ( x ) (f (2h )  1)


= f(x). lim
= h.( f (2h )  1)


h 0
h

Replace x by 2x and y = 0 in the rule : f ( 2h )  1
= f(x). lim
f(x) = f (2 x ).f (0)


h 0 2h

f ′(x) = f(x) . f ′(0)
f ( 2h )


f (x)  f (x) f ′(x) = 2f(x)
f ( 0)


f ′(x) = lim n f(x) = 2x + c


h 0

h f(x) = e2x + c


f ( x )( f (2h )  1) f(0) = 1 ⇒ c = 0
= lim


h 0 ∴ f(x) = e2x

h


Things to Remember

1. Right Hand Derivative (R.H.D.) f (a − h ) − f (a )



f (a  h )  f (a ) L.H.D. = f ′(a–) = lim exists. In such a case

= f '(a )  lim −h

h→0
h 0 f ′(a) = f ′(a–).

h ps
Left Hand Derivative (L.H.D.) Similarly, if a function f is defined only in the right


neighbourhood of a point x = a (for x ≥ a), it is said to be
el
 f (a  h )  f (a )
= f '(a )  lim differentiable at a if
eh

h
h 0

f (a  h )  f (a )
f is differentiable at x = a if and only if R.H.D. = f ′(a+) = lim
je

exists.
h


h→0
L.H.D. = R.H.D.
iit

In such a case f ′(a) = f ′(a+).



2. A function will fail to have a derivative at a point where

9. A function f(x) is said to be differentiable in an open

@

the graph has



(i) a corner, where the one-sided derivatives differ. interval (a, b) if it is differentiable at all interior points


(ii) an oscillation point, where the one-sided derivative(s) in (a, b).


does (do) not exist. 10. A function f(x) is said to be differentiable in a closed

(iii) a vertical tangent, where the absolute value of slope interval [a, b] if


of the secant approaches ∞. (i) it is differentiable at all interior points in (a, b).


(iv) a discontinuity. (ii) R.H.D. = f ′(a+) exists at the left endpoint a.




3. The curve y = f(x) has a vertical tangent line at the (ii) L.H.D. = f ′(a–) exists at the right endpoint b.



point (x0, f(x0)) provided that f is continuous at x0 and 11. (i) All polynomial, exponential, logarithmic and


|f ′(x)| → ∞ as x → x0 trigonometric functions (inverse trigonometric not
4. The graph of a continuous function f has a cusp at x0 if included) are differentiable at each point in their

f ′(x) approaches ∞ from one side and –∞ from the other domain.
side. (ii) Modulus function and signum function are non


5. If a function f is differentiable at x = a then it must be differentiable at x = 0. Hence, y = | f(x) | and

continuous at x = a. y = sgn(f(x)) should be checked at points where
If f is continuous at x = a, then f may or may not be f(x) = 0.
(iii) Power function y = xp, 0 < p < 1 is non-differentiable

differentiable at x = a.


6. If a function f is discontinuous at x = a then it is non- at x = 0. Hence, y = (f(x))p should be checked at points

differentiable at x = a. where f(x) = 0.
(iv) The inverse trigonometric functions
7. If a function f is non-differentiable at x = a but both the


y = sin–1x, cos–1x, cosec–1x, and sec–1x

one-sided derivatives exist (though being unequal), then

are not differentiable at the points x = ± 1. Hence,

f is continuous at x = a. y = sin–1(f(x)), cos–1(f(x)), cosec–1(f(x)), and sec–1(f(x))
8. If a function f is defined only in the left neighbourhood should be checked at points where f(x) = ± 1.

of a point x = a (for x ≤ a), it is said to be differentiable at (v) Greatest integer function and fractional part


a if functions are non differentiable at all integral x.

FREE BOOKS FOR JEE & NEET =>(@iitjeeadv)


Differentiability 3.55

Hence, y = [f(x)] and y = {f(x)} should be checked at (c) If lim f ′(x) and lim f ′(x) are infinite, then f(x)
x a  x a 



points where f(x) = n, n ∈ I.
has an infinite derivative at x = a and hence it is
(vi) Further, a function should be checked at all those
non-differentiable there and f ′(x) is discontinuous


points where discontinuity may arise.
at x = a.
12. An alternative limit form of the derivative (ii) If any of the limits lim f ′(x) or lim f ′(x) does

f ( x ) − f (a )



x a  x a 
f ′(a) = lim .
x−a not exist, then we cannot conclude anything about

x →a

f (a  h )  f (a  h ) the differentiability of the function. In such a case,


13. If f ′(a) exists, then lim = f ′(a). we should try to find the derivative using its basic
h 0

2h
definition (i.e. first principles).
14. (i) If f ′(a) exists and ψ(h) → 0 as h → 0, then


f (a   (h ))  f (a )
18. A function f(x) is twice differentiable at x = a if its


lim = f ′(a) derivative f ′(x) is differentiable at x = a i.e.
(h )

h 0
f (a  h )  f (a )
(ii) If f ′(a) exists, and ψ(h) → 0 and φ(h) → 0 as h → 0, the limit f"(a) = lim exists.


h 0 h


f (a   (h ))  f (a  (h )) 19. If f(x) and g(x) are differentiable at x = a, then the following
then lim = f ′(a)


h 0  (h )  (h ) functions are also differentiable at x = a.
15. Let the function y = f(x) be defined by (i) cf(x) is differentiable at x = a, where c is any constant.



x= x(t) and y= y(t), where t is the parameter.

(ii) f(x) ± g(x) is differentiable at x = a.



y ( t  )  y ( t ) (iii) f(x). g(x) is differentiable at x = a.


dy  y ( t  )  y ( t ) (iv) f(x)/g(x) is differentiable at x = a, provided g(a) ≠ 0.
Then,  lim  lim
ps


  x ( t   )  x ( t )   x ( t  )  x ( t )

dx 0 0
20. If f(x) is differentiable at x = a and g(x) is non-differentiable


el
at x = a, then we have the following results :
16. In general, the limit of the derivative may not exist,
eh

(i) Both the functions f(x) + g(x) and f(x) – g(x) are

but when it exists then it is equal to the value of the


derivative. In such a case, we say that the derived function non-differentiable at x = a.
je

f ′(x) is continuous, or the function f(x) is continuously (ii) f(x). g(x) is not necessarily non-differentiable at


x = a. We need to find the result by first principles.
iit

differentiable.
17. (i) If lim f ′(x) and lim f ′(x) both exist or are infinite 21. If f(x) and g(x) both are non-differentiable at x = a, then
@


x a 


x a  we have the following results.
then lim f ′(x) = f ′(a–) (i) The functions f(x) + g(x) and f(x) – g(x) are not
x a 


necessarily non-differentiable at x = a. However,
i.e L.H.L. of f ′(x) = L.H.D. of f(x) at x = a
atmost one of f(x) + g(x) or f(x) – g(x) can be


and lim f ′(x) = f ′(a+)
differentiable at x = a. That is, both of them cannot


x a 
i.e R.H.L. of f ′(x) = R.H.D. of f(x) at x = a. be differentiable simultaneously at x = a.

(a) If lim f ′(x) = lim f ′(x), then f(x) is differentiable (ii) f(x). g(x) and f(x)/g(x) are not necessarily non-


x a  x a  differentiable at x = a. We need to find the result by


and its derivative f ′(x) is continuous at x = a. applying first principles.
(b) If lim f ′(x) ≠ lim f ′(x), then f(x) is non- 22. If f(x) is differentiable at x = a and g(x) is differentiable
x a  x a 



differentiable and its derivative f ′(x) is at x = f(a) then the composite function (gof)(x) is
discontinuous at x = a. differentiable at x = a.

Objective Exercises
Single Correct Answer Type

x  x2
 if x  x0
1. If f(x) = , x ≠ 0 and f(0) = 0 then, 2. Let f(x) = 
1+ e 1/ x
 x  x0




a x b if
(A) f(x) is continuous at x = 0 and f ′(x) = 1
The values of the coefficients a and b for which the function


(B) f(x) is discontinuous at x = 0

is continuous and has a derivative at x0, are


(C) f(x) is continuous at x = 0 and f ′(x) does not exists
(A) a = x0, b = –x0 (B) a = 2x0, b = –x02


(D) f(x) is continuous at x = 0 and f ′(x) = 0




(C) a = a = x0 , b = –x0 (D) a = x0, b = – x02
2






FREE BOOKS FOR JEE & NEET =>(@iitjeeadv)
3.56  
Differential Calculus for JEE Main and Advanced

3. The number of points where function f(x) = minimum 12. Let f (x) is a function continuous for all x ∈ R


{x 3 – 1, –x + 1, sgn (–x)} is continuous but not except at x = 0 such that f ′(x) < 0 ∀ x ∈ (–∞, 0)
differentiable is and f(x) > 0 ∀ x ∈ (0, ∞). If lim f ( x )  3 , lim f ( x )  4


(A) One (B) Two x0  x0





(C) Zero (D) None of these and f (0) = 5, then the image of the point (0,1) about the

 




4. Total number of the points where the function line y lim f (cos3 x  cos 2 x )
x0

f(x) = min {|x| – 1, |x – 2| – 1| is not differentiable
(A) 3 points (B) 4 points lim f (sin 2 x  sin 3 x ) , is
= x x 0






(C) 5 points (D) None of these
 12 9   12 9 




5. The set of values of x for which the function defined as (A)  ,  (B)  , 
 25 25   25 25 






1  x x 1
f (x) = (1  x )(2  x ) 1 x  2  16 8   24 7 
(C)  ,  (D)  , 

 25 25   25 25 




3  x x2
fails to be continuous or differentiable, is 13. Let f be an injective and differentiable function such that



(A) {1} (B) {2} f(x). f(y) + 2 = f(x) + f(y) + f(xy) for all non negative real
x and y with f ′(0) = 0, f ′(1) = 2 ≠ f(0), then




(C) {1, 2} (D) φ
(A) x f ′(x) – 2f(x) + 2 = 0




6. The number of points where


(B) x f ′(x) + 2 f(x) – 2 = 0

f(x) = (x + 1)2/3 + |x – 1|√3, is non-differentiable is


(C) x f ′(x) – f(x) + 1 = 0

(A) 1 (B) 2







(C) 3 (D) none (D) 2 f(x) = f’(x) + 2
ps






7. The set of all points where f(x) = 3 x 2 | x | – |x| – 1 is not 14. Let f be a function such that f(x + y) = f(x) + f(y) for all
el

differentiable is x and y and f(x) = (2x2 + 3x) g(x) for all x where g(x) is
continuous and g(0) = 3. Then f ′(x) is equal to
eh

(A) {0} (B) {–1, 0, 1}







(C) {0, 1} (D) none of these (A) 9 (B) 3
je








8. It f(x) is a differentiable function from R → Q then (C) 6 (D) none




iit

100 15. Let f(x) be a function such that f(x + y) = f(x) + f(y) and
 (1)r f (r) is

f(x) = sin x g(x) for all x, y ∈ R. If g(x) is a continuous
@

r 0
function such that g(0) = K, then f’(x) is equal to
(A) 0 (B) –1 (A) K (B) Kx








(C) 1 (D) none
(C) Kg(x) (D) none








| x |  x (31/ x  1)
9. The function f(x) = , x ≠ 0, f(0) = 0 is  3x 2  2 x  1 1
31/ x  1  2 for x 

 6 x  5 x  1 3
1
(A) discontinuous at x = 0 16. Let f(x) =  1 then f ′  
  3



(B) continuous at x = 0 but not differentiable there  4 for x
 3

(C) both continuous and differentiable at x = 0
(A) is equal to –9 (B) is equal to –27


(D) differentiable but not continuous at x = 0




(C) is equal to 27 (D) does not exist

 1




 g ( x ).cos x if x0 17. Given f(x) is a differentiable function of x, satisfying
10. Let f(x) =  where g(x) is an even

f(x) . f(y) = f(x) + f(y) + f(xy) – 2 and that f(2) = 5. Then

 0 if x0
f(3) is equal to
function differentiable at x = 0, passing through the origin.
Then f ′(0) (A) 10 (B) 24




(A) is equal to 1 (B) is equal to 0 (C) 15 (D) none








(C) is equal to 2 (D) does not exist  2x  1 ,xQ
18. The function f(x) =  x 2  2 x  5 , x  Q is




( x 2  2 x  3  sin  x ) n  1 

11. Let f (x) = lim , then
( x 2  2 x  3  sin  x ) n  1
n 

(A) continuous no where



(A) f(x) is continuous and differentiable for all x ∈ R. (B) differentiable no where




(B) f(x) is continuous but not differentiable for all x ∈ R. (C) continuous but not differentiable exactly at one point


(D) differentiable and continuous only at one point and


(C) f(x) is discontinuous at infinite number of points.




(D) f(x) is discontinuous at finite number of points. discontinuous elsewhere


FREE BOOKS FOR JEE & NEET =>(@iitjeeadv)
Differentiability 3.57

19. Let f(x) be differentiable at x = h then 27. If f(x + y + z) = f(x). f(y) . f(z) for all x, y, z and


( x  h ) f ( x )  2h f ( h ) f(2) = 4, f ′(0) = 3, then f’(2) equals
lim is equal to (A) 12 (B) 9
xh

x →h





(C) 16 (D) 6
(A) f(h) + 2hf ′ (h) (B) 2 f(h) + hf ′ (h)





28. If f(x) = lim (cos x) + cos–1 (sin x); then the points of
2k




(C) hf(h) + 2f ′ (h) (D) hf(h) – 2f ′ (h)


k
non-differentiability of f(x) is




20. If f(x) . f(y) = f(x) + f(y) + f(xy) – 2 ∀ x, y ∈ R and if f(x)
(A) x = 1 (B) x = – 1

is not a constant function, then the value of f(1) is





(C) x = nπ(n ∈ 1) (D) none of these
(A) 1 (B) 2









(C) 0 (D) – 1 29. If f(x) = Max. {1, (cos x + sin x),


(sin x – cos x)} 0 ≤ x ≤ 5π/4, then




21. If f(x) = |1 – x|, then the points where sin–1(f |x|) is non-


(A) f(x) is not differentiable at x = π/6

differentiable are



(B) f(x) is not differentiable at x = 5π/6



(A) {0, 1} (B) {0, –1} (C) f(x) is continuous for x ∈ [0, 5π/4]







(C) {0, 1, –1} (D) none of these (D) None of these






22. Let f(x) be defined for all x ∈ R and the continuous. 30. Let f : R → R satisfying |f(x)| ≤ x2 ∀ x ∈ R, then


Let f(x + y) – f(x– y) = 4xy ∀ x, y = ∈ R and (A) ‘f’ is continuous but non-differentiable at x = 0


f(0) = 0 then
(B) ‘f’ is discontinuous at x = 0


(A) f(x)is bounded (C) ‘f’ is differentiable at x = 0





1  1 (D) None of these


(B) f(x) + f    f  x    2 31. Choose the incorrect statement given that f is differentiable
ps
   


x x

(A) If f is odd and f ′(c) = 3 then f ′(–c) = 3


1  1 (B) If f is even and f ′(c) = 3 then f ′(–c) – 3
el
(C) f ( x )  f    f  x    2


    (C) If f is even then f ′(0) = 0


x x
eh


(D) none of these (D) If f is odd then f ′(0) = 0


je


32. Consider the function f(x)
23. If g′(x) exists for all x, g′ (0) = 2 and g(x + y) = eyg(x) +

iit

exg(y) ∀ x, y. Then x3 if x  0
(A) g(2x) = –2exg(x) (B) g’(x) = g(x) + 2ex  2
@

x if 0  x  1




(C) lim g(h)/h = 3 (D) None of these =  ,
2x  1 if 1  x  2





h→0
 2
24. If y = |1–|2–|3–|4–x|||| ; then number of points where y is  x  2 x  3 if x  2

not differentiable; is then f is continuous and differentiable for

(A) 1 (B) 3 (A) x ∈ R (B) x ∈ R – {0, 2}








(C) 5 (D) > 5 (C) x ∈ R – {2} (D) x ∈ R – {1, 2}









x
 2 , x0 33. The number of points on [0, 2] where
25. f(x) =  2 x  | x |

then f(x) is
1  x{x}  1 0  x  1

 , x0
f(x) = 
(A) Continuous but non-differentiable at x = 0 2  {x} 1  x  2



(B) Differentiable at x = 0 fails to be continuous or derivable is



(C) Discontinuous at x = 0 (A) 0 (B) 1






(D) None of these (C) 2 (D) 3






26. Let f(x) = cos x and g(x) = 34. Which one of the following functions best represent the


minimum {f ( t ) : 0 ≤ t ≤ x} , x ∈[0, π] graph as shown adjacent ?
g(x) = sin x − 1 , x>π
Y


then (0, 1)

(A) g(x) is discontinuous at x = π


(B) g(x) is continuous for x ∈ [0, ∞)


(C) g(x) is differentiable at x = π


(D) g(x) is differentiable for x [0, ∞) 0 X


FREE BOOKS FOR JEE & NEET =>(@iitjeeadv)
3.58  
Differential Calculus for JEE Main and Advanced

1 1 42. Let f be a differentiable function on the open interval (a, b).


(A) f(x) = (B) f(x) = Which of the following statements must be true?
1+ x 2 1+ | x |





–| x |
I. f is continuous on the closed interval [a, b]
(D) f(x) = a|x| (a > 1)



(C) f(x) = e




II. f is bounded on the open interval (a, b)



35. The number of points where the function
III. If a < a1 < b1 < b, and f(a1) < 0 < f(b1), then there is a

f(x) = (x2 – 1) | x2 – x – 2| + sin (| x |)



number c such that a1 < c < b1 and f(c) = 0

is not differentiable is

(A) 0 (B) 1 (A) I and II only (B) I and III only









(C) 2 (D) 3 (C) II and III only (D) only III









36. Let f(x) be differentiable at x = h then 43. Let f : R → R, f(x – f(y)) = f(f(y)) + xf(y) + f(x) – 1 ∀ x,


( x  h ) f ( x )  2h f ( h ) y ∈ R, if f(0) = 1 and f ′(0) = 0, then
lim is equal to
xh

x →h
x2
(A) f(h) + 2hf ′(h) (B) 2 f(h) + hf ′(h) (A) f(x) = 1 –
2
(B) f(x) = x2 + 1









(C) hf(h) + 2f ′(h) (D) hf(h) – 2f ′(h)
 2x  1 




37. The function f(x) = maximum x (2  x ), 2  x is non-   (C) f(x) =  
 x 1 
(D) none of these






differentiable at x equal to 44. If f : [–2a, 2a] → R is an odd function such that

(A) 1 (B) 0, 2
f(x) = f(2a – x) for x ∈ (a, 2a). if the left hand derivative




(C) 0, 1 (D) 1, 2 of f(x) at x = a is zero, then the left hand derivative of




ps
 sin | x 2  5x  6 | f(x) at x = –a is
 , x  2, 3
el
38. Let f(x) =  x 2  5x  6 . (A) 1 (B) –1






eh

 1 , x  2 or 3 (C) 0 (D) none






The set of all points where f is differentiable is 45. Suppose that the differentiable functions u, v, f,
je


(A) (∞, ∞) (B) (–∞, ∞) – {2} g : R → R satisfy lim u(x) = 2, lim v(x) = 3,
iit





(C) (–∞, ∞) – {3} (D) (–∞, ∞) – {2, 3}
x x
f ( x )
@

f (x)




lim f(x) = lim g(x) = ∞ and  u(x) = v(x)
1 f (a  2 h 2 )  f (a  2 h 2 ) x x g( x ) g( x )
39. If f ′(a) = , then lim =
h  0 f (a  h 3  h 2 )  f (a  h 3  h 2 )

4 f (x)
then lim is equal to (given that it exists)
x  g( x )
(A) 0 (B) 1




(C) –2 (D) none (A) 1 (B) 1/2








[ x ]  {x} x 1 (C) 2 (D) None




 46. Suppose f is a differentiable function such that
40. If f(x) =  1 , then [where [ . ] and

 x 1 f(x + y) = f (x) + f (y) + 5xy for all x, y and f ′(0) = 3. The

 [ x ]  {x}
2
minimum value of f (x) is
{ . } represents greatest integer part and fractional part (A) – 1 (B) – 9/10





respectively.]
(C) – 9/25 (D) None
(A) f(x) is continuous at x = 1 but not differentiable






(B) f(x) is not continuous at x = 1 47. Let h(x) be differentiable for all x and let f(x) = (k x + ex)



(C) f(x) is differentiable at x = 1 h(x) where k is some constant. If h(0) = 5, h’(0) = –2 and
f’(0) = 18 then the value of k is equal to



(D) lim f(x) does not exist


x→1
(A) 5 (B) 4




41. If f(x) has isolated point discontinuity at x = a such that (C) 3 (D) 2.2





|f(x)| is continuous at x = a then
(A) |f(x)| must be differentiable at x = a 48. If for a function f(x) : f(2) = 3, f ′(2) = 4, then lim [f(x)],




x→2
(B) lim f(x) does not exist
where [ . ] denotes the greatest integer function, is


x →a
(C) lim f(x) + f(a) = 0 (A) 2 (B) 3


x →a





(D) f(a) = 0 (C) 4 (D) dne






FREE BOOKS FOR JEE & NEET =>(@iitjeeadv)
Differentiability 3.59

[ x ]  {x} (C) f(x) is differentiable at x = 1


x 1




 (D) lim f(x) does not exits
49. If f(x) =  1 , then



x→1
x 1


 [ x ]  {x}
2
50. Let f ″(x) be continuous at x = 0 and f″(0) = 4 then value


[where [ . ] and { . } represent greatest integer and 2f ( x )  3f (2 x )  f (4 x )
of lim is

fractional part functions respectively] x 0 x2
(A) f(x) = is continuous at x = 1 but not differentiable (A) 11 (B) 2







(B) f(x) is not continuous at x = 1 (C) 12 (D) none







Multiple Correct Answer Type for JEE Advanced

51. Let f (x)=x − 1( [x] − [−x]) , then which of the 56. If a function f : R → R satisfies


following statement(s) is/are correct? |Σ 3k (f(x + ky) – f(x – ky))| ≤ 1 for every n ∈ N and for


(where [x] denotes greatest integer function) all x y ∈ R, then

(A) f (x) is continuous at x = 1 (A) |3n (f(x + ny) – f(x – ny))| ≤ 2




(B) f (x) is derivable at x = 1 2


(C) f (x) is non-derivable at x = 1 (B) |f(u) – f(v)| ≤ n
3




(D) f (x) is discontinuous at x = 1. (C) f(x) is a constant function




(D) f ′(x) = 1 ∀ x ∈ R
52. Which of the following function will have tangent at
ps



indicated point 57. If we define new derivative of function f to be
el
 2 1 f 3 ( x  x )  f 3 ( x )
 x sin , x0 ∆(f(x)) = lim
eh

, then
(A) f(x) =  x

x at x = 0 x 0
 0




, x0 (A) ∆ (x) = 3x3
je

π



iit

(B) f(x) = sgn (sin x) at x = π (B) ∆ (tan x) at x = equals 6




4



(C) ∆ (sin x) at x = 0 equals 1
@

(C) f(x) = sgn (x2) at x = 0









(D) f(x) = |x – 1|11/10 at x = 1 3π2
(D) ∆ (cos–1 x) at x = 0 equal –



53. Let f (x) = sin πx, g (x) = sgn (x) and h (x) = gof(x) then


4

(A) h(x) is discontinuous at infinite number of points  1 
58. if f(x) = minimum  cos x , ,{sin x}  , 0 ≤ x ≤ 2 π ,


(B) h′(x) = 0 for all x ∈ R – I
 

2


(C) lim h ( x ) does not exist where { . } represents fractional part function, then f(x) is
x →I



(D) h(x) is periodic with period 1. differentiable at
3π π


tan[ x ] (A) (B)




54. If f(x) = , where [.] denotes the 2 4
[1 | log(sin 2 x  1) |] π π

(C) (D)




greatest integer function then f(x) is 3 2
(A) discontinuous ∀ x ∈ Ι 59. If f(x) = minimum {x3 – 1, – x + 1, sgn (–x)}, then f(x) is



(B) continuous ∀ x ∈ R (A) continuous at x = 0


(B) differentiable at x = 0


(C) non differentiable ∀ x ∈ Ι



(D) a periodic function with no fundamental period (C) continuous at x = 2


(D) differentiable at x = 2

55. Let f and g be two functions defined as follows :



60. If [ . ] denotes greatest integer function, then for the
x+ x  x for x  0

f(x) = for all x & g(x) =  2 then function y = sin π (|x| + [x]), which of the following is
 x for x  0

2
correct
(A) (gof)(x) & (fog)(x) are both continuous for all x ∈ R (A) not continuous at x = –1/2




(B) (gof)(x) & (fog)(x) are unequal functions (B) continuous at x = 0




(C) (gof) is differentiable at x = 0 (C) differentiable in [–1, 0)






(D) (fog)(x) is not differentiable at x = 0 . (D) differentiable in (0,1 ]




FREE BOOKS FOR JEE & NEET =>(@iitjeeadv)
3.60  
Differential Calculus for JEE Main and Advanced

61. If f(x) is differentiable everywhere and f(a) = 0, then at 67. Given that f′(x) is continuous at x = a. State which of the


x = a, f′(a) is non-zero following statements are true
(A) | f | is differentiable f ( h ) − f (a )
(A) f ′(a) = lim



(B) | f |2 is differentiable h−a



h →0


(C) f | f | is differentiable f (a ) − f (a − h )
(B) f ′(a) = lim



(D) f + | f |2 is differentiable



h →0 h


1 f (a + 2 t ) − f (a )
62. Let f (x) = x 2 sin for 0 < x ≤ 1 and f (0) = 0. If (C) f ′(a) = lim



x t →0 t
g(x) = x2 for x ∈ [0, 1] then which of the following f (a + 2 t ) − f (a + t )
(D) f ′(a) = lim

statement(s) is/are correct?



t →0 2t
(A) f (x) is differentiable in [0, 1]
 2


1
(B) lim
f (x)
does not exist. sin x.cos , x  0
68. Let f ( x )   x


x→0 g ( x )


 0, x0
f (x)
(C) lim exist
(A) f is continuous at x = 0



x→0 g( x )



(B) f ′(0) = 1


f '( x )
(D) lim does not exist. (C) f ′(0) does not exist




x→0 g '( x ) (D) f ′ is not continuous at x = 0.


 x 1 , x  1 ea / x  e  a / x
ps
63. If f(x) =  and g(x) = f(x) + f(|x|), then 69. Let f(x) = g′(x) where g′ is the derivative
2  x , x  1 ea / x  e  a / x


el
g(x) is of g and is a continuous function and a > 0 then lim f(x)
eh

x→0
(A) not continuous at x = –1



exists if
je

(B) not continuous at x = 1




(C) continuous at x = 0 (A) g(x) is polynomial
iit





(D) not differentiable at x = 0 (B) g(x) = x


@


64. A function f(x) satisfies the relation (C) g(x) = x2



(D) g(x) = x3 h(x) where h(x) is a polynomial
f(x + y) = f(x) + f(y) + xy (x + y) ∀ x, y ∈ R.


70. Let f : R → R be a function such that,

If f ′(0) = –1, then


(A) f(x) is a polynomial function x  y f ( x ) + f ( y)
f  = , f(0)=3 and f ′(0)=3, then


(B) f(x) is an exponential function  

3 3


(C) f(x) is twice differentiable for all x ∈ R f (x)


(D) f ′(3) = 8 (A) is differentiable in R


x


65. Function f(x) = max (|tan x|, cos|x|) is
(B) f ′ (x) = 3

(A) non-differentiable at 4 points in (–π, π)


(C) f(x) is continuous in R


(B) discontinuous at 2 points in (–π, π)



(D) f(x) is bounded in R




(C) non-differentiable at only 2 points in (–π, π)
Assertion (A) and Reason (R)


(D) in (–π, π) there are only 2 points where f(x) is


continuous but not differentiable (A) Both A and R are true and R is the correct explanation

66. Let a differentiable function f(x) be such that of A.

1 1 (B) Both A and R are true but R is not the correct explanation
|f(y) – f(y)| ≤ |x – y| ∀ x, y ∈R and f ′(x) ≥ .

of A.

2 2
Then the number of points of intersection of the graph of (C) A is true, R is false.


y = f(x) with (D) A is false, R is true.

(A) the line y = x is one 71. Consider the function f(x) = x2 – 2x and g(x) = –|x|




(B) the curve y = –x3 is one Assertion (A) : The composite function



(C) the curve 2y = |x| is three F(x) = f(g(x)) is not derivable at x = 0.




(D) None of these Reason (R) : F ′ (0+) = 2 and F′(0–) = –2.



FREE BOOKS FOR JEE & NEET =>(@iitjeeadv)
Differentiability 3.61

72. Assertion (A) : There is no polynomial function f such 80. Assertion (A) :



that f(x + y) = f(x) + yf(f(x)) for all x, y ∈ R
f(x) = ( x  1) 2 p 1 ( x  2) 2 p 1 2
1
....( x  n ) 2 p n
1
is


f (x  h)  f (x)
Reason (R) : f ′(x) = lim = f(f(x)). differentiable everywhere where p1, p2, ......pn ∈N.

h 0 h


Reason (R) : f(x) = x|x| is differentiable everywhere.
If f is of degree n then the equation n – 1 = n2 has no



positive integral solution. Comprehension – 1
73. Assertion (A) : There are exactly three functions
e{x }  1, x  0
2

f : R → R such that 
 sin x  tan x  cos x  1
f(x) f(y) = f(x) + f(y) – f(xy) ...(1) Let f(x) =  2 , x0



1 + f(x + y) = f(xy) + f(x) f(y) ...(2)  2 x  ln( 2  x )  tan x



Reason (R) : Adding (1) and (2), we get 0, x0

1 + f(x + y) = f(x) + f(y). where { } represents fractional part function. Lines L1 and


f (x  h)  f (x) f (h )  1 L2 represent tangent and normal to curve y = f (x) at x = 0.
Now f ′(x) = lim  lim = f ′(0).
h 0 h h  0 h Consider the family of circles touching both the lines L1
⇒ f(x) = ax + b where a & b can be determined by using and L2.


function’s value at x = 0, 1. 81. Ratio of radii of two circles belonging to this family cutting
74. Assertion (A) : The function y = sin–1 (cos x) is not

each other orthogonally is

differentiable at x = nπ , n ∈ I is particular at = π.
dy − sin x (A) 2 + 3 (B) 3
Reason (R) :




= so the function is not
| sin x |

dx
ps (C) 2 + 2 (D) 2 – 2
differentiable at the points where sin x = 0.





82. A circle having radius unity is inscribed in the triangle
el
75. Consider two functions


f(x) = sin x and g(x) = | f(x)|. formed by L1 and L2 and a tangent to it. Then the minimum
eh

Assertion (A) : The function h(x) = f(x) g(x) is not area of the triangle possible is

je

differentiable in [0, 2π]. (A) 3 + 2 (B) 3 – 2






Reason (R) : f (x) is differentiable and g (x) is not
iit

(C) 3 + 2 2 (D) 3 – 2 2

differentiable in [0, 2π]




@

83. If centres of circles belonging to family having equal radii


76. Assertion (A) : The function

'r' are joined, the area of figure formed is

x|
f (x) = c1e| 3
+ c2 | x | , where c1, c2 are constants (A) 2r2 (B) 4r2





is differentiable at x = 0, provided c1 = 0. (C) 8r2 (D) r2





Reason (R) : e|x| is not differentiable at x = 0.
Comprehension – 2

77. Consider two functions f(x) = sin x and g(x) = | f(x) |.
Let f(x) be a real valued function not identically zero, which

Assertion (A) : The function h (x) = f (x) g(x) is not satisfies the following conditions.

differentiable in [0, 2π] I. f(x + y 2n + 1) = f(x)+ {f(y)}2n + 1, n ∈ N, x, y are any real numbers
Reason (R) : f (x) is differentiable and g (x) is not

II. f ′(0) ≥ 0.

differentiable in [0, 2π]


84. The value of f ′(10) is
 [x]  

78. Consider the function f(x) = cot–1  sgn    where (A) 10 (B) 0
 2x  [ x ]  






(C) 2n + 1 (D) 1




[ ] denotes the greatest integer function
85. The function f(x) is
Assertion (A) : f(x) is discontinuous at x = 1.

(A) odd

Reason (R) : f(x) is non derivable at x = 1.


(B) even




 x 1
 0x2 (C) neither even nor odd
79. Let f(x) = sgn x and g(x) =  then


1  x 2x4
2

 (D) even as well as odd


Assertion (A): The function (fog)(x) is differentiable at 86. The function f(x) is


x = 2. (A) Continuous and differentiable everywhere
Reason (R) : If f(x) is differentiable at x= g(x) is


(B) Continuous everywhere but not differentiable at some points



differentiable at x = a then g(x) is differentiable at x = a (C) Discontinuous at exactly one point


and f(x) is differentiable at x = g(a). (D) Discontinuous at infinitely many points


FREE BOOKS FOR JEE & NEET =>(@iitjeeadv)
3.62  
Differential Calculus for JEE Main and Advanced

Comprehension – 3 91. If lim f ( x )  0 and lim f ( x )  0 and f(x) be continuous


x  x


Suppose f, g and h be three real valued function defined on R.
1 on R. Then
Let f (x) = 2x + |x|, g (x) = (2x – |x| ) and h(x) = f(g(x)) (A) f (x) is bounded on R and attains both maximum and
3



87. The range of the function minimum on R.

(B) f (x) is unbounded on R but attains minimum on R.



k(x) = 1 + (cos–1(h(x)) + cot–1(h(x))) is equal to (C) f (x) is bounded on R and attains either maximum or



minimum on R.
1 7  5 11  (D) f is unbounded.
(A)  ,  (B)  , 



4 4 4 4 





92. f (x) is defined on [0, 1] and


1 5  7 11 
(C)  ,  (D)  ,   x a sin( x  c ) x  0

4 4 4 4  where a, c ∈ R and c > 0




f(x)= 
0
 x0


88. The domain of definition of the function

l(x) = sin–1 (f(x)) – g(x)) is equal to (A) f ′(0) exists if 0 < a < 1




3  (B) f (x) is continuous if a < 0
(A)  ,   (B) (–∞, 1]


8  (C) f ′(x) is bounded and continuous if a > c






(D) f (x) and f ′(x) are continuous if a > 1 + c.
 3


(C) [– 1, 1] (D)   , 
 




8
Comprehension – 5
89. The function T(x) = f(g(f(x))) + g(f(g(x))), is
ps
Let f : R → R be a differential function satisfying

(A) continuous and differentiable in (– ∞, ∞).
 x  y  2  f ( x )  f ( y)
el


(B) continuous but not derivable ∀ x ∈ R. f  for all real x and y and f ′(2) = 2.
 3  3
eh


(C) neither continuous nor derivable ∀ x ∈ R.


93. The range of f(|x|) is
je

(D) an odd function.



(A) [0, ∞) (B) [1, ∞)


iit





Comprehension – 4 (C) [2, ∞) (D) None of these




@

A function f (x) is said to be differentiable at x = c, if


94. If f(x) = |f(|x|) – 3| for all x ∈ R, then for g(x)
f ′(c+) = f ′(c–) = a finite quantity. Also f (x) is said to be bounded

(A) one non-differentiable point
if | f (x) | < n ∀ x for which f (x) is defined (where n is finite)



(B) two non-differentiable point


  (C) three non-differentiable point
90. Let f : R → R and c ∈ R such that lim n  f  c    f (c) 
1



  n (D) four non-differentiable point

n 


= a (a ∈ R) then 95. The number of solutions of the equation
(A) f ′(c) exists and is equal to a

x2 + (f (|x|)2) = 9 are



(B) f ′(c) does not exists (A) 0 (B) 2






(C) f ′(c) exists but is not equal to a (C) 3 (D) 5






(D) f ′(c) may or may not exist.


Match the Columns for JEE Advanced

96. Column-I Column-II




 tan 1 x , | x | 1 2
 x −1
(A) Let f(x) =  x 2  1 , | x | < 1, then f(x) is (P) –1
 , | x | 1




4
 4
not differentiable at x equal to

(B) f(x) = (x2 – 4) |x2 – 5x + 6| + cos |x| is non derivable at x equal to (Q) 1




dy
(C) If sin(x + y) = ex + y – 2, then is equal to (R) 2




dx

FREE BOOKS FOR JEE & NEET =>(@iitjeeadv)


Differentiability 3.63

(D) Let f: R → R is defined by the equation (S) 3






f(x + y) = f(x) f(y) x, y ∈ R, f (0) ≠ 0 and f’(0) = 2

f '( x )
then is equal to (T) None of the above values



f (x)
97. Column-I Column-II


a x  1 if 0  x  3
(A) Let g(x) =  , if g(x) is differentiable on (0, 5) (P) 0
 bx  2 if 3  x  5




then (a + b) equals

(B) Let a and b be real numbers and let f(x) = a sin x + b 3 x + 4, ∀ x ∈ R (Q) 1




If f(log10 (log3 10)) = 5 then the value of f(log10 (log10 3)) is equal to

sin 4 x  sin 4 2 x  sin 4 3x
(C) lim equals (R) 2
sin 3 x  sin 3 2 x  sin 3 3x




x0
1

(D) Number of solution(s) of the equation 2 log10 x


 8  ( x  8) log10 2
is (S) 3




(T) 4


98. Column-I Column-II



(A) If f(xy) = f(x) . f(y) and f is differentiable at x = 1 such that (P) 1




f ′(1) = 1 also f(1) ≠ 0, then f ′(7) equals
ps

(B) If [.] denotes greatest integer function, then number of points (Q) 13
el




at which the function f(x) = |x2 – 3x + 2| + |sin x| – [x – 1/2],

eh

– π ≤ x ≤ π, is non differentiable, is

(C) Let f(x) = [a + 7 sin x], x ∈ (0, π), a ∈ I, [.] denotes greatest integer
je

(R) 5




function. Then number of points at which f(x) is not differentiable is
iit

(D) If for a continuous function, f, f(0) = f(1) = 0, f’(1) = 2 and (S) 3




@

y (x) = f(ex) ef(x), then y’(0) is equal to (T) 2





99. Column-I Column-II


  2 
  2  sin (1  {x} )  sin 1  {x}
1 1

  , x0
 2 ({x}  {x}3 )

(A) If f (x) =  (P) 2
 x0




k,



 A sin 1  {x} cos 1  {x}
1 1

 , x0
 2{x} 1  {x}

 A 
is continuous at x = 0, then the value of sin2k + cos 2   , is


 2
(where {x} denotes fractional part function)

(B) If f (x) = [2 + 5 | n | sin x] where n ∈ I has exactly 19 points (Q) –2





of non-differentiability in (0, π), then possible value(s) of 'n' are


(where [x] denotes greatest integer function)

 1 1
  3 x   3  x
  4  4
  , x0
x 1
(C) Let f (x) =  1 . (R) 3
     3  x





3 x

 4 4

 0, x0

FREE BOOKS FOR JEE & NEET =>(@iitjeeadv)


3.64  
Differential Calculus for JEE Main and Advanced

[ x 1]

If P = f ′(0–) – f ′(0+), then lim


 exp   x  2  ln 4   4
 16
4  16
x

x P

is less than (where [x] denotes greatest integer function)



1
(D) The number of points at which g(x) = 2
is not (S) –3





1+
f (x)
1
differentiable where f(x) = 1 , is (T) 5



1+
x
100. Column-I Column-II


(A) If f : [0, 1] → [0, 1] is continuous, then the number (P) one




of roots of the equation f(x) = x3 is

(B) If f is derivable, then between the consecutive roots of (Q) at least one




f ′(x) = 0, the number of roots of f(x) = 0 is

(C) The number of values of x at which f(x) = |1 – |x – 1|| is (R) at most one




not differentiable

 x, if x is rational
(D) Let f(x) =  (S) more than one
 2  x , if x is irrational




ps
The number of values of x at which f(x) is continuous is

el
eh

Review Exercises for JEE Advanced


je
iit

x 1 1
6. Examine the differentiability of

for x ≠ 0, f(0) = 0. Is the
@

1. Let f(x) =
f(x) = x  2 2x  4 + x − 2 2 x − 4 over its domain.

x

function f(x) continuous and differentiable at x = 0 ? 7. Let f(x) = x3 – 3x and

2. Let g(x) be polynomial of degree atmost two and f(x) be  min(f ( t ) : 0  t  x ) , 0  x  2

continuous at x = 0 and differentiable at x = 1. Find 
g(x) = 2x  5 , 2 x 3.
 5xe1/ x  2 x


( x  2) , x3
2
 , x0
 3  e
1/ x
Draw g(x) and discuss continuity and differentiability of
, 0  x 1.

g(x) if f(x) =  g( x ) g in (0, 4).

 1
( x  1) 2 sin  x , x 1 x 2  a , 0  x  1

8. Let f(x) = 
 (1  x ) 2 2x  b , 1  x  2


 x  1 ,  2  x  0
 3x  b , 0  x  1
 d (f ( x ))
3. Let f(x) =  x 2  x , 0  x  2 and and g(x) =  . If
 x

3
, 1 x  2

d (g ( x ))

2  x , 2  x  3
 exists at x = 1, find the values of a, b and also its value.

g(x) = f(|x|) + |f(x)| check continuity and differentiability
9. If f and g be two functions having the same domain D, if

of g(x) over [–2, 3].

f and g be derivable at x0 ∈ D, and if f(x0) ≠ g(x0), then
4. Check the continuity and differentiability of prove that each of the function max. {f, g} and min. {f, g}

 2 x2 1  is derivable at x0. What happens if f(x0) = g(x0)?
f(x) = sin–1  .
2  f : [0, 1] → [1, 2] be a differentiable function, then

 x  10. (i)


x prove that the number of solution of f(x) – ex(x – 1) –
5. Let f (x) = (x2 – 4) |(x3 – 6x2 + 11x – 6)| + . log2(1 + x) = 0 will be at least one.

1+ | x |
Find the set of points at which the function f(x) is not (ii) Let f : [1, e] → [0, 1] be continuous then prove that



differentiable. f(x) = n x has atleast one solution in [1, e].

FREE BOOKS FOR JEE & NEET =>(@iitjeeadv)


Differentiability 3.65

11. If a differentiable function f(x) satisfies the relation 20. Let f (x) = max. {|x2 – 2 |x| |, |x|} and g (x) = min.


x x {|x2 – 2 |x| |, |x|} then show that f(x) is not differentiable at
f   = x f(x) – yf(y) ∀ x, y ∈ R & f(e) = 1/e, then 5 points and g (x) is non differentiable at 7 points.

y y
log x 21. Consider function f : R – {–1, 1} → R.


prove that f(x) = . x
x f(x) = . Find f ′(x), draw the graph of f′ and


12. Given a real valued function f(x) as follows : 1− | x |
prove that it is not derivable at the origin.

x 2 + 2 cos x − 2


1
f(x) = for x < 0; f(0) = & 22. Suppose the function f satisfies the conditions :


4

x 12 (i) f(x + y) = f(x)f(y) for all x and y.



sin x − n (e x cos x ) (ii) f(x) = 1 + x.g(x) where lim g ( x )  1
x 0



f(x) = for x > 0. Test the continuity and
6x 2 Show that the derivative f ′(x) exists and f ′(x) = f(x) for


differentiability of f(x) at x = 0. all x.
13. Let f be given by 23. If f(x) = |x – 1| . ([x] – [–x]), then find f ′(1+) and


 x 2 sin( x −4 / 3 )
 when x ≤ 0 f ′(1–) where [x] denotes greatest integer function.
f(x) =  .

 when x = 0 24. Let f : R → R be continuous function | f(x) | ≤ x4n, n ∈ N.

0


Show that f has a derivative for all values of x, and that Prove that f(x) is differentiable at x = 0.

f ′(0) = 0. Prove that f ′ is not, however, continuous at x = 0 25. Find the values of

14. Draw the graph of the function f(x) = x – |x – x2|, –1 ≤ x ≤ 1 f (3  h ) 2  f (3  h 2 )
(a) lim if f ′(3) = 2 ;

and discuss its continuity in [–1, 1] and differentiability in 2h 2


h→0
ps
(–1, 1). f (a  2 h 2 )  f (a  2 h 2 ) 1
if f ′(a) =
el
 tan[ x ] 2 (b) lim 2
.


  ax 3  b , 0  x 1 h→0 h 4
eh

15. If f(x) =  ax 2 is
  5e 1/ x
2


1
 2 cos x  tan x , 1 x  2  , x0
je

26. If f(x) =  3  e1/ x Examine f(x), xf(x) and


π 26

1  , x0
differentiable in [0, 2], then a = 
iit

and b = – . 0

k1 4 k2 x2f(x) for continuity and differentiability at x= 0
@


Find k12 + k22 { where [ ] denotes 1  x , (0  x  1)
greatest integer function}. 27. f(x) =  x  2 , (1  x  2) . Discuss the continuity

 a

 1   4  x , (2  x  4)
 x . cos  3  , x  0
16. If f ( x )   x  is continuous but non-
& differentiability of y = f (f(x)) for 0 ≤ x ≤ 4.

 0, x0

 ax ( x  1)  b , x  1
differentiable at x = 0, then all possible values of a. 
28. The function f(x) =  x  1 , 1  x  3 find the values
a1/ x  a 1/ x

 2
17. Examine the function , f (x) = x . ,x≠ 0  px  qx  2 , x  3
a1/ x  a 1/ x

of the constant a, b p and q so that
(a > 0) and f (0) = 0 for continuity and existence of the (i) f(x) is continuous for all x.



derivative at the origin . (ii) f ′(1) does not exist.



18. Let f(x) = x4 – 8x3 + 22x2 – 24 and (iii) f ′(x) is continuous at x = 3.



min f ( t ) ; x ≤ t ≤ x + 1, −1 ≤ x ≤ 1 29. Discuss the continuity on 0 ≤ x ≤ 1 & differentiability at
g(x) =  Discuss

 x − 10 ; x > 1 x = 0 for the function f(x) = x.
the differentiability of g(x) in [–1, 2]. 1  1 
sin   where x ≠ 0, x ≠ 1/r π &

sin
19. For the function g whose graph is given, arrange the  x.sin 1 

x

following numbers in increasing order.  x
0, g ′(–2), g ′(0), g ′(2), g ′(4) f(0) = f(1/rπ) = 0, r = 1, 2, 3, ..........


Y 30. Suppose that instead of the usual definition of the
y=g(x)

derivative Df(x), we define a new of derivative, D*f(x),
by the formula,
–1 0 1 2 3 4 X
f 2 (x  h)  f 2 (x)
D*(x) = lim ,

h→0 h

FREE BOOKS FOR JEE & NEET =>(@iitjeeadv)


3.66  
Differential Calculus for JEE Main and Advanced

where f 2(x) means [f(x)] 2. Derive the formula for 34. A differentiable function f satisfies the relation




computing the derivative D* of a product & quotient. xf(y) + yf(x) = (x + y) f(x) . f(y) ∀ x , y ∈ R.
31. Let f(x + y) – 2f(x – y) + f(x) – 2f(y) = y – 2 for all real x If f ′(0) = 0, find f(x).

and y. If f’(x) exist, find f(x). 35. A differentiable function f satisfies the relation


32. Let f(xy) = xf(x) + yf(y) for all real x and y. If f is a f(x + y) = f(x) + f(y) + f(x).f(y) ∀ x , y ∈ R.

differentiable function and f(0) = 0, find f(x). If f(0) ≠ –1 and f ′(0) = – 1, find f(x).
33. A differentiable function f satisfies the relation f(xyz) =

f(x) + f(y) + f(z) ∀ x, y, z ∈ R+ and f ′(1) = 1. Find f(x).

Target Exercises for JEE Advanced


1. Let 'f' be a function whose graph is obtained by summing Discuss continuity and differentiability of f(x) over



the ordinate values of the graph of another function ′g ′ its domain.
after shifting its graph by 1 unit leftward and rightward. 8. Suppose f is differentiable on the interval (a , ∞) and


1  x , | x |  1
 lim (f ( x )  f ( x ))   (  R ) .
If g(x) =  discuss the continuity and x
0 , | x | 1

 Prove that lim f ( x )   and lim f ( x )  0 .

differentiability of f(x). x x

2. Check the differentiability of 9. ps Let f be a continuous and differentiable function in (a, b),

lim f ( x )  


 1 

x a 
 

 2  ex 

f(x) = ( x  e)2 , xe


el
at x = e.
 , xe and lim f ( x )   and f ′(x) + f2(x) ≥ – 1 for a < x < b.


 0 x b

eh

 Prove that b  a   .
je

 x (1 + x sin 1 ) , x>0
 10. If
iit

x


x x x
3. Let f(x) =  − | x |(1 + x cos 1 ) , x < 0 . 1  cos 2 x cos 2 .cos 2 .......cos 2 n 1
@



2 8 2

x f(n) = lim
X 0 x2

 0 ,x=0
 and λ = lim f(n). Find the absolute value of 27abc for
Show that f(x) is not differentiable at x = 0.

n→∞

which the function
4. A differentiable function f satisfies the relation f(x2 + y2) =
a

1
(f(x))2 + (f(y))2 ∀ x , y ∈ R. Find f(x).   cot x x  0

5. If f(x) + f(y) = f (x 1 − y 2 + y 1 − x 2 ) ∀ x, g(x) =   x0


 1 1 dy f ( x )  b
y ∈   ,  and = 1, find f(x).  xc x  0
 2 2 dx x  
6. Let f(x) = x3 – x2 + x + 1 and g(x) = 3 – x is differentiable at x = 0.


We define a function  ax ( x  1)  b ; x 1

max{f (u ) , 0  u  x} , 0  x  1 
11. Let f(x) =  x  2 ; 1  x  3 is continuous for
h(x) = 

min{g ( v) , 1  v  x} ,1  x  2  2

 px  qx  2 ; x3
Discuss the continuity and differentiability of h(x) in the all x except x = 1 but |f(x) | is differentiable every where

interval [0, 2]. and f’(x) is continuous
for e  x  1 k
 0 at x = 3 and | a + b + p + q | = , then find the value of k.


18
7. If g(x) =  1 2  . where { }
1  3 sin(l nx )  for 1  x  e 12. Two real valued, continuous and differentiable functions

 

f(x) and g(x) defined on R satisfy the conditions
denotes the fractional part function.
f(x + y) = f(x)g(y) + g(x)f(y)
for 1  x  e

 xg ( x ) g(x + y) = g(x)g(y) – f(x)f(y)
f(x) =  .

 x ( g ( x )  1) otherwise If g′(0) = 0, prove that f2(x) + g2(x) = 1.

FREE BOOKS FOR JEE & NEET =>(@iitjeeadv)
Differentiability 3.67

13. Sketch the graph of the function


 1 

 x 20. If f(x) = cos x. cos  cos x  when
if | x | 1  


 1 | x |
f(x) =  x ∈ (0, 2π) – {π/2, 3π/2},
 x



 1 | x | if | x | 1 and f(x) = 0 when x ∈ {π/2, 3π/2}.



Show that f is continuous but not differentiable at


and find the domain of f ′(x). x = π/2, 3π/2.

e −1/ x for x ≠ 0
2
14. Use the graph of f to answer each question.
21. Prove that the function f(x) = 

Y  0 for x = 0


f
B C is infinitely differentiable at the point x = 0.


A D E
22. A function f : R → [1, ∞) satisfies the equation


X f(xy) = f(x) f(y) – f(x) – f(y) + 2


(i) Between which two consecutive points is the average rate If f is differentiable on R and f(2) = 5, then show




of change of the function greatest ? f ( x ) −1
that f ′(x) = . f ′(1) Hence, determine f(x).

(ii) Is the average rate of change of the function between x


A and B greater than or less than the instantaneous
23. Let f : R → R satisfy |f(x)| ≤ x2ex ∀ x ∈ R, then show
rate of change at B ?
that f(x) is differentiable at x = 0.
(iii) Sketch a tangent line to the graph between the points


C and D so that the slope of the tangent line is the
ps | x 1 | 1
24. Let f(x) = and g(x) = , then find
same as the average rate of change of the function 11/ x 1
el
between C and D. 1+
f (x)
eh

(iv) Give any sets of consecutive points for which


the points where g(x) is not differentiable.


je

the average rates of change of the function are



approximately equal. 25. If f(x + y) = f(x) + f(y) x, y ∈ R, f(1) = 1 then find
iit

15. Given f(x + y + z) = f(x) f(y) f(z) for all real x, y, z if
2f (tan x ) − 2f (sin x )
@

f(2) = 4 and f ′(0) = 3, then show that f(0) = 1 and lim
x 2 f (sin x )
f ( 2)  12 . x→0

16. Let f(x) = x3 – 3x2 + 6  x  R and  x  1 ,  1  x  0

max .{f ( t ), x  1  t  x  2},  3  x  0 26. L e t f ( x ) =  2 . Discuss the
 x , 0  x  1

g( x )   .
1  x for x  0

 differentiability of h(x) = f(|sin x|) + |f(sin x)| in [0, 2π].
Test continuity of g(x) for x  [3, 1] .
27. Let f : R → R satisfy f(xy) = f(x). f(y) – f(x + y) + 1 for


17. If f(x) = x2 – 2 | x | and all x, y ∈ R. If f(x) is differentiable and f ′(0) = 1 then

 min .{f ( t ) : 2  t  x}, 2  x  0 determine all functions ‘f’.
g( x )   .
 max .{f ( t ) : 0  t  x}, 0x3

28. Suppose that function g(t) and h(t) are defined for all

(a) Draw the graph of f(x) and discuss its continuity and values of t and that g(0) = h(0) = 0. Can lim g(t)/(h(t)


t→0
differentiability.
exist? If it does exist, must it equal zero? Give reason for
(b) Find and draw the graph of g(x). Also, discuss the your answer.


continuity.
29. Given a function f, we say that f is differentiable at
18. Suppose that f(x) is continuous at a point ‘a’. Prove that

x0 ∈ Df if there exists a real number A such that

it is differentiable at ‘a’ if and only if there exists some
| f ( x ) − L( x ) | | f ( x ) − f ( x 0 ) − A( x − x 0 ) |
linear function L(x) such that tends to ‘0’ lim .
| x −a | x→x0 | x − x0 |

as x tends to ‘a’.
Prove that if f is differentiable at x0 then the value of
19. Suppose that f(x) is a differentiable function such that

A in the above definition is the derivative f ′(x0). (Note

f ′(x) = g(x), g′′(x) exists and |f(x)| ≤ 1 for all real x. that the magnification function A (x – x0) is called the
If {f(0)}2 + {g(0)}2 = 9, then show that for some c ∈ (–3, 3), differential of f at x0, and A is called the differential
g(c)g"(c) < 0. coefficient.)

FREE BOOKS FOR JEE & NEET =>(@iitjeeadv)


3.68  
Differential Calculus for JEE Main and Advanced

30. A differentiable function f satisfies 33. A differentiable function f satisfies the relation



(x – y) f(x + y) – (x + y) f(x–y)  xy 
∀ x , y ∈ R – {–1}, where

f  = f(x) . f(y)
= 4xy(x2 – y2) ∀ x , y ∈ R, where f(1) = 1.  1  xy 

Find f(x). f(0) ≠ 0 and f ′(0) = 1. Find f(x).


31. A differentiable function f satisfies the relation f(x+y) – 34. A differentiable function f satisfies the relation

f(x – y) = f(x). f(y) ∀ x , y ∈ R.


[1 + f(x).f(y)] f(x + y) = f(x) + f(y) ∀ x , y ∈ R.
If f ′(0) = 0, find f(x).


If f(0) = 0 and f ′(0) = 1, find f(x).

32. Let f be defined on (a, b) and differentiable at c, where


35. Given a differentiable function f(x) defined for all real x,

a < c < b. Suppose xn → c and zn → c, where


and is such that f(x + h) – f(x) ≤ 6h2 for all real h and x.
xn < c < zn for all n.

Show that f(x) is constant.
f ( x n ) − f (z n )
Prove that → f ′(c).
x n − zn

Previous Year's Questions (JEE Advanced)

A. Fill in the blanks : (D) differentiable in (–1, 1)





 1 (E) none of these
( x  1) sin  | x | if x  1
2


1. Let f(x) =  ( x  1) 6. The set of all points where the function
ps



x
 1 if x  1 f(x) = is differentiable, is [IIT - 1987]
el
(1+ | x |)


be a real-valued functions. Then the set of points where (A) (–∞, ∞) (B) [0, ∞)
eh

f(x) is not differentiable is........... [IIT - 1981]




(C) (–∞, 0) ∪ (0, ∞) (D) (0, ∞)





je

2. Let f(x) = x |x|. The set of points where f(x) is twice (E) none of these

[IIT - 1992]


differentiable is............... Let f : R → R be a differentiable function and f(1) = 4.
iit

7.


f (x)
B. Multiple Choice Questions with ONE correct 2t
@

Then the value of lim  dt is



answer : x 1
4
x 1 [IIT - 1990]
3. For a real number y, let [y] denotes the greatest integer (A) 8f ′(1) (B) 4 f ′(1)






less than or equal to y : Then the fuction (C) 2 f ′(1) (D) f ′(1)




tan([ x  ]) 8. Let [ . ] denote the greatest integer function and
f(x) = is [IIT - 1981]

1  [ x ]2 [IIT - 1993]


f(x) = [tan2 x], then :

(A) discontinuous at some x
(A) lim f(x) does not exist


(B) continuous at all x, but the derivative f ′(x)



x→0



does not exist for some x (B) f(x) is continuous at x = 0



(C) f ′(x) exist for all x, but the derivative f ′′(x does not (C) f(x) is not differentiable at x = 0





exist for some x (D) f ′(0) = 1


(D) f ′(x) exits for all x 9. The function f(x) = (x2 – 1) |x2 – 3x + 2| + cos (|x|) is Not



4. If f(x) = x ( x  x 1) , then [IIT - 1985] differentiable at - [IIT - 1999]



(A) f(x) is continuous but not differentiable at x = 0 (A) – 1 (B) 0






(B) f(x) is differentiable at x = 0 (C) 1 (D) 2






(C) f(x) is not differentiable at x = 0 10. Find left hand derivative of f(x) =[x]sin (πx) at k, k an




(D) none of these integer if [IIT - 2001]


5. Let [x] denote the greatest integer less than or equal to x. (A) (–1)k (k – 1)π (B) (–1)k – 1 (k – 1)π

If f(x) = [x sin π x], then f(x) is





(C) (–1)k (k –1)kπ (D) (–1)k–1 (k – 1)kπ
[IIT - 1986]




(A) continuous at x = 0 11. Let f : R → R be a function which is defined by




(B) continuous in (–1, 0) f(x) = max {x, x3}. The set of all points where f(x) is not
differentiable is - [IIT - 2001]


(C) differentiable at x = 1




FREE BOOKS FOR JEE & NEET =>(@iitjeeadv)
Differentiability 3.69

(A) {–1, 1} (B) {–1, 0} (B) differentiable at x = 0 but not differentiable at x = 2









(C) {0, 1} (D) {–1, 0, 1} (C) not differentiable at x = 0 but differentiable at x = 2






12. Which of the following functions is differentiable at (D) differentiable neither at x = 0 nor at x = 2



[IIT - 2012]

x=0? [IIT - 2001]

(A) cos (|x|) + |x| (B) cos (|x|) – |x| C. Multiple Choice Questions with ONE or






(C) sin (|x|) + |x| (D) sin (|x|) – |x|
MORE THAN ONE correct answer :




13. The domain of the derivative of the function

 tan 1 x , if | x | 1 20. If x + |y| = 2y, then y as a function of x is




f (x)   1 is [IIT - 2002] [IIT - 1984]


 | x | 1 , if | x |  1 (A) defined for all real x


2




(B) continuous at x – 0



(A) R – {0} (B) R – {1} (C) differentiable for all x








(C) R – {–1} (D) R – {–1, 1} dy 1




14. If f(x) is differentiable and strictly increasing function, (D) such that =
3
for x < 0



dx

then the value of lim f ( x ) − f ( x ) is
2
21. The function f(x) = 1 + |sin x| is [IIT - 1986]
x→0 f ( x ) − f (0)



(A) continuous now here




[IIT - 2004] (B) continuous everywhere



(A) 1 (B) 0 (C) differentiable nowhere







(C) –1 (D) 2 (D) not differentiable at x = 0






15. The function given by y = ||x| – 1| is differentiable for all (E) not differentiable at infinite number of points.



real number except the points [IIT - 2004]
ps
| x  3 |, x 1

(A) {0, 1, –1} (B) ± 1 
el
22. The function f(x) =  x 2 3x 13 is





   , x 1

(C) 1 (D) –1
eh

 4




2 4
16. If f is a differentiable function and f(1/n) = 0 n ≥ 1 and [IIT - 1988]

n ∈ Ι, then [IIT - 2005]
je

(A) continuous at x = 1

(A) f(x) = 0, x ∈ (0, 1]


(B) differentiable at x = 1
iit





(B) f(0) = 0, f ′(0) = 0 (C) continuous at x = 3
@




(C) f(0) = 0 = f ′(x), x ∈ (0, 1] (D) differentiable at x = 3





(D) f(0) = 0 and f ′(0) need not to be zero 23. Let h(x) = min {x, x2}, for every real number of x. Then



17. Let f(x) be differentiable on the interval (0, ∞) such [IIT - 1998]


t 2f (x)  x 2f (t) (A) h is continuous for all x



f(1) = 1, and lim = 1 for each x > 0. (B) h is differentiable for all x
tx

t x


(C) h’(x) = 1 , for all x > 1
Then f(x) is [IIT - 2007]



(D) h is not differentiable at two values of x’


1 2x 2 1 4 x 2


(A) + (B)   0,
 x0
[IIT - 1994]





3x 3 3x 3 24. Let f(x) =  then for all x
2
x ,
 x0


1 2 1
(C)  (D) (A) f ′ is differentiable (B) f is differentiable
x x2









x (C) f ′ is continuous (D) f is continuous




( x  1) n  x sin(1 / x ) , x  0
18. Let g ( x )  ; 0 < 2, m and n are integers, 25. Let g(x) = x f(x), where f(x) =  At x = 0
log cos m ( x  1)

 0 , x0

m ≠ 0, n > 0 and let p be the left hand derivative of |x – 1| [IIT - 1994]

at x = 1. If lim g(x) = p, then :
x →1
[IIT - 2008] (A) g is differentiable but g’ is not continuous



(B) g is differentiable while f is not
(A) n = 1 n = 1 (B) n = 1, m = –1


(C) both f and g are differentiable





(C) n = 2, m = 2 (D) n > 2, m = n



(D) g is differentiable and g’ is continuous




 2 


x cos , x  0
19. Let f ( x )  
26. The function f(x) = max {(1 – x), (1 + x), 2},
, x  , then f is

x
x ∈ (–∞, ∞) is [IIT - 1995]

 0, x  0

 (A) continuous at all points


(A) differentiable both at x = 0 and at x = 2 (B) differentiable at all points




FREE BOOKS FOR JEE & NEET =>(@iitjeeadv)
3.70  
Differential Calculus for JEE Main and Advanced

(C) differentiable at all points except at x = 1 and x = –1 




(D) continuous at all points except at x = 1 and (A) f(x) is continuous at x  



2


x = –1, where it is discontinuous (B) f(x) is not differentiable at x = 0
27. Let f : R → R be any function. Define g : R → R by



(C) f(x) is differentiable at x = 1

g(x) = |f(x)| for all x. Then g is - [IIT - 2000]



3
(D) f(x) is differentiable at x = −


(A) onto if f is onto 2






(B) one-one if f is one-one 33. Let g : R → R be a differentiable function with g(0) = 0,



(C) continuous if f is continuous g′(0) = 0 and g′(1) ≠ 0. Let



(D) differentiable if f is differentiable
 x


28. If f(x) = min. {1, x2, x3}, then [IIT - 2006]  g(x), x ≠ 0
f (x) =  | x |


(A) f(x) is continuous ∀ x ∈ R  0,


 x=0



(B) f(x) is continuous & differentiable everywhere
and h(x) = e|x| for all x ∈ R. Let (f oh) (x) denote f (h(x))


(C) f(x) is continuous but not differentiable ∀ x ∈ R






(D) f(x) is not differentiable at two points and (hof )(x) denote h(f (x)), then which of the following



is (are) true ?


 
 x   , x (A) f is differentiable at x = 0



2
 2 (B) h is differentiable at x = 0
  cos x ,    x  0


29. If f ( x )   , then [IIT - 2011]
(C) foh is differentiable at x = 0


2


 x 1 (D) hof is differentiable at x = 0 [2015]
 0  x 1



 n x , x 1
34. Consider the hyperbola H : x2 – y2 = 1 and a circle S with


ps
centre N(x2, 0). Suppose that H and S touch each other
π at a point P(x1, y1) with x1 > 1 and y1 > 0. The common
(A) f(x) is continuous at x = –
el
tangent to H and S at P intersects the x-axis at point M.


2
(B) f(x) is not differentiable at x = 0 If (l, m) is the centroid of the triangle ∆PMN, then the
eh


(C) f(x) is differentiable at x = 1 correct expression(s) is(are)


je

3 dl 1
(D) f(x) is differentiable at x = – (A) = 1 − 2 for x1 > 1
iit


2


dx1 3x1
30. Let f : R → R be a function such that f(x + y) = f(x) +
@

dm x1

f(y), ∀ x, y ∈ R. If f(x) is differentiable at x = 0, then =
(B)
dx1 3 ( ) for x1 > 1

[IIT - 2011] x12 − 1



(A) f(x) is differentiable only in a finite interval containing dl 1
=1+


zero (C) for x1 > 1
3x12


dx1
(B) f(x) is continuous ∀ x ∈ R
dm 1


(C) f ′(x) is constant ∀ x ∈ R (D) = for y1 > 0 [2015]



dy1 3


(D) f(x) is differentiable except at finitely many points


31. Let f :  →  be a function such that  1   1 
35. Let f :  − , 2 → R and g :  − , 2 → R be functions

   2 

2
f ( x  y)  f ( x )  f ( y),  x , y  
defined by f (x) = [x – 3] and g(x) = |x| f (x) + |4x – 7| f (x),
2

If f(x) is differentialbe at x = 0, then [IIT - 2011]





where [y] denotes the greatest integer less than or equal


(A) f(x) is differentiable only in a finite interval containing to y for y ∈ R. Then


zero
 1 
(B) f(x) is continuous  x   (A) f is discontinuous exactly at three points in  − , 2
 2 




(C) f '(x) is constant  x  
 1 



(D) f(X) is differentiable except at finitely many points. (B) f is discontinuous exactly at four points is  − , 2


 2 


  
 x  2 , x   2 (C) g is not differentiable exactly at four points is


  1 
  − ,2
32. If f ( x )   cos x,   x  0 , then [IIT - 2011]  2 
2


 (D) g is not differentiable exactly at five points in
 x  1, 0  x  1


ln x , x  1  1 
− ,2
 2 
 [2016]

FREE BOOKS FOR JEE & NEET =>(@iitjeeadv)
Differentiability 3.71

36. Let a, b ∈ R and f : R → R be defined by f (x) F. Subjective Problems :






= a cos (|x3 – x|) + b |x| sin (|x3 + x|). Then f is x2
(A) differentiable at x = 0, if a = 0 and b = 1 43. Let f(x) = ,0≤x<1 [IIT - 1983]



2


(B) differentiable at x = 1, if a = 1 and b = 0 3
= 2x2 – 3x + , 1 ≤ x ≤ 2


(C) not differentiable at x = 0, if a = 1 and b = 0


2


(D) not differentiable at x = 1, if a = 1 and b = 1 [2016] Discuss the continuity of f, f ′ and f ′′ on [0, 2].




44. Let f(x) = x3 – x2 + x + 1 and [IIT - 1985]
37. Let f : (0, ∞) → R be differentiable function such that f′(x)



g(x) = max {f(t); 0 ≤ t ≤ x}, 0 ≤ x ≤ 1


f (x)



= 2− for all x ∈ (0, ∞) and f (1) ≠ 1 then  
=3–x 1<x≤2



x Discuss the continuity and differentiability of the function


 1 g(x) in the interval (0, 2).
(A) lim f ′   = 1
 x Let f(x) be defined in the interval [–2, 2] such that


x →0+ 45.


 1 f(x) =1, –2 ≤ x ≤ 0


(B) lim xf   = 2 = x – 1, 0 < x ≤ 2
 x


x →0+


and g(x) = f(|x|) + |f(x)|
(C) lim x f ′ ( x ) = 2
2


x →0+ Test the differentiability of g(x) on x ∈ [–2, 2]



(D) |f (x)| ≤ 2 for all x ∈ (0, 2) [2016]

[IIT - 1986]




38. Let f : R → R, g : R → R and h : R → R be differentiable 46.
Let f(x) be a function satisfying the condition f(–x) = f(x)
for all real x. If f’(0) exists, find its value. [IIT - 1987]


function such that f (x) = x3 + 3x + 2, g(f (x)) = x and
ps




h(g(g(x)) = x for all x ∈ R. Then 47. Draw a graph of the function y = [x] + |1 – x|,

–1 ≤ x ≤ 3. Determine the points, if any, where this function
el
1
(A) g′(2) = (B) h′(1) = 666
is not differentiable. [IIT - 1989]
eh




15


(C) h(0) = 16 (D) h(g(3)) = 36 [2016] 48. A function R → R satisfies the equation f(x + y) = f(x) f(y) for





je

39. For every twice differentiable function f : R →[–2, 2] with all x, y in R and f(x) ≠ 0 for any x in R. Let the function be

iit

(f(0))2 + (f ¢(0))2 = 85, which of the following statement(s) differentiable at x = 0 and f’(0) = 2. Show that f’(x) = 2 f(x)

is (are) TRUE? for all x in R . Hence, determine f(x). [IIT - 1990]
@

(a) there exist r, s ∈R, where r < s such that f is one-one


xy f ( x ) + f ( y)


on the open interval (r, s). 49. Let f   = for all real x and y. If f ′(0)
(b) there exits x0 ∈(–4, 0) such that |f′(x0)| ≤ 1  2  2



(c) lim f ( x ) = 1 exists and equals – 1 and f(0) = 1. Find f(2)


x →0 [IIT - 1995]
(d) there exists a ∈ (–4, 4) such that f(a) + f″(a) = 0 and

   1  1 


f′(a) ≠ 0 [2018] 
50. Let f(x) =  xe
|x| x 
; x0 test whether


D. Assertion & Reasoning :

 0 ; x0

40. Let f and g be real valued functions defined on interval [IIT - 1997]

(–1, 1) such that g′′(x) is continuous, g(0) ≠ 0, f′′(0) = 0 (a) f(x) is continuous at x = 0



and f(x) = g(x) sin x. (b) f(x) is differentiable at x = 0


Statement–1 : lim [g(x) cot x – g(0) cosec x] = f ′′(0) 51. Determine the values of x for which the following function

x →0

fails to be continuous or differentiable
Statement–2 : f ′(0) = g(0) [IIT - 2008]


1  x ; x 1
E. Integer Answer Type : 
f(x) = (1  x )(2  x ) ; 1  x  2

41. If the function f(x) = x3 + ex/2 and g(x) = f–1(x), then the 3  x


 ; x2

value of g′(1) is : [IIT - 2009]
Justify your answer. [IIT - 1997]

42. Let y'(x) + y(x) g'(x) = g(x), y(0) = 0, x ∈  , where


52. Discuss the continuity and differentiability of the function

df ( x )

f'(x) denotes and g(x) is a given non-constant [IIT - 1998]

dx 
 2  1  x ; | x | 1
2
differentiable function on  with g(0) = g(2) = 0. Then f(x) =  (1 x )
 ; | x | 1
2

[IIT - 2011]

the value of y(2) is  2e

FREE BOOKS FOR JEE & NEET =>(@iitjeeadv)
3.72  
Differential Calculus for JEE Main and Advanced

53. Let α ∈ R. Prove that a function f : R → R is differentiable If right hand derivative at x = 0 exists for f(x). Find



at α if and only if there is a function g: R → R which is derivative of g(x) at x = 0 [IIT - 2005]


continuous at α and satisfies f(x) – f(α) = g(x) (x – α) G. Match the Columns
for all x ∈ R.


[IIT - 2001] 58. Column-I Column-II

 x  a if x  0



(A) x |x| continuous in (–1, 1) (P)
54. Let f(x) = | x  1 | if x  0 , and







 (B) | x | differentiable in (–1, 1) (Q)





 x  1 if x  0 (C) x + [x] strictly increasing in (–1, 1) (R)





g(x) = ( x  1) 2  b if x  0 , where a and b are non- (D) |x – 1| + |x + 1| not differentiable at least at one (S)







point in (–1, 1)
negative real numbers Determine the composite function 59. Let f 1: R → R, f 2: [0, ∞) → R, f 3: R → R and f 4: R → [0, ∞)






g of. If (g o f) (x) is continuous for all real x, determine be defined by
the values of a and b. Further, for these value of a and | x | if x < 0,
b, is gof differentiable at x = 0 ? Justify your answer. f 1(x) =  x
[IIT - 2002]

e if x ≥ 0;




55. If a function f : [– 2a, 2a] → R, is an odd function such f 2(x) = x2;




that f(x) = f(2a – x) for x ∈ [a, 2a] and left hand derivative sin x if x < 0,
at x = a is 0, then find the left hand derivative at x = – a. f 3(x) = 

x if x≥0



[IIT - 2003] and


 f (f (x)) if x < 0,
1 x  c f 4(x) =  2 1
 b sin ; 1 / 2  x  0
f 2 (f1 (x)) − 1 if x ≥ 0.


 2
ps

56. Let f(x) = 1 / 2 ; x0 List I List II
el


P. f 4 is 1. onto but not one-one

 ax / 2
e 1 1





; 0x Q. f 3 is 2. neither continuous nor one-one
eh








x 2
R. f 2 o f 1 is 3. differentiable but not one-one
If f(x) be a differentiable function at x = 0, & |c| < 1/2,






je

S. f 2 is 4. continuous and one-one



then find the value of ‘a’ and prove that 64 b2 = (4 – c2).





iit

P Q R S

[IIT - 2004]




(A) 3 1 4 2

@







(B) 1 3 4 2
57. If f(x – y) = f(x) . g(y) – f(y) . g(x) and






(C) 3 1 2 4

g(x – y) = g(x) . g(y) + f(x) . f(y) for all x, y ∈ R.






(D) 1 3 2 4 [2014]







Previous Year's Questions (JEE Main Papers)

1. Let f be a differentiable function such that 8f (x)


x 2 + y2 x 2 + y2


 1 dy (c) (d) [2013, online]
+ 6f   – x = 5 (x ≠ 0) and y = x2 f (x), then at y2





 x x2

dx
x = –1 is dy
3. If y = sec (tan–1 x), then is equal to

15 15 dx
(a) (b) − x=1




14 14 1
(a) (b) 1
1 1




(c) − (d) [2013, online] 2
1





14 14 (c) 2 (d) [2013]





2
For a > 0, t ∈  0, π  , let x =
sin −1 t cos −1 t
2. a , y= a . Then
 2 x2 − x

d
4. Let f(x) = , x ≠ 0, –2. Then (f –1(x)) (wherever
2
x 2 + 2x


 dy  dx
1 +   equals defined) is equal to
 dx 
−1 3
x2 y2 (a) (b)
(a) (b) (1 − x ) 2
(1 − x ) 2




2




y x2

FREE BOOKS FOR JEE & NEET =>(@iitjeeadv)


Differentiability 3.73

−3
x > 0, then f   is equal to
1 3
(c) (d) [2013, online]
(1 − x ) 2
(1 − x ) 2  2





5. If g is the inverse of a function f and f ′(x) = 1 , then 23 13
(a) (b)


1 + x5





18 6
g′(x) is equal to 25 31
(c) (d) [2016, online]
(a) 1+ x5 (b) 5x4






9 8




1 − x, x <1
(c) (d) 1 + (g(x))5 [2014] 
1 + (g ( x ))5 13. If the function f(x) =  is





−1
a + cos ( x + b) 1 ≤ x ≤ 2


6. Let f(x) = x|x|, g(x) = sin x and h(x) = go f(x). Then

a
(a) h(x) is not differentiable at x = 0 differentiable at x = 1, then is equal to


(b) h(x) is differentiable at x = 0, but h′(x) is not π + 2 b
π −2


continuous at x = 0 (a) (b)





2 2
(c) h′(x) continuous at x = 0, but it is not differentiable
−π − 2


at x = 0 (c) (d) –1 – cos–1(2) [2016]






(d) h′(x) is differentiable at x = 0 [2014, online] 2



15 15
1 14. If y =  x + x 2 − 1  +  x − x 2 − 1 then
7. If f(x) = x2 – x + 5, x > , and g(x) is its inverse function,    


then g′(7) equals 2
d2 y dy
(a) −
1
(b)
1 ( x 2 − 1) 2
+x is equal to

ps dx dx




3 13
1 1 (a) 125 y (b) 224 y2
el
(d) − [2014, online]




(c) (c) 225 y2 (d) 225 y [2017 online]





3 13





eh

2
8. Let f : R → R be function such that |f (x)| ≤ x2 for all d y dy
15. If 2x = y1/5 + y–1/5 and (x2 – 1) + λx + ky = 0 then


x ∈ R. Then at x = 0, f is
je

2
d x

l + k is equal to dx
(a) continuous but not differentiable
iit

(a) –23 (b) –24




(b) continuous as well as differentiable




(c) 26 (d) –26 [2017 online]


@

(c) neither continuous nor differentiable









(d) differentiable but not continuous [2014, online]  6x x 
16. If for x ∈(0, 1/4), derivative of tan −1 



 d y d x 2 2  is
 1 − 9x 3 

9. If y = enx, then   is equal to
 dx 2  dy 2

x ⋅ g ( x ) then g(x) is equal to
(a) nenx (b) ne–nx 3 9




(c) 1 (d) –ne–nx [2014] (a) (b)
1 + 9x 3
1 + 9x 3









k x + 1, 0 ≤ x ≤ 3 3x x 3x
10. I f t h e f u n c t i o n g ( x ) =  is (c) (d) [2017]
 mx + 2, 3 < x ≤ 5

1 − 9x 3 1 − 9x 3





differentiable, then the value of k + m is d2 y
16 17. If x2 + y2 + sin y = 4, then the value of at the point
dx 2

(a) 2 (b)




5 (–2, 0) is
10
(c) (d) 4 [2015] (a) –34 (b) –32









3 (c) –2 (d) 4 [2018 online]
11. For x ∈ R, f(x) = |log2 – sin x| and g(x) = f(f(x)), then






(a) g is not differentiable at x = 0 cos x x 1


(b) g′(0) = cos(log 2)
18. If f(x) = 2 sin x x 2
2 x , then lim f ′ ( x )


(c) g′(0) = –cos(log 2)

x →0 x


(d) g is differentiable at x = 0 and g′(0) = –sin(log2) tan x x 1


[2016] (a) exists and is equal to 0



12. If f(x) is differentiable function in the internal (0, ∞) (b) exists and is equal to –2



t 2f (x) − x 2f (t) (c) exists and is equal to 2


such that f(1) = 1 and lim = 1 for each (d) does not exist [2018 online]
t→x t−x



FREE BOOKS FOR JEE & NEET =>(@iitjeeadv)
3.74  
Differential Calculus for JEE Main and Advanced

24. Let f: (–1, 1) → R be a function defined by


 2 × 3x   1


19. If f(x) = sin −1  x 
, then f ′  −  is equal to max{− | x |, − 1 − x 2 } . If k be the set of all points at
 1+ 9   2

which f is not differentiable then k has exactly
(a) 2 log 3 (b) − 3 log 3 (a) three elements (b) one elements





(c) five elements (d) two elements




[2019]
(c) − 3 log 3 3 log 3 [2018 online]






(d)
25. If x log(log x) – x2 + y2 = 4 (y > 0), then dy at x = e is





20. For each x ∈R, let f(x) = |x – 1|, g(x) = cos x and ϕ(x) =


dx

f(g(2 sin x) – g(f(x)), then ϕ is equal to
(a) differentiable at each point of R e 1 + 2e
(a) (b)


(b) not differentiable at 0





4 + e2 2 4 + e2


(c) not differentiable at 1 2e − 1 1 + 2e


(d) differentiable only in (– /2, /2) [2018] (c) (d) [2019]






2 4+e 2
4 + e2


p
p

21. Let S = {t ∈R : f(x) = |x – π|. (e|x| – 1) sin |x| is not
 −1, − 2 ≤ x < 0

differentiable at t}. Then the set S is equal to 26. Let f(x) =  2 and g(x) = |f(x)| + f(|x|). Then
 x − 1, 0 ≤ x ≤ 2


(a) {0} (b) {π}




(c) {0, π} (d) ϕ (empty set) [2018] in the interval (–2, 2), g is





(a) differentiable at all points
d2 y



22. If x = 3 tan t and y = 3 sec t, then the value of at t = (b) not differentiable at two points
dx 2



(c) not continuous
ps


π (d) not differentiable at one point [2019]



is 27. Let f be a differentiable function such that f(1) = 2 and
el
4

f′(x) = f(x) for all x ∈ R. If h(x) = f(f(x)) then h′(1) is equal to
eh

3 1 (a) 4e (b) 4e2


(a) (b)








2 2 (d) 2e2
je

3 2 (c) 2e [2019]





d y
iit

1 1 28. For x > 1, if (2x)2y = 4e2x – 2y, then (1 + log 2x)2 is equal
(c) (d) [2019]

dx





6 6 2
@

to
max{| x |, x 2 }, |x|≤ 2 x log 2 x + log 2
23. Let f(x) =  (a) log 2x (b)




x
8 − 2 | x |, 2 < |x| ≤ 4

x log 2 x − log 2
Let S be the set of points in the interval (–4, 4) at which f (a) x log 2x (b) [2019]

is not differentiable. Then S:





x

(a) is an empty set 29. Let K be the set of all real values of x where the function

f(x) = sin |x| – |x| + 2(x – p) cos |x| is not differentiable,



(b) equals {–2, –1, 1, 2}


(c) equals {–2, –1, 0, 1, 2} then the set K is equal to



(d) equals {–2, 2} (a) { } (b) {0}


p


(c) f (an empty set)


(d) {0, } [2019]




p

ANSWERS
Concept Problems—A Practice Problems—A
1. f ′(0 ) = – 1 ; f ′(0 ) = 1; non-differentiable
+ –
7. – h; 0.


2. m = – 1, n = π. 2 −2
8. (a) –1,1 (b) ,

f ′(1+) = – 1; f ′(1–) = 3 ln 3 hence f is not differentiable




3. a a

at x = 1 (c) 1, 0 (d) 0, 0.





4. (b) (2a–1/3)/3 9. f(x) is not differentiable at x = 0



5. yes; y = 1. 14. 0


6. non-differentiable at x = 0 15. 0


FREE BOOKS FOR JEE & NEET =>(@iitjeeadv)
Differentiability 3.75

Concept Problems—B Practice Problems—C


1. (a) –1, 2 (b) 1, 3 11. continuous but non-differentiable.





2. (a) 0 (b) 1, 3 12. differentiable.





3. (a) 5 (b) 2, 3 13. x = 0, ± 1





4. – 4 (discontinuity), – 1 (corner), 2 (discontinuity), 5 14. a = 35/9, b = 10/3



(vertical tangent).
Y
15. Non differentiable.



16. Y
f′


Y
f'
5. No f

(1 ,1 ) 1 X O
O X O X
7. a = 2, b = 0

8. continuous and differentiable.

9. cont. but not diff.at x = 0 ; diff. & cont. at x = π/2

10. a = 1, b = 1 17. f is cont. but not diff. at x = 1, discont. at x = 2 &

discontinuous and non-differentiable at x = 1.

11. x = 3. cont.& diff. at all other points

18. 3 ; The bold line represents the graph of y = f(x).
Practice Problems—B

Y
ps
12. continuous; diff at x = 1, non diff. at x = 0

continuous but non diff at x = 0 y=cosx
el
13.


14. a=2 y=1 -cosx
eh

15. diff.



O
je

16. discontinuous & not derivable at x = 1, X




iit

continuous but not derivable at x = 2


18. discontinuous at x = 1 and hence non-differentiable. y=sinx
@



It is non-differentiable at three points namely
Concept Problems—C

  5
1. (a) x = – 3, 1 (b) x = ± 1. x , , .
4 2 3





2. yes
19. g o f (x) = 0 for all x ∈ R and


3. (a) –1, 1
0 if x is an int eger


(b) x = 10/3, L.H.D. = –3, R.H.D. = 3  
f o g = n if x  R and n  x  n  1 or


5. f ′(o) does not f ′ (o–) = 0 f ′ (o+)

 
 x  R and  n  1  x   n


6. f(x) is cont. but not diff. at x = 0


7. (a) Y g o f is differentiable everywhere and f o g is differentiable


on R – I.
cos x , 0x
20. g ( x )  
 sin x  1 , x  

0
X Adjacent figure represents the graph of g(x). Clearly, g(x)

is continuous but non-differentiable at x = π.
Y

(b) All x (c) f ′(x) = 2|x|




O X
8. discontinuous at x = 0, 1, 2, 3 hence non-differentiable

9. The Dirichlet function is nowhere continuous, and hence

it is nowhere derivable.
10. diff every where.


FREE BOOKS FOR JEE & NEET =>(@iitjeeadv)
3.76  
Differential Calculus for JEE Main and Advanced

Concept Problems—D 11. Cont. but non diff. at x = 1, discont. and non diff. at x = 2, 3


12. non diff. at x = π, 2π
1. 6 cos 9


13. p>1


2. f(x) = x6 , a = 2 14. non-diff at x = 2, 3 if a ≠ 4, 9; non diff at x = 3 if


3. (i) f(x) = 2x, a = 5 a = 4; non-diff at x = 2 if a = 9.


(ii) f(x) = cos x, a = π or f(x) = cos (π + x), a = 0
Practice Problems—F


4. The only doubtful point is 0. But |f(x)/x| = |x| → 0

as x → 0, so f ′(0) = 0. For x > 0, f ′(x) = 2x and for 2. x, |x|


x < 0, f ′(x) = –2x. therefore f ′(x) = 2|x| is continuous. 3. –1


5. 1 4. 0



6. 2/3 5. 2x




non diff. at x = 0 1
7. 7. , n2


2
8. a = 6, b = –3.
8. x2 + x ex


Concept Problems—E 10. f(x) = 2x


2. (a) x for x ≥ 0, – x for x < 0


(b) Yes
OBJECTIVE EXERCISES


(c) 1 for x > 0, – 1 for x < 0


4 1/ 3 1. C 2. B 3. A
3. (a) x (b) Yes ps










3 4. A 5. B 6. A






4
el
(c) 7. A 8. D 9. B






9x 2 / 3


10. B 11. A 12. D
eh





4. (a) g′(x) = 2x for x ≤ 1, g′(x) = 2 for x > 1 13. A 14. A 15. A








je

g"(x) = 2 for x ≤ 1, g"(x) = 0 for x > 1 16. B 17. A 18. D









(b) g is differentiable ; g ′ is not
iit

19. A 20. B 21. D











5. – sin a 22. C 23. B 24. D
@








25. C 26. B 27. A
Practice Problems—D






28. C 29. C 30. C






6. (a) y" = 6 |x|; y"(0) = 0. 31. D 32. A 33. C








1 2 1 1 1 34. C 35. C 36. A
(b) y" = 2 sin − cos − 2 sin at x ≠ 0, y"(0)








x x x x x 37. D 38. D 39. C






does not exist, since y ′ (x) is discontinuous at x = 0. 40. A 41. C 42. D






8. 2secx x tan x 43. A 44. C 45. A







9. n–1<k<n 46. B 47. C 48. D







49. A 50. C 51. A,C
Concept Problems—F






52. A,D 53. A,B,C 54. B,D






1. (a) Yes (b) No 55. A,C 56. A,B,C 57. D










2. No 58. B,D 59. A,B,C 60. B,C,D







3. (a) No (b) No 61. B,D 62. B,D 63. A,B,C,D




If g(x0) ≠ 0 then f is non-diff. as x = x0. If g(x0) = 0 then






4. 64. A,C,D 65. A,B,D 66. A,B

f is differentiable is x = x0.






67. A, B, C 68. A,C,D 69. C,D






Practice Problems—E 70. B,C 71. A 72. A






73. D 74. A 75. D






6. Yes
76. A 77. D 78. B

7. No






79. C 80. B 81. A

8. (b) f is twice but not thrice differentiable at x = x0 and the








derivatives are 0. 82. C 83. B 84. D






9. They are of opposite signs. 85. A 86. A 87. B







10. non-differentiable. 88. D 89. B 90. D







FREE BOOKS FOR JEE & NEET =>(@iitjeeadv)
Differentiability 3.77

91. C 92. D 93. C Y








94. C 95. B




96. (A)–(P,Q), (B)–(S), (C)–(P), (D)–(R) 1



97. (A)–R,(B)–S, (C)–P, (D)–Q



98. (A)–(P), (B)–(R), (C)–(Q), (D)–(T) 1
0 X –1


99. (A)–P; (B)–(P, Q), C–PRT, D–R


–1
100. (A)–(Q), B–(R), (C)–(S), (D)–(P)





REVIEW EXERCISES for JEE ADVANCED 23. f (1 )  3, f (1 )  1


25. (a) 2 (b) 1
continuous but non-differentiable.


1. 26. f(x) is not cont. not diff, xf(x) is cont. and non diff. x2f(x)


2. g(x) = x is cont and diff.

3. discont and non-diff at x = 0, 2; cont but non-diff at x = ± 1. 27. cont but not diff at x = 1, discont. at x = 2, 3


4. discont and non-diff at x = 0; cont but non-diff at x = ± 2 28. a ≠ 1, b = 0, p = 1/3, q = – 1


and –1,1
29. cont in 0 ≤ x ≤ 1, but not diff. at x = 0


5. {1, 3}
30. D*[f(x). g(x)] = f2 (x) D*g(x) + g2(x). D*f(x);

non-diff at x = 4

6.

7. discontinuous and non-differentiable at x = 2. g 2 ( x ).D * f ( x ) − f 2 ( x ).D * g ( x )
D* =

[g ( x )]4

8. a = – 1, b = – 2, value = 2/3.

9. If f(x0) = g(x0), then the two functions may or may not be 31.
ps x+1


differentiable at x = x0. 32. f(x) = 0
el
12. f is cont. but not derivable at x = 0. 33. f(x) = ln x
eh


14. continuous but non-differentiable at x = 0. 34. f(x) = 1, f(x) = 0.


je

15. 180 35. f(x) = e–x – 1




16. a ∈ (0, 1]
iit

TARGET EXERCISES for JEE ADVANCED




17. If a ∈ (0, 1),f ′ (0+) = −1,f ′ (0−) = 1
@

⇒ continuous but not derivable ; If a = 1, f (x) = 0 which
1. Continuous but non-differentiable at x = –2, –1, 0, 1, 2

is constant ⇒ continuous and derivable;

2. not differentiable at x = e; f ′(e+) = 1, f ′(e–) = 0.
If a > 1, f ′(0−) = − 1, f ′(0+) = 1

4. f(x) = 0, f(x) = 1/2, f(x) = x. 5. f(x) = sin–1 x)

⇒ continuous but not derivable.


6. continuous but non-differentiable at x = 1.

non diff. at x = 1

18. 7. f is continuous and differentiable for all x ∈ domain ,



19. g′(0), 0, g′(4), g′(2), g′(– 2) except non-differentiable at x = 1

20. f(x) is non differentiable at x = α, β, 0, γ, δ and g(x) is 10. 256


non differentiable at x = α, β, 0, – 2, 2 11. 54


Y 13. R – {–1, 1}

Y

X
 x
 1  x if x  0, x  1 X
21. f(x) = 
 x if x  0, x   1


 1  x

 1 14. (i) A and B (ii) Greater
if x  0, x  1




 Y
 1  x 
2
f
and f ′(x) =  (iii) BC (iv) B and C, D and E


1
 if x  0, x   1 A D




E
 1  x 2
 X

FREE BOOKS FOR JEE & NEET =>(@iitjeeadv)


3.78  
Differential Calculus for JEE Main and Advanced

16. g(x) is continuous in [–3, 1] except x = 0 35. BC 36. AB






17. (a) continuous for all x ∈ R not differentiable for all 37. A 38. BC 39. ABD








x  R  {0} 40. B 41. 2 42. 0







(b) g(x) is not continuous at x = 0 43. f and f ′ are continuous and f ′′ is discontinuous on [0, 2]




44. cont. on (0, 2) and differentiable on (0, 2) = {1}
22. 1 + x2 24. {±1, 0, 1 – 2 })



not differentiable at x = 0, 1


n2 45.
26. Non - diff. at x = π, 2π


25. 46. f ′(0) = 0 47. x = 0, 1, 2, 3



2




27. x + 1 48. f(x) = e2x 49. –1





28. For g(t) = mt and h(t) = t, limitt → 0 g(t)/(h(t) = m, which 50. cont. but not diff.


need not be zero. 51. f(x) is not continuous and thus not differentiable at x = 2


30. f(x) = x3 31. f(x) = 0 continuous but not differentiable at discontinuous and not

52.



1 x e2 x  1 differentiable at x = –1.
33. f(x) = . 34. f(x) = 2 x .
1 x e 1



PREVIOUS YEAR'S QUESTIONS 54. g(f(x)) = a = 1, b = 0,


(JEE ADVANCED)
1. {0} 2. R – {0} gof differentiable at x = 0




55. 0
3. D 4. B
ps


56. a=1




5. ABD 6. A
el
57. 0




7. A 8. B (A)→(P, Q, R), (B)→(P, S), (C)→(R, S), (D)→(P, Q)
eh

58.




9. D 10. A 59. D



je

11. D 12. D




iit

13. D 14. C
QUESTIONS FROM PREVIOUS



@

15. A 16. B
YEAR'S (AIEEE/JEE MAIN PAPERS)




17. A 18. C



19. B 20. ABD 1. (c) 2. (d) 3. (d) 4. (b)












21. BDE 22. ABC 5. (d) 6. (c) 7. (c) 8. (b)











23. ACD 24. BCD 9. (d) 10. (a) 11. (b) 12. (c)











25. AB 26. AC 13. (a) 14. (d) 15. (b) 16. (b)













27. C 28. AC 17. (a) 18. (b) 19. (a) 20. (a)












29. ABCD 30. BC 21. (d) 22. (d) 23. (c) 24. (a)












31. BC 32. ABCD 25. (c) 26. (d) 27. (a)









33. AD 34. ABD 28. (d) 29. (c)







FREE BOOKS FOR JEE & NEET =>(@iitjeeadv)
HINTS & SOLUTIONS

Objective Exercises
Single Correct Answer Type

 x 4. f (x) = min{| x | −1, | x − 2 | −1}


, x≠0



1. f (x) = 1 + e1/ x f(x) is not differentiable at x = 0, 1, 2

0, x=0 Hence, A is correct.

LHD =


−h
−1/h
−0
lim 1 + e
1
= lim =1 y = |x| –1
h →0 −h h →0 1 + e −1/h

h y = |x–2| –1
−0
RHD → 1 + e1/h
lim =0 (–1,0) (1,0)

h →0 h
Since LHD and RHD exist, f(x) is continuous but not (2,–1)

differentiable at x = 0

Hence, C is correct. 1 − x, x <1
2. For f(x) to be continuous at x = x0. 
5. f (x) = (1 − x)(2 − x), 1 ≤ x ≤ 2
ps

ax 0 + b = x 02 
3 − x, x > 2.
(1)


el
For f(x) to be differentiable
f(x) is discontinuous at x = 2 & non-differentiable

LHD ⇒

eh

at x = 2. However, it is differentiable at x = 1 since



( x 0 − h )2 − x 02 h 2 − 2x 0 h LHD = RHD = −1
je

lim = lim = 2x 0
h →0 −h h →0 −h Hence, B is correct.
iit

RHD ⇒
@

y = 1–x
a ( x0 + h ) + b − ( x0 )
2
lim =a
h →0 h

For differentiable, LHD = RHD. (2,1)

⇒ a = 2x 0 & b = − x 02

Hence, B is correct. (1,0) (2,0) y
{ }
=
3–
3. f (x) = min x − 1, − x + 1, sgn(− x)
3
y = (1–x)(2–x) 2

At x = 0, f(x) is differentiable as LHD = RHD = 0
At x = 2, f(x) is continuous but not differentiable. 6. y = x p , p < 1 is non differentiable

Hence, A is correct.
at x = 0
y =| x |p , p > 1 is

y = Sgn(–x)
(0,1) differentiable at x = 0
f(x) is non differentiable at x = −1 only .
0 1 2
(1,0) Hence, A is correct.
(0,–1) y = Sgn(–x) 7. f (x) = 3 x 2 | x | − | x | −1

=| x |5/3 − | x | −1

y = –x+1 is non differentiable at x = 0 because of | x |
y= –x3–1 Hence, A is correct.

FREE BOOKS FOR JEE & NEET =>(@iitjeeadv)


3.80  
Differential Calculus for JEE Main and Advanced

8. If f (x) is differentiable function from R → Q. (


12. y lim f cos x − cos x = x lim f sin x − sin x
3 2 2 3
) ( )

x →0 x →0


⇒ f (x) is constant function.
cos3 x < cos 2 x


100
∑ (−1)r f (r) = f (0) − f (1) + f (2) − f (3) +…+ f (100) sin 2 x > sin 3 x


r =0
⇒ 4y = 3x ⇒ line eqn

= f (100) which is some constant


Let image be (h, k)

Hence, D is correct.


 h k + 1
 − x31/ x ⇒ Pt  , will fulfill the equation of line
 1/ x , x>0  2 2 
 3 −1


 k +1  h
 − x 31/ x + 2

9. f (x) = 
( ) x<0
4
 2  = 3 2 
  
 3 −1

1/ x


⇒ 4k + 4 = 3h ...................................................(I)
0 x=0



 4 k − 1/ 2
 − =
 3 h/2

LHD at x = 0

⇒ –4h = 3k – 3 ...................................................(II)


lim
h 3 ( −1/h
+2 )× 1 =2
Solving (I) & (II)
ps 24 −7
h →0 3 −1/h
−1 −h h= , k=
25 25

el
RHD at x = 0

Hence, D is correct.
eh

−h31/h 1 f (x + h) − f (x)
lim × =1 13. f ′ (x) = lim
h →0 31/h −1 h
je

h →0

h

So, f (x) is not derivable at x = 0
iit

f (x(1 + h / x)) − f (x)


= lim
but it is continuous h →0
@

h
Hence, B is correct.

f (x) ⋅ f (1 + h / x) + 2 − 2 f (x) − f (1 + h / x )
10. Since g(x) is even and differentiable at x = 0 , = lim
h →0 h
g ′(0) = 0

 f (1 + h / x) − 2 

g (h) = lim(f (x) − 1)  
⇒ lim =0 h →0  h 
h →0 h

f (1 + h / x) − 2 1

1 = (f (x) − 1) lim × (f (1) = 2)
g(h) cos   h →0 h/x x
f ′ (0) = lim h

f ′ (1)
h →0 h = ( f (x) − 1)

g(h) x
⇒ lim =0

h →0 h xf ' (x) = 2f (x) − 2


⇒ f ′ (0) = 0 Hence, A is correct
f (x + h) − f (x)

Hence, B is correct. 14. f ′ (x) = lim

h →0 h
 
11. f (x) = lim 1 − 
2 f (h)
= lim = f ' (0)

( )
h →0 h

n →∞ n
 x + 2x + 3 + sin πx
2
+1 

  (2h 2 + 3h)g(h)
f ′ (0) = lim
As x + 2x + 3 + sin πx > 2 ∀x ∈ R .
2
h →0 h

f (x) = 1 for all x. = lim(2h + 3)g(h)

h →0
Hence, A is correct.

FREE BOOKS FOR JEE & NEET =>(@iitjeeadv)
Differentiability 3.81

= (3)g(0) ⇒ x 2 − 4x + 4 = 0


= 3× 3 = 9 ⇒ (x − 2) 2 = 0

f ′ (x) = f ′ (0) = g


⇒x=2


Hence, A is correct. For f (x) to be differentiable at x = 2
f (x + h) − f (x) 2(2 + h) + 1 − 5 (2 + h) 2 − 2(2 + h) + 5 − 5
15. f ′ (x) = lim
= lim = lim

h →0 h h →0 h h →0 h


f (h) (h rational) (h irrational)
= lim = f ′ (0)



h →0 h
Both are equal to 2.

f ′ (0) = lim
sin h g(h) Hence, D is correct.
19. lim (x + h)f (x) − 2hf (h)
h →0 h

g(0) = k x−h


x →h

Hence, A is correct. xf (x) − hf (h) hf (x) − hf (h)
lim + lim
x →h x−h x →h x−h

3x 2 + 2x − 1 (3x − 1)(x + 1) x +1
16. f (x) = = = = f (h) + hf ′ (h) + hf ′ (h)
6x − 5x + 1 (3x − 1)(2x − 1) 2x − 1

2
1/ 3 + h + 1 = f (h) + 2hf ′ (h)
+4
  / 3 + h) − 1

′ 1 2(1
ps
Hence, A is correct.
f   = lim
 3  h →0 h 20. Put x = 1, y = 1 ⇒ f 2 (1) − 3f (1) + 2 = 0
el

4 / 3 + 8 / 3 + 9h − 4 ⇒ f (1) = 1, 2
eh

= lim

h →0 h(2(1/ 3 + h) − 1) If f (1) = 1, put y = 1 to get f (x) = 1
je

= –27. which is a constant function
iit

Hence f (1) = 2

Hence, B is correct.
@

Hence, B is correct.
17. f ′ (x) = lim f (x + h) − f (x)
21. y = sin −1 (|1− | x ||)

h →0
h
This function is non differentiable at
f (x) ⋅ f (1 + h / x) − 2f (x) + 2 − f (1 + h / x ) x = 0, ± 1 and 1 − x = 1 ⇒ x = ±2
= lim
h →0 h

⇒ x = ±2

(f (x) − 1) ′

= f (1) Hence, D is correct .
x
22. f (x + y) − f (x − y) = 4xy

df dx
Now, = f ′ (1) .

f (x) − 1 x Put x = 0

f ( y ) = f ( − y ) ⇒ f (x) is even function
Integrating both sides w.r.t x, ln f (x) − 1 = n ln x + ln C

⇒ f (x) = 1 + cx n Put y = x


Put x = 1, y = 2 to get f(1)=2 f (2x) = 4x 2 = (2 x) 2

Use f (1) = 2 and f (2) = 5 to get f (x) = x 2 . It is unbounded.

f (x) = x 2 + 1 1 1  1
2
f (x) + f   = x 2 + 2 =  x −  + 2
Hence f (3) = 10  
x x  x
Hence, A is correct.

 1
= f x − +2
18. For f (x) to be continuous,  x

2x + 1 = x 2 − 2x + 5 Hence, C is correct.

FREE BOOKS FOR JEE & NEET =>(@iitjeeadv)
3.82  
Differential Calculus for JEE Main and Advanced

23. g(x + y) = e y g(x) + e x g(y) 29. f (x) = max{1, cos x + sin x, sin x − cos x}



Put x = 0, y = 0 ⇒ g(0) = 0  π   π 
= max 1, 2 sin  + x  , 2 sin  x −  

e g(x) + e g(h) − g(x) − e g(0)
h x x   4   4 
g ′(x) = lim
h →0 h π
f(x) is not differentiable at x = ,π

g ′(x) = g(x) + 2e x
Hence, C is correct.
2


Hence, B is correct
24. y = | 1− | 2− | 3− | 4 − x | || | 1

f(x) is not differentiable when
x = 4 , | 3− | 4 − x ||= 0,| 2− | 3− | 4 − x ||= 0,|1− | 2− | 3− | 4 − x |= 0
y is not differentiable at more than 5 points 0 π π 5π
2 4
Hence, D is correct.
 1
 2x + 1 , x > 0
 30. | f (x) | ≤ x 2


 1
25. f (x) =  x<0 | f (0) | ≤ 0 ⇒ f (0) = 0
 2x − 1


1 x=0 | f (h) | ≤ h 2



ps f (h) − f (0)
⇒ lim ≤ lim h
lim f (x) = 1 , lim f (x) = −1 , f (0) = 1 h →0 h h →0
el
x →0+ x →0−


f ′ (0) ≤ 0
eh

Hence, C is correct.

26. y = cosx is decreasing in [0, π]

je

⇒ f ′ (0) = 0

cos x, x ∈ [0, π]
iit

g(x) = 

Hence, C is correct.
sin x − 1, x > π
@

31. If f(x) is even and differentiable for all x



lim cos x = −1 , lim f (x) = −1 , f (π) = −1 ⇒ f′(0) = 0
x →π− x →π+


Hence, option D is correct.
At x= π, LHD = 0 RHD = 1. Not differentiable
Hence, B is correct. x3 , x<0

27. Put x = y = z = 0 ⇒ f (0) = 1 x 2 , 0 ≤ x <1
32. f (x) = 
Put z = 0 2x − 1, 1≤ x < 2

⇒ f (x + y) = f (x) ⋅ f (y).  2
 x − 2x + 3, x≥2

′ f (x) ⋅ f (h) − f (x)
f (x) = lim 3x 2 , x<0
h →0 h 
2x, 0 ≤ x <1

f ′ (x) = 3f (x) ′
f (x) = 
2, 1≤ x < 2

Put x = 2 2x − 2,
 x≥2
f ′ (2) = 12

f (x) is continuous and differentiable for x ∈ R.

Hence, A is correct.

Hence, A is correct.
−1
28. f (x) = lim (cos x) + cos (sin x)
2k
k →∞  x 2 + 1, 0 ≤ x < 1


cos −1 sin x, x ≠ nπ 33. f (x) = 3 − x, 1 ≤ x < 2

= 2, x=2
−1
1 + cos sin x x = nπ 
f (x) is discontinuous at x = nπ ⇒ f (x) is discontinuous at x = 2 and non-derivable at x = 1


Hence, C is correct. Hence, C is correct.

FREE BOOKS FOR JEE & NEET =>(@iitjeeadv)


Differentiability 3.83

34. As x → ±∞, f (x) → 0


( ) (
f a + 2h 2 − f a − 2h 2 )

39. lim
As x > 0, f ′′ (x) > 0 2


h →0 4h

′′
As x < 0, f (x) > 0 2h 3 − 2h 2 4h 2
× ×
( ) ( )

& at x = 0, f ′ (x) is non differentiable. f a + h3 − h 2 − f a − h3 + h 2 2h 3 − 2h 2
Hence, C is correct.


1 4
( )
35. f (x) = x 2 − 1 | (x − 2)(x + 1) | + sin | x | f ′ (a) × ′
f (a)
×
−2
= −2

y = ( x + 1) | (x + 1) | is differentiable at x = −1


Hence, C is correct.

f (x) is non differentiable at x = 0 & 2 [x] + {x}, x <1

Hence, C is correct. 
40. f (x) =  1
36. Put x = h + t where t → 0 x ≥1


 ,
 [x] + {x}2
(2h + t)f (h + t) − 2hf (h)
lim
t →0
t lim f (x) = 1 , lim f (x) = 1 , f (1) = 1
x →1− x →1+


f (h + t) − f (h) {1 − h}
= 2h lim + lim f (h + t) f (1 − h) − f (1)
t →0 t t →0 LHD → lim = lim = does not exist
h →0 −h h →0 −h

= f (h) + 2hf ′ (h)

Hence, A is correct.
ps

Hence, A is correct 41. −f (a) = xlim
→a
f (x)

el
37. f (x) = max{ x(2 − x), 2 − x} Hence, C is correct.

eh

42. Statement III is true using Intermediate Value Theorem.


Now as y = x(2 − x) Hence, D is correct.
je

⇒ y 2 = 2x − x 2 + 1 − 1 43. f (x − f (y)) = f (f (y)) + x f (y) + f (x) − 1


iit

y 2 + (x − 1) 2 = 1 Put y = 0
@


f (x − 1) = f (1) + x + f (x) − 1

f(x) has corner at x = 1 and has infinite derivative at x = 2

f (x) is non differentiable at x = 1, 2.

Put x = 1

Hence, D is correct.

f (1) = 1/2 .

1
f (x − 1) = x + f (x) −
y = x(2–x) 2

Now,
f (0) = 1 + f (1) − 1/ 2
(0,0) f (1) = 2 + f (2) − 1/ 2 .
(1,0) (2,0)

y = 2–x
f (x − 2) = x − 1 + f (x − 1) − 1/ 2
1
f (x − 1) = x + f (x) −
(
 sin x 2 − 5x + 6 ) 2

 , x < 2, or x > 3 Add all equations,
 x 2 − 5x + 6
 x(x + 1) x
(
38. f (x) =  sin x 2 − 5x + 6 ) f (0) =
2
− + f (x)
2

− , 2<x<3

 x 2 − 5x + 6 x2
 f (x) = 1 −
1 x = 2 or 3 2

f (x) is non differentiable at x = 2 & 3 as it is discontinuous Hence, A is correct.

there. 44. Conceptual Problem.
Hence, D is correct. C is Correct.

FREE BOOKS FOR JEE & NEET =>(@iitjeeadv)


3.84  
Differential Calculus for JEE Main and Advanced

45. xlim
→∞
µ(x) = 2, lim v(x) = 3, lim f (x) = lim g(x) = ∞
x →∞ x →∞ x →∞ ( ) (
f ′ (x) = k + e x h(x) + kx + e x h ′ (x) )



f (x) f (x) Put x = 0
+ µ(x) = v(x)
g ′(x)


g(x)
f ′ (0) = (k + 1)h(0) + h ′ (0)

f (x) 18 = 5k + 5 − 2
Let lim =a


x →∞ g(x) k=3



f (x) f (x) Hence, C is correct.
lim + µ(x) = lim v(x)
x →∞ g ′(x) g(x ) x →∞ 48. If f ′ (2) = 4 ⇒ f (x) is differentiable and continuous




⇒ a + 2a = 3 at x = 2
using LH Rule
a =1 as f (2) = 3

Hence, A is correct. lim f (x) = 3 + h
x → 2+
f (x) + f (h) + 5xh − f (x) − f (0)
46. f ′ (x) = lim lim f (x) = 3 − h
h →0 h x → 2−

= lim
f (h)
+ 5x (∵ f (0) = 0) So, lim [f (x)] will not exist.
h →0 h x →2
Hence, D is correct.


f ′ (x) = f ′ (0) + 5x

49. Similar to Q . 40.
ps

= 5x + 3
2f (x) − 3f (2x) + f (4x)
50. lim

el
5

f (x) = x 2 + 3x x →0 x2
eh

2 Applying L-Hospital Rule,




−D
je

Minimum value of f (x) = =−


9 2f ′ (x) − 6f ' (2x) + 4f ′ (4x)
4a 10 lim
iit

x →0 2x
Hence, B is correct.

@

47. f (x) = kx + e x h(x) ( ) lim


2f '' (x) − 12f '' (2x) + 16 f '' (4x)

x →0 2
Put x = 0, f (0) = 5

= 12


Differentiate w.r.t x Hence, C is correct.

Multiple Correct Answer Type

51. f (x) =| x − 1| ([x] − [− x]) B) LHD → Not defined & f (x) is discontinuous at




x = π.
lim f (x) = 0, lim f (x) = 0, f (1) = 0
x →1+ x →1− 1, x ≠ 0
RHD → Not defined C) f (x) = 

0, x = 0

You might also like